You are on page 1of 303

https://sscstudy.

com/

https://sscstudy.com/
https://sscstudy.com/

Contents
Oneliner Current Affairs 1-32
General Knowledge 1-50

General Mental Ability 1-84


1. Analogy 3-8 9. Logical Sequence of Words 47-49
2. Classification 9-12 10. Inserting the Missing Characters 50-53
3. Series 13-18 11. Ranking Test 54-57
4. Coding and Decoding 19-24 12. Sitting Arrangement 58-61
5. Alphabet Test and Jumbling 25-30 13. Puzzle Test 62-65
6. Mathematical Operations 31-34 14. Venn Diagram 66-71
7. Direction Sense Test 35-41 15. Cube and Dice 72-75
8. Blood Relations 42-46 16. Non-Verbal Reasoning 76-84

Numerical Ability 1-102


1. Number System 3-9 10. Simple and Compound Interest 49-53
2. HCF and LCM 10-13 11. Mixture and Alligation 54-57
3. Square Root and Cube Root 14-20 12. Time and Work 58-62
4. Indices and Surds 21-24 13. Time, Speed and Distance 63-69
5. Simplification 25-28 14. Area and Perimeter 70-75
6. Average 29-32 15. Volume and Surface Area 76-80
7. Ratio and Proportion 33-37 16. Algebra 81-87
8. Percentage 38-41 17. Geometry 88-95
9. Profit, Loss and Discount 42-48 18. Data Interpretation 96-102

Quantitative Reasoning 3-10


Logical and Analytical Reasoning 3-14
Practice Sets (1-5) 3-22

https://sscstudy.com/
https://sscstudy.com/

CUET (UG)
All About

Exam
Common University Entrance Test (CUET (UG) – 2022) will be conducted in 13 mediums across India for admission into
the Undergraduate Programmes in Computer Based Test (CBT) mode for all the Central Universities (CUs) like DU, JNU,
JMI, BHU, AMU, etc. in India for the academic year 2022-23.

EVENTS DETAILS
Online submission of Application Form 06 April to 06 May, 2022 (up to 5:00 pm)

Last date of successful transaction of Examination Fee 06 May, 2022 (up to 11:50 pm)

Correction in the particulars of Application Form


To be announced later on the website
on Website only

Downloading of Admit Card from NTA Website To be announced later on the website

Date(s) of Examination First and second week of July, 2022

Duration of Examination Slot 1: 195 minutes (3:15 hours)


Slot 2: 225 minutes (3:45 hours)

Timing of Examination Slot 1: 09.00 AM to 12.15 PM (IST)


Slot 2: 03.00 PM to 06.45 PM (IST)

TESTS DESIGN There are three Sections in the design of the test:

Section IA 13 Languages (As a medium and “Language”) Section IB 20 Languages

Section II 27 Domain Speci c Subjects Section III General Test

MERIT LIST
Merit list will be prepared by participating Universities/ organisations. Universities may conduct their individual
counselling on the basis of the score card of CUET (UG) 2022 provided by NTA.

https://sscstudy.com/
https://sscstudy.com/

Broad features of CUET (UG) - 2022 are as follows:


Questions to
Section Subjects/Tests Question Type Duration
be Attempted

Section IA- There are 13* different 40 questions to Language to be tested through 45 minutes
Languages languages. Any of these be attempted out Reading Comprehension (based on for each
languages may be chosen. of 50 in each different types of passages – Factual, language
language Literary and Narrative, [Literary Aptitude
Section IB- There are 19** languages. and Vocabulary]
Languages Any other language apart
from those offered in Section
IA may be chosen.

Section II- There are 20*** Domains 40 questions to Ÿ Input text can be used for MCQ 45 minutes
Domain specific subjects being be attempted out based questions for each
offered under this Section. A of 50 Domain
Ÿ MCQs based on NCERT Class XII Specific
candidate may choose a
maximum of Six (06) Domains syllabus only Subjects
as desired by the applicable
University/Universities.

Section III- For any such undergraduate 60 questions to Ÿ Input text can be used for MCQ 60 minutes
General programme/programmes be attempted out based questions
Test being offered by Universities of 75
Ÿ General Knowledge, Current
where a General Test is being
used for admission. Affairs, General Mental Ability,
Numerical Ability, Quantitative
Reasoning (Simple application of
basic mathematical concepts
arithmetic/algebra geometry/
mensuration/stat taught till Grade
8) Logical and Analytical Reasoning

* Languages (13): Tamil, Telugu, Kannada, Malayalam, Marathi, Gujarati, Odiya, Bengali, Assamese, Punjabi, English,
Hindi and Urdu
** Languages (20): French, Spanish, German, Nepali, Persian, Italian, Arabic, Sanskrit, Sindhi, Kashmiri, Konkani, Bodo,
Dogri, Maithili, Manipuri, Santhali, Tibetan, Japanese, Russian, Chinese.
*** Domain Specific Subjects (27): 1. Accountancy/ Book Keeping, 2. Biology/ Biological Studies/Biotechnology
/Biochemistry, 3. Business Studies, 4. Chemistry 5. Computer Science/ Informatics Practices 6. Economics/ Business
Economics 7. Engineering Graphics 8. Entrepreneurship 9. Geography/Geology 10. History 11. Home Science 12.
Knowledge Tradition and Practices of India 13. Legal Studies 14. Environmental Science 15. Mathematics 16. Physical
Education/ NCC /Yoga 17. Physics 18. Political Science 19. Psychology 20. Sociology 21. Teaching Aptitude 22. Agriculture
23. Mass Media/ Mass Communication 24. Anthropology 25. Fine Arts/Visual Arts (Sculpture/ Painting)/Commercial Arts,
26. Performing Arts – (i) Dance (Kathak/ Bharatnatyam/Oddisi/ Kathakali/Kuchipudi/ Manipuri (ii) Drama- Theatre (iii)
Music General (Hindustani/ Carnatic/ Rabindra Sangeet/ Percussion/ Non-Percussion).
27. Sanskrit/For all Shastri (Shastri 3 years/ 4 years Honours) Equivalent to B.A./B.A. Honours courses i.e. Shastri in Veda,
Paurohitva (Karmakand), Dharamshastra, Prachin Vyakarana, Navya Vyakarana, Phalit Jyotish, Siddhant Jyotish,
Vastushastra, Sahitya,Puranetihas, Prakrit Bhasha,Prachin Nyaya Vaisheshik, Sankhya Yoga, Jain Darshan,Mimansa,Advaita
Vedanta, Vishihstadvaita Vedanta, Sarva Darshan, a candidate may choose Sanskrit as the Domain.]
Ÿ A candidate can choose a maximum of any 3 languages from Section IA and Section IB taken together. However, the
(one of the langueges chosen needs to be in lieu of 6th domain specific Subjects).
Ÿ Section II offers 27 Subjects, out of which a candidate may choose a maximum of 6 Subjects.
Ÿ Section III comprises General Test.
Ÿ For choosing Languages (upto 3) from Section IA and IB and a maximum of 6 Subjects from Section II and General Test
under Section III, the candidate must refer to the requirements of his/her intended University.

https://sscstudy.com/
https://sscstudy.com/

Broad features of CUET (UG) - 2022 are as follows:


Mode of the Test Computer Based Test-CBT

Test Pattern Objective type with Multiple Choice Questions

Medium 13 languages ( Tamil, Telugu, Kannada, Malayalam, Marathi, Gujarati, Odiya, Bengali,
Assamese, Punjabi, English, Hindi and Urdu)

Section IA & IB: Language to be tested through Reading Comprehension (based on different
types of passages– Factual, Literary and Narrative [Literary Aptitude & Vocabulary]

Section II : As per NCERT model syllabus as applicable to Class XII only


Syllabus
Section III : General Knowledge, Current Affairs, General Mental Ability, Numerical Ability,
Quantitative Reasoning (Simple application of basic mathematical concepts arithmetic/
algebra geometry/mensuration/stat taught till Grade 8), Logical and Analytical Reasoning

Registration Registration will be online at https://cuet.samarth.ac.in/. (available from 06.04.2022)

Level of questions for CUET (UG) -2022:


All questions in various testing areas will be benchmarked at the level of Class XII only. Students having studied Class XII
Board syllabus would be able to do well in CUET (UG) – 2022.
Number of attempts : If any University permits students of previous years of class XII to take admission in the current
year also, such students would also be eligible to appear in CUET (UG) – 2022.
Choice of Languages and Subjects : Generally the languages/subjects chosen should be the ones that a student has
opted in his latest Class XII Board examination. However, if any University permits any flexibility in this regards, the same
can be exercised under CUET (UG) -2022 also. Candidates must carefully refer to the eligibility requirements of various
Central Universities in this regard. Moreover, if the subject to be studied in the Undergraduate course is not available in
the list of 27 Domain Specific Subject being offered, the candidate may choose the Subject closest to his/her choice for
e.g. For Biochemistry the candidate may choose Biology.

https://sscstudy.com/
https://sscstudy.com/

ONELINER CURRENT AFFAIRS 1

ONELINER

CURRENT AFFAIRS
Ministry of Tribal Affairs has and 31, 2022, to celebrate the culture
National Affairs

sanctioned ` 2.26 Crores for second- and traditional heritage of the



PM Narendra Modi on March 29, 2022 largest fair in India, MedaramJatara Union Territory
participated in ‘Grih Pravesham’ of 2022 Festival held from February 16
more than 5 lakh beneficiaries of

The Ministry of Culture has nominated
to 19, 2022 in Telangana
Pradhan Mantri Awaas Yojana-Gramin the ‘Sacred Ensembles of Hoysala’

Union Minister NitinGadkari and Bihar located in Karnataka on January 31,
in Madhya Pradesh
Chief Minister Nitish Kumar jointly 2022 for UNESCO’s World Heritage

India has launched an evacuation inaugurated the ……… ‘Rail-cum- List for 2022-23
mission named Operation Ganga on Road-Bridge’ in Bihar on February 11, ■
Home Delivery of medicines under
February 27, 2022 to evacuate Indian 2022. 14.5 KM long
…….. for all entitled armed forces
nationals from Ukraine due to the ■
Jammu & Kashmir has become the first personnel and their families started on
Russia-Ukraine tension
Union Territory on February 8, 2022 to January 31, 2022.

Union Cabinet has approved the be integrated with the Services e-Health Assistance and
pan-India implementation of Ayushman National Single Window System (NSWS) Teleconsultation (SeHAT)
Bharat Digital Mission (ABDM) with a ■
‘Pradhan Mantri Kisan Sampada ■
Health Minister Mansukh Mandaviya
budget of ` 1600 crore for 5 years
Yojana (PMKSY)’ has been extended launched revamped website and Mobile

Union Minister of Culture, G Kishan till ……….. with an allocation of ` 4600 App …….. of Central Government
Reddy, inaugurated Devayatanamon crore on February 8, 2022. March 2026 Health Schemeon Jan 24, 2022. MyCGHS
February 25, 2022, a one-of-a-kind ■
……….. government has started an ■
The government of India has launched
odyssey (conference) of temple
open-air classroom programme, called a web portal ………. to share Key
architecture of India, at
Paray Shikshalaya (Neighborhood Performance Indicators (KPI) related to
Hampi (Karnataka)
School), for students studying in the Coal Sector on January 21, 2022.

Prime Minister Shri NarendraModi primary and pre-primary classes on Koyla Darpan
inaugurated a 550-tonne capacity February 6, 2022. West Bengal ■
India’s First “District Good Governance
‘Gobar-Dhan (Bio-CNG) Plant’ through ■
Khadi and Village Industries Index” for 20 districts of Jammu and
video conferencingon February 19,
Commission has canceled the “Khadi Kashmir released on January 21, 2022
2022 in Indore (Madhya Pradesh)
Certification” on February 4, 2022 of its and………..district has topped the

The Union Minister of Ports, Shipping oldest Khadi Institution named Index. Jammu
and Waterways SarbanandaSonowal Mumbai Khadi and ■
Who was inaugurated ‘AzadiKe Amrit
flagged off India’s first water taxi Village Industries Association
Mahotsav se Swarnim Bharat Ki
service on February 17, 2022 in ■
The Aravalli Biodiversity Park in Ore’programme on January 20, 2022
Maharashtra
Gurugram, Haryana, has been declared via video conferencing?

Union Home Minister Amit Shah on February 2,2022 as India’s first PM Narendra Modi
launched ‘Smart Card Arms License’ Other Effective area-based Conservation ■
The government of India has merged
and ‘Shastra App’on February 16, 2022 Measures (OECM) site
the flame of the Amar Jawan Jyoti on
of the Delhi Police ■
India’s first geological park is being set January 21, 2022 at Delhi’s India Gate

Ministry of Education has approved a up on February 1, 2022 at Lamheta with the flame at the adjoining
new scheme “New India Literacy village of Jabalpur district in National War Memorial
Programme on February 16, 2022 for Madhya Pradesh ■
Prime Minister Narendra Modi has
the period of 2022-2027 to cover all the ■
The annual SpitukGustor Festival was declared on January 16, 2022 to mark
aspects of Adult Education
observed in Ladakh on January 30 January 16 as National Start-up Day

https://sscstudy.com/
https://sscstudy.com/

2 CUET (UG) Section III : General Test


The state governments of Haryana and ■
Prime Minister NarendraModi has ■
PM Narendra Modi laid the foundation
Himachal Pradesh have signed a MoU inaugurated the SaryuNahar National stone and dedicated various National
on January 21, 2022to revive the Project on December 15, 2021 in Highway and Road projects on
Saraswati river through construction of Balrampur, Uttar Pradesh November 9, 2022 to the nation in
AdiBadri dam Pandharpur (Maharashtra)

Union Minister Social Justice and

PM Narendra Modi inaugurated 11 Empowerment, Dr Virendra Kumar has ■
Union Minister Dr. Jitendra Singh has
new government medical colleges and a launched National Helpline on dedicated to the nation “Sardar Patel
new campus of the Central Institute of December 14, 202 1against atrocities on Leadership Centre” on October 31, 2022
Classical Tamil (CICT) on January 12, SCs, STs at LalBahadurShastri National
2022 in Tamil Nadu ■
PM Narendra Modi has inaugurated Academy of Administration (LBSNAA)

PM NarendraModi inaugurated the 25th the first phase of the ` 339 crore in Mussoorie
National Youth Festival on Jan 12, 2022 KashiVishwanath Corridor project that ■
The Union Minister of Power & New
via video conferencingin Puducherry connects the Kashi Vishwanath temple and Renewable Energy, R K Singh has
and Ganga ghats launched a new market segment on

Defence Minister Rajnath Singh
inaugurated Kalpana Chawla Centre on ■
PM Narendra Modi has inaugurated October 28, 2022
January 10, 2022 for Research in Space and laid the foundation stone on Green Day Ahead Market (GDAM)
Science & Technology at December 7, 2021of multiple projects ■
The Atal Innovation Mission (AIM) of
Chandigarh University worth ` 18000 crore at NITI Aayog has launched a Digi-book
Dehradun, Uttarakhand named ………. on October 26, 2022.

Ministry of Railways has approved the
renaming of the Kevadiya railway

NITI Aayog launched the ‘e-Sawaari Innovations for You
station in Narmada district of Gujarat on India Electric Bus Coalition’ in ■
Prime Minister NarendraModi has
Jan 5, 2022, as Ekta Nagar railway station partnership with Convergence Energy
inaugurated the Kushinagar
Service Ltd (CESL) and World Resources

Prime Minister Narendra Modi has International Airport on October 20,
Institute, India (WRI India)
declared on January 5, 2022 that 2022 in Uttar Pradesh
starting from the year 2022, December

PM NarendraModi has laid the ■
Prime Minister NarendraModi
26 will be observed as ‘Veer Baal Diwas’ Foundation Stone of Noida
dedicated ……. new Defence PSUs,
Every Year
International Airport on November 25,
carved out of OFBs, to the nation on
2022 at Jewar (Uttar Pradesh)

Telangana has bagged the top position October 16, 2022. 7

President of India, Ram NathKovind
among all other States of the country in ■
Prime Minister NarendraModi
visited Sui village in the………….., to
having the highest number of ODF plus inaugurated ……… National Master
inaugurate various public facilities in
villages under the Plan, from Pragati Maidan in New Delhi
the village on November 18, 2022.
Swachh Bharat Mission (Grameen) on October 14, 2022. PM Gati Shakti
Bhiwani district (Haryana)

PM Narendra Modi inaugurated 13 ■
PM Narendra Modi has announced on

Centre government has launched ……..
Projects and laid the foundation stone November 19, 2022 that his in Kolkata for digital monitoring of port
of 9 projects worth on January 4, 2022, government will repeal the three operation on Oct 11, 2022.
during an event held at contentious agricultural laws ‘MyPortApp’
Hapta Kangjeibung in Imphal, Manipur

Union Minister, Piyush Goyal virtually ■
Who was launched the Indian Space

PM NarendraModi inaugurated and laid launched India’s first Digital Food Association (ISpA) via
foundation stones of 23 projects worth Museum on November 18, 2022 in videoconferencing on October 11, 2022.
over ` 17500 croreon December 30, 2021 Thanjavur, Tamil Nadu Prime Minister Narendra Modi
in Haldwani (Uttarakhand) ■
PM Narendra Modi has inaugurated ■
Union Minister of Law and Justice,

Uttar Pradesh Government has the ……….. long Purvanchal Express in KirenRijiju has inaugurated the first
renamed the Jhansi Railway Station in Sultanpur district, Uttar Pradesh on Sports Arbitration Centre of India on
Jhansi as ‘VeeranganaLaxmibai November 16, 2022. 341 km Oct 4, 2022 at
Railway Station’ on December 29, 2021, ■
The central government of India Ahmedabad in Gujarat
as a tribute to Queen of Jhansi promulgated two ordinances on ■
India on October 14, 2021 re-elected for

PM Narendra Modi has inaugurated November 15, 2022 for extending the which council?
and laid the foundation stone of several tenure of Directors of the ED and CBI United Nations Human Rights Council
development projects worth over 650 for up to 5 years ■
Who has topped the IIFL Wealth
crores rupees on Dec 23, 2021 in Goa ■
Union Cabinet chaired by Prime Hurun India Rich List 2021?

PM Narendra Modi has laid the Minister Narendra Modi on November Mukesh Ambani
foundation stone of Ganga Expressway 11, 2022 has approved the declaration
of November 15 as

The world’s largest Khadi national flag
on Dec 22, 2021 in has recently been unveiled in Ladakh
Shahjahanpur, Uttar Pradesh Janjatiya Gaurav Divas

https://sscstudy.com/
https://sscstudy.com/

ONELINER CURRENT AFFAIRS 3


The Greenfield Sindhudurg Airport has ■
Which UT has topped among union ■
The Union Minister for Commerce and
been inaugurated in which state under territories in the FSSAI’s 3rd State Industry Piyush Goyal inaugurated
the UDAN Scheme? Maharashtra Food Safety Index 2021? …………… on April 13, 2021 to provide
Jammu & Kashmir a platform for connecting aqua farmers

Which cartoon character has been
India’s highest altitude herbal park and buyers. e-SANTA
roped in as the official mascot for the ■

Namami Gange Programme? situated at a height of 11,000 feet was ■


NITI Aayog on April 13, 2021 launched
Chacha Chaudhary inaugurated in Uttarakhand a national digital repository on health
Which year has been set as the target and nutrition, called Poshan Gyan

The VAYO NAMAN Programme was ■

organised by Ministry of Social Justice for becoming 'energy independent' by ■


Education Minister Ramesh Pokhriyal
and Empowerment PM Modi? 2047 ‘Nishank’ launched on April 12, 2021

In which city, Narendra Singh Tomar the ‘Implementation Plan for School

The Alibaug white onion of which state
has inaugurated world’s second-largest Education’, called SARTHAQ
has recently received the GI tag?
Maharashtra refurbished gene bank? New Delhi

The 2021 State of the Education Report

Which Airport has been renamed as International Affairs
(SOER) for India: “No Teacher, No Maharaja Agrasen International ■
The BIMSTEC Summit 2022 was held
Class”has been launched by which Airport? Hisar Airport on March 30, 2022 in Virtual mode. It
organisation? UNESCO India ■
Which state has government has was hosted by Sri Lanka
launched ‘Mission Vatsalya’ for women The UN Sustainable Development

Which state has launched Vatan Prem ■

who lost their husbands to COVID-19? Solutions Network (SDSN) published


Yojana? Gujarat
Maharashtra the World Happiness Report 2022 on

Which state government has launched ■
PM Narendra Modi launched……, an March 18, 2022. In this, India is ranked
‘Mera Kaam Mera Maan’ Scheme to
e-voucher- based digital payment at 136th Place
Help Unemployed Youth? Punjab
solution, on August 2, 2021. e-RUPI ■
India has been ranked at ………. place

Which state will host the Defence Expo ■
Which state has celebrated Wanchuwa in the International IP Index 2022
2022? Gujarat Festival 2021? Assam released by US Chamber of Commerce

Who has been elected as the new Chief ■
The Parliament on July 27, 2021 passed on February 25, 2022 and US has
Minister of Gujarat? Bhupendra Patel which bill, aims to replace over topped. 43rd

In which state, Nirmala Sitharaman 90-year-old Lighthouse Act, 1927? ■
India and France have inked a roadmap
has inaugurated ‘My Pad, My Right’ Marine Aids to Navigation Bill, 2021 on February 22, 2022 to enhance their
project? Tripura ■
The Central government on July 19, bilateral exchanges on the blue

The name of Rajiv Gandhi national 2021 decided to set-up which institute economy and Ocean Governance
park in Assam has been changed to in Noida (Gautam Buddha Nagar)? ■
Prime Minister NarendraModi
Orang National Park Indian Institute of Heritage delivered the inaugural address at

India’s 1st Emergency Landing Facility ■
The Ministry of Ayush launched which World Sustainable Development ……..
on National Highway has been mobile app in the event for the 7th via video conferencing on February 16,
inaugurated in which state? Rajasthan International Day of Yoga on June 11, 2022. Summit

Who has been featured in Time 2021, virtually. Namaste Yoga ■
Who has become the first country in
Magazine’s list of ‘100 Most Influential ■
Which State Government of has the world to allow unmanned aircraft
People of 2021’? PM Narendra Modi, officially notified the Dihing Patkai vehicles (UAVs/Drones) to operate in
Mamata Banerjee, Adar Poonawalla Wildlife Sanctuary as a National Park civilian airspace? Israel

Who has been appointed as Punjab’s on June 9, 2021? Assam ■
Military officer Paul-Henri Sandaogo
new Chief Minister? ■
Ladakh LG RK Mathur launched which Damiba appointed as interim President
Charanjit Singh Channi scheme to Distributes 12300 Tablets on February 12, 2022 of Burkina Faso

Which state’s Hathei chilli and Among Students of Government School ■
Norway has topped and India is placed
Tamenglong mandarin orange has been on June 6, 2021? YounTab at ….. position, among 167 nations, in
given the GI tag? Manipur ■
Raimona Reserve Forest in Assam has the EIU’s 2021 Democracy Index. 46th

Which state has become first state to been upgraded as the National Park. It ■
………. tops in terms of digital skills
recognise Community Forest Resource will now be the readiness as per Salesforce Global
Rights in Urban Region? Chhattisgarh Sixth National Park in Assam Index on February 7, 2022. India

Which state government has renamed

Union Minister Piyush Goyal launched ■
North Korea successfully tested its
Kakori Train Conspiracy to Kakori Train which scheme on April 19, 2021 to Hwasong-12 intermediate-range
Action? Uttar Pradesh provide financial assistance to startups? ballistic missile on January 30, 2022
Startup India Seed Fund from the Jagang Province area

https://sscstudy.com/
https://sscstudy.com/

4 CUET (UG) Section III : General Test


PM of Portugal ……… , has been to a four-and-a-half day starting ■
World Health Organisation (WHO) has
re-elected after his centre-left Socialist January 1,2022. The UAE classified the new COVID-19 variant
Party secured landslide victory in the B.1.1.529 on November 29, 202 1as

Japan has introduced a minibus
2022 Portuguese legislative electionon Omicron India
lookalike world’s first dual-mode
January 30, 2022. Antonio Costa ■
Russian Navy successfully test-fired the
vehicle (DMV)on December 29, 2021 in

The world’s largest canal lock has been its town of Kaiyo ………… Hypersonic Cruise Missile
inaugurated on January 28, 2022 at from Frigate – Admiral Gorshkov

Egypt has been inducted on Dec 29, warship in the Russian Arctic waters on
Ijmuiden, a small port city, in the
2021as the fourth new member of the November 22, 2021. Zircon
Port of Amsterdam, The Netherlands
BRICS

PM of India NarendraModi hosted the New Development Bank (NDB) ■
The WePOWER India Partnership
first ………. on 27 January 2022, Forum was held on November 17, 2021,

…….. has launched Inmarsat-6 F1, a to boost the South Asia
through virtual platform.
communication satellite, through a
India-Central Asia Summit Women in Power Sector Professional
Mitsubishi Heavy Industries (MHI) Network (WePOWER) in India

The Russian, Chinese and Iranian H-IIA204 rocket on Dec 28, 2021. Japan
navies undertook naval exercises ■
……….. successfully launched a new

35-year-old ……… , has won Chile satellite named Shijian-21 on
………… in the Gulf of Oman from presidential elections, to become the
January 18 to 22, 2022. CHIRU-2Q22 October 28, 2021. China
youngest-ever President-elect of Chile

……….. delivered all the contracted on December 22, 2021. Gabriel Boric ■
India, Israel, United Arab Emirates and
70,000 AK-203 Kalashnikov assault ………. have decided to launch a new

………. was sworn in as new Austria’s quadrilateral economic forum on
rifles to the Indian Armed Forces on chancellor by Alexander Van der
January 25, 2022. Russia October 21, 2021. The USA
Bellen, President of Austria in Hofburg

The Indonesian parliament has Palace, Vienna, Austria on ■
……….., the Leader of the Labour Party
approved a bill on January 21, 2022to December 20, 2021. Karl Nehammer in Norway, has assumed the charge of
change the capital of Indonesia from the Prime Minister of Norway with

Dubai became the world’s first
Jakarta to Nusantara effect from October 14, 2021.
government to turn 100% paperless on
Jonas Gahr Store

India Extended $ 190-million line of December 15, 2021, the announcement
credit to ………. for the Metro Express was made by UAE Crown Prince ■
German rail operator, Deutsche Bahn
Project on January 20, 2021. and industrial group, Siemens launched

Indian-American political advisor
Mauritius the first automated & driverless ………
GautamRaghavanelected as the head of
of the world October 12, 2021. train

The Oxford University Press (OUP) has the …………. of Presidential Personnel
picked up ……… as the Children’s on Dec 13, 2021. White House Office ■
Colonel Mamady Doumbouya has been
Word of the Year 2021, based on their sworn-in as the Interim President of

United Nations General Assembly
recent research on January 20, 2021. which country? Guinea
(UNGA) has granted Observer Status to
Anxiety ………. by adopting resolution 76/123 ■
What is the rank of India in the latest

PM Narendra Modi addressed the on December 10, 2021. Henley Passport Index for 2021? 90th
……….. Davos Agenda Summit 2022, International Solar Alliance (ISA) ■
The Multidimensional Poverty Index
through video-conferencing on (MPI) Report is released by UNDP

German lawmakers ……….. selectedas
January 17, 2022.
the new chancellor on December 9, ■
Which University/Institute has topped
World Economic Forum’s (WEF’s) 2021, putting an end to 16 years of the ‘THE World University Ranking

The iconic ‘Infinity Bridge’ in ………. conservative rule under Angela Merkel. 2022’? University of Oxford
was formally opened to traffic for the Olaf Scholz
first time on January 16, 2022.

Which country will host the G-20

The President of Gambia, ………….., leaders’ summit in 2023? India
Dubai (United Arab Emirates) won the second term as President

Antigua and ……… has officially during Gambia’s presidential election

With which country, India has
joined the International Solar Alliance on December 7, 2021. Adama Barrow launched the revamped Strategic Clean
(ISA) as a member country on Energy Partnership (SCEP)? USA

Who has become the World’s newest
January 5, 2022. Barbuda republic, around 400 years after it

Aziz Akhannouch has been appointed

………. has assumed the rotating became a British colony on as the new Prime Minister of Morocco
presidency of the Council of the December 1, 2021? Barbados ■
‘AUKUS’ is a new security partnership
European Union with effect from ■
PetrFiala has been sworn in as the new for Indo-Pacific between
January 1, 2022. France Prime Minister of the Czech Republic USA, UK and Australia

……….. becomes first country to on Nov 29, 2021by President ■
Who has been chosen as the new Prime
change its existing five-day workweek MilosZeman Minister of Japan? Fumio Kishida

https://sscstudy.com/
https://sscstudy.com/

ONELINER CURRENT AFFAIRS 5


Which country has become the 9th ■
Which country has banned the ■
The ……….. was India’s top trading
member of the Shanghai Cooperation Full-Face Coverings in Public Places? partner in the calendar year 2021 with
Organisation (SCO)? Iran Switzerland a trade of $ 112.3 billion. US

What was the rank of India in Global ■
Which country has launched the first ■
…….. has launched a new mobile app
Innovation Index 2021? 46th Arctic-monitoring Satellite named Saa?thi on investor education
‘Arktika-M’? Russia on January 19, 2022. SEBI

Narendra Modi has become the first
Indian PM to chair ……… debate on

Which nation’s President has signed a ■
The RBI has included ……….. Payment
August 9, 2021. law enabling him to run for President Bank in the ‘Second Schedule to the
United Nations Security Council (UNSC) again? Russia Reserve Bank of India Act, 1934’ on

Who is the new President of Vietnam January 4, 2022. Airtel

India has launched a tech platform
elected on April 5? Nguyen Xuan Phuc ■
The global cryptocurrency app Crypto
named ‘UNITE Aware’, in collaboration
Wire has launched India’s first index of
with United Nations ■
Russia has become the first country in
cryptocurrencies named ……… on
Nikol Pashinyan has been re-appointed the world to register the coronavirus

January 3, 2022. IC15
as the PM of vaccine for animals on April 5, 2021,
Armenia
named Carnivac-Cov ■
Who has issued the guidelines for

Which country has become the 5th facilitating small value digital
country to sign International Solar transactions in offline mode on
Alliance (ISA) Framework Agreement? Economy & Banking January 1, 2022? Reserve Bank of India
Germany ■
The ICICI Bank Ltd. and HDFC Bank
on March 28, 2022 signed an

……… and Payment Services Ltd. has

Who has been appointed as the new collaborated with Paytm for card
agreement for a 15% state in
Prime Minister of Lebanon, in place of tokenisation to protect data of the
India Debt Resolution Company Limited
Saad Hariri, on July 26, 2021? cardholders on Jan. 1, 2022. SBI Cards
Najib Mikati ■
RBI Governor Shaktikanta Das
inaugurated the ......... in Bengaluru on

The RBI has extended the card-on-file

Which European country ratified the March 24, 2022. (CoF) tokenization deadline on
Framework Agreement of the Reserve Bank Innovation Hub (RBIH) December 27, 2021 by 6 months i.e., to
International Solar Alliance (ISA) on June 30, 2022
July 22, 2021? Sweden ■
The Reserve Bank of India (RBI) on
March 11, 2022 barred which Payments

The RBI has announced on December

According to SIPRI Yearbook 2021 on Bank from opening new accounts with 23, 2021 the empanelment of CSB
June 14, 2021, India has ……. highest immediate effect? Bank, a private sector lender as an
nuclear weapons stockpile while United Paytm Payments Bank Agency Bank
States tops. sixth

New Development Bank (NDB) will

Bank of Baroda has announced on

Stockholm International Peace become the first multilateral agency to December 23, 2021, that it has won the
Research Institute (SIPRI) released the open an office in the Gujarat first position in overall digital
SIPRI Yearbook 2021 on June 14, 2021 International Finance Tech (GIFT) City transactions amongst large banks for
FY 2020-21

Which nation has registered the world's ■
JP Morgan (largest bank in the USA)
first COVID-19 vaccine for animals? announced that it has become the

Paytm Payments Bank has been given
Russia …………. to enter in the metaverse on the scheduled bank status on Dec 10,
February 18, 2022. First Bank 2021 by the Reserve Bank of India

India has jointly set up a friendship park
with which nation? Republic of Korea ■
RBI reopens Voluntary Retention Route

The Labour ministry has released the
(VRR) on February 10, 2022. With new series of wage rate index (WRI)

Arouca Bridge 516, the world’s longest
investment limit of ` 250000 crore with the base year being ……….. on
pedestrian suspension bridge and was
November 26, 2022. 2016
officially inaugurated on May 2, 2021 ■
The NPCI and UPI ecosystem observed
in northern Portugal February 1 to 7,2022 as ‘UPI Safety and

The RBI has issued a revised
Awareness Week’ and the whole of ……………. Framework on November

Which country is building the world’s
February as 5, 2022 for banks to enable supervisory
first platypus sanctuary? Australia
UPI Safety and Awareness Month intervention at “appropriate time”.

PM Narendra Modi participated in the Prompt Corrective Action (PCA)
‘Leaders’ Summit on Climate’ on ■
As per the first revised estimates
April 23, 2021, hosted by President of released by NSO, the GDP contracted

Which Indian Conglomerate has
by ……….. in FY21 (2020-21). 6.6% acquired the financial services
USA Joe Biden
company, DHFL? Piramal Enterprises

Italy has launched pilot project named ■
Reserve Bank of India has canceled the
license of Independence Co-operative

What is the interest rate fixed on the
“The Mega Food Park” virtually on
Bank Ltd., ………….. w.e.f. February 3, Post Office Savings Account in the
April 17, 2021 at Fanidhar Mega Food
2022. Nashik, Maharashtra third quarter (Oct-Dec) of 2021-22? 4%
Park, in Gujarat

https://sscstudy.com/
https://sscstudy.com/

6 CUET (UG) Section III : General Test


What is the interest rate fixed on ■
The UP-based ……….. commenced ■
NASA planned on February 7, 2022 to
Sukanya Samriddhi Accounts for operations as a SFB with effect from retire the International Space Station
Quarter-3 (Oct-Dec) of 2021-22? 7.6% April 26, 2021. by 2031 by crashing it into the

Which company has recently acquired Shivalik Small Finance Bank Ltd. Pacific Ocean
100% stake in digital lending startup ■
Who has launched its first Overseas ■
India has become the first country in
Credit Mate? Paytm Fund? SBI Mutual Funds the world on February 7, 2022to
Which bank has launched NAV-eCash administer a DNA vaccine named
■ ■
The RBI joined the Central Banks and
card for Indian Navy on INS ZyCoV-D (ZydusCadila, Ahmedabad)
Supervisors ………. as a Member on
Vikramaditya? SBI against COVID-19
April 26, 2021.

Which company has recently launched Network for Greening the ■
IIT-Hyderabad has launched an
the ‘postpe’ solution based on ‘Buy Financial System AI-based job portal named ………..
Now Pay Later’ platform? BharatPe that helps people with disabilities

According to the budget 2021-22,
acquire relevant skills and find jobs.

………… buys 3.9% stake in Bank of Pensioners (senior citizens) over
Swarajability
India via open market acquisition. LIC ............. of age are exempted from filing
returns. 75 years ■
Indian Institute of Science (IISc)

What is the current policy repo rate? 4% commissioned on February 2, 2022, one

The Reserve Bank of India (RBI) has

Which bank has topped the MeitY extended the deadline for processing of India’s most powerful
Digital Payment Scorecard for 2020-21? auto-debit payments till Sept. 30, 2021 supercomputers Param Pravega
Bank of Baroda ■
The Global Center of Excellence in

Who topped the Forbes billionaires list

S&P Global Ratings has projected India’s 2021 with a net worth of $177 billion? Affordable and Clean Energy
GDP for FY22 at 9.5% (GCoE-ACE) has been inaugurated on
Jeff Bezos
January 28, 2022 at

Which Bank has launched ‘Dukandar ■
Which bank has launched ‘UNI– IIT, Dharwad in Karnataka
Overdraft Scheme’? HDFC Bank CARBON CARD’ with HPCL?
Union Bank of India

India’s largest electric vehicle (EV)

Which is the highest-ranked Indian charging station has been inaugurated
company on the Fortune Global 500 list in Sector 52 of Gurugramon January 28,
for 2021 in terms of revenue? Science & 2022 at
Reliance Industries
Technology Delhi-Jaipur National Highway

Which bank has been authorized by the ■
The Indian armed forces have signed a

The World Bank on March 24, 2022
Reserve Bank of India (RBI) to act as contract with the Swedish defence
created the world’s first kind of .........,
an ‘Agency Bank’? company ………. for the supply of
Induslnd Bank
also known as the ‘Rhino Bond’ in
supoort of South Africa’s efforts to single-shot anti-armour weapon AT4

……… has allowed payments banks to protect the endangered species? on January 24, 2022. Saab
act as investment bankers. SEBI ■
ISRO Successfully Tests Vikas engine
Wildlife Conservation Bond (WCB)

Which organization has introduced that would power India’s first human-

The government of India has organised carry rocket on January 22, 2022 in
‘Regulatory General Anti-Avoidance
a week-long science exhibition titled Mahendragiri, Tamil Nadu
Rule (GAAR)’ for round tripping? RBI
………. from February 22 to 28, 2022.

Which digital payments platform has ■
The Israeli Defence Ministry conducted
Vigyan Sarvatra Pujyate
received direct broking license from successful test flight of the ……..…

Three more Rafale fighter aircraft anti-ballistic missile system and its
IRDAI? PhonePe
landed in India from France on Feb 22, interceptors on Jan 18, 2022. Arrow-3

The Central government on July 22, 2022, with
2021 approved the PLI scheme for

Indian Navy and Russian Navy
Indian specific enhancements
which product? Specialty Steel undertook ………… on January 14,

The Indian Navy received the 12th 2022 at the port of Cochin, in the

The LIC on July 20, 2021 launched a anti-submarine warfare aircraft P-8I Arabian Sea. PASSEX
new health insurance scheme by which from the US-based aerospace company ■
DRDO successfully test-fired an
name? Aarogya Rakshak ………. on February 23, 2022. Boeing extended range sea-to-sea variant of

El Salvador has become the first ■
……….. notified Green Hydrogen and the BrahMos supersonic cruise missile
country in the world on June 7, 2021 to Green Ammonia Policy on February 20, on January 11, 2022 for the
grant legal tender status to bitcoin 2022. Ministry of Power Indian Navy

What is the projected GDP of India for ■
India’s first commercial-scale biomass-

Covid vaccine of children between the
FY22 according to the Economic Survey based hydrogen plant will come up at age group of ………. started from
2021? 11% January 3, 2022. 15 to 18 years
Khandwa (Madhya Pradesh)

https://sscstudy.com/
https://sscstudy.com/

ONELINER CURRENT AFFAIRS 7


INS Khukri, the first indigenously built ■
The 8th edition of the Exercise Mitra ■
IIT, Ropar has developed a device
Missile Corvette, was decommissioned Shakti-21 was held between the Indian dubbed as ……… which can be used as
on December 29, 2021 after 32 years of Army and Sri Lanka a substitute of Continuous Positive
service at Airway Pressure machine. Jivan Vayu

Name the web portal launched by the
Visakhapatnam, Andhra Pradesh government to find solutions to India’s ■
Indian Navy’s oldest hydrographic

Indian Army has launched on waste problems, through community survey ship INS Sandhayak was
December 27, 2021a contemporary participation? Waste to wealth decommissioned on June 7, 2021 at
messaging application named Naval Dockyard in Visakhapatnam

JIMEX 2021 is the 5th edition of the
ASIGMA (Army Secure IndiGeneous annual bilateral maritime exercise of ■
IIT Ropar develop ‘FakeBuster’ tool to
Messaging Application) India with which country? Japan detect imposters in video conferences &

The nuclear-capable ballistic missile manipulated Faces on social media

The first ever automated, driverless
‘Agni Prime’ was successfully tested off train in the world has been unveiled in ■
India is developing a mobile tech
the coast of Odisha on December 20, which country? Germany platform called ‘UNITE AWARE’, to
2021 by DRDO make use of technology in the safety

Indian Army contingent participated in

India’s first commercial-scale and security of UN peacekeepers
Exercise ZAPAD 2021 with Russia
biomass-based hydrogen plant will ■
Indian Navy’s first destroyer ………
come up on December 13, 2021 at the ■
Name the typhoon that has recently hit
decommissioned after 41 years of
Khandwa district of Madhya Pradesh Taiwan. Chanthu
service on May 21, 2021 INS Rajput

The 3rd edition of the Trilateral ■
Which Indian frigate has conducted ■
The 19th edition of the Indian and
Maritime Exercise named SITMEX–21 maritime partnership exercise with
French Navy bilateral exercise ………..
held from 15 to 16 Nov. 21 of India, Sudanese Navy in Red Sea? INS Tabar
was conducted in the Arabian Sea from
Singapore and Thailand in ■
SpaceX has launched the first April 25-27 2021. VARUNA-2021
Andaman Sea all-tourist crew into orbit onboard Crew
Dragon capsule. Name the three days

The seventh Indian Navy frigate of the
mission. Inspiration4
Sports
P1135.6 class was launched at the ■
Which team won the prestigious
Yantar Shipyard on November 13, 2021 ■
Name the Cyclonic Storm which has Obaidulla Khan Heritage Hockey Cup
at Kaliningrad in Russia recently affected the state of Odisha Tournament 2022, that was held in

A new Indian Coast Guard Ship (ICGS) and Andhra Pradesh. Gulab Bhopal (Madhya Pradesh) from
………… has been dedicated to the ■
36th edition of CORPAT held between March 21-27, 2022?
nation on October 28, 2021. Sarthak India and Bangladesh Railway Sports Promotion Board (RSPB)

DRDO has carried successful test-firing ■
Which company has launched ‘AI For ■
Indian badminton sensation PV Sindhu
of surface-to-surface ballistic missile All’ initiative in collaboration with on March 27, 2022 won women’s
Agni-5 on October 27, 2021from CBSE, Ministry of Education? Intel singles title at which tournament
APJ Abdul Kalam Island in Odisha Swiss Open 2022

Which armed force has recently

Armed Forces of India and the United inducted its first two women officers ■
India concluded its campaign at the
Kingdom (UK) launched Tri-Service (Prakriti and Diksha) in combat for the Singapore Weightlifting International
exercise ‘Konkan Shakti 2021’ from first time? Indo Tibetan Border Police 2022 with ……… medals, including six
October 24 to 27, 2021 off the Konkan golds on February 27, 2022. eight
coast in the Arabian Sea

What name has been given to India’s
Evacuation Mission from Afghanistan?

Haryana defeated the Indian Railway,

The fifth edition of India-Japan Operation Devi Shakti to win Men’s title in ……….. on
Maritime Bilateral Exercise ………. was February 13, 2022.
held in the Arabian Sea from October 6

Which organisation on July 23, 2021 Senior National
to 8 ,2021. JIMEX announced to set-up 35 new centres in Volleyball Championship 2021-22
the country by December 2021?

The 8th edition of the India – Sri Lanka National Centre for Seismology (NCS)

In women’s category of Senior National
bilateral joint Exercise “Mitra Volleyball Championship, ………..
Shakti-21” conductedfrom October 4

Which new generation missile was defeated the Indian Railway, to lift the
to 15, 2021at Combat Training School, successfully flight-tested by the DRDO trophy on February 12, 2022. Kerala
Ampara in Sri Lanka on July 23, 2021? Akash-NG

English club Chelsea defeated ………..

The second phase of multilatral

Which Indigenously developed missile Palmeiras, 2-1, to win the FIFA Club
maritime exercise ‘Malabar 2021’ was successfully test-fired by the World Cup final 2021, held on
concluded on October 15, 2021 in the DRDO on July 21, 2021? February 12, 2022.
Bay of Bengal Man Portable-Anti Tank Guided Missile Brazilian club

https://sscstudy.com/
https://sscstudy.com/

8 CUET (UG) Section III : General Test


The Rajasthan CM Ashok Gehlot and ■
Rohan Bopanna and Ram kumar ■
Which team has won the Durand Cup
BCCI President SouravGanguly laid the Ramanathan (India) won on January 8, 2021? FC Goa
foundation stone of world’s third- 2022 2022 Adelaide International 1 ■
Name the winner of the inaugural
largest cricket stadium on February 5, Tennis Tournament Meltwater Champions Chess Tour
2022 in Jaipur ■
India has lifted the under-19 Asia (MCCT). Magnus Carlsen

Indian men’s hockey player ……….. cricket Cup by defeating Sri Lanka by ■
Name the hockey players who have
has won the World Games Athlete of nine wickets on January 3, 2022 in a won the FIH Women’s Player of the
the Year award for 2021on January 28, rain-interrupted One-Day International Year and FIH Men’s Player of the Year
2022. PR Sreejesh final in Dubai 2020-21?

Indian teenager Unnati Hooda beat ■
Senior Indian off-spinner, Harbhajan GurjitKaur and Harmanpreet Singh
compatriot Smit Toshniwal on Singh has announced retirement on ■
What is the name of the official mascot
January 30, 2022, to claim the women’s December 27, 2021 from all forms of
for U-17 Women’s World Cup 2022?
singles title at cricket
IBHA
Odisha Open badminton ■
Shuttler Kidambi Srikanth became the
tournament 2022 ■
Which country will host the FIFA U-17
first Indian man to win a silver medal
Women’s World Cup in 2022? India

Argentina defeated Chile on on December 22,2021 at
January 28, 2022 ,to win its 6th BWF World Championships ■
Praveen Kumar has clinched silver in
women’s field hockey championship Paralympics 2020. He is related to

India won a total of six medals, which sport? High Jump
title at
Women’s Pan American Cup 2022 including two gold and 4 silver medals
in the Asian Rowing Championship on

Who has won bronze medal in archery

Japan defeated South Korea in the final December 22, 2021 in Thailand in men's individual recurve open for
on January 28, 2022 to win their 3rd India at the Tokyo Paralympics 2020?
title at Women’s Hockey Asia Cup ■
India claimed 41 medals (12 Gold, Harvinder Singh
tournament 2022 and India defeated 15 Silver, 14 Bronze) at the 4th Asian
Youth Para Games (AYPG) on

Who has won the formula one Dutch
China to win bronze medal
December 11, 2021, Asia’s biggest event Grand Prix 2021? Max Verstappen

The women’s team from Ladakh has held at Riffa city (Bahrain) ■
Who has been named the ICC Men’s
lifted the 9th National Women’s Ice

In cricket, Tamil Nadu has lifted the player of the month for August?
Hockey Championship on January 21,
Syed Mushtaq Ali Trophy defeating Joe Root
2022 in Himachal Pradesh
Karnataka on November 24, 2021 after ■
Who has won the women’s singles title

In badminton, ace Indian shuttler PV chasing 152 in of US Open 2021? Emma Raducanu
Sindhu bagged the women’s singles Arun Jaitley Stadium (Delhi)
title at the Syed Modi International ■
Which country has been suspended
Tournament on January 23, 2022, in ■
India ends with 7 medals at 2021 Asian from 2022 Beijing Winter Olympics by
Lucknow Archery Championships Karnataka on the IOC? North Korea
Nov. 22 ,2021 in Dhaka (Bangladesh) ■
Who has won the Formula One Italian

Bharath Subramaniyam from Chennai
has become on January 9, 2022 the ■
Australia won their maiden T20 World Grand Prix 2021?
73rd Grandmaster of India Cup title as they defeated New Zealand Daniel Ricciardo
in the final by 8 wickets on ■
Who has won Norway Chess Open 2021

India’s first para-badminton academy
November 14, 2021 in Dubai Masters section? D. Gukesh
has been set up in
Lucknow, Uttar Pradesh ■
China beat Japan 3-1 in a thrilling final ■
Name the Formula 1 race driver who
in Aarhus, Denmark, on October 20, has won the Hungarian GP 2021.

Indian badminton player LakshyaSen
2021 to reclaim the Uber Cup Esteban Ocon
defeated World Champion Loh Kean
Yew of Singapore on Jan 17, 2022, in the ■
Chennai Super Kings (CSK) has ■
Who has become India’s 70th Chess
men’s singles final of India Open 2022 defeated Kolkata Knight Riders (KKR) Grandmaster? Raja Rithvik
on October 17,2021 in the finals to win

ICSD has given its approval to the All ■
Which country has won CONCACAF
the 2021 IPL Title
India Sports Council of the Deaf Gold Cup 2021 in Football? USA
(AISCD) on January 16, 2022 to ■
Which franchise won the title of IPL
organise the World Deaf 2021 after beating KKR in the final on

India men’s hockey team defeated ……
T20 Cricket Championship in 2023 to won the bronze medal at the Tokyo
October 15, 2021?
Chennai Super Kings (CSK)
Olympics on 5th August. Germany

World number six Rafael Nadal won
the men’s singles tennis title on ■
Indian player Rupinder Pal Singh has

………… has become the first Indian
January 9, 2022 at 2022 recently announced retirement from to win Olympic Gold medal in athletics.
Melbourne Summer Set 1 which sports? Hockey Neeraj Chopra

https://sscstudy.com/
https://sscstudy.com/

ONELINER CURRENT AFFAIRS 9


33rd Summer Olympics Games in 2024 ■
Who won Monte Carlo Masters 2021 in ■
President Ram Nath Kovind approved
will be hosted in which city? Tennis and captured maiden Masters conferment of the Jeevan Raksha
Paris, France 1000 title on April 20, 2021. Padak Series of Awards 2021on

Which country has won the maximum Stefanos Tsitsipas (Greece) January 25, 2022, a civilian lifesaving
number of medals in Tokyo Olympics award, to 51 brave citizens of the country

Who has been selected for the
2020? US Chhattisgarh Veerni Award? ■
PS Vinothraj directed film Koozhangal

Who has been named the ICC Men’s Dutee Chand from India won the best film award in
player of the month for July? the Asian Film Competition section on

FIFA has suspended which nation’s
Shakib Al Hasan January 24, 2022 at the
football federation? Pakistan
20th Dhaka International Film Festival

Who has become first Indian woman to ■
Which football team has won the ISL
win two Olympic medals in badminton ■
Former Japan Prime Minister Shinzo
League Football Winners Shield?
sports? PV Sindhu Abe was conferred with the Netaji
Mumbai City FC

PV Sindhu defeated ‘He Bingjiao’ from Award 2022 on January 23,2022 by

Which State/UT has topped the medals Netaji Research Bureau
……….. by 21-13, 21-15 in women’s
tally at the 2nd Khelo India Winter
singles bronze medal match at Tokyo ■
37 Years old Shaylyn Ford was crowned
National Games in Gulmarg?
Olympics 2020. China on January 15, 2022 as the winner of
Jammu & Kashmir

Which cricket team won the inaugural Mrs World 2022
ICC World Test Championship 2019-21 ■
Who has won the BBC’s Indian ■
The 2015 film BajrangiBhaijaan fame
after beating India on June 23, 2021? sportswoman of the year award?
actress, Harshaali Malhotra has been
New Zealand Koneru Humpy
awarded on January 11, 2022 the

Who has won the 2021 French Grand ■
Who is the 1st Indian woman cricketer 12th Bharat Ratna
Prix, held on June 20, 2021? to score 10000 international runs? Dr. Ambedkar Award 2022
Max Verstappen Mithali Raj ■
The Forensic Science Laboratory (FSL)
(Netherlands-Red Bull)
won ………… in the Silver category for

Who has been won the 2021 Azerbaijan Awards & Honours its work towards combating Crime and
Grand Prix, held on June 6, 2021 Violence against Children on

Which Indian Olympic medallist
Sergio Perez (Mexico- Red Bull) January 10, 2022. SKOCH Award
weightlifter won the ‘BBC Indian

The IBF, the apex body of broadcasters Sports Woman of the Year 2021’ award ■
The Power of the Dog and the West
in India, is being renamed on May 29, on March 28 2022? Side Story, won 3 each awards in 79th
2021 as IBDF Mirabai Chanu edition of ………… ceremony held on

Name a Hong Kong-based ■
American mathematician Dennis January 9, 2022.
mountaineer, has created the record of Sullivan has been awarded with which Golden Globe Awards 2022
world’s fastest ascent of Everest by a prize for his contributions to topology ■
Zishaan A Latif won …….. in the Photo
woman, within the time of just under and dynamic systems, on March 25, Journalism category on January 5,
26 hours. Tsang Yin-hung 2022? Abel Prize 2022 2022. Ramnath Goenka Award

Tamil Nadu teenager Arjun Kalyan on Physicist Professor, ……….. has
■ ■
People for the Ethical Treatment of
April 20, 2021 has become the become the first Indian to be awarded Animals (PETA) India has named
68th Chess Grandmaster of India the Boltzmann Medal on March 2, Bollywood star ………… its 2021

Casper Ruud of Norway beat 15th- 2022. Deepak Dhar Person of the Year on December 29,
ranked Denis Shapovalov of Canada on 2021. Alia Bhatt

The ICC has selected New Zealand
May 25, 2021 to win Geneva Open 2021
batter Daryl Mitchell on February ■
PM NarendraModi unveiled hologram

Max Verstappen (Red 2,2022as the winner of the statue of NetajiSubhas Chandra Bose at
Bull-Netherlands) has won the……, ICC Spirit of Cricket Award 2021 India Gate on December 26, 2021 on
held on May 23, 2021. Parakram Divas

The government of Assam has
2021 Monaco Grand Prix
conferred its highest civilian award ■
DivyaHegde, an Indian Climate Action

American professional golfer………., ….…… on Industrialist Ratan Tata on Entrepreneur from Udupi, Karnataka,
created history by winning the 2021 January 27, 2022. Assam Baibhav has won
PGA Championship, at the age of 50 on UN Women’s Award for Leadership

The Padma Award 2022 has been
May 23, 2021. Phil Mickelson Commitment
conferred upon 128 winners on

English professional player ………. has January 26, 2022, which include 4 ■
Seven-time Formula One champion
become the World Snooker Champion Padma Vibhushan, 17 Padma Bhushan received, …….., a knighthood at
2021 for the fourth time on May 3, and 107 Padma Shri Windsor Castle, London on
2021. Mark Selby December 17, 2021. Lewis Hamilton

https://sscstudy.com/
https://sscstudy.com/

10 CUET (UG) Section III : General Test


Actor-model ………… made history as ■
Which Indian organisation has been ■
Who has won the AM Turing Award
she was crowned Miss Universe 2021, honored with the 2021 Right Livelihood 2020? Alfred Aho
21 years after India last brought home Award? Legal Initiative for Forest ■
Who has been selected for the
the title on December 12, 2021. and Environment (LIFE), Delhi
prestigious Maharashtra Bhushan
Harnaaz Sandhu ■
Who has been conferred with the Award? Asha Bhosle

Assamese poet Nilmani Phookan Jr. Swami Brahmanand Award 2021? ■
International Federation of Film
won the 56th Jnanpith Award and Anand Kumar
Archives (FIAF) will confer the
Konkani novelist ………. won the 57th ■
Who has been honoured with the prestigious 2021 FIAF Award to whom?
Jnanpith Award on December 8, 2021. National Florence Nightingale Award Amitabh Bachchan
Damodar Mauzo 2020? SV Sarasvati ■
Who has been honoured with

Nizamuddin Revival Project of ■
Who has won the Goalkeeper ‘Global International Woman of Courage
NizamuddinBasti community, in New Goals Award 2021’? Award 2021? Gowsalya Shankar
Delhi has won the UNESCO Asia-Pacific Ghumzile Mlambo-Ngcuka
Awards on December 6, 2021 for ■
Which singer’s album ‘Folklore’ has
Cultural Heritage Conservation 2021

Who has been named the winner of the won the ‘Album of the year’ award at
prestigious National Lokmanya Tilak the 63rd Grammy Awards 2021?

Who has been conferred with the Tata Award 2021? Cyrus Poonawala Taylor Swift
Literature Live! Lifetime Achievement
Award for 2021 on November 23, 2021? ■
Name the Ashok Chakra awardee of ■
Name the Hindi Writer who has been
Anita Desai 2021, who has been given the award selected for Vyas Samman 2020.
posthumously. Babu Lal Sharad Pagare

South African playwright and novelist,
Damon Galgut has won the 2021

Who has won Prof. CR Rao Centenary ■
Which actor has won the ‘Best Actor
Booker Prize on November 8, 2021 for Gold Medal award? (Male)’ award at Dadasaheb Phalke
The Promise Jagdish Bhagwati and C. Rangarajan International Film Festival Awards

Who has been awarded the National 2021? Akshay Kumar

Who was honoured with the
prestigious 51st DadasahebPhalke Youth Award in 2021? Mohammad Azam ■
Who has been chosen to receive
Award at the 67th National Film ■
The Central government on July 19, Saraswati Samman 2020?
Awards ceremony on October 25, 2021? 2021 launche which new award? Sharankumar Limbale
Rajinikanth National Logistics Excellence Awards ■
Who has won the Best Actress award at

………… won 22nd Lal Bahadur ■
Who has received Kirti Chakra award the 67th National Awards 2021?
Shastri National Awardon October 12, in 2021? Altaf Hussain Bhat Kangana Ranaut
2021? Dr. Randeep Guleria ■
Who has won the Wildlife

Noted Zimbabwean novelist ………….

Telugu filmmaker B. Gopal, alias has been declared as the winner of Photographer of the Year People’s
Bejawada Gopal, has been chosen on 2021 Pen Pinter Prize on June 9, 2021. Choice Award 2021? Robert Irwin
October 11, 2021for the Tsitsi Dangarembga ■
Who has been crowned VLCC
fourth Satyajit Ray Award Femina Miss India World 2020?

Dr. Shakuntala Haraksingh Thilsted, an

Name the winner of the 2021 Nobel Indian-born global nutrition expert, won Manasa Varanasi
Prize for Medicine? the prestigious 2021 World Food Award ■
Which city’s Police has received the
David Julius and Best Marching Contingent Trophy

Who has been honoured with the 51st
Ardem Patapoutian 2021? Delhi
Dadasaheb Phalke Award? Rajnikanth

Who has won the Bird Photographer of ■
Who has won the Sanctuary Lifetime

Philippines Journalist ………honored
the Year 2021 award? Alejandro Prieto Service Award 2020? Theodore Baskaran
with 2021 UNESCO/ Guillermo Cano

Who has won the Women’s Prize for World Press Freedom Prize on May 3, ■
Which State’s Chief Minister has got
Fiction 2021 for her novel ‘Piranesi’? 2021. Maria Ressa the SKOCH Chief Minister of the Year
Susanna Clarke Award? Andhra Pradesh

The Marathi film “Puglya” has won the

Who has been honoured with the Best Foreign Language Feature award ■
Who has won the Nelson Mandela
ryabhata Award for 2021 by on April 17, 2021 at World Humanitarian Award?
Astronautical Society of India (ASI)? Moscow International Film Festival, 2021 Ravi Gaikwad
Dr. G. Satheesh Reddy ■
Who has been honored with the 51st ■
Who has won the Best Actor (Male)

Who is the winner of the fourth Satyajit Dadasaheb Phalke Awards, the highest award at 66th Filmfare Awards 2021?
Ray Award in 2021? BejawadaGopal film honour in India on April 3, 2021? Irrfan Khan
legendary actor Rajinikanth

https://sscstudy.com/
https://sscstudy.com/

ONELINER CURRENT AFFAIRS 11


Which has won the prestigious ■
Nightingale of India and Queen of ■
Who has been appointed as the new
International King Bhumibol World Melody .......... has died in Mumbai’s Chief of Air Staff (CAS) of the IAF?
Soil Day Award 2020? Breach Candy Hospital due to multiple Vivek Ram Chaudhari
Indian Council of Agricultural Research organ failures on February 5, 2022. ■
Who has been appointed as the first
Lata Mangeshkar

Which State’s tableau has won the 1st Chairman of the newly set up Indian
prize on Republic Day 2021 parade? ■
Veteran Indian and Marathi film and Space Association (ISpA)? Jayant Patil
Uttar Pradesh television actor, ........ , has passed away ■
Who has been appointed as the new
on February 5, 2022. Ramesh Deo
Advisor to Prime Minister Narendra
In the News ... ■
Lieutenant General GAV Reddy has Modi? Amit Khare
Name the former Chief Minister of been appointed on February 2, 2022 as
■ ■
Who has been appointed as the
Sikkim, who passed away on March 28, the Director-General of the
Chairman of Central Board of Direct
2022 at the age of 92 years. Defence Intelligence Agency Taxes (CBDT)? JB Mohapatra
Bhim Bahadur Gurung ■
Gopinath appointed as first deputy ■
Who has become the first woman CMD

Who has been appointed as the New managing director of ......... from of Engineers India Ltd.?
MD and CEO of Maruti Suzuki India on January 21 2022. IMF Vartika Shukla
March 24, 2022? Hisashi Tekuchi ■
Andhra Pradesh girl .......... becomes ■
Who has been appointed as the

RBI has approved the re-appointment first Indian to complete NASA’s IASP secretary-general of Rajya Sabha?
of Rakesh Sharma as Managing programme on January 20, 2022. PPK Ramacharyulu
Director and Chief Executive Officer of Jahnavi Dangeti

Who has been appointed as the
IDBI Bank ■
Professional mountaineer, Padma Shri president of the International Road

India’s ................ has been elected the and a retired army officer Major Federation (IRF) India? Satish Parekh
new Chairman of the International ........….... has passed away on
January 19, 2022. ■
Name the Uttarakhand Governor who
Rubber Study Group (IRSG) for a has resigned.
period of two years on February 23, Hari Pal Singh Ahluwalia
Baby Rani Maurya
2022. KN Raghavan ■
Padma Vibhushan awardee,Legendary
Kathak maestro, composer and singer,

Who has been appointed as joint

Bollywood actor Shah Rukh Khan has secretary in Ministry of Cooperation?
been appointed on February 23, 2022 as ............ has passed away, due to
age-related ailments on January 16, Abhay Kumar Singh
the brand ambassador for Thums Up
2022. Pandit Birju Maharaj ■
Oscar Fernandes passed away recently.

Who has been appointed as the new He was a/an Politician
Director of The Institute of Economic

………… elected Kuwait’s Haitham
Growth on February 20, 2022? Al-Ghais as new secretary general on ■
Who has been appointed as the new
Chetan Ghate
January 6, 2022. OPEC CEO of Yahoo? Jim Lanzone

Who has been elected as the President

Alka Mittal becomes first women head ■
Who has been appointed as the
of The Institute of Chartered of ............... on January 1, 2022. Governor of Manipur? La Ganesan
Accountants of India (ICAI) on Oil and Natural Gas Corporation (ONGC) ■
Who has been appointed as 25th CGA?
February 13, 2022 for the year 2022-23? ■
Virender Singh (VS) Pathania took Deepak Das
Debashis Mitra charge on Dec. 31, 2021 as the Director-
General of the Indian Coast Guard (ICG)

Who among the following has become

Actor and athlete ........ (Bheem in the world’s youngest mountaineer to scale
TV series “Mahabharat”) passes away ■
Somalian Prime Minister ……… has K2? Shehroze Kashif
at the age of 74 years on February 10, been suspended on December 28, 2021.
2022. Praveen Kumar Sobti

Which BJP leader took oath as the 23rd
Mohamed Hussein Roble
Chief Minister of Karnataka on July 28,

Who has been appointed as the Brand ■
Senior bureaucrat, Vivek Johri has been 2021? Basavaraj S. Bommai
Ambassador of Indian state of appointed on November 30, 2022 as the
Uttarakhand on February 8, 2022?

Chinmoy Chatterjee passed away
new Chairman of the Central Board of
recently. He was a ……… . Footballer
Bollywood actor Akshay Kumar Indirect Taxes and Customs (CBIC).

Which eminent multilingual Kannada

Who has been appointed as the new ■
Who takes charge as CMD of National
film actress passed away in Bengaluru
director of Vikram Sarabhai Space Research Development Corporation
at the age of 76 years on July 26, 2021?
Centre (VSSC) on February 7, 2022? (NRDC) on October 18, 2021?
Amit Rastogi Jayanthi
Dr. S. Unnikrishnan Nair
Name the newly appointed MD & CEO

Which Indian businessman was

Who has been appointed as the official ■

of National Securities Depository declared ‘bankrupt’ by a UK High Court


brand ambassador of MediBuddy on
(NSDL). Padmaja Chunduru on July 26, 2021? Vijay Mallya
February 7, 2022. Amitabh Bachchan

https://sscstudy.com/
https://sscstudy.com/

12 CUET (UG) Section III : General Test


Which Union Minister was appointed ■
Who has been appointed as the new
as the Deputy Leader of the House in Mission Director for AIM?
Miscellaneous
the Rajya Sabha on July 19, 2021? Dr Chintan Vaishnav ■
Union Minister of Agriculture and
Mukhtar Abbas Naqvi Farmers Welfare Narendra Singh

National Assembly of Vietnam has
Tomar on March 28, 2022 released a

Who has won the 2021 Iranian voted ………. as the new Prime book titled
presidential election on June 18, 2021? Minister of the country on April 5, Indian Agricultrue Towards 2030 :
Ebrahim Raisi 2021. Pham Minh Chinh Pathways for Enhancing Farmers Income,

Name of former Defence Minister, took ■
Niger President Mohamed Bazoum has Nutritional Security and Sustainable Food
oath as the Prime Minister of the Israel appointed ……….as the new PMof the and Farm Systems
on June 13, 2021. Naftali Bennett country effective from April 3, 2021. ■
Indian author Anirudh Suri has come

Asian Games gold medal-winning Ouhoumoudou Mahamadou out on February 25, 2022 with his new
former boxing star …………has passed ■
Who has been appointed as the new book titled
away, battling liver cancer on June 7, The Great Tech Game : Shaping
Chairman and Managing Director of
2021. Dingko Singh Geopolitics and the Destinies of Nations.
SIDBI? S. Ramann

Name of Syrian President has been ■
Award-winning US author Jimmy Soni

Name the first women cricket
re-elected for the fourth straight 7-year released on February 22, 2022 his new
commentator who passed away on
term on May 28,2021. Bashar al-Assad book titled The Founders :
April 4. Chandra Nayudu The Story of PayPal and the

The social media giant Facebook has ■
Tarun Bajaj has been appointed as the Entrepreneurs Who Shaped Silicon Valley
named ………. as its Grievance Officer
Revenue Secretary ■
The book titled “A Nation To Protect”
for India on its website. Spoorthi Priya

Who has been recommended as the authored by Priyam Gandhi Mody was

Who took the oath as the Chief launched by Union Health
Minister of Kerala for the second next Chief Justice of India (CJI) w.e.f.
Minister Mansukh Mandaviya
consecutive time on May 20, 2021? April 4, 2021? NV Ramana
Pinarayi Vijayan

A short biography of Indian athlete

Who has been re-appointed as the
………… titled ‘Golden Boy Neeraj

Name the Justice was sworn in as the Chairman of the Central Board of
Chopra’ authored by Navdeep Singh
48th Chief Justice of India on April 24, Direct Taxes?
Gill has been released on February 3,
2021. Pramod Chandra Mody
2022. Neeraj Chopra
Nuthalapati Venkata Ramana ■
Who has been appointed as the new ■
The Book titled ‘Fearless Governance’

BJP leader Himanta Biswa Sarma CEO of Unique Identification Authority authored by ………. has been released
Replaced ……… as Assam’s New Chief of India (UIDAI)? on January 31, 2022. Dr. Kiran Bedi
Minister on May 9, 2021. Saurabh Garg

India’s first innovation centre for
Sarbananda Sonowal ■
In which city, International Yoga graphene is set to come up on

AINRC founder leader ……….. has Festival has been organised? Rishikesh January 28, 2022 in Thrissur, Kerala
been sworn in as the New CM of ■
Name the Indian-American who has ■
A new book titled …………., based on
Puducherry, for a record fourth time,
been appointed Acting Chief of Staff of West Bengal CM and Trinamool
on May 7, 2021. N. Rangasamy
NASA. Bhavya Lal Congress supremo Mamata Banerjee

Padma Bhushan awardees Former released on January 24, 2022.
Attorney General of India, ……….has

Who has been appointed as the First
Mamata Beyond 2021
passed away, battling Covid-19 on Chief Compliance Officer of Facebook?
April 30, 2021. Soli Sorabjee Henry Moniz ■
India’s “Supermom” tigress, popularly
known as ……….. , has passed away at

Name the Election Commissioner

Who has been appointed acting chief of
Pench Tiger Reserve (PTR) in Madhya
assumed charge with effect from the Central Bureau of Investigation Pradesh, due to old-ageon January 16,
April 13, 2021 as Chief Election (CBI)? Praveen Sinha 2022. Collarwali
Commissioner. Sushil Chandra ■
Who has been appointed as the CEO of ■
The 18th edition of the Kachai Lemon

Name the US educated female law Amazon, after Jeff Bezos step down? Festival was held in Manipur on Jan. 13
professor has been elected as the new Andy Jassy and 14, 2022 at the Local Ground of
President of Kosovo on April 7, 2021. ■
Who has become India’s Youngest Kachai Village in Ukhrul district
Vjosa Osmani
Female Pilot at 25? Ayesha Aziz ■
India’s first heli-hub with all aviation

Name the Deputy Governor of RBI who facilities is proposed on Jan. 12, 2022 to

In which state Mandu Festival is
has retired from his service. be built at Gurugram in Haryana
celebrated? Odisha
BP Kanungo

https://sscstudy.com/
https://sscstudy.com/

ONELINER CURRENT AFFAIRS 13


Arundhati Bhattacharya, the former ■
India’s first Sports Arbitration Centre ■
Who is the author of the book ‘Human
chair of the SBI, has come out on has been unveiled in which city? Rights and Terrorism in India’?
Jan. 11, 2022 with her new book titled Ahmedabad Subramanian Swamy
Indomitable : A Working Woman’s ■
What is the theme of the International ■
What was the theme for World Ozone
Notes on Work, Life and Leadership.
Day of Older Persons 2021? Day 2021? Montreal Protocol -

A new book titled Gandhi’s Assassin: Digital Equity for All Ages Keeping us, our food and vaccines cool
The Making of NathuramGodse and ■
Which Central Armed Police Force ■
International Day of Democracy is
His Idea of India, authored by
celebrated its 83rd Raising Day on celebrated on ……… every year.
journalist ………, has been released on
July 27, 2021? CRPF September 15
January 4, 2022. Dhirendra K. Jha

Who is the author of the book ‘The ■
In India, “Muslim Women Rights Day”

Union Minister Jitendra Singh
Custodian of Trust – A Banker’s is observed across the country on
inaugurated 24th Conference on
Memoir’? Rajnish Kumar August 1
e-Governance 2020-21 on January 7,
2022 in Hyderabad,Telangana ■
Who is the writer of the book, titled ■
Who is the author of the new book
‘Know Your Rights and Claim Them : A titled ‘Balakot Air Strike: How India

……….. has retained the tops Atal
Guide for Youth’? Avenged Pulwama’? Manan Bhatt
Rankings (ARIIA) 2021 for third
Angelina Jolie &Geraldine Van Bueren
straight year on January 1, 2022. ■
What was the theme of the 75th
IIT Madras ■
Who is the author of the book titled Independence Day of India?
‘My Own Mazagon’? Ramesh Babu Nation First, Always First

Ministry of Power is celebrating the
Energy Conservation Week from ■
What was the theme for International ■
Cricketer Suresh Raina has launched
December 8 to 14, 2021 under Literacy Day 2021? his autobiography on June 14, 2021
Azadi ka Amrit Mahotsav Literacy for a human-centered recovery: titled ‘Believe-What Life taught me’
Narrowing the digital divide

Who is the author of the book named ■
Harry Potter author JK Rowling is all
‘Chronicles from the Land of the ■
Which city has become the first Indian set to release on Oct. 12, 2021 a new
Happiest People on Earth’? city to vaccinate 100% against children’s book titled The Christmas Pig
Wole Soyinka COVID-19? Bhubaneswar

https://sscstudy.com/
https://sscstudy.com/

14 CUET (UG) Section III : General Test

Practice Questions
National Affairs 9. The government of India is committed 17. Which UT has become the first Union
to become a net zero carbon emitter by Territory in the country to have ‘District
1. The first-ever road project, made out which year? Good Governance Index’?
of waste steel has come up in which city (a) 2040 (b) 2030 (a) Chandigarh
of India? (c) 2050 (d) 2070 (b) Ladakh
(a) Surat (b) Gurugram (c) Jammu and Kashmir
(c) Hyderabad (d) Jamshedpur 10. Name the scheme launched by the (d) Puducherry
Ministry of Social Justice and
2. Which state has topped the NITI Empowerment for the welfare of the 18. Consider the follow ing statements :
Aayog’s Export Preparedness Index Transgender community and the 1. According to the Ministry of Defence,
2021? beggars? for the first time, an Indian Air Force
(a) Tamil Nadu (b) Maharashtra (a) SMILE (b) WINNER (IAF) shown grand flypast by 75
(c) Gujarat (d) Uttar Pradesh (c) PROUD (d) VALUE
aircraft or helicopters.
3. Which state’s handmade 11. Which Indian city has been declared 2. 1000 indigenously developed drones
Narasingapettai Nagaswaram got as the most congested cities in the used for the ‘Beating the Retreat’
geographical identification tag under the country as per the TomTom Traffic Index ceremony, along with projection
category of musical instruments of class Ranking 2021? mapping shown for the first time.
15? (a) New Delhi (b) Bengaluru 3. Beating Retreat ceremony held at
(a) Kerala (b) Maharashtra (c) Mumbai (d) Chennai
Attari-Wagah border on 73rd Republic
(c) Odisha (d) Tamil Nadu
12. Which of the follow ing has launched Day
4. Which Indian city has been ranked ‘Operation AAHT’ to curb human Which of the above is/are correct?
second-most noise polluted city globally, trafficking? (a) Only 1 (b) 1 and 2
as per the UNEP ‘Annual Frontier Report, (a) National Security Guard (c) 2 and 3 (d) All of these
2022’? (b) Central Reserve Police Force
(a) Mumbai (b) Moradabad (c) Railway Protection Force 19. Kazhuveli Wetland has been declared
(c) Kanpur (d) Ghaziabad (d) Border Security Force as the 16th Bird Sanctuary of which of
the follow ing states?
5. Which state’s ‘Jingkieng Jri or Living 13. Which state government launched a
4-year mentorship programme ‘Project (a) Tamil Nadu (b) Karnataka
Root Bridge’ has been included in (c) Andhra Pradesh (d) Kerala
UNESCO’s tentative list of World Arohan’?
Heritage Sites? (a) Tamil Nadu (b) Uttar Pradesh 20. Which state’s Chief Minister has
(a) Meghalaya (b) Mizoram (c) Assam (d) Jharkhand launched the ‘Milk Price Incentive
(c) Tripura (c) Assam
14. Consider the follow ing statements : Scheme’?
(a) Nagaland (b) Sikkim
6. India has launched which operation to 1. Asia’s biggest Bio-CNG GOBAR- Dhan (c) Uttarakhand (d) West Bengal
bring back its nationals from Ukraine? plant has been inaugurated in Indore
(a) Operation Yamuna (Madhya Pradesh). 21. Which State government has set up
(b) Operation Saraswati 2. The plant is based on zero-landfill General Category Commission (Samanya
(c) Operation Ganga models. Varg Aayog) for upper castes?
(d) Operation Bharat (a) Andhra Pradesh
Of the above correct statement(s) is/are
(a) Both 1 and 2 (b) Only 1 (b) Uttar Pradesh
7. Which of the follow ing organisations
(c) Only 2 (d) None of these (c) Arunachal Pradesh
has launched the ‘Waste to Wealth
(d) Himachal Pradesh
Creation’ programme for women in the
15. Which State/UT has topped the India
Sundarbans in West Bengal? 22. First Drone Mela in the country has
Press Freedom Report 2021 released by
(a) National Development Council been organised at
the Rights and Risks Analysis Group
(b) NITI Aayog (RRAG)? (a) Jabalpur (b) Orchha
(c) Small Industries Development Bank of (a) Uttar Pradesh (c) Gwalior (d) Bhopal
India (SIDBI) (b) Tripura
(d) Central Vigilance Commission (c) Madhya Pradesh
23. Which State’s Askot Wildlife
(d) Jammu & Kashmir Sanctuary has been declared as the Askot
8. Which state’s Governor has flagged off Wildlife Sanctuary Eco Sensitive Zone
historic first-ever all-women officers 16. India’s first Graphene Innovation (ESZ)?
offshore sailing expedition of Indian Centre w ill be established in which of the (a) Nagaland
Army? follow ing state? (b) Sikkim
(a) Karnataka (b) Telangana (a) Karnataka (b) Telangana (c) Uttarakhand
(c) Odisha (d) Tamil Nadu (c) Tamil Nadu (d) Kerala (d) West Bengal

https://sscstudy.com/
https://sscstudy.com/

CURRENT AFFAIRS ONELINER 15

24. Which State government has 34. Consider the follow ing statements : 42. Rajiv Gandhi Kisan Nyay Yojana
launched ‘Khel Nursery Scheme 2022-23’ 1. India’s first ever emergency landing (RGKNY) is the initiative of which Indian
to promote sports? strip at National Highway for the State/UT?
(a) Haryana (b) Bihar Indian Air force Planes has been (a) Punjab (b) Kerala
(c) Rajasthan (d) Assam launched in Rajasthan. (c) Chhattisgarh (d) Andhra Pradesh

25. Which state has topped the IPF 2. With this inauguration, the NH-925 has 43. The Mera Kaam Mera Maan
Smart Policing Index 2021 released by become India’s first national highway (MKMM) scheme has been launched by
the Indian Police Foundation (IPF)? to be used for emergency landing of which state to help in enhancing skills of
(a) Andhra Pradesh IAF aircraft. unemployed youths?
(b) Gujarat Of the above correct statement(s) is/are (a) Maharashtra (b) Madhya Pradesh
(c) Uttar Pradesh (a) Both 1 and 2 (b) Only 1 (c) Punjab (d) Rajasthan
(d) Rajasthan (c) Only 2 (d) None of these
44. PM Modi dedicated to the nation
26. Which state in India has approved 35. India’s largest open-air fernery has renovated complex of Jallianwala Bagh
the proposal to launch the first Cyber been inaugurated at Smarak. The memorial is based in which
Tehsils? (a) Ranikhet (b) Darjeeling city?
(a) Gujarat (b) Uttarakhand (c) Dehradun (d) Rishikesh (a) New Delhi (b) Amritsar
(c) Madhya Pradesh (d) Rajasthan (c) Kolkata (d) Srinagar
36. Match the follow ing.
27. The Indian Railways is setting up the 45. The Rural Enterprises Acceleration
tallest pier railway bridge of the world in a VAYO NAMAN 1. Ministry of Commerce Programme ‘Saath’ has been launched by
Programme and Industry
the state of which State/UT for Self Help Group
(a) Uttarakhand (b) Tamil Nadu b Clean India 2. Ministry of Science & (SHG) women?
(c) Gujarat (d) Manipur Programme Technology (a) Chandigarh (b) Rajasthan
c peve CARE 3. Ministry of Youth Affairs (c) Jammu & Kashmir (d) Uttar Pradesh
28. Which Indian state/UT recently and Sports
observed the first Aharbal festival to 46. The government of India an July 29,
promote tourism? d National Single 4. Ministry of Social
Window System Justice and
2021 decided to give 27% reservation to
(a) Jammu & Kashmir (NSWS) Empowerment OBCs and 10% to EWSs for
(b) Assam undergraduate and postgraduate
(c) Kerala (a) a-3, b-4, c-2, d-1 (b) a-4, b-2, c-3, d-1 programmes from the academic year
(d) Rajasthan (c) a-4, b-3, c-2, d-1 (d) a-4, b-3, c-1, d-2 2021-22 in which field?
(a) Defene sector (b) Medial education
29. Which city’s Patalpani Railway 37. Which state has won the FSSAI’s (c) Economic sector (d) None of these
Station has been renamed after tribal State Food Safety Index (SFSI) 2020-21 in
Icon Tantya Bhil? large state category? 47. In which state did the Union Home
(a) Patna (b) Jaipur (a) Tamil Nadu (b) Madhya Pradesh Minister Amit Shah launh the ‘Harit
(c) Indore (d) Delhi (c) Gujarat (d) Maharashtra Sohra Afforestation Campaign’ on July 25,
2021?
30. Which city has been designated as 38. Which cartoon character has been (a) Nagaland (b) Mizoram
a part of UNESCO Creative Cities roped in as the official mascot for the (c) Tripura (d) Meghalaya
Network (UCCN) in November 2021 by Namami Gange Programme?
UNESCO? (a) Shakitmaan (b) Chacha Chaudhary 48. The Ministry of Defence has unveiled
(a) Shimla (b) Kargil (c) Chhota Bheem (d) Motu Patlu a new system for automated sanction and
(c) Srinagar (d) Prayagraj disbursement of defence pension. What is
39. What is the theme of the 7th edition the name given to that system?
31. India has been chosen as the Host of the Swachh Survekshan (SS), launched (a) MITRA (b) ROSHINI
Nation for the G20 Summit in which of by the government for the year 2022? (c) SPARSH (d) PRAYAS
the upcoming years? (a) People First (b) Citizen Participation
(a) 2023 (b) 2022 (c) Waste to Wealth (d) Sustainable Living 49. Name the new Union Cabinet
(c) 2024 (d) 2025 Minister of Health and Family Welfare to
40. Which state has received the GI tag replace Dr. Harsh Vardhan in cabinet
32. India’s tallest air purification tower for Judima, a home-made rice w ine? reshuffle.
has been installed in which State/UT of (a) Kerala (b) Assam (a) Mahendra Nath Pandey
the country? (c) Nagaland (d) Odisha (b) Mansukh Mandaviya
(a) Delhi (b) Tamil Nadu (c) G. Kishan Reddy
(c) Assam (d) Chandigarh 41. The Ministry of Information and
(d) Narayan Tatu Rane
Broadcasting has constituted a
33. Which institution has topped the Committee to review the existing 50. Select the incorrect match w ith
overall category ranking of the NIRF guidelines of Journalist Welfare Scheme. respect to the newly appointed Governor
India Rankings 2021? Who is the head of this committee? of the States
(a) IIT Madras (a) Sachidanand Murthy (a) Himachal Pradesh
(b) IISc Bengaluru (b) Ravinder Kumar – Rajendra Vishwanath Arlekar
(c) AIIMS Delhi (c) Pankaj Salodia (b) Madhya Pradesh
(d) IIT Delhi (d) Ashok Kumar Tandon – Mangubhai Chhaganbhai Patel

https://sscstudy.com/
https://sscstudy.com/

16 CUET (UG) Section III : General Test

(c) Mizoram 57. Education Minister Ramesh Pokhriyal 63. Prime Minister Shri Narendra Modi
– Hari Babu Kambhampati
released ‘Implementation Plan for School has recently inaugurated ‘Maitri Setu’
(d) Maharashtra – Thaawarchand Gehlot
Education’ under the National Education bridge, to strengthen connectivity w ith
51. Who has been named as the brand Policy (NEP) 2020. What is the name given which country?
ambassador of Khadi Prakritk Paint, to this plan? (a) Nepal (b) Bangladesh
India’s first and only paint made from (a) SMARAN (b) SARTHAQ (c) Maldives (d) Sri Lanka
cow dung? (c) ARTH (d) UDDESHYA
64. In 2021, India is observing which
(a) Nitin Gadkari (b) Narendra Modi
58. How many MoUs have been signed edition of the Dandi March celebrations,
(c) Smriti Irani (d) Amit Shah
between India and Bangladesh during the which was led by Mahatma Gandhi?
52. The Indian Railways has set up visit of PM Modi to the foreign nation to (a) 75 (b) 91 (c) 82 (d) 100
India’s first movable freshwater tunnel attend golden jubilee celebrations of
aquarium ‘Aquatic Kingdom’ in which Bangladesh’s independence in 1971? 65. The Rajya Sabha and Lok Sabha
(a) 12 (b) 9 Television channels have been merged to
city?
(c) 7 (d) 5 form a single entity called Sansad
(a) Shimla (b) Cochin
(c) Bengaluru (d) Surat
Television. Who has been appointed as
59. The Gram Ujala Scheme has been the CEO of this Sansad Television?
53. Choose the correct statements launched by government to offer LED (a) Om Birla (b) M. Venkaiah Naidu
bulbs at ` 10 in rural areas. The scheme (c) Ravi Capoor (d) Naresh Jain
1. Prime Minister Narendra Modi has
w ill cover how many states in the first
unveiled the mYoga mobile app.
phase? 66. PM Modi has inaugurated the
2. mYoga app is available in English, (a) 5 (b) 7 Chilahati-Haldibari rail link, which w ill
Hindi and French. (c) 3 (d) 10 connect India w ith which country?
3. mYoga app is preloaded with many (a) Thailand (b) Myanmar
yoga training videos and audio practice 60. Which state has launched the (c) Nepal (d) Bangladesh
sessions. ‘Launch Pad Scheme’ for above 18 years
Code
youths, coming out of child care 67. The Government of India has signed
(a) 1 and 2 (b) 1 and 3 institutions? an MoU on June 22, 2021 w ith the
(c) 1, 2 and 3 (d) None of these (a) Haryana (b) Madhya Pradesh Government of Fiji for cooperation in the
(c) Uttarakhand (d) Kerala field of
54. India’s first ever private Liquefied (a) Defense sector
Natural Gas (LNG) facility plant has been 61. Which film has won the best (b) Agriculture and allied sectors
inaugurated in which city? non-feature film award at the 67th (c) Sports sector
(a) Pune (b) Dehradun National Film Awards? (d) IT Sector
(c) Ahmedabad (d) Nagpur (a) The Shower
(b) An Engineered Dream 68. Name the Yoga dedicated mobile app
55. Which among these is the newly (c) Elephants Do Remember launched by the Government of India in
introduced ministry in the Union Cabinet (d) Khisa collaboration w ith WHO on the occasion
of Government of India? of International Yoga Day on June 21, 2021
(a) Ministry of Relations 62. The world’s first Double Stack Long (a) s-Yoga (b) p-Yoga
(b) Ministry of Services Haul Container Train was flagged off by (c) m-Yoga (d) i-Yoga
(c) Ministry of Synergy PM Modi recently. The train w ill run
(d) Ministry of Cooperation between which states? 69. Which city has topped the Ease of
(a) Mahatshtra and Bihar Living Index 2020, for being the most
56. What is the rank of India in the (b) Haryana & Rajathan liveable city of India?
Human Freedom Index 2020? (c) Jammu & Kashmir & Delhi (a) Hyderabad (b) Bengaluru
(a) 124 (b) 107 (d) Tamil Nadu and Odisha (c) New Delhi (d) Mumbai
(c) 92 (d) 111

ANSWERS
1. (a) 2. (c) 3. (d) 4. (b) 5. (a) 6. (c) 7. (c) 8. (b) 9. (d) 10. (a)
11. (c) 12. (c) 13. (c) 14. (a) 15. (d) 16. (d) 17. (c) 18. (d) 19. (a) 20. (c)
21. (d) 22. (c) 23. (c) 24. (a) 25. (a) 26. (c) 27. (d) 28. (a) 29. (c) 30. (c)
31. (a) 32. (d) 33. (a) 34. (a) 35. (a) 36. (c) 37. (c) 38. (b) 39. (a) 40. (b)
41. (d) 42. (c) 43. (c) 44. (b) 45. (c) 46. (b) 47. (d) 48. (c) 49. (b) 50. (d)
51. (a) 52. (c) 53. (c) 54. (d) 55. (d) 56. (d) 57. (b) 58. (d) 59. (a) 60. (b)
61. (b) 62. (b) 63. (b) 64. (b) 65. (c) 66. (d) 67. (b) 68. (c) 69. (b)

https://sscstudy.com/
https://sscstudy.com/

CURRENT AFFAIRS ONELINER 17

International Affairs
1. Which country hosted BIMSTEC 13. PM Modi hosted the first 22. Aziz Akhannouch has been appointed
Summit 2022? India-Central Asia Summit virtually. How as the new Prime Minister of which
(a) Sri Lanka (b) India many countries are officially recognised country?
(c) Bangladesh (d) Thailand as Central Asian region countries? (a) Turkey (b) Algeria
(a) 5 (b) 9 (c) Morocco (d) Israel
2. Robert Abela has been re-elected as (c) 16 (d) 24
the Prime Minister of which country? 23. Choose the correct statement
(a) Portugal (b) Maldives 14. Three countries have jointly topped regarding Fumio Kishid.
(c) Cyprus (d) Malta the Corruption Perceptions Index (CPI) 1. He has been appointed as the 100th
2021 released by the Transparency Prime Minister of China.
3. Which city has been adjudged as the International. Which of these country
world’s most polluted capital city in 2. Hereplaced outgoing party leader
among them?
IQAir’s 2021 World Air Quality Report? Prime Minister Yoshihide Suga.
(a) New Zealand, Finland and Denmark
(a) Dushanbe (b) Muscat (b) Norway, New Zealand and Finland Codes
(c) Dhaka (d) New Delhi (c) France , Finland and Japan (a) Both 1 and 2 (b) Only 1
(d) Denmark , Israel and Norway (c) Only 2 (d) None of these
4. India, along w ith which country,
agreed on ‘Roadmap on Blue Economy 15. Which country will build world’s first 24. The World Expo 2020 has been
and Ocean Governance’? floating city by 2025? organised in Dubai. What is the theme of
(a) The USA (b) The UAE (a) India (b) China
the expo?
(c) France (d) Japan (c) Singapore (d) South Korea (a) Culture and Education
(b) Connecting Minds, Creating the
5. France government has decided to 16. Which country became the first Future
w ithdraw its military from Mali after nine country to transition to 4.5 day work (c) Live Green, Live Better
years, and shift its military base to which week? (d) Building The World of Tomorrow
country? (a) Qatar
(a) Chad (b) Senegal (b) Oman 25. What was the theme of the 76th
(c) Niger (d) Mauritania (c) Saudi Arabia session of the United Nations General
(d) United Arab Emirates Assembly (UNGA) held on September 25,
6. Paul-Henri Sandaogo Damiba has been
declared as President of 2021?
17. Which country has joined the ISA as (a) The Future We Want, the UN We Need
(a) Chad (b) Ecuador its 101st member?
(c) Mauritania (d) Burkina Faso (b) Effective responses to global crises
(a) Singapore (b) Germany
(c) France (d) The USA (c) Making the United Nations relevant to all
7. Which country has topped the EIU’s people
2021 Democracy Index?
18. Which country has planned to (d) Building resilience through hope
(a) Singapore (b) Australia
establish world’s first Bitcoin City?
(c) New Zealand (d) Norway 26. The first-ever, bronze statue of
(a) United States (b) Switzerland
8. Xiomara Castro sworn in as the first (c) Singapore (d) El Salvador Bitcoin’s anonymous creator Satoshi
woman President of which of the Nakamoto has been unveiled in which
follow ing country? 19. Which country has set the city?
(a) Nicaragua (b) Guatemala Guinness Record for the world’s largest (a) San Salvador (b) Dubai
(c) Mexico (d) Honduras orchestra? (c) Budapest (d) Madrid
(a) Venezuela (b) Argentina
9. Which country has topped in the (c) Chile (d) Ecuador 27. PM Modi virtually addressed the 6th
Corruption Perception Index (CPI) 2021 Eastern Economic Forum (EEF) recently.
released by Transparency International? 20. Which country has topped the global Which country was the host of the EEF
(a) New Zealand (b) Denmark list of TRACE Bribery Risk Rankings 2021? Summit 2021?
(c) Finland (d) All of these (a) Iceland (a) France (b) Russia
(b) Singapore (c) Germany (d) Australia
10. Nusantara has been selected as the (c) Finland
new capital of which of the follow ing (d) Denmark 28. The Children’s Climate Risk Index’
country? introduced by UNICEF on August, 2021.
(a) South Korea (b) Japan 21. What was the theme of the BRICS Accordingly, which of the follow ing
(c) Vietnam (d) Indonesia Summit 2021? statement is correct?
(a) BRICS@15: Stronger Partnership for a
11. What is the rank of India in the latest 1. It ranks countries based on children’s
Brighter Future
Henley Passport Index 2022? exposure to climate and environmental
(b) BRICS@15: Intra-BRICS Cooperation for
(a) 73 (b) 83 (c) 94 (d) 104
shocks
Continuity, Consolidation and Consensus
(c) BRICS@15: Collaboration for inclusive
2. Climate crisis puts Indian kids at
12. Which word has been chosen as the ‘extremely high risk’
growth and shared prosperity in the 4th
Oxford University Press Children’s Word 3. The 33 extremely high risk countries
Industrial Revolution
of the Year 2021? emit only 9.38 % of global greenhouse
(d) BRICS@15: Economic Growth for an
(a) Anxiety (b) Coronavirus gas emissions
(c) Sanitizer (d) Isolation Innovative Future

https://sscstudy.com/
https://sscstudy.com/

18 CUET (UG) Section III : General Test

Codes (a) Black list (b) Grey list 46. Russia has started its biggest Nuclear
(a) Only 1 (b) Only 2 (c) Red list (d) None of these
Power Project with which nation?
(c) Only 3 (d) All of these
37. Where was 16th meeting of Security (a) China (b) US (c) Japan (d) India
29. Name the President of Afghanistan Council Secretaries of Shanghai
who has stepped down after the control Cooperation Organisation (SCO) held?
47. The first BRICS Employment
of the country was taken by the Taliban Working Group (EWG) Meeting of 2021
(a) Dushanbe (Tajikistan)
forces? was held recently. Which country is the
(b) Baku (Azerbaijan)
(c) Ashgabat (Turmenistan) chair of BRICS in 2021?
(a) Ashraf Ghani (b) Mohammed Omar
(c) Rashid Khan (d) Abdullah Abdullah (d) Tashkant (Uzbekistan) (a) South Africa (b) Russia
(c) India (d) Brazil
30. Which country has topped the 2021 38. Which country signed the framework
Global Manufacturing Risk Index? agreement of the International Solar 48. Which country has topped the World
(a) France (b) Australia Alliance (ISA) and Instrument of Press Freedom Index 2021?
(c) United States (d) China Ratification w ith India in June, 2021? (a) Norway (b) Finland
(a) Portugal (b) Sweden (c) Switzerland (d) Sweden
31. Which of the follow ing option is not (c) Italy (d) Denmark
true regarding UN resolution on vision? 49. Which country has been inducted as
(a) It encourages countries to institute a 39. A 1098-carat diamond has been the latest and 23rd member of the Indian
whole of government approach to eye discovered by researchers, which is Ocean Rim Association (IORA)?
care. deemed to be the world’s third-largest (a) France (b) Afghanistan
diamond ever found. The diamond has (c) Australia (d) Malaysia
(b) It calls on international financial
institutions and donors to provide been recovered in which country? 50. India has handed over 1 MW
targeted financing. (a) Mauritius (b) Zimbabwe Romainville Island Solar Power Plant to
(c) According to the resolution, at least (c) Botswana (d) South Africa
2 billion people are living with vision
which country?
impairment. 40. Ebrahim Raisi has won the Iranian (a) Sri Lanka (b) Indonesia
(d) None of the above presidential election 2021 of which (c) Mauritius (d) Seychelles
country ? 51. The ‘Malala Yousafzai Scholarship
32. Abiy Ahmed has won the second (a) Ukraine (b) Qatar (c) Syria (d) Iran
five-year term as the Prime Minister of Act’ has been passed by which country for
which of the follow ing countries? 41. The 47th G7 Leaders’ Summit 2021 the welfare of Pakistani-diaspora female?
(a) Chile (b) Ethiopia took place in a Hybrid format at (a) Germany (b) India
(c) Nigeria (d) Egypt Cornwall, (c) United States (d) France
(a) France (b) USA 52. Which country has topped the World
33. The government of India has signed (c) United Kingdom (d) Germany
an MoU on June 22,2021 w ith the Happiness Report 2021?
Government of Fiji for cooperation in the 42. Isaac Herzog, has been elected as (a) Germany (b) Singapore
field of the new President of which country? (c) Finland (d) Norway
(a) Defense sector (a) Israel (b) Oman 53. Which country was the largest arms
(b) Agriculture and allied sectors (c) Turkey (d) Lebanon exporter in the world during 2016-2020
(c) Sports sector
43. Which country has become the first as per latest SIPRI report?
(d) IT Sector
in the world to grant legal tender status (a) United States (b) China
34. The tallest sandcastle of the world to bitcoin? (c) Russia (d) Germany
has been inaugurated in which country? (a) Costa Rica (b) El Salvador 54. As per the 2020 IQAir World Air
(a) Russia (b) Denmark (c) Italy (d) France (c) Ecuador (d) Colombia Quality Report, which city has been
35. What is the rank of India in terms of 44. Bashar al-Assad is the President of adjudged as the most polluted capital city
recipient of FDI inflows in the year 2020, which country, who has been re-appointed in the world?
as per the World Investment Report 2021 for fourth straight term recently? (a) Beijing (b) Kathmandu
by the UNCTAD? (c) Dhaka (d) New Delhi
(a) Syria (b) Mali
(a) 3rd (b) 5th (c) 7th (d) 6th (c) The Bahamas (d) Oman 55. Which country was the host of the
36. Financial Action Task Force (FATF) 45. Isaac Herzog, has been elected as ‘One Planet Summit’ 2021, held in
released its updated list of countries in the new President of which country? collaboration w ith UN and World Bank?
June, 2021, where Pakistan remained in (a) Israel (b) Oman (a) Australia (b) France
the (c) Turkey (d) Lebanon (c) India (d) Germany

ANSWERS
1. (a) 2. (d) 3. (d) 4. (c) 5. (c) 6. (d) 7. (d) 8. (d) 9. (d) 10. (d)
11. (b) 12. (a) 13. (a) 14. (a) 15. (d) 16. (d) 17. (d) 18. (d) 19. (a) 20. (d)
21. (b) 22. (c) 23. (c) 24. (b) 25. (d) 26. (c) 27. (b) 28. (d) 29. (a) 30. (d)
31. (d) 32. (b) 33. (b) 34. (b) 35. (b) 36. (b) 37. (a) 38. (d) 39. (c) 40. (d)
41. (c) 42. (a) 43. (b) 44. (a) 45. (a) 46. (a) 47. (c) 48. (a) 49. (a) 50. (d)
51. (c) 52. (c) 53. (a) 54 (d) 55. (b)

https://sscstudy.com/
https://sscstudy.com/

CURRENT AFFAIRS ONELINER 19

Economy & Banking


1. The Dearness Allowance for Central 11. Choose the correct statements 19. The RBI has granted small finance
government employees has been hiked by regarding Economic Survey 2021-22 bank licence to Unity Small Finance
how much percent? 1. The central theme of Economic Survey Bank. It is a consortium of which entities?
(a) 1% (b) 2% 2021-22 is the Agile Approach. (a) Centrum and BharatPe
(c) 3% (d) 4% (b) PFC Ltd. and Aditya Birla
2. The Economic Survey has predicted 8
(c) Tata Capital and Bandhan
2. Reserve Bank Innovation Hub (RBIH) to 8.5% GDP growth in Financial Year
(d) LIC and L&T Finance
has been inaugurated in which city and 2022-23.
what is the initial capital contribution for 3. India is the 3rd largest start-up 20. Which company has launched the
RBIH? ecosystem in the world after US and ‘postpe’ solution based on ‘Buy Now Pay
(a) Mumbai; ` 200 crore China. Later (BNPL)’ platform?
(b) Bengaluru; ` 100 crore Codes (a) Paytm (b) BharatPe
(c) New Delhi; ` 200 crore (a) 1 and 2 (b) 2 and 3 (c) Whatsapp (d) PhonePe
(d) Chennai; ` 100 crore (c) 1, 2 and 3 (d) None of thsee
21. Bank of Baroda has launched its
3. The ‘AutoFirst’ application has been 12. Which of these countries has been digital banking platform under which
launched by which bank to offer fully added as a new member of the BRICS New name to bring all banking services under
automated auto loans? Development Bank in December 2021? one roof?
(a) HDFC Bank (a) Nepal (b) Italy (a) bob Ahead (b) bob Now
(b) ICICI Bank (c) Iran (d) Egypt (c) bob Unite (d) bob World
(c) Kotak Mahindra Bank
(d) IndusInd Bank 13. Which bank has launched 22. Match the follow ing.
feature-rich savings bank product for
4. Which country will become the a. Public Provident Fund 1. 6.9%
women named “Mahila Mitra Plus”?
first country to adopt India’s UPI (a) RBL Bank b. National Savings 2. 7.6%
platform? (b) Karur Vysya Bank Certificate
(a) Nepal (b) Myanmar (c) Federal Bank c. Sukanya Samriddhi 3. 6.8%
(c) Bangladesh (d) Sri Lanka (d) DCB Bank Yojana
5. Which bank has become the first to 14. Which payments bank has recently d. Kisan Vikas Patra 4. 7.1%
enter in the Metaverse? gat the RBI’s approval to function as a (a) a-3, b-4, c-2, d-1 (b) a-4, b-2, c-3, d-1
(a) Barclays (b) JPMorgan scheduled payments bank? (c) a-4, b-3, c-2, d-1 (d) a-4, b-3, c-1, d-2
(c) Citibank (d) UBS (a) Paytm Payments Bank
(b) Airtel Payments Bank 23. Who has topped the IIFL Wealth
6. Which city in India has highest number (c) NSDL Payments Bank Hurun India Rich List 2021?
of dollar-millionaire households according (d) Jio Payments Bank
to the Hurun India Wealth Report 2021? (a) Shiv Nadar (b) Gautam Adani
(c) Mukesh Ambani (d) SP Hinduja
(a) Bengaluru (b) Kolkata 15. Which company has partnered w ith
(c) Delhi (d) Mumbai Mastercard on Tokenisation for 24. Which among the follow ing banks in
Card-Based Payments? the only one left under the RBI’s PCA
7. In accordance w ith the Bloomberg (a) PhonePe regime?
Billionaires Index, who among the (b) Paytm
follow ing has become Asia’s richest (a) Central Bank of India
(c) Google Pay (b) UCO Bank
person as on February 8, 2022? (d) Amazon Pay (c) Bank of Maharashtra
(a) Aziz Premji (b) Gautam Adani
(d) Canara Bank
(c) Ratan Tata (d) Mukesh Ambani 16. Which organisation has recently
launched the HARBINGER 2021 – 25. The government has recently
8. The RBI has directed certain Innovation for Transformation hackathon? enhanced the income limit of disabled
Non-Banking Financial Companies (a) SEBI (b) SIDBI dependents to be eligible for family
(NBFCs) to mandatorily implement ‘Core (c) SBI (d) RBI
Financial Services Solution (CFSS)’ by pension for life from Ministry of Defence.
(a) March 2025 (b) March 2024 17. Under RBI’s new directives, what is What is the new limit?
(c) December 2024 (d) September 2025 the minimum initial capital requirement (a) less than 20% of the last pay drawn
to set-up a new universal bank? (b) less than 10% of the last pay drawn
9. How many digital banking systems (a) ` 1000 crore (b) ` 800 crore (c) less than 30% of the last pay drawn
w ill be set-up in 75 districts by Scheduled (c) ` 700 crore (d) ` 500 crore (d) less than 40% of the last pay drawn
Commercial Banks?
(a) 25 (b) 50 (c) 75 (d) 100 18. CBI has recently registered the 26. PM Modi inaugurated a special
biggest ever bank fraud case by which commemorative coin of what value to
10. Nirmala Sitharaman announced a company? mark the birth anniversary of ISKCON
new tax rule for taxpayers where a (a) Berger Paints founder Srila Bhaktivedanta Swami
taxpayer can file an updated return on (b) ABG Shipyard Prabhupada?
payment of taxes w ithin? (c) Mercator Limited (a) ` 100 (b) ` 125
(a) 1 year (b) 2 years (c) 3 years (d) 4 years (d) Shalimar Works (c) ` 200 (d) ` 250

https://sscstudy.com/
https://sscstudy.com/

20 CUET (UG) Section III : General Test

27. Centre has launched Pradhan Mantri 33. Assertion (A) India was the fifth (a) World Bank
(b) Asian Development Bank
Gatishakti scheme for holistic largest recipient of FDI inflows in the
(c) International Monetary Fund
development of infrastructure sector and world in 2021.
(d) World Economic Forum
boosting employment. What is the approx Reason (R) Received $ 64 billion FDI in
amount that will be invested in the 2020, which is an increase of 50%. 40. NITI Aayog has recommended the
scheme? Code names of which bank for privatisation in
(a) ` 50 trillion (b) ` 100 trillion (a) A is true but R is false. 2021-22?
(c) ` 150 trillion (d) ` 200 trillion (b) Both A and R are true and R is the (a) Indian Overseas Bank and Punjab
correct explanation of A. National Bank
28. The RBI has announced an increase
in the incentives given to commercial (c) Both A and R are true but R is not the (b) Bank of India and Canara Bank
banks for distribution of coins per bag correct explanation of A. (c) Central Bank of India and Indian
from September 2021. What w ill be the (d) Both A and R are false. Overseas Bank
new incentive per bag? (d) Canara Bank and Bank of Maharashtra
(a) ` 45 (b) ` 65 (c) ` 75 (d) ` 135
34. Which Bank has launched an
initiative called ‘Salaam Dil Sey’ to pay 41. LIC Cards Services (LIC CSL) has
29. Which of these countries have been tribute and show gratitude to doctors launched a contactless prepaid gift card,
inducted as the new member country of across the country for their tireless ‘Shagun’, in collaboration w ith
the New Development Bank (NDB) set-up service during the pandemic ? (a) Bank of India
by BRICS nation? (a) SBI (b) HDFC Bank (b) ICICI Bank
(c) ICICI Bank (d) PNB (c) SBI
1. Bangladesh 2. UAE 3. Uruguay
(d) IDBI Bank
4. Syria 5. Zimbabwe 35. The government has recently
Codes extended the superannuation age of LIC. 42. Which Indian bank has joined hands
(a) 1 , 2 and 4 (b) 2, 3 and 4 Chairman.What is the revised age for the w ith NPCI to link its digital wallet
(c) 1, 2 and 3 (d) All of these same? Pockets w ith UPI ID?
(a) 65 years (b) 70 years (a) Canara Bank (b) SBI
30. The SAMRIDH programme has (c) HDFC Bank (d) ICICI Bank
(c) 62 years (d) 60 years
recently been launched by which ministry
to boost start-up ecosystem in India? 36. In June 2021, Union Minister Nitin 43. TRIFED has collaborated with NITI
(a) Ministry of Electronics & Information Gadkari launched ‘MSME Prerna’, a first Aayog to implement Van Dhan Yojana
Technology of its kind programme by scheme in the Tribal Aspirational Districts
(b) Ministry of Statistics and Programme (a) Canara Bank (b) SBI identified by NITI Aayog. How many Tribal
Implementation (c) Indian Bank (d) PNB Aspirational Districts are there presently?
(c) Ministry of Micro, Small and Medium (a) 29 (b) 41
Enterprises 37. Which NBFC has received approval (c) 39 (d) 18
(d) Ministry of Health & Family Welfare from RBI to take over PMC bank and
re-launch it as a Small Finance Bank? 44. The National Digital Financial
31. Recently Government of India has (a) Credila Financial Services Private Infrastructure (NADI) is a next-generation
amended Insolvency and Bankruptcy Limited Digital Financial Infrastructure, being
Board of India (Insolvency Resolution (b) Satin Creditcare Network Limited build by body?
Process for Corporate Persons) (c) HDB Financial Services Limited (a) National Payments Corporation of India
Regulations, 2016.It w ill be effective (NPCI)
(d) Centrum Financial Services Limited
from which date ? (b) State Bank of India (SBI)
(a) July 14, 2021 (b) July 19, 2021 38. What is the rank of India in terms of (c) NITI Aayog
(c) July 22, 2021 (d) July 24, 2021 recipient of Foreign Direct Investment (d) Institute for Development and Research
(FDI) inflows in the year 2020, as per the in Banking Technology (IDRBT)
32. Which among these sectors have World Investment Report 2021 by the
been included as MSMEs sector by UNCTAD? 45. RBI has excluded Lakshmi Vilas Bank
government for purpose of Priority Sector (a) 3rd (b) 5th from the Second Schedule of the Reserve
Lending? (c) 7th (d) 6th Bank of India Act, 1934, follow ing the
(a) Retail and Wholesale trade merger of the Lakshmi Vilas Bank w ith
(b) Forestry and logging 39. Which financial body has signed (a) DBS Bank India Ltd
(c) Fishing and aquaculture $484 million loan w ith Indian government (b) HSBC Bank
to improve transport connectivity in the (c) United Overseas Bank
(d) Bee-keeping and production of honey
Chennai–Kanyakumari Industrial Corridor (d) Citibank
and beeswax
(CKIC)?

ANSWERS
1. (c) 2. (b) 3. (a) 4. (a) 5. (b) 6. (d) 7. (b) 8. (d) 9. (c) 10. (b)
11. (c) 12. (d) 13. (c) 14. (a) 15. (c) 16. (d) 17. (a) 18. (b) 19. (a) 20. (b)
21. (d) 22. (c) 23. (c) 24. (a) 25. (c) 26. (b) 27. (b) 28. (b) 29. (c) 30. (a)
31. (a) 32. (a) 33. (d) 34. (b) 35. (c) 36. (c) 37. (d) 38. (b) 39. (b) 40. (c)
41. (d) 42. (d) 43. (c) 44. (d) 45. (a)

https://sscstudy.com/
https://sscstudy.com/

CURRENT AFFAIRS ONELINER 21

Science & Technology


1. The first edition of Indian Ocean Naval 11. India’s largest Electric Vehicle (EV) (a) Apple (b) Meta
Symposium (IONS) Maritime Exercise charging station inaugurated at (c) Alphabet (d) Samsung
2022 (IMEX-22) was held in which state? (a) Jaipur (Rajasthan )
(a) Gujarat (b) Goa (b) Mumbai (Maharashtra) 20. Which country has become the first
(c) Kerala (d) Maharashtra (c) Chennai (Tamil Nadu) in the world to approve a pill for treating
(d) Gurugram (Haryana) cases of symptomatic COVID-19?
2. The Indian Army version of Medium (a) Britain (b) Singapore
Range Surface to Air Missile (MRSAM) 12. Which of the follow ing company has (c) The USA (d) Australia
has been developed by DRDO in unveiled an artificial intelligence
partnership w ith which country? supercomputer ‘AI Research Super 21. What is the rank of India in the
(a) Israel (b) Russia Cluster (RSC)’? Climate Change Performance Index
(c) France (d) United States (a) IBM (b) Alphabet Inc. (CCPI) 2022 by Germanwatch?
(c) Tesla (d) Meta Inc. (a) 10th (b) 12th
3. Name the joint military training (c) 7th (d) 15th
exercise between the Indian army and 13. Choose the correct statements
Uzbekistan which kicked off in March regarding MPATGM : 22. Name the fourth Scorpene class
2022 at Yangiarik. submarine received by Indian Navy under
1. DRDO successfully flight tested the
(a) Ex-Shared Destiny the Project 75.
final deliverable configuration of Man
(b) Ex-Dustlik Portable Anti-Tank Guided Missile (a) INS-Vela (b) INS-Vagir
(c) Ex-Khanjar (c) INS-Vayu (d) INS-Veer
(MPATGM).
(d) Ex-Sea Breeze
2. The indigenously developed anti-tank 23. The Zircon hypersonic cruise missile
4. Which Naval Ship was recently missile is a low weight, fire & forget was successfully test-fired by which
presented the prestigious President’s missile and is launched from a man country?
Colour? portable launcher, integrated with (a) Egypt (b) Israel
(a) INS Valsura (b) INS Gomati thermal sight. (c) Russia (d) United Arab Emirates
(c) INS Brahmaputra (d) INS Betwa Codes
(a) Only 1 (b) Only 2 24. The World Health Organisation has
5. Which Naval Command of Indian Navy (c) Both 1 and 2 (d) Neither 1 nor 2 designated the coronavirus variant,
has conducted offshore security exercise B.1.1.529 named ………… as ‘Variant of
‘Prasthan’ off the Mumbai coast? 14. What type of orbit placement has Concern (VOC)’.
been planned by the NASA for recently (a) Alpha (b) Beta
(a) Western Naval Command
(b) Southern Naval Command
launched James Webb Telescope? (c) Delta (d) Omicron
(c) Northern Naval Command (a) LEO (b) L2
(d) Eastern Naval Command (c) Geo-synchronous (d) None of these 25. Consider the follow ing statements :
1. Norway topped the Global Drug Index
6. Which country hosted multilateral 15. Which country has topped the 2021.
exercise MILAN 2022? “World Talent Ranking Report 2021”
published by IMD World Competitive Centre? 2. India ranked at 18 out of 30 countries
(a) India (b) France with an overall score of 46/100.
(c) The UK (d) Malaysia (a) Switzerland (b) Norway
(c) Sweden (d) Finland Of the above statement(s), the correct is/are
7. Which nation has successfully tested (a) Both 1 and 2 (b) Only 1
its new naval air defence system called 16. Which country has topped the Global (c) Only 2 (d) None of these
‘C-Dome’? Health Security (GHS) Index 2021?
(a) Thailand (b) Finland
26. The multinational ‘Blue Flag-2021’
(a) Israel (b) Iran air combat exercise was hosted by which
(c) Turkey (d) North Korea (c) Australia (d) USA
country?
8. The ISRO successfully launched which 17. Which organisation has successfully (a) Turkey (b) Russia
earth observation satellite, in its first launched a long-range Supersonic Missile (c) Israel (d) Singapore
launch of the year 2022? Assisted Release of Torpedo (SMART)
System from Abdul Kalam Island, off the 27. Which country has recently unveiled
(a) EOS-2 (b) EOS-5 the world’s largest hydrogen fuel cell
(c) EOS-3 (d) EOS-4 Odisha coast?
(a) IAF (b) DRDL
power plant?
9. Which country test fired its (c) DRDO (d) HAL (a) South Korea (b) Israel
intermediate-range ballistic missile (c) Germany (d) Singapore
Hwasong-12? 18. Which institution has topped the Atal
Ranking of Institutions on Innovation 28. i-Drone, a drone-based vaccine
(a) China (b) Japan delivery model for Northeast states, has
(c) North Korea (d) South Korea Achievements (ARIIA) 2021?
(a) IIT Bombay (b) IIT Madras
been developed by which organisation?
10. Which country has delivered all the (c) IIT Kanpur (d) IIT Delhi (a) ISRO
contracted 70000 AK-203 assault rifles to (b) Indian Council of Medical Research
India? 19. Isomorphic Labs is a newly launched (c) DRDO
(a) Israel (b) Afghanistan wholly owned subsidiary of which (d) Indian Institute of Technology,
(c) Germany (d) Russia company? Madras

https://sscstudy.com/
https://sscstudy.com/

22 CUET (UG) Section III : General Test

29. The ISRO has decommissioned which 37. How many Indian beaches have been 46. On July 29, 2021 Russia’s module
communications satellite through 11 awarded w ith the Blue Flag Certification docked w ith international space station.
Re-orbiting manoeuvres after 14-year by the Foundation for Environment What is the name of this module?
service? Education in Denmark? (a) Chaka (b) Nauka
(a) INSAT-2E (a) 10 (b) 8 (c) 12 (d) 9 (c) Luna (d) None of these
(b) KALPANA-1
(c) GSAT-29 38. JIMEX 2021 is the 5th edition of the 47. The Indian Army has named a firing
(d) INSAT-4B annual bilateral maritime exercise of range at Gulmarg in Jammu & Kashmir
India w ith which country? after which of these Bollywood actress?
30. Which organisation for the first time (a) Germany (b) Thailand (a) Priyanka Chopra
successfully demonstrated India’s (c) Italy (d) Japan (b) Vidya Balan
indigenous technology, Quantum Key (c) Diya Mirza
Distribution (QKD) link between Prayagraj 39. The digital platform ‘e-Source,’ is (d) Anushka Sharma
and Vindhyachal in Uttar Pradesh? being developed by which institution to
(a) DRDL (b) DRDO tackle the problem of e-waste? 48. Choose the correct statements :
(c) ISRO (d) HAL (a) IIT Madras (b) IIT Delhi 1. The Union Defence Minister Rajnath
(c) IIT Kanpur (d) IIT Hyderabad Singh has informed that India’s first
31. Match the follow ing. Indigenous Aircraft Carrier (IAC-I),
40. Consider the follow ing statements : is planned to be commissioned by
(a) Gaofen-5 02 1. North
Korea 1. Rajnath Singh dedicated indigenously 2022.
(b) Tsirkon (Zircon) 2. US
built Coast Guard Ship Vigraha to the 2. Once commissioned, the carrier will
hypersonic cruise missile nation in Chennai. be rechristened as INS Vikrant, in
(c) Hypersonic Air-breathing 3. Russia 2. ICGS Vigraha is seventh in the series memory of India’s first aircraft
Weapon Concept (HAWC) of Offshore Patrol Vessels. carrier.
(d) Anti-aircraft missile 4. China Of the above correct statement(s) is/are 3. The IAC-1 caarier is being built at the
(a) Only 1 (b) Only 2 Cochin Shipyard Limited , Kochi,
(a) a-3, b-4, c-2, d-1 (b) a-4, b-2, c-3, d-1
(c) Both 1 and 2 (d) None of these Kerala, under public-private
(c) a-4, b-3, c-2, d-1 (d) a-4, b-3, c-1, d-2
partnership.
32. Which among these space-tech 41. The annual military exercise Code
KAZIND-21 is being conducted between
startups has become the first private (a) 1 and 2 (b) 2 and 3
the Indian Army and the Army of (c) 1, 2 and 3 (d) None of these
company to formally sign agreement w ith
ISRO for using its expertise and facilities? (a) Kyrgyzstan (b) Kazakhstan
(c) Kenya (d) Kuwait 49. The Ministry of Defence has inked
(a) Dhruva Space
(b) Agnikul Cosmos ` 499 crore contract w ith which entity for
42. India’s first indigenously developed the manufacture and supply of Akash
(c) Bellatrix Aerospace Quantum Computer Simulator (QSim)
(d) Skyroot Aerospace missiles to the Indian Air Force (IAF)?
Toolkit has been launched. The toolkit has
(a) Hindustan Aeronautics Limited
33. i-Drone, a drone-based vaccine been developed by which institution? (b) Bharat Heavy Electricals Limited
delivery model for Northeast states, has 1. Centre for Development of Advanced (c) Bharat Dynamics Limited
been developed by which organisation? Computing (C-DAC) (d) Ordnance Factory Board
(a) ISRO 2. Indian Institute of Technology
Roorkee 50. Which space agency has planned to
(b) Indian Council of Medical Research
3. Indian Institute of Science, Bengaluru launch the world’s first wooden satellite,
(ICMR)
Codes WISA Woodsat, into orbit by the end of
(c) DRDO
(a) 1 and 2 (b) 2 and 3 the year 2021?
(d) Indian Institute of Technology, Madras (a) NASA (b) ESA
(c) 1 and 3 (d) All of these
(c) ISRO (d) Roscosmos
34. Hwasong-8 is a new hypersonic 43. India’s first green hydrogen
missile, which was tested successfully by electrolyzer manufacturing unit has been 51. The Ministry of Defence has recently
which country recently? unveiled in which city? inked contract w ith which company for
(a) North Korea (b) South Korea (a) Bengaluru (b) Visakhapatnam construction of two Pollution Control
(c) Japan (d) Taiwan (c) Chennai (d) Hyderabad Vessels (PCVs) for Indian Coast Guard
(ICG)?
35. The maiden successful test fight of 44. What is the name of India’s first (a) Garden Reach Shipbuilders & Engineers
which new advanced version of the Akash Cattle Genomic Chip for the conservation Ltd.
Missile was carried out by DRDO? of pure varieties of indigenous cattle breeds?
(a) Akash Prime (b) Akash Delux (b) Mazagon Dock Shipbuilders Ltd.
(a) BharGau (b) GauDham (c) Goa Shipyard Ltd.
(c) Akash Fine (d) Akash A1 (c) IndiGau (d) GauLok
(d) Hindustan Shipyard Ltd.
36. Name the system launched by 45. Which Indian ship participated in the
Supreme Court for transferring Maiden Maritime Partnership Exercise
52. With which of the follow ing naval
e-authenticated copies from courts to force, India has conducted a joint
2021 conducted by the Indian Navy w ith
prisons? exercise in Gulf of Aden?
Algerian Navy?
(a) SAFER (b) FASTER (a) SAARC (b) SCO
(a) INS Talwar (b) INS Tabar
(c) WINNER (d) GIFT (c) NATO
(c) INS Teg (d) INS Trikand
(d) European Union

https://sscstudy.com/
https://sscstudy.com/

CURRENT AFFAIRS ONELINER 23

53. Which space agency has planned to 62. The supercomputer named ‘Simorgh’ 69. Russia launched 38 foreign satellites
launch the world’s first wooden satellite, has been developed by which country from 18 countries into orbit in March
WISA Woodsat, into orbit by the end of recently? 2021. Which rocket system was used for
the year 2021? (a) France (b) Iran this launch?
(a) NASA (c) South Korea (d) Finland (a) Soyuz-2.1a (b) Soyuz-2.1b
(b) European Space Agency (c) Soyuz-2.1v (d) Soyuz-2.1s
(c) ISRO 63. Union cabinet has approved the
(d) Roscosmos proposal of exporting the indigenously- 70. The Indian Army has
developed Akash Missile System. The decommissioned M-46 Catapult Guns and
54. Prime Minister Narendra Modi missile comes under which category? Tampella Mortars, two of the longest
virtually addressed the VivaTech 2021. (a) Surface to Surface Missile serving artillery gun systems from
The event has been organised in which (b) Air to Surface Missile service. The caliber of M-46 Catapult
city? (c) Surface to Air Missile Guns and Tampella Mortars were ............
(a) London (b) Rome (c) Paris (d) Dubai (d) Air to Air Missile and ........... respectively.
(a) 190 mm and 190 mm
55. The Conference of BRICS Network 64. The Military Junta of which nation has (b) 120 mm and 150 mm
Universities 2021, has been organised at arrested the President, Prime Minister and (c) 100 mm and 110 mm
which Indian institute? Defence Minister? (d) 130 mm and 160 mm
(a) IIT Kanpur (b) IIT Bombay (a) Zambia (b) Mali
(c) IIT Delhi (d) IIT Guwahati (c) Ethiopia (d) Eritrea 71. The web portal and mobile app,
‘e-Sampada’ has been launched by which
56. The mobile app ‘Namaste Yoga’, 65. DRDO conducted successful maiden sector?
devoted to Yoga, has been launched by test firing of the 5th generation Python-5 (a) Central Public Works Department
which Ministry? missile in April,2021. The state of the art (b) Directorate of Printing
(a) Ministry of AYUSH missile is which type of missile? (c) Land & Development Office
(b) Ministry of Youth Affairs and Sports (a) Air-to-Air Missile (d) Directorate of Estates
(c) Ministry of Health and Family Welfare (b) Surface-to-Air Missile
(d) Ministry of Information & Broadcasting (c) Air-to-Surface Missile 72. The first-ever ‘Hot Air Balloon
(d) Surface-to-Surface Missile Wildlife Safari’ of India in a tiger reserve
57. Kappa and Delta, which has been in has been launched in which tiger reserve?
news recently, is related to 66. The Nehru Zoological Park is the (a) Sariska Tiger Reserve
(a) Satellite launch (b) Covid variant first zoo in India to get the ISO 9001:2015 (b) Satpura Tiger Reserve
(c) Cyclone (d) Food Production Quality Management Standards (c) Bandhavgarh Tiger Reserve
58. CBSE has collaborated w ith which Certificate from United Kingdom. The zoo (d) Kanha Tiger Reserve
tech giant to introduce Coding and Data is situated in which city?
(a) Bengaluru
73. The Union cabinet has recently
Science as new subjects in 2021-22 approved new guidelines for DTH
academic sessions? (b) Nagpur
(c) Jaipur broadcasting services, as per which, the
(a) Google (b) Microsoft (c) IBM (d) TCS DTH services w ill now be issue licences
(d) Hyderabad
for a period of .......... years.
59. The sustainable electric 67. India has signed an agreement
two-wheeler,named ‘PiMo’ has been (a) 20 (b) 10
w ith which country for cooperation in (c) 15 (d) 30
developed by which institution?
India’s first human space mission,
(a) IIT Madras (b) IIT Delhi 74. The mobile app ‘Satark Nagrik’ and
(c) IIT Kanpur (d) IIT Kharagpur Gaganyaan?
(a) Russia (b) Israel
Departmental Vigilance Officers (DVO)
60. The multilateral military exercises, (c) United States (d) France Portal has been launched by which state
‘Steadfast Defender 2021’, has been for easy grievance redressal?
organised by which group of 68. NITI Aayog has launched a national (a) Delhi (b) Madhya Pradesh
inter-governmental organisation? digital repository on health and nutrition. (c) Jammu & Kashmir (d) Uttar Pradesh
(a) G-20 (b) NATO (c) OPEC (d) BRICS What is the name given to this
repository? 75. DRDO has partnered w ith which
61. Kappa and Delta, which has been in country for the development of MRSAM
(a) Aushadhi Gyan
news recently, is related to (b) Poshan Gyan
missile?
(a) Satellite launch (b) Covid variant (c) Fitness Gyan (a) France (b) Russia
(c) Cyclone (d) Food production (d) Aarogya Gyan (c) Israel (d) United States

ANSWERS
1. (b) 2. (a) 3. (b) 4. (a) 5. (a) 6. (a) 7. (a) 8. (d) 9. (c) 10. (d)
11. (d) 12. (d) 13. (c) 14. (b) 15. (a) 16. (d) 17. (c) 18. (b) 19. (c) 20. (a)
21. (a) 22. (a) 23. (c) 24. (d) 25. (a) 26. (c) 27. (a) 28. (b) 29. (d) 30. (b)
31. (c) 32. (d) 33. (b) 34. (a) 35. (a) 36. (b) 37. (a) 38. (d) 39. (a) 40. (c)
41. (b) 42. (d) 43. (a) 44. (c) 45. (b) 46. (b) 47. (b) 48. (c) 49. (c) 50. (c)
51. (c) 52. (d) 53. (b) 54. (c) 55. (b) 56. (a) 57. (b) 58. (b) 59. (a) 60. (b)
61. (b) 62. (b) 63. (c) 64. (b) 65. (a) 66. (d) 67. (d) 68. (b) 69. (a) 70. (d)
71. (d) 72. (c) 73 (a) 74. (c) 75. (c)

https://sscstudy.com/
https://sscstudy.com/

24 CUET (UG) Section III : General Test

Sports
1. Who has been roped in as the new 11. Match the follow ing columns Championship in 2022 held in Himachal
President of Badminton Association of Pradesh?
A. Rio Open 2022 1. Dabang Delhi
India (BAI) for four years from (a) Chandigarh
2022-2026? B. Mexican Open 2022 2. Andrey Rublev (b) Ladakh
(a) Jyotiraditya Scindia C. Dubai Tennis 3. Rafael Nadal (c) Delhi
(b) Kiren Rijiju Championships 2022 (d) Indo-Tibetan Border Police
(c) Himanta Biswa D. PKL 8 4. Carlos Alcaraz
(d) Tarun Gogoi 19. Name the Indian shuttler who has
(a) a-3, b-4, c-2, d-1 (b) a-4, b-2, c-3, d-1 won silver at BWF World Tour Finals
2. Which player has won the Formula (c) a-4, b-3, c-2, d-1 (d) a-4, b-3, c-1, d-2 2021.
One 2022 Saudi Arabian Grand Prix? (a) Saina Nehwal (b) PV Sindhu
(a) Max Verstappen (b) Lewis Hamilton 12. Which cricket team has won the (c) Sania Mirza (d) N. Sikki Reddy
(c) Charles Leclerc (d) Carlos Sainz Jr. under-19 Asia Cricket Cup 2021 in Dubai?
(a) Nepal (b) Pakistan 20. Who among the follow ing has been
3. PV Sindhu beat which player to lift (c) India (d) Sri Lanka honoured w ith Sports Journalists’
women’s singles title at the 2022 Sw iss Federation of India (SJFI) Medal 2021?
Open Badminton Tournament? 13. Who among the follow ing has won (a) Ravi Shastri (b) Rahul Dravid
(a) Pornpawee Chochuwong the Men's Singles title of Australian Open (c) Sunil Gavaskar (d) Anil Kumble
(b) Busanan Ongbamrungphan 2022?
(c) Yeo Jia Min (a) Daniil Medvedev (b) Novak Djokovic 21. Who has been named Time
(d) Ratchanok Intanon (c) Alexander Zverev (d) Rafael Nadal magazine’s 2021 Athlete of the Year?
(a) Victoria Azarenka (b) Jen Brady
4. Wrist Assured is the autobiography of 14. Name the Indian cricketer who has (c) Naomi Osaka (d) Simone Biles
which former cricket player? been named in 2021 ICC Women’s T20I
(a) EAS Prasanna (b) Dilip Vengsarkar team of the year. 22. Who has become the first Indian man
(c) GR Viswanath (d) Sunil Gavaskar (a) Mithali Raj (b) Smriti Mandhana to w in a silver medal at BWF World
(c) Jhulan Goswami (d) Shafali Verma Championships?
5. Name the Indian cricket player who (a) Srikanth Kidambi
has been felicitated w ith the ‘Sports Icon’ 15. Which player has won the Tata Steel (b) Sandeep Gupta
award at the Maldives Sports Awards Chess Tournament 2022 in Masters (c) Varun Kapur
2022? section? (d) B. Sai Praneeth
(a) Rohit Sharma (a) Magnus Carlsen
(b) Hikaru Nakamura
23. Name the w inner of the Mexico City
(b) Virender Sehwag
(c) Garry Kasparov Grand Prix 2021.
(c) S. Sreesanth
(d) Suresh Raina (d) Fabiano Caruana (a) Charles Leclerc (b) Max Verstappen
(c) Sergio Perez (d) Lewis Hamilton
6. Which country has topped the medal 16. Consider the follow ing statements
table of Winter Olympics Games 2022 regarding Syed Modi International Title 24. Which player has won the WTA
held in Beijing? 2022 : Finals 2021 tennis title?
(a) Garbine Muguruza
(a) India (b) Norway 1. PV Sindhu clinched the Syed Modi
(b) Simona Halep
(c) Germany (d) China International Title 2022 after beating (c) Victoria Azarenka
Malvika Bansod in the women’s singles (d) Karolína Pliskova
7. Who won Mexican Open 2022 men’s final at the Babu Banarasi Das Indoor
singles title?
Stadium. 25. Who has been appointed as the new
(a) Rafael Nadal
2. It was also Sindhu’s first BWF title permanent CEO of the International
(b) Cameron Norrie
since winning the BWF World Cricket Council (ICC)?
(c) Daniil Medvedev
(d) Stefanos Tsitsipas Championships in 2019, after losing (a) Mark Boucher (b) Dave Richardson
twice in the finals last year—at the (c) Jay Shah (d) Geoff Allardice
8. Sadia Tariq won a gold medal for India Swiss Open and the BWF World Tour
in which event? Finals.
26. Which team has won the Syed
Choose the correct statements Mushtaq Ali Trophy 2021-22 in cricket?
(a) Wushu (b) Fencing
(c) Boxing (d) Weightlifting (a) Only 1 (b) Only 2 (a) Karnataka (b) Tamil Nadu
(c) Both 1 and 2 (d) None of these (c) Madhya Pradesh (d) Gujarat
9. Which team has won the FIFA Club
World Cup Championship 2021? 17. Which player has won the women’s 27. Who has become the youngest
single Indian Open (Badminton) 2022? bowler to take 400 T20 w ickets?
(a) Chelsea (b) Manchester City
(c) Liverpool (d) Palmeiras (a) Busanan Ongbamrungphan (a) Rashid Khan (b) Jasprit Bumrah
(b) Ratchanok Intanon (c) Stuart Broad (d) Dale Steyn
10. Which country has become the (c) Akane Yamaguchi
world's top ranked T20 team in the latest (d) Nozomi Okuhara
28. Which player has won the 2021
ICC T20 Team Rankings? Denmark Open men’s single badminton
18. Which team has won the 9th championship?
(a) South Africa (b) India
(c) Australia (d) New Zealand National Women’s Ice Hockey (a) Kento Momota (b) Viktor Axelsen
(c) Chen Long (d) Anders Antonsen

https://sscstudy.com/
https://sscstudy.com/

CURRENT AFFAIRS ONELINER 25

29. India beat which team to w in the (a) KD Jadhav 46. India’s Asian Gold-medalist Milkha
(b) Dhanraj Pillay
2021 SAFF Championship? Singh, who has passed away on June 18,
(c) Milkha Singh
(a) Sri Lanka (b) Nepal 2021, was associated w ith which sports
(d) Major Dhyan Chand
(c) Malaysia (d) Japan event?
38. Indian para-athlete Mariyappan (a) Boxing (b) Athletics
30. How many medals have been won by Thangavelu has claimed a silver in which (c) Weightlifting (d) Shooting
Indian Shooting team at the 2021 ISSF
event at Tokyo 2020 Paralympics?
Junior World Championship? 47. Who among the follow ing has won
(a) Discuss Throw (b) Javelin Throw
(a) 43 (b) 31 (c) 52 (d) 28 the French Grand Prix 2021?
(c) High Jump (d) Shooting
(a) Lewis Hamilton (b) Max Verstappen
31. Choose the correct statement 39. Avani Lekhara has won the first gold (c) Sergio Perez (d) Charles Leclerc
regarding US Open 2021. medal for India at Tokyo Paralympics
1. Russia’s Daniil Medvedev won the 2020 in which event? 48. Name the w inner of women’s singles
men’s singles title. (a) Archery (b) Shooting title at French Open 2021.
2. Leylah Fernandez won the women's (c) Tennis (d) Badminton (a) Sofia Kenin
singles title. (b) Elise Mertens
Codes 40. Indian para-athlete Sumit Antil has (c) Katerina Siniaková
(a) Both 1 and 2 (b) Only 1 claimed gold medal in which event at the (d) Barbara Krejcikova
(c) Only 2 (d) None of these ongoing Tokyo Paralympics?
(a) Discuss Throw (b) Javelin throw
49. CRICURU is India’s first AI-enabled
32. How many medals have been won by (c) Shotput (d) Long Jump coaching website, launched by which
Indian contingent at the 2021 World Indian cricket player?
Archery Championships? 41. Who has become the first Assamese (a) Sourav Ganguly (b) Yuvraj Singh
(a) 3 Bronze (b) 1 Gold woman to assure Indian an Olympic (c) Virender Sehwag (d) Sachin Tendulkar
(c) 2 Silver (d) 3 Silver medal?
(a) Mirabai Chanu (b) Mary Kom
50. Sanjeet Kumar, is an Indian sports
33. Jyothi Surekha Vennam has recently (c) Lovlina Borgohain (d) Hima Das person, related to which sports category?
won silver medal in which sports event (a) Weightlifting (b) Shooting
for India at the international platform? 42. Name the w inner of the 2021 (c) Sprinting (d) Boxing
(a) Archery (b) Badminton Wimbledon women’s singles title in
(c) Tennis (d) Shooting Tennis? 51. Pooja Rani has won the gold medal
(a) Sofia Kenin (b) Ashleigh Barty on May 30,2021 for India in which sports
34. What is the official motto of the (c) Simona Halep (d) Karolina Pliskova event?
Beijing 2022 Winter Olympics and (a) Wresling (b) Shooting
Paralympics? 43. Name the chess player who has (c) Boxing (d) Sprinting
(a) Together for a Shared Future recently become the youngest-ever
(b) Faster, Higher, Stronger – Together Grandmaster in chess history by breaking 52. In Tennis, who has won the men’s
(c) Passion lives here the previous record held by Sergey single title at the Italian Open 2021?
(d) With glowing hearts Karjakin. (a) Novak Djokovic
(a) Abhimanyu Mishra (b) Anish Giri (b) Rafael Nadal
35. Name the Indian para-athlete who (c) Nihal Sarin (d) Samay Raina (c) Stefanos Tsitsipas
has won two medals at the ongoing Tokyo (d) Daniil Medvedev
Games in Shooting event? 44. The BCCI has formed a working
(a) Mariyappan Thangavelu group to look into Domestic Cricket, 53. Which football team has won the
(b) Sharad Kumar including the compensation package, UEFA Champions League 2020-21?
(c) Devendra Jhajharia amid Pandemic. How many members are (a) Chelsea
(d) Avani Lekhara there in this group? (b) Manchester City
(a) 9 (b) 7 (c) Liverpool
36. Harvinder Singh has become the (c) 11 (d) 15 (d) West Ham United
first ever Indian para-athlete to w in a
medal in …… event at the Paralympics 45. Who has been named as the Captain 54. Iga Sw iatek has won the women’s
(a) Shotput (b) Archery for the Indian Men’s Hockey Team, single title at the Italian Open 2021
(c) Discuss Throw (d) Badminton participating in Tokyo Olympics 2021? Tennis tournament. She represents which
(a) Rupinder Pal Singh country?
37. The National Sports Day is observed (b) Harmanpreet Singh
in India to mark the birth anniversary of (a) Russia (b) Poland
(c) Manpreet Singh (c) Spain (d) Mexico
which Indian sports person? (d) Surender Kumar

ANSWERS
1. (c) 2. (a) 3. (b) 4. (c) 5. (d) 6. (b) 7. (a) 8. (a) 9. (a) 10. (b)
11. (c) 12. (c) 13. (d) 14. (b) 15. (a) 16. (c) 17. (a) 18. (b) 19. (b) 20. (c)
21. (d) 22. (a) 23. (b) 24. (a) 25. (d) 26. (b) 27. (a) 28. (b) 29. (b) 30. (a)
31. (b) 32. (d) 33. (a) 34. (a) 35. (d) 36. (b) 37. (d) 38. (c) 39. (b) 40. (b)
41. (c) 42. (b) 43. (a) 44. (b) 45. (c) 46. (b) 47. (b) 48. (d) 49. (c) 50. (d)
51. (c) 52. (d) 53. (a) 54. (b)

https://sscstudy.com/
https://sscstudy.com/

26 CUET (UG) Section III : General Test

Awards and Honours


1. Which film has won the Best Picture Of the above correct statement(s) is/are 17. Which city has received the Swachh
award at the 94th Academy Awards? (a) Both 1 and 2 (b) Only 1 Survekshan Award 2021 for being the
(a) Belfast (c) Only 2 (d) None of these Cleanest City of India?
(b) CODA (a) Ahmedabad (b) Indore
(c) The Power of the Dog 9. How many peoples have been (c) Mumbai (d) Surat
(d) King Richard awarded ‘Padma Bhushan’ in Padma
Awards 2022? 18. Match the follow ing columns :
2. Match the follow ing columns : (a) 4 (b) 17
A. Vihaan Agarwal 1. Tata Literature Live!
A. Stockholm Water 1. Mario Marcel (c) 52 (d) 74 and Nav Lifetime Achievement
Prize for 2022 Agarwal Award 2021
10. Who has won the award for the Best
B. TIME100 Impact 2. Dennis Parnell B. M. Mukundan 2. Indian Personality of
National Costume at the Mrs. World 2022 the Year 2021
Awards 2022 Sullivan
pageant?
C. Abel Prize 2022 3. Deepika Padukone C. Hema Malini and 3. JCB Prize for
(a) Suman Rao (b) Sanjana Vij Prasoon Joshi Literature 2021
D 2022 Governor of 4. Wilfried Brutsaert (c) Vartika Singh (d) Navdeep Kaur
. the year award D. Anita Desai 4. International
11. Who has been conferred w ith the Children’s Peace
Codes Prize 2021
A B C D A B C D Assam’s highest civilian honour ‘Assam
(a) 3 4 2 1 (b) 4 2 3 1 Baibhav’ for the year 2021? Codes
(c) 4 3 2 1 (d) 4 3 1 2 (a) Ratan Tata (b) Adar Poonawalla A B C D A B C D
(c) Azim Premji (d) Narayana Murthy (a) 3 4 2 1 (b) 4 2 3 1
3. Name the w inner of Miss World 2021 (c) 4 3 2 1 (d) 4 3 1 2
beauty pageant? 12. Prime Minister Narendra Modi was
(a) Milena Sadowska conferred with which country’s highest 19. Who has been awarded the Best
(b) Karolina Bielawska civilian award, Ngadag Pel gi Khorlo [Order Actor award at the BRICS Film Festival
(c) Izabella Krzan of the Dragon King (Druk Gyalpo)]? 2021?
(d) Rozalia Mancewicz (a) Vietnam (b) Bhutan (a) Vikram Gokhale
(c) Japan (d) Thailand (b) Jitendra Bhikulal Joshi
4. Which state has topped under the Best (c) Kunal Kapoor
State category at the third National Water 13. Who has won “Woman of the Year (d) Dhanush
Awards? Award” at World Athletics Awards 2021?
(a) Uttar Pradesh (b) Madhya Pradesh (a) Manasi Joshi 20. Name the w inner of the 2021 Booker
(c) Karnataka (d) Gujarat (b) Koneru Humpy Prize for Fiction.
(c) PV Sindhu (a) Richard Powers (b) Maggie Shipstead
5. Which airport has been adjudged as (d) Anju Bobby George (c) Marion Hänsel (d) Damon Galgut
the ‘Best Airport’ under the General
Category at Wings India 2022? 14. Name the Sahitya Akademi and 21. Name the w inners of the 2021
(a) Rajiv Gandhi International Airport Padma Shri awardee Assamese poet who Global Leadership Award, by the US India
has won the 56th Jnanpith Award. Business Council (USIBC)?
(b) Chhatrapati Shivaji International Airport
(a) Akkitham Achuthan Namboothiri (a) Shiv Nadar and Mallika Srinivasan
(c) Kempegowda International Airport
(b) Amitav Ghosh (b) N. Chandrasekaran and Jim
(d) Netaji Subhash Chandra Bose
(c) Shanka Ghosh Taiclet
International Airport
(d) Nilmani Phookan (c) Sundar Pichai and Adena Friedman
6. Which Indian mathematician was (d) Adar Poonawalla and Adil Zainulbhai
15. Who has won the 2021 Miss
conferred ‘Ramanujan Prize for Young Universe crown?
Mathematicians 2021’? 22. Which programme has won the
(a) Adline Castelino (b) Vartika Singh outstanding drama series at the 2021
(a) Neena Gupta (c) Andrea Meza (d) Harnaaz Sandhu Emmy Award?
(b) R. Praggnanandhaa
(c) Dr. B. Uma Shankar (a) Mare of East town (b) The Crown
16. Match the follow ing columns : (c) Ted Lasso (d) Hacks
(d) Dr. Kuntal Ghosh
A. SAIL 1. RedInk Award for
7. Which film won Best Film award at ‘Journalist of the Year’ 2020 23. Dr. Firdausi Qadri and Muhammad
the Dadasaheb Phalke International Film Amjad Saqib, who were seen in the news
B. Divya 2. THE Asia Awards 2021
Hedge recently, are the recipients of which
Festival Awards 2022?
award?
(a) Shershaah (b) 83 C. OP Jindal 3. UN Women’s Award for
Global Leadership Commitment (a) Booker Prize
(c) Mimi (d) Pushpa
University (b) Ramon Magsaysay Award
8. Consider the follow ing statements : D. Danish 4. Golden Peacock
(c) Pulitzer Award
Siddiqui Environment Management (d) Abel Prize
1. Bill Gates has been conferred with
Award 2021
‘Hilal-e-Pakistan’. 24. Name the w inner of Bird
2. Hilal-e-Pakistan is the highest civilian Codes Photographer of the Year (BPOTY)
award of the Islamic Republic of A B C D A B C D 2021?
Pakistan. (a) 3 4 2 1 (b) 4 2 3 1 (a) Felipe Foncueva (b) Alejandro Prieto
(c) 4 3 2 1 (d) 4 3 1 2 (c) Jonas Classon (d) Maofeng Shen

https://sscstudy.com/
https://sscstudy.com/

CURRENT AFFAIRS ONELINER 27

25. Name the w inners of the 2021 Nobel accomplishments in the area of tobacco 41. Name the w inner of the Pandit
Prize in Physics. control Deendayal Upadhyay Telecom Skill
(a) Syukuro Manabe, Klaus Hasselmann and (a) Narendra Modi (b) Smriti Irani Excellence Award
Giorgio Parisi (c) Harsh Vardhan (d) Piyush Goyal (a) Sreenivas Karanam
(b) Andrea M. Ghez, Jim Peebles and Michel 33. Who is the w inner of the 2021 (b) Sreelakshmi Suresh
Mayor (c) Deepanjali Dalmia
International Booker Prize?
(c) Reinhard Genzel, Didier Queloz and Kip (d) Ranveer Allahbadia
(a) David Diop
Thorne (b) Olga Tokarczuk
(d) Arthur (b) McDonald, Peter Higgs and
42. Name the Indian film which has won
(c) Marieke Lucas Rijneveld the Best Foreign Language Feature award
Adam Riess (d) Han Kang at Moscow International Film Festival
26. Name the Indian activist who has 34. Who among the follow ing has won (a) Choked (b) Panga
been felicitated w ith the prestigious Pulitzer Prize 2021 in ‘International (c) Bulbbul (d) Puglya
Commonwealth Points of Light award by Reporting category’ for reporting on
the UK government recently?
43. Name the w inner of the Sir Richard
China’s detention camps for Muslims? Hadlee Medal for the 2020-21 season at
(a) Yasir Nadeem al Wajidi (a) Megha Rajagopalan (b) Alison Killing the New Zealand Cricket Awards?
(b) Hucheshwar Gurusidha Mudgal (c) Christo Buschek (d) All of the above
(c) Bharat Bhushan Tyagi (a) Glenn Turner
(d) Syed Osman Azhar Maqsusi 35. Who among the follow ing has won (b) Ross Taylor
(c) Kane Williamson
the Miss Universe title 2020?
27. Which amongst the follow ing is not (d) Chris Cairns
(a) Julia Gama (b) Andrea Meza
included in the India-based enterprises (c) Adline Castelino (d) Kimberly Jimenez
announced Best Small Business by United
44. Name the Indian filmmaker who has
been honored with the second highest
Nations? 36. Who is the w inner of the
French civilian honour “Knight of the
(a) Edible Routes Private Limited UNESCO/Guillermo Cano World Press
Order of Arts and Letters”.
(b) Oorja Development Solutions India Freedom Prize 2021?
(c) Taru Naturals (a) Sunil Bohra (b) Anurag Kashyap
(a) Cheche Lazaro (b) Rodrigo Duterte
(d) Zydus Wellness (c) Shaan Vyas (d) Guneet Monga
(c) Maria Ressa (d) Mike Enriquez

28. Who won the Maharashtra Bhushan 45. Name the female player who has
37. Which nation honoured Nobel
Award 2021? bagged “The Best FIFA Women’s Player”
Laureate Amartya Sen w ith its top award
(a) Sachin Tendulkar (b) Lata Mangeshkar
for 2020?
in social sciences?
(c) Asha Bhosle (d) Ratan Tata (a) Ellen White (b) Jill Scott
(a) Denmark (b) France (c) Lucy Bronze (d) Toni Duggan
(c) Spain (d) Germany
29. The European Union’s Sakharov
Prize 2020 has been awarded to the 46. Who has been conferred w ith the
38. Who has been chosen as the Tansen Samman for 2020?
opposition movement of which country? “People’s Advocate” of the UN Climate (a) Pt. Satish Vyas
(a) Belarus (b) Chile Change Conference (COP26)? (b) Pt. Vidyadhar Vyas
(c) Argentina (d) Singapore (a) Louis Theroux (c) Pt. Ulhaas Kashalkar
(b) Steve Backshall (d) Pt. Dalchan Sharma
30. Who won the Lokmanya Tilak (c) Trevor McDonald
National Award 2021? (d) David Attenborough 47. Who has been awarded the
(a) Sanjay Gupta (b) Deepak Tilak Saraswati Samman, 2020?
(c) Cyrus Poonawalla (d) Adar Pooonawalla 39. Who has won the title of Sir Garfield (a) K. Siva Reddy
Sobers Award for ICC Male Cricketer of (b) Sharankumar Limbale
31. Who has been awarded the the Decade 2020? (c) Vasdev Mohi
prestigious Central European University
(a) Kumar Sangakkara (b) Ravi Ashwin (d) Sitanshu Yashaschandra
(CEU) Open Society Prize for 2021? (c) Virat Kohli (d) Steve Smith
(a) Sharankumar Limbale 48. Which film has won the best film
(b) Rumana Sinha Sehgal 40. Which film has won the Best Picture award at the 66th Filmfare Awards for
(c) KK Shailaja Award at the 93rd Academy Awards or the year 2021?
(d) Guneet Monga Oscars 2021? (a) Thappad
(a) The Father (b) Tanhaji : The Unsung Warrior
32. Name the Indian who has been (c) Angrezi Medium
(b) Judas And The Black Messiah
awarded w ith the ‘WHO Director- (d) Ludo
(c) Nomadland
General Special Recognition Award’ for (d) Minari

ANSWERS
1. (b) 2. (c) 3. (b) 4. (a) 5. (c) 6. (a) 7. (b) 8. (b) 9. (b) 10. (d)
11. (a) 12. (b) 13. (d) 14. (d) 15. (d) 16. (c) 17. (b) 18. (c) 19. (d) 20. (d)
21. (a) 22. (b) 23. (b) 24. (b) 25. (a) 26. (d) 27. (d) 28. (c) 29. (a) 30. (c)
31. (c) 32. (c) 33. (a) 34. (d) 35. (b) 36. (c) 37. (c) 38. (d) 39. (c) 40. (c)
41. (a) 42. (d) 43. (c) 44. (d) 45. (c) 46. (a) 47. (b) 48. (a)

https://sscstudy.com/
https://sscstudy.com/

28 CUET (UG) Section III : General Test

In the News
1. Who has been elected as the new 10. Who among the follow ing has been 19. Indian social worker Sindhutai
Director-General of the International appointed as chairman of Pfizer India? Sapkal, who has passed away recently,
Labour Organisation (ILO)? (a) KV Kamath was fondly referred as
(a) Gilbert Houngbo (b) Rajiv Mehrishi (a) Women of Peace
(b) Muriel Penicaud (c) Mukundakam Sharma (b) Iron Lady of India
(c) Kanayo Nwanze (d) Pradip Shah (c) Mother of Orphans
(d) Kang Kyung-wha (d) Lioness of Maratha
11. Who among the follow ing took
2. Ramesh Chandra Lahoti, who has charge as the Director of Vikram 20. Who has been appointed as the next
passed away recently, was the former Sarabhai Space Centre (VSSC)? Chief of the Naval Staff to succeed
.............. of India? (a) Unnikrishnan Nair Admiral Karambir Singh?
(a) Lok Sabha Speaker (b) Badrinath Srinivasan (a) Anil Kumar Chawla
(b) Chief Justice (c) Dileep Sanghani (b) R. Hari Kumar
(c) Chief Election Commissioner (d) Vikram Dev Dutt (c) Shekhar Sinha
(d) ISRO Chairman (d) Surinder Pal Singh Cheema
12. Who has been named as the next
3. Name the creator of the Graphics chief economist of the International 21. Keshav Desiraju who has passed
Interchange Format (GIF), who has Monetary Fund (IMF)? away recently had formerly served at
passed away recently. (a) Geoffrey Okamoto which post in the Union Cabinet?
(a) Alan Ladd Jr (b) Eugene Parker (b) Gita Gopinath (a) Economic Secretary
(c) Stephen Wilhite (d) Rod Marsh (c) Pierre-Olivier Gourinchas (b) Health Secretary
(d) Kristalina Georgieva (c) Finance Secretary
4. Who has been elected as the fellow of (d) Agricultural Secretary
the Royal Society of Edinburgh (RSE) in 13. Who among the follow ing has been
Scotland? appointed as the head of United Nations 22. Consider the follow ing statements :
(a) Chanda Kochhar Children’s Fund (UNICEF)? 1. Air Chief Marshal Vivek Ram
(b) Vandana Luthra (a) Audrey Azoulay (b) Catherine Russell Chaudhari assumed charge as the Chief
(c) Soumya Swaminathan (c) Henrietta H. Fore (d) Rafael Grossi of the Air Staff (CAS).
(d) Kiran Mazumdar-Shaw 2. Air Marshal Sandeep Singh has been
14. Choose the correct statement
5. Which country’s former President regarding Elon Musk. appointed as the new and 46th Vice
Shahabuddin Ahmed passed away Chief of the Air Staff (VCAS) of the
1. Elon Musk named TIME Magazine’s Indian Air Force (IAF).
recently? ‘Person of the Year’ for 2021.
(a) Pakistan (b) Saudi Arabia Of the above correct statement(s)
2. Musk is also the founder and CEO of is/are
(c) Afghanistan (d) Bangladesh
rocket company SpaceX. (a) Both 1 and 2 (b) Only 1
6. Choose the correct statement(s) Codes (c) Only 2 (d) None of these
regarding Ketanji Jackson. (a) Both 1 and 2 (b) Only 1
1. Ketanji Jackson nominated as first
(c) Only 2 (d) None of these 23. The 23-year-old Shashvat Nakrani is
the youngest self-made individual in
Black Woman to US Supreme Court. 15. Abdalla Hamdok has resigned as the Hurun India Rich List 2021. He is the
2. She would replace liberal Supreme Prime Minister of which country? co-founder of which company?
Court Justice Stephen Breyer. (a) Rwanda (b) Turkey (a) Paytm (b) PhonePe
Codes (c) Sudan (d) Somalia (c) WinZO (d) BharatPe
(a) Both 1 and 2 (b) Only 1
(c) Only 2 (d) None of these 16. Who has been appointed as the next 24. Kamla Bhasin, who has passed away
President of Federation of Indian recently, was a well known author and
7. KN Raghavan has recently been Chambers of Commerce and Industry poet, besides being
appointed as the Chairman of which (FICCI)? (a) Journalist
international body? (a) Soma Sankara Prasad (b) Women’s rights activist
(a) International Rubber Study Group (b) Ramalingam Sudhakar (c) Basketball player
(b) International Steel Group (c) Sanjiv Mehta (d) Filmmaker
(c) International Dairy Federation (d) Arun Kumar Mishra
(d) International Automotive Task Force 25. Name the mountaineer who has
17. Name the India’s first woman achieved the feat of becoming the fastest
8. Who has been appointed as India’s psychiatrist, who passed away recently. Indian to scale two peaks?
first National Maritime Security (a) G S Lakshmi (b) Sarada Menon (a) Shivangi Pathak (b) Geeta Samota
Coordinator (NMSC)? (c) Janani Narayanan (d) Vrinda Rathi (c) Chhanda Gayen (d) Premlata Agrawal
(a) Ravneet Singh (b) Karambir Singh
(c) G. Ashok Kumar (d) Sunil Lanba 18. Who has been appointed as the new 26. Who has been appointed as the
Chairman and CEO of Railway Board? Chairman of the Central Board of Direct
9. Nobel Prize w inner Luc Montagnier (a) Sanjeev Mittal Taxes (CBDT)?
passed away recently. He was a/an (b) Vinay Kumar Tripathi
(a) JB Mohapatra (b) Rajesh Damor
(a) Astrophysicist (b) Cardiologist (c) Naresh Salecha
(c) Pramod Y. Devikar (d) Deepa Rasal
(c) Paleontologist (d) Virologist (d) Ravinder Gupta

https://sscstudy.com/
https://sscstudy.com/

CURRENT AFFAIRS ONELINER 29

27. Which Indian banker has been (a) Rashi (b) Sudharma (a) Ranganath Misra
(c) Brahman (d) Purana (b) MN Venkatachalliah
appointed as the independent director of
(c) Arun Mishra
the Hongkong and Shanghai Banking 37. Name the CEO of NITI Aayog, whose (d) S. Rajendra Babu
Corporation (HSBC) Asia? tenure has been extended by government
(a) Rajnish Kumar for one year till June 2022? 47. Name the female IAF officer who has
(b) Aditya Puri become the first woman flight test
(a) Arvind Panagariya (b) Rajiv Kumar
(c) Arundhati Bhattacharya engineer of India?
(c) Amitabh Kant (d) Nripendra Misra
(d) Anshula Kant
(a) Padmavathy Bandopadhyay
28. Who has been appointed as the 38. The renowned software developer (b) Mitali Madhumita
and the creator of the McAfee antivirus (c) Aashritha V Olety
Chairperson of Stop TB Partnership
software has passed away at the age of (d) Sophia Qureshi
Board?
75, due to suicide. Name that software
(a) Harsh Vardhan 48. Who has been appointed as the new
pioneer
(b) Mansukh Mandaviya Deputy Chief of Naval Staff?
(c) Kiren Rijiju (a) Kim McAfee (b) Charles McAfee
(c) Nick McAfee (d) John McAfee (a) Ravneet Singh (b) G. Ashok Kumar
(d) Dharmendra Pradhan
(c) Satish Soni (d) Jagjit Singh Bedi
29. Who has been roped in by the Delhi 39. What was the profession of Raj
Kaushal who has passed away recently at 49. Who has been sworn in as the new
government as the brand ambassador for
the age of 49? Chief Minister of Assam to replace
its ‘Desh Ke Mentors’ programme?
(a) Politician (b) Sportsman Sarbananda Sonowal?
(a) Virat Kohli (b) Sonu Sood
(c) Ranvir Singh (d) Kapil Dev (c) Journalist (d) Filmmaker (a) Prafulla Kumar Mahanta
(b) Himanta Biswa Sarma
30. Who has been appointed as the 40. Satya Nadella has been appointed as (c) Atul Bora
Chairman of the reconstituted Advisory the Chairman of Microsoft in June, 2021. (d) Kailash Nath Sarma
Board for Banking and Financial Frauds He replaced whom?
(ABBFF) by CVC? (a) Manu Sawhney (b) Mark Lowcock 50. Name the women mountaineer who
(a) Ajay Banga (c) Gina Haspel (d) John Thompson has recently created historic record of
(b) TM Bhasin world’s fastest ascent of Everest by a
(c) Kris Gopalakrishnan 41. Tadang Minu is the first Indian woman
(d) NR Narayana Murthy female to be appointed as a member of (a) Gerlinde Kaltenbrunner
the Coaches Committee of the (b) Oh Eun-sun
31. Who has been sworn in as International Boxing Association (AIBA). (c) Moni Mulepati
Karnataka's new Chief Minister? She hails from which State? (d) Tsang Yin-hung
(a) Arun Singh (b) Thaawarchand Gehlot (a) Manipur (b) Sikkim
(c) CT Ravi (d) Basavaraj Bommai (c) Ladakh (d) Arunachal Pradesh 51. Who has been appointed as the new
Director of the CBI?
32. Which nation's President has sacked 42. Who has been appointed as the new (a) Subodh Kumar Jaiswal
the Prime Minister and suspend the Chairman of Microsoft? (b) Vishwas Nangare Patil
Parliament? (a) Elon Musk (b) Sunder Pichai (c) Amulya Patnaik
(a) Armenia (b) Turkmenistan (c) Larry Page (d) Satya Nadella (d) Anil Deshmukh
(c) Tunisia (d) Indonesia
43. Centre has extended the term of 52. Who among the follow ing has been
33. Dr. PK Warrier, who has passed away which Deputy Governors of RBI by two appointed as the new Chairperson of the
at the age of 100 recently, was a veteran years, till June 2023? National Human Rights Commission
(a) Agricultural Scientist (a) Michael Patra (NHRC) of India?
(b) Economist (b) Mahesh Kumar Jain (a) Ranganath Misra
(c) Astrologer (c) Rabi Sankar (b) MN Venkatachalliah
(d) Ayurveda practitioner (d) M. Rajeshwar Rao (c) Arun Mishra
(d) S. Rajendra Babu
34. Who has been appointed as the 44. Who has been appointed as the
Chairman of Indian Federation of United President of 76th UN General Assembly 53. Noted Indian environmentalist
Nations Associations (IFUNA)? (UNGA) for 2021-22? Sunderlal Bahuguna passed away due to
(a) Justice Sudhanshu Dhulia (a) Peter Mohan (b) Munir Akram COVID-19 on May 21, 2021. He was a
(b) Justice Rajesh Bindal (c) Omar Hilale (d) Abdulla Shahid prominent leader of which among the
(c) Justice Mohammad Rafiq follow ing movements?
(d) Justice Shambhu Nath Srivastava 45. What was the profession of TM (a) Narmada Bachao Andholan
Kalliannan Gounder, who has passed (b) Chipko Movement
35. Indian politician Virbhadra Singh, away at the age of 101? (c) Save Silent Valley Movement
who has passed away was the former (a) Actor (d) Appiko Movement
Chief Minister of which State? (b) Agricultural Scientist
(a) Himachal Pradesh (b) Gujarat (c) Freedom Fighter 54. Jagannath Bidyadhar Mohapatra has
(c) Haryana (d) Bihar (d) Sportsperson been roped in as the new Chairman of
which of these organisation?
36. KV Sampath Kumar, the editor of 46. Who among the follow ing has been (a) CBDT (b) NITI Aayog
world’s first Sanskrit Daily, has passed appointed as the new Chairperson of the (c) CBI (d) SIDBI
away. What is the name of the Sanskrit
NHRC of India?
Daily?

https://sscstudy.com/
https://sscstudy.com/

30 CUET (UG) Section III : General Test

55. Pinarayi Vijayan has been sworn in (a) C. Rangarajan (c) Ramesh Pokhriyal Nishank
(b) M. Narasimham (d) Bhagat Singh Koshyari
as the Chief Minister of Kerala for second
(c) S. Venkitaramanan
straight term. He is the .............. CM of 72. Lou Ottens, the inventor of the audio
(d) YV Reddy
the State. cassette tape has passed away. He was
(a) 12th (b) 14th (c) 11th (d) 15th 64. Justice Nuthalapati Venkata Ramana from which country?
has been sworn in as the ............. Chief (a) Ireland (b) Finland
56. Who has been appointed as the new Justice of India (CJI). (c) Netherlands (d) Sweden
CBI Chief?
(a) 40th (b) 36th
(a) Rakesh Asthana (b) YC Modi (c) 52nd (d) 48th 73. The Padma Bhushan awardee
(c) Subodh Jaiswal (d) SS Deswal Laxman Pai has passed away. He was
65. Name the Indian mountaineer, who conferred w ith the prestigious honor for
57. Arya Rajendran, has been elected as has become the first female of the country contribution in which field?
the youngest mayor in India. She is from to summit the Mt Annapurna in April 2021, (a) Agriculture (b) Mathematics
which State? the tenth highest mountain in the world. (c) Literature (d) Painting
(a) Tamil Nadu (b) Assam (a) Gurmayum Devi (b) Malavath Purna
(c) Kerala (d) Goa (c) Priyanka Mohite (d) Santosh Yadav 74. Which Indian celebrity has been
listed in the 2021 Young Global Leaders
58. Who has been sworn in as the new 66. GVG Krishnamurty has passed away. (YGLs) forum by World Economic Forum
Chief Minister of Tamil Nadu after 2021 He was the former ............. of India (a) Anushka Sharma (b) Shraddha Kapoor
Assembly election? (a) Home Minister (c) Deepika Padukone (d) Alia Bhatt
(a) MK Tamilarasu (b) MK Stalin (b) President
(c) M. Karunanidhi (d) MK Alagiri (c) Election Commissioner 75. Name the Indian economist who has
(d) Chief of Army Staff been appointed as the Assistant
59. Name the newly appointed Chief Secretary-General and Head of the New
Minister of Union Territory of Puducherry 67. Who has been appointed as the new York Office of the UNEP
(a) V. Vaithilingam (b) V. Narayanasamy Chief Election Commissioner of India?
(a) Prakash Lohia (b) Arora Akanksha
(c) N. Rangasamy (d) YS Bharati (a) Sushil Chandra (b) Rajiv Kumar (c) V. Sundramoorthy (d) Ligia Noronha
(c) Navin Chawla (d) Nagendra Singh
60. Chaudhary Ajit Singh, who has 76. Who has been appointed as the new
passed away recently was the founder of 68. Who has been appointed as the new Chairman and Managing Director of SIDBI?
which political party? Chairman of Steel Authority of India
(a) S. Ramann
(a) Ambedkar Samaj Party (ASP) Limited (SAIL)? (b) Devendra Kumar Singh
(b) Rashtriya Lok Dal (RLD) (a) Mistu Mahajbin (b) Soma Mondal (c) V. Satya Venkata Rao
(c) Bahujan Samaj Party (BSP) (c) Meeta Baghel (d) Radha Pyari (d) Pankaj Jain
(d) Rashtriya Janata Dal (RJD)
69. Who has been appointed as the new 77. Chandra Nayudu, who has passed
61. Mamata Banerjee has been sworn in Chairman & CEO of Railway Board? away, was first women commentator of
as the Chief Minister of West Bengal after (a) Sanjay Pal which sports?
massive w in in the 2021 Assembly (b) Amit Dhawan (a) Football (b) Cricket
election. This w in is her ............. (c) Suneet Sharma (c) Tennis (d) Hockey
consecutive w in as the head of the state. (d) Harish Kumar Sahu
(a) fourth (b) second 78. Name the Indian billionaire who has
(c) third (d) fifth 70. Justice Hima Kohli has been topped the Forbes list of India’s 10
appointed as the first-ever female Chief richest billionaires for the year 2021
62. Name the fourth Deputy Governor of Justice of the High Court of
(a) Gautam Adani (b) Shiv Nadar
the RBI, who has recently been appointed (a) Karnataka (b) Delhi (c) Mukesh Ambani (d) Uday Kotak
in place of BP Kanungo (c) Ladakh (d) Telangana
(a) T. Rabi Sankar (b) GK Chadha 79. Name the Deputy Governor of RBI
(c) DM Nanjundappa (d) TN Srinivasan 71. Name the new Chief Minister of who has retired from his service.
Uttarakhand, who has been appointed to
(a) BP Kanungo
63. Name the former Governor of RBI, replace the former CM Trivendra Singh (b) Mahesh Kumar Jain
considered as the “father of banking Rawat? (c) Michael Patra
reforms in India”, who has passed away (a) Tirath Singh Rawat (d) Rajeshwar Rao
due to COVID-19 related illness. (b) Dhan Singh Rawat

ANSWERS
1. (a) 2. (b) 3. (c) 4. (d) 5. (d) 6. (a) 7. (a) 8. (c) 9. (d) 10. (d)
11. (a) 12. (c) 13. (b) 14. (a) 15. (c) 16. (c) 17. (b) 18. (b) 19. (c) 20. (b)
21. (b) 22. (a) 23. (d) 24. (b) 25. (b) 26. (a) 27. (a) 28. (b) 29. (b) 30. (b)
31. (d) 32. (c) 33. (d) 34. (d) 35. (a) 36. (b) 37. (c) 38. (d) 39. (d) 40. (d)
41. (d) 42. (d) 43. (b) 44. (d) 45. (c) 46. (c) 47. (c) 48. (a) 49. (b) 50. (d)
51. (a) 52. (c) 53. (b) 54. (a) 55. (a) 56. (c) 57. (c) 58. (b) 59. (b) 60. (b)
61. (c) 62. (a) 63. (b) 64. (d) 65. (c) 66. (c) 67. (a) 68. (b) 69. (c) 70. (d)
71. (a) 72. (d) 73. (d) 74. (c) 75. (d) 76. (a) 77. (b) 78. (c) 79. (a)

https://sscstudy.com/
https://sscstudy.com/

CURRENT AFFAIRS ONELINER 31

Miscellaneous
1. What is the theme of World Theatre 10. South Maubuang in which state, has (a) Puneet Dalmia
Day in the year 2022? been declared as the first model ODF (b) Rakeysh Omprakash Mehra
(a) Theatre and a Culture of Peace Plus village of the state? (c) Suresh Raina
(b) Tales of Theatre (a) Meghalaya (b) Mizoram (d) Narotam Sekhsaria
(c) Theatre of Nations (c) Tripura (d) Assam
(d) Theatre and Drama in Education 20. What was the theme for World AIDS
11. Which of the follow ing state has got Day 2021?
2. Who has authored a children’s book first-of-its-kind state-level bird atlas in (a) Communities make the difference
‘The Little Book of Joy’? India? (b) End inequalities, End AIDS and End
(a) Desmond Tutu (b) Tenzin Gyatso (a) Karnataka (b) Telangana Pandemics
(c) Penpa Tsering (d) Both (a) and (b) (c) Tamil Nadu (d) Kerala (c) On the fast track to end AIDS
(d) Global Solidarity and Shared
3. United Nations General Assembly has 12. Which day has been declared by Responsibility
proclaimed which day as the International Prime Minister Narendra Modi to be
Day to Combat Islamophobia? observed as ‘National Start-up Day’ in 21. International Literacy Day is
(a) March 14 (b) March 16 India? observed every year on
(c) March 17 (d) March 15 (a) January 5 (b) January 16 (a) September 05 (b) September 06
(c) January 18 (d) January 22 (c) September 07 (d) September 08
4. The Dredging Museum named
‘Nikarshan Sadan’ has been inaugurated 13. Which Ministry has come out w ith 22. What is the theme of the World
in, which city? the pictorial comic book titled ‘India’s Suicide Prevention Day (WSPD) 2021?
(a) Chennai (b) Vishakhapatnam Women Unsung Heroes’? (a) Take a minute, change a life
(c) Kochi (d) Panaji (a) Ministry of Culture (b) Creating hope through action
(b) Ministry of Women & Child Development (c) 40 seconds of action
5. India’s first Biosafety level-3
containment mobile laboratory has been (c) Ministry of Defence (d) Working Together to Prevent Suicide
inaugurated in (d) Ministry of Sports
23. Nuakhai is a harvest festival,
(a) Jaipur (Rajasthan) 14. On which day World Leprosy Day is celebrated by the native of which Indian
(b) Nashik (Maharashtra) observed every year?
(c) Chennai (Tamil Nadu) state?
(a) Last Thursday of January (a) Karnataka (b) Tamil Nadu
(d) Gurugram (Haryana)
(b) Last Friday of January (c) Andhra Pardseh (d) Odisha
6. What is the theme of National Science (c) Last Saturday of January
Day 2022? (d) Last Sunday of January 24. Who is the author of the book and
(a) The Future of STI ‘My Life in Full: Work, Family, and Our
15. Who among the follow ing is the Future’?
(b) Future of STI : Impact on Education author of the book titled ‘Operation (a) Nita Ambani
Skills and Work Khatma’? (b) Melinda French Gates
(c) Women in Science (a) RC Ganjoo (b) Ashwini Bhatnagar (c) Indra Nooyi
(d) Integrated Approach in S&T for (c) Kiran Bedi (d) Both (a) and (b) (d) Satya Nadella
Sustainable Future
16. Oxford University Press has declared 25. Who is the author of the book
7. The book titled ‘A Nation To Protect’ ………… as Children’s Word of the Year ‘Accelerating India : 7 Years of Modi
has been authored by 2021, based on their recent research. Government’?
(a) Narayan Rane (a) Coronavirus (b) Isolate (a) KK Shailaja (b) Pinarayi Vijayan
(b) Priyam Gandhi Mody (c) Anxiety (d) Pandemic (c) MK Stalin (d) KJ Alphons
(c) Mansukh Mandaviya
(d) Pranab Mukherjee 17. PM Modi unveiled 216-foot statue of 26. Which nation will host SCO Defence
saint Ramanujacharya in which of the Ministers’ meet?
8. Match the follow ing columns : follow ing city? (a) Afghanistan (b) Tajikistan
A. Maru Mahotsav 1. Telangana (a) Varanasi (b) Chennai (c) Uzbekistan (d) Turkmenistan
(c) Hyderabad (d) Bengaluru
B. Khajuraho Festival 2. Jammu and
Kashmir 27. Ramappa Temple has become the
18. Which Railway Station has been 39th Indian site to be inscribed in
C. Kanchoth festival 3. Madhya Pradesh named as the ‘Veerangana Laxmibai UNESCO’s World Heritage List. It is
D. Medaram Jatara 4. Rajasthan Railway Station’? located in which state?
Festival (a) Habibganj Railway Station (a) Telangana (b) Karnataka
(a) a-3, b-4, c-2, d-1 (b) a-4, b-2, c-3, d-1 (b) Bina Junction Railway Station (c) Tamil Nadu (d) Kerala
(c) a-4, b-3, c-2, d-1 (d) a-4, b-3, c-1, d-2 (c) Kanpur Central Railway Station
(d) Jhansi Railway Station 28. Who is the author of the book ‘The
9. The government of India has Ramayan of Shri Guru Gobind Singh Ji’?
organised the annual Polio National 19. Who has authored his autobiography (a) Baljit Kaur Tulsi
Immunisation Day 2022 on which day? titled ‘The Ambuja Story: How a Group of (b) Rohit Baban Deo
(a) February 26 (b) February 28 Ordinary Men Created an Extraordinary (c) Sanjay Kishan Kaur
(c) February 27 (d) February 25 Company’? (d) Pushpa Virendra Ganediwala

https://sscstudy.com/
https://sscstudy.com/

32 CUET (UG) Section III : General Test

29. Choose the correct statements (a) Sikkim (b) Assam 44. The ‘Maharaja Chhatrasal
(c) Tamil Nadu (d) Punjab
1. Armenia’s acting Prime Minister Nikol Convention Centre’ has been inaugurated
Pashinyan has won the country’s 37. On the occasion of World in which of these UNESCO World
parliamentary elections 2021, with 54% Environment Day, PM Modi has launched Heritage locations?
of the vote. (a) Sanchi (b) Khajuraho
three E 100 ethanol dispensing stations
2. Pashinyan, who is the leader of the Civil under a pilot project in which city? (c) Konark (d) Hampi
Contract party, first came to power in
(a) Hyderabad (b) Ahmedabad 45. Who is the author of the book
2018, following the country’s first free
(c) Pune (d) New Delhi
and fair election. ‘COVID-19: Sabhyata ka Sankat aur
Code 38. Who is the author of the book Samadhan’?
(a) Only 1 (b) Only 2 ‘‘LANGUAGES OF TRUTH: Essays (a) Kailash Satyarthi (b) Narendra Modi
(c) Both 1 and 2 (d) None of these (c) Tathagata Roy (d) Amish Tripathi
2003-2020’’?
30. In India, which day is observed as (a) VS Naipaul
46. The National Safe Motherhood Day
the National Fish Farmers’ Day? (b) Salman Rushdie
marks the birth anniversary of which
(a) July 7 (b) July 8 (c) July 9 (d) July 10 (c) Arundhati Roy
(d) Kiran Desai Indian political activist?
(a) Kasturba Gandhi
31. Who is the author of the book ‘The (b) Kamala Nehru
Nutmeg’s Curse: Parables for a Planet in
39. Who is the author of the book
‘Stargazing: The Players in My Life’? (c) Indira Gandhi
Crisis’? (d) Vijaya Lakshmi Pandit
(a) Sourav Ganguly
(a) Arundhati Roy (b) Salman Rushdie
(b) Kapil Dev
(c) Chetan Bhagat (d) Amitav Ghosh
(c) Ravi Shastri
47. PM Narendra Modi released the
(d) Mohammad Azharuddin Hindi translation of the book ‘Odisha
32. What is the theme of the 2021 Itihaas’. Who is the author of this book?
International Yoga Day? (a) Biren Mitra
40. Who is the author of the book
(a) Yoga for Peace (b) Harekrushna Mahtab
‘Savarkar: A contested Legacy
(b) Yoga for well-being (c) Nilamani Routray
(c) Yoga at Home and Yoga with Family
(1924-1966)’?
(a) Ashok Desai (b) Hindol Sengupta (d) Biswanath Das
(d) Yoga for Climate Action
(c) Bibek Debroy (d) Vikram Sampath 48. Which country is the sponsor for
33. ‘Believe- What Life Taught me’, is the International Children’s Book Day 2021?
autobiography of which Indian player? 41. What was the theme of the 2021
International Nurses Day? (a) United States (b) New Zealand
(a) Gautam Gambhir (b) Shikhar Dhawan (c) Australia (d) Denmark
(c) Suresh Raina (d) Yuvraj Singh (a) Nurses: A Voice to Lead – Health for All
(b) Nurses: A Voice to lead – Health is a 49. India’s biggest floating solar power
34. What is the theme of World Day human right plant is being set-up at which place?
Against Child Labour 2021? (c) Nurses: A Voice to Lead – Nursing the (a) Talcher (b) Vijayawada
(a) Generation Safe & Healthy World to Health (c) Ramagundam (d) Vindhyachal
(b) Protect children from child labour, now (d) Nurses: A Voice to Lead – A vision for
more than ever future healthcare 50. The World Health Day is observed
(c) Children shouldn’t work in fields, but on every year on which day?
dreams 42. The book titled “Elephant In The (a) April 5 (b) April 6
(d) Act now: End child labour! Womb” is a debut book by which Indian (c) April 7 (d) April 4
celebrity?
35. Name the first village in India, where (a) Shilpa Shetty (b) Kalki Koechlin 51. National Maritime Day is celebrated
the entire population above 18 years has (c) Twinkle Khanna (d) Anushka Sharma every year in India on which day?
been vaccinated. (a) April 4 (b) April 5
(a) Malana (b) Nako 43. India’s biggest floating solar power (c) April 6 (d) April 7
(c) Diskit (d) Weyan plant is being set up at which place?
(a) Talcher 52. The Tulip Festival is organised in
36. Raimona Reserve Forest has recently (b) Vijayawada which State/UT of India every year?
been declared as a National Park. Where (c) Ramagundam (a) Jammu & Kashmir (b) Kerala
is it located? (d) Vindhyachal (c) Himachal Pradesh (d) Nagaland

ANSWERS
1. (a) 2. (d) 3. (d) 4. (b) 5. (b) 6. (d) 7. (b) 8. (c) 9. (c) 10. (b)
11. (d) 12. (b) 13. (a) 14. (d) 15. (d) 16. (c) 17. (c) 18. (d) 19. (d) 20. (b)
21. (d) 22. (b) 23. (d) 24. (c) 25. (d) 26. (b) 27. (a) 28. (a) 29. (c) 30. (d)
31. (d) 32. (b) 33. (c) 34. (d) 35. (d) 36. (b) 37. (c) 38. (b) 39. (c) 40. (d)
41. (d) 42. (b) 43. (c) 44. (b) 45. (a) 46. (a) 47. (b) 48. (a) 49. (c) 50. (c)
51. (b) 52. (a)

https://sscstudy.com/
https://sscstudy.com/

GENERAL KNOWLEDGE 3

CHAPTER 01

General Knowledge
Important Days First in the World
Day Date First Radio Telescope Satellite launched Japan
National Youth Day January 12 into Space

National Tourism Day January 25 First country to use glass Egypt and Mesopotamia

Martyrs’ Day January 30 First country to make map Sumeria (3800 BC) (1500
BC)
World Leprosy Eradication Day January 30
First Space Ship Landed on Mars Viking-I (July 1976)
National Science Day February 28
World’s first Multipurpose River Valley Tennessee River Valley
International Women’s Day March 8 Project Project (USA)
World Disabled Day March 15 First Space Shuttle Launched Columbia
World Consumer Rights Day March 15 First rocket to go near the Sun Helius ‘B’
World Forestry Day March 21 First country to make Constitution America
World Day for Water March 22 First country to start underground metro rail Britain
World TB Day March 24 First unmanned mission on Moon LUNA-9
World Health Day April 7 First van to carry man on Moon Apollo - 11
World Heritage Day April 18 First country to do Artificial Satellite Russia
Earth Day April 22 Experiment
Worker’s Day (International Labour Day) May 1 Country to give voting right to women New Zealand
World Red Cross Day May 8 First country to appoint lokpal Sweden
National Technology Day May 11 First country to imposed carbon tax New Zealand
Anti-tobacco Day May 31 First clonned animal Dolly (a sheep)
World Environment Day June 5 World’s first Atomic Energy Plant EBR (USA)
World Yoga Day June 21 First in World (Male)
World Population Day July 11
First Asian to head the International Jagmohan Dalmiya
World Senior Citizen’s Day August 8 Cricket Council
Teacher’s Day September 5 First man to climb Mount Everest Sherpa Tenzing Norgay
International Literacy Day (UNESCO) September 8 and Sir Edmund Hillary
World Ozone Day September 16 (29th May, 1953)
First man to go into space Major Yuri Gagarin (USSR)
World Tourism Day September 27
First man to work in space Alexei Leonel (Rerssiaj)
Gandhi Jayanti; International Day of Non-violence October 2
First person to give information about Nicolous Copernicus
World Food Day October 16
planets and their motion around the Sun
UN Day October 24
First man to compile Encyclopaedia Aspheosis (Athens)
Children’s Day; Diabetes Day November 14
First person to go on both the poles Albert P Carey
Citizen’s Day November 19 (North and South)
World AIDS Day December 1 First man to reach North Pole Robert Peary
Human Rights Day December 10 First man to reach South Pole Roald Amundsen
Good Governance Day December 25 First man to climb on Mt Everset without Phu Dorji Sherpa
(Birthday of Atal Bihari Vajpayee) oxygen

https://sscstudy.com/
https://sscstudy.com/

4 CUET (UG) Section III : General Test

First in World (Female) Country Capital Currency


First woman President of a country Maria Estela Peron (Argentina) Myanmar Nai-pe-da Kyat
First woman in the world to cross the Arti Pradhan (India) Nepal Kathmandu Nepalese Rupee
Strait of Gibralter Portugal Lisbon Euro
First woman Cosmonaut in space Valentina Tereshkova (USSR) Qatar Doha Riyal
First female to find asteroid Caroline and Derschell Russia Moscow Rouble
First non-white female to win Nobel Toni Morrison Saudi Arabia Riyadh Rial
Prize in Literature South Africa Pretoria Rand
First woman to reach Antarctica Caroline Michaelson Spain Madrid Euro
First woman to have a space-walk Svetlana Yevgenyevna Savitskaya Sri Lanka Sri Jayawardene Sri Lankan Rupee
First Youngest girl to reached Purna Malavath Pura Kotte
Mount Everest Sweden Stockholm Krona
Countries, their Capitals and Currencies Switzerland Berne Swiss Franc
Country Capital Currency Syria Damascus Syrian Pound
Afghanistan Kabul Afghani Tajikistan Dushanbe Somoni
Argentina Buenos Aires Peso Thailand Bangkok Baht
Australia Canaberra Australian Dollar Tanzania Dodoma Shilling
Bangladesh Dhaka Taka Turkey Ankara Lira
Belgium Brussels Euro Turkmenistan Askhabad Manat
Bhutan Thimpu Ngultrum United Arab Emirates Abu Dhabi Dirham
(UAE)
Brazil Brasilia Real
United Kingdom (UK) London Pound Sterling
Brunei Bandar Seri Begawan Brunei Dollar
United States of Washington D C Dollar
Cambodia Phnom Penh Riel
America (USA)
Canada Ottawa Canadian Dollar
Uzbekistan Tashkent Som
Chile Santiago Peso
Vietnam Hanoi Dong
China Beijing Renmminbi Yuan
Yemen Sana’s Riyal
Cuba Havana Cuban Peso
East Timor Dili US Dollar Countries and their Parliaments
Egypt Cairo Egyptian Pound
Name of the Name of the Name of the Name of the
Iceland Reykjavik Krona Country Parliament Country Parliament
India New Delhi Rupee Afghanistan Shora Australia Parliament
Indonesia Jakarta Rupiah Bangladesh Jatiyo Sansad Bhutan Tshongdu
Iran Tehran Toman
Brazil National Congress Canada Parliament
Iraq Baghdad Iraqi Dinar
China National People’s Denmark Folketing
Ireland Dublin Euro Congress
Israel Jerusalem Shekel Egypt People’s Assembly France National Assembly
Japan Tokyo Yen
Germany Bundestag India Sansad
Kenya Nairobi Shilling
Iran Majlis Iraq National Assembly
North Korea Pyongyang Won
Israel Knesset Japan Diet
South Korea Seoul Won
Maldives Majlis Mongolia Khural
Kuwait Kuwait city Kuwaiti Dinar
Kyrgyzstan Bishkek Som Nepal Rashtriya Panchayat Pakistan National Assembly

Laos Vientiane Kip Poland Sejm Russia Duma


Malaysia Kuala Lumpur Ringgit Spain Cortes Generales Sweden Riksdag
Maldives Male Rufiyaa Saudi Arabia Majlis Al Shura Sudan Majlis Watani
Mauritius Port Louis Mauritius Rupee Taiwan Yuan Turkey Grand National
Mexico Mexico City Mexican Peso Assembly
Morocco Rabat Dirham USA Congress UK Parliament

https://sscstudy.com/
https://sscstudy.com/

GENERAL KNOWLEDGE 5

National Emblems of Major Countries Discovery Discoverer Discovery Discoverer


Country Emblem Country Emblem Solar System Copernicus Circumnavigation Magellan
Australia Kangaroo Italy White Lily of World

Bangladesh Water Lily Japan Chrysanthemum Planets Kepler Mount Everest Edmund Hillary

Belgium Lion Netherlands Lion South Pole Amundsen First person to set Neil Armstrong
foot on the moon
Canada White Lily New Zealand Southern Cross,
Kiwi, Fern North Pole Robert Peary Tasmania Island Tasman

Chile Candor and Norway Lion China Marco Polo Cape of the Good Baurtho
Huemul Hope Romelodeis

France Lily Pakistan Crescent


National Monuments of Some Famous Countries
Germany Corn Flower Spain Eagle
Monument Country Monument Country
India Lioned Capital United Kingdom Rose
Emperial Palace (Tokyo) Japan Leaning Tower of Pisa Italy
Iran Rose USA Golden Rod
Eiffel Tower (Paris) France Pyramid (Giza) Egypt
Sobriquets (Places and Persons) Places Great Wall of China China Opera House (Sydney) Australia
Sobriquets Primary Name Kremlin (Moscow) Russia Statue of Liberty (New York) USA
Bengal’s Sorrow River Damodar Kinder Disk Denmark Taj Mahal (Agra) India
City of the Golden Gate San Francisco (USA)
Persons
City of the Golden Temple Amritsar
City of Seven Hills/Eternal City Rome Sobriquet Name of Person

City of Skyscrapers New York (USA) Adi Kavi Valmeeki


Garden City Bangaluru Anna C N Annadurai
Gateway of India Bombay Frontier Gandhi/Badshah Khan Abdul Ghaffar Khan
Gift of the Nile Egypt Deenbandhu C F Andrews
Holy Land Palestine Deshbandhu C R Das
Island of Cloves Zanzibar Fuhrer Adolf Hitler
Island of Pearls Bahrain Loknayak Jaiprakash Narayan
Land of the Kangaroo Australia Kaviguru Rabindranath Tagore
Land of the Golden Pagoda Myanmar (Burma) Lady with the Lamp Florence Nightingale
Land of the Midnight Sun Norway
Lokamanya Bal Gangadhar Tilak
Land of the Rising Sun Japan
Mahamana Madan Mohan Malaviya
Land of Thousand Lakes Finland
Mahatma Gandhiji
Land of White Elephants Thailand
Man of Blood and Iron Bismark
Pink City Jaipur
Netaji Subhash Chandra Bose
Queen of the Adriatic Venice (Italy)
Panditji Jawaharlal Nehru
Queen of the Arabian Sea Kochi
Qaid-e-Azam Mohmmed Ali Jinnah
Roof of the World Pamer
Rajaji C Rajgopalachari
Sorrow of China Hwang Ho
Saint of the Gutters Mother Teresa
Sugar Bowl of the World Cuba
Venice of the East Alappuzha Crematoriums of Famous Persons
Venice of the North Stockholm (Sweden)
Crematorium Famous Person Crematorium Famous Person
White City Belgrade
Raj Ghat Mahatma Gandhi Shanti Van Jawaharlal Nehru
World’s Bread Basket Prairies of North America
Vijay Ghat Lal Bahadur Shastri Shakti Sthal Indira Gandhi
Geographical Discoveries Kisan Ghat Ch Charan Singh Abhay Ghat Morarji Desai
Discovery Discoverer Discovery Discoverer Veer Bhumi Rajiv Gandhi Samata Sthal Jagjivan Ram
America Christopher New Foundland Gobot Sebastian Ekta Sthal Giani Zail Singh, Karma Bhumi Dr. Shankar
Columbus Chandra Shekhar Dayal Sharma
Sea Route to Vasco Da Hudson Bay Henry Hudson Uday Bhoomi K R Narayana Mahaprayan Dr. Rajendra
India via Cape Gama Ghat Prasad
of Good Hope

https://sscstudy.com/
https://sscstudy.com/

6 CUET (UG) Section III : General Test

Largest, Longest and Highest in the World The largest Fresh Water Lake Kolleru Lake (Andhra Pradesh)
Speciality Entity The biggest River Islands Majuli, Brahmaputra river (Asom)
Largest Archaepelago Indonesia The largest Lake Wular lake (Jammu and Kashmir)
Largest Bird Ostrich The highest Dam Tehari Dam (Uttarkhand)
Smallest Bird Humming bird The highest Waterfall Kunchikal Falls (Karnataka)
Longest Canal Suez Canal (455 m, 1493 ft)
Highest Capital of the World Lapaz (Bolivia) The deepest River Valley Bhagirathi and Alaknanda
Largest City (Area) Mount Isa (Australia) The longest River Bridge Bhupen Hazarika Setu (Assam)
Largest City (Population) Tokyo (Japan) The biggest Conti Lover Bridge Rabindra Setu or Howrah Bridge
Largest Continent Asia (Kolkata)
Smallest Continent Australia The state with longest Coastline Gujarat (1600 km)
Largest Country (Area) Russia The longest river without Delta Narmada and Tapti
Smallest Country (Area) Vetican City The longest Sea Bridge Anna Indira Gandhi Bridge (Tamil Nadu)
Largest Country (Population) China or Pamban Bridge (2.3 km)
Largest Animal Blue Whale The largest Artificial Lake Govind Sagar Bhakhra Nangal (HP)
Largest Desert Sahara (Africa) The longest River of Godavari (1465 km)
Largest Delta Sunderban Southern India
(Ganga-Brahmaputra Delta) The longest Railway Platform Gorakhpur, 1.3 km (Uttar Pradesh)
Largest Dam Grand Coolie (USA) The longest Road Grand Trunk Road (Kolkata to Delhi)
Highest Dam Zingping-I (China) The longest Corridor Corridor of Ramnathswami Temple at
Rameshwaram ( Tamil Nadu)
Largest irrigation project Lloyd Barrage (Pakistan)
Largest Island Greenland The highest Road Road at Umling La
(in East Ladakh Sector)
Largest Sea Mediterranean Sea
The highest Airport Leh Airport (Ladakh)
Deepest Lake Baikal
The largest Desert Thar (Rajasthan)
Highest Lake Titikaka
The largest Delta Sunderbans (Paschim Banga)
Largest Lake Caspian Sea
The state with maximum Madhya Pradesh (11.24%)
Largest Freshwater Lake Superior Lake Forest Area
(USA & Canada)
The largest Zoo Arignar Anna Zoological Garden
Largest Saline Lake Caspian Sea
(Tamil Nadu)
Highest Mountain Peak Mount Everest (8850 m)
The biggest Stadium Sadar Patel Stadium (Ahemdabad)
Largest Mountain Range Andes (South America)
The longest National Highway NH-44 ( Srinagar to Kanyakumari)
Largest Museum Louvre Museum (Paris)
The highest Award Bharat Ratna
Largest Ocean Pacific Ocean
Largest Park Green Land National Park (Denmark) The highest Gallantry Award Param Vir Chakra

Coldest Place Verkhoyansk (Siberia) The largest Gurudwara Golden Temple, Amritsar
Hottest Place Death Velly, (USA) The largest Cave Temple Kailash Temple (Ellora, Maharashtra)
Longest Platform Gorakhpur (UP, India) The highest Peak Godwin Austin I, K 2 (8611 m)
Longest Railwayline Trans-Siberian Railway The largest Mosque Jama Masjid (Delhi)
Largest River Amazon (South America) The longest Tunnel Chennani-Nashri Tunnel
Longest River Nile (Africa) (Jammu and Kashmir)
The largest Auditorium Sri Shanmukhanand Hall (Mumbai)
Superlatives (India) The largest Animal Fair Sonepur (Bihar)
(Biggest, Highest, Largest, Longest, Smallest etc.)
The largest Cave Amarnath (Jammu and Kashmir)
The longest River The Ganga (2525 km)
The highest Gate Way Buland Darwaza, Fatehpur Sikri
The longest Canal Indira Gandhi Canal or Rajasthan (Uttar Pradesh)
Canal (Rajasthan) (649 km)
The tallest Statue Statue of Unity (Gujarat)
The longest Dam Hirakud Dam (Odisha) (26 km)
The largest Public Sector Bank State Bank of India
The longest Sea Beach Marina Beach (Chennai) (13 km)
The most Populous City Mumbai (Maharashtra)
The highest Lake Devtal Lake (17745 ft), Garhwal
(Uttarakhand) The biggest Church Saint Cathedral at Old Goa (Goa)
The largest Saline Water Lake Chilka Lake (Odisha) The highest Battlefield Siachen Glacier (5753 m)

https://sscstudy.com/
https://sscstudy.com/

GENERAL KNOWLEDGE 7

First in India First Cricketer to get Padma Bhushan CK Naidu


Newspaper Bengal Gazette (James Hickey) First Indian to get through ICS Satyendra Nath
Tagore (1869)
Vernacular Daily Samachar Darpan
First Indian to swim across the Mihir Sen (1958)
Hindi Newspaper Udant Martand
English Channel
Telegraph Line Diamond Harbour to Kolkata
First Indian Cricketer to get Bharat Ratna Sachin Tendulkar
International Telephone Service Mumbai to London (1851)
Silent Movie Raja Harish Chandra First in India (Female)
(Dadasaheb Phalke 1913) First Indian Female Chairperson of Sarojini Naidu (1925)
Talkie Movie Alam Ara (Ardeshir Irani-1931) Indian National Congress
Aircraft Carriage Warship INS Vikrant First Woman Cabinet Minister Rajkumari Amrit Kaur (1947)
Satellite Aryabhatta (19th April, 1975) First Woman Chairman of the UN Vijaya Laxmi Pandit
Satellite dedicated exclusively for EDUSAT General Assembly
Education purposes First Woman President of India Pratibha Devi Singh Patil
Successful indigenous launch vehicle SLV-3 First Woman Speaker of Lok Sabha Meira Kumar (2009)
Nuclear Reactor Apsara First Woman Deputy Chairman of Margaret Alva (1962)
Lunar Mission Chandrayaan-I (October, 2008) Rajya Sabha
Hydroelectric Project Sivasamudram (1902) First Woman Prime Minister of India Indira Gandhi
Asian Games Delhi (1951) First Woman to reach Antarctica Meher Moos (1976)
Census 1872 First Woman IAS Officer Anna Rajam George (1950)
Regular DEcadal Census 1881 Onwards First Female Chief Justice Leela Seth (1991)
Biosphere Reserve Nilgire First Woman to win the Jnanpith Award Asha Poorna Devi (1976)
National Haley National Park First Woman to get the Bharat Ratna Indira Gandhi
(Jim Corbett), 1936 First Female Nobel Prize Winner Mother Teresa (1979)
Chairman of UPSC Ross Barker First woman to complete century in Thirush Kamini
First E-court Ahmedabad World Cup Cricket
Court exclusively dedicated to women Malda (WB) First actress to be nominated to the Nargis Dutt
Technology Park Technopark, Rajya Sabha
Thiruvananthapuram First Woman Chairperson of UPSC Rose Millian Bathew (Kharbuli)
First Foriegn Minister Sushma Swaraj
First in India (Male)
First Olympic Winner P.T. Usha
First Governor-General of India William Bentinck (1828)
First Female Pilot of Indian Navy Shivangi Singh
Last Governor-General of India Lord Mountbatten
First Female Excavation Engineer Shivani Meena
First and last Indian Governor-General of C Rajgopalachari
Free India Important Books and Authors
First Commander-in-Chief of Free India General KM Kariappa
Book Author
First Field Marshal of India General SHFJ
Panchatantra Vishnu Sharma Buddhacharita Ashvaghosha
Manekshaw (1971)
Kadambari Banabhatta Shahnama Firdausi
First Indian to go in Space Rakesh Sharma
Sursagar Surdas Mudrarakshasha Vishakhadatta
First Indian to climb the Mount Everest without Sherpa Ang Dorje
Oxygen Rajtarangini Kalhana Arthashashtra Chanakya
(Kautilya)
First Indian to become the Managing Director Gautam Kaji
Uttarramcharita Bhavbhuti Padmavat Mallik Mohammad
of World Bank
Jayasi
First Chairman of National Human Rights Rangnath Mishra
Geetanjali, Gora Ravindranath Bharat-Bharti Maithilisharan
Commission
Tagore Gupta
First Indian to get Nobel Prize in Physics CV Raman (1930)
Anamika, Parimal Suryakant Chidambara Sumitranandan
First Indian to get Nobel Prize in Literature Rabindranath
Tripathi Nirala Pant
Tagore (1913)
First Indian to get Nobel Prize in Economics Kurukshetra, Ramdhari Malgudi Days, R K Narayan
Dr Amartya Sen
Urvashi Singh ‘Dinkar’ Guide
First Indian to get Nobel Prize in Medicines Dr Har Govind Khorana
(Physiology) Chandrakanta Devki Nandan Devadasa, Sharatchandra
Khatri Charitrahin Chattopadhyay
First Indian to get Bharat Ratna Dr S Radhakrishnan,
C Rajgopalachari and Mother Maxim Gorki Mein Kemph Adolf Hitler
Dr CV Raman War and Peace Leo Tolstoy The Insider P V Narsimha
First Person to be Honoured with the G Sankara Kurup Rao
Jnanpith Award (Malayalam) Ignited Minds A P J Abdul Long Walk to Nelson Mandela
First Person to get Bharat Ratna Lal Bahadur Shastri Kalam Freedom
(Posthumously) Half a life V S Naipaul Satanik Verses Salman Rushdie

https://sscstudy.com/
https://sscstudy.com/

8 CUET (UG) Section III : General Test

United Nations Organisation (UNO)


Ä The United Nations (UN) is a world organisation formed in 24th October, 1945. It came into existence after World
War II, when the leaders of the world, including American President Roosevelt and British Prime Minister
Churchill, decided to create a world organisation that would help to ensure peace.
Ä The original membership of 51 nations has grown to 193 members. The 193rd member being the newly created South
Sudan. The United Nations Headquarters is in New York City. The UN also has offices in Nairobi (Kenya), Geneva
(Switzerland) and Vienna (Austria).
Ä Six official languages are spoken and used in documents at the United Nations: Arabic, Chinese, English, French,
Russian and Spanish, but the working languages are English and French only.
Ä The General Assembly is the main place for discussions and policy making in the United Nations.
Ä The Security Council has primary responsibility for the maintenance of international peace and security. The
Security Council is made up of 15 members.
Ä There are five permanent members of the Secutiry Council-China, France, Russia, United Kingdom and USA and
10 non-permanent members elected for 2 years terms starting on 1st January.
UN and its Specialized Agencies
Organization Year of Establishment Headquarters
United Nations (UN) Oct 24, 1945 New York
International Atomic Energy Agency (IAEA) July 29, 1957 Vienna (Austria)
Food and Agriculture Organization (FAO) Oct 16, 1945 Rome (Italy)
United Nations Educational, Scientific and Cultural Nov 4, 1946 Paris (France)
Organization (UNESCO)
World Health Organization (WHO) July 22, 1946 Geneva (Switzerland)
International Labour Organization (ILO) April 11, 1919 Geneva (Switzerland)
International Finance Corporation (IFC) July, 1956 Washington D C
International Civil Aviation Organization (ICAO) Dec, 1944 Montreal (Canada)
Universal Postal Union (UPU) July 1, 1875 Berne (Switzerland)
World Meteorological Organization ( WMO) 1951 Geneva (Switzerland)
International Maritime Organization (IMO) March, 1948 London (Britain)
World Intellectual Property Organization ( WIPO) 1967 Geneva (Switzerland)

Some Other International Organizations


Year of
Organization Headquarters No. of Member States
Establishment
Arab League 1945 Cairo (Egypt) 22
African Union 1963 Addis Ababa (Ethiopia) 53
Asia-Pacific Economic Cooperation (APEC) 1989 Singapore 21
Asian Development Bank (ADB) 1966 Manila (Philippines) 59
Association of South-East Asian Nations (ASEAN) 1967 Jakarta (Indonesia) 10
Commonwealth 1931 London (Britain) 54
European Union (EU) 1991 Brussels (Belgium) 27
Commonwealth of Independent States (CIS) 1991 Kirava (Belarus) 12
G-8 (Group of Eight) 1975 — 8
INTERPOL 1923 Lyon (France) 184
North Atlantic Treaty Organization (NATO) 1949 Brussels (Belgium) 30
Organization of Petroleum Exporting Countries (OPEC) 1960 Vienna (Austria) 13
Non-Aligned Movement (NAM) 1961 (First Conference at Belgrade)
Secretariat at Kathmandu (Nepal) 120
South Asian Association for Regional Cooperation (SAARC) 1985 Kathmandu (Nepal) 8

https://sscstudy.com/
https://sscstudy.com/

GENERAL KNOWLEDGE 9

Indian Defence Ranks of Commissioned Officers


Ä The President of India is the supreme commander of Army Air Force Navy
the Indian Defence System. General Air Chief Marshal Admiral
Ä The whole administrative control of the Armed forces Lt. General Air Marshal Vice Admiral
lies in the Ministry of Defence. Major General Air Vice Marshal Rear Admiral
Ä Indian Defence System has been divided into three Brigadier Air Commodore Commodore
services-Army, Navy and Air Force. Colonel Group Captain Captain
Lt. Colonel Wing Commander Commander
Indian Army Major Squadron Leader Lt Commander
Ä The Indian Army is organized into seven commands Captain Flt Lieutenant Lieutenant
Lieutenant Flying Officer Sub-Lieutenant
Command Headquarters
Western Command Chandigarh Indian Defence Training Institutions
Eastern Command Kolkata Training Institution Place Estd. in
Northern Command 56 APO Rashtriya Indian Military College (RIMC) Dehradun 1922
Southern Command Pune Army Cadet College (ACC) Dehradun 1929
Indian Military Academy (IMA) Dehradun 1932
Central Command Lucknow
National Defence Academy (NDA) Khadakwasla, Pune 1941
Army Training Command Shimla
High Altitude Warfare School (HAWS) Gulmarg 1948
South Western Command Jaipur
National Defence College (NDC) New Delhi 1960
Officers Training Academy (OTA) Chennai 1963
Indian Air Force
Counter Insurgency and Jungle Warfare Vairengte 1970
Ä It is organized into seven commands School (Mizoram)
Command Headquarters College of Defence Management Secunderabad 1970
Western Command New Delhi (Andhra Pradesh)

Central Command Allahabad College of Combat/Army War College Mhow (Madhya 1971
Pradesh)
Eastern Command Shillong
Army School of Physical Training (ASPT) Pune 1978
South Western Command Gandhi Nagar
Army Air Defence College (AADC) Gopalpur (Odisha) 1989
Training Command Bangaluru
Maintenance Command Nagpur Indian Para-Military Forces
Southern Command Thiruvananthapuram Forces Estd. in
Indo-Tibetan Border Police 1962
Indian Navy National Security Guards 1984
Central Industrial Security Force 1969
Ä It is organized into three commands
Assam Rifles 1835
Command Headquarters Border Security Force 1965
Eastern Command Vishakhapatnam Central Reserve Police Force 1939
Southern Command Cochin National Cadet Corps 1948
Western Command Mumbai Intelligence Bureau 1920
Central Bureau of Investigation 1953
Ä Each command is headed by Vice Admiral.
Ä The Navy is headed by the “Chief of the Naval Staff” Defence Research and Development
of the rank of Admiral. Organisation (DRDO)
Ä Each command is headed by Vice Admiral. It was established in 1958 to provide a solid base to the
Ä INS Shakti was India’s first indigenously built national security system.
submarine. Prithvi
Ä INS Kadamba is India’s largest naval base situated at Surface to surface missile with a short range of 150 km
Karwar, Karnataka. with 1000 kg warhead and 250 km with 500 kg warhead.
Ä It was commissioned in 2005 under the project (Prithvi-I Army version, Prithvi-II-Air Version,
‘Seabird’. Prithvi-III-Naval.)

https://sscstudy.com/
https://sscstudy.com/

10 CUET (UG) Section III : General Test

Agni Awards, Honours and Prizes


Ä Surface to surface intermediate range ballistic missile
(IRBM) with a range of 1000 km-2500 km. Nobel Prize
Ä It was instituted by the inventor of dynamite, Alfred
Ä Angi V- Surface to surface Intercontinental Ballistic
Bernard Nobel (1833-96).
Missile (ICBM) with a range of 5000-6000 km.
Ä The award is given on December 10, which is the
Akash death anniversary of its founder.
Surface to surface missile with a range of 25 km. Ä Nobel Prize is given every year to those eminent persons
who have made pioneering achievements in the field of
Nag Physics, Chemistry, Medicine, Peace, Literature and
Third generation ‘fire and forget’ anti-tank missile with Economics.
a range of 4 km. Ä Apart from Economics, all other categories have been
3Trishul given since 1901. Economics Nobel Prize was
Surface to air missile with a range of 500m to 9 km. It is instituted in 1967 and was first given in 1969.
being developed for all the three services. Indian Nobel Prize Winners

MBT-Arjun Name Field Year

India’s Main Battle Tank (MBT) has been designed and Rabindra Nath Tagore Literature 1913
developed by DRDO. It has Global Positioning System Dr. CV Raman Physics 1930
(GPS) to facilitate to find its position. Dr. Hargovind Khurana Medicine 1968
Mother Teresa Peace 1979
Lakshya
Dr. S Chandrashekhar Physics 1983
It is the Pilotless Target Aircraft (PTA).
Dr. Amartya Sen Economics 1999
Nishant VS Naipaul Literature 2001
It is the Remotely Piloted Vehicle (RPV) for the Venkataraman Ramakrishnan Chemistry 2009
survelliance purpose. Kailash Satyarthi Peace 2014
Tejas Abhijeet Banerjee Economics 2019
Smallest, light weight, single-engine, single-seat,
multi-role, fourth generation combat aircraft—the Pulitzer Prize
Light Combat Aircraft (LCA).
Ä It was instituted in 1917 and named after the US
Publisher Joseph Pulitzer.
Pinaca Ä It is conferred annually in the USA for
It is a lethal ground based multibarrel Rocket Launcher accomplishments in journalism, literature and music.
weapon system. It has a range of 39 km and has a
capability to fire up to 12 rockets within seconds. Magsaysay Awards
Ä They were instituted in 1957 and named after Ramon
Brahmos Magsaysay, the late President of Philippines.
It is a cruise missile jointly developed by India and Russia. Ä This award is given annually on August 31, for
outstanding contributions to public service, community
India’s Atomic Research leadership, journalism, literature and creative arts
Ä India’s journey to atomic energy research started with and international understanding.
the establishment of the Atomic Energy Commission Ä First Indian to get this award was Vinobha Bhave.
on August 10, 1948, under the chairmanship of Dr
Homi J Bhabha. Man Booker Prize
Ä Subsequently, the Department of Atomic Energy
Ä It is the highest literary award given to the authors of
(DAE) was established in 1954 for implementation of British, Irish and Commonwealth countries.
atomic energy programmes. Ä It was instituted in 1968 by the Booker Company and
Ä Bhabha Atomic Research Centre (BARC) was the British Publishers Association along the lines of
established in 1957 at Trombay (Maharashtra). It is Pulitzer Prize of US.
India’s largest atomic research centre. Apsara (India’s Ä Booker Prize has been renamed as Man Booker Prize.
first atomic reactor), Cirus, Zerlina, Dhruva, Purnima
Oscar Awards
I and II and Kamini (India's first fast breeder nuclear Ä These awards were instituted in 1929 and conferred
reactor, at Kalpakkam) are the BARC’S atomic
annually by the Academy of Motion Pictures in USA.
reactors.

https://sscstudy.com/
https://sscstudy.com/

GENERAL KNOWLEDGE 11

Ä These are considered the most prestigious awards in Gallantry Awards


the cinema world. The first Indian to get an Oscar was Param Vir Chakra is the highest decoration of valour award. It is the most
Bhanu Athaiya for the movie ‘Gandhi’. conspicuous act of bravery or some act of valour or self-sacrifice in the
presence of the enemy, whether on land, at sea or in the air. The medal is
Ä Satyajit Ray was awarded Oscar for lifetime made of bronze.
achievements in cinema in 1992. Mahavir Chakra is the second highest gallantry award for acts of
conspicuous gallantry in the presence of the enemy whether on land, at
Bharat Ratna sea or in the air. The medal is made of standard silver.
Ä It is the highest civilian award of India. It is presented Vir Chakra is awarded for acts of gallantry in the presence of enemy,
by the Government of India. whether on land, at sea or in the air. The medal is made of standard silver.
Ä It is presented for exceptional public service and rarest Ashok Chakra This is awarded for valour, courageous action or sacrifice,
away from the battlefield. It is highest military award during peacetime.
achievements in the field of art, literature, sport and
Kirti Chakra The decoration is awarded for conspicuous gallantry. It is
science. made of standard silver and is circular in shape. The obverse and the
Ä It was instituted in 1954 and the first recepient were C reverse are exactly the same as in Ashoka Chakra.
Rajagopalachari, Dr Radhakrishnan and CV Raman. Shaurya Chakra The decoration is awarded for an act of gallantry during
peacetime.
Ä Padma Vibhushan is the second highest civilian award
for distinguished services in any field including Bharatiya Jnanpith Award
Government service. Ä Instituted in 22nd May, 1961, carries a cash prize of
Ä Padma Bhushan and Padma Shree are the other ` 5 lakh, a citation and a bronze replica of Vagdevi
important civilian awards. (Saraswati).
Sahitya Akademy Awards Ä This award is given for the best literary writing by an
It was instituted in 1955 and is given for any exclusive Indian citizen in a language listed in Eighth Schedule
of the Indian Constitution.
writing in any of the 22 languages including English
literature during last 5 years. Gandhi Peace Prize
Shanti Swaroop Bhatnagar Awards
Ä Established in 2nd October, 1994, on the occasion of
the 125th birthday anniversary of Mahatma Gandhi,
These awards are given to the Indian scientists for their
carries a cash prize of ` 1 crore.
exceptional performance. Ä Indian Government instituted this annual prize to
Arjuna Awards encourage and promote the significance of Gandhian
Ä These were instituted in 1961 and given by Sports values over the world.
Ministry, Government of India. National Film Awards
Ä These are given for the special achievements in Ä The National Film Awards are the most prominent film
different types of sports. award ceremony in India, established in 1954 and it is
administered, along with the international film festival
Dronacharya Awards
of India and the Indian Panorama, by the Indian
Ä These were instituted in 1985 and given by Sports government’s Directorate of Film Festivals since 1973.
Ministry, Government of India. Ä Due to the national scale of the National Film Awards,
Ä These are given to sports coaches. it is considered to be the equivalent of the American
Rajiv Gandhi Khel Ratna Academy Awards.
(Major Dhayan Chand Khel Ratna Award) Filmfare Awards
Ä Rajiv Gandhi Khel Ratna Award formerly known as Ä The filmfare awards are presented annually by the
Major Dhyan Chand Khel Ratna Award is the highest times group to honour both artistic and technical
sporting honour of India. excellence of professionals in the Hindi language film
Ä This award was instituted in 1991-92. It is awarded industry of India.
annually by the ministry of Youth Affairs and Sport. Ä They were initially referred to as the Clare Awards
after the editor of the Times of India, Clare Mendonca.
Dadasaheb Phalke Award
Ä Dadasaheb Phalke is known as the Father of Indian Sangeet Natak Akademi Puraskar (Akademi Award)
Cinema. The highest National Film Award is named Ä Awarded by the Sangeet Natak Akademi India’s
after him in 1969. National Academy of Music, Dance and Drama.
Ä Mrs Devika Rani Roerich was the first person to Ä It is the highest Indian recognition given to practicing
receive Dadasaheb Phalke Award in 1969. artists in the categories of music, dance, theatre, other
Ä It was instituted in 1962 and is presented for traditional/folk/tribal/dance/music/theatre and Puppetry
commendable display by the players. and contribution/scholarship in performing arts.

https://sscstudy.com/
https://sscstudy.com/

12 CUET (UG) Section III : General Test

Lalit Kala Akademi Ratna Musical Instruments and Instrumentalists


Ä Instituted in 1955 by the Government of India is an Instruments Instrumentalists
honour for the fine arts given to eminent artists for Stringed Instruments
their lifetime achievements in the field of visual arts. Been Asad Ali Khan, Zia Moin-ud-din Khan
Ä It is awarded by the Lalit Kala Akademi, India’s Santoor Shiv Kumar Sharma
National Academy of Art, it is the highest honour in
Sarod Buddhadev Dasgupta, Ali Akbar Khan, Amjad Ali khan,
the fine arts conferred by the Government of India.
Bahadur Khan, Sharan Rani, Zarin S Sharma
Classical Dancers of India Sarangi Ustad Binda Khan
Dance Dancer Sitar Ravi Shankar, Hara Shankar Bhattacharya, Nikhil,
Bharatnatyam Bala Saraswati, CV Chandrasekhar, Leela Samson, Banerjee, Vilayat Khan, Mustaq Ali Khan
Mrinalini Sarabhai, Padma Subramanyam, Rukmini Surb Ahar Sajjad Hussain, Annapurna
Devi, Sanyukta Panigrahi, Sonal Mansingh, Yamini
Veena Doraiswamy Iyengar, Chittibabu, Emani Sankara
Krishnamurti
Shastri, Dhanammal, S Bala Chandran, KR
Kathak Bharti Gupta, Birju Maharaj, Damayanti Joshi, Durga
Kumaraswamy
Das, Gopi Krishna, Kumudini Lakhia, Sambhu
Maharaj, Sitara Devi Violin Gajanan Rao Joshi, MS Gopal Krishnan, TN
Krishnan, Baluswamy, Dikshitar, Dwaran
Kuchipudi Josyula Seetharamaiah, Vempathi Chinna Sathyam
Venkataswamy Naidu Lalyuli G Jayaraman,
Manipuri Guru Bipin Sinha, Jhaveri Sisters, Nayana Jhaveri, Nirmala Mysore T Chowdiah, VG Jog
Mehta, Savita Mehta
Wind Instruments
Odissi Debaprasad Das, Dhirendra Nath Patnaik, Indrani
Rahman, Kelucharan Mahapatra, Priyambada Mohanty Flute TR Mahalingam, N Ramani, Hari Prasad Chaurasia,
Kathakali Mrinalini Sarabhai, Guru Shankaran, Namboodripad, Pannalal Ghosh
Thottam Shankaran, Kutti Nayyar, Shankar Kurup, Nadaswaran Sheikh Chinna Moula, Neeruswamy Pillai, Rajaratanam
KC Pannikar, TT Ram Kulti Pillai
Mohiniattam Protima Devi, Sanyukta Panigrahi, Sonal Mansingh, Shehnai Bismillah Khan
Pankaj Charan Das, Kelucharan Mahapatra, Madhvi Percussion (Strikting Thumping) Instruments
Mudgal, etc.
Mridangam Palghat Mani Iyer, Karaikudi R Mani, Palghat Raghu
State and Folk Dances Pakhawag Pt Ayodhya Prasad, Gopal Das, Babu Ram Shanker
Andhra Pradesh Kuchipudi, Ghantamardala, Ottam Thedal Pagaldas

Asom Bihu, Bichhua, Natpuja, Maharas, Kaligopal, Tabla Zakir Hussain, Nikhil Ghosh, Kishan Maharaj, Alla
Bagurumba, Naga dance, Khel Gopal, Tabal Rakha Khan, Pandit Samta Prasad, Kumar Bose, Latif
Chongli, Canoe, Jhumura Hobjanai Khan
Bihar Jata-Jatin, Bakho-Bakhain, Panwariya, Kanjira Pudukkotai Dakshinamurthi Pillai
Sama-Chakwa, Bidesia
Cultural Institutions
Gujarat Garba, Dandiya Ras, Tippani Juriun, Bhavai
Haryana Jhumar, Phag, Daph, Dhamal, Loor, Gugga, Khor, Institutions Headquarters
Gagor ¡ Anthropological Survey of India, 1945 Kolkata
Himachal Pradesh Jhora, Jhali, Chharhi, Dhaman, Chhapeli, Mahasu, ¡ Archaeological Survey of India, 1861 New Delhi
Nati, Dangi ¡ Asiatic Society, 1784 (Sir William Jones) Kolkata
Jammu & Kashmir Rauf, Hikat, Mandjas, Kud Dandi Nach, Damali
¡ Indira Gandhi National Centre for Arts, 1985 New Delhi
Karnataka Yakshagan, Huttari, Suggi, Kunitha, Karga, Lambi
¡ Lalit Kala Akademi (National Academy of Fine Arts), New Delhi
Kerala Kathakali (Classical), Ottam Thulal, Mohiniattam, 1954
Kaikottikali
¡ National Archives of India, 1981 New Delhi
Maharashtra Lavani, Nakata, Koli, Lezim, Gafa, Dahikala
Dasavtar or Bohada ¡ National School of Drama, 1959 New Delhi
Odisha Odissi (Classical), Savari, Ghumara, Painka, Munari, ¡ Sahitya Academy, 1954 New Delhi
Chhau ¡ Sangeet Natak Academy, 1953 New Delhi
Paschim Banga Kathi, Gambhira, Dhali, Jatra, Baul, Marasia, Mahal, ¡ Library of Tibetan Works and Archives Dharmashala
Keertan etc
¡ Science City Kolkata
Punjab Bhangra, Giddha, Daff, Dhaman etc
¡ Victorial Memorial Hall Kolkata
Rajasthan Ghumar, Chakri, Ganagor, Jhulan Leela, Jhuma,
Suisini, Ghapal, Kalbeliya ¡ Birla Industrial and Tech Museum Kolkata
Tamil Nadu Bharatnatyam, Kumi, Kolattam, Kavadi ¡ Central Institute of Buddhist Studies Leh
Uttar Pradesh Nautanki, Raslila, Kajri, Jhora, Chappeli, Jaita ¡ Nava Nalanda Mahavihara Nalanda (Birla)
Uttarakhand Garhwali, Kumayuni, Kajari, Jhora, Raslila, Chappeli ¡ National Gallery of Modern Art New Delhi

https://sscstudy.com/
https://sscstudy.com/

GENERAL KNOWLEDGE 13

Sports Cricket World Cup


The first Cricket World Cup was organised in England
Olympics in 1975. A separate women’s Cricket World Cup has
Ä Olympics games were started in 776 BC on Mount been held every four years since 1973.
Olympia in the honour of Greek God ‘Zeus’.
Hockey World Cup
Ä The modern Olympic Games were started in Athens
The first Hockey World Cup was organised in Barcelona
the capital of Greece on 6th April, 1896 with great
(Spain) in 1971. Women’s Hockey World Cup has been
efforts made by French nobleman, Baron Pierre de
held since 1974. The 14th Hockey World Cup were be
Coubertin.
held in the Bhubaneswar, Odisha (India) in 2018.
Ä The Olympic Games are organised after every four
15th Hockey World Cup (Men’s) will be held in Odisha
years.
(India) in 2023.
Ä The Olympic Flag is made up of White Silk
and contains five interwined rings as the Olympic Football World Cup
Emblem. Ä The Football World Cup is organised by FIFA
Ä The five interlaced rings are arranged in 3-2 pattern (Federation of International Football Association). The
on a white background, with the blue ring to the World Cup is called ‘Jules Rimet Cup’ named after the
extreme left, followed by yellow, black, green and red, name of FIFA President Jules Rimet. The first
in the same order. Football World Cup was organised in Uruguay in 1930.
Ä Blue for Europe, Black for Africa, Red for Americas Ä In 1942 and 1946, the Football World Cup was not
(North and South America), Yellow for Asia and Green played due to World War II.
for Oceania (Australia and New Zealand). Ä Brazil is the only nation to have participated in every
Ä The official Olympic Motto is ‘‘Citius, Altius, World Cup so far. The 2014, 2018 and 2022
Fortius’’, a Latin phrase meaning Swifter, Higher, Football World Cup scheduled to be held at Brazil,
Stronger. Russia and Qatar respectively.
Ä The Head Office of International Olympic Committee Important Sport Terms
(IOC) is at Lausanne (Switzerland). Sport Terms
Commonwealth Games Basketball Dunk, Front court, Lay up, Held ball, Pivot, Rebound,
Steal
Ä The first Commonwealth Games were held in 1930 in
Hamilton, Canada. Cricket Bye, Draw, Googly, Topspin, Over throw, Duck, Hit
wicket
Ä Since 1930, the games have been conducted every four
Football Bend dribble, Dissent, Dummy, Feint, Free kick, Header,
years except for 1942 and 1946 due to World War II. Red card, Throwins
Ä The Commonwealth Games Federation (CGF) is the Hockey Bully, Striking, Circle, Post back
organization which is responsible for the direction and Chess Castle, Diagonaes, Files, Pawns, Peices, Promote,
control of the Commonwealth Games. Gambit, Pawn
Ä There are currently 54 members of the Commonwealth Boxing Jab, laying on Knock, Second out habbit punch,
of Nations and 71 teams participated in the games. Upper cut
Ä 21st Commonwealth Games of 2018 were held at Badminton Loab, Let, Drive, Drop, Love
Goldcoast city, Queens land (Australia). Polo Chuker, Bunker
Ä 22nd commonwealth Games of 2022 will be held at Baseball Diamond, Home run, Put out, Strike, Ant-rubber.
Berminghom (England) Rifle Shooting Target, Muzzle fulb, Bulls eye
Wrestling Half, Nelson, Hold Sager, Rebuts
The Asian Games
Golf Fore some, Stymie, T, put hole, Caddy, Nib lick, Iron, The
Ä The Asian Games, also called the Asiad, are a green, Bunkeer
multi-sport event held every four years among athletes
Billiards Jigger, Pot, Break pot, In off, Cans, Bolting, Long,
from all over Asia. Hazard, Cue
Ä The games are regulated by the Olympic Council of Swimming Breast Stroke, Twist, Butterfly, Crawl, Spring Board
Asia (OCA), under the supervision of the International Volley Ball Antennae, Attack hit, Libero, Service, Set up, Blocking,
Olympic Committee (IOC). Dribbling
Ä 18th Asian Games of 2018 were held at Jakarta Lawn Tennis Advantage, Ace, Dence, Volley, Foot Foult, Smash,
(Indonesia). Grand-Slam, Slice, Love
Ä 19th Asian Games of 2022 will be held in Hangzhou, Table Tennis End line, Flat hit, Foil, Service, Phnholder grip, Reverse,
China. Top-spin, Couter-hitting, Let

https://sscstudy.com/
https://sscstudy.com/

14 CUET (UG) Section III : General Test

Nickname of Players Number of Players


Sports
(in each team)
Player Nickname Players Nickname
Golf Several individuals compete
Major Hockey ka Jadugar Rahul Dravid The wall
simultaneously
Dhyanchand
Badminton, Tennis and Table 1 or 2
Milkha Singh Flying Sikh Harbhajan Singh Bhajji, The
Tennis (Singles and Doubles respectively)
Turbanator
Gymnastic Several individuals compete
PT Usha Payyoli Express, Javagal Srinath Mysore
simultaneously
Golden Girl Express
Shoaib Aktar Rawalpindi Express Paes and Bhupati Indian Express Important Books of Ancient India
Important Cups and Trophies Books Authors

Sport Cup and Trophy Mudrarakshas Vishakha Dutta


Cricket Irani Trophy, Duleep Trophy, Ranji Trophy, Vizzy Trophy, Mahabhasya Patanjali
Asia Cup, Deodhar Trophy, CK Naidu Trophy, Ashtadhyayi Panini
Cooch-Behar Trophy
Vikramankadevacharita Bilhan
Football Durand Cup, Nizam Gold Cup, Rovers Cup, Sanjay Gold
Cup, Santosh Trophy, Subroto Mukherjee Cup, Vittal Ramcharit Sandhyakar Nandi
Trophy, Nehru Gold Cup Prabandha Kosh Rajashekhara
Hockey Agha Khan Cup, Azlan Shah Cup, Nehru Trophy, Arthashastra Chanakya (Kautilya)
Dhyanchand Trophy, Beighton Cup, Scindia Gold Cup, Modi
Gold Cup, Indira Gandhi Gold Cup, Rangaswami Cup, Khan Nitishastra Sukra
Abdul Gaffar Cup. Brihat-Katha Kosha Harisena
Golf Canada Cup, Muthian Gold Cup, Ryder Cup, Walker Cup Kamasutra Vatsayana
Table Tennis Corbillion Cup (women), Jayalaxmi Cup (women),
Svapna-Vasavdatta Bhasa
Swaythling Cup (men)
Lawn Tennis Davis Cup, Hamlet Cup, Australian Open, French Open, Buddacharita Asvaghosh
Wimbledon, US open Fu-Kuo-Ki Fa-hien
Badminton Thomas Cup (men), Uber Cup (women), Narang Cup Kiratarjuniyam Bharavi
Boxing Aspy Adjania Trophy Panchatantra Vishnusharma
Rowing Wellington Trophy Surya Siddhanta Aryabhatta
Bridge Ruia Trophy
Pancha Siddhantika Varahamihir
Polo Ezra Cup, Winchestor Cup, Radha Mohan Cup
Bagvatgita Ved Vyas
Important National Sports Gita-Govinda Jayadeva
Country National Sport Country National Sport
Important Socio-Religious Organisations
Australia Cricket India Hockey
Canada Lacrose Japan Sumo Organisation Place Founder Year

China Table Tennis Malaysia Badminton Atmiya Sabha Calcutta Raja Rammohan Roy 1815
England Cricket Scotland Rugby/Football Brahmo Samaj Calcutta Raja Rammohan Roy 1828
Spain Bull Fighting USA Baseball Brahmo Samaj of Calcutta Keshab Chandra Sen 1866
India
Number of Players in Some Population Sports Sadharan Brahmo Calcutta Anand Mohan Bose & 1878
Number of Players Samaj Shivnath Shastri
Sports
(in each team) Dharma Sabha Calcutta Radhakant Deb 1829
Hockey, Football and Cricket 11 Tattvabodhini Calcutta Devendranath Tagore 1839
Water Polo 7 Sabha
Kho Kho 9 Paramhansa Bombay Dadoba Pandurang, 1849
Babaddi 7 Mandali Gopalhari Deshmukh
Billiards/Snooker 1 Radhaswami Agra Tulsi Ram (Shiv Dayal 1861
Movement Saheb)
Boxing/Chess 1
Bridge 2 Prarthana Samaj Bombay Atmaram Pandurang 1867

Netball 7 Arya Samaj Bombay Dayanand Saraswati 1875


Volleyball 6 Theosophical New York Madan H P Blavatsky & 1875
Society Colonel H S Olcott
Basketball 5

https://sscstudy.com/
https://sscstudy.com/

GENERAL KNOWLEDGE 15

Organisation Place Founder Year Major Rivers of the World


Deccan Education Poona G G Agarkar 1884 River Origin
Society
Nile Victoria lake
Indian National Bombay M G Ranade 1887
Amazon Andes (Peru)
Social Conference
Yangtze Tibetan Kiang Plateau
Dev Samaj Lahore Shivnarayan Agnihotri 1887
Ramkrishna Belur Swami Vivekanand 1897 Mississippi Missouri Itaska Lake (USA)
Mission Yenisei Tannu-Ola Mountains
Servant of India Bombay G K Gokhale 1905 Huang Ho Kunlun Mountains
Society
Ob Altai Mountains, Russia
Social Service Bombay N M Joshi 1911
League Congo Lualaba and Luapula rivers
Seva Samiti Allahabad H N Kunjru 1914 Amur North East China
Rahnumai Bombay Dadabhai Naoroji, Naoroji 1851 Lena Baikal Mountains
Mazdyasan Sabha Fardonji, S S Bengalee Mekong Tibetan Highlands
Major Mountain Ranges Niger Guinea

Range Location Length (km)


Important Straits of the World
Andes South America 7200
Himalayan, Karakoram South Central Asia 5000 Strait Water Bodies joined Area
and Hindu kush Bab-el- Red Sea and Arabian Sea Arabia and Africa
Rockies North America 4800 Mandeb
Great Dividing Range East Australia 3600 Bering Arctic Ocean and Bering Sea Alaska and Asia
Atlas North-West Africa 1930
Bosphorus Black Sea and Marmara Sea Turkey
Western Ghats Western India 1610
Dover North Sea and Atlantic Ocean England and Europe
Caucasus Europe 1200
Alaska USA 1130 Florida Gulf of Mexico and Atlantic Ocean Florida and Bahamas
Alps Europe 1050 Islands
Gibralter Mediterranean Sea and Atlantic Spain and Africa
Major Mountain Peaks Ocean
Mountain Malacca Java Sea and Bay of Bengal India and Indonesia
Mountain Peak Location Location
Peak
Palk Bay of Bengal and Indian Ocean India and Sri Lanka
Mt Everest Nepal-Tibet Tirich Mir Pakistan
(highest in the world) Megellan South Pacific and South Atlantic Chile
Ocean
K2 (Godwin Austin) India (POK) Aconcagua Argentina
Sunda Java Sea and Indian Ocean Indonesia
Dhaulagiri Nepal Cotopaxi Ecuador
Annapurna Nepal Kilimanjaro Tanzania Important Lakes of the World
Gurla Mandhata Tibet
Lake Location Lake Location
Famous Plateaus of the World Caspian Asia Tanganyika Africa
Superior Canada and USA Baikal Russia
Plateau Situation
Victoria Africa Great Bear Canada
Tibetan Plateau Between Himalayas and Kunlun Mountains
Huron Canada and USA Aral Kazakshtan
Deccan Plateau Southern India
Michigan USA Great Slave Canada
Arabian Plateau South-West Asia
Plateau of Brazil Central-Eastern South America List of Waterfalls
Plateau of Mexico Mexico
Waterfall Location
Plateau of Colombia USA
Angel Falls Venezuela
Plateau of Madagascar Madagascar
Tugela Falls South Africa
Plateau of Alaska North-West North America
Monge Norway
Plateau of Bolivia Andes Mountains
Yosemite United States
Great Basin Plateau South of Colombia Plateau, USA
Catarata Yumbilla Peru
Colorado Plateau South of Great Basin Plateau, USA

https://sscstudy.com/
https://sscstudy.com/

16 CUET (UG) Section III : General Test

Riverside Cities Important International Boundary Lines


Town River Town River Name In Between
Akyab (Myanmar) Irrawaddy Baghdad (Iraq) Tigris
Radclife Line (1947) India and Pakistan (Indo-Pak)
Basara (Iraq) Tigris and Belgrade Danube
Mac Mohan (1914) India and China (Indo-China)
Euphrates
Berlin (Germany) Spree Bristol (UK) Avon Durand Line (1896) Pakistan and Afghanistan
Budapest (Hungary) Danube Cairo (Egypt) Nile Hindenburg Line Germany and Poland
Canton Si-Kiang Glasgow (Scotland) Clyde Maginot Line France and Germany
Hamburg (Germany) Elbe Jamshedpur Subarnarekha Oder Neisse Line Germany and Poland
Kabul Kabul Khartoum (Sudan) Nile Siegfried Line Fortification between Germany and France
Karachi Indus Lahore Ravi
38th Parallel Line North and South Korea
Lisbon (Portugal) Tangus London (UK) Thames
49th Parallel Line USA and Canada
Lucknow Gomti Montreal (Canada) Ottawa
24th Parallel Line Pakistan claims that it is the boundary between India
Nanking Yang-tse-Kiang New Castle(UK) Tyre
and Pakistan in Rann of Kuchch
New Orleans (USA) Mississippi New York(USA) Hudson
17th Parallel Line North Vietnam and South Vietnam
Paris (France) Seine Philadelphia (USA) Delaware
Rome (Italy) Tiber Shanghai Yang-tse-Kiang Highest Peaks of India
Srinagar Jhelum Warsaw (Poland) Vistula
Highest Peak Height (in m) State
Yangon (Myanmar) Irawady
MT K2 8611 PoK (India)
Great Deserts Kanchenjunga 8598 Sikkim
Name Country/Region Nanda Devi 7817 Uttarakhand
Sahara (Libyan, Nubian) North Africa Saltoro Kangri 7742 Jammu and Kashmir
Australian (Gibson, Simpson), Victorian Great Sandy) Australia
Kangto 7090 Arunachal Pradesh
Arabian (Rub al Khali, An-Nafud) Arabia
Reo Purgyil 6816 Himachal Pradesh
Dast-e-Lut (Barren Desert) Iran
Saramati 3841 Nagaland
Dast-e-Kavir (Salt Desert) Iran
Desierto de Sechura Peru Sandakphu 3636 Paschim Banga
Atacama North Chile Khayang 3114 Manipur
Patagonia Argentina Anaimudi 2695 Kerala
Kalahari Botswana Dodda Betta 2636 Tamil Nadu
Namib Namibia
Important Indian Towns on Rivers
Major Islands of the World
Town
Rank Name Area (km2 )3 Country Town River River
1. Greenland 2130800 Dernmark
Jamshedpur Subarnrekha Delhi Yamuna
2. New Guinea 785753 Indonesia
Kanpur Ganga Surat Tapti
3. Borneo 748168 Indonesia, Malaysia
4. Madagascar 587713 Madagascar Ferozpur Sutlej Allahabad At the confluence of the
5. Baffin Island 507451 Canada Ganga and Yamuna
6. Sumatra 443066 Indonesia Varanasi Ganga Haridwar Ganga
Badrinath Alaknanda Ludhiana Sutlej
Famous Grasslands of the World
Srinagar Jhelum Ayodhya Saryu
Grassland Country
Steppe Eurasia Ahmedabad Sabarmati Patna Ganga
Pustaz Hungary Kota Chambal Jabalpur Narmada
Prairie USA Panji Mandavi Ujjain Kshipra
Pampas Argentina and Uruguay (South America)
Guwahati Brahmaputra Kolkata Hooghly
Weld South Africa
Cuttack Mahanadi Hyderabad Musi
Downs Australia
Canterbury New Zealand Nasik Godavari Lucknow Gomti

https://sscstudy.com/
https://sscstudy.com/

GENERAL KNOWLEDGE 17

Important Rivers of India


Name Originates from Falls into Name Originates from Falls into
Ganges Combined Sources Bay of Bengal Brahmaputra Near Mansarovar Lake Bay of Bengal
Sutlej Mansarovar Rakas Lakes Chenab Narmada Amarkantak Gulf of Khambat
Indus Near Mansarovar Lake Arabian Sea Tapti Betul District in Madhya Gulf of Khambat
Ravi Kullu Hills near Rohtang Pass Chenab Pradesh
Beas Near Rohtang Pass Sutlej Mahanadi Raipur District in Chhattisgarh Bay of Bengal
Jhelum Verinag in Kashmir Chenab Luni Aravallis Rann of Kuchchh
Yamuna Yamunotri Ganga Ghaggar Himalayas Near Fatehabad
Chambal Singar Chouri Peak, Vindhyan Yamuna Sabarmati Aravallis Gulf of Khambat
escarpment Krishna Western Ghats Bay of Bengal
Ghaghara Matsatung Glacier Ganga Godavari Nasik district in Maharashtra Bay of Bengal
Kosi Near Gosain Dham Peak Ganga Caurery Brahmagir Range of Western Bay of Bengal
Betwa Vindhyanchal Yamuna Ghats
Son Amarkantak Ganga Tungabhadra Western Ghats Krishna

Important Sanctuaries and National Parks


Name Location Reserve For
¡ Kaziranga National Park Asom One Horned rhinoceros, gaur, elephant, leopard and wild buffalo
¡ Sonai Rupa Sanctuary Asom Elephant, sambhar, wild boar and one-horned rhinoceros
¡ Namdapha Sanctuary Arunachal Pradesh Elephant, panther, sambhar, tiger, cheetal and king cobra
¡ Gautam Buddha Sanctuary Bihar Tiger, leopard, sambhar, cheetal and barking deer
¡ Achanakmar Sanctuary Chhattisgarh Tiger, boar, cheetal, sambhar and bison
¡ Valvadore National Park Gujarat Wolf and black buck
¡ Wild Ass Sanctuary Gujarat Wild ass, wolf, nilgai and chinkara
¡ Gir Forest Gujarat India’s biggest wildlife sanctuary famous for Gir lions
¡ Dachigam Sanctuary Jammu and Kashmir Kashmiri stag
¡ Banerghatta National Park Karnataka Elephant, cheetal, deer and grey partridge and green pigeon
¡ Bhadra Sanctuary Karnataka Elephant, cheetal, panther, sambhar and wild boar
¡ Bandipur Sanctuary Karnataka Elephant, tiger, panther, sambhar, deer and birds
¡ Dandeli Sanctuary Karnataka and Tamil Nadu Tiger, panther, elephant, cheetal, sambhar and wild boar
¡ Tungabhadra Sanctuary Karnataka Panther, cheetal, sloth bear and four-horned antelope
¡ Nagarhole National Park Karnataka Nagarhole National Park
¡ Pachmarhi Sanctuary Karnataka Tiger, panther, boar, sambar, nilgai and barking deer
¡ Gandhi Sagar Sanctuary Madhya Pradesh Cheetal, sambhar, chinkara and wild birds
¡ Bandhavgarh National Park Madhya Pradesh Tiger, panther, cheetal, nilgai and wild boar
¡ Simlipal Sanctuary Madhya Pradesh Elephant, tiger, leopard, gaur and cheetal
¡ Ghana Bird Sanctuary Water birds, black-buck, cheetal and sambar
¡ Kanchenzunga National Park Rajasthan Sikkim Snow leopard, musk deer and Himalayan boar
¡ Vedanthangal Bird Sanctuary Tamil Nadu Important bird sanctuary
¡ Chandraprabha Sanctuary Uttar Pradesh Gir lions, cheetal and sambhar
¡ Dudhwa National Park Uttar Pradesh Tiger, panther, sambar, cheetal, nilgai and barking deer
¡ Corbett National Park Uttarakhand Tiger, leopard, elephant and sambhar (named in memory of Jim Corbett)
¡ Jaldapara Sanctuary Paschim Banga Rhinoceros
¡ Sunderban Tiger Reserve Paschim Banga Tiger, deer, wild boar, crocodile and Gangetic dolphin.

https://sscstudy.com/
https://sscstudy.com/

18 CUET (UG) Section III : General Test

Practice Questions
1. Browning of paper in old books is caused by 15. In cricket, a run taken when the ball passes the
(a) frequent use (b) lack of aeration batsman without touching his bat or body is called
(c) collection of dust (d) oxidation of cellulose (a) drive (b) leg bye (c) bye (d) bosie
2. Who was popularly known as Africa's Gandhi? 16. Bernard Shaw is the author of the drama
(a) Mir Karzai (b) Nelson Mandela (a) Twelfth Night (b) Arms and the Man
(c) Firoz Gandhi (d) MK Gandhi (c) Death of a Salesman (d) Murder in the Cathedral
3. Name the body which has replaced the UN 17. Gol Gumbaz, the largest dome in India, is located in
Commission on Human Rights. the State of
(a) Secretariat (a) Bihar (b) Karnataka
(b) Human Rights Council (c) Maharashtra (d) Madhya Pradesh
(c) Trusteeship Council
(d) Economic and Social Council
18. National Consumer Rights Day is observed in India
on
4. The Great Wall of China was built by (a) December 24 (b) September 25
(a) Li-tai-pu (b) Shih Huang-ti (c) October 25 (d) November 25
(c) Lao-tze (d) Confucius
19. Yen is the currency of
5. In which year Colombia hosted the NAM summit (a) Yugoslavia (b) Mexico
(a) 1996 (b) 1997 (c) 1994 (d) 1995 (c) Japan (d) Thailand
6. The Central Food Technological Research Institute 20. Which one of the following is a ‘Surface to Air
(CFTRI) is located at missile’?
(a) Nagpur (b) Ernakulam (a) Trishul (b) K-15 Sagarika
(c) Mysore (d) Dehradun (c) Brahmos (d) Agni
7. Head office of coca cola company is in
21. Nehru Trophy is associated with
(a) Atlanta (b) Florida
(a) Hockey (b) Football
(c) New York (d) New Jercy
(c) Kabaddi (d) Table Tennis (Women)
8. Name the country where the first Industrial
22. Odissi classical dance originated in
Revolution took place (a) Orissa (b) Andhra Pradesh
(a) Germany (b) France
(c) Rajasthan (d) Gujarat
(c) America (d) Great Britain
23. Who discovered America?
9. The Indian Institute of Science is located at (a) Vasco-Da-Gama (b) Columbus
(a) Kolkata (b) Mumbai
(c) Captain Cook (d) Amundsen
(c) Chennai (d) Bengaluru
24. The famous painting ‘Monalisa’ was the creation of
10. Who had played key role in the formation of Lokpal (a) Michael-Angelo (b) Leonardo-Da-Vinci
bill in India? (c) Piccasso (d) Van Gogh
(a) Baba Amte
(b) Medha Patekar 25. The first woman to swim across seven important
(c) Vipin Hazarika seas is
(d) Anna Hajare (a) Chandini (b) Bula Choudhary
(c) Mridula Rajiv (d) Priya Shanbhag
11. Which of the following devices is generally used to
check multiple choice questions? 26. Who said that “India’s soul lives in villages”?
(a) MICR (b) Bar Code Reader (a) Vinoba Bhave (b) Jayaprakash Narayan
(c) OCR (d) OMR (c) Nehru (d) Gandhiji
12. Which one of the following is an ‘Air-to-air’ missile? 27. The headquarters of WTO is at
(a) Astra (b) Prithvi (a) New York (b) Doha
(c) Agni (d) Akash (c) Uruguay (d) Geneva
13. Flight-recorder is technically called 28. Which state is called the ‘Rice Bowl’ of India?
(a) altitude meter (b) dark box (a) Andhra Pradesh (b) Tamil Nadu
(c) blind box (d) black box (c) Kerala (d) Karnataka
14. National Library, the largest in India, is located in 29. Flight Recorder is technically called
(a) Kolkata (b) Chennai (a) Dark box (b) Blind box
(c) Mumbai (d) Delhi (c) Black box (d) Altitude meter

https://sscstudy.com/
https://sscstudy.com/

GENERAL KNOWLEDGE 19

30. Which of the following countries has entered into an 44. India won Champions Trophy 2013 under
agreement with India for the supply of Advanced Captainship of
Jet Trainer (HAWK)? (a) Virat Kohli (b) M S Dhoni
(a) Russia (b) USA (c) Ravindra Jadeja (d) Shikhar Dhawan
(c) England (d) France
45. What is called Dedicated Freight Corridor?
31. Yuan is the currency of (a) Railway track for freight movement
(a) Japan (b) China (b) Road link for freight movement
(c) Italy (d) Yugoslavia (c) A business hub of Uttar Pradesh
(d) None of the above
32. Santosh Trophy is associated with
(a) Hockey (b) Football 46. Who was the first Railway Minister of Independent
(c) Basketball (d) Badminton India?
(a) Morarjee Desai (b) John Mathai
33. Kathakali classical dance originated in (c) Sachchidananda Sinha (d) Dr. Ambedkar
(a) Kerala (b) Karnataka
(c) Rajasthan (d) Tamil Nadu 47. The term ‘Grand Slam’ is associated with which one
of the following game?
34. The 16 years old school boy, Arjun Vajpai who (a) Cricket (b) Swimming
became the youngest Indian to successfully climb (c) Lawn Tennis (d) Badminton
the world’s highest peak, Mount Everest, on May
22, 2010, is from which state? 48. How many countries are permanent member of the
(a) Uttar Pradesh Security Council of United Nations Organization?
(b) Madhya Pradesh (a) 4 (b) 6
(c) National Capital Territory of Delhi (c) 5 (d) 7
(d) Uttarakhand
49. Mary Kom belongs to which sports?
35. Which one of the following is an ‘Air-to-Air’ missile? (a) Wrestling (b) Boxing
(a) Prithvi (b) Agni (c) Shooting (d) Badminton
(c) Akash (d) Astra 50. The name of the White Revolution is associated
36. Who presented the first Union Budget of with
Independent India? (a) K Rangarajan (b) Verghese Kurien
(a) Dr. Rajendra Prasad (c) M S Swaminathan (d) J V Narlikar
(b) Jawaharlal Nehru 51. India’s Permanent Research Station ‘Dakshin
(c) R K Shanmukham Chetty Gangotri’ is located at
(d) Dr. Ambedkar (a) Indian Ocean (b) Himalayas
37. ‘Antara’ is the news agency of (c) Antarctica (d) Arabian Sea
(a) Saudi Arab (b) Indonesia 52. Anushka Shankar is the daughter of which famous
(c) Japan (d) Russia Indian?
38. ‘Reuters’ is the news agency of (a) Ravishankar (b) Uday Shankar
(a) Britain (b) Canada (c) Mamta Shankar (d) Tanushree Shankar
(c) France (d) None of these 53. Who, among the following is the propagator of
39. ‘Xinhua’ is the news agency of ‘Art of Living’?
(a) North Korea (b) Japan (a) Sri Sri Ravishankar (b) Swami Ramdev
(c) China (d) South Korea (c) Swami Agnivesh (d) Rajneesh

40. The Holy City Kedarnath devastated due to 54. The organization ‘World Wide Fund for Nature’
(a) Cloud burst (b) Earthquake (WWF) was established in 1961 at
(c) Bombing (d) Over commercialisation (a) France (b) UK
(c) Switzerland (d) Spain
41. ‘Sabawoon’ is the news agency of
(a) Turkey (b) Egypt 55. A popular dance form of Jammu and Kashmir is
(c) Afghanistan (d) Kazakhstan (a) Garba (b) Chhau
(c) Chakri (d) Bihu
42. Who one among the following Ex-Chief Minister did 56. ‘Otis’ is name associated with
not go to the jail in Scamsor Corruption Charges? (a) telegraph (b) submarine
(a) Om Prakash Chautala (b) Lalu Prasad (c) computer (d) lift
(c) Mayawati (d) Madhu Koda
57. The name of the eminent activist associated with
43. World famous book ‘Das Capital’ was written by Lokpal Bill is
(a) Angels (b) Karl Marx (a) Medha Patkar (b) Anna Hazare
(c) R.C. Dutta (d) None of these (c) Sonia Gandhi (d) Baba Amte

https://sscstudy.com/
https://sscstudy.com/

20 CUET (UG) Section III : General Test

58. Zubin Mehta, the Indian-born maestro is a 72. Which city is called the ‘City of Nawabs’?
(a) vocalist (b) painter (a) Delhi (b) Amritsar (c) Agra (d) Lucknow
(c) musician (d) poet
73. Which of the following festivals is associated with
59. The dance form popular in North East India is full moon day?
(a) Lai Haroba (b) Parhaun (a) Id-ul-Fitr (b) Easter
(c) Roof (d) Bhangra (c) Holi (d) Diwali
60. CRIS stands for 74. Which of the following is a classical dance from
(a) Centre for Railway Internal Services North India?
(b) Centre for Railway Information Systems (a) Kathak (b) Kathakali
(c) Centre for Rail and Indian Stations (c) Kuchipudi (d) Bharat Natyam
(d) Centre for Railway International Services
75. Which of the following name is not associated with
61. Among the following wonders of the world, the one painting?
that is still surviving is (a) K L Saigal (b) Jamini Roy
(a) Pyramids of Egypt (b) Colossus of Rhodes
(c) Manjit Bawa (d) M F Hussain
(c) Pharos of Alexandria (d) Statue of Zeus at
Olympia 76. Who among the following is not a Dada Saheb
Phalke Award winner?
62. ‘Devdas’ is written by (a) Amitabh Bachchan (b) Asha Bhonsle
(a) Sarat Chandra Chattopadhyay
(c) Pran (d) Mrinal Sen
(b) Vikram Seth
(c) Munshi Premchand 77. Who among the following is an Olympic Medal
(d) Rabindranath Tagore winner?
(a) P T Usha (b) Sania Mirza
63. ‘Court’ is the playing area for (c) Mahesh Bhupati (d) Saina Nehwal
(a) badminton (b) lawn tennis
(c) basketball (d) All of these 78. Which of the following award is given to recognise
outstanding achievement in sports?
64. The term not associated with football is (a) Padma Shri (b) Arjuna Award
(a) forward (b) corner (c) Param Vir Chakra (d) Ashok Chakra
(c) pass (d) penalty corner
79. The temple situated near Pushkar Lake in
65. Which of the following university is an Open Rajasthan is related to
University? (a) Lord Ganesh (b) Lord Vishnu
(a) Calcutta University (b) Mumbai University (c) Lord Mahesh (d) Lord Brahma
(c) IGNOU (d) Nalanda University
80. If someone is injured in an accident and broken his
66. Which of the following countries is not a permanent knee joint then he or she needs to consult
member of UN Security Council? (a) Orthopedic (b) Paediatrician
(a) USA (b) UK (c) Oncologist (d) Urologist
(c) France (d) Spain
81. If you have the option of buying a refrigerator with
67. ‘Green Peace International’ (Netherlands) was 1 star, 3 star, 5 star rating given by BEE, then
founded in which one you will prefer?
(a) 1961 (b) 1971 (c) 1984 (d) 1974 (a) 3 star (b) 5 star
68. ‘World Watch Institute’ is located in (c) 1 star (d) does not matter
(a) Rome (b) Paris
82. Name the city which hosts the literary festival
(c) Geneva (d) Washington
(largest in Asia Pacific)
69. Dada Saheb Phalke Award is given by the Ministry (a) Jaipur (b) Lucknow
of (c) Kolkata (d) New Delhi
(a) Social Welfare
83. Sri Pranab Mukherjee was elected as MP from
(b) Corporate Affairs
which of the following places before becoming the
(c) Home Affairs
President of India?
(d) Information and Broadcasting (a) Howrah (b) Jangipur
70. ‘Berlin’ is situated on the bank of river (c) Malda (d) Raiganj
(a) Denube (b) Volga
84. Who won the Best Actor Award for the film Paan
(c) Spree (d) Dnieper
Singh Tomar?
71. Which of the following is a ‘Fire and forget’ (a) Irrfan Khan
anti-tank missile? (b) Aamir Khan
(a) Trishul (b) Agni (c) Farhan Akhtar
(c) Akash (d) Nag (d) Ranbir Kapoor

https://sscstudy.com/
https://sscstudy.com/

GENERAL KNOWLEDGE 21

85. Hugo Chavez was the President of which country? 99. First Automated Teller Machine (ATM) is started in
(a) Cuba (b) Venezuela which of the following city?
(c) Argentina (d) Brazil (a) Kolkata (b) Mumbai
(c) New Delhi (d) Chennai
86. ‘Cue ball, break shot, pool table’ are terms associated
with which of the following games? 100. Who is the first woman to become an IPS Officer?
(a) Billiards (b) Squash (a) Ujjwala Rai (b) Anna George Malhotra
(c) Golf (d) Cricket (c) Kiran Bedi (d) Karnelia Sorabji
87. Who won the Champions Trophy, 2013? 101. Who is the first Indian cricketer to make a triple
(a) India (b) Sri Lanka century in Test Cricket?
(c) England (d) South Africa (a) Sachin Tendulkar (b) Virendra Sehwag
(c) Rahul Dravid (d) Sunil Gavaskar
88. On which river’s bank ‘Rome’ is located ?
(a) Don (b) Dnieper 102. Who is the writer of Abhigyanshakuntalam?
(c) Tiber (d) Po (a) Kalidas (b) Surdas
(c) Banbhatt (d) Tulsidas
89. Name the train which covers longest distance in
India. 103. ‘Baghdad’ is loated on the bank of
(a) Jammu Tawi Express (a) Vistula (b) Tigris
(b) Jammu Kanniyakumari Express (c) Eupherates (d) None of these
(c) Himsagar Express
(d) Vivek Express
104. Which is the capital of Afghanistan?
(a) Herat (b) Kandhar
90. Which of the following two terminal stations August (c) Kabul (d) Multan
Kranti Rajdhani Express is running?
(a) New Delhi - Howrah
105. Which is the currency of Japan?
(a) Yen (b) Yuan
(b) New Delhi - Chennai
(c) Rupaih (d) Rial
(c) New Delhi - Mumbai
(d) New Delhi - Hyderabad 106. Find the most suitable answer.
A tree always has
91. How many commands are made in Indian Navy?
(a) branches (b) leaves
(a) Two (b) Three
(c) fruit (d) root
(c) Four (d) Five
92. In which year Border Security Force (BSF) was 107. Which of the following two stations ‘Samjhota
established? Express’ is running?
(a) 1947 (b) 1950 (a) Delhi - Islamabad (b) Candla - Krachi
(c) 1960 (d) 1965 (c) Howrah - Dhaka (d) Attari - Amritsar

93. Which of the following Fighter Groups are include 108. Which of the following place is famous for ‘Rock
in Air Force? Garden’?
(a) Mig-21 Veriant (a) Bengaluru (b) Chandigarh
(b) Mig-29 S, Mig-25 S, Mig-27 S (c) Ahmedabad (d) Lucknow
(c) Jaguar 109. How many language are used on a ten rupee note?
(d) All of the above (a) 2 (b) 7
(c) 10 (d) 15
94. In which year Shatabadi Express was started?
(a) 1969 (b) 1972 110. The state emblem of India has been adopted from
(c) 1988 (d) 1977 (a) Palace of Chandragupta Maurya at Patna
(b) Ashokan Pillar at Allahabad
95. Which is the first Indian satellite to be launched? (c) Ashokan Pillar at Delhi
(a) Aryabhatt (b) Bhaskar-I (d) Lion Capital of Ashoka at Sarnath
(c) Rohini (d) Insat
111. How many spokes are in Indian National Flag ?
96. Ustad Amjad Ali is associated with (a) 24 (b) 23
(a) Sarod (b) Tabla (c) 26 (d) 22
(c) Violin (d) Sitar
112. The National Anthem of India was adopted by the
97. Which of the following folk dance is represent by Constituent Assembly on
Mridalini Sarabhai? (a) 26 Jan, 1950 (b) 15 Aug, 1947
(a) Bharatnatyam (b) Kathak (c) 24 Jan, 1950 (d) 26 Nov, 1949
(c) Kuchipuri (d) Manipuri17.
113. The National Calendar was adopted on
98. Who is the first Indian to climb Mount Everest? (a) 26 Jan, 1950 (b) 26 Nov, 1949
(a) Navog Sherpa (b) KD Yadav (c) 15 Aug, 1947 (d) 22 March, 1957
(c) Bachendri Pal (d) Tenjing Norge

https://sscstudy.com/
https://sscstudy.com/

22 CUET (UG) Section III : General Test

114. The National Song was composed by 129. First Indian woman to summit Mount Everest was
(a) Rabindranath Tagore (a) Santosh Yadav (b) Bachhendri Pal
(b) Maithilisharan Gupta (c) Bula Choudhary (d) None of these
(c) Bankim Chandra Chatterji
(d) Ramdhari Singh ‘Dinkar’
130. Who was the first woman to conquer Everest twice ?
(a) Arati Saha (b) Junko Tabei
115. Which is the National Flower of India ? (c) Bachhendri Pal (d) Santosh Yadav
(a) Rose (b) Lotus
(c) Lilli (d) Jasmine
131. First Indian woman to swim across seven seas of
the world was
116. The National Animal of India is (a) Bula Choudhary (b) Arati Saha
(a) Lion (b) Elephant (c) Bachhendri Pal (d) None of these
(c) Tiger (d) Rhinoceros
132. Who was the first Indian woman to cross English
117. The National Calendar begins on Channel twice ?
(a) 1 Chaitra (b) 1 Vaishakha (a) Bula Choudhary (b) Bachhendri Pal
(c) 1 Asadha (d) 1 Magh (c) Arati Saha (d) None of these
118. Which is the National Tree of India ? 133. First woman to became the President of UN
(a) Pipal (b) Banyan General Assembly was
(c) Sal (d) Neem (a) Madeleine Albright (b) Kim Campbell
(c) Vijayalakshmi Pandit (d) None of these
119. The National Fruit of the country is
(a) Banana (b) Coconut 134. First woman Chief Minister of a state in India was
(c) Mango (d) Apple (a) Vijayalakshmi Pandit (b) Sarojini Naidu
(c) Annie Beasant (d) Sucheta Kriplani
120. The National Bird of India is
(a) Duck (b) Eagle 135. Who became the first woman Lok Sabha speaker in
(c) Cuckoo (d) Peacock India ?
(a) Annie Beasant (b) Sucheta Kriplani
121. Which of the following is the National Sports of (c) Meira Kumar (d) Vijayalakshmi Pandit
India ?
(a) Football (b) Cricket 136. Who was the first US President to die in office ?
(c) Kabaddi (d) Hockey (a) George Washington (b) Thomas Jefferson
(c) William Henry Harrison (d) John Adams
122. India won the first Olympic Hockey Gold at
Amsterdam in 137. The first US President to be assassinated was
(a) 1932 (b) 1928 (a) Abraham Lincoln (b) Richard Nixon
(c) 1936 (d) 1948 (c) John Adams (d) None of these
123. The first man to land on the Moon was 138. The first and only US President to resign from
(a) Neil Armstrong (b) Yuri Gagarian office was
(c) Valentina Tereshkova (d) None of these (a) John Adams (b) Richard Nixon
(c) Thomas Jefferson (d) None of these
124. First woman of the world in space
(a) Dr Sally K Ride (b) Anonsheh Ansari 139. Who was the first Indian batsman to complete
(c) Marie Collins (d) Valentina Tereshkova 10000 runs ?
(a) Lala Amarnath (b) Sunil Gavaskar
125. Who was the first Indian to reach space (c) Sachin Tendulkar (d) Rahul Dravid
(a) Mihir Sen (b) Rakesh Sharma
(c) Major Somnath Sharma (d) S Mukherjee 140. Who became the first Indian cricketer to score a
triple century ?
126. Which was the first artificial satellite launched into (a) Sunil Gavaskar (b) Mohammad Azharuddin
orbit ? (c) Sachin Tendulkar (d) Virendra Sehwag
(a) Explorer 1 (b) Vostok 1
(c) Sputnik 1 (d) Apollo 8 141. Who became the first Indian to win an individual
Olympic Gold ?
127. First manned space vehicle was (a) K D Jadhav (b) Abhinav Bindra
(a) Vastok 1 (b) Apollo 11
(c) Sushil Kumar (d) Karnam Malleswari
(c) Apollo 8 (d) Explorer 1
142. Who is the first Indian woman medal winner in
128. First Asian woman to swim across the English Olympics ?
Channel was (a) S Vijayalakshmi
(a) Bula Choudhary
(b) Kamaljit Sandhu
(b) Arati Saha
(c) Karnam Malleswari
(c) Santosh Yadav
(d) None of the above
(d) None of the above

https://sscstudy.com/
https://sscstudy.com/

GENERAL KNOWLEDGE 23

143. The first Indian to cross the English Channel was 160. ‘Water Lily’ is the National Symbol of
(a) Mihir Sen (b) Santosh Yadav (a) UK (b) France
(c) Arati Saha (d) None of these (c) Bangladesh (d) All of these
144. Which was the first newspaper published in India ? 161. The National Symbol of Iran is
(a) The Hindu (b) Bengal Gazettear (a) Rose (b) Lily
(c) Samachar Darpan (d) Uddant Martand (c) Eagle (d) Moon and Stars
145. The first post-office in India was established at 162. The National Symbol of Russia is
(a) Kolkata (b) Delhi (a) Eagle (b) Double headed eagle
(c) Mumbai (d) Allahabad (c) Mapple Bird (d) None of these
146. The first postal stamp in India was issued in 163. ‘Mapple Bird’ is the National Symbol of
(a) 1855 (b) 1853 (c) 1857 (d) 1852 (a) Canada (b) UK
(c) Chile (d) None of these
147. In India, Radio Transmission Service was started at
Kolkata and Mumbai in 164. Which of the following is the international
(a) 1927 (b) 1866 (c) 1933 (d) 1874 boundary between India and Pakistan ?
(a) Red Cliff Line (b) Mac Mohan Line
148. The first Television Centre of India was established
(c) Maginot Line (d) Durand Line
in 1959 at
(a) Kolkata (b) Bangaluru 165. ‘49th Parallel’ is the boundary between
(c) New Delhi (d) Chennai (a) France and Germany
(b) North Korea and South Korea
149. The coloured programme on television was started
(c) USA and Canada
on
(d) None of the above
(a) 15 Aug, 1980 (b) 15 Aug, 1986
(c) 15 Aug, 1982 (d) 15 Aug, 1974 166. ‘38th Parallel’ is the boundary between
(a) North Korea and South Korea
150. Which was the first Indian satellite launched ?
(b) France and Germany
(a) Aryabhatta (b) Rohini
(c) USA and Canada
(c) Insat 1A (d) Insat 2A
(d) Germany and Poland
151. First indigenously built Indian satellite was
(a) Aryabhatta (b) Insat 2A 167. ‘17th Parallel’ is the boundary between
(a) North Vietnam and South Vietnam
(c) Insat 1A (d) Rohini
(b) USA and Canada
152. The first hydel plant in India was established at (c) USA and Mexico
(a) Darjeeling (b) Arunachal Pradesh (d) Peru and Chile
(c) Asom (d) Uttar Pradesh
168. The boundary line between West Germany and
153. The first indigenously designed and built Indian Poland is
Missile was (a) Hindenberg Line (b) 17th Parallel Line
(a) Nag (b) Trishul (c) Prithvi (d) Agni (c) 38th Parallel Line (d) Maginot Line
154. The first underground nuclear testing of India was 169. Who is popularly known as ‘Rajaji’ ?
carried in (a) C Rajgopalachari (b) C F Andrews
(a) 1971 (b) 1974 (c) 1977 (d) 1978 (c) Raja Ramanna (d) None of these
155. The first test-tube baby of India was 170. Who of the following is popularly known as ‘Tiger of
(a) Baby Harsha (b) Baby Khatoon Snow’ ?
(c) Baby Pinki (d) Baby Julu (a) Sir Walter Scott (b) Richard Golden
(c) Tenzing Norge (d) None of these
156. ‘Lion’ is the National Symbol of
(a) Belgium (b) Norway 171. Who of the following is known as ‘Blood and Iron
(c) Netherlands (d) All of these Man’ ?
(a) Sardar Patel (b) Bismark
157. ‘Eagle’ is the National Symbol of
(c) Hitler (d) None of these
(a) Spain (b) Sudan
(c) Germany (d) All of these 172. The official document of UK is known as
(a) Blue Book (b) Green Book
158. ‘Lily’ is the National Symbol of
(c) Yellow Book (d) Orange Book
(a) UK (b) France (c) Canada (d) Belgium
173. ‘Yellow Book’ is the official document of
159. ‘White Lily’ is the National Symbol of
(a) UK (b) Germany
(a) Italy (b) UK
(c) France (d) Italy
(c) France (d) Both ‘a’ and ‘b’

https://sscstudy.com/
https://sscstudy.com/

24 CUET (UG) Section III : General Test

174. ‘Orange Book’ is the official document of 176. ‘Anand Van’ is associated with
(a) Germany (b) Italy (a) Vinoba Bhave
(c) Portugal (d) Netherlands (b) Baba Amte
(c) Indira Gandhi
175. The place ‘Nirmal Hriday’ is associated with (d) Mahatma Gandhi
(a) Rabindranath Tagore
(b) Mother Teresa 177. ‘Phoenix Farm’ is associated with
(c) Guru Nanak (a) Mahatma Gandhi (b) J L Nehru
(d) Baba Amte (c) Rabindranath Tagore (d) None of these

ANSWERS
1. (d) 2. (b) 3. (b) 4. (b) 5. (d) 6. (c) 7. (a) 8. (d) 9. (d) 10. (d)
11. (d) 12. (a) 13. (d) 14. (a) 15. (c) 16. (b) 17. (b) 18. (a) 19. (c) 20. (a)
21. (b) 22. (a) 23. (b) 24. (b) 25. (b) 26. (c) 27. (d) 28. (a) 29. (c) 30. (b)
31. (b) 32. (b) 33. (a) 34. (a) 35. (c) 36. (c) 37. (b) 38. (a) 39. (c) 40. (a)
41. (c) 42. (c) 43. (b) 44. (b) 45. (a) 46. (b) 47. (c) 48. (c) 49. (b) 50. (b)
51. (c) 52. (a) 53. (a) 54. (c) 55. (c) 56. (d) 57. (b) 58. (c) 59. (a) 60. (b)
61. (a) 62. (a) 63. (d) 64. (d) 65. (c) 66. (d) 67. (b) 68. (d) 69. (d) 70. (c)
71. (d) 72. (d) 73. (c) 74. (a) 75. (a) 76. (a) 77. (d) 78. (b) 79. (d) 80. (a)
81. (b) 82. (a) 83. (b) 84. (a) 85. (b) 86. (a) 87. (a) 88. (c) 89. (d) 90. (c)
91. (b) 92. (d) 93. (d) 94. (c) 95. (a) 96. (a) 97. (a) 98. (d) 99. (b) 100. (c)
101. (b) 102. (a) 103. (b) 104. (c) 105. (a) 106. (d) 107. (d) 108. (b) 109. (d) 110. (d)
111. (a) 112. (c) 113. (d) 114. (c) 115. (b) 116. (c) 117. (a) 118. (b) 119. (c) 120. (d)
121. (d) 122. (b) 123. (a) 124. (d) 125. (b) 126. (c) 127. (a) 128. (b) 129. (b) 130. (d)
131. (a) 132. (a) 133. (c) 134. (d) 135. (c) 136. (c) 137. (a) 138. (b) 139. (b) 140. (d)
141. (b) 142. (c) 143. (a) 144. (b) 145. (a) 146. (d) 147. (a) 148. (c) 149. (c) 150. (a)
151. (b) 152. (a) 153. (c) 154. (b) 155. (a) 156. (d) 157. (d) 158. (b) 159. (d) 160. (c)
161. (a) 162. (b) 163. (a) 164. (a) 165. (c) 166. (a) 167. (a) 168. (a) 169. (a) 170. (c)
171. (b) 172. (a) 173. (c) 174. (d) 175. (b) 176. (b) 177. (a)

https://sscstudy.com/
https://sscstudy.com/

GENERAL KNOWLEDGE 25

CHAPTER 02

General Science
Inventions and Discoveries
Invention Year Inventor Country
Invention Year Inventor Country
Proton 1919 Rutherford N Zealand
Artificial Heart 1957 Willem Kolff Netherlands
Quantum Theory 1900 Plank Germany
Atomic Bomb 1945 J Robert Oppenheimer USA
Radar 1922 AH Taylor and USA
Leo C Young
Atomic Theory 1803 Dalton Britain
Radiocarbon dating 1947 Libby USA
Automatic Rifle 1918 John Browning USA
Relativity Theory 1905 Einstein Germany
Ball-Point Pen 1888 John J Loud USA
Stethoscope 1819 Laennec France
Barometer 1644 Evangelista Torricelli Italy
Telegraph Code 1837 Samuel FB Morse USA
Bicyle 1839- Kirkpatrick Macmillan Britain
40 Telescope 1608 Hans Lippershey Netherlands
Bifocal Lens 1780 Benjamin Franklin USA Television 1926 John Logie Baird Britain
(Mechanical)
Bunsen Burner 1855 R Willhelm von Bunsen Germany
Transformer 1831 Michael Faraday Britain
Calculus 1670 Newton Britain
Transistor 1948 Bardeen, Shockley USA
Cloning, DNA 1973 Boyer, Cohen USA and Brattain
Diesel Engine 1895 Rudolf Diesel Germany Wireless (telegraphy) 1896 G Marconi Italy
Dynamo 1832 Hypolite Pixii France X-ray 1895 WK Roentgen Germany
Electron 1897 Thomson J Britain
Fountain Pen 1884 Lewis E Waterman USA Milestones in Medicine
Galvanometer 1834 Andre-Marie Ampere France Discovery/ Discoverer/
Year Country
Invention Inventor
Gramophone 1878 Thomas Alva Edison USA
Aspirin 1889 Dreser Germany
Hydrogen Bomb 1952 Edward Teller USA
Ayurveda 2000-1000 BC — India
Intelligence Testing 1905 Simon Binet France
Bacteria 1683 Leeuwenhock Netherlands
Laser 1960 Theodore Maiman USA
Lift (Mechanical) 1852 Elisha G Otis USA Biochemistry 1648 Jan Baptista Van Belgium
Helmont
Lighting Conductor 1752 Benjamin Franklin USA
Blood Plasma 1940 Drew USA
Logarithms 1614 Napier Britain storage (Blood
bank)
Microphone 1876 Graham Bell USA
Blood Transfusion 1625 Jean-Baptiste France
Movie Projector 1893 Thomas Edison USA
Denys
Pacemaker 1952 Zoll USA
Chemotherapy 1493-1541 Paracelsus Switzerland
Pasteurization 1867 Louis Pasteur France
Circulation of 1628 William Harvey Britain
Periodic Table 1869 Mendeleyev Russia blood
Pistol, Revolver 1836 Colt USA Heart Transplant 1967 Christian S Africa
Surgery Barnard
Printing Press 1455 Johann Gutenberg Germany

https://sscstudy.com/
https://sscstudy.com/

26 CUET (UG) Section III : General Test

Discovery/ Discoverer/ Instrument Uses


Year Country
Invention Inventor Fathometer It measures the depth of the ocean.
Penicillin 1928 Alexander Britain Hydrometer It measures the specific gravity of liquids.
Fleming
Hygrometer It measures humidity in air.
Rabies Vaccine 1860 Louis Pasteur France
Lactometer It determines the purity of milk.
Rh-factor 1940 Karl Landsteiner USA
Microphone It converts the sound waves into electrical
Vaccination 1796 Edward Jenner Britain vibrations and to magnify the sound.
Vaccine, Polio 1954 Jonas Salk USA Microscope It is used to obtain magnified view of small
objects.
Vaccine, Polio-oral 1960 Albert Sabin USA
Periscope It is used to view objects above sea level
Vaccine, Rabies 1885 Louis Pasteur France (used in sub-marines).
Yoga 200-100 BC Patanjali India Pyrometer It measures very high temperature.
Radar It is used for detecting the direction and range of
Unit of Measurements an approaching plane by means of radio
Quantity Unit (SI) Quantity Unit (SI) microwaves.
Length Metre Weight Newton or Seismograph It measures the intensity of earthquake shocks.
Kilogram Stethoscope An instrument which is used by the doctors to hear
Time Second Impulse Newton-second and analyse heart and lung sounds.
Mass Kilogram Heat Joule Telescope It views distant objects in space.
Area Square metre Temperature Kelvin Thermometer This instrument is used for the measurement of
temperatures.
Volume Cubic metre Absolute Kelvin
temperature Voltmeter It measures the electric potential difference
between two points.
Velocity Metre/second Resistance Ohm
Acceleration Metre/second Electric current Ampere Ores of Metals
square Name of the
Ore Chemical Formulae
Density Kilogram/metre Electromotive Volt Elements
cube force Aluminium (Al) (a) Bauxite Al 2O 3 ⋅ 2H2O
Momentum Kilogram Electric energy Kilo watt hour (b) Kryolite Na 3 AlF6
metre/second Iron (Fe) (a) Haematite Fe 2O 3
Work Joule Electric power Kilo watt or watt (b) Magnetite Fe 3O 4
Energy Joule Charge Coulomb (c) Iron Pyrite FeS2
Force Newton Luminous flux Candela Copper (Cu) (a) Copper Pyrite CuFeS2
Pressure Pascal or Intensity of Decibel (b) Copper Glance Cu2S
Newton/sq sound Zinc (Zn) (a) Zinc Blende ZnS
metre (b) Calamine ZnCO 3
Frequency Hertz Power of lens Dioptre Sodium (Na) (a) Rock Salt NaCl
Power Watt Depth of sea Fathom (b) Sodium Carbonate Na 2CO 3
Lead (Pb) (a) Galena PbS
Important Scientific Instruments
(b) Anglesite PbCl 2
Instrument Uses
Mercury (Hg) (a) Cinnabar HgS
Altimeter It measures altitudes and is used in aircrafts. (b) Calomel Hg 2Cl 2
Ammeter It measures strength of electric current (in
amperes). Common and Chemical Names of
Anemometer It measures force and velocity of wind. Some Compounds
Audiometer It measures intensity of sound. Common Name Chemical Name Chemical Formulae

Audiophone It is used for improving imperfect sense of hearing. Dry Ice Solid Carbondioxide CO 2
Barometer It measures atmospheric pressure. Bleaching Powder Calcium Oxychloride CaOCl 2
Binocular It is used to view distant objects. Nausadar Ammonium Chloride NH4Cl
Dynamo It converts mechanical energy into electrical energy. Caustic Soda Sodium Hydroxide NaOH

https://sscstudy.com/
https://sscstudy.com/

GENERAL KNOWLEDGE 27

Common Name Chemical Name Chemical Formulae Some Important Facts of Human Body
Potash Alum Potassium K 2SO 4 ⋅ Al 2 (SO 4 )3 Number of cells in body 75 trillion
Aluminium Sulphate ⋅ 24H2O
Longest bone Femur (thigh bone)
Epsom Magnesium MgSO 4 ⋅ 7H 2 O Smallest bone Ear ossicle, stapes
Sulphate
Blood volume 6.8 litres (in 70 kg body)
Quick Lime Calcium Oxide CaO
1 Normal BP 120/80 mm Hg
Plaster of Paris Calcium Sulphate (CaSO 4 ) H 2 O
2 Life span of RBC 120 days
Gypsum Calcium Sulphate CaSO 4 ⋅ 2H 2 O Life span of WBC 3-4 days

Green Vitriol Ferrous Sulphate FeSO 4 ⋅ 7H 2 O Universal blood donor O Rh-ve

Blue Vitriol Copper Sulphate CaSO 4 ⋅ 5H 2 O Universal blood recipient AB

White Vitriol Zinc Sulphate ZnSO 4 ⋅ 7H 2 O Average body weight 70 kg

Marsh Gas Methane CH 4 Normal body temperature 98.4° F or 37°C


Dental formula adult : 2123/2123 = 32; child :
Vinegar Acetic Acid CH 3 COOH
2120/2120 = 22 milk teeth
Baking Powder Sodium Bicarbonate NaHCO 3
Gestation period 9 months (253-266 days)
Washing Soda Sodium Carbonate Na 2 CO 3 ⋅ 10H 2 O
Largest gland Liver
Magnesia Magnesium Oxide MgO
Largest muscle in the body Gluteus maximus (Buttock muscle)
Laughing Gas Nitrous Oxide N2O
Longest cell Neuron (nerve cell)
Chloroform Tricholoro Methane CHCl 3
Menstrual cycle 28 days
Vermelium Mercuric Sulphide HgS
Minimum distance for proper vision 25 cm
Borax Borax Na 2 B 4 O 7 ⋅ 10H 2 O
pH of urine 6.0
Alcohol Ethyl Alcohol C 2 H 5 OH
Nutrition
Sugar Sucrose C12 H 22 O11
Ä The components of food are : Carbohydrates, Fats,
Heavy Water Duterium Oxide D2O Proteins, Minerals, Vitamins, Water and Roughage.
Sand Silicon Oxide SiO 2 Carbohydrates
Ä Constitutes 3 elements : Carbon, Hydrogen and Oxygen.
Some Important Alloys
Ä Main source of energy providers.
Name Composition Use Ä 1 gm, gives 17 kg of energy (or 4.1 kcal of energy).
Brass Cu (60 to 80%), For making
Zn (40 to 20%) household utencils Fats
Ä Provides twice the energy of carbohydrates
Bronze Cu (75 to 90%), For making coins,
Sn (25 to 10%) idols, utencils (1 g provides 37 kJ of energy).
Ä An enzyme called Lipase digests fats. It breaks down into
German Silver Cu (60%), Zn (25%), For making utencils
Ni (15%) fatty acids and glycerol.
Duraliumin Al (95%), Cu (4%), In aircraft
Ä Daily requirement : 50 g.
Mg (0 ⋅ 5%), Mn ( manufacturing
0 ⋅ 5%) Proteins
Ä Made up of Carbon, Hydrogen, Oxygen and Nitrogen.
Steel Fe (98%), C (2%) For making nails,
screws, bridges Ä Important for growth and repair of the body.
Stainless Steel Fe (82%) Cr, Ni (18%) For making cooking
Ä Made up of amino acids.
utencils, knives Ä Kwashiorkar and Marasmus are the diseases which
An alloy is a mixture of two or more metals. occur due to deficiency of protein.

https://sscstudy.com/
https://sscstudy.com/

28 CUET (UG) Section III : General Test

Minerals
Types of Minerals
Mineral Element Source Significance Effect of Deficiency
Macro Element
Calcium (Ca)# Milk, cereals, cheese, Required for formation of teeth and bones, blood Weak teeth and bones; retarded body
green vegetables clotting, functions of nerves and muscles growth
Phosphorus (P) Milk, meat, cereals Required for formation of teeth and bones and Weak teeth and bones; retarded body
acid-base balance; component of ATP, DNA, RNA growth and physiology
Sulphur (S) Many proteins of food Component of many amino acids Disturbed protein metabolism.
Potassium (K) Meat, milk, cereals, fruits Required for acid-base balance, water regulation and Low blood pressure, weak muscles;
and vegetables function of nerves risk of paralysis
Chlorine (Cl) Table salt Required for acid base balance; component of gastric Loss of appetite; muscle cramps
juice
Sodium (Na) Table salt Required for acid-base and water balance and Low blood pressure, Loss of appetite;
nervous functions muscle cramps
Magnesium (Mg) Cereals, green vegetables Cofactor of many enzymes of glycoly- sis and a number Irregularities of metabolism principally
of other metabolic reactions dependent upon ATP affecting nervous functions
Iron (Fe) Meat, eggs, cereals, green Component of haemoglobin and cytochromes Anaemia, weakness and weak
vegetables immunity
Iodine (I) Milk, Cheese, Sea food, Important component of thyroxine hormone Goitre, Cretinism
iodized salt
Micro Element
Fluorine (F) Drinking water, tea, sea Maintenance of bones and teeth Weak teeth, larger amount causes
food mottling of teeth
Zinc (Zn) Cereals, milk, eggs, meat, Cofactor of digestive and many other enzymes Retarded growth, anaemia, rough skin,
sea food weak immunity and fertility
Copper (Cu) Meat, dry fruits, pods, Cofactor of cytochrome oxidase enzyme. Necessary Anaemia, weak blood vessels and
green vegetables, sea food for iron metabolism and development of blood vessels connective tissues
and connective tissues
Manganese (Mn) Dry fruits, cereals, tea, fruits Cofactor of some enzymes of urea synthesis and Irregular growth of bones, cartilages
and green vegetables transfer of phosphate group and connective tissues

# The salf of Ca required by our body is Ca 3 (PO 4 )2 .

Vitamins
Necessary for normal growth, good health, good vision, proper digestion of the body etc. They do not provide energy
to our body. Vitamins can be divided into two categories
Ä Water-soluble Vitamin B-complex, Vitamin C.

Ä Fat-soluble Vitamin A, Vitamin D, Vitamin E, Vitamin K.

Types of Vitamins
Vitamin Chemical Name Property Deficiency Disease
A Retinol General health giving vitamin, can be stored in liver Night blindness
B1 Thiamine For growth, carbohydrate metabolism, functioning of heart Beri-Beri
B2 Riboflavin For keeping skin and mouth healthy Cheilosis
C Ascorbic Acid For keeping teeth, gums and joints healthy. Gets destroyed on Scurvy
heating
D Calciferol For normal bones and teeth, can be stored in liver Rickets
E Tocopherol For normal reproduction, removes scars and wrinkles Sterility
K Phylloquinone For normal clotting of blood Haemophilia

https://sscstudy.com/
https://sscstudy.com/

GENERAL KNOWLEDGE 29

Water Communicable or Infectious Diseases


Important in digestion, transportation, excretion
and to regulate body temperature (body contains Diseases Caused by Virus
65% water). Disease Pathogen responsible

Roughage Smallpox Variola virus


Ä Fibrous material present in the cell wall of plants. Chickenpox Varicella virus
Ä Mainly contains cellulose. Common cold Rhino virus
Ä It doesn't provide energy but only helps in Influenza flu Orthomixo virus
retaining water in the body. Measles (Rubella) Measles virus (paramyxo virus)
Blood Mumps Mumps virus (paramyxo virus)
Ä Blood is a fluid connective tissue. Poliomylities Polio virus (entero virus)
Ä Its quantity is 6.8 litres in man and 500 ml less in Rabies (Hydrophobia) Rabies virus (rhabdo virus)
woman.
Dengue fever or Breakbone fever Dengue virus (arbo virus)
Ä Constitutes 6–8% of body weight and has a pH of
7.4. Acquired Immuno Deficiency Human T-cell Leukemia virus
Syndrome (AIDS) (HTLV-III); called LAV (retro virus)
Types of Blood Groups
Diseases Caused by Bacteria
Blood Group Can donate to Can receive from
Disease Pathogen responsible
A A, AB A, O Diphtheria Irregular rod (Corynebacterium diptheriae)
B B, AB B, O Pneumonia Diplococcus pneumoniae
AB AB A, B, AB, O Tuberculosis Irregular rod (mycobacterium tuberculosis)
O A, B, AB, O O Plague or Bubonic plague Short rod, Yersinia pestis
Tetanus or lockjaw Clostridium tetani
RH Factor Typhoid or enteric fever Salmonella typhi
● It is a blood antigen found in RBC. Cholera Vibrio cholerae
● A person can be Rh+ or Rh– depending upon the presence
Syphilis Spiral-shaped organism (Treponema pallidum)
of Rh factor in RBC.
● Blood transfusion technique was developed by James Leprosy Mycobacterium leprae
Blundell. Botulism Clostridium botulinum

Nervous System Diseases Caused by Protozoans


Ä The nerves, the brain and the spinal cord Disease Pathogen responsible
constitute the nervous sytem. Malaria Plasmodium
Ä Nervous system controls and regulates the Sleeping sickness (Trypanosomiasis) Trypanosoma brucei
activities of all the other systems of the body. Kala-azar or black sickness Leishmania donovani
Brain
Common Diseases and Affected Body Part
Brain is the main organ of the nervous system. It
consists of cerebrum, cerebellum and medulla Disease Affected Part
oblongata. AIDS Immune system of the body
Cerebrum Arthritis Joints
It controls the voluntary actions and is the seat of Asthma Bronchial muscles
intelligence.
Cataract Eyes
Cerebellum Conjunctivitis Eyes
It controls the voluntary actions and is the seat of
Glaucoma Eyes
intelligence.
Diabetes Mellitus Pancreas and blood
Medulla Oblongata
Dermatitis Skin
Lowest part of the brain and is connected with the
spinal cord. Diphtheria Throat
It controls the involuntary actions. Dementila Brain

https://sscstudy.com/
https://sscstudy.com/

30 CUET (UG) Section III : General Test

Disease Affected Part National Laboratories and Research Institutions


Eczema Skin Name Place
Goitre Thyroid Central Leather Research Institute Chennai, T.N.
Hepatitis Liver Central Mining Research Station Dhanbad, Jharkhand
Jaundice Liver Indian Institute of Science Bengaluru, Karnataka
Malaria Spleen Indian Space Research Organisation Bengaluru, Karnataka
Meningitis Brain
All India Institute of Medical Sciences New Delhi
Paralysis Nerves and limbs
Polio Legs Nuclear/Atomic Research Centre
Pyarrhoea Teeth Name of the centre Place
Pneumonia Lungs Bhabha Atomic Research Centre Trombay, near Mumbai, MS
Typhoid Intestines Saha Institute of Nuclear Physics Kolkata, Paschim Banga
Tuberculosis Lungs
Tata Institute of Fundamental Mumbai, Maharashtra
Tonsillitis Tonsils gland in throat Research

Environmental Science
Ecology Components of Ecosystem
Ä Ecology is the study of interaction between living Ecosystem has two components
organisms and their environment.
Ä The term ‘ecology’ was coined by Ernst Haeckel in I. Abiotic Component
1886. Non-living portion of an ecosystem can be divided into
three parts
Ä Odum (1971) defined ecology as ‘study of structure
and function of nature’. (i) Climate, e.g. light, wind, temperature, water etc.
(ii) Inorganic substances These includes various
Ecosystem elements, e.g. sulphur, nitrogen, calcium, phosphorus.
Ä The term ecosystem was given by A G Tansley (iii) Organic compounds, e.g. proteins, carbohydrates, lipids.
(1935).
Ä Interaction between living and non-living components
II. Biotic Component
is called ecosystem. These includes all living organisms on the basis of mode
of nutrition biotic components are of following types :
There are two types of ecosystems :
(i) Autotrophs These include all photosynthesizing
I. Natural Ecosystem plants these are self food producing members and
prepare their food in the presence of sunlight,
These are operated by themselves under natural
chlorophyll, CO2 and water.
conditions without any major interference by man. These
are of two types (ii) Heterotrophs These include organism, which cannot
prepare their own food material and they obtain food
(i) Terrestrial eg, grassland, forest, desert ecosystem, from autotrophs. These are of two types on the nature
etc. of food.
(ii) Aquatic It is further of two types : (a) Herbivores Obtain food from plants.
(a) Fresh water Which may be lotic (running water (b) Carnivores Obtain food from various herbivores.
as springs stream or river) or lentic (standing water They further are of two types :
as lake, ponds, ditch, swamp, etc). Ä Lower carnivores (obtain food from herbivores)

(b) Marine Ocean and sea. Ä Higher carnivores (obtain food from carnivores)

(iii) Decomposers These are micro-organisms include


II. Artificial Ecosystem bacteria and fungi, etc. They cause breakdown of dead
These are maintained artificially by man. It is also and decaying material into simple organic and
known as man made ecosystem. eg, crop land ecosystem, inorganic components. These are further used by the
aquarium. autotrophs.

https://sscstudy.com/
https://sscstudy.com/

GENERAL KNOWLEDGE 31

Food Chain Ä Pyramid of biomass may be upright in grassland and


Ä The process of transfer of energy from producer forest ecosystem whereas it invert in pond ecosystem.
through a series of organism to consumers is known as
food chain. III. Pyramid of Energy
eaten by eaten by
Ä Energy content or productivity of different trophic level
e.g., Grass Rabbit Hawk is shown in pyramid of energy.
(producer) (primary (secondary
consumer) consumer)
Ä Pyramid of energy is always upright in all ecosystems.
Ä In nature, there are two types of food chains are Pollution
present : Ä Odum defined the term ‘pollution’.
(i) Grazing food chain starts with green plants, Ä Undesirable change in physical, chemical and
which are eaten by herbivores, which in turn eaten biological properties of air, water and soil, which
by carnivores. directly or indirectly affect human beings, is called
(ii) Detritus food chain starts with dead organic pollution.
matter acted upon by decomposers. Ä Pollution causing agents or substances are called
Food Web pollutants.
Ä Various food chains are interconnected with each other I. Air Pollution
forming an interlocked system, which is known as Ä It is due to excess of different gases, smoke, particulate
food web. matters, chemicals, allergens, etc. Important primary
Ä Food web are very important in maintaining air pollutants are CO, SO2 , NO2 , HF, etc.
equilibrium of ecosystem. Ä Secondary air pollutants are O3 , Peroxy Acetyl Nitrate
(PAN), etc.
Differences between Food Chain and Food Web
Ä Most important gaseous air pollutant is SO2.
Food chain Food web
Ä SO 2 is produced due to combustion of fossil fuels.
It is straight single pathway It consists of number of
interconnected food chains
Ä Excess of sulphides is the cause of SO2 injury in plants.
Ä SO 2 above 1 ppm affects human beings in terms of
Members of higher trophic Members of higher trophic level
level feed upon a single type can feed as a member of irritation of eyes, respiratory tract, bronchitis asthma
of organisms of lower trophic alternative organisms of power etc.
level trophic level Ä Maximum SO2 pollution in India occurs in Kolkata.
It increases adaptability and It increases adaptability and Ä Yellowing and blacking of Taj Mahal in Agra is due to
competitiveness of the competitiveness of the organisms. SO2 released by Mathura refinery. This type of
organisms pollution is called ‘Stone cancer’.
Ä Increase in global temperature mainly due to CO2
Ecological Pyramids
concentration is called Green-house effect. The
Ä The producers and consumers can be arranged into mixture of SO2 and NO2 with water is called acid rain.
several feeding or nutritional groups, each is known as
trophic level. Ä Major air pollution disaster in India occurred in 1984
Ä Trophic structure of an ecosystem can be express by at Bhopal due to release of MIC (Methyl Isocyanate)
means of ecological pyramids. from insectiside factory of union carbide.
Ä The concept of ecological pyramid was given by II. Water Pollution
Charles Elton in 1927.
Ä It is mainly due to addition of sewage, detergents,
Each food level is also called trophic level.
domestic wastes, infectious agents, industrial wastes,
Types of Ecological Pyramids minerals, etc, in the ordinary water.
These are of three types : Ä Degree of water pollution is measure in terms of BOD
(Biological Oxygen Demand).
I. Pyramid of Number Ä If BOD is less than 1500 mg/L, it indicates less
In this, number of individuals at each trophic level is
pollution. If BOD is 1500-4000 mg/L, it indicates
shown. Pyramid of number is upright in case of
medium pollution.
grassland and pond ecosystem, intermediate in forest
ecosystem and inverted in parasitic food chain. Ä If BOD is more than 4000 mg/L, it indicates high
pollution.
II. Pyramid of Biomass Ä Mercury (Hg) is very persistent effluent.
Ä In this, the biomass of each trophic level is shown in
Ä Cadmium (Cd) accumulation in liver, kidney and
pyramid.
thyroid causes itai-itai (ouch-ouch) disease.

https://sscstudy.com/
https://sscstudy.com/

32 CUET (UG) Section III : General Test

III. Soil Pollution Ä Kanpur is the most noise polluted city in UP.
Ä Toxicity of soil increases as a result of chemical wastes, Ä Large green plants, which are planted in high noise
which has adverse effect on human beings. Excessive pollution zone are known as green mufflers as they
use of nitrogen fertilizers has increased level of have the capacity to absorb sound waves.
nitrates in soil, which is responsible for blue-baby V. Radioactive Pollution
syndrome.
Ä Chief sources are nuclear explosions.
Ä Major cause of soil pollution is disposal of plastics.
Ä Sr-90 accumulates in bone marrow and causes cancer
Ä Plastic articles are non-biodegradable, ie, they cannot
(leukaemia).
be decomposed.
Ä Cs137 accumulates in muscles and causes muscular
Ä Biological pathogens play important role in soil
pain.
pollution.
Ä Radioactive disaster was first occurred in 1945 in
IV. Noise Pollution Hiroshima and Nagasaki (Japan), when USA was
Ä Unwanted sound is called noise it is measured in term dropped nuclear bombs during Second World War.
of decibles (dB). Ä Recently in 1986, nuclear accident occurred at
Ä Generally, sound above 80 dB is termed as noise. Chernobyl (USSR).

https://sscstudy.com/
https://sscstudy.com/

GENERAL KNOWLEDGE 33

Practice Questions
1. Myopia is a disease connected with 15. Decibel is a term connected with
(a) lungs (b) brain (a) air (b) water (c) sound (d) soil
(c) ears (d) eyes
16. Which type of mirror is used in the head lights of
2. The weight of a body acts through the centre of vehicles?
(a) gravity (b) mass (a) Plane mirror (b) Concave mirror
(c) both (a) and (b) (d) buoyancy (c) Convex mirror (d) Parabolic mirror
3. A wheel-barrow is an example of 17. The height of a geo-stationary satellite from the
(a) class III lever (b) pulley system Earth’s surface is approximately
(c) class I lever (d) class II lever (a) 36000 km (b) 42000 km
4. Combustion of a candle is a/an (c) 30000 km (d) None of these
(a) photochemical reaction (b) physical change 18. The best conductor of electricity among the
(c) endothermic reaction (d) exothermic reaction following is
5. The splitting of white light into its components is (a) Copper (b) Iron
due to (c) Aluminium (d) Silver
(a) transmission (b) dispersion 19. The angle in which a cricket ball should be hit to
(c) reflection (d) refraction travel maximum horizontal distance is
6. A small drop of oil spreads over water because (a) 60° with horizontal (b) 45° with horizontal
(a) oil has a higher viscosity (c) 30° with horizontal (d) 15° with horizontal
(b) water has a higher viscosity 20. The minimum number of geostationary satellites
(c) oil has a higher surface tension needed for uninterrupted global coverage is
(d) water has a higher surface tension (a) 3 (b) 4 (c) 2 (d) 1
7. An object which absorbs all colours and reflects 21. Natural rubber is the polymer of
none appears (a) Isoprene (b) Styrene
(a) blue (b) black (c) white (d) grey (c) Butadiene (d) Ethylene
8. Radiations which is not emitted during 22. In addition to hydrogen, the other abundant
radioactivity is element present on Sun’s surface is
(a) γ-rays (b) cathode rays (a) Helium (b) Neon
(c) α-rays (d) β-rays (c) Argon (d) Oxygen
9. Magnetic keepers are used to protect magnets from 23. A nuclear reaction must be balanced in terms of
(a) self-demagnetization (a) mess and volume (b) energy and weight
(b) demagnetization due to heating (c) number of electrons (d) mass and energy
(c) Earth’s magnetic field
24. For taking the clinical photograph of bones, we ues
(d) effect of other magnets
(a) UV rays (b) IR rays
10. The quantity ‘weight’ is measured by (c) X-rays (d) Cosmic rays
(a) beam balance (b) common balance 25. A micron is equal to
(c) spring balance (d) balance wheel (a) 01
. mm (b) 0.01 mm
11. The principle due to which circular patches of light (c) 0.001 mm (d) 0.0001 mm
is seen under a tree during day time, is similar to 26. Persons suffering from myopia are advised to use
that of image formation by a (a) convex lens (b) concave lens
(a) concave lens (b) pinhole (c) plano-convex lens (d) plano-concave lens
(c) photographic camera (d) convex lens 27. Optic fibres are mainly used for which of the
12. A compass needle cannot be used to detect following?
(a) magnetic North-South direction (a) Communication (b) Weaving
(b) polarity of a magnet (c) Musical instruments (d) Food industry
(c) strength of a magnet 28. Which of the following are most penetrating?
(d) direction of magnetic field (a) α-rays (b) β-rays
13. In which medium, velocity of sound is maximum? (c) γ-rays (d) X-rays
(a) Metals (b) Air (c) Water (d) Polymer
29. Old-written material, which cannot be read easily,
14. Decibel is used to measure the intensity of can be read by
(a) magnetic field (b) sound (a) cosmic rays (b) ultraviolet rays
(c) light (d) heat (c) infrared rays (d) None of these

https://sscstudy.com/
https://sscstudy.com/

34 CUET (UG) Section III : General Test

30. Cryogenic engines find applications in 39. A bomb is dropped at a point from a moving
(a) rocket technology aeroplane. The pilot observes that
(b) frost-free refrigerators (a) the bomb traverses a curved path and falls some
(c) sub-marine propulsion distance behind that point
(d) researches in superconductivity (b) the bomb traverses a curved path and falls some
distance ahead
31. The best colour(s) for a sun umbrella will be (c) the bomb drops vertically downward
(a) black
(d) the bomb remains stationary in the air for some time
(b) black on top and white on the inside
(c) White on top and black on the inside 40. Rise of mercury in a barometer indicates
(d) printed with all the seven colours of rainbow (a) fair weather (b) storm
(c) rain (d) Both (b) and (c)
32. Which of the following laws is explained by the
statement that matter can neither be created for 41. The source of our energy is basically the
destroyed? (a) ocean (b) sun
(a) Law of Conservation of Energy (c) atmosphere (d) space
(b) Le Chatelier's principle
(c) Law of Conservation of Mass
42. When light passes from air into glass it experiences
(d) Law of Osmosis change in
(a) frequency and wavelength
33. Aviation fuel for jet aeroplanes consists of purified (b) frequency and speed
(a) petrol (b) kerosene (c) wavelength and speed
(c) gasoline (d) diesel (d) frequency, wavelength and speed
34. Which of the following is the correct combination of 43. Which of the following are the discoveries of
the inventors and the inventions? Einstein?
(a) Galileo Galilei - Transistors (a) Radioactivity and photoelectric effect
(b) Elisha Thomson -Windmill (b) Radioactivity and theory of relativity
(c) JL Baird - Television (c) Theory of relativity and photoelectric effect
(d) Sir Frank Whittle - Laser (d) X-rays and photoelectric effect
35. Who among the following laid the foundation of 44. The nuclides having the same number of neutrons
nuclear science in the country? but different number of protons or mass number
(a) Vikram Sarabhai are known as
(b) Homi J Bhabha (a) isobars (b) isoelectronic
(c) MGK Menon (c) isotopes (d) isotones
(d) Raja Ramanna
45. Why is there a spluttering sound when water is
36. Match the following sprinkled on hot oil?
Names Contribution (a) Boiling point of the water is higher than that of oil
(b) Boiling point of the oil is higher than that of water
A. SN Bose 1. Theory of numbers
(c) Oil and water do not mix
B. S Chandrasekhar 2. Elementary particles (d) Oil and water combine to form a compound
statistics
C. S Ramanujam 3. Astronomy 46. Which falls down faster in a vacuum-a feather, a
D. Aryabhatta 4. Theory of black holes wooden ball or a steel ball?
(a) A wooden ball
Codes (b) A feather
A B C D (c) A steel ball
(a) 2 1 4 3 (d) All will fall down at the same speed in a vacuum because
(b) 4 3 2 1 there will be no air resistance
(c) 2 4 1 3
(d) 3 1 4 2 47. A light year is a unit of
37. What happens to the level of mercury in the barometer (a) time
tube when it is taken down a coal mine? It (b) distance
(a) falls (b) remains unaltered (c) speed of light
(c) rises (d) rises and then falls (d) intensity of light

38. In a barometer, mercury is preferred over water 48. The technique of calculating the age of fossil
because organisms is
(a) mercury is a good conductor of heat (a) Radiocarbon dating
(b) mercury shines and therefore its level can be read easily (b) Electroporation
(c) mercury has high density and low vapour pressure (c) Counting the annual rings
(d) mercury is available in pure form (d) Micro manipulation

https://sscstudy.com/
https://sscstudy.com/

GENERAL KNOWLEDGE 35

49. A dynamo is used to convert (c) this can damage the appliances
(a) mechanical energy into electrical energy (d) this can damage the domestic wiring due to overheating
(b) electrical energy into mechanical energy
(c) electrical energy into magnetic energy 58. A ship rises as it enters the sea from a river
(d) magnetic energy into mechanical energy because the
(a) water in sea is hard water
50. Shaving mirror is (b) larger quantity of water in the sea pushes the ship
(a) convex (b) concave upwards
(c) plane (d) parabolic (c) density of sea water is lower than that of the river water
51. Transfer of heat energy from the sun to the moon (d) density of sea water is higher than that of the river
takes place by water
(a) radiation only 59. Consider the following statements
(b) radiation and conduction
The fog during winter season is more on the day
(c) radiation and convection
when
(d) radiation, conduction and convection
1. sky is clear 2. sky is cloudy
52. Match the following 3. wind is blowing
List I List II Which of the statements given above is/are correct?
(a) Only 1 (b) 1 and 3
A. Cusec 1. Pressure
(c) 2 and 3 (d) Only 2
B. Byte 2. Intensity of earthquakes
C. Richter 3. Rate of flow 60. Which of the following is good nuclear fuel ?
(a) Uranium-238 (b) Neptunium-239
D. Bar 4. Computer memory
(c) Thorium-236 (d) Plutonium-239
Codes
A B C D 61. In an atomic explosion, enormous energy is
(a) 2 4 3 1 released which is due to the
(b) 3 4 2 1 (a) conversion of neutrons into protons
(c) 3 2 1 4 (b) conversion of chemical energy into heat energy
(d) 4 1 2 3 (c) conversion of chemical energy into nuclear energy
53. Microphone is a device in which (d) conversion of mass into energy
(a) sound waves are directly transmitted 62. Which of the following is used as a moderator in
(b) electrical energy is converted into sound waves directly nuclear reactor?
(c) sound waves are converted into electrical energy and then (a) Ordinary water (b) Radium
reconverted into sound after transmission (c) Thorium (d) Graphite
(d) amplification is not required at all
63. Electron was first identified by
54. The blue colour of the water in the sea is due to (a) JJ Thomson (b) Daniel Rutherford
(a) absorption of other colours by water molecules (c) J Kepler (d) James Chadwick
(b) reflection of the blue light by the impurities in sea water
(c) reflection of blue sky by sea water and scattering of blue 64. Kinetic energy depends
light by water molecules (a) on the mass of the moving body
(d) None of the above (b) the velocity or speed of the moving body
(c) the pressure of the moving body
55. A lighted candle gets extinguished when covered with (d) both mass and velocity of the moving body
a tumbler because of
(a) adequate supply of air 65. Match the following
(b) inadequate supply of air Device Conversion
(c) presence of non-luminous matter
A. Battery charger 1. Electrical energy into
(d) None of the above mechanical energy
56. A weather balloon is not fully inflated on the ground B. Battery 2. Mechanical energy into
because electrical energy
(a) it cannot rise up in the air when fully inflated C. Washing machine 3. Chemical energy into
(b) it cannot withstand the outside pressure if fully inflated electrical energy
(c) if the balloon is fully inflated, it may not remain stable D. Dynamo 4. Electrical energy into
in a storm chemical energy
(d) the air inside the balloon expands as it rises and may
burst Codes
A B C D
57. One should not connect a number of electric (a) 4 3 1 2
appliances to the same power socket because (b) 4 3 2 1
(a) the appliances will not get full voltage (c) 1 2 3 4
(b) the appliances will not get proper current (d) 3 4 1 2

https://sscstudy.com/
https://sscstudy.com/

36 CUET (UG) Section III : General Test

66. If you swim one km, how many miles do you swim? 79. Nylon is made up of
(a) 0.5 (b) 0.62 (c) 0.84 (d) 1.6 (a) Polyethylene (b) Polypropylene
(c) Polyamide (d) Polyester
67. The temperature of a body is an indicator of
(a) the total energy of the molecules of the body 80. Rectified spirit is
(b) the average energy of the molecules of the body (a) methylated spirit (b) tincture iodine
(c) the total velocity of the molecules of the body (c) 95% ethyl alcohol (d) 100% ethyl alcohol
(d) the average kinetic energy of the molecules of the body
81. Cosmetic powders are prepared from
68. Why are two blankets warmer than one? (a) asbestos (b) talc
(a) Two blankets have more wool and hence, provide (c) gypsum (d) serpentine
greater warmth
(b) Two blankets enclose air which does not allow the cold to 82. The unit of radioactivity is
penetrate (a) angstrom (b) candela
(c) Two blankets compress the air in between the body and (c) fermi (d) curie
the blankets and this compression produces that 83. The gas used for artificial ripening of green fruit is
(d) None of the above (a) acetylene (b) ethylene
69. Tiredness is felt on the top of the mountains (c) ethane (d) carbon dioxide
because of
(a) high temperature 84. Glass is
(a) super cooled liquid (b) crystalline solid
(b) low temperature
(c) liquid crystal (d) None of the above
(c) high pressure outside the body
(d) low pressure outside the body 85. The percentage of lead in lead pencil is
(a) 0 (b) 100 (c) 77 (d) 65
70. Oil rises up the wick in a lamp because
(a) oil is very light 86. An electric iron has a heating element made of
(b) of the diffusion of oil through the wick (a) Copper (b) Tungsten
(c) of the surface tension phenomenon (c) Nichrome (d) Zinc
(d) of the capillary action phenomenon
87. The process of covering water pipes made of iron
71. Radioactivity was discovered by
with zinc layer to prevent corrosion is called
(a) Rutherford (b) Henry Becquerel
(a) Zinc plating (b) Alloy formation
(c) Roentgen (d) Einstein
(c) Vulcanization (d) Galvanization
72. Aquaregia is a mixture of the following in the ratio
1 : 3 by volume 88. The particle required to continue the chain process
(a) conc. HNO3 and conc. HCI
of Uranium fission is
(a) Electron (b) Proton
(b) conc. HNO3 and conc. H2SO4
(c) Neutron (d) Positron
(c) dil, HCI and dil. HNO3
(d) conc. HCI and conc. HNO3 89. Which of the following is used to increase the
anti-knocking property of petrol?
73. In deep-sea diving, divers use a mixture of gases
(a) Tetramethyl lead (b) Tetraethyl lead
consisting of oxygen and
(a) hydrogen (b) nitrogen (c) Trimethyl lead (d) Triethyl lead
(c) argon (d) helium 90. The chief source for the production of nitrogeneous
74. 'Atomic theory' of matter was given by fertilizers is
(a) Avogadro (b) Dalton (a) Ammonia (b) Nitric acid
(c) Newton (d) Pascal (c) Nitrogen (d) Nitrogen dioxide

75. Which among the following is a covalent 91. Which of the following is the major constituent of LPG?
compound? (a) Methane (b) Ethane
(a) Calcium chloride (b) Magnesium fluoride (c) Propane (d) Butane
(c) Sodium chloride (d) Carbon tetrachloride 92. Which one of the following elements is used in the
76. Radioactive materials should be kept in the manufacture of fertilizers?
container made of (a) Fluorine (b) Potassium
(a) Pb (b) Steel (c) Fe (d) Al (c) Lead (d) Aluminium

77. Sweat contains water and ..... . 93. Which atom is not found in proteins?
(a) salt (b) vinegar (a) Cartbon (b) Nitrogen
(c) citric acid (d) lactic acid (c) Phosphorus (d) Sulphur

78. Brass is an alloy of 94. The chemical used in a photographic plate is


(a) Cu and Zn (b) Al and Mg (a) Barium chloride (b) Silver bromide
(c) Cu, Al and Zn (d) Cu and Sn (c) Silver nitrate (d) Silver chloride

https://sscstudy.com/
https://sscstudy.com/

GENERAL KNOWLEDGE 37

95. The element used for making solar cells is 109. Which one of the following fertilizers has more
(a) Magnesium (b) Sodium nitrogen content?
(c) Calcium (d) Silicon (a) Urea (b) Ammonium Nitrate
(c) Potassium Nitrate (d) Ammonium Phosphate
96. Which of the following is used for wrapping of
fractured bones? 110. Dialysis is used for people with defective kidneys. It
(a) White cement (b) White lead involves the process of
(c) Zinc oxide (d) Plaster of Paris (a) adsorption (b) osmosis
(c) electropheresis (d) diffusion
97. Carbon, diamond and graphite are together called
(a) isomers (b) allotropes 111. The substance coated on plastic tape-recorder tapes
(c) isomorphs (d) isotopes is
(a) zinc oxide (b) manganse oxide
98. Which of the following metals was first discovered (c) iron sulphate (d) iron oxide
by man?
(a) Bronze (b) Silver 112. Match the following
(c) Iron (d) Copper
List I List II
99. An element that does not occur in nature but can be
A. Best conductor of heat 1. Gold
produced artificially is and electricity
(a) thorium (b) radium
(c) plutonium (d) uranium B. Most abundant metal 2. Lead
C. Most malleable metal 3. Aluminium
100. The element found on the surface of the moon is
(a) tin (b) tungsten D. Poorest conductor of 4. Silver
(c) tantalum (d) titanium heat

101. The average salinity of sea water is Codes


A B C D
(a) 20% (b) 25% (a) 1 3 2 4
(c) 30% (d) 35% (b) 2 3 4 1
(c) 3 2 4 1
102. The metallic constituents of hard water are (d) 4 3 1 2
(a) magnesium, calcium and tin
(b) iron, tin and calcium 113. Water is a compound because
(c) calcium, magnesium and iron (a) it exists as a solid, a liquid or a gas
(d) magnesium, tin and iron (b) it contains hydrogen and oxygen
103. The high-temperature superconductors are (c) it contains two different elements joined by chemical
(a) metel alloys bonds
(b) ceramic oxides (d) it cannot be split up into simpler substances by chemical
(c) inorganic polymers means
(d) pure rare earth metals 114. Of the following metals which one used to pollute
104. The variety of coal in which the deposit contains the air of a big city?
recognisable traces of the original plant material is (a) Copper (b) Chromium
(a) anthracite (b) lignite (c) Lead (d) Cadmium
(c) peat (d) bitumen
115. The element present in the largest amount in rocks
105. The highest grade of coal whose surface is shining and minerals is
and has the highest calorific value, is (a) carbon (b) silicon
(a) Bituminous (b) Lignite (c) hydrogen (d) aluminium
(c) Anthracite (d) Peat
116. Which of the following is the lightest metal?
106. What are soaps? (a) Mercury (b) Silver
(a) Salts of silicates (c) Lithium (d) Lead
(b) Esters of heavy fatty acids 117. The most malleable metal is
(c) Sodium or potassium salts of heavier fatty acids (a) Silver (b) Gold
(d) Mixture of glycerol and alcohols (c) Aluminium (d) Sodium
107. In vulcanisation, natural rubber is heated with 118. Balloons are filled with
(a) Carbon (b) Sulphur (a) helium (b) oxygen
(c) Silicon (d) Phosphorus (c) nitrogen (d) argon

108. Which of the following is used as a filler in rubber 119. Which of the following is used in beauty parlours
tyres? for hair setting?
(a) Graphite (b) Coal (a) Chlorine (b) Sulphur
(c) Coke (d) Carbon black (c) Phosphorus (d) Silicon

https://sscstudy.com/
https://sscstudy.com/

38 CUET (UG) Section III : General Test

120. Which of the following is the lightest gas? (b) lack of oxygen in the surroundings of sigrees
(a) Nitrogen (b) Hydrogen (c) accumulation of smoke and dust over the sigrees
(c) Ammonia (d) Carbon dioxide (d) interrupted release of moisture from the material
121. Match the following 132. de Vries proposed
Alloy Composition (a) Theory of Speciation
(b) Theory of Crossing Over
A. Bronze 1. Lead, antimony, tin (c) Theory of Natural Selection
B. Brass 2. Copper, zinc, nickel (d) Theory of Mutation
C. German silver 3. Copper, zinc
133. Haemoglobin in the blood is a complex protein rich
D. Type metal 4. Copper, tin
in
Codes (a) iron (b) silver
A B C D (c) copper (d) gold
(a) 2 1 4 3
(b) 2 1 3 4 134. Vitamin-C is also known as
(c) 3 2 1 4 (a) amino acid (b) ascorbic acid
(d) 4 3 2 1 (c) acetic acid (d) azelaic acid

122. Among the various allotropes of carbon 135. Which of the following is not a stem modification?
(a) diamond is the hardest, lamp black is the softest (a) Bulb of onion (b) Corm of arvi
(b) diamond is the hardest, coke is the softest (c) Tuber of sweet-potato (d) Tuber of potato
(c) coke is the hardest, graphite is the softest
(d) diamond is the hardest, graphite is the softest 136. Clitellum is found in
(a) leech (b) snail
123. Which one of the following is the basis of modern (c) nereis (d) earthworm
periodic table?
(a) Atomic mass (b) Atomic number 137. Plant genetic material in ‘Gene-Bank’ is preserved
(c) Atomic size (d) Atomic volume at- 196°C in liquid nitrogen as
(a) pre-mature seed (high moisture)
124. Which is/are the important raw material(s) (b) ripe fruit
required in cement industry? (c) seedling and meristem
(a) Limestone (b) Gypsum and clay (d) mature seed (dry)
(c) Clay (d) Limestone and clay
138. Which is not an anasthetic agent in surgical
125. Glass is made of the mixture of operations?
(a) quartz and mica (b) sand and salt (a) Nitrous oxide (b) Acetone
(c) sand and silicates (d) None of these (c) Chloroform (d) Ether
126. Which one of the following metals has the highest 139. Insulin
density? (a) constricts blood vessels (b) stimulates lactation
(a) Gold (b) Iron
(c) increases blood sugar (d) decreases blood sugar
(c) Platinum (d) Lead
140. Animals having open circulatory system possesses
127. The quantum theory was first enunciated by (a) pseudocoel (b) haemocoel
(a) Niel Bohr (b) Albert Einstein
(c) coelenteron (d) spongocoel
(c) Max Planck (d) Max Born
128. The noble gases do not react with any other atom 141. Flowers emit fragrance to
because (a) purify air (b) drive away flies
(a) they are heavy elements (c) attract insects (d) All of the above
(b) they are stable compounds
(c) their outermost orbits contain eight electrons 142. Brine refers to
(d) None of the above (a) pure water (b) starch water
(c) salt water (d) sweet water
129. Washing soda is the common name for
(a) Calcium carbonate (b) Calcium bicarbonate 143. Haemoglobin contains the metal
(c) Sodium carbonate (d) Sodium bicarbonate (a) Copper (b) Molybdenum
(c) Iron (d) Magnesium
130. Which of the following is the basic characteristic of
a living organism? 144. Which of the following prevents bleeding of gums?
(a) Ability to breathe (b) Ability to reproduce (a) Nicotinamide (b) Thiamine
(c) Ability to move (d) Ability to eat (c) Ascorbic acid (d) Vitamin B
131. The open ‘Sigrees’ or coal stoves often require 145. The locomotion of insects to the source of light is
fanning to sustain burning because of called
(a) tendency of carbon dioxide to form a layer along with (a) phototactic (b) thermotactic
dust smoke (c) phototropic (d) hydrotropic

https://sscstudy.com/
https://sscstudy.com/

GENERAL KNOWLEDGE 39

146. Rabies is a 162. Water from soil enters into the root hairs owing to
(a) helminthic disease (b) viral disease (a) Atmospheric pressure (b) Capillary pressure
(c) bacterial disease (d) protozoan disease (c) Root pressure (d) Osmotic pressure
147. Tuberculosis is transmitted through 163. The vitamin necessary for coagulation of blood is
(a) droplet transmission (b) blood transfusion (a) Vitamin B (b) Vitamin C
(c) contaminated water (d) sexual contact (c) Vitamin K (d) Vitamin E
148. Antibodies are mainly synthesised from 164. The average life span of red blood corpuscles is
(a) megakaryocyte (b) monocyte about
(c) lymphocyte (d) histiocyte (a) 100-200 days (b) 100-120 days
(c) 160-180 days (d) 150-200 days
149. Which of the following is a parthenocarpic fruit?
(a) Banana (b) Apple 165. Dormancy period of animals during winter season
(c) Mulberry (d) Strawberry is called
(a) Aestivation (b) Regeneration
150. Viruses are (c) Hibernation (d) Mutation
(a) cellular (b) acellular
(c) unicellular (d) multicellular 166. Minamata disease was caused by
(a) Mercury (b) Lead
151. Too much consumption of tea or coffee can result in (c) Cadmium (d) Zinc
deficiency of
(a) vitamin B12 (b) calcium 167. The first effective vaccine against polio was
(c) vitamin C (d) albumin prepared by
(a) JH Gibbon (b) Jonas E Salk
152. Penicillin was discovered by
(c) Robert Edwards (d) James Simpson
(a) Charles Goodyear (b) Michael Faraday
(c) Six Alexander Fleming (d) William Harvey 168. Plague is transmitted by
(a) House fly (b) Tse-tse fly
153. Insulin was discovered by
(c) rat flea (d) mosquito
(a) F Banting (b) Edward Jenner
(c) Ronald Ross (d) SA Wakesman 169. Itai-itai disease is due to poisoning caused by
(a) mercury (b) arsenic
154. The radioactive Strontium-90 causes
(a) Brain cancer (b) Skin cancer (c) cadmium (d) asbestos
(c) Lung cancer (d) Bone cancer 170. Blue-green algae are included in the group
155. Which of the following is a good source of vitamin (a) eubacteria (b) cyanobacteria
‘E’? (c) protozoa (d) fungi
(a) Meat (b) Ghee 171. Which vitamin is provided by sunlight to the body?
(c) Yellow Yolk (d) Fresh Vegetables (a) Vitamin A (b) Vitamin B
156. Penicillin is isolated from (c) Vitamin C (d) Vitamin D
(a) Fungi (b) Algae 172. The vitamin which is very liable and easily
(c) Virus (d) Bacteria destroyed during cooking as well as storage is
157. The total number of bones in the human body is (a) Vitamin A (b) Vitamin B 6
(a) 206 (b) 260 (c) Vitamin C (d) Vitamin K
(c) 306 (d) 360
173. The compound used in anti-malarial drug is
158. Bile is secreted by (a) aspirin (b) neoprene
(a) Gall bladder (b) Liver (c) isoprene (d) chloroquin
(c) Bile duct (d) Pancreas
174. Which of the following tests helps in diagnosis of
159. Name the vitamin not found in any animal food. cancer?
(a) Vitamin BI2 (b) Vitamin C (a) X-ray (b) Urine test
(c) Vitamin D (d) Vitamin K (c) Blood test (d) Biopsy test

160. The largest organ of human body is 175. Which of the following vitamins is synthesised in
(a) Heart (b) Brain the body by intestinal bacteria?
(c) Liver (d) Kidney (a) Vitamin B1 (b) Vitamin B 4
(c) Vitamin D (d) Vitamin K
161. Which one of the following is known as the
‘immovable property’ in the cell? 176. Which acid is produced when milk gets sour?
(a) Carbohydrate (b) Fat (a) Acetic acid (b) Tartaric acid
(c) Protein (d) Nucleic acid (c) Lactic acid (d) Butyric acid

https://sscstudy.com/
https://sscstudy.com/

40 CUET (UG) Section III : General Test

177. Which of the following vitamins can be synthesised 186. A person who lives exclusively on milk, egg and
by our body? bread is likey to become a victim of
1. Vitamin B6 2. Vitamin D (a) rickets (b) scurvy
3. Vitamin E 4. Vitamin K (c) xerophthalmia (d) None of these
(a) 1 and 2 (b) 2 and 3
187. Match the following
(c) 2 and 4 (d) 1 and 4
List I List II
178. The most important requirement for the people of a
A. Adrenal gland 1. Below the right portion
flood hit area is of the liver
(a) small pox vaccination (b) BCG injection
B. Pituitary gland 2. By the side of the
(c) inoculation against cholera (d) None of these
kidneys
179. Match the following C. Islets of Langerhans 3. At the base of the brain
List I List II D. Gall bladder 4. Part of the pancreas
A. BCG Vaccine 1. Malaria
Codes
B. BPL Vaccine 2. Sore throat A B C D
C. Chloroquin 3. Tuberculosis (a) 3 2 4 1
D. Penicillin 4. Rabies (b) 2 3 4 1
(c) 4 1 2 3
Codes (d) 1 2 4 3
A B C D
(a) 3 4 1 2
188. A girl ate sweets while fanning the flies away. Due
(b) 3 4 2 1 to this, she suffered from a disease diagnosed as
(c) 4 3 1 2 (a) kwashiorkar (b) tuberculosis
(d) 4 3 2 1 (c) diphtheria (d) cholera

180. Which of the following are correctly matched? 189. Calcium content is maximum in
1. Tse tse fly-Sleeping sickness (a) bajra (b) maize
2. Female-anopheles mosquito-Malaria (c) sorghum (d) wheat
3. Sand fly-Kala azar 190. Which of the following has been found useful in
(a) 1 and 2 (b) 2 and 3 keeping cholesterol level down?
(c) 1 and 3 (d) 1, 2 and 3 (a) Tulsi (b) Turmeric
(c) Garlic (d) Serpentina
181. During sleep a man's blood pressure
(a) increases (b) decreases 191. Acid rain is formed due to contribution from the
(c) remains constant (d) fluctuates following pair of gases
(a) Methane and Ozone
182. Sugar syrup is used for the preservation of fruits
(b) Oxygen and Nitrous oxide
because it
(a) kills the microorganisms present in them (c) Methane and Sulphur dioxide
(b) helps to improve their taste and flavour (d) Nitrogen oxides and Sulphur dioxide
(c) drains moisture to inhibit growth of microorganisms 192. Most serious environmental effect posed by
(d) mixes well with the fruits hazardous waste is
(a) air pollution
183. Bionics is the science of
(b) contamination of ground water
1. application of mathematics to the study of living
(c) increased use of land for landfils
things
2. application of the knowledge of the living world (d) destruction of habitat
characteristics to the world of machines 193. Which of the following is not a primary contribute
3. study of relation of an organism to its environment to the greenhouse effect?
4. study of laws of life
(a) Carbon dioxide (b) Carbon mono oxide
(a) 1 and 2 (b) Only 2 (c) Only 3 (d) 1 and 4 (c) Argon (d) Methane gas
184. Salt is used in our food because 194. The increase in the concentration of CO2 in our
(a) it makes the food tasty
(b) we sweat and lose salt environment in last fifty years, since 1960 is about
(c) it helps in production of hydrochloric acid required to (a) 20% (b) 10% (c) 14% (d) 6%
digest the food 195. How the biological oxygen demand gets affected with
(d) it makes water absorb food particles better
the increased presence of organic matter in water?
185. The vitamin which is generally excreted by humans (a) The oxygen demand increases
in urine is (b) The oxygen demand decreases
(a) vitamin A (b) vitamin D (c) The oxygen demand remain unchanged
(c) vitamin C (d) None of these (d) None of the above

https://sscstudy.com/
https://sscstudy.com/

GENERAL KNOWLEDGE 41

196. The headquarters of International Whaling 207. Which one of the following is not a site for in-site
Commission is located at method of conservation of flora?
(a) New York (b) Kyoto (a) Biosphere Reserve (b) Botanical Garden
(c) Perth (d) Cambridge (c) National Park (d) Wildlife Sanctuary

197. Increased defoliation in plants is caused by 208. Consider the following


(a) ozone depletion (b) acid rains 1. Photosynthesis
(c) global warming (d) ground pollution 2. Respiration
198. National Biodiversity Authority is located at 3. Decay of organic matter
(a) Shimla (b) Hyderabad 4. Volcanic action
(c) Chennai (d) Kolkata Which of the above add carbon dioxide to the carbon
cycle on the Earth?
199. According to World Health Organisation which is (a) 1 and 4 (b) 2 and 3
the most polluted city in the world? (c) 2, 3 and 4 (d) All of these
(a) Los Angeles, California
(b) Mexico City, Mexico 209. A sandy and saline area in the natural habitat of
(c) New Delhi, India an Indian animal species. The animal has no
(d) Shanghai, China predators in that area but it’s existen is threatened
due to the destruction of it’s habitat. Which one of
200. The best way to dispose plant waste is
(a) burning (b) composting
the following could be that animal?
(c) dumping (d) incineration (a) Indian Wild Buffalo (b) Indian Wild Ass
(c) Indian Wild Boar (d) Indian Gazelle
201. Which among the following greenhouse gas has the
210. The Himalayan Range is very rich in species
highest radiative forces?
(a) Carbon dioxide (b) Methane
diversity. Which one among the following is the
(c) Chlorofluoro carbons (d) Ozone most appropriate reason for this phenomena?
(a) It has a high rainfall that supports luxuriant vegetation
202. Milankovitch cycles (b) It is a confluence of different bio geographical zones
(a) refer to shifts in the temperature of surface water in the (c) Exotic and invasive species have not been it introduced
middle latitudes of Pacific Ocean in this region
(b) are changes in Earth’s rotation and orbit around the (d) It has less human interference
Sun that may trigger climate variation
(c) describe the timing of the Northern lights in the 211. Consider the following
thermosphere 1. Carbon dioxide 2. Oxides of Nitrogen
(d) describe upwelling and downwelling in the ocean 3. Oxides of Sulphur
Which of the above is/are the emission(s) from coal
203. Keeling’s reports from Mauna Loa demonstrated combustion zone at thermal power stations?
(a) an increase in Carbon dioxide from the 1950 to present (a) Only 1 (b) 2 and 3
(b) that sediments deposited on the seafloor can yield dues (c) 1 and 3 (d) 2 and 3
about past climates 212. The formation of ozone hole in the Antarctica
(c) Carbon dioxide gradient decreases from urban to rural
region has been a cause of concern, what could be
(d) the presence of El-Nino
the reason for the formation of this hole?
204. Kyoto is to carbon dioxide as montreal is to (a) Presence of prominent tropospheric turbulence and
(a) living modified organisms inflow of chlorofluoro carbons
(b) persistent organic pollutants (b) Presence of prominent polar front and stratospheric
(c) chlorofluoro carbons clouds and inflow of chlorofluoro carbons.
(d) ozone (c) Absence of polar front and stratospheric clouds and
inflow of methane and chloro flouro carbons
205. Which of the following terms best applies to the (d) increased temperature at polar region due to global
practice of environ- mental science? warming
(a) Highly specialised (b) Integrative
(c) Abstract (d) Theoretical 213. Regarding carbon credits, which one of the
following statements is not correct?
206. Salinisation occurs when the irrigation water (a) The carbon credit system was ratified in conjunction
accumulate in the soil evaporates, leaving behind with the Kyoto protocol
salts and minerals. What are the effects of (b) Carbon credits are awarded to countries or group that
Salinisation on the irrigated land? have reduced green house gases below their emission quota.
(a) It greatly increases the crop production (c) The goal of the carbon credit system is to limit the
(b) It makes some soils impermeable increase of carbon dioxide emission.
(c) It raises the water table (d) Carbon credits are traded at a price fined from time to
(d) It fills the air spaces in the soil time by United Nations Environment Programme

https://sscstudy.com/
https://sscstudy.com/

42 CUET (UG) Section III : General Test

214. Mon-863 is a variety of maize. It was in the news (d) Ministry of Rural Development
from which one of the following reasons? 219. Which one of the following is also known as Top slip?
(a) It is genetically modified dwarf variety, which is (a) Simlipal National Park
resistant to drought (b) Periyar Wildlife Sanctuary
(b) It is genetically modified variety, which is pest resistant (c) Manjira Wildlife Sanctuary
(c) It is genetically modified variety with ten times higher (d) Indira Gandhi Wildlife Sanctuary and National Park
protein content than regular maize crop
(d) It is a genetically modified variety used exclusively for 220. Which one of the following is located in Bastar
bio-fuel production region?
(a) Bandhavgarh National Park
215. Where was the world summit on sustainable (b) Randeli Sanctuary
development (Rio-10) held? (c) Rajaji National Park
(a) Davos (b) Nova Seotia (d) Indravati National Park
(c) Johannesburg (d) Shanghai
221. Which one of the following country is the first
216. Out of all the Biosphere reserves in India, four country in the world to propose a carbon tax for its
have been recognised on the world network by people to address global warming?
UNESCO. Which one of the following is not one of (a) Australia (b) Germany
them? (c) Japan (d) New Zealand
(a) Gulf of Mannar (b) Kangchenjunga
(c) Nanda Devi (d) Sunderbans 222. Amongst the following Indian states, which one has
minimum total forest cover?
217. Which one among the following has the maximum (a) Sikkim (b) Goa
numbers of National Park? (c) Haryana (d) Kerala
(a) Andaman and Nicobar Islands
(b) Arunachal Pradesh 223. Genetic engineering approval commission, whose
(c) Asom permission is required for cultivation of any
(d) Meghalaya genetically modified crop such as Bt-Cotton in
India, is under Union Ministry of
218. Which one of the following Union Ministries is (a) Agriculture
implementing the biodiesel mission (as nodal (b) Environment and forest
ministry)? (c) Commerce and industry
(a) Ministry of Agriculture (d) Rural development
(b) Ministry of Science and Technology
(c) Ministry of New and Renewable Energy

ANSWERS
1. (d) 2. (a) 3. (d) 4. (d) 5. (b) 6. (d) 7. (b) 8. (b) 9. (a) 10. (c)
11. (b) 12. (c) 13. (a) 14. (b) 15. (c) 16. (b) 17. (a) 18. (d) 19. (b) 20. (c)
21. (a) 22. (a) 23. (d) 24. (c) 25. (c) 26. (b) 27. (a) 28. (c) 29. (c) 30. (a)
31. (c) 32. (c) 33. (b) 34. (c) 35. (b) 36. (c) 37. (c) 38. (c) 39. (b) 40. (a)
41. (b) 42. (c) 43. (c) 44. (d) 45. (b) 46. (d) 47. (b) 48. (a) 49. (a) 50. (b)
51. (a) 52. (b) 53. (c) 54. (c) 55. (b) 56. (d) 57. (d) 58. (d) 59. (a) 60. (d)
61. (d) 62. (d) 63. (a) 64. (d) 65. (a) 66. (b) 67. (d) 68. (b) 69. (d) 70. (d)
71. (b) 72. (a) 73. (d) 74. (b) 75. (d) 76. (a) 77. (a) 78. (a) 79. (c) 80. (c)
81. (b) 82. (d) 83. (b) 84. (a) 85. (a) 86. (c) 87. (d) 88. (c) 89. (b) 90. (a)
91. (d) 92. (b) 93. (c) 94. (b) 95. (b) 96. (d) 97. (b) 98. (d) 99. (c) 100. (d)
101. (d) 102. (c) 103. (b) 104. (c) 105. (c) 106. (c) 107. (b) 108. (d) 109. (a) 110. (b)
111. (d) 112. (d) 113. (c) 114. (c) 115. (b) 116. (c) 117. (b) 118. (a) 119. (b) 120. (b)
121. (d) 122. (d) 123. (b) 124. (d) 125. (c) 126. (c) 127. (c) 128. (c) 129. (c) 130. (c)
131. (a) 132. (d) 133. (a) 134. (b) 135. (c) 136. (d) 137. (d) 138. (b) 139. (d) 140. (b)
141. (c) 142. (c) 143. (c) 144. (c) 145. (a) 146. (b) 147. (a) 148. (c) 149. (a) 150. (b)
151. (a) 152. (c) 153. (a) 154. (d) 155. (d) 156. (a) 157. (a) 158. (a) 159. (c) 160. (c)
161. (a) 162. (b) 163. (c) 164. (b) 165. (c) 166. (a) 167. (b) 168. (c) 169. (c) 170. (b)
171. (d) 172. (c) 173. (d) 174. (d) 175. (d) 176. (c) 177. (c) 178. (c) 179. (a) 180. (d)
181. (d) 182. (c) 183. (a) 184. (c) 185. (c) 186. (b) 187. (b) 188. (d) 189. (b) 190. (c)
191. (d) 192. (b) 193. (c) 194. (c) 195. (a) 196. (d) 197. (a) 198. (c) 199. (b) 200. (b)
201. (b) 202. (b) 203. (b) 204. (c) 205. (b) 206. (d) 207. (b) 208. (c) 209. (b) 210. (b)
211. (d) 212. (b) 213. (d) 214. (b) 215. (c) 216. (b) 217. (a) 218. (d) 219. (d) 220. (d)
221. (d) 222. (c) 223. (b) 228. () 229. () 230. ()

https://sscstudy.com/
https://sscstudy.com/

GENERAL KNOWLEDGE 43

CHAPTER 03

Computer Awareness
The word ‘computer’ has been originated from the Latin Control Unit
word ‘compute’ which means to calculate. Computer is It gives command to transfer data from the input devices
an electronic device which receives data, processes it to memory and from memory to arithmetic logic unit. It
and gives meaningful result. To know the work of transfers the results from memory to output unit.
computer, some important terms are as follows
Ä Data The word data has been derived from Latin
Memory
word ‘datum’ means fact. It is a collection of facts and It temporarily/permanently holds the data received from
figures which are not in directly usable form. input device. It holds the data being processed and
Ä Processing This is an act of performing some actions
intermediate results being generated. It holds the system
software and the application software in use.
to convert it into usable form.
Ä Information It is processed data or final result which
Memory is mainly classified into two types
is in directly usable form and conveys some (a) Random Access Memory (RAM) All active
meaningful results. programs and data are stored in RAM so that
Ä Instruction It is a type of action to be taken on data.
they may readily available and easily accessed by
the CPU.
Ä Basic Operations Input, storing, processing, output,
(b) Read Only Memory (ROM) The data stored on
control. ROM cannot be changed. It is non-volatile which
means that it can keep its contents even without a
Components of a Computer power source.
Input Unit
Output Unit
Ä It accepts data and instructions from the user.
Ä It accepts processed data from CPU.
Ä It converts the accepted instructions into the machine
language.
Ä It converts results from machine language into simple
language.
Ä It gives converted instructions to CPU for processing.
Ä It displays results to user.
Central Processing Unit (CPU)
Ä It performs all processing work of computer. Software
Ä It is also known as brain of computer.
It is a set of instructions that directs the computer
to process information. It can be classified into three
Ä It controls all the parts of computer system.
types.
Central Processing Unit (CPU) has three components
Arithmetic and Logic Unit
System Software
It coordinates the operation of the various hardware
It performs all arithmetic and logic operations. It
components of the computer.
consists of number of small locations, termed as
registers. It has two parts e.g. DOS, Windows 95/98/2000/ME and UNIX.
(a) Arithmetic Section All complex arithmetic Application Software
expressions are performed by repetitive
combination of these operations. It is a set of program designed for specific uses or
applications
(b) Logic Section It performs all logic operations like
comparing, selecting and merging of data. e.g. MS Word, MS Excel, etc.

https://sscstudy.com/
https://sscstudy.com/

44 CUET (UG) Section III : General Test

Utilities Software Year Inventor Invention


Utility programs are those that are very often requested 1642 Blaise Pascal First Calculating Machine
by many applications programs. It is used to debug the 1671 Gottfried Von Leibnitz Calculator (Modified Pascal’s
software errors, correct the data through the editor, sort machine)
data, etc, e.g. Pkzip, Antivirus software, etc. 1801 Joseph Jacquard Card of holes for weaving
patterns
Bits, Bytes and Words 1823-34 Charles Babbage Difference Engine and
Analytical Engine
Unit Size Description
1880 Herman Hollerith Tabulating machine using
Bit One binary digit Stores either a binary 0 or 1
punch cards
Byte Eight bits One character Lady Ada Lovelace Binary Number System
Word 16 to 64 bits One character 1930 Howard Aitken and MARK-I
Kilobyte (KB) 1 thousand bytes About one page of Grace Hopper (IBM)
double-spaced text 1937-38 Dr John Vircent ABC
Atanassoff (Atanasoff-Berry-Computer)
Megabyte (MB) 1 million bytes About the size of a short book
1946 JP Eckert and JW ENIAC (Electronic Numerical
Gigabyte (GB) 1 billion bytes 1000 short books
Mauchly Integrated And Calculator)
Terabyte (TB) 1 trillion bytes An entire library 1940s H Goldshine, AW Stored Program Concept
Petabyte (PB) 1 quadrillion bytes Just about all the libraries in Burks and John Von
the US Neumann
1947-49 John Von Neumann EDSAC (Electronic Delayed
Historical Overview of Computer Storage Automatic Computer)
Year Inventor Invention 1950 Moor School in USA EDVAC (Electronic Discrete
16th China Abacus Variable Automatic Computer)
Century 1951 Eckert and JW UNIVAC-I (Universal Automatic
1617 John Napier Napier’s Bones Mauchly Computer)

Computer Generations
Operating
Generation Year Switching Device Storage Device Speed Language Application
System
First 1949-54 Vacuum Tubes/ Acoustic delay lines and 333 Mainly Machine and Mostly scientific later simple
Electronic Valves later magnetic drum, micro batch assembly business systems
1KB memory, seconds languages, Simple
monitors
Second 1954-64 Transistors Magnetic core, main 10 micro Multi-bag High level Extensive business
memory, tape and disk seconds remaining, languages, applications, Engineering
peripheral memory, 100 Time Fortran, Cobol, design optimisation
KB main memory sharing Algol, Batch, scientific research
Operating system.
Third 1964-80 Integrated Circuits High speed magnetic 100 Real time, Fortran IV, Cobol Database management
cores Large disks 100 nano Time 68 PI/I, Time systems, On-line systems
MB, 1 MB main seconds sharing sharing Operating
memory system,
Fourth 1980-till Large scale Semi-conductor 300 Time Fortran 77, Pascal, Personal computers,
now integrated circuits. memory, Winchester nano sharing, ADA, Cobol-74 Distributed systems Integrated
Microprocessors disk, 10 MB main seconds networks CAD/ CAM Real time control,
(LSI) memory, 1000 MB disks Graphics oriented systems

https://sscstudy.com/
https://sscstudy.com/

GENERAL KNOWLEDGE 45

Types of Computer A modern computer has the following hardwares :


(i) Motherboard
Based on Uses (ii) Electric Supply
Computers can be classified based on their uses in (iii) Video Display Controller
following ways : (iv) Removable Media Devices
Analog Computer (v) Secondary Storage
It measures pressure, temperature, length, etc, of (vi) Sound Card
physical quantities and converts into its numerical (vii) Peripherals
values. They are mainly used for scientific and
engineering purposes. Input Devices
We give unprocessed data to computer with the help of
Digital Computer
input devices and get output after the data has been
It is used in counting of numbers, words or other special
processed. We can understand that output and the input
symbols. It is also used in fuel control of automobiles
devices may be hand operated or automated.
and banking processes.
The act of processing is mainly regulated by CPU.
Hybrid Computer
It is the combined form of analog and digital computer. Hand Operated Input Devices
Informations are continuously converted into data that Keyboard and Mouse are the most commonly used
are received in the form of input. Output may be in form hand operated input devices.
of analog and digital. They are used in factories and Some of the hand operated input devices along with
machines. keyboard and mouse are as follows :
Based on Sizes Concept Keyboard
Computers can be classified based on their size in Concept keyboard is related to a special function or item.
following ways The labels of picture or text may reside above its
buttons.
Micro Computer
It is the smallest computer in which ALU and CPU are Track Ball
based on the same chip. It is used as an alternate of mouse. The user has to move
its ball in order to utilise it that causes the pointer to
Personal Computer move on the screen.
It is single user system used generally in business.
Joystick
Mini Computer It is very popular among the people fond of computer
It is more powerful than micro computer and can be games, but today different types of game controllers are
used by many users at the same time. It processes data replacing them gradually. Joystick is used to control the
very fast. machine such as crane in the process of building
Mainframe Computer constructions.
They are giant in size and of highly storage capacity. Digital Camera
They can process an ample amount of data very rapidly. Digital camera can take the images and also records the
They are used in banks, large organizations and public videos. The images and videos taken by digital cameras
sectors. are recorded in files. These files can get copied into the
Super Computer computer that can be further edited for many purposes.
They are capable of processing the huge amount of data Microphone
in fraction of seconds. They are used in very complex Microphone is used to input the sound. It can be used to
calculations. enter text in any word processing application with voice
recognition software in the computer.
Hardware
All the components of computer, parts and devices that Touch Screen
we can touch are known as Hardware. Computer is There is a network of light rays in touch sensitive visual
composed of several physical parts of the computer display unit or in screen that recognises the touch.
hardware on which a number of softwares are installed Touch screen is used in many mobile phones. They are
to do the desired work of operating system and operator. often used in cash machines and shopping centres.

https://sscstudy.com/
https://sscstudy.com/

46 CUET (UG) Section III : General Test

Video Digital Projector


Video digital takes the images from video camera or Projector is that hardware device which projects an
television and stores it into the computer for further use. image on the flat surface like computer screen. These
The video sequences taken by video digital are mainly devices are generally used in meetings, seminars and
used in multimedia presentation. presentations because each person can easily see the
Scanner image projected by it sitting in a big room.
Scanner is used to convert an image into its digital form.
Sound Card
It is similar to photocopier but it makes digital copy of
an image instead of its hard copy. It is used to scan the It is also known as Sound Board or Audio Card.
text with the software optical character recognition. Sound card is an expansion card or unified circuit that
enables computer to produce sound through speaker or
Graphic Tablet headphone.
There is a flat pad on the graphic tablet upon which a
user writes with the help of a special pen. Pictures can Speaker
also be drawn on the screen using that pen in this way. It is the device which presents the sound produced
during any programme. This is connected to sound card
Keyboard and gives the sound produced by card in the form of
Keyboard is the most commonly used input device in output.
computer. There are three types of keys on the keyboard
(a) Alphanumeric keys (b) Special keys Video Card
(c) Function keys It is also known as Graphics Card, Video Board or
Alphanumeric keys include the whole alphabets from A Video Controller. This is an internal circuit board that
to Z and digits from 0 to 9 and different characters such allows the image displaying devices like monitor to
as space, / × ` ; ! @ # $ % ^ & − + = \ { } [ ]. display the images.
Special Keys include ENTER, Backspace, Delete, Shift,
Capslock, Ctrl, Alt, etc.
Internet
Ä An Internet is a global connection of computers linked
Keys from F1 to F12 in the keyboard are known as
together by cables and telephones and makes the
Function Keys.
communication possible among them.
Mouse Ä For such communication, the address of the
It is the most popular point and draw input device which destination and a safe means of moving data are
is connected to a terminal or microcomputer through a required.
cable wire. Ä The rules governing the sending and receiving of data
on the Internet are implemented in two parts as
Output Devices Transmission Control Protocol (TCP) and
The means through which computer gives output are Internet Protocol (IP).
known as Output Devices. Ä Transmission Control Protocol divides the data into
Some of the most popular output devices are as follows : little data packets.
1. Monitor 2. Printer 3. Projector 4. Sound Card Ä The rule of Internet Protocol here is to put destination
5. Speaker 6. Video Card addressing information on such packets.
Ä Hosts are in general, individual machines at a
Monitor particular location.
Video display screen fixed in hard layer and computer is Ä Domains are general category that a computer on the
also known as Monitor or Video Display Terminal. internet belongs to.
Monitor means only that devices in which there is no Ä E-mail is mainly used for sending electronic piece of
any electronic device other than the device required for text.
showing the image. Ä Use net and news groups are set up by people sharing
common interests ranging from current topics to
Printer cultural heritages.
Printer is that external hardware device which gives the Ä Telnet is a program that allows an Internet host
output in printed format after receiving data from computer to become a terminal of another host on the
computer. Internet.
This is such an operated external device used with Ä Gopher displays a set of resources on the Internet in
computer that prints text, image or photo. the form of menus or lists of items.

https://sscstudy.com/
https://sscstudy.com/

GENERAL KNOWLEDGE 47

Ä Hypertext Markup Language (HTML) is a Some antivirus programs are capable of ‘On the Fly’
language to create documents for world wide web. scanning of received e-mails along with scanning of the
Ä Standard Generalised Markup Language (SGML) open files. This process is known as ‘On-Access’
is used to define the structure and manage the content scanning.
of a digital document. Antivirus software does not alter the efficiency of the
Ä World Wide Web (www) is a giant collection of host software for the communication of viruses. These
hypertext pages on the Internet. softwares need to be updated regularly so that the new
Ä Hyper Text Transfer Protocol (HTTP) is used to viruses can be detected.
manage all the linkings between one hypertext The loss incurred by the viruses can also be negated by
document and another. the regular backup of data on other mediums. If any
Ä On the Internet, a home page may be related to a data get misplaced due to this virus then we can retrack
single person, a specific subject, or to a corporation. it by backup.
Ä Uniform Resource Locator (URL) is a way to avail
Important Facts Related to Computer Virus
of Internet resources.
● The Creeper virus was first detected on ARPANET, the forerunner
Ä Browser is an application program which is used for of the Internet, in the early 1970s.
exploring Internet resources. ● The Reaper Program was created to delete creeper.
Ä File Transfer Protocol (FTP) is used to access a ● The actual term ‘virus’ was first used in David Gerrold’s 1972
remote Internet host. novel, when HARLIE was one.
Ä Modem is a device allowing a computer to transmit ● A program called ‘‘Elk Cloner’’ was the first computer virus to
information over a telephone line. It acts as a appear ‘‘in the wild’’ that is, outside the single computer or lab
modulator and demodulator while the process of where it was created.
transmitting and receiving. ● It attached itself to the APPLE DOS 3.3 operating system and
spread via floppy disk.
Ä Wide Area Information Service (WAIS) is a type of ● The first PC virus in the wild was a boot sector virus ‘‘Brain’’
service to access text databases or libraries on the created in 1986.
Internet. ● With the spread of personal computer, traditional computer
viruses emerged in 1980. Bulletin board-driven software sharing
Computer Virus contributed directly to the spread of Trojan horse programs.
A computer virus is a computer program that can copy Shareware and boot leg software were equally common vectors
itself and infect a computer. A virus can spread from one for viruses on BBS.
computer to another when its host is taken to the target ● Macro Viruses have become common since the mid 1990. Most
of these viruses are written in the scripting languages for
computer, for instance because a user sent it over a
Microsoft Programs such as word and Excel and spread
network or the Internet, or carried it on a removable throughout Microsoft Office by infecting documents and
medium such as a floppy disk, CD, DVD or USB drive. spreadsheets.
Viruses can increase their chances of spreading to other ● Viruses that spread using cross site scripting were first reported
computers by infecting files on a network file system or in 2002 and were academically demonstrated in 2005. There
a file system that is accessed by another computer. have been multiple instances of the cross site scripting viruses in
the wild, exploiting websites such as My Space and Yahoo.
Antivirus Software and Other Preventive The most popular five computer virus came into light so far as
follows :
Measures
(i) I Love You (2000) It spreads through e-mails.
Antivirus software can destroy the viruses by finding (ii) Code Red (2001) It affects windows server.
them. The antivirus software uses two common methods (iii) Nimada (2001) It is the most rampant virus.
in order to detect viruses.
(iv) Melissa (1999) It spreads through Word Macro and e-mails.
1. By using the Virus Signature Definitions list (v) Sasser (2004) It spreads by attaching to port 445 on the
2. By using Heuristics Algorithm network windows system.

https://sscstudy.com/
https://sscstudy.com/

48 CUET (UG) Section III : General Test

Practice Questions
1. Assembly language is 13. Which of the following displays programs that are
(a) Machine language currently running? A single click on one of the
(b) High-level programming language program icons will bring the window up?
(c) Low-level programming language (a) Menu bar (b) Task bar
(d) Language for assembling computers (c) Title bar (d) Status bar
2. Which of the following is not one of the syntax 14. ……… is a process of summarising data from more
rules? than one source.
(a) The order in which you list the function’s arguments (a) Data Validation (b) Data Consolidation
(b) The precedence of the arguments (c) Data Form (d) Data Filter
(c) Whether or not the function has arguments
(d) Properly spelling the function’s name 15. When we create a chart on a separate sheet in the
same workbook, it is called a
3. The binary system is a number system to the base (a) View chart (b) View sheet
(a) 2 (b) 4 (c) Embedded chart (d) Chart sheet
(c) 8 (d) 10
16. In order to tell Excel that we are entering a
4. Which of the following is not an example of formula in cell we must begin the with an
hardware? operator such as
(a) Scanner (b) Printer (a) $ (b) @ (c) = (d) #
(c) Monitor (d) Interpreter
17. Using the chart wizard, we can quickly and easily
5. Which contents are lost when the computer turns turn our
off? (a) data into charts and vice-versa
(a) Storage (b) Input (b) charts into charts
(c) Output (d) Memory (c) data into charts
(d) charts into most advanced charts
6. Which of the following is hardware and not
software? 18. Excel can display upto ....... fields of a data form.
(a) Excel (b) Printer driver (a) 16 (b) 32
(c) Operating System (d) CPU (c) 64 (d) 128
7. On a CD-RW, you can 19. A numeric value can be treated as a label value if it
(a) read and write information proceeds with
(b) only read information (a) apostrophe (’) (b) exclamation (!)
(c) only write information (c) hash (#) (d) ampersand (&)
(d) read, write and rewrite information
20. Which menu option can be used to split windows into
8. To be able to “boot”, the computer must have a(n) two?
(a) Compiler (b) Loader (a) Format > Windows (b) View > Windows > Split
(c) Operating System (d) Assembler (c) Windows > Split (d) View > Split
9. This is not a function category in Excel 21. Data can be arranged in a worksheet in an easy to
(a) Logical (b) Data Series understand manner using
(c) Financial (d) Text (a) auto formatting (b) applying styles
(c) changing fonts (d) All of these
10. In Excel, this is a prerecorded formula that
provides a shortcut for complex calculations 22. Which one is the example of spreadsheet package?
(a) Value (b) Data Series (a) Visicalc (b) Unify
(c) Function (d) Field (c) Ada (d) Snowball
11. The speed at which the monitor accepts data is 23. Legends are used in
called (a) Chart Wizard (b) Function Wizard
(a) bandwidth (b) interlacing (c) Auto Sum (d) Validation
(c) response time (d) scanning
24. Which is the comparison operator?
12. The copy command saves to (a) = (b) /
(a) the desktop (c) * (d) None of these
(b) the clipboard
(c) microsoft word
25. From which tool bar we can change chart type?
(a) Chart Tool Bar (b) Formatting Tool Bar
(d) paste
(c) Formula Bar (d) Clipboard Bar

https://sscstudy.com/
https://sscstudy.com/

GENERAL KNOWLEDGE 49

26. Which sign is used to specify a cell range? 35. Which protocol provides e-mail facility among
(a) : (colon) (b) / (slash) different hosts?
(c) * (asterisk) (d) - (hyphen) (a) SMTP (b) FTP
(c) TELNET (d) SNMP
27. In which tool bar will you find Auto Sum short
button? 36. 192.9.200.153 is an
(a) Formatting Bar (b) Standard Bar (a) Ethernet Address
(c) Clipboard Bar (d) Formula Bar (b) IP Address
(c) Computer Address
28. In which table, we find the pivot table command? (d) Any of these
(a) Tools (b) Data
(c) Insert (d) Window 37. The storage that supplements the primary internal
storage of a computer is known as
29. Which menu keeps the Goal seek command? (a) secondary storage
(a) Data (b) Tools
(b) primary storage
(c) Insert (d) Format
(c) back-end storage
30. By which option, we can show and print the (d) background storage
gridlines?
38. A browser is a
(a) Page Setup (b) Format
(a) line that delineates the edge of a programme
(c) Auto Format (d) Style
window
31. What are ‘Laptops’? (b) software programme especially designed for viewing
(a) Computers used in clinical laboratories web pages on the internet
(b) A computer manufactured by Compaq (c) tool for resizing the window
(c) A computer having voice recognition system (d) device used for modifying the window
(d) Lightweight computers, small enough to fit in a small
39. A folder
suitcase
A. is an organizational tool used to store files
32. The basic architecture of computer was developed B. is synonymous with directory, but folder is the newer
by preferred term
(a) John Von Neumann (b) Charles Babbage C. contains data
(c) Blaise Pascal (d) Garden Moore (a) Only A
(b) Only B
33. What does CPU stand for? (c) Only C
(a) Central Processing Unit (b) Central Process Unit
(d) A and C
(c) Central Printing Unit (d) Central Peripheral Unit
40. The part of a computer system containing the
34. Ethernet, LAN, Token Ring and Token Bus are circuitry that does the adding, subtracting,
types of multiplying, dividing and comparing, called as
(a) WAN (a) Arithmetic Logic Unit
(b) communication channels (b) Control
(c) LAN (c) Memory
(d) physical medium (d) CPU

ANSWERS
1. (c) 2. (b) 3. (a) 4. (d) 5. (d) 6. (d) 7. (d) 8. (c) 9. (b) 10. (c)
11. (a) 12. (b) 13. (d) 14. (b) 15. (d) 16. (c) 17. (c) 18. (b) 19. (a) 20. (c)
21. (d) 22. (b) 23. (c) 24. (d) 25. (b) 26. (a) 27. (d) 28. (b) 29. (b) 30. (d)
31. (d) 32. (a) 33. (a) 34. (b) 35. (a) 36. (a) 37. (c) 38. (b) 39. (d) 40. (a)

https://sscstudy.com/
https://sscstudy.com/

GENERAL MENTAL ABILITY 3


CHAPTER 01

Analogy
‘Analogy’ means ‘similarity’, i.e. having similar features. SN Types of Relationship Examples
Analogy test is an inference of similarity between two
15. Quantity & Unit Mass : Kilogram, Length :
relations. In it, words are used between which specific Metre
logical relations exist. It is a comparison between two
16. Utility relationship Chair : Sitting, Bed :
elements that are usually thought to be different, but
Sleeping
have something in common.
17. Comparative Black : Coal, Red : Blood
Questions on analogy test the ability of a candidate to
understand the relationship between two given elements 18. Animal and Sound Snake : Hiss, Duck : Quack
and apply the same relationship to find the one asked in 19. Place relationship Delhi : Red fort,
the question. Punjab : Ludhiana
Analogy can be of four types from which questions are 20. Purpose relationship Sandpaper : Abrasion
generally asked in competitive exams. 21. Specialist & Subject Heart : Cardiologist,
Skin : Dermatologist
1. Word Analogy
22. Individual and Young ones Man : Child, Cow : Calf
In questions based on word analogy, two words are given
23. Country-Capital India : New Delhi,
related to each other in some way. The candidate has to Japan : Tokyo
find that relationship and apply the same to choose the
correct alternative which is either in the form of a word 24. Country-Currency Iraq : Dinar, Russia : Rouble
or a pair of words. Some of the important word 25. Instrument & Barometer : Pressure,
relationships are as following Measurement Hygrometer : Humidity
26. Individual & Group Sailor : Crew, Cattle : Herd
SN Types of Relationship Examples
27. Individual & Dwelling Dog : Kennel, Horse : Stable
1. Synonymous relationship Slim : Thin, Meet : Contact
28. Study & Topic Ornithology : Birds,
2. Opposite relationship Poor : Rich, Never : Always
Seismology : Earthquakes
3. Cause and effect Shoot : Kill, Fast : Hunger
29. Worker & Working place Chef : Kitchen,
relationship
Teacher : School
4. Game and place of playing Tennis : Court, Boxing : Ring
30. Worker & Product Farmer : Crop, Poet : Poem,
5. Worker & Tool Writer : Pen, Hunter : Gun Chef : Food
6. Tool & Object Pen : Paper, Eraser : Paper Ex. 1 Find out the missing word from the given
7. Whole & Part Car : Wheel alternatives, which bears the same relationship to the
8. Part & Whole Handle : Chair third/fourth word, as the first two bear
9. Word and Intensity Hot : Warm Cattle : Herd :: Sheep : ?
(a) Flock (b) Swarm (c) Shoal (d) Mob
10. Classification Cow : Mammal, Ice : Water
Sol. (a) Herd is a group of cattle. Similarly, flock is a
11. Function Surgeon : Operates collection of sheep.
12. Male and female Bull : Cow, Brother : Sister Ex. 2 Cat is related to mouse in the same way as
13. Sequential Morning : Afternoon, Nine : Bird is related to …… .
Ten (a) Worm (b) Snake
14. Finished Product & Raw Bread : Flour, Fabric : Yarn (c) Animal (d) Crow
Material Sol. (a) In this case, cat feeds on Mouse. Similarly, bird
feeds on worm.

https://sscstudy.com/
https://sscstudy.com/

4 CUET (UG) Section III : General Test

Ex. 3 Find a pair similar to ‘Arrow : Bow’ from the Ex. 5 Out of the four given alternatives, choose that
following. alternative as your answer which is similar to the
(a) Football : Hand (b) Salad : Knife given words. L B W : Slip : Cover
(c) Bullet : Rifle (d) Smoke : Water (a) Dence (b) Dribble
Sol. (c) As, arrow is released with the help of (c) Corner (d) Chinaman
bow, similarly a bullet is released with the help of Sol. (d) LBW, slip and cover, all are related to cricket.
rifle. Similarly, chinaman is a term used in cricket.
Ex. 4 Select the alternative that does not have a Ex. 6 Find the similarity in the following.
similar relationship as the given pair. Elephant, Camel, Buffalo, Giraffe
Forward : Backward (a) The milk produced by all of them cannot be
(a) Hope : Despair (b) Love : Hate consumed by people
(c) Anger : Wrath (d) Light : Dark (b) All of them have horns
Sol. (c) In the given pair both the words are opposite to (c) None of them are mammals
each other. (d) The young ones of all of them are called calf
But in pair (c) i.e. Anger : Wrath, both the words have Sol. (d) The young ones of elephant, camel, buffalo and
similar meaning. giraffe are called calf.

2. Letter Analogy
In letter or alphabet analogy, both of question and answer pairs consists of letters or alphabets. The candidate has to
examine the question pair and find the relationship on the basis of which either a group of letters or a pair of group
of letters is to be find out.
To solve these questions the candidate has to remember the positional values of letters in English alphabetical order.
Forward 1 2 3 4 5 6 7 8 9 10 11 12 13 14 15 16 17 18 19 20 21 22 23 24 25 26
Alphabet A B C D E F G H I J K L M N O P Q R S T U V W X Y Z
Backward 26 25 24 23 22 21 20 19 18 17 16 15 14 13 12 11 10 9 8 7 6 5 4 3 2 1

Ex. 7 Complete the second pair in the same way as Sol. (d) As, 32 + 2 = 11.
first pair CD : GH :: LM : ? Similarly, 72 + 2 = 51
(a) DC (b) EG (c) AB (d) IK
Sol. (c) As, in the first term i.e CD, the letters are Ex. 10 Find the set among the four sets which is like
consecutive and the second letter comes after the first the given set ( 13 : 20 : 27).
letter and so is the term GH. Similarly, LM and AB (a) (3 : 11 : 18) (b) (18 : 25 : 32)
follows the same pattern. (c) (18 : 27 : 72) (d) (7 : 14 : 28)

Ex. 8 Choose the pair from the given alternatives Sol. (b) As, 13 20 27 Similarly, 18 25 32
which is similar to the given pair
+7 +7 +7 +7
BCD : EFG :: ? : ?
(a) L M N : O P Q (b) P R S : S T V 4. Mixed Analogy
(c) A B C : M N O (d) X W V : Z Y Z
Sol. (a) As, B C D In this type of analogy, letters and numbers are given,
Similarly, L M N following a certain relationship. The relationship may be
+3 +3 +3 +3 +3 +3

O P Q
based on the position of alphabets or letters, sum or
E F G
product of the position of alphabet or letters etc. The
3. Number Analogy candidate is required to understand the relationship and
complete the analogy.
In this analogy, either a number or a group of numbers
are related to each other in the first pair according to a Ex. 11 Complete the second pair in the same way as
certain pattern. The candidates has to find that pattern first
and choose the related number or a pair of numbers.
NEWS : 14, 5, 23, 19 : : PAPER : ?
Ex. 9 Choose the number from the given (a) 16, 5, 16, 1, 18 (b) 18, 5, 16, 1, 16
alternatives which will complete the second pair in (c) 16, 1, 16, 5, 18 (d) 32, 2, 32, 10, 36
the same way as first pair. Sol. (a) Here, each letter is assigned its positional value
3 : 11 :: 7 : ? in forward order
(a) 22 (b) 29 (c) 18 (d) 51 So, PAPER → 16, 1, 16, 5, 18

https://sscstudy.com/
https://sscstudy.com/

GENERAL MENTAL ABILITY 5

Practice Questions
1. Goitre is related to Iodine as Anaemia is related to 14. Ohm : Watt : Ampere
(a) Vitamin (b) Blood (c) Iron (d) Weakness (a) Electricity (b) Volt (c) Hour (d) Light
2. Needle is related to Thread as Pen is related to 15. Ear : Nose : Lips
(a) Ink (b) Cap (c) Paper (d) Wood (a) Finger (b) Lungs (c) Heart (d) Kidney

3. If H2 O : Hydrogen, then KOH : …… Directions (Q. Nos. 16-18) Find out the common feature
(a) Cobalt (b) Phosphorous among the given words and pick the alternative that
(c) Potassium (d) Krypton mentions the properties common to the given words.

Directions (Q. Nos. 4-8) In each of the following 16. Nose : Eyes : Ears
questions, there is a certain relationship between two given (a) They are internal part of human body
(b) They are not the external part of human body
words on the left side of :: and one word is given on the right
(c) They are parts of the body below waist
side of :: while another words is to be found from the given
(d) They are parts of the body above neck
alternatives having the same relation with this word as the
words of the given pair bear. 17. Dhoni : Yuvraj : Dravid
(a) Cricketers (b) Athlete
4. Consolation : Grief :: Sedative : …
(c) Politicians (d) Singers
(a) Chloroform (b) Anesthesia
(c) Pain (d) Burn 18. Cinema : Press : Television
(a) They are means of entertainment
5. Bird : Wings :: Fish : ?
(b) They are means of mass media
(a) Gills (b) Head (c) Fins (d) Legs
(c) All are public undertakings
6. Contemporary : Historic :: … : Ancient (d) They give word wide news
(a) Past (b) Classic (c) Modern (d) Future
Directions (Q. Nos. 19-25) In these questions, there is a
7. Blunt : Sharp :: Sow : certain relationship between the two terms to the left of ::
(a) Reap (b) Seeds (c) Farmer (d) Crop and the same relationship exists between the two terms to the
8. Circle : Circumference :: Square : right. One of the terms either to the left or to the right is
(a) Sides (b) Area (c) Perimeter (d) Diagonal missing. Find out this term.
19. KMF : LLH :: RMS : ?
Directions (Q. Nos. 9-11) The following questions consist (a) SLR (b) SLU (c) SSU (d) SUS
of two words each that have a certain relationship to each
other followed by four lettered pair of words. Choose the pair 20. FJUL : BOQQ :: LHRX : ?
having the same relationship. (a) BKPR (b) MNCC (c) HRYY (d) HMNC

9. Hands : Gloves 21. EJOT : VQLG :: BGLQ : ?


(a) Bank : Fog (b) Socks : Feet (a) AEIM (b) AFKP (c) YTOJ (d) ZUPK
(c) Fish : Water (d) Legs : Shoes 22. JLNP : OMKI :: SUWY : ?
10. Ass : Bray (a) FGHI (b) MLKJ (c) PLHD (d) XVTR
(a) Flies : Squeak (b) Hen : Mew
23. A F K P : B G L Q :: ? : D I N S
(c) Fox : Snout (d) Sheep : Bleat
(a) C H M R (b) D N I S (c) H D K P (d) C F M S
11. Brook : River
(a) Pen : Paper (b) Yard : Alley
24. FLOWER : REWOLF :: FRUITS : ?
(a) STRUIF (b) STUIRF (c) STIURF (d) STUIFR
(c) Path : Highway (d) Vein : Artery
12. Select the alternative that has a different relationship 25. B J N T : C I O S :: D H P V : ?
as the given pair. (a) E G Q U (b) E L Q W
(c) E L P V (d) E I O U
Inside : Outside
(a) Day : Night (b) Sun : Star Directions (Q. Nos. 26-30) Each of these questions
(c) Light : Dark (d) White : Black consists of a pair of letters or groups having a certain
relationship followed by alternatives. Select the pair
Directions (Q. Nos. 13-15) Out of the four given
similarly related to the given pair.
alternatives, choose that alternative as your answer which
is similar to the given words. 26. BC : FG
13. Pen : Pencil : Rubber (a) PQ : ST (b) HI : LM
(a) Page (b) Cell (c) Pillow (d) TV (c) AD : PQ (d) JK : LM

https://sscstudy.com/
https://sscstudy.com/

6 CUET (UG) Section III : General Test

27. MN : OP 42. 7 : 24
(a) AD : GH (b) AB : PQ (a) 30 : 100 (b) 23 : 72
(c) QR : ST (d) RS : TV (c) 19 : 58 (d) 11 : 43

28. CEH : IKN Directions (Q. Nos. 43-45) In each of the following
(a) ACD : FHJ (b) CDF : IJK questions, choose that set of numbers from the four
(c) EFH : KMN (d) OQT : UWZ alternatives, which is similar to the given set.
29. LLAMS : SMALL 43. (2, 14, 16)
(a) SRENID : DINERS (b) CHART : TRACH (a) (2, 7, 8) (b) (2, 9, 16)
(c) BARK : KRAB (d) TREE : EERT (c) (3, 21, 24) (d) (4, 16, 18)

30. LOWER : WORLE 44. (32, 24, 8)


(a) GLAZE : AGELZ (b) AMONG : OMNAG (a) (26, 32, 42) (b) (34, 24, 14)
(c) WORDS : ROSWD (d) ENTRY : RNYET (c) (24, 16, 0) (d) (42, 34, 16)

Directions (Q. Nos. 31-38) In these questions, there is a 45. (525, 813, 714)
certain relationship between two given numbers on the left (a) 353 (b) 329
(c) 606 (d) 520
side of :: and one number is given on the right side of :: while
another number is to be found from the given alternatives. Directions (Q. Nos. 46-55) In each of these questions,
Find the missing term. find out the missing term from the given alternatives.
31. 18 : 30 :: 36 : ? 46. C : 16 :: F : ?
(a) 54 (b) 62 (c) 64 (d) 66 (a) 30 (b) 40
(c) 49 (d) 50
32. 3 : 243 :: 5 : ?
(a) 425 (b) 465 (c) 546 (d) 3125 47. M×N : 13×14 :: F×R : ?
(a) 7×19 (b) 5×17
33. 6 : 222 :: 7 : ?
(c) 14×15 (d) 6×18
(a) 210 (b) 336 (c) 343 (d) 350
169
34. 68 : 130 :: ? : 350 48. MK : :: JH : ?
121
(a) 210 (b) 216 (c) 222 (d) 240 100 100
(a) (b)
64 81
35. 42 : 56 :: 72 : ? 64 81
(a) 81 (b) 90 (c) 92 (d) 100 (c) (d)
120 100
36. 7 : 56 :: 9 : ? T X
49. : 2 :: :?
(a) 63 (b) 81 (c) 90 (d) 99 J H
(a) 2 (b) 3
37. 42 : 56 :: 110 : ? 23
(a) 18 (b) 132 (c) (d) 4
7
(c) 136 (d) 140
E 22 B
50. : :: :?
38. 48 : 122 :: 168 : ? H 19 I
(a) 215 (b) 225 24 25
(a) (b)
(c) 290 (d) 292 17 18
3 17
(c) (d)
Directions (Q. Nos. 39-42) Each of these questions 7 19
consists of a pair of numbers that have a certain
relationship to each other followed by four other pairs of 51. MLO : 121114 :: GJL : ?
numbers given as alternatives. Select the pair having the (a) 192022 (b) 060911 (c) 160813 (d) 2218175
same relation. 52. GREAT : 25 :: NUMBER : ?
39. 27 : 9 (a) 36 (b) 38 (c) 27 (d) 24
(a) 64 : 8 (b) 125 : 5
53. STBP : 1920216 :: MNGO : ?
(c) 135 : 15 (d) 729 : 81
(a) 1314715 (b) 1413715 (c) 5173141 (d) 3114715
40. 5 : 35
(a) 7 : 77 (b) 9 : 45
54. X M A E : 16 :: V T N G : ?
(a) 21 (b) 17 (c) 35 (d) 18
(c) 11 : 55 (d) 3 : 24
G ×Q B×V
41. 8 : 256 55. : 20 :: :?
Q V
(a) 7 : 343 (b) 9 : 243
(c) 10 : 500 (d) 5 : 75 (a) 25 (b) 27 (c) 20 (d) 17

https://sscstudy.com/
https://sscstudy.com/

ANSWERS
1. (c) 2. (a) 3. (c) 4. (c) 5. (c) 6. (c) 7. (a) 8. (c) 9. (d) 10. (d)
11. (c) 12. (b) 13. (a) 14. (b) 15. (a) 16. (d) 17. (a) 18. (b) 19. (b) 20. (d)
21. (c) 22. (d) 23. (a) 24. (c) 25. (a) 26. (b) 27. (c) 28. (d) 29. (a) 30. (c)
31. (d) 32. (d) 33. (d) 34. (c) 35. (b) 36. (c) 37. (b) 38. (c) 39. (d) 40. (a)
41. (c) 42. (b) 43. (a) 44. (c) 45. (c) 46. (c) 47. (d) 48. (a) 49. (b) 50. (b)
51. (b) 52. (a) 53. (a) 54. (d) 55. (a)

Hints & Solutions


1. (c) Goitre is caused by the deficiency of 13. (a) As, Pen, Pencil and Rubber are Similarly, S U W Y X V T R
–1
Iodine. Similarly, Anaemia is caused by the stationary items. Similarly, page is also a
–1
deficiency of Iron. stationary item
–1

14. (b) As, Ohm, Watt, Ampere are –1


2. (a) Thread is required by the Needle to
function. Similarly, Ink is required by the Pen to measurement units of electricity. Similarly, volt
function. is the measurement unit of electricity. 23. (a)
H2O → Hydrogen As, A F K P Similarly, C H M R
3. (c) As, 15. (a) Ear, Nose and Lips all are external
+1 +1 +1 +1 +1 +1 +1 +1
parts of the human body and so is the finger.
Here, ‘H’ signifies Hydrogen.
B G L Q D I N S
Similarly, 16. (d) ‘Nose’, ‘Eyes’ and ‘Ears’ are the parts
KOH → Potassium of human body above neck. 24. (c) As, F L O W E R R EW O L F
17. (a) It is clear that the common feature,
Here, ‘K’ signifies Potassium. among Dhoni, Yuvraj and Dravid is that they
are cricketers.
4. (c) As, cosolation soothe the grief, Similarly,
sedative alleviate the pain. 18. (b) All are means of mass media.
5. (c) As, bird uses wings to fly. Similarly, fish 19. (b) As, K M F Similarly, R M S
Similarly, F R U I T S S T I U R F
uses fins to swim. +1 –1 +2 +1 –1 +2

6. (c) Contemporary and historic are opposite L L H S L U


to each other, similarly modern and ancient are
20. (d) As, F J U L
opposite to each other.
–4 +5 –4 +5
7. (a) As, blunt and sharp are opposite to
each other, similarly sow and reap are B O Q Q
opposite to each other. 25. (a) As, B J N T
Similarly, L H R X
+1 –1 +1 –1
8. (c) As, the boundary of the circle is known –4 +5 –4 +5
as circumference, similary the boundary to the C I O S
square is known as perimeter. H M N C
Similarly, D H P V
9. (d) Hands are the parts of the body where 21. (c) Here, each letter is replaced by an +1 –1 +1 –1
Gloves are worn. Similarly, Legs are the parts alphabet which occupies the same place as
of the body where Shoes are worn. E G Q U
the letter in reverse English alphabetical
10. (d) Bray is the sound produced by Ass. order. 26. (b) As, B C Similarly, H I
Similarly, Bleat is the sound produced by So, B G L Q → Y T O J +4 +4 +4 +4
Sheep.
22. (d) As, F G L M
11. (c) As, River is bigger form of brook.
Similarly, Highway is bigger form of path. J L N P O M K I 27. (c) As, M N Similarly, Q R
–1
+2 +2 +2 +2
12. (b) In the given pair, both the words are –1
opposite to each other. But in pair (b) both the –1 O P S T
words, i.e. sun and star are similar in nature. –1

https://sscstudy.com/
https://sscstudy.com/

8 CUET (UG) Section III : General Test

28. (d) As, C E H Similarly, O Q T 39. (d) As, 27 : 9 ⇒ 33 : 32 50. (b) Here, each letter is assigned its
+6 +6 +6 +6 +6 +6 3 2 positional value in reverse English alphabetical
similarly, 729 : 81 ⇒ 9 : 9 order.
I K N U W Z
E 22
40. (a) The first number is multiplied by the As, =
29. (a) The second group is a meaningful H 19
next prime number to obtain the second
B 25
word formed by reversing the order of the number. As, 5 : 5 × 7 . Similarly, 7 : 7 × 11 Similarly, =
letters of the first group. I 18
83
41. (c) As, 8 : 256 ⇒ 8 : −1
30. (c) 2 51. (b) As, 13 M  → 12
As, L O W E R
3 −1
10 12 L  → 11
Similarly, 10 : 500 ⇒ 10 :
2 −1
15 O  → 14
W O R L E
42. (b) As, 7 : 24 ⇒ 7 : (7 × 3) + 3 i.e. MLO : 121114
−1
Similarly, 07 G  → 06
Similarly, W O R D S Similarly, 23 : 72 ⇒ 23 : (23 × 3) + 3
−1
10 J  → 09
43. (c) −1
12 L  → 11
R O S W D As, 2 14 16 Similarly, 3 21 24
∴ GJL : 060911
31. (d) As, 18 ×7 ×7
: 30 Similarly, 36 : 66 52. (a) As, GREAT has 5 letters, it is
×8 ×8
represented by square of 5 i.e 25. similarly,
(18×2)–6 (36×2)–6 NUMBER has 6 letters. So, it is represented by
44. (c)
square of 6 i.e. 36
32. (d) As, 3 As, 32 24 8 Similarly, 24 16 0
: 243 Similarly, 5 : 3125
53. (a) Here, each letter is represented by its
–8 –16 –8 –16
×34 ×54 position in English alphabetical order.

33. (d) As, 6, ( 63 + 6) = 222 45. (c) As, 525 ⇒ 5 + 2 + 5 = 12 As, S T B P Similarly, M N G O
813 ⇒ 8 + 1 + 3 = 12
Similarly, 7, (7 3 + 7 ) = 350 714 ⇒ 7 + 1 + 4 = 12 19 20 2 16 13 14 7 15
34. (c) As, 68 = 43 + 4:130 = 53 + 5 Similarly, 606 = 6 + 0 + 6 = 12
54. (d) As, X M A E
Similarly, 222 = 6 3 + 6 : 350 = 7 3 + 7
46. (c) C is the 3rd letter of the English
2
35. (b) As, 42 = 6 × 7: 56 = 7 × 8, alphabet and ( 3 + 1) = 16
24 13 1 5
Similarly, 72 = 8 × 9 Similarly, F is the 6th letter of the English
So, missing number = 9 × 10 = 90 alphabet and ( 6 + 1)2 = 49
2+4 1+3 1 5
36. (c) As, 7 : 56 Similarly, 9 : 90 47. (d) M and N are 13th and 14th letters of
the English alphabets, respectively. So, M × N 6 + 4 + 1 + 5 =16
7×(7+1) 9×(9+1) corresponds to 13 × 14. Similarly, F and R are Similarly, V T N G
6th and 18th letters of the English alphabet,
37. (b) As, 42 = 7 × 6 and 56 = 7 × ( 6 + 2 ) respectively. So, F × R corresponds to 6 × 18.
22 20 14 7
Similarly, 110 = 11 × 10
48. (a) M and K are 13th and 11th letters of
So, the required number
13 2 169
English alphabet, respectively and = 2+2 2+0 1+4 7
= 11 × (10 + 2 ) = 11 × 12 = 132 112 121
38. (c) As, 48 : 122 Similarly, J and H are 10th and 8th letters of the 4 + 2 + 5 + 7 =18
10 2 100
English alphabet, respectively and 2 = .
8 64 55. (a) Here, each letter is assigned its position
(72–1) (7+4)2 +1
value in reverse English alphabetical series.
Similarly, 168
49. (b) T and J are 20th and 10th letters of the G × Q 20 × 10
: 290 As, = = 20
English alphabet, respectively and 20 ÷10 = 2 . Q 10
Similarly, X and H are 24th and 8th letters of the B × V 25 × 5
(132–1) (13+4)2 +1 Similarly, = = 25
English alphabet, respectively and 24 ÷ 8 = 3. V 5

https://sscstudy.com/
https://sscstudy.com/

General Mental Ability 9

CHAPTER 02

Classification
Classification means to group various objects on the group and only then one can be able to solve such
basis of a certain common quality that they possess and questions.
spot the odd one out. In this test, questions are designed Ex. 3 Choose the group of letters which is different
to test the candidate’s ability of classifying given objects from others.
and find the one that doesn’t belong to the group. (a) B D (b) I K (c) P N (d) S U
In this type of questions, all the items except one, follow Sol. (c) Here,
a certain rule or pattern or they possess some common +2 +2
B D, I K
quality or characteristics between them and one which is –2 +2
odd does not possess the common quality or P N, S U
characteristics. The candidate has to find that odd item.
All except PN follows the same pattern. So, PN is the odd
Classification can be of three types from which questions one.
are generally asked in competitive exams, i.e.
Ex. 4 Choose the odd letter group.
1. Word Classification 2. Alphabet Classification (a) BDGK (b) JLOS (c) NPSW (d) MORU
3. Number Classification Sol. (d) Here,
B D G K J L O S
1. Word Classification ;
+2 +3 +4 +2 +3 +4
In this classification, either words or pairs of words are
given and the candidate has to find out the basis of N P S W M O R U
;
similarity between them so as to separate the odd word +2 +3 +4 +2 +3 +3
or pair of words.
All except MORU, follows the same pattern. So, MORU is
Ex. 1 Choose the odd one out.
the odd one.
(a) Optical disc (b) Magnetic Storage
(c) Solid State Drive (d) Petabyte
Sol. (d) Except petabyte, all others are storage divices, 3. Number Classification
but petabyte is a unit of measurement used to measure In this type, certain numbers are given either in simple
the data. group or pairs, out of which all except one share some
Ex. 2 Choose the odd pair of words. common property and hence are alike while one is
(a) Cow : Calf (b) Dog : Bitch different. This number is to be chosen as the answer.
(c) Lion : Cub (d) Tortoise : Turtle
Ex. 5 Choose the number which is different from
Sol. (b) Except in Dog : Bitch, all other pairs have the
others in the group.
second word as the young one of the first. In Dog:Bitch,
second is the female of the first. (a) 12 (b) 25 (c) 37 (d) 49
Sol. (c) Clearly, 37 is the only prime number in the
group.
2. Alphabet Classification
Ex. 6 Choose the number/pair/group which is
(Letter Classification) different from others.
Sometimes we are given a group of alphabets and we are (a) 50-66 (b) 22-38 (c) 64-80 (d) 63-77
asked to find the odd letter group out of them. The Sol. (d) Clearly, in each of the pairs except (d), the
candidate must understand the relationship within each second number is 16 more than the first.

https://sscstudy.com/
https://sscstudy.com/

10 General Mental Ability

Practice Questions
Directions (Q. Nos. 1-16) In each of these questions, four 19. (a) Principal : School (b) Pages : Note book
words have been given out of which three are alike in some (c) Letters : Word (d) Students : Class
manner while the fourth one is different. Choose out the
20. (a) Dim : Bright (b) Wrong : Right
odd one.
(c) Shallow : Deep (d) Genuine : Real
1. (a) Pear (b) Apple (c) Guava (d) Orange
21. (a) Nitrogen & oxygen : Air
2. (a) Football (b) Volleyball (c) Cricket (d) Chess (b) Sulphur & Phosphorous : Match stick
3. (a) Tortoise (b) Duck (c) Hydrogen & Oxygen : Water
(c) Snake (d) Whale (d) Magnesium & Silver : Stainless steel
4. (a) Violet (b) Blue 22. (a) Water : Thirst (b) Talent : Education
(c) Green (d) White (c) Food : Hunger (d) Air : Suffocation
5. (a) Tea (b) Cinchona 23. (a) Sky : Cloud (b) Purse : Wallet
(c) Rubber (d) Chalk (c) Cupboard : Almirah (d) Chair : Stool
6. (a) Bajra (b) Mustard 24. (a) Tree : Stem (b) Face: Eye
(c) Rice (d) Wheat (c) Chair : Sofa (d) Plant : Flower
7. (a) Rashtrapati Bhavan 25. (a) Shirt : Dress (b) Boy : Girl
(b) Chatrapati Shivaji Terminus (c) Mango : Fruit (d) Table : Furniture
(c) Taj Mahal
(d) Sun Temple 26. (a) Pen - Ink (b) Pot - Water
(c) Bottle - Tonic (d) Cylinder - Air
8. (a) Tonnes (b) Quintals
(c) Grams (d) Kilometres 27. (a) Bulb - Filament (b) Pencil - Lead
(c) Leaf - Flower (d) Fruit - Seed
9. (a) Dagger (b) Hammer
(c) Knife (d) Sword 28. Which among the following is a mismatched pair of
religion practiced and its holy book?
10. (a) King (b) Queen (a) Islam : Quran (b) Sikhism : Guru Granth Saheb
(c) Bishop (d) Minister (c) Jainism : Upanishad (d) Christianity : Bible
11. (a) Feeling (b) Joy Directions (Q.Nos. 29-43) In each of these questions,
(c) Anxiety (d) Anger some groups of letters are given, all of which except one,
12. (a) Aluminum (b) Iron share a common similarity while one is different. Choose
(c) Copper (d) Brass the odd one out.
29. (a) A (b) O (c) U (d) Y
13. (a) Silent valley (b) Silicon valley
(c) Indus valley (d) Damodar valley 30. (a) GE (b) MK (c) WU (d) QN

14. (a) Spade (b) Knife 31. (a) AE (b) AI (c) IO (d) EI
(c) Axe (d) Blacksmith 32. (a) ABD (b) FGI (c) LMO (d) STU

15. (a) Garlic (b) Chilli 33. (a) BDW (b) DFU (c) FHS (d) GIQ
(c) Ginger (d) Potato 34. (a) VRT (b) RMP (c) YUW (d) FBD
16. (a) Manure (b) Nitrogen 35. (a) DEB (b) HIF (c) NOL (d) RTP
(c) Ammonia (d) Urea 36. (a) HJN (b) JLP (c) PRU (d) QSW
Directions (Q. Nos. 17-27) In these questions, four pairs 37. (a) CHG (b) LMM (c) BBC (d) HEG
of words are given, out of which the words in three pairs 38. (a) RAT (b) OUT (c) BED (d) LOT
bear a certain common relationship. Choose the pair in 39. (a) D H F (b) K O M (c) R V T (d) W Y X
which the words are differently related. 40. (a) A Y B Z (b) B X C Y (c) D V E W (d) M P O N
17. (a) Shoe : Leather (b) Iron : Axe 41. (a) C P B O (b) B O D Q
(c) Table : Wood (d) Jewellery : Gold (c) A N H U (d) E S G T
18. (a) Saturn : Planet 42. (a) P R S Q (b) U W X V
(b) Sun : Star (c) L O N M (d) C E F D
(c) Milky Way : Constellation 43. (a) M O N J K (b) A C A Z V
(d) Titan : Satellite (c) G M R G S (d) M E T E I

https://sscstudy.com/
https://sscstudy.com/

GENERAL MENTAL ABILITY 11

Directions (Q. Nos. 44-52) In each of the following Directions (Q.Nos. 53-60) Choose the odd numeral
questions, four numbers are given. Out of these, three are pair/group in each of the following questions.
alike in a certain way but the rest one is different. Choose 53. (a) 34 : 43 (b) 55 : 62
the one that is different from the rest four. (c) 62 : 71 (d) 83 : 92
44. (a) 17 (b) 27 (c) 29 (d) 37 54. (a) 12 : 96 (b) 13:117
45. (a) 29 (b) 53 (c) 85 (d) 125 (c) 15 : 120 (d) 16 : 128
46. (a) 37 (b) 45 (c) 49 (d) 65 55. (a) 12 : 72 (b) 24 : 36
47. (a) 35 (b) 49 (c) 50 (d) 63 (c) 60 : 74 (d) 84 : 96
48. (a) 120 (b) 168 56. (a) 18 : 108 (b) 23 : 138
(c) 290 (d) 380 (c) 20 : 80 (d) 26 : 156
49. (a) 126 (b) 217 57. (a) 62 : 37 (b) 74 : 40
(c) 345 (d) 513 (c) 85 : 60 (d) 103 : 78
50. (a) 13 (b) 17 58. (a) 34 : 12 (b) 43 : 30
(c) 23 (d) 37 (c) 52 : 21 (d) 62 : 19
51. (a) 4867 (b) 5555 59. (a) 7 : 18 (b) 9 : 26
(c) 6243 (d) 6655 (c) 11 : 36 (d) 13 : 42
52. (a) 272 (b) 210 60. (a) 133 : 98 (b) 150 : 115
(c) 240 (d) 304 (c) 182 : 140 (d) 188 : 153

ANSWERS
1. (d) 2. (d) 3. (d) 4. (d) 5. (d) 6. (b) 7. (a) 8. (d) 9. (b) 10. (d)
11. (a) 12. (d) 13. (b) 14. (d) 15. (b) 16. (b) 17. (b) 18. (c) 19. (a) 20. (d)
21. (d) 22. (b) 23. (a) 24. (c) 25. (b) 26. (a) 27. (c) 28. (c) 29. (d) 30. (d)
31. (b) 32. (d) 33. (d) 34. (b) 35. (d) 36. (c) 37. (d) 38. (b) 39. (d) 40. (d)
41. (d) 42. (c) 43. (a) 44. (b) 45. (d) 46. (c) 47. (c) 48. (d) 49. (c) 50. (c)
51. (d) 52. (c) 53. (b) 54. (b) 55. (c) 56. (c) 57. (b) 58. (d) 59. (c) 60. (c)

Hints & Solutions


1. (d) Orange is the only citrus fruit in the 11. (a) Except feeling, all others denote a collection of pages, a word is a collection of
group. various types of feelings. letters and a class is a collection of students.
20. (d) In all pairs except Genuine : Real, the
2. (d) Except chess, all others are outdoor 12. (d) Only brass is an alloy and all others
two words are antonyms of each other.
games. are metal. Hence, brass is different.
21. (d) The major components of air are
3. (d) Except whale, all others lay eggs. 13. (b) Except silicon valley all others real Nitrogen & Oxygen, of match stick are Sulphur
places, while bangalore is known as silicon & Phosphorous and of water are Hydrogen &
4. (d) All except white, are the colours of the valley.
rainbow. Oxygen. But Stainless Steel does not contain
14. (d) All except Blacksmith, are tools. Magnesium & Silver.
5. (d) All except chalk, are obtained from plants.
22. (b) In all other except Talent : Education,
6. (b) All except Mustard, are food grains 15. (b) All except Chilli, are modified stems.
lack of first causes the second.
while Mustard is an oilseed. 16. (b) All except Nitrogen, are used as 23. (a) In all pairs except Sky : Cloud, the two
fertilisers. words denote things which serve the same
7. (a) Except Rashtrapati Bhavan, all others
are declared as word heritage site by 17. (b) In all pairs except Iron : Axe, first is a purpose.
UNESCO. product made from the second. 24. (c) In all pairs except Chair : Sofa, second
8. (d) All except kilometers are units to is a part of the first.
18. (c) Saturn is a planet, sun is a star and
measure weights. titan is a satellite, but milky way is not a 25. (b) In all pairs except Boy : Girl, second
constellation it is a galaxy. denotes the class to which the first belongs.
9. (b) All except Hammer, are sharp-edged
and have a cutting action. 19. (a) Except (a), in all other pairs second 26. (a) Except Pen-Ink in all other, the second
word is the collection of first i.e. a note book is is kept in the first
10. (d) All except Minister, are chessmen.

https://sscstudy.com/
https://sscstudy.com/

12 CUET (UG) Section III : General Test

27. (c) In all pairs except Leaf-Flower, the 40. (d) Here, 51. (d) Here,
second is contained in the first. +1 +1 4867 ⇒ 4 + 8 + 6 + 7 = 25 i.e divisible by 5
28. (c) The holy book of Jainism is Agam 5555 ⇒ 5 + 5 + 5 + 5 = 20 i.e divisible by 5
Literature. Hence, Jainism : Upanished, is odd A Y B Z B X C Y
6243 ⇒ 6 + 2 + 4 + 3 = 15 i.e divisible 5
one out.
+1 +1 6655 ⇒ 6 + 6 + 5 + 5 = 22 i.e not divisible by 5
29. (d) All letters except Y are vowels. +1 –2 All except 6655 follow the same pattern.
Hence, 6655 is the odd one.
30. (d) Here, D V E W M P O N
−2
G → −2
E, M → −2
K, W → U, Q → N
−3 52. (c) Here,
+1 +2
272 ⇒ 2 + 7 + 2 = 11 i.e prime number
All except QN follow the same pattern.
All except MPON follow the same pattern. 210 ⇒ 2 + 1 + 0 = 3 i.e prime number
So, QN is the odd one.
240 ⇒ 2 + 4 + 0 = 6 i.e composite number
So, MPON is the odd one.
31. (b) Except AI, all other groups contain two 304 ⇒ 3 + 0 + 4 = 7 i.e prime number
consecutive vowels of the English alphabet. 41. (d) Here, number of alphabets between All except 240 follow the same pattern.
+1 +2 +1 +2 alternate letters is similar Hence, 240 is the odd one.
32. (d) Here, A → B → D, F → G → I,
+1 +2 +1 +1 –1 +2
L → M → O, S → T → U 53. (b) In all other pairs, second number is
C P B O B O D Q 9 more than the first except 55 : 62.
all except STU follow the same pattern. So,
STU is the odd one –1 +2 54. (b) Here,
+2 opposite
33. (d) Here, B → D ←→ W, +7 +1
12 : 96 13 : 117
+2 opposite
D → F ←→ U, A N H U E S G T ×8 ×9
+2 opposite
F → H ←→ S, +7 +2 15 : 120 16 : 128
+2
G → I → Q
Hence, ESGT is the odd one. ×8 ×8
All excpet GIQ follow the same pattern. So,
GIQ is the odd one. 42. (c) Here, all except 13 : 117 follows the same pattern.
+4 +2 +5 +2 +1 +1 Hence, 13 : 117 is the odd one
34.(b) Here, V ← R → T, R ← M → P, P R S Q U W X V
+4 +2 +2 +4 55. (c) The HCF of two numbers in all other
Y ← U → W, F ← B → D +1 +1
pairs is 12 except 60 : 74.
All except RMP follow the same pattern. So, L O
–1
N M C E
+1
F D
RMP is the odd one. 56. (c) In all pairs except 20 : 80, second
+1 +1
number is 6 times the first.
+1 −3 +1 −3
35. (d) Here, D → E → B, H → I → F,
57. (b) In all pairs except 74 : 40, the
+1 +2 −3−4
N → O → L , R → T → P All except LONM follow the same pattern. difference between the two numbers is 25.
Hence, LONM is the odd one.
All except RTP follow the same pattern. So, 58. (d) Here
RTP is the odd one. 43. (a) Except MONJK, in all other alternatives
34 : 12 ⇒ 3 + 4 + 1 + 2 = 10
+4 +4
atleast one letter is repeated.
+2 +2
36. (c) Here, H → J → N, J → L → P, 43 : 30 ⇒ 4 + 3 + 3 + 0 = 10
44. (b) Each of the numbers except 27, is a
+2 +3
+2 +4 52 : 21 ⇒ 5 + 2 + 2 + 1 = 10
P → R → U, Q → S → W prime number.
62 : 19 ⇒ 6 + 2 + 1 + 9 = 18
All except PRU follow the same pattern. So, 45. (d) 125 is the only number in the group
All except 62 : 19 follow the same pattern.
PRU is the odd one. which is a perfect cube.
Hence, 62 : 19 is the odd one.
37. (d) HEG is the only group of letters that 46. (c) 49 is the only perfect square in the
contains a vowel while others consists group. 59. (c)
consonants only. : 18 :
47. (c) Each of the numbers except 50 is 7 9 26
38. (b) OUT is the only group of letters that divisible by 7.
(7×4)–10 (9×4)–10
contains two vowels, while other consists one 48. (d) Each of the numbers except 380, is
vowel. 13 : 42 11 : 36
either one less or one more than the square of
+4 −2 a certain number.
39. (d) Here, D → H → F, (13×4)–10 (11×4)–8
+4 −2 49. (c) Each of the numbers except 345, is
K → O → M,
+4 −2 one more than the cube of a certain number. All except 11 : 36 follow the same patter.
R → V → T, Hence, 11:36 is the odd one.
+2 −1 50. (c) Each of the given numbers is a prime
W → Y → X 60. (c) In all pairs except 182 : 140, first
number. But the number obtained on reversing
All except WYX follow the same pattern. So, the digits of each of the numbers except 23, is number is 35 more than the second.
WYX is the odd one also a prime number.

https://sscstudy.com/
https://sscstudy.com/

GENERAL MENTAL ABILITY 13

CHAPTER 03

Series
A series is a sequence of letters or numbers or a 2. Letter Series
combination of both obtained by some particular In this type, a series of letters, either single or in groups
predefined rule. The candidate is required to study the is given. The terms of the series form a definite pattern
given series, identify the pattern followed and complete or sequence as regards the positions of letters in the
the given series with the most suitable alternative or English alphabet. The candidate is required to find out
find the wrong term in the series. that pattern and apply it to find the missing term.
The following types of questions are generally asked in Ex. 4 Find the next term in the series
exams related to series.
A, C, F, J, O, ?
(a) T (b) U (c) S (d) V
1. Number Series Sol. (b) The pattern of the series is,
In these questions, a number series is given following a A C F J O U
particular sequence or rule. The candidate has to either
complete the series or find the wrong term given at a +2 +3 +4 +5 +6
specific place in the series. Ex. 5 Find the next term in the series
Ex. 1 Find the missing term in the series AIQ, BJR, CKS, DLT, ?
5, 10, 15, 20, 25, ? (a) ENU (b) EMV
(a) 30 (b) 40 (c) ENV (d) EMU
(c) 35 (d) 45 Sol. (d) The pattern of the series is,
Sol. (a) The pattern of the series is, +1 +1 +1 +1
A B C D E
5 10 15 20 25 30 +1 +1 +1 +1
I J K L M
+1 +1 +1 +1
+5 +5 +5 +5 +5 Q R S T U
Ex. 2 Find the missing (?) in the series
Ex. 6 Find the next term in the series
3, 7, 13, 21, 31, ?
(a) 41 (b) 42 (c) 43 (d) 44 BMO, EOQ, HQS, ?
Sol. (c) The pattern of the series is, (a) KSU (b) LMN
(c) SOV (d) SOW
3 7 13 21 31 43
Sol. (a) The pattern of the series is,
+4 +6 +8 +10 +12 +3 +3 +3
B E H K
+2 +2 +2 +2
+2 +2 +2
M O Q S
Ex. 3 Find the wrong term in the series
+2 +2 +2
4, 12, 30, 68, 146, 302, 622 O Q S U
(a) 12 (b) 30 (c) 146 (d) 302
Sol. (d) The pattern of the series is, 3. Alpha Numeric Series
304
This series is a combination of letter series and number
4 12 30 68 146 302 622 series which move according to a set pattern. The
×2+4 ×2+6 ×2+8 ×2+10 ×2+12 ×2+14
candidate has to find the pattern for both to get the next
term of the series

https://sscstudy.com/
https://sscstudy.com/

14 CUET (UG) Section III : General Test

Ex. 7 Find the next term in the series 5. Conditional Letter, Number
3 F, 6G, 11 I, 18 L ?
(a) 21 O (b) 25 N
and Symbol Sequence
(c) 25 P (d) 27 P In this type of questions, a jumbled sequence of some
Sol. (d) The pattern of the series is, letters, numbers and / or symbols are given. The
+3 +5 +7 +9 candidate is required to find the total number of a
3 6 11 18 27
particular number / letter/ symbol in the sequence
+1 +2 +3 +4
F G I L P applying certain condition.
Ex. 8 Choose the missing term. Ex. 11 How many such symbols are there in the
P 3 C, R 5 F, T 8 I, V 12 L, ? following arrangement each of which is immediately
(a) X 17 O (b) L 12 S preceded by a number but not immediately followed
(c) X 12 T (d) I 17 O by a consonant?
Sol. (a) The pattern of the series is,
B 5 R 1 @ EK 4 F 7  D A M 2 P 3 % 9 H I W 8 * 6 U I $ V Q
+2 +2 +2 +2
P R T V X #
+2 +3 +4 +5 (a) None (b) One (c) Two (d) Three
3 5 8 12 17
+3 +3 +3 +3 Sol. (d) Let us see
C F I L O
✗ ✗
4. Continuous Pattern Series
B5R1@EK4F7©DAM2P3%9HIW8★6UJ$VQ#
In this type, a group of letters usually given in small
letters are repeated in a systematic way to establish a
series. However, some letters are missing from the ✓ ✓ ✓
series. These missing terms are then given in a proper ✓ = Condition fulfilled ✗ = Condition not fulfilled
sequence as one of the alternatives. The candidate is Clearly, there are three such symbols.
required to choose this alternative as the answer.
Ex. 12 How many 7’s immediately preceded by 6 but
Ex. 9 Find the missing letters in the series
not immediately followed by 4 are there in the
mc __ m __ a __ ca __ ca __ c __ mc following series?
(a) acmmma
(b) camcam 74276436753578437672406743
(c) aaccmm (a) One (b) Two (c) Four (d) Six
(d) acmmca Sol. (b) Let us see
Sol. (a) The series is m c a / m c a / m c a / m c a / m c a / ✗ ✗ ✗ ✗
mc.
So, the missing letters are ‘acmmma'. 74276436753578437672406743
Ex. 10 Find the missing letters in the series
ba–cb–b–bab– ✓ ✓
(a) acbb (b) bacc ✓ = Condition fulfilled
(c) bcaa (d) cabb
✗ = Condition not fulfilled
Sol. (b) The series is babc / babc / babc. So, the missing
Clearly, there are two such 7’s
letters are ‘bacc’.

https://sscstudy.com/
https://sscstudy.com/

GENERAL MENTAL ABILITY 15

Practice Questions
Directions (Q.Nos. 1–10) In the following questions, a 18. 1, 5, 5, 9, 7, 11, 11, 15, 12, 17
number series is given with one term missing. Choose the (a) 11 (b) 12
correct alternative that will continue the pattern. (c) 17 (d) 15
1. 6, 9, 12, 15, 18, ? Directions (Q. Nos. 19-28) In the following questions,
(a) 21 (b) 20 (c) 19 (d) 22 the terms of an alphabet series are given with one or more
2. 13, 14, 18, 27, ?, 68, 104 terms missing. Choose the missing terms.
(a) 36 (b) 41 (c) 43 (d) 54 19. T, R, P, N, L, ?, ?
(a) J, G (b) J, H
3. 6, 11, 21, 36, 56, ?
(c) K, H (d) K, I
(a) 42 (b) 51 (c) 81 (d) 91
20. H, I, K, N, ?
4. 1, 1, 8, 4, 27, 9, ?, 16
(a) O (b) Q
(a) 32 (b) 48
(c) R (d) S
(c) 64 (d) 72
21. A, I, P, V, A, E, ?
5. 5, 11, 23, 47, 95, ?
(a) E (b) F
(a) 190 (b) 191 (c) 161 (d) 169
(c) G (d) H
6. 10, 14, 28, 32, 64, 68, ?
22. E, J, ?, T, Y, D
(a) 132 (b) 72 (c) 136 (d) 86
(a) B (b) O (c) F (d) J
7. 325, 259, 204, 160, 127, 105, ?
23. AI, BJ, CK, ?
(a) 94 (b) 96 (c) 98 (d) 100
(a) DL (b) DM (c) GH (d) LM
8. 82, 70, 76, 64, 70, 58, ?
24. AZ, CX, FU, ?
(a) 52 (b) 76 (c) 64 (d) 48
(a) IR (b) IV (c) JQ (d) KP
9. 1, 5, 14, 30, 55, 91, ?
25. PMT, OOS, NQR, MSQ, ?
(a) 130 (b) 140 (c) 150 (d) 160
(a) LUP (b) LVP (c) LVR (d) LWP
10. 3, 15, 4, 16, 5, 17, 6, ?, 7
26. BCA, FGE, ?, NOM, RSQ, VWU
(a) 12 (b) 13 (c) 15 (d) 18
(a) IJH (b) KLJ
Directions (Q.Nos. 11–18) In the following questions, (c) KJI (d) JKI
one term in the number series is wrong. Find out the wrong 27. ADGJ, CFIL, EHKN, ?
term. (a) FILO (b) HKNQ
11. 3, 10, 27, 4, 16, 64, 5, 25, 125 (c) DGJM (d) GJMP
(a) 3 (b) 4 (c) 10 (d) 27
28. CBDA, GFHE, KJLI, ?
12. 5, 27, 61, 122, 213, 340, 509 (a) NOPM (b) MNOP
(a) 27 (b) 61 (c) PMNO (d) ONPM
(c) 122 (d) 509
Directions (Q.Nos. 29-36) In the following questions, a
13. 4, 10, 22, 46, 96, 190, 382 alpha-numeric series is given with one or more terms
(a) 4 (b) 10 (c) 96 (d) 382 missing. Choose the missing term.
14. 125, 126, 124, 127, 123, 129 29. 2B, 4C, 8E, 14H, ?
(a) 126 (b) 124 (c) 123 (d) 129 (a) 16 K (b) 20 I (c) 20 L (d) 22L

15. 15, 16, 22, 29, 45, 70 30. 2, A, 9, B, 6, C, 13, D, ?


(a) 16 (b) 22 (a) 9 (b) 10
(c) 45 (d) 70 (c) 12 (d) 19

16. 2, 6, 24, 96, 285, 568, 567 31. KM5, IP8, GS11, EV14,?
(a) BX17 (b) BY17
(a) 6 (b) 24
(c) CY17 (d) CY18
(c) 285 (d) 567

17. 1, 3, 12, 25, 48 32. C4X, F9U, I16R, ?


(a) K 25 P (b) L 25 P
(a) 3 (b) 12
(c) L 25 O (d) L 27 P
(c) 25 (d) 48

https://sscstudy.com/
https://sscstudy.com/

16 CUET (UG) Section III : General Test

33. 2 Z 5, 7 Y 7, 14 X 9, 23 W 11, 34 V 13, ? 45. a _ bccb _ ca _ cca _ baab_c


(a) 27 U 24 (b) 45 U 15 (a) ababc (b) abcaa
(c) 47 U 15 (d) 47 V 14 (c) accab (d) bacaa

34. N 5 V, K 7 T, ?, E 14 P, B 19 N 46. How many 4’s are there in the series which comes
(a) H 9 R (b) H 10 Q between two 5’s ?
(c) H 10 R (d) I 10 R 344545421454574545
(a) 4 (b) 5
35. Q1F, S 2 E, U 6 D, W 21 C, ? (c) 6 (d) 3
(a) Y 44 B (b) Y 66 B
(c) Y 88 B (d) Z 88 B 47. How many 7’s are there in the given series which are
immediately preceded by 6 but not immediately
36. 2 A 11, 4 D 13, 12 G 17, ? followed by 8?
(a) 36 I 19 (b) 36 J 21 348761567849675
(c) 48 J 21 (d) 48 J 23 (a) 1 (b) 2
Directions (Q.Nos. 37-45) In the following letter series, (c) 3 (d) 4
some letters are missing which are given in that order as 48. In the following series of numbers, find out how many
one of the alternatives. Choose the correct alternative. times 1, 3 and 7 have appeared together, 7 being in
37. ab _ _ baa _ _ ab _ the middle and 1 and 3 on either side of 7?
(a) aaaaa (b) aabaa 2931737771331738
(c) aabab (d) baabb 571377173906
(a) 3 (b) 4
38. a _ ba _ b _ b _ a _b (c) 5 (d) None of these
(a) abaab (b) abbab
(c) aabba (d) bbabb 49. How many even numbers are there in the following
sequence of numbers each of which is immediately
39. bca _ b _ aabc _ a _ caa followed by an odd number as well as immediately
(a) acab (b) bcbb preceded by an even number?
(c) cbab (d) ccab 86768932753422355228119
(a) One (b) Three
40. ab _ d _ aaba _ na _ bad na _ b (c) Five (d) None of these
(a) andaa (b) babda
(c) badna (d) dbanb 50. How many times n comes before p in the given series?
mstnpzxnpnpqynpranpst
41. a _ bbc _ aab _ cca _ bbcc (a) 2 (b) 3
(a) bacb (b) acba (c) 4 (d) 5
(c) abba (d) caba
51. In the following series, how many Ks are there which
42. _ a _ b _ abaa _ bab _ abb are immediately preceded by N and immediately
(a) aaabb (b) ababb followed by U?
(c) babab (d) babba ABCDKNLJMNKSTRZNKUANKUBWXNKLS
(a) 6 (b) 2 (c) 3 (d) 4
43. ac_cab_baca_aaa_aba
(a) aabc (b) aacb 52. Which of the following is the 10th to the right of the
(c) babb (d) bcbb 19th from the right end of the below arrangement?
F4T2E%MP5W9@LQR6UH3Z7*AT B8
44. abca_bcaab_ca_bbc_a
V#G$YD
(a) ccaa (b) bbaa
(a) M (b) T (c) # (d) 2
(c) abac (d) abba

ANSWERS
1. (a) 2. (c) 3. (c) 4. (c) 5. (b) 6. (c) 7. (a) 8. (c) 9. (b) 10. (d)
11. (c) 12. (a) 13. (c) 14. (d) 15. (b) 16. (b) 17. (c) 18. (b) 19. (b) 20. (c)
21. (d) 22. (b) 23. (a) 24. (c) 25. (a) 26. (d) 27. (d) 28. (d) 29. (d) 30. (b)
31. (c) 32. (c) 33. (c) 34. (c) 35. (c) 36. (d) 37. (b) 38. (d) 39. (a) 40. (a)
41. (b) 42. (d) 43. (a) 44. (c) 45. (a) 46. (d) 47. (a) 48. (a) 49. (d) 50. (d)
51. (b) 52. (b)

https://sscstudy.com/
https://sscstudy.com/

Hints & Solutions


1. (a) The pattern of the series is, 11. (c) The correct sequence is 19. (b) The pattern of the series is,
6 9 12 15 18 21 3, 3 2, 3 3, 4, 4 2, 4 3, 5, 5 2, 5 3 –2 –2 –2 –2
T R P N L
2
+3 +3 +3 +3 +3 So, 10 is wrong and must be replaced by 3 –2 –2
i.e. 9. J H
2. (c) The pattern of the series is,
12. (a) The pattern of the series is, 20. (c) The pattern of the series is,
13 14 18 27 43 68 104
24 +1 +2 +3 +4
+1 +4 +9 +16 +25 +36
H I K N R
5 27 61 122 213 340 509
21. (d) The pattern of the series is,
(1)2 (2)2 (3)2 (4)2 (5)2 (6)2 +8 +7 +6 +5
23–3 33–3 43–3 53–3 63–3 73–3 83–3 A I P V
3. (c) The pattern of the series is, +4 +3
So, 27 is the wrong term. A E H
6 11 21 36 56 81
13. (c) The pattern of the series is, 22. (b) The pattern of the series is,
+5 +10 +15 +20 +25 94 E
+5
J
+5
O
+5
T
+5
Y
+5
D
+5 +5 +5 +5 4 10 22 46 96 190 382 23. (a) The pattern of the series is,
+1 +1 +1
4. (c) The pattern of the series is, +6 +12 +24 +48 +96 +192 A B C D
1 1 8 4 27 9 64 16 +1 +1 +1
So, 96 is the wrong term. I J K L

13 12 23 22 33 32 43 42 14. (d) The pattern of the series is, 24. (c) The pattern of the series is,
+1 +1 +2 +3 +4
5. (b) The pattern of the series is, A C F J
125 126 124 127 123 129 –2 –3 –4
5 11 23 47 95 191 Z X U Q
–1 –1 128
×2+1 ×2+1 ×2+1 ×2+1 ×2+1 25. (a) The pattern of the series is,
So, 129 is the wrong term. –1 –1 –1 –1
6. (c) The pattern of the series is, P O N M L
15. (b) The pattern of the series is, +2 +2 +2 +2
10 14 28 32 64 68 136 20 M O Q S U
–1 –1 –1 –1
+4 ×2 +4 ×2 +4 ×2 15 16 22 29 45 70 T S R Q P

7. (a) The pattern of the series is, +1 +4 +9 +16 +25 26. (d) The pattern of the series is,
+4 +4 +4 +4 +4
325 259 204 160 127 105 94 12 22 32 42 52 B F J N R V
So, 22 is the wrong term. C
+4 +4
K
+4
O
+4
S
+4
W
G
–66 –55 –44 –33 –22 –11
+4 +4 +4 +4 +4
16. (b) The pattern of the series is, A E I M Q U
+11 +11 +11 +11 +11 25
27. (d) The pattern of the series is,
8. (c) The pattern of the series is, 2 6 24 96 285 568 567
+2 +2 +2
A C E G
82 70 76 64 70 58 64 ×6–6 ×5–5 ×4–4 ×3–3 ×2–2 ×1–1 +2 +2 +2
D F H J
–12 +6 –12 +6 –12 +6 So, 24 is the wrong term. +2 +2 +2
G I K M
+2 +2 +2
9. (b) The pattern of the series is, 17. (c) The pattern of the series is, J L N P
27
1 5 14 30 55 91 140 28. (d) The pattern of the series is,
1 3 12 25 48 +4 +4 +4
+4 +9 +16 +25 +36 +49
C G K O
12–02 22–12 42–22 62–32 82–42 +4 +4 +4
B F J N
22 32 42 52 62 72 So, 25 is the wrong term. D
+4
H
+4
L
+4
P
+4 +4 +4
10. ((a)) The pattern of the series is, 18. (b) The pattern of the series is, A E I M
+4 +2 +4 +2
+1 +1 +1
13 29. (d) The pattern of the series is,
1 5 5 9 7 11 11 15 12 17 +2 +4 +6 +8
3 15 4 16 5 17 6 18 7 2 4 8 14 22
+1 +2 +3 +4
+4 +2 +4 +2 B C E H L
+1 +1 +1 +1
So, 12 is the wrong term.

https://sscstudy.com/
https://sscstudy.com/

18 CUET (UG) Section III : General Test

30. (b) The pattern of the series is, 38. (d) The series is a b b /a b b / a b b / a b b. 48. (a) Clearly, the given number series is as
+1 +1 +1 Thus, the pattern ‘abb’ is repeated. follows

39. (a) The series is b c a a / b c a a / b c a a/ 293173777133173857


2 A 9 B 6 C 13 D 10 1377173906
b c a a. Thus, the pattern ‘bcaa’ is repeated.
+7 –3 +7 –3 ∴Required pattern
40. (a) The series is abadna / abadna/
1/ 3 7 1/ 3
abadna / ab. Thus, the pattern ‘abadna’ is =
31. (c) The pattern of the series is, Preceding middle following
–2 –2 –2 –2 repeated.
K I G E C Therefore, total number of such patterns = 3
+3 +3 +3 +3 41. (b) The series is aabbcc / aabbcc / aa
M P S V Y bbcc. Thus, the pattern ‘aabbcc’ is repeated. 49. (d) Clearly, the given sequence of
+3 +3 +3 +3
5 8 11 14 17 numbers is as follows
42. (d) The series is baa / bba/ baa / bba /
baa / bb. Thus, the pattern baa/bba is 8676 8 93275342 2 35522 8 119
32. (c) The pattern of the series is,
repeated. Required pattern
+3 +3 +3
C F I L
43. (a) The series is Even number Even number Odd number
=
4(2)2 9(32) 16(42) 25 52 Preceded Middle Followed
–3 –3 –3
a c a / c a b / a b a/ c a b / a a a / c a b / a.
So, total number of such even numbers
X U R O
44. (c) The series is = Four
33. (c) The pattern of the series is, abc /aabc/aabbc/aabbc/a.
50. (d) m s t n p z x n p n p q y n p r a n p s t
+5 +7 +9 +11 +13
2 7 14 23 34 47 45. (a) The series is aabcc / bbcaa / ccabb Clearly, required number of n = 5 times
–1 –1 –1 –1 –1
Z Y X W V U /aabcc.
+2 +2 +2 +2 +2 51. (b)
5 7 9 11 13 15 Thus, the pattern move in a cyclic order and
✗ ✗ ✗
in each group, the middle letter occurs only
34. (c) The pattern of the series is, once.
–3 –3 –3 –3 ABCDKNLJMNKSTRZNKUANKUBWXNKLS
N K H E B 46. (d)
+2 +3 +4 +5 ✗ ✗ ✗ ✗ ✗ ✓ ✓
5 7 10 14 19
.P = Condition fulfilled
–2 –2 –2 –2 ✗ = Condition not fulfilled
V T R P N 3 4 4 5 4 5 4 2 1 4 5 4 5 7 4 5 4 5
∴Required number of K = 2 times
35. (c) The pattern of the series is, ✓ ✓ ✓
+2 +2 +2 +2
52. (b) 1 2 3 4 5 6 7 8 9 10 11 12 13 14
Q S U W Y .P = Condition fulfilled F 4  T 2E%MP 5 W 9 @ L
×1+1 ×2+2 ×3+3 ×4+4 ✗ = Condition not fulfilled 15 16 17 18 19 20 21 22 23 24 25 26
1 2 6 21 88
∴ Required 4 = 3 times Q R 6 U H 3 Z 7 * A T B
–1 –1 –1 –1
F E D C B 27 28 29 30 31 32 33
47. (a) 8 V # G $ Y D
36. (d) The pattern of the series is, ✗ ✗
Total terms = 33
×2 ×3 ×4
2 4 12 48 ∴19th term from right = ( 33 + 1 − 19)
+3 +3 +3 3 4 8 7 6 1 5 6 7 8 4 9 6 7 5
A D G J = 34 − 19
+2 +4 +6 ✓ = 15 th from left = Q
11 13 17 23
P = Condition fulfilled ∴ 10th term to the right of 19th term
37. (b) The series is ab a / ab a / a b a / ab a. ✗ = Condition not fulfilled = 10th term to the right of Q
So, the missing letters are aabaa ∴ Required 7 = 1 time
= (15 + 10)th = 25th term from left = T

https://sscstudy.com/
https://sscstudy.com/

GENERAL MENTAL ABILITY 19

CHAPTER 04

Coding and Decoding


Coding is a method of transmitting information to Ex. 3 In a certain coding system, ‘SHEEP’ is written
someone using some suitable codes, so that it is not as ‘GAXXR’ and ‘BLEAT’ is written as ‘HPXTN’. How
understood by others. Decoding is the method of finding can ‘SLATE’ be written in that same coding system?
the actual meaning of those codes. (a) GPTNX (b) GPTXN
In questions based on coding-decoding, a word is coded (c) GPXN (d) PTGXN
in a particular way and the candidates are asked to code Sol. (a) In both the words ‘SHEEP’ and ‘BLEAT’, the
the other word in the same way. letter E is common and code for E is X. Hence, using
direct letter coding method, we have
There are mainly four types of questions which are asked.
S G and B H ∴ S G
H A L P
1. Letter Coding L P
E X E X A T
In this category, certain alphabets are coded as certain
E X A T T N
other alphabets. The candidate is required to
P R T N E X
understand the pattern and solve the problems based on
that pattern. Ex. 4 If in a certain code language ‘NOTION’ is
Ex. 1 In a certain code language ‘MIGHT’ is written written as ‘MLGRLM’, then how will ‘VECTOR’ be
as ‘GHMTI’, then how will ‘EARTH’ be written in that written in that language?
(a) EVXLGI (b) EVGXIL (c) EVXGIL (d) EVXGLI
code?
Sol. (d) As,
(a) RTEHA (b) RTEAH
(c) RTAEH (d) RETHA 14 15 20 9 15 14 13 12 7 18 12 13
N O T I O N M L G R L M
Sol. (a) As, Opposite letter
M I G H T G H M T I Opposite letter
1 2 3 4 5 3 4 1 5 2 Opposite letter
(position) (new position) Opposite letter
Opposite letter
Similarly, Opposite letter
E A R T H R T E H A
1 2 3 4 5 3 4 1 5 2
22 5 3 20 15 18 5 22 24 7 12 9
(position) (new position) Similarly, V E C T O R E V X G L I
Opposite letter
Ex. 2 In a certain code language ‘COLD’ is coded as Opposite letter
‘DPME’, then how will ‘NAME’ be written in that Opposite letter
Opposite letter
language?
Opposite letter
(a) OBME (b) OBNF (c) BOFN (d) EMAE Opposite letter
Sol. (b) As, C O L D
+1 +1 +1 +1
2. Number/Symbol Coding
D P M E
In this category, certain alphabets are coded as certain
Similarly, N A M E numbers /symbols. The candidate is required to
+1 +1 +1 +1 understand the pattern and solve the problem based on
that pattern.
O B N F

https://sscstudy.com/
https://sscstudy.com/

20 CUET (UG) Section III : General Test

Ex. 5 If in a certain code language ‘PUT’ is written 3. Substitution Coding


as ‘16, 21, 20’, then how will ‘BAT’ be written in that
language? In this category, a series of words is given and each word
(a) 2, 4, 20 (b) 5, 2, 11
of this series is substituted with another word. The
(c) 4, 5, 21 (d) 2, 1, 20 candidate is required to answer the question in the
Sol. (d) Here, each letter assigned its positioned value substituted code language.
in English alphabetical series. Ex. 9 If ‘Orange’ is called ‘Butter’, ‘Butter’ is called
PUT = 16, 21, 20 ‘Soap’, ‘Soap’ is called ‘Ink’ and ‘Ink’ is called ‘Red’,
and BAT = 2, 1, 20 then what is used for washing clothes?
(a) Red (b) Butter
Ex. 6 If X = 24 and BE = 7, then RING = ? (c) Ink (d) Soap
(a) 41 (b) 47
Sol. (c) We use ‘Soap’ to wash our clothes. But here
(c) 48 (d) 49 ‘Soap’ is called ‘Ink’. Therefore, ‘Ink’ is used for washing
Sol. (c) Here, each letter is coded as its position in the clothes.
English alphabetical order and then the word is coded as
the sum of the position of the alphabets in English 4. Message Coding
alphabetical series.
X = 24 (positional value in English alphabet) In this category, some messages are given in coded
BE = 2 + 5 = 7 language. The candidate is required to find the code for a
particular word or a message based on the information
Similarly, RING = 18 + 9 + 14 + 7 = 48 provided.
Ex. 7 In a certain code, the following numbers are Ex. 10 In a certain code language ‘he is good’ is
coded by assigning signs written as ‘pa ka na’ and ‘ she is brave’ is written as ‘ ra
ka da’ find the code for ‘is’.
1 2 3 4 5 6 7 8 9 (a) ka (b) na
< + = ↑ → > ≠ — (c) da (d) ra
Sol. (a)
Which number can be decoded from the given he is good pa ka na
symbols? she is brave ra ka da
→ ≠ > = <
Here ‘is’ is common in both the messages. Similarly, the
(a) 63181 (b) 68731 code ‘ka’ is common in the codes of both the messages.
(c) 62781 (d) 63118
So, ‘is’ → ka.
Sol. (b) According to the given table
→ ≠ > = <
Ex. 11 In a certain code language ‘go to school’ = 125,
‘study in school’ = 146 and ‘run to school’ = 135.
↓ ↓ ↓ ↓ ↓
Which digit is used for ‘run’?
6 8 7 3 1 (a) 6 (b) 2
(c) 3 (d) 1
Ex. 8 In RESEARCH is $#!#%$ & @, then SCARE is Sol. (c) We have,
(a) !&%$# (b) !@%$#
go to school 1 2 5 ...(i)
(c) !$%#& (d) !@%#$
Sol. (a) As, Similarly study in school 1 4 6 ...(ii)

R E S E A R C H S C A R E run to school 1 3 5 ...(iii)

Now, from Eq. (iii), to → 5, school → 1


$ # ! # % $ & @ ! & % $ # ∴ run → 3

https://sscstudy.com/
https://sscstudy.com/

GENERAL MENTAL ABILITY 21

Practice Questions
1. If ‘FISH’ is written as ‘EHRG in a certain code, then 12. If in a certain language, CONDEMN is written as
how would JUNGLE be written in that code? CNODMEN, then how will TEACHER be written in
(a) ITMFKD (b) ITNFKD that code?
(c) KVOHMF (d) TIMFKD (a) TAECEHR (b) TCAEHER
(c) TAECHER (d) TAEECHR
2. If SUMMER is coded as RUNNER, then code for
WINTER will be 13. In a certain code, DECEMBER is written as,
(a) SUITER (b) VIOUER ERMBCEDE. Which word will be written as
(c) WALKER (d) SUFFER ERMBVENO in that code?
(a) AUGUST (b) SEPTEMBER
3. If in a certain code, ‘BASIC’ is written as ‘DDULE’, (c) OCTOBER (d) NOVEMBER
then how is ‘LEADER’ written in that code?
(a) NGCFGT (b) NHCGGU 14. In a certain code, FIRE is written as QHOE and
(c) OGDFHT (d) OHDGHU MOVE as ZMWE. Following the same rule of coding,
what should be the code for the word OVER?
4. If the code for MOTHER is JRQKBU, then what is the (a) MWED (b) MWEO
code for PRINCIPAL? (c) MWOE (d) MWIO
(a) MRFKZLMXI (b) SULQFLSDO
15. In a certain code, STOVE is written as FNBLK, then
(c) MUFQZLMDI (d) MRFKZFMXI
how will VOTES be written in that code?
5. If ‘GOLD’ is written as ‘HOME’, ‘COME’ is coded as (a) FLKBN (b) LBNKF
‘DONE’ and ‘CORD’ is coded as ‘DOSE’, then how (c) LKNBF (d) LNBKF
would you code SONS? 16. If in a code language ‘PARENT’ is written as
(a) TPOT (b) TOOT
‘BDFGJK’ and ‘CHILDREN’ is written as
(c) TOOS (d) TONT
‘MOXQUFGJ’ then how is ‘REPRINT’ written in that
6. If EHFNRQ is the code for BECKON, then which same code?
word has the code QDFWXULQ? (a) FGBFXJK (b) FGBUXJK
(a) NCAUTIRN (b) NACUTIRN
(c) FGBFXGD (d) BGFXJK
(c) NATCRIUN (d) NACTURIN
17. In a code language ‘ORGANISATION’ is written as
7. In a certain code, SWITCH is written as TVJSDG. ‘CBDWLQJWYQCL’ and ‘OPERATION’ as
Which word would be written as CQFZE?
‘CXFBWYQCL’. How would ‘SEPERATION’ be coded?
(a) BARED (b) BRAED
(a) EJXEBYQCL (b) JFQYWBCXQL
(c) BREAD (d) BRADE
(c) JFXFBWYQCL (d) QCLYWBFXJE
8. If in a certain language, MACHINE is coded as
18. If the letters in PRABA are coded as 27595 and
LBBIHOD, then which word would be coded as
THILAK is coded as 368451, then how can BHARATI
SLTMFNB?
be coded?
(a) RKSLEMA (b) TKULGMC
(a) 9567568 (b) 9675538
(c) RMSNEOA (d) TMUNGOC
(c) 9657538 (d) 9567538
9. In a certain code, INACTIVE is coded as VITCANIE. 19. If NOIDA is written as 39658, then how will INDIA
How is COMPUTER written in the same code? be written?
(a) PMOCRETU (b) ETUPMOCR (a) 36568 (b) 63568
(c) UTEPMOCR (d) MOCPETUR (c) 63569 (d) 65368
10. In a certain code, KAVERI is coded as VAKIRE. How 20. If ‘FLARE’ is coded as 21, 15,26, 9, 22, then how
is MYSORE written in that code? would ‘BREIF’ be coded in the same language?
(a) EROSYM (b) SYMROE (a) 25, 9, 22, 21, 18 (b) 5, 37, 11, 19, 13
(c) SYMERO (d) None of these (c) 13, 19, 11, 37, 5 (d) 25, 9, 22, 18, 21

11. If SPIDER is written as PSDIRE in a certain code, 21. If ‘LINGER’ is ‘123456’ and ‘FORCE’ is ‘56789’ then
then how could COMMON be written in that code? ‘FIERCE’ will be
(a) OCOMMO (b) OCMMNO (a) 345667 (b) 456678
(c) OCMOMN (d) OCMMON (c) 345677 (d) Cannot be determined

https://sscstudy.com/
https://sscstudy.com/

22 CUET (UG) Section III : General Test

22. If B = 2, BAG = 10, then BOX = ? (a) Well (b) Island


(c) Sky (d) Air
(a) 36 (b) 39
(c) 41 (d) 52 33. ‘If ‘dust’ is called ‘air’, ‘air’ is called ‘fire’, ‘fire’ is called
‘water’, ‘water’ is called ‘colour’, ‘colour’ is called ‘rain’
23. If OWL = 50 and N = 14, then TIME is
and ‘rain’ is called ‘dust’, then where do fish live?
(a) 45 (b) 47
(a) Fire (b) Water
(c) 43 (d) 49
(c) Colour (d) Dust
24. If CAT =12, then MAN = ?
34. If the animals which can walk are called ‘swimmers’,
(a) 14 (b) 24
animals who crawl are called ‘flying’, those living in
(c) 16 (d) 15
water are called ‘snakes’ and those which fly in the
25. If ASHA equal 79, then VINAY BHUSHAN = ? sky are called ‘hunters’, then what will a lizard be
(a) 211 (b) 200 called?
(c) 144 (d) 180 (a) Swimmers (b) Snakes
(c) Flying (d) Hunters
26. If MOBILITY is coded as 46293927, then
EXAMINATION is coded as 35. If ‘blue’ is called ‘green’, ‘green’ is called ‘white’,
(a) 45038401854 (b) 56149512965 ‘white’ is called ‘red’, ‘red’ is called ‘black’, then what
(c) 57159413955 (d) 67250623076 is the colour of clear sky?
(a) Blue (b) Green
27. If ARC is written as $@* and HIT is #&%, then
(c) White (d) Black
CHAIR is
(a) #*&$@ (b) #*$&% 36. If ‘god is great’= ‘cp an bo’, ‘great help done’ = ‘er cp fs’
(c) *#$&@ (d) *#$&% and ‘he is great’ = ‘bo cp dq', then what represents ‘he
is god’?
28. In a certain code language ‘SAFER’ is written as
(a) cp er bo (b) an bo cp
‘5@3#2’ and ‘RIDE’ is written as ‘2©%#’ how would
(c) dq bo cp (d) an bo dq
‘FEDS’ be written in that code?
(a) 3 # © 5 (b) 3 @ % 5 37. If ‘ cinto baoli tsi nzro’ means ‘her village is
(c) 3 # % 5 (d) 3 # % 2 Sarurpur’, mhi cinto keepi tsi oind’ means ‘her first
love is literature’ and ‘oind geit tsi cinto pki’ means
29. If in a certain code language ‘STAR’ is written as ‘5 $
‘literature collection is her hobby’, which word would
* 2’, ‘TORE’ is written as $ 3 2 @, then how will
mean ‘literature’?
‘OATS’ be written in that language?
(a) cinto (b) baoli
(a) 3 * 5 $ (b) 3 * $ 5
(c) oind (d) geit
(c) 3 $ * 5 (d) 3 5 * $
30. If in a certain code language ‘GONE’ is written as ‘5 @ 38. In a certain code, 592 means ‘grapes are sweet’, 374
means ‘I like oranges’ and 267 means ‘oranges are
© 9’ and ‘’SEAL is written as ‘6 9 % *’, then how will
sour’. Which digit represents sour?
‘LOGS’ be written in that language?
(a) 9 (b) 5
(a) * 9 © 6 (b) * 9 © 6
(c) * @ 65 (d) * @ 56 (c) 6 (d) Cannot be determined

31. If ‘sky’ is ‘star’, ‘star’ is ‘cloud’, ‘cloud’ is ‘earth’, ‘earth’ 39. In a certain code language, ‘134’ means ‘good and tasty’,
is ‘tree’ and ‘tree’ is ‘book’ , then where do the birds ‘478’ means ‘see good pictures’ and ‘729’ means ‘pictures
fly? are faint’. Which of the following digits stands for ‘see’ ?
(a) Cloud (b) Sky (a) 9 (b) 2
(c) Star (d) None of these (c) 1 (d) 8

32. If ‘sand’ is called ‘air’ , ‘air’ is called ‘plateau’, plateau’ 40. If VOTER = 41352, HEATER = 743654, TEASER =
is called ‘well’, ‘well’ is called ‘island’ and ‘island’ is 645834, which number represents S?
called ‘sky’, then from where will a woman draw (a) 1 (b) 5
water? (c) 8 (d) 7

ANSWERS
1. (a) 2. (b) 3. (b) 4. (c) 5. (b) 6. (d) 7. (c) 8. (b) 9. (b) 10. (c)
11. (b) 12. (a) 13. (d) 14. (b) 15. (b) 16. (a) 17. (c) 18. (c) 19. (b) 20. (d)
21. (d) 22. (c) 23. (b) 24. (a) 25. (d) 26. (b) 27. (c) 28. (c) 29. (b) 30. (d)
31. (c) 32. (b) 33. (c) 34. (c) 35. (b) 36. (d) 37. (c) 38. (c) 39. (d) 40. (c)

https://sscstudy.com/
https://sscstudy.com/

GENERAL MENTAL ABILITY 23

Hints & Solutions


1. (a) As, F I S H 7. (c) As, S W I T C H Similarly, N O V E M B E R
–1 –1 –1 –1 +1 –1 +1 –1 +1 –1

E H R G T V J S D G
Similarly, J U N G L E E R M B V E N O
Similarly, B R E A D
–1 –1 –1 –1 –1
+1 –1 +1 –1 +1 14. (b) As, F I R E and M O V E
I T M F K D
C Q F Z E
S U M M E R Q H O E Z M W E
2. (b) As, 8. (b) As, M A C H I N E
–1 +1 +1 –1 +1 –1 +1 –1 +1 –1 Similarly, O V E R
R U N N E R L B B I H O D
Similarly, W I N T E R Similarly, T K U L G M C M W E O
–1 +1 +1 –1 +1 –1 +1 –1 +1 –1
15. (b) As, S T O V E
V I O U E R S L T M F N B
9. (b) As, F N B L K
3. (b) As, B A S I C
+2 +3 +2 +3 +2
I N A C T I V E
Similarly, V O T E S
D D U L E
V I T C A N I E
Similarly, L E A D E R L B N K F
Similarly,
+2 +3 +2 +3 +2 +3
C O M P U T E R 16. (a) As, P A R E N T
N H C G G U

4. (c) As, M O T H E R E T U P M O C R B D F G J K
–3 +3 –3 +3 –3 +3
C H I L D R E N
10. (c) As, K A V E R I
J R Q K B U and
Similarly, P R I N C I P A L M O X Q U F G J
V A K I R E
–3 +3 –3 +3 –3 +3 –3 +3 –3 Similarly, R E P R I N T
Similarly, M Y S O R E
M U F Q Z L M D I
F G B F X J K
5. (b) Here, each consonant is move(a) one S Y M E R O
step forward and each vowel remains 17. (c) As,
unchanged. I
11. (b) As, S P D E R O R G A N I S A T I O N
As, G O L D , C O M E
+1 +1 +1 +1 +1
P S D I R E C B D W L Q J W Y Q C L
H O M E D O N E Similarly, C O M M O N O P E R A T I O N
and C O R D and
+1 +1 +1
O C M M N O C X F B W Y Q C L
D O S E
12. (a) As, C O N D E M N Similarly, S E P E R A T I O N
Similarly, S O N S
+1 +1 +1
C N O D M E N J F X F B W Y Q C L
T O O T Similarly, T E A C H E R 18. (c) As,
6. (d) As, E H F N R Q
P R A B A and T H I L A K
–3 –3 –3 –3 –3 –3
T A E C E H R
B E C K O N
2 7 5 9 5 3 6 8 4 5 1
13. (d) As, D E C E M B E R
Similarly, Q D F W X U L Q Similarly, B H A R A T I
–3 –3 –3 –3 –3 –3 –3 –3

N A C T U R I N E R M B C E D E
9 6 5 7 5 3 8

https://sscstudy.com/
https://sscstudy.com/

24 CUET (UG) Section III : General Test

19. (b) As, N O I D A 25. (d) As, 30. (d) As,


A S H A G O N E and S E A L
3 9 6 5 8
Position in reverse ⇒ 26, 8, 19, 26 = 79 5 @ c 9 6 9 % *
Similarly, I N D I A English alphabet
Similarly,
Similarly, L O G S
6 3 5 6 8 V I N A Y B H U S HA N
Position
20. (d) Here, each letter is coded by its in reverse
* @ 5 6
position in reverse English alphabetical order English ⇒ 5+18+13+26+2+25+19+6+ 8+19+26+13
alphabet = 180 31. (c) Birds fly in the sky and here sky is
As, F L A R E called star. So, Birds fly in star.
Position in reverse English alphabet
32. (b) We know that, water is drawn from
21, 15, 26, 9, 22 26. (b) As, M O B I L I T Y
well and here well is called island. So, woman
Similarly, B R E I F will draw water from island.
13 15 2 9 12 9 20 25
33. (c) Fish lives in water and here water is
25, 9, 22, 18, 21 1+3 1+5 2 9 1+2 9 2+0 2+5 called colour. So, fish lives in colour.

21. (d) As, L I N G E R 4 6 2 9 3 9 2 7


34. (c) We know that lizard crawls and
Similarly, animals who crawl are called flying so, lizard is
called flying.
1 2 3 4 5 6 E X A M I N A T I O N

F O R C E 35. (b) Colour of clear sky is blue and here


5 24 1 13 9 14 1 20 9 15 14 blue is called green. So, colour of clear sky is
green.
5 6 7 8 9 5 2+4 1 1+3 9 1+4 1 2+0 9 1+5 1+4
36. (d) god is great cp an bo
Similarly, F → 5
5 6 1 4 9 5 1 2 9 6 5 great help done er cp fs
I → 2
he is great bo cp dq
E → 5 / 9 27. (c) As,
Here, he = dq, is = bo, god = an
R → 6 / 7 A R C and H I T
∴he is god = dq bo an ⇒ an bo dq
C → 8
E → 5 / 9 $ @ * # & %
37. (c)
Hence, code for ‘FIERCE’ cannot be her village is sarurpar cinto baoli tsi nzro
determined Similarly, C H A I R
her first love is literature mhi cinto keepi tsi oind

22. (c) As, B = 2 (positional value in English literature collection is her hobby oind geit tsi cinto pki

alphabet) * # $ & @
B A G
28. (c) As, ∴Code for literature is ‘oind’

2 +1 +7 = 10 S A F E R and R I D E 38. (c) 5 9 2 grapes are sweet

Similarly, B O 3 7 4 I Iike oranges


X
5 @ 3 # 2 2 © % # 2 6 7 oranges are sour
2+15+24 = 41 Similarly, F E D S ∴6 = sour

23. (b) N = 14 39. (d) good and tasty 1 3 4


(positional value in English alphabet) 3 # % 5
see good pictures 4 7 8
O W L = 15 + 23 + 12 = 50
Similarly, T I M E = 20 + 9 + 13 + 5 = 47
29. (b) As, pictures are faint 7 2 9
S T A R and T O R E
24. (a) As, C A T ∴Code for see ⇒ 8

5 $ * 2 $ 3 2 @ 40. (c) Given,


3 +1 +20 = 24, 24÷2=12
V O T E R = 4 1 3 5 2
Similarly, M A Similarly, O A T S
N H E A T E R= 7 4 3 6 5 4
T E A S E R= 6 4 5 8 3 4
13+1+14 = 28, 28÷2= 14 3 * $ 5
∴S = 8

https://sscstudy.com/
https://sscstudy.com/

GENERAL MENTAL ABILITY 25

CHAPTER 05

Alphabet Test and


Jumbling
As the name says, this test is related to the English Ex. 2 How many letters are there between 8th
alphabets. Questions asked in this test are based on the letter from left and 7th letter from right in the English
alphabetical order of letters, formation of words either alphabet?
meaningful or meaningless various arrangements of (a) 7 (b) 11
letters etc. to which the candidates have to be familiar (c) 8 (d) 9
with. Sol. (b) Here,
1 2 3 4 5 6 7 8 9 10 11 12 13 14 15 16 17 18 19 20 21 22 23 24 25 26
1. Finding a Particular Letter in A B C D E F G H I J K L M N O P Q R S T U V W X Y Z

English Alphabetical Order From left 11 letters From right


8th 7th
In this type of questions, the questions are asked to find
out the letter to the left or right of another letter in the So, there are 11 letters between 8th letter from left and
7th letter from right.
English alphabetical order. Questions in other format
may also be asked based on the arrangement of
alphabetical order. 2. Letter-Word Problems
In this type, a word is given and it is asked to find the
Important Rules different letter pairs having as many letters between
1. nth letter to the left of mth letter from right end them as in English alphabet. Also, in some questions, a
= (m + n)th letter from the right end. word is given and then asked how many letters remains
2. nth letter to the right of mth letter from left end = (m + n)th letter same at their position, if they are arranged in ascending
from the left end.
or descending order. Also, some questions are asked
3. nth letter to the left of mth letter from left end = (m − n)th letter
from the left end.
based on letter rearrangement.
4. nth letter to the right of mth letter from right end = (m − n)th letter Ex. 3 How many such pairs of letters are there in
from the right end. the word CORPORATE each of which has as many
letters between them in the word as in the English
Ex. 1 Find the 11th letter to the left of 20th letter alphabet both forward and backward?
from left in the English alphabet. (a) None (b) One
(a) D (b) J (c) Two (d) Four
(c) K (d) I
Sol. (d)
Sol. (d) Here, C O R P O R A T E
1 2 3 4 5 6 7 8 9 10 11 12 13 14 15 16 17 18 19 20 21 22 23 24 25 26
A B C D E F G H I J K L M N O P Q R S T U V W X Y Z
To the left There are four such pairs.
11th Ex. 4 If the first and third letters in the word
From left 20th NECESSARY were interchanged, also the fourth and
Hence, 11th letter to the left of 20th letter from left is I. sixth letters, and the seventh and the ninth letters,
Alternate Method which of the following would be the seventh letter
In English alphabet, 11th letter to the left of 20th letter from the left?
from left = (20 − 11)th letter from left = 9th letter from (a) A (b) Y
left = I (c) R (d) E

https://sscstudy.com/
https://sscstudy.com/

26 CUET (UG) Section III : General Test

Sol. (b) On interchanging the positions of the letters, Ex. 8 If it is possible to form a word with the first,
fourth, seventh and eleventh letters of the word
N E C E S S A R Y ‘SUPERFLUOUS’. Write the first letter of that word,
otherwise X is the answer.
The new letter sequence will be CENSSEYRA. (a) S (b) L
(c) O (d) E
So, the seventh letter from the left is Y.
Sol. (b) ‘LESS’ is the word which is formed by using
Ex. 5 If the letters in the word FATHER are written first, fourth, seventh and eleventh letters of the word
in alphabetical order, then how many letters will not ‘SUPERFLUOUS’. First letter of this word is ‘L’.
change their position?
(a) None (b) One Ex. 9 Choose one word out of the given alternatives,
(c) Two (d) Three which cannot be formed from the letters of the word
Sol. (b) ‘CONSULTATION’.
(a) CONSTANT (b) NATION
Word F A T H E R
(c) SALUTE (d) STATION
In alphabetical order A E F H R T
Sol. (c) Carefully looking at the words, we find that the
There is only one such alphabet. word ‘CONSULTATION’ does not contain the letter ‘E’.
So, the word ‘SALUTE’ cannot be formed.
3. Alphabetical Order of Words Ex. 10 Find the word which can be formed by using
Arranging words in alphabetical order means to arrange the letters of the given word — RATIONALISATION.
them in the order in which they appear in the (a) NATIONALIST (b) NATIONALISTICS
dictionary, i.e. according to the order of letters with (c) REALISATION (d) SITUATION
which they begin.
Sol. (a) From the given word, ‘NATIONALIST’ can be
To arrange the words as per dictionary follow the given formed because all the letters of this word are present in
steps. the given word.
(a) Take the first alphabet of each word and arrange
the words in the order in which these alphabets 5. Jumbling
appear in English alphabetical series.
In this type of questions, a set of English letters is given
(b) If more than one word begin with the same in a jumbled order. The candidate is required to arrange
alphabet the such words should be arranged in the these letters to form a meaningful word.
order of second alphabet in the word and so on
Note Always try to place the letters according to the numbers
Ex. 6 Arrange the words in the alphabetical order provided in options rather than doing it on the basis of your
and tick the one that comes at the third place. vocabulary knowledge.
(a) Rigour (b) Remove Ex. 11 Select the combination of numbers that forms
(c) Retrospect (d) Revive a meaningful word.
Sol. (d) The arrangement of words would be, Remove, P N O A C L M I
Retrospect, Revive, Rigour,. Clearly, Revive at the third
place. 1 2 3 4 5 6 7 8
Ex. 7 Arrange the given words according to English (a) 2, 7, 8, 6, 4, 3, 1, 5 (b) 4, 7, 5, 2, 6, 8, 1, 3
(c) 7, 1, 8, 5, 6, 2, 4, 3 (d) 5, 3, 7, 1, 6, 4, 8, 2
dictionary.
Sol. (d) C O M P L A I N
1. Episode 2. Epistle
3. Episcope 4. Epigraph 5 3 7 1 6 4 8 2
(a) 2, 3, 1, 4 (b) 4, 3, 1, 2 Clearly, the given letters, when arranged in the order of
(c) 3, 2, 1, 4 (d) 1, 2, 3, 4
‘5, 3, 7, 1, 6, 4, 8, 2,’ form the word ‘COMPLAIN’.
Sol. (b) The correct sequence of words would be
Epigraph, Episcope, Episode, Epistle i.e. 4, 3, 1, 2 Ex. 12 Rearrange the letters A, R, T, Y and D to
form a meaningful word and select from the given
4. Word Formation alternatives, the word which is almost opposite in
In this type, letters from some positions of a given word
meaning to the word so formed.
are selected and then arranged to form another (a) Dirty (b) Quiet
(c) Quick (d) Queek
meaningful word. Sometimes words are given in the
options and the candidate has to find out the word that Sol. (c) The word that can be formed by rearranging the
given letters is ‘Tardy’ which means ‘Sluggish’ and
can or cannot be formed from the given one.
opposite of this word is ‘Quick’.

https://sscstudy.com/
https://sscstudy.com/

GENERAL MENTAL ABILITY 27

Practice Questions
1. In the English alphabet, which letter is 15th from the 12. PRISON
right? (a) One (b) Two
(a) D (b) M (c) Three (d) More than three
(c) O (d) L
13. KLING
2. Which letter is exactly midway between G and Q in (a) Three (b) Four
English alphabet? (c) Five (d) Six
(a) K (b) L
(c) N (d) J 14. CHILDREN
(a) Two (b) Three
3. Which letter will be fifth to the right of letter which is (c) Four (d) Five
fourth to the left of I in English alphabet?
(a) O (b) N 15. PRODUCTION
(c) J (d) K (a) Nil (b) One
4. Which letters are exactly in the middle of the sixth (c) Two (d) Three
letter from the left and fourteenth letter from the 16. If the positions of the first and the sixth letters in the
right end in English alphabet? word DISTRIBUTE are interchanged. Similarly, the
(a) MN (b) IJ positions of the second and the seventh, the third and
(c) PM (d) OL the eighth and so on. Which of the following letters
5. Which letter will be the 3rd letter to the right of 19th will be the fifth from left after interchanging the
letter from left in the English alphabet? positions?
(a) X (b) U (a) E (b) I
(c) P (d) V (c) S (d) T

6. In the English alphabet, which letter will be 8th letter 17. If the positions of the third and tenth letters of the
to the left of 16th letter from your left? word DOCUMENTATION are interchanged, and
(a) H (b) X likewise, the positions of the fourth and seventh
(c) I (d) Y letters, the second and sixth letters is also
interchanged. Which of the following will be eleventh
7. If the order of the English alphabet is reversed, then
letter from the right end?
which letter would be exactly in the middle?
(a) C (b) I
(a) L (b) M
(c) T (d) U
(c) N (d) None of these

8. If the English alphabet is written in the reverse order,


Directions (Q. Nos. 18 and 19) If the given words are
which will be the fifth letter to the left of the ninth written in alphabetical order, then find the number of
letter from the right? letters that will not change their place.
(a) P (b) N 18. S T O N E D
(c) D (d) W (a) None (b) One
9. If every alternate letter starting from B is deleted (c) Two (d) Three
from the given alphabet, then which of the following 19. ENGLISH
will be the tenth letter from the right end? (a) One (b) Two
(a) G (b) D (c) Three (d) None
(c) Q (d) H
Directions (Q Nos. 20 and 21) Arrange the words in
10. If A interchanges position with B, similarly C and D
alphabetical order and tick the one that comes at the
interchange positions and so as E and F and so on
upto Y and Z, then which of the following will be second place.
seventh from the left? 20. (a) Plane (b) Plain
(a) A (b) F (c) L (d) H (c) Plenty (d) Player

Directions (Q. Nos. 11-15) How many pairs of letters are 21. (a) Cathedral (b) Catenation
there in the given word each of which has as many letters (c) Caterpillar (d) Category
between them in the word as in the English alphabet?
22. Which of the following words will come at the fourth
11. CREATIVE place according to the dictionary?
(a) One (b) Two (a) Converse (b) Current
(c) Three (d) Four (c) Curator (d) Cushion

https://sscstudy.com/
https://sscstudy.com/

28 CUET (UG) Section III : General Test

23. Which word will come at third place in the English 34. How many meaningful words can be formed from the
dictionary? letters ADRW, using each letter only once in each word?
(a) Radical (b) Radiate (a) None (b) One
(c) Racket (d) Radius (c) Two (d) Three

24. Arrange the following word in alphabetical order and 35. Rearrange the letters AYDOT to form a meaningful
find the third word? word. The last letter of this word is the answer.
Singer, Single, Sinister, Simple (a) A (b) T
(a) Single (b) Singer (c) Y (d) D
(c) Sinister (d) Simple
36. If the letters, ERVSECI can be rearranged to form a
Directions (Q. Nos. 25 and 26) Arrange the given words meaningful word what will be the fifth letter from the
as they occur in the dictionary and then choose the correct right?
(a) R (b) E
sequence.
(c) V (d) No word can be formed
25. 1. Select 2. Seldom 3. Send
37. If it is possible to form a word with the first, fourth,
4. Selfish 5. Seller seventh and eleventh letters of the word
(a) 1, 2, 4, 5, 3 (b) 2, 1, 5, 4, 3 INTERPRETATION, then which of the following will
(c) 2, 1, 4, 5, 3 (d) 2, 5, 4, 1, 3 be third letter of that word, otherwise X is your
26. 1. Protein 2. Problem 3. Proverb answer?
(a) T (b) R
4. Property 5. Project
(c) E (d) X
(a) 1, 2, 3, 4, 5 (b) 2, 1, 4, 3, 5
(c) 2, 5, 4, 1, 3 (d) 3, 4, 5, 2, 1 Directions (Q. Nos. 38-41) Letters of the words given
below have been jumbled up and you are required to
27. If the words in the sentence, ‘She showed several sample
snaps’ are rearranged in the alphabetical order, then construct the words. Each letter has been numbered and
what will be the middle word? each word is followed by four options. Choose the option
(a) Snaps (b) Sample
which gives the correct order of the letters as indicated by
(c) Several (d) She the numbers to form words.
38. 1 2 3 4 5 6
Directions (Q. Nos. 28-30) Find the word that can be
G I C O D N
formed from the letters of the given word.
(a) 2, 1, 4, 3, 6, 5 (b) 4, 3, 2, 6, 5, 1
28. PRAGMATIC (c) 6, 5, 2, 3, 1, 4 (d) 3, 4, 5, 2, 6, 1
(a) GUITAR (b) AGMARK
(c) GAME (d) MAGIC 39. V A R S T E
1 2 3 4 5 6
29. MEASUREMENT (a) 2, 3, 1, 6, 4, 5 (b) 3, 2, 4, 5, 6, 1
(a) MASTER (b) MANTLE (c) 4, 5, 2, 3, 1, 6 (d) 6, 3, 4, 5, 2, 1
(c) SUMMIT (d) ASSURE
40. E M I H T R
30. ENVIRONMENT 1 2 3 4 5 6
(a) MOVEMENT (b) ENTERTAIN (a) 1, 2, 3, 4, 5, 6 (b) 4, 1, 6, 2, 3, 5
(c) EMINENT (d) ENTRANCE (c) 5, 1, 6, 4, 3, 2 (d) 6, 1, 2, 3, 5, 4
Directions (Q.Nos. 31 and 32) From the given alternative 41. R T A N U E
words, select the word which cannot be formed using the 1 2 3 4 5 6
letters of given word. (a) 1, 3, 2, 6, 4, 5 (b) 3, 2, 4, 6, 1, 5
(c) 4, 3, 2, 5, 1, 6 (d) 4, 6, 5, 2, 3, 1
31. INFLATIONARY
(a) FLAIR (b) FAULTY 42. When we arrange the letters of the word ‘VRAKIE’ in
(c) NATIONAL (d) RATION correct order, we get the name of a river. Fifth letter
from left in that name, is
32. DETERMINATION (a) R (b) E
(a) NATION (b) DETERMINE (c) I (d) K
(c) TERMINATE (d) DIET
43. If a meaningful word can be formed by rearranging
33. How many meaningful three letter words can be the letters ‘USCALA’. The first letter of the word so
formed with the letters, W, N and O using each letter formed is, the answer. If no such word can be formed
only once in each word? the answer is X.
(a) None (b) One (a) C (b) S
(c) Two (d) Three (c) A (d) L

https://sscstudy.com/
https://sscstudy.com/

GENERAL MENTAL ABILITY 29

44. What is the last letter of the word which is formed by 46. DAXEPN
rearranging the following letters? (a) INCREASE (b) REDUCE
C P A O E C K (a bird) (c) STILL (d) DECREASE
(a) E (b) K 47. HRADTE
(c) P (d) C
(a) DECREASE
45. The letters of the word ‘NUMKIPP’ are disorder. If (b) LOSS
they are arranged in proper order, the name of a (c) REDUCTION (d) SCARCITY
vegetable is formed. What is the last letter of the word
48. DNHBEI
so formed?
(a) FRONT (b) SIDE
(a) K (b) M
(c) BACK (d) LAST
(c) N (d) P
49. Unscramble the letters ‘CCITRKE’ to form an English
Directions (Q. Nos. 46-48) In each of these questions,
word and find the fifth letter of the unscrambled
jumbled letters of a meaningful word are given. You are
word?
required to rearrange these letters and select from the given
(a) C (b) K
alternatives, the word which is almost similar in meaning
(c) E (d) I
to the rearranged word.

ANSWERS
1. (d) 2. (b) 3. (c) 4. (b) 5. (d) 6. (a) 7. (d) 8. (b) 9. (a) 10. (d)
11. (c) 12. (d) 13. (d) 14. (c) 15. (d) 16. (a) 17. (c) 18. (a) 19. (a) 20. (a)
21. (b) 22. (d) 23. (b) 24. (a) 25. (c) 26. (c) 27. (d) 28. (d) 29. (a) 30. (c)
31. (b) 32. (b) 33. (d) 34. (c) 35. (c) 36. (a) 37. (d) 38. (d) 39. (c) 40. (b)
41. (c) 42. (a) 43. (a) 44. (b) 45. (c) 46. (a) 47. (d) 48. (c) 49. (b)

Hints & Solutions


1. (d) 7. (d) The new letter series obtained on 12. (d)
reversing the order of the English alphabet is P R I S O N
ABCDEFGHIJKLMNOPQRSTUVWXYZ
ZYXWVUTSRQPONMLKJIHGFE
15th from right DCBA So, PS, RN, RO, ON, four pairs are present.
Since, the series has an even number of
2. (b) Letters between G and Q letters (i.e. 26).
13. (d)
K L I N G
G H I J K L M N O P Q There is no such letter which lies exactly in the
middle.
Middle letter So, KI, IG, LN, KL, KG, KN, i.e. six pairs are
8. (b) The new alphabet series is
Here, L is midway between G and Q. present.
ZYXWVUTSRQPONMLKJIHGFE
3. (c) Fourth letter to the left of I = E DCBA 14. (c) C H I L D R E N
The ninth letter from the right is I. The fifth
∴Fifth letter to the right of E = J
letter to the left of I is N.
4. (b) Sixth letter from left = F So, EI, HI, IN, HN, i.e. four pairs are present.
9. (a) The new alphabet series is
Fourteenth letter from right = M
ACEGIKMOQSUWY
Letters between F and M =
15. (d) P R O D U C T I O N
The tenth letter from the right is G.
G H I J K L
10. (d) The new alphabet series is So, PI, RU and ON, i.e. three pairs are
BADCFEHGJILKNMPORQTSVU present.
Letters exactly XWZY
in the middle 16. (a) The new letter sequence is
Clearly, the seventh letter from the left is H. IBUTEDISTR as shown below
5. (d) 19th letter from left = S 1 2 3 4 5 6 7 8 9 10
3rd letter to the right of S = V 11. (c) D I S T R I B U T E
C R E A T I V E
6. (a) 16th letter from the left = P
8th letter to the left of P = H So, CE, AE and TV, three pairs are present.
So, the fifth letter from the left is E.

https://sscstudy.com/
https://sscstudy.com/

30 CUET (UG) Section III : General Test

17. (c) D O C U M E N T A T I O N 25. (c) Seldom, Select, Selfish, Seller, Send 37. (d) Since, 1st, 4th, 7th and 11th letters
1 2 3 4 5 6 7 8 9 10 11 12 13 i.e. 2, 1, 4, 5, 3. are I, E, R and T, respectively. hence, two
meaningful words RITE and TIRE can be formed.
26. (c) Problem, Project, Property, Protein, ∴ Required answer = X
Proverb i.e. 2, 5, 4, 1, 3
The new letter sequence is DETNMOUTACION. 38. (d) CODING → (3, 4, 5, 2, 6, 1)
So, the eleventh letter from the right is T. 27. (d) The correct alphabetical order is
Sample, Several, She, Showed, Snaps. 39. (c) STARVE → (4, 5, 2, 3, 1, 6)
18. (a)
40. (b) HERMIT → (4, 1, 6, 2, 3, 5)
28. (d) All letters of the word MAGIC are
Word S T O N E D 41. (c) NATURE → (4, 3, 2, 5, 1, 6)
present in the given word.
Alphabetical order D E N O S T
29. (a) All letters of the word MASTER are 42. (a) After arranging the given letters
There is no such letter. present may be only once but this word can in proper sequence, we get the river name
19. (a) be formed. KAVERI and its fifth letter from left is ‘R’.
30. (c) All letters of th word EMINENT are 43. (a) The correct word is CASUAL and first
Word E N G L I S H
present in the given word. letter is ‘C’.
Alphabetical order E G H I L N S
31. (b) FAULTY cannot be formed using the 44. (b) The name of the bird is ‘PEACOCK’
letters of the given word, as it contains U. and the last letter is ‘K’.
20. (a) Plain, Plane, Player, Plenty
Clearly, plane is at second place. 32. (b) DETERMINE cannot be formed using 45. (c) The name of that vegetable would be
the letters of the given word, as it contains three E.
21. (b) Category, Catenation, Caterpillar, ‘PUMPKIN’ and last letter of the word is ‘N’.
Cathedral 33. (d) Three meaningful words can be 46. (a) The word is ‘EXPAND’ and the
Clearly, catenation is at second place. formed using W, N and O i.e WON, OWN and meaning is ‘INCREASE’.
NOW
22. (d) Converse, Curator, Current, Cushion 47. (d)The word is ‘DEARTH’ and the meaning
Clearly, cushion is at fourth place. 34. (c) Two meaningful words can be formed is ‘SCARCITY’.
viz. DRAW and WARD
23. (b) Racket, Radar, Radiate, Radical, 48. (c) The word is ‘BEHIND’ and the meaning
Clearly, Radiate is at third place.
35. (c) Meaningful word = TODAY
is ‘BACK’.
∴ Last letter = Y
24. (a) Simple → Singer → Single → Sinister 49. (b) Word formed is ‘CRICKET’ and the
36. (a) Meaningful word = SERVICE
Clearly, Single is at third place. fifth letter of this word is ‘K’.
∴Fifth letter from right = R

https://sscstudy.com/
https://sscstudy.com/

GENERAL MENTAL ABILITY 31

CHAPTER 06

Mathematical Operations
Mathematical operation can be defined as the Sol. (c) Converting alphabets into mathematical
simplification of expression containing numbers and symbols, we get
different mathematical signs. While simplifying a 12
18 × 12 ÷ 4 + 5 − 6 = 18 × + 5 − 6 = 53
mathematical problem, one must follow ‘VBODMAS’ 4
rule. The order of various operations is the same as the
order of letters in ‘VBODMAS’ from left to right. 2. Interchanging of Signs
V → Viniculum, (—) and Numbers
B → Bracket, ( ), { }, [ ] In this type of questions, the given equation becomes
O → of correct and fully balanced when either two signs of the
equation or both the numbers and signs of the equation
D → Divide, ( ÷ )
are interchanged . The candidate is required to find the
M → Multiply, ( × ) correct pair of signs and numbers from the given
A → Addition ( + ) alternatives.
S → Subtraction (−) Ex. 3 If ‘−’ means ‘+’, ‘+’ means ‘−’, ‘×’ means ‘÷’ and
The various types of questions that are asked in ‘÷’ means ‘×’, then which of the following equation is
competitive exams are as follows correct?
(a) 30 − 5 + 4 ÷ 10 × 5 = 62 (b) 30 + 5 ÷ 4 − 10 × 5 = 22
1. Problem Solving by (c) 30 + 5 − 4 ÷ 10 × 5 = 28 (d) 30 × 5 − 4 ÷ 10 + 5 = 41
Substitution Sol. (d) From option (d),
In this type, there are substitutes for various LHS = 30 × 5 − 4 ÷ 10 + 5
mathematical symbols or the numerals and questions are ↓ ↓ ↓ ↓
÷ + × −
asked related to calculation of an expression or choosing = 30 ÷ 5 + 4 × 10 − 5 = 6 + 4 × 10 − 5
the correct equation. = 6 + 40 − 5
Ex. 1 If ‘+ ’ means ‘× ’, ‘−’ means ‘÷’, ‘×’ means ‘+’ and = (6 + 40) − 5 = 41 = RHS
‘÷’ means ‘−’; compute the value of the expression (using VBODMAS rule)
17 + 6 × 13 ÷ 8 = ? ∴ LHS = RHS
(a) 100 (b) 107 Ex. 4 Which of the given interchanges in signs
(c) 110 (d) 109 would make the given equation correct?
Sol. (b) Given 17 + 6 × 13 ÷ 8
3 ÷ 5 × 8 + 2 − 10 = 13
↓ ↓ ↓ (a) + and − (b) × and ÷
17 × 6 + 13 − 8 (c) ÷ and − (d) ÷ and +
= 102 + 13 − 8 Sol. (d) On interchanging + and − as in option (a) the
= 115 − 8 = 107 result is
24
Ex. 2 If P denotes ÷, Q denotes ×, R denotes + and S 3 ÷ 5 × 8 − 2 + 10 = + 8 ≠ 13
5
denotes –, then what is the value of 18 Q 12 P 4 R 5 S
On interchanging symbols × and ÷ as given in option (b),
6=? we get
(a) 64 (b) 81 5
(c) 53 (d) 24 3 × 5 ÷ 8 + 2 − 10 = 3 × + 2 − 10 ≠ 13
8

https://sscstudy.com/
https://sscstudy.com/

32 CUET (UG) Section III : General Test

On interchanging ÷ and − as given in option (c), we get ⇒ 11 × 7 + 5 = 82


2 ⇒ 77 + 5 = 82
3 − 5 × 8 + 2 ÷ 10 = 3 − 5 × 8 +
10 ⇒ 82 = 82
2
= 3 − 40 + ≠ 13
10 4. Trick Based Mathematical
On interchanging ÷ and + as given in option (d), we get Operations
8
3 + 5 × 8 ÷ 2 − 10 = 3 + 5 × − 10 In this type of questions, there is some logic behind the
2
given expressions. The candidate is required to find that
= 3 + 20 − 10 = 13
logic and then solve the questions accordingly.
3. Balancing the Equation Ex. 6 If 9 × 5 × 2 = 529 and 4 × 7 × 2 = 724, then
In this type of questions, the signs given in one of the
3× 9× 8=?
(a) 983 (b) 839
alternatives are required to fill up the blank spaces for (c) 938 (d) 893
the signs in order to balance the given equation.
Sol. (a) As,
Ex. 5 Select the correct set of symbols 9×5×2=5 2 9 and 4×7×2=7 2 4
44 4 7 5 = 82
(a) ×, −, ÷ (b) +, ÷, −
(c) +, −, ÷ (d) ÷, ×, + Similarly, 3×9×8=9 8 3
Sol. (d) From option (d), we get
44 ÷ 4 × 7 + 5 = 82

Practice Questions
1. If the mathematical operator ‘÷ ’ means ‘ ×’ , ‘ + ’ means 7. If P means ‘÷’, Q means ‘+’, R means ‘−’ and S means ‘×’,
‘ − ’ , ‘ × ’ means ‘+’ and ‘−’ means ‘÷ ’, then then the value of 10 R 192 P 48 S 48 P 96 Q 1 is
25 + 18 − 3 × 7 ÷ 3 = ? (a) 10 (b) 9
(c) 8 (d) 7
(a) 25 (b) 21
(c) 19 (d) 40 8. If P denotes ‘+’, Q denotes ‘−’, R denotes ‘×’ and S denotes
2. If ‘+’ stands for ‘× ’, ‘−’ for ‘÷’ , ‘ ×’ for ‘− ’ and ‘÷’ for ‘+’, ‘÷’, which of the following statement is correct?
find the value of 26 + 74 − 4 × 5 ÷ 2. (a) 36 R 4 S 8 Q 7 P 4 = 10
(b) 16 R 12 P 49 S 7 Q 9 = 200
(a) 220 (b) 376
(c) 32 S 8 R 9 = 160 Q 12 R 12
(c) 478 (d) 488
(d) 8 R 8 P 8 S 8 Q 8 = 57
3. If ‘+’ ‘means ‘multiplication’, ‘−’ means ‘division’, ‘×’
9. If ‘−’ stands for division + stands for subtraction ÷
means ‘subtraction’ and ‘÷’ means ‘addition’, then
9 + 8 ÷ 8 − 4 × 9 is stands for multiplication ‘×’ stands for addition, then
(a) 65 (b) 11 which one of the following equations is correct ?
(c) 26 (d) 56 (a) 70 − 2 + 4 ÷ 5 × 6 = 44 (b) 70 − 2 + 4 ÷ 5 × 6 = 21
(c) 70 − 2 + 4 ÷ 5 × 6 = 341 (d) 70 − 2 + 4 ÷ 5 × 6 = 36
4. If ‘×’ stands for +, ÷ stands for ‘−’, ‘−’ stands for ‘×’ and ‘
+’ stands for ‘÷’ then find the value of following 10. If ‘−’ stands for ‘÷’ ‘+’ stands for ‘×’, ‘÷’ for ‘−’ × for ‘+’,
equation. which one of the following equations is correct ?
54 ÷ 16 − 3 × 6 + 2 = ? (a) 30 − 6 + 5 × 4 ÷ 2 = 27 (b) 30 + 6 − 5 ÷ 4 × 2 = 30
(c) 30 × 6 ÷ 5 − 4 + 2 = 32 (d) 30 ÷ 6 × 5 + 4 − 2 = 40
(a) 9 (b) 12
(c) 8 (d) 15
Directions (Q.Nos. 11-16) Which of the following
5. If P means ‘÷ ’, R means ‘× ’, Q means ‘+’ and S means alternatives would make the equation correct?
‘− ’, then
11. 42 ÷ 4 + 2 − 3 × 5 = 29
48 P 8 Q 6 R 9 S 31 = ?
(a) 60 (b) 29 (c) 31 (d) 54 (a) + and × (b) + and −
(c) − and × (d) ÷ and +
6. If A means ‘+’ B means ‘−’ C means ‘× ’ and D means ÷,
then 12. 2 × 3 + 6 − 12 ÷ 4 = 17
18 C 14 A 6 B 16 D 4 = ? (a) × and + (b) + and −
(a) 254 (b) 238 (c) 188 (d) 258 (c) + and ÷ (d) − and ÷

https://sscstudy.com/
https://sscstudy.com/

GENERAL MENTAL ABILITY 33

13. 12 ÷ 2 − 6 × 3 + 8 = 16 21. 6 5 4 = 34
(a) ÷ and + (b) − and + (a) − and + (b) × and +
(c) × and + (d) ÷ and × (c) ÷ and × (d) × and −

14. 5 + 6 ÷ 3 − 12 × 2 = 17 22. 27 3 19 10 = 90
(a) × , − , ÷ (b) + , ÷, −
(a) ÷ and × (b) + and × (c) + , − , ÷ (d) × , + , −
(c) + and ÷ (d) + and −
23. 7 4 8 2 = 24
15. 8 × 20 ÷ 3 + 9 − 5 = 38 (a) −, × and × (b)−, × and ÷
(a) 3, 9 (b) 3, 8 (c) × , − and ÷ (d) × , ÷ and −
(c) 8, 9 (d) 3, 5
24. 8 * 8 * 1 * 7 = 8
16. (18 ÷ 9) + 3 × 5 = 45 (a) × ÷ + (b) + ÷ ×
(c) ÷ × + (d) + × ÷
(a) × and ÷ (b) + and ÷
(c) 18 and 5 (d) 3 and 9 25. Replace # sign with the mathematical operators ‘+’, ‘÷’
17. If ‘+’ and ‘÷’, ‘×’ and ‘−’ are interchanged in the and ‘−’ and ‘=’ to get a balanced equation out of (27 #
equation 17 ÷ 7 − 27 + 7 × 37, then its value will be 15 # 2) # 10 # 4. Choose the right sequence from
(a) 7 (b) 17 (c) 27 (d) 37
below.
(a) + ÷ = − (b) − + = ÷
Directions (Q. Nos. 18 and 19) On making (c) + − ÷ = (d) + = ÷ −
interchanges in signs and numbers, which of the 26. If 4 × 6 × 2 = 351 and 3 × 9 × 8 = 287, then 9 × 5 × 6 = ?
equations would be correct? (a) 270 (b) 845
18. Given interchanges : Signs + and × and numbers 4 (c) 596 (d) 659
and 5. 27. If 5 × 3 × 9 = 395 and 9 × 7 × 5 = 759, then 7 × 6 × 4 = ?
(a) 5 × 4 + 20 = 40 (b) 5 × 4 + 20 = 85
(c) 5 × 4 + 20 = 104 (d) 5 × 4 + 20 = 95 (a) 676 (b) 476
(c) 647 (d) 764
19. Given interchanges : Signs − and × and numbers 3
and 6. 28. If 2463 = 36 and 5552 = 30, then 6732 = ?
(a) 6 − 3 × 2 = 9 (b) 3 − 6 × 8 = 10 (a) 32 (b) 36
(c) 6 × 3 − 4 = 15 (d) 3 × 6 − 4 = 33 (c) 34 (d) 39
29. If 2 + 6 + 9 = 926 and 1 + 8 + 2 = 218 then
Directions (Q. Nos. 20-24) If the following equations 4+3+1=?
has to be balance, then the signs of which of the (a) 314 (b) 341
following options will be used? (c) 143 (d) 431
20. 65 40 11 = 36 30. If 2 = 6, 4 = 12 and 8 = 24, then 10 = ?
(a) − and + (b) × and ÷ (a) 30 (b) 20
(c) ÷ and + (d) + and × (c) 15 (d) 2

ANSWERS
1. (d) 2. (c) 3. (a) 4. (a) 5. (b) 6. (a) 7. (b) 8. (d) 9. (b) 10. (a)
11. (d) 12. (a) 13. (b) 14. (a) 15. (d) 16. (b) 17. (a) 18. (c) 19. (b) 20. (a)
21. (b) 22. (d) 23. (c) 24. (c) 25. (c) 26. (b) 27. (c) 28. (a) 29. (c) 30. (a)

https://sscstudy.com/
https://sscstudy.com/

Hints & Solutions


1. (d) Given, 25 + 18 − 3 × 7 ÷ 3 = ? LHS = 42 + 2 − 3 × 5 = 42 + 2 − 15 24. (c) From option (c),
↓ ↓ ↓ = 44 − 15 = 29 = RHS 8 ÷ 8 × 1+ 7 ⇒ 1× 1+ 7
= 25 − 18 ÷ 3 + 7 × 3 12. (a) On interchanging × and + , we get ⇒ 1 + 7 = 8 = RHS
= 25 − 6 + 7 × 3 Hence, option (c) is correct.
= 25 − 6 + 21 = 19 + 21 = 40 Given expression = 2 + 3 × 6 − 12 ÷ 4
= 2 + 3 × 6 − 3 = 2 + 18 − 3 = 17
25. (c) From option (c),
2. (c) Using the correct symbols, we have (27 # 15 # 2) # 10 # 4
given expression
13. (b) On interchanging − and + , we get ⇒ (27 + 15 − 2 ) ÷10 = 4
37
= 26 × 74 ÷ 4 − 5 + 2 = 26 × − 5+ 2 ⇒ ( 42 − 2 ) ÷ 10 = 4
2 Given expression = 12 ÷2 + 6 × 3 − 8
= 6 + 6 × 3 − 8 = 6 + 18 − 8 = 16 ⇒ 40 ÷ 10 = 4
= 13 × 37 − 5 + 2 = 481 − 5 + 2 = 478
⇒ 4=4
3. (a) Given, 9 + 8 ÷ 8 −4 × 9 14. (a) On interchanging ÷ and ×, we get the
equation as, 26. (b) As, 4 × 6 × 2 = 3 5 1
↓ ↓ ↓ ↓ –1
9 × 8 + 8 ÷ 4 −9 5 + 6 × 3 − 12 ÷ 2 ⇒ 5 + 18 − 6 –1

= 9 × 8 + 2 − 9 = 72 + 2 − 9 ⇒ 23 − 6 = 17 = RHS –1

= 74 − 9 = 65 15. (d) On interchanging 3 and 5, we get the and 3×9×8=2 8 7


equation as –1
4. (a) Using the correct symbols, we have –1
given expression 8 × 20 ÷ 5 + 9 − 3 ⇒ 8 × 4 + 6
–1
54 − 16 × 3 + 6 ÷ 2 ⇒ 32 + 6 = 38 = RHS
⇒ 54 − 48 + 3 ⇒ 57 − 48 = 9 Similarly, 9×5×6=8 4 5
16. (b) On interchangng + and ÷ we get the
–1
5. (b) Given, 48 P 8 Q 6 R 9 S 31 equation as,
–1
(18 + 9) ÷ 3 × 5 ⇒ 27 ÷ 3 × 5
↓ ↓ ↓ ↓ –1
⇒ 9 × 5 = 45 = RHS
48 ÷ 8 + 6 × 9 − 31
48 27. (c) As, 5 × 3 × 9 = 3 9 5
= + 6 × 9 − 31 17. (a) Given, 17 ÷ 7 − 27 + 7 × 37
8 ↓ ↓ ↓ ↓
= 6 + 54 − 31 = 60 − 31 = 29 17 + 7 × 27 ÷ 7 − 37 and 9×7×5=7 5 9
27
6. (a) Using the correct symbols, we have = 17 + 7 × − 37
given expression 7
18 × 14 + 6 − 16 ÷ 4 = 17 + 27 − 37 = 44 − 37 = 7 Similarly, 7×6×4=6 4 7
⇒ 252 + 6 − 4 ⇒ 252 + 2 = 254 18. (c) On interchanging + and × and 4 and 5
in option (c), we get the equation as
7. (b) Given, 10 R 192 P 48 S 48 P 96 Q 1
↓ ↓ ↓ ↓ ↓ 4 + 5 × 20 = 104 = RHS 28. (a) As, 2463 = (2 + 4 + 6) × 3
= 10 − 192 ÷ 48 ×48 ÷ 96 +1 = 12 × 3 = 36
48
19. (b) On interchanging − and × and 3 and
= 10 − 4 × +1 6 in option (b) we get the equation as and 5552 = ( 5 + 5 + 5) × 2
96 = 15 × 2 = 30
1 6 × 3 − 8 = 10
= 10 − 4 × + 1 Similarly , 6732 = ( 6 + 7 + 3) × 2 = 32
2 ⇒ 18 − 8 = 10 = RHS
= 10 − 2 + 1 = 8 + 1 = 9 29. (c) As, 2 + 6 + 9 = 9 2 6
20. (a) From option (a)
8. (d) Using the proper notations in option (d), 65 − 40 + 11 = 76 − 40
we get the statement as 8 × 8 + 8 ÷ 8 − 8 = 36 = RHS
= 8 × 8 + 1 − 8 = 64 + 1 − 8 = 65 − 8 = 57 and 1+8+2=2 1 8
21. (b) From option (b),
9. (b) Using the proper notations in option (b), 6 × 5 + 4 ⇒ 30 + 4 = 34 = RHS
we get the statement as 70 ÷ 2 − 4 × 5 + 6
⇒ 35 − 20 + 6 ⇒ 41 − 20 = 21 = RHS 22. (d) From option (d), Similarly, 4+3+1=1 4 3
27 × 3 + 19 − 10 = 90
10. (a) Using the proper notation in option (a), ⇒ 81 + 19 − 10 = 90
we get the statement as ⇒ 100 − 10 = 90
30 ÷ 6 × 5 + 4 − 2 ⇒ 5 × 5 + 2 ⇒ 90 = 90 30. (a) As, 2 = 6 , 4 = 12 and 8 = 24
⇒ 25 + 2 = 27 = RHS
×3 ×3 ×3
23. (c) From option (c),
11. (d) From option (d), 7 × 4 − 8 ÷2 Similarly, 10 = 30
42 ÷ 4 + 2 − 3 × 5 = 29 ⇒ 28 − 4 = 24 = RHS
↓ ↓ ×3
Hence, option (c) is correct.
42 + 4 ÷ 2 − 3 × 5 = 29

https://sscstudy.com/
https://sscstudy.com/

GENERAL MENTAL ABILITY 35

CHAPTER 07

Direction Sense Test


Direction sense test deals with a sort of direction puzzle. The different types of questions which are asked in
A successive follow up of directions is formulated and exams related to direction sense test are as given below.
the candidate is required to find the final direction or
the distance between two points. 1. Finding Out the Final Direction
To solve the questions based on direction sense test the In this type, questions are based on final direction of a
candidates must have knowledge of some important person/object or direction of a person with respect to
concepts, which are given below. starting point.
1. The four main directions are North (N), South (S),
East (E) and West (W) and four cardinal directions Ex. 1 Deepa moved a distance of 75 m towards the
are North-East (NE), North-West (NW), South-East North. She then turned to the left and walking for
(SE) and South-West (SW). about 25 m, turned left again and walked 80 m.
Finally, she turned to the right. In which direction
N-W
Left N Right
was she moving finally?
N-E
Right Left (a) North (b) South
W E (c) West (d) South-West
Left Right
Sol. (c) According to the given information,
S-W S-E
Right S Left C 25 m B
N
2. Pythagoras theorem,
AC2 = AB2 + BC2 W E 80 m 75 m
2 2
i.e. AC = AB + BC
S E D
3. In the morning, sunrises in the East, the shadow of (end point) A (starting point)
person or object falls in the West. In the evening, the Clearly, she is moving towards West.
sunset in the West, the shadow of a person or object
falls in the East. Ex. 2 A man is facing West. He turns 45° in the
A clockwise direction and then another 180° in the same
Perpendicular

Hy direction and then 270° in the anti-clockwise


po
te
nu
direction. Which direction is he facing now?
se (a) South (b) North-West
(c) West (d) South-West
B Base C
Sol. (d) According to the given information,
4. Movement in the direction of clock is called B
270° N
clockwise movement and in the direction opposite to N-W N-E
that of clock is called anti-clockwise movement 180°
45° W E
360° 360° A
325° 45° 45° 325° O
S-W S-E
270° 90° Anti- 90° 270° S
Clockwise clockwise
D C
225° 135° 135° 225°
180° 180° Clearly, he is facing South-West now.

https://sscstudy.com/
https://sscstudy.com/

36 CUET (UG) Section III : General Test

Ex. 3 If S-E becomes West, N-E becomes South and If there is no straight distance, then distance is
so on. What will North become? calculated using Pythagoras theorem, i.e.
(a) North-East (b) North-West In this type of questions, the candidate have to find the
(c) South-East (d) South-West final distance between the starting and final points or
Sol. (c) The directions are as follow distance between two points / persons / things.
N Ex. 6 One day, Ravi left home and cycled 10 km
(S-E)
N-W N-E Southwards, turned right and cycled 5 km and turned
(E) (S) right and cycled 10 km and turned left and cycled
W E 10 km. How many kilometres will he have to cycle to
(N-E) (S-W) reach his home straight.
S-W S-E (a) 10 km (b) 15 km
S
(N) (W) (c) 20 km (d) 25 km
(N-W)
Sol. (b) According to the given information,
Clearly, North will become South-East.
10 km D N
Ex. 4 Early morning after sunrise Sangeeta went A (home)
E
for a walk. Her brother, Ramesh, who was coming (end point)
10 km 10 km W E
towards, her from opposite direction, saw that
Sangeeta’s shadow had fallen to his right. From
which direction was Ramesh coming? C 5 km B S
(a) South (b) North Thus, his distance from initial position A
(c) East (d) West = AE = AD + DE = BC + DE
Sol. (b) The direction will be as follows. = (5 + 10) km = 15 km
N Ramesh
3. Finding the Direction
W E
Sun
and Distance
S In this type of questions, the candidate have to
determine both the distance and direction.
Sangeeta shadow Sangeeta
Ex. 7 Kunal walks 10 km towards North. From
Clearly, Ramesh was coming from North.
there, he walks 6 km towards South. Then, he walks
Ex. 5 In a clock at 12 : 30, hour needle is in North 3 km towards East. How far and in which direction is
direction while minute needle is in South direction. In he with reference to his starting point?
which direction would be minute needle at 12 : 45? (a) 5 km, West (b) 7 km, West
(a) North-West (b) South-East (c) 7 km, East (d) 5 km, North-East
(c) West (d) East Sol. (d) According to the given information,
Sol. (c) The hour hand of the clock is North direction B
and minute hand is in South direction N
At 12 : 30 6 km N-W N-E
At 12 : 45
North North
W E
12 1 12 1 10 km
11 11 3 km
10 2 10 2 C D S-W S-E
Hour Hour (end point) S
hand hand
West 9 minute
3 Eest West 9 3 Eest
Minute
8 hand
4 hand A
8 4 (starting point)
7
6 5 7
6 5
South South Then, AC = (AB − BC)
= (10 − 6) = 4 km
As, it is clear from the above diagram that minute hand
is in West direction. Kunal’s distance from starting point A
AD = AC2 + CD2 (Pythagoras theorem)
2. Finding the Total Distance 2 2
= 4 + 3 = 5 km
The starting and end points are marked using left and
right turns and the distance is calculated between them. Also, D is to the North-East of A.

https://sscstudy.com/
https://sscstudy.com/

GENERAL MENTAL ABILITY 37

4. Questions Based on Map Ex. 9 The shortest distance between Shruti’s school
and her home is
In these questions, a map of a journey of a man from his
(a) 14 km (b) 12 km
starting point to end point is given. On the basis of this (c) 13 km (d) 10 km
map the candidates are required to determine the Sol. (Ex. Nos. 8 and 9) According to given information,
position of starting point with respect to end point,
N Temple
position of end point with respect to starting point or the N-W N-E 7 km D
C
position of other place with respect to starting or end W E 5 km
Shruti’s
School
point.
S-W S-E 5 km
S
Directions (Ex. Nos. 8 and 9) Analyse the following A
information and answer the questions based on it. Shruti’s 5 km B 7 km E
Home
The map of Shruti’s journey from her home to school is
given below 8. (b) From the given diagram, Shruti’s school is in
Shruti’s
School
North-East direction with respect to her home.
Temple
9. (c) From the given diagram,
7 km
Shruti’s BE = CD = 7 km
home 90° Right
Then, AE = 5 + 7 = 12 km
90° Left 5 km
and DE = CB = 5 km
5 km East
On the basis of above map, answer the questions given ∴ Shortest distance, AD = (AE)2 + (ED)2
below.
= (12)2 + (5)2
Ex. 8 The direction of Shruti’s school with respect
= 144 + 25
to her home is
(a) North (b) North-East = 169
(c) South-West (d) East = 13 km

Practice Questions
1. A boy goes in South direction, then he turns towards 5. I am facing North. I turn 135º in clockwise direction,
left and travels for some distance. After that he turns then 180º in anti-clockwise direction. What direction
right and moves certain distance. At last he turns left am I facing now?
and travel again for some distance. Now, in which (a) North-East (b) West
direction is he moving? (c) South-East (d) North-West
(a) South (b) West
(c) East (d) North 6. Rama is facing East. He turns 90º in clockwise
direction, and then 135º in anti-clockwise direction
2. I was facing East from where I turned to my left and and then again 90º in clockwise direction. Which
walked 12 ft, then I turned towards right and walked
direction is she facing now?
6 ft, After that I walked 6 ft in South direction and at
(a) South-East (b) South
last I walked 6 ft in the West. Then, in which (c) North-East (d) South-West
direction am I standing from the original point?
(a) West (b) East 7. A person walks away from his house at 8:00 am and
(c) South (d) North observes his shadow to his right. Then, he turns
towards his left and then again towards his right.
3. Ram is to the South of Aishwarya and to the West of Which direction is he facing now?
Rani. If Priyanka is to the South of Ram, then in which (a) West (b) South
direction is Priyanka with respect to Rani? (c) North (d) East
(a) South (b) North-East
(c) South-West (d) North 8. One day, during Sunset, two friends Sudhir and Amit
were talking, facing each other. If Amit’s shadow was
4. There are four roads. I have come from the South and on his right then in which direction is Aman facing? If
want to go the temple. The road to the right leads me Aman faces the direction opposite of what Sudhir is
towards the coffee house while straight road leads to facing?
the college. In which direction is the temple? (a) North (b) West
(a) North (b) East (c) South (d) West (c) East (d) South

https://sscstudy.com/
https://sscstudy.com/

38 CUET (UG) Section III : General Test

9. The time in a clock is quarter past twelve. If the hour walked 15 m. At what distance is he from his starting
hand points to the East, then towards which direction point and in which direction?
the minute hand is pointing? (a) 35 m, East (b) 35 m, North
(a) South-West (b) South (c) 40 m, East (d) 60 m, East
(c) West (d) North
19. Starting from a point P, Rohan walked 20 m towards
10. A direction pole was situated on the road crossing. South. He turned left and walked 30 m. He then
Due to an accident, the pole turned in such a manner turned left and wallked 20 m. He again turned left
that the pointer which was showing East, started and walked 40 m and reached point Q. How far and in
showing South. Sita, a traveller went to the wrong which direction is the point Q from the point P?
direction thinking it to be West. In what direction (a) 20 m, West (b) 10 m, East
actually she was travelling? (c) 10 m, West (d) 10 m, North
(a) North (b) West (c) East (d) South
20. Mohan left for his office in his car. He drove 15 km
11. While facing East you turn to your left and walk towards North and then 10 km towards West. He then
10 yards. Then, turn to your left and walk 10 yards turned to the South and covered 5 km. Further he
and now turn 45° to your right and go straight to turned to the East and moved 8 km. Finally, he
cover 50 yards. Now, in what direction are you with turned right and drove 10 km. How far and in which
respect to the starting point? direction is he from his starting point?
(a) North-East (b) North (a) 2 km, West (b) 5 km, East
(c) South-East (d) North-West (c) 3 km, North (d) 6 km, South
12. A house faces North. A man coming out of his house 21. Ramesh walks 10 m towards South. Turning to the
walked straight for 10 m, turned left and walked left, he walks 20 m and then moves to his right. After
25 m. He then turned right and walked 5 m and again
moving a distance of 20 m, he turns to the right and
turned right and walked 25 m. How far is he from his
then walks 20 m. Finally, he turns to the right and
house?
moves a distance of 10 m. How far and in which
(a) 15 m (b) 55 m
(c) 60 m (d) 65 m
direction is he from the starting point?
(a) 10 m, North (b) 20 m, South
13. A cyclist goes 30 km to North and then turning East (c) 20 m, North (d) 10 m, South
he goes 40 km. Again, he turns to his right and goes
20 km. After this, he turns to his right and goes 22. Raju starts from a place P towards North and reaches
40 km. How far is he from his starting point? place Q. From there, he turns towards North-West
(a) 6 km (b) 10 km and reaches place R. Then, he turns towards
(c) 25 km (d) 40 km South-West and walks to a place S. From there, he
14. A boat moves from port towards East. After sailing for turns towards North-West and finally reaches place T.
9 miles, it turns towards right and covers another Which of the following figures shows the movement of
12 miles. If it wants to go back to the port, what is the Raju?
shortest distance now from its position? R
T T R
(a) 21 miles (b) 20 miles Q Q
(c) 18 miles (d) 15 miles (1) S (2)
S
15. A cyclist rides 40 km to the East, turns North and
P P
rides 20 km, again turns left and rides 20 km. How R
R
far is he from his starting point? T
Q S Q
(a) 0 km (b) 10 km S
(3) (4)
(c) 15 km (d) 20 2 km
P T P
16. A man travels 4 km due North, then travels 6 km due
East and further travels 4 km due North. How far is Directions (Q. Nos. 23 and 24) In the below diagram,
he from the starting point? the position of seven points are shown.
(a) 6 m (b) 14 km G F
(c) 8 km (d) 10 km A
17. A and B starts walking from same point. A goes North D
and covers 3 km, then turns right and covers 4 km. B E
B C
goes West and covers 5 km. Then, turns right and
covers 3 km. How far apart are they from each other? Here, Point A is 11 m North of point B. Point C is 11 m
(a) 10 km (b) 9 km East of point B. Point D is 6 m North of point C. Point E
(c) 8 km (d) 5 km is 7 m West of point D. Point F is 8 m North of point E.
18. Rohit walked 25 m towards South. Then he turned to Point G is 4 m West of point F.
his left and walked 20 m. He then turned to his left On the basis of the above information, answer the
and walked 25 m. He again turned to his right and questions asked.

https://sscstudy.com/
https://sscstudy.com/

GENERAL MENTAL ABILITY 39

23. How far is point F form point A? Directions (Q. Nos. 27 and 28) Study the following
(a) 43 m (b) 4 m map carefully and answer the questions asked.
(c) 3 m (d) 5 m
Agarwal Av
24. How far and in which direction is point G from point Lena Bank

Plaza Way
A? N N
Building
(a) 3 m, North (b) 5 m, North Gupta St
(c) 4 m, North (d) 4 m, South Jain Metro
Oil Company HQ O Plaza
P

Jain Nagar Rd
Directions (Q. Nos. 25-26) In the following map the West st Plaza St
Arihant

Josh St
Rock AV
position of 5 checkposts A, B, C, D and E are shown. Publications Sharma
Building Sweets
Study the map carefully and answer the questions

Rd
Garh Road
asked.

Delhi
Checkpost

Dun Rd
In the map PVS Mall
B Raj St
=1 km

TP St
Checkpost
E M
Checkpost Greater Av
D
Checkpost
A 27. Astha is in Arihant Publications Building and can see
Checkpost Sharma sweets in her front which direction is she
C facing?
25. If a checkpost A is in the East direction of checkpost (a) East (b) West
C, then in which direction is the checkpost B which (c) North (d) South
respect to chckpost A? 28. Yatharth starts from location ‘P’ and proceeds as
(a) North-West (b) North-East follows right onto plaza st heading West, second right
(c) South-East (d) South-West heading North, first left heading West and stops as
26. Find the shortest distance between checkpost E and location ‘N’. Where is location ‘O’ in relation to his
checkpost A. current location?
(a) 17 km (b) 13 km (a) South-East (b) South-West
(c) 18 km (d) 20 km (c) North (d) East

ANSWERS

1. (c) 2. (d) 3. (c) 4. (d) 5. (d) 6. (a) 7. (b) 8. (a) 9. (b) 10. (a)
11. (d) 12. (a) 13. (b) 14. (d) 15. (d) 16. (d) 17. (b) 18. (a) 19. (c) 20. (a)
21. (b) 22. (a) 23. (d) 24. (a) 25. (a) 26. (b) 27. (a) 28. (a)

https://sscstudy.com/
https://sscstudy.com/

40 CUET (UG) Section III : General Test

Hints & Solutions


1. (c) The direction diagram is as follows, N Then, turned his left in the direction of West
N-W N-E
and walked 10 yards. Now, he turned 45° to
Starting point
N his right and walked 50 yards straight in the
W E same direction. Now, the direction of the man
45°
W E with respect to his starting point is
North-West.
S-W S S-E
S
12. (a) According to the question,
Hence, now Rama is facing in South-East
Clearly, he is moving towards East direction. direction. 25 m
C D
N
2. (b) The direction diagram is as follows, 7. (b) According to the question, the shadow 5m
6 ft is towards his right at 8:00 am that means the B A W E
25 m
person is walking towards South direction.
10 m
6 ft House S
12 ft N N O House
W E ∴ Required distance = OD = OA+AD
6 ft W E
S = OA+BC = 10+5
Final position
S = 15 m
Clearly, he is facing South direction.
Initial
position
13. (b) According to the question,
8. (a) According to the question, at Sunset 40 km
Clearly, I am towards North from my initial position. Amit’s shadow is towards his right that means he A B
is facing North. Now, Sudhir is standing in front of N
3. (c) According to the question, 30 km 20 km
him that means Sudhir is facing South. Now,
Aishwarya W E
Aman faces the direction opposite of Sudhir. D C
40 km
∴ Aman is facing North direction. S
N-W N N-E Final position
9. (b) Time quarter past twelve means that the
Ram Rani W E Starting O
time is 12 : 15. position
S-W S S-E Original position After shifting
N
∴Required distance,
11
12
1 8
9
10 OD = OA – AD
Priyanka 10 2 7 11
W E = OA – BC = 30 – 20 = 10 km
9 3 6 12
Clearly, Priyanka is towards South-West of Rani.
8 4 5 1
7 5 4 2
S 14. (d) According to the question,
4. (d) According to the question, 6 3
Port 9 miles A
College
N Hence, when hour hand is pointing towards O N
East, then the minute hand is pointing towards
12 miles W E
Temple Coffee W E South.
house
Left Right N W S
10. (a)
S B
Starting
point W E S N
∴ Required distance, OB = OA 2 + AB 2
Clearly, temple is towards West. S E [using Pythagoras theorem]
5. (d) Difference in degrees The, pointer which was showing West started = 9 2 + 12 2
showing South. Hence, the pointer turned 90° = 225 = 15 miles
= 180º – 135º = 45º anti-clockwise.
clockwise.
(anti-clockwise) (clockwise) 15. (d) According to the question,
Now, Sita went to the direction thinking it as
N West. The original direction will be +90°
N-W N-E C 20 km
B N
45° clockwise i.e. North direction.
11. (d) Let A be the initial position of the man. 20 km W E
W E
He initially faced East and then turned his left in Starting
the direction of North and walked 10 yards. point O D A S
40 km
S-W D
S S-E
45° In ∆OCD, OD = OA − AD = OA − BC
50 yards
Hence, now I am facing in North-West direction. 10 yards = 40 − 20 = 20 km
B
C and CD = AB = 20 km.
6. (a) Clockwise turn = 90º + 90º = 180º
Now, required distance,
Anti-clockwise turn = 135º 10 yards
Difference =180º – 135º = 45° clockwise OC = CD 2 + OD 2 = 20 2 + 20 2
A = 800 = 20 2 km

https://sscstudy.com/
https://sscstudy.com/

GENERAL MENTAL ABILITY 41

16. (d) According to the question, 20. (a) According to the question, ∴ AF = AG 2 + GF 2
C B
10 km
A = 9 + 16 = 5 m
5 km N
Final D
4 km N C 24. (a) Point G is in the North direction from
position 8 km
15 km W E
A 10 km point A and AG = 3 m [from above solution]
6 km B W E
4 km Final E S
O Starting 25. (a) Since the checkpost A is in the East
position point
S direction of checkpost C, so the checkpost B
Starting D
point O Point E is to the West of point O. is in North-West direction with respect to
Now, required distance, EO = AB − CD checkpost A.
In ∆OCD,
and OD = AB = 6 km = 10 − 8 = 2 km 26. (b) According to the figure,
⇒ CD = BC + BD Hence, he is 2 km towards West from his Checkpost Checkpost
starting point. D E
= BC + OA
= 4 + 4 = 8 km 21. (b) According to the question,
Now, required distance O Starting point
OC = OD 2 + CD 2 10 m N
20 m A
Checkpost C Checkpost M (Let)
= 6 2 + 8 2 = 100 = 10 km A B W E Checkpost
Final 20 m
17. (b) According to the question, E
position 10 S Distance between checkpost M and
m
D
20 m
C checkpost A = 12 × o
B’s Final position 4 km
G A’s Final Point E is to the South of Point O. = 12 × 1 = 12 km
C D position
3 km 3 km Now, Required distance = OE = OA + AE Distance between checkpost E and
O Starting point = OA + BC − ED = 10 + 20 − 10 = 20 m checkpost M = Distance between checkpost D
E 5 km Hence, Ramesh is 20 m towards, South from and checkpost C = 5 × o
N his starting point.
= 5 × 1 = 5 km
W E 22. (a) According to the question, the ∴Required shortest distance, EA
direction diagram is as given below.
S = (MA)2 + (EM)2
T R N
∴ Required distance, GD = GC + CD Q = (12)2 + (5)2
= EO + CD S W E
= 5 + 4 = 9 km = 144 + 25
P S = 169
18. (a) According to the question,
O This situation is traced by option (a). = 13 km
C 15 m N
Starting D
point Final Sol. (Q. Nos. 23 and 24) According to the 27. (a) According to the question,
25 m 25 m position W E information, the direction diagram is as given N
below.
A B S
20 m W E
∴ Required distance, OD=OC+CD N Arihant Publications Sharma S
= BA + CD Building Sweets
= 20 + 15 = 35 m W E
Also, point D is to the East of point O. Clearly, Sharma Sweets is in East direction of
∴ He is 35 m towards East from his starting point. S Arihant Publications Building, so Astha is
facing towards East direction.
19. (c) According to the question,
23. (d) According to the question, the
Starting point direction diagram is as given below.
28. (a) According to the question,
P N
C 4m
Q C N
G F
20 m

Final 20 m N N-W N-E


W E
position W E
A 3m
30 m B
S S-W S-E
O S
We can see that point ‘Q’ is to the West of P
A
point ‘P’.
Now, Required distance = PQ = QC − PC AG = BG − AB
= QC − AB = (CD + EF ) − ( AB ) Clearly, location O is in South-East direction in
= 40 − 30 = 10 m = ( 6 + 8) − 11 = 3 m relation to his current location i.e., N.
∴ Q is 10 m to the West of P. and GF = 4 m

https://sscstudy.com/
https://sscstudy.com/

42 CUET (UG) Section III : General Test

CHAPTER 08

Blood Relations
Blood relation between two individuals is defined as a
Relation Relation name
relation between them by the virtue of their birth. Here,
a chain of relationship is given in the form of Mother’s or father’s daughter Sister
information and the success of candidate depends upon Son’s wife Daughter - in-law
his/her knowledge of blood relations. These relationships Husband’s or wife’s sister Sister-in-law
can be used to solve various types of problems related to
Daughter’s husband Son-in-law
blood relations.
Husband’s or wife’s brother Brother-in-law
Important Relations Brother’s son Nephew
Relations from paternal side Brother’s daughter Niece
Relation Relation name Sister’s husband Brother-in-law
Father’s father Grandfather Brother’s wife Sister-in-law
Fother’s mother Grandmother Grandson’s or Granddaughter’s Great
daughter or son granddaughter
Father’s brother Uncle or son
Father’s sister Aunt
There are three types of questions, which are asked in
Children of uncle or aunt Cousin competitive exams.
Wife of uncle Aunt
Husband of aunt Uncle
1. Jumbled-up Descriptions
In this type, a round about description is given in the
Relations from Maternal Side form of certain small relationships and candidate is
Relation Relation name required to analyse the whole chain of relations and
Mother’s father Maternal grandfather
then find out the direct relationship between the
concerned persons.
Mother’s mother Maternal grandmother
Ex. 1 Pointing towards a person in the photograph,
Mother’s brother Uncle
Anjali said, ‘He is the only son of the father of my
Mother’s sister Aunt
sister’s brother’. How is that person related to Anjali?
Children of maternal uncle or Aunt Cousin (a) Mother (b) Father
Wife of maternal uncle Aunt (c) Maternal uncle (d) Brother
Husband of maternal aunt Uncle Sol. (d) The relations may be analysed as Sister’s
brother-Brother, Brother’s father-Father. Father’s
Other Relations son-Brother. So, the person in the photograph is Anjali’s
brother as shown in the figure given below
Relation Relation name
(+)
Grandfather’s or Grandmother’s son Father or Uncle Father
Grandfather’s or Grandmother’s only son
Grandfather’s or Grandmother’s daughter Aunt
Father Brother ( (+)
(–)
Male
Female )
Anjali Sister Brother (son)
Mother’s or father’s son Brother
(–) (+)

https://sscstudy.com/
https://sscstudy.com/

GENERAL MENTAL ABILITY 43

Ex. 2 Kitty said, ‘‘Uthara is one of the two 3. Coded Relations


daughters of my mother’s brother’s wife’’. How is
In this type, the relationships are represented by certain
Kitty’s mother related to Uthara’s sister? specific codes or symbols such as + ,− ,÷ ,× ,$, @ etc.
(a) Maternal Aunt (b) Mother Candidate has to analyse the codes to determine the
(c) Grandmother (d) Sister
required relationship.
Sol. (a) According to the question,
Ex. 5 If ‘P+Q’ means ‘P is the father of Q’, ‘P×Q’
Brother Wife
means ‘P is the brother of Q’, ‘P-Q’ means ‘P is the
Mother Maternal Aunt Daughter mother of Q’, then which of the following is definitely
Kitty
Sister
Uthara true about C−A+B?
(a) B is the son of A (b) A is the son of C
From the above, it is clear that Kitty’s mother is the (c) B is the father of A (d) C is the mother of B
maternal aunt of Uthara’s sister. Sol. (b) According to the given information,
C
2. Relation Puzzle (–)
In this type, mutual blood relations of more than two (Son) (Mother)
persons are mentioned. Candidate has to analyse the
A
information and find the required relation.
(+)
Ex. 3 If ‘A’ is the son of ‘B’ and is the father of ‘C’, (Father)
how is ‘B’ related to ‘C’?
B
(a) Father (b) Son
Clearly, A is the son of C.
(c) Grandparent (d) Grandchild
Sol. (c) According to the given information, Ex. 6 If ‘P × Q’ means P is mother of Q.
● P + Q’ means P is sister of Q.
B
● P ÷ Q’ means P is father of Q.
Grandparent

Son

(+)A
● P − Q’ means P is brother of Q.
Then, which of the following means J is paternal
Father
grandfather of W?
C (a) W ÷ T ÷ J (b) J ÷ T × W
(c) J × T × W (d) None of these
From the above it is clear that, B is the grandparent
of C. Sol. (d) From option (a),
W ÷ T ⇒ W is the father of T
Ex. 4 A and B are brothers. C and D are sisters. A ‘s
son is D’s brother. How is B related to C ? T ÷ J ⇒ T is the father of J
So, W is the grandfather of J.
(a) Father (b) Brother
(c) Grandfather (d) Uncle From option (b),
Sol. (d) According to the given information, J ÷ T ⇒ J is the father of T
T × W ⇒ T is the mother of W.
Brothers
A(+) B(+) So, J is the maternal grandfather of W

(Sisters) Brother Uncle


( (+)
(–)
Male
Female ) From option (c),
J × T ⇒ J is the mother of T
(–)C (–)D Son (+) T × W ⇒ T is the mother of W
So, J is the maternal grandmother of W.
Clearly, B is uncle of C. So, all the given relations are false.

https://sscstudy.com/
https://sscstudy.com/

44 CUET (UG) Section III : General Test

Practice Questions
1. Introducing Reena, Monika said, ‘She is the only 11. Showing a man, a woman said, “His brother’s father is
daughter of my father’s only daughter.’ How is the only son of my grandfather”. How is the woman
Monika related to Reena? related to the man?
(a) Aunt (b) Niece (a) Aunt (b) Sister
(c) Mother (d) Cousin (c) Daughter (d) Mother
2. Pointing to a man, a woman said, ‘His mother is the 12. Mathew told his friend Shyam, pointing to a
only daughter of my mother.’ How is the woman photograph, “Her father is the only son of my mother”.
related to the mother? The photograph is of whom?
(a) Mother (b) Daughter (a) Mathew’s niece (b) Mathew’s mother
(c) Sister (d) Grandmother (c) Mathew’s daughter (d) Mathew’s sister
3. Pointing to Ketan, Namrata said, ‘He is the son of my 13. Looking at a photograph, a person said, “I have no
father’s only son’. How is Ketan’s mother related to brother or sister but that man’s father is my
Namrata? father’s son”. At whose photograph was the person
(a) Daughter (b) Aunt looking at?
(c) Sister (d) Sister-in-law (a) His son’s (b) His nephew
4. Pointing to a man on the stage, Rashi said, ‘He is the (c) His father’s (d) His own
brother of the daughter of the wife of my husband’. How 14. Bina is the daughter of Mohan who is the only
is the man on the stage related to Rashi? son-in-law of Meena. Meena has only one child. Kiran
(a) Son (b) Husband is the granddaughter of Meena. How is Kiran Related
(c) Cousin (d) Nephew
to Bina?
5. How is Sameer related to Akbar if Sameer introduced (a) Sister (b) Daughter
Akbar as his maternal grandmother’s only son’s son? (c) Maternal aunt (d) Mother
(a) Brother (b) Son
15. E is the daughter of P who is the husband of the only
(c) Maternal Uncle (d) Cousin
daughter-in-law of K. How is E related to K?
6. Deepak said to Nitin, ‘That boy playing football is the (a) Daughter (b) Granddaughter
younger of the two brothers of the daughter of my (c) Grandmother (d) Mother
father’s wife.’ How is the boy playing football related
to Deepak? 16. P’s father is Q’s son. M is the paternal uncle of P and N
is the brother of Q. How is N related to M?
(a) Son (b) Brother
(c) Cousin (d) Nephew (a) Brother (b) Nephew
(c) Cousin (d) None of these
7. Malini said, ‘‘Rohit is my maternal aunt’s mother’s
only son’s son’’. How is Malini related to Rohit? 17. Q is the brother of R. P is the sister of Q. T is
(a) Cousin (b) Mother the brother of S. S is the daughter of R. How is P
(c) Sister (d) Daughter related to T?
(a) Uncle (b) Aunt
8. Ram said, ‘‘Sita is my paternal great grandfather’s (c) Brother (d) Mother
only son’s only daughter-in-law’’. How is Sita related
to Ram? 18. M is the son of N. O is the father of N. P is the father
(a) Maternal Aunt (b) Paternal Aunt of M. How is N related to P?
(c) Mother (d) Sister (a) Wife (b) Husband
(c) Father (d) Mother
9. Pointing to a photograph, a person tells his friend,
‘She is the granddaughter of the elder brother of my 19. P’s father Q, is B’s paternal uncle and A’ s
father’. How is the girl in the photograph related to husband M, is P’s only paternal uncle. How is A
this man? related to B?
(a) Niece (b) Sister (a) Cousin (b) Aunt
(c) Aunt (d) Sister-in-law (c) Mother (d) Data inadequate
10. Pointing to a gentleman, Deepak said, ‘His only 20. Q is the son of P, X is the daughter of Q,R is the
brother is the father of my daughter’s father’. How is aunty (Bua) of X and L is the son of R, then what is L
the gentleman related to Deepak? to P?
(a) Grandfather (b) Father (a) Grandson (b) Grand daughter
(c) Brother-in-law (d) Uncle (c) Daughter (d) Nephew

https://sscstudy.com/
https://sscstudy.com/

GENERAL MENTAL ABILITY 45

21. If ‘P$Q’ means ‘P is father of Q’, ‘P # Q’ means ‘P is 24. If,


mother of Q’, ‘P * Q’ means ‘P is sister of Q’, then how is • ‘P ÷ Q’ means ‘P is sister of Q’
D related to N in N# A $ B * D? • ‘P × Q’ means ‘P is brother of Q’
(a) Nephew • ‘P − Q’ means ‘P is mother of Q’
(b) Grandson • ‘P + Q’ means ‘P is father of Q’
(c) Grand daughter Then, which of the following means ‘ M is maternal
(d) Data inadequate uncle of T’ ?
22. If ‘P × Q’ means ‘P is the daughter of Q’, ‘P + Q’ means (a) M ÷ K + T
‘P is the father of Q’, ‘ P ÷ Q, means ‘P is the mother of (b) M × K + T
Q’ and ‘P − Q’ means ‘P is the brother of Q’, then in the (c) M × K − T
expression A ÷ B + C − E × F, how is A related to F? (d) M ÷ K − T
(a) Mother (b) Aunt
(c) Daughter-in-law (d) None of these
25. If
• ‘P × Q’ means ‘P is wife of Q’
23. ‘P−Q’ means ‘P is the mother of Q’, ‘P × Q’ means ‘P is • ‘P ÷ Q’ means ‘P is father of Q’
the father of Q’ and ‘P+Q’ means ‘P is the daughter of • ‘P + Q’ means ‘P is son of Q’
Q’. Now, if M − N × T + Z, then which of the following is • ‘P − Q’ means ‘P is sister of Q’
not true?
(a) T is N’s daughter Then which of the following represent’s S is mother
(b) N is wife of Z of T?
(c) M is mother-in-law of Z (a) S × M ÷ H − T (b) S × M + H − T
(d) T is grand daughter of M (c) M × S ÷ H − T (d) M × S ÷ H + T

ANSWERS
1. (c) 2. (a) 3. (d) 4. (a) 5. (d) 6. (b) 7. (a) 8. (c) 9. (a) 10. (d)
11. (b) 12. (c) 13. (a) 14. (a) 15. (b) 16. (d) 17. (b) 18. (a) 19. (c) 20. (a)
21. (d) 22. (d) 23. (b) 24. (c) 25. (a)

Hints & Solutions


1. (c) Monika’s fathers’s only brother- Deepak’s younger brother. So, the 9. (a) Brother of father-uncle Uncle’s
daughter-Monika. So, Reena is Monika’s boy is Deepak’s brother. granddaughter-Daughter of uncle’s
daughter, i.e. Monika is Reena’s mother. 7. (a) According to the question, son-Daughter of Cousin-Niece

2. (a) Only daughter of my mother- Myself. So, Only


s
10. (d) Father of Deepak’s daughter’s
on
the woman is man’s mother. father-Deepak’s father. So, the man is the
Grand mother

Mother brother of Deepak’s father, i.e. Deepak’s uncle.


Maternal

3. (d) Namrata’s father’s only son- Namrata’s


brother. So, Ketan is the son of Namrata’s Son 11. (b) Here, only son of woman’s grandfather
brother. Maternal is the father of woman. Also man’s brother’s
aunt Rohit father is the father of that woman. Now, it is
Thus, Ketan’s mother is the wife of Namrata’s
clear that the woman is the sister of that man.
brother, i.e. Namrata’s sister-in -law. Malini
12. (c) Here only son of Mathew’s mother
4. (a) Wife of Rashi’s husband-Rashi Cousin
means Mathew himself. Now, it is clear that
Brother of daughter-Son Clearly, Rohit is the cousin of Malini. the photograph is of Mathew’s daughter.
So, the man on the stage is Rashi’s son.
8. (c) According to the question, 13. (a) Here, person’s father’s only son is the
5. (d) According to the question, person himself and the man in the photograph
is the son of that person. So, the person is
al

Father looking at his son’s photograph.


n

Only son
er

Only son
at

Mother
M

14. (a) According to the question,


Grand Great
Daughter-in-law -
mother Brother Son grandfather son
Mother Father Only law Meena
in-
Grand daughter

Grand
Sita Child
Sameer Akbar father Wife
Mohan
Father er
Cousin
M oth Daughter
Ram Daughter
Clearly, Akbar is the cousin of Sameer.
(–)Bina
Sister Kiran
6. (b) Father’s wife- Mother, Mother’s Clearly, Sita is the mother of Ram.
daughter-Sister, Deepak’s sister’s younger Clearly, Kiran is the sister of Bina.

https://sscstudy.com/
https://sscstudy.com/

46 CUET (UG) Section III : General Test

15. (b) According to the question, 19. (c) According to the question, 23. (b) According to the question,
K Couple Brother
Daughter-in-law A(–) M(+) Q(+) M (–)
Grand daughter

(–) Father Father


P+ (Mother)
Husband
Mother
Daughter r B P
he
ot (+) Couple
M Clearly, A is mother of B N (–)
Z
E(–)
20. (a) According to the question.
Clearly, E is the granddaughter of K. (Father)
P
16. (d) According to the question, Son T–
Brother Grandson
Q(+) N(+) Sister
(+) Q R(–) Clearly, N is husband of Z
Son Uncle
Daughter Son
Brother 24. (c) From option
?(+) M(+) (–) ( − ) sister
X L(+) (a) M ÷ K + T = M ← K ← T
(+) father

Father
Now, it is clear that L is the grandson of P. M is the aunt of T
P Father
21. (d) According to the question, (b) M × K + T = M ←
Brother
K ← T
Clearly, N is uncle of M. M is the paternal uncle of T
N(–)
17. (b) According to the question, Brother Mother
Mother (c) M × K − T = M ← K ← T
Sister Brother
P(–) Q(+) R M is the maternal uncle of T.
A(+)
So, option (c) gives the stated
Daughter Father relation.
Brother Sister
S(–) T(+) B(–) D 25. (a) From option (a),
Aunt D is either granddaughter or grandson of N. S × M → S is wife of M
Clearly, P is aunt of T. 22. (d) According to the question, M ÷ H → M is father of H
H − T → H is sister of T
18. (a) According to the question, A (–) Mother-in-law
O(+) Couple
Mother S(–) M(+)
Father Married
(–) Couple B(+) Couple (–)
N P(+) F Father

Son Father Daughter Sister


Father T H(–)
(+) Brother (–)
(+)M C E
Clearly, S is mother of T.
Clearly, N is the wife of P. A is mother-in-law of F.

https://sscstudy.com/
https://sscstudy.com/

GENERAL MENTAL ABILITY 47

CHAPTER 09

Logical Sequence
of Words
The arrangement of words in an order based on reality Ex. 2 1. Phrase 2. Letter
and facts that are universally accepted is called logical 3. Word 4. Sentence
sequence of words. In this type of questions, a group of (a) 1, 2, 3, 4 (b) 1, 3, 2, 4
interrelated words are given in any order. The candidate (c) 2, 3, 1, 4 (d) 2, 3, 4, 1
is required to find out the proper logical arrangement of Sol. (c) A group of letter makes a word.
those words.
A group of words makes a phrase. A group of phrases
Directions (Ex. Nos. 1-3) In each of the following makes a sentence.
questions arrange the words in a meaningful/logical So, the correct order becomes 2, 3, 1, 4.
order and then select the appropriate sequence from the
alternatives given below each group of words. Ex. 3 1. Consultation 2. Illness
3. Doctor 4. Treatment
Ex. 1 1. Birth 2. Death
5. Recovery
3. Marriage 4. Education
(a) 2, 3, 1, 4,5 (b) 2, 3, 4, 1, 5
(a) 1, 2, 3, 4 (b) 4, 2, 3, 1
(c) 1, 4, 3, 2 (d) 3, 1, 2, 4 (c) 4, 3, 1, 2, 5 (d) 5, 1, 4, 3, 2
Sol. (c) Clearly, the given words when arranged in Sol. (a) Clearly, Illness occurs first.
order of various events as they occur in man’s life, form Then, one goes to a doctor and after consultation
the sequence, with him, undergoes treatment to finally attain
Birth → Education → Marriage → Death recovery.
(a) (d) (c) (b) So, the correct order becomes 2, 3, 1, 4, 5.
So, the correct order becomes 1, 4, 3, 2.

Practice Questions
Directions (Q. Nos. 1-20) In each of the following 4. 1. Phrase 2. Letter 3. Word 4. Sentence
questions arrange the words in a meaningful/logical (a) 1, 2, 3, 4 (b) 1, 3, 4, 4
order and then select the appropriate sequence from the (c) 2, 3, 1, 4 (d) 2, 3, 4, 1
alternatives given below each group of words.
5. 1. District 2. Village 3. State 4. Block
1. 1. Honey 2. Flower 3. Bee 4. Wax (a) 2, 1, 4, 3 (b) 2, 3, 4, 1
(a) 1, 3, 4, 2 (b) 2, 1, 4, 2 (c) 2, 4, 1, 3 (d) 3, 2, 1, 4
(c) 2, 3, 1, 4 (d) 4, 3, 2, 1
6. 1. Plant 2. Fruit 3. Seed 4. Flower
2. 1. Butterfly 2. Cocoon 3. Egg 4. Worm (a) 3, 2, 4, 1 (b) 3, 1, 2, 4
(a) 1, 3, 4, 2 (b) 1, 4, 3, 2 (c) 3, 1, 4, 2 (d) 3, 2, 1, 4
(c) 2, 4, 1, 3 (d) 3, 4, 2, 1
7. 1. Twilight 2. Dawn 3. Noon 4. Night
3. 1. Weaving 2. Cotton 3. Cloth 4. Thread (a) 2, 1, 3, 4 (b) 2, 3, 1, 4
(a) 2, 4, 1, 3 (b) 2, 4, 3, 1 (c) 1, 2, 3, 4 (d) 1, 3, 2, 4
(c) 4, 2, 1, 3 (d) 3, 1, 4, 2

https://sscstudy.com/
https://sscstudy.com/

48 CUET (UG) Section III : General Test

8. 1. Pupa 2. Larva 14. 1. Punishment 2. Prison 3. Arrest 4. Crime


3. Moth 4. Eggs 5. Judgement
(a) 4, 2, 1, 3 (b) 4, 1, 2, 3 (a) 5, 1, 2, 3, 4 (b) 4, 3, 5, 2, 1
(c) 4, 3, 2, 1 (d) 4, 3, 1, 2 (c) 4, 3, 5, 1, 2 (d) 2, 3, 1, 4, 5
9. 1. Mother 2. Child 15. 1. Vegetable 2. Market 3. Cutting
3. Milk 4. Cry 4. Cooking 5. Food
5. Smile (a) 1, 2, 3, 4, 5 (b) 2, 1, 3, 4, 5
(a) 1, 5, 2, 4, 3 (b) 2, 4, 1, 3, 5 (c) 3, 1, 2, 5, 4 (d) 5, 2, 1, 3, 4
(c) 2, 4, 3, 1, 5 (d) 3, 2, 1, 5, 4 16. 1. Frog 2. Eagle
10. 1. Study 2. Job 3. Grasshopper 4. Snake
3. Examination 4. Earn 5. Grass
5. Apply (a) 1, 3, 5, 2, 4 (b) 3, 4, 2, 5, 1
(a) 1, 2, 3, 4, 5 (b) 1, 3, 2, 5, 4 (c) 5, 3, 1, 4, 2 (d) 5, 3, 4, 2, 1
(c) 1, 3, 5, 4, 2 (d) 1, 3, 5, 2, 4 17. 1. Postbox 2. Letter 3. Envelope 4. Delivery
11. 1. Tired 2. Night 5. Clearance
(a) 3, 2, 4, 5, 1 (b) 3, 2, 1, 5, 4
3. Day 4. Sleep
(c) 3, 2, 1, 4, 5 (d) 2, 3, 1, 4, 5
5. Work
(a) 1, 3, 5, 2, 4 (b) 3, 5, 1, 4, 2 18. 1. Compose 2. Appreciation
(c) 3, 5, 1, 2, 4 (d) 3, 5, 2, 1, 4 3. Money 4. Think
12. 1. Key 2. Door 5. Sing
(a) 1, 4, 2, 5, 3 (b) 4, 1, 5, 2, 3
3. Lock 4. Room
(c) 2, 4, 3, 5, 1 (d) 5, 4, 2, 1, 3
5. Switch on
(a) 5, 1, 2, 4, 3 (b) 4, 2, 1, 5, 3 19. 1. Index 2. Contents 3. Title 4. Chapters
(c) 1, 2, 3, 5, 4 (d) 1, 3, 2, 4, 5 5. Introduction
(a) 3, 2, 5, 1, 4 (b) 2, 3, 5, 4, 1
13. 1. Table 2. Tree 3. Wood 4. Seed
(c) 5, 1, 4, 2, 3 (d) 3, 2, 5, 4, 1
5. Plant
(a) 4, 5, 3, 2, 1 20. 1. Windows 2. Walls 3. Floor 4. Foundation
(b) 4, 5, 2, 3, 1 5. Roof 6. Room
(c) 1, 3, 2, 4, 5 (a) 4, 5, 3, 2, 1, 6 (b) 4, 3, 5, 6, 2, 1
(d) 1, 2, 3, 4, 5 (c) 4, 2, 1, 5, 3, 6 (d) 4, 1, 5, 6, 2, 3

ANSWERS
1. (c) 2. (d) 3. (a) 4. (c) 5. (c) 6. (c) 7. (b) 8. (a) 9. (b) 10. (d)
11. (c) 12. (d) 13. (b) 14. (c) 15. (b) 16. (c) 17. (b) 18. (b) 19. (a) 20. (c)

https://sscstudy.com/
https://sscstudy.com/

Hints & Solutions


1. (c) Meaningful order of events, 8. (a) Meaningful order of events, 15. (b) Meaningful order of events,
Flower → Bee → Honey → Wax Eggs → Larva → Pupa → Moth Market → Vegetable → Cutting
i.e. 2, 3, 1, 4 i.e. 4, 2, 1, 3 → Cooking → Food
i.e. 2, 1, 3, 4, 5
2. (d) Meaningful order of events, 9. (b) Meaningful order of events,
Egg → Worm → Cocoon → Butterfly Child → Cry → Mother → Milk → Smile 16. (c) Meaningful order of events,
i.e. 3, 4, 2, 1 Grass → Grasshopper → Frog → Snake
i.e. 2, 4, 1, 3, 5
→ Eagle
3. (a) Meaningful order of events i.e. 5, 3, 1, 4, 2
10. (d) Meaningful order of events,
Cotton → Thread → Weaving → Cloth
Study → Examination → Apply → Job → Earn 17. (b) Meaningful order of events,
i.e. 2, 4, 1, 3
i.e. 1, 3, 5, 2, 4
Envelope → Letter → Postbox → Clearance
4. (c) Meaningful order of events, 11. (c) Meaningful order of events, → Delivery
Letter → Word → Phrase → Sentence i.e. 3, 2, 1, 5, 4
Day → Work → Tired → Night → Sleep
i.e 2, 3, 1, 4 18. (b) Meaningful order of events,
i.e. 3, 5, 1, 2, 4
5. (c) Meaningful order of events, Think → Compose → Sing → Appreciation
12. (d) Meaningful order of events,
Village → Block → District → State → Money
Key → Lock → Door → Room → Switch on i.e. 4, 1, 5, 2, 3
i.e. 2, 4, 1, 3 i.e. 1, 3, 2, 4, 5
19. (a) Meaningful order of events,
6. (c) Meaningful order of events, 13. (b) Meaningful order of events, Title → Contents → Introduction → Index
Seed → Plant → Flower → Fruit Seed → Plant → Tree → Wood → Table → Chapter
i.e. 3, 2, 5, 1, 4
i.e. 3, 1, 4, 2 i.e. 4, 5, 2, 3, 1
20. (c) Meaningful order of events,
7. (b) Meaningful order of events, 14. (c) Meaningful order of events,
Dawn → Noon → Twilight → Night Foundation → Walls → Windows → Roof
Crime → Arrest → Judgment → Punishment → Floor → Room
i.e. 2, 3, 1, 4 → Prison
i.e. 4, 3, 5, 1, 2 i.e. 4, 2, 1, 5, 3,6

https://sscstudy.com/
https://sscstudy.com/

GENERAL MENTAL ABILITY 50

CHAPTER 10

Inserting the Missing


Characters
Inserting the missing character is filling-up of the empty Sol. (b) In the first column, 12 × (18 ÷ 3) = 72
spaces in letter and number puzzles given in pictorial In the third column, 16 × (32 ÷ 4) = 128
forms. Let the missing number be x.
In such type of questions, some figures are given bearing Then, in the second column, we have
certain characters, numbers or a combination of both 14 × (24 ÷ x) = 112
which follow a particular rule for their placement at ⇒ 24 ÷ x = 8 or x = 3
different positions of the figure. The candidate is
required to find this pattern and insert the missing Ex. 4 Find the missing alphabet in the figure.
character or number in the figure based on the method
? B
used.
Ex. 1 Select the missing number from given P H
responses. J L
81 729 64 512 49 ?
(a) U (b) V
(c) T (d) X
2 +8 10
9 8 7
Sol. (c) As, B → J
(a) 444 (b) 515 (c) 343 (d) 373 2 +8 16
Sol. (c) As, 9 × 9 = 81 H → P
12 +8 20
⇒ 81 × 9 = 729 and 8 × 8 = 64 Similarly, L → T
⇒ 64 × 8 = 512 So, missing character = T
Same as, 7 × 7 = 49 ⇒ 49 × 7 = 343
Ex. 5 Find the missing character in the figure.
Ex. 2 Select the missing number from the given
responses. P G
? 14 F N
22 21
5 10
7 9 27 15
K E
? J
(a) 2 (b) 3 (c) 4 (d) 18
Sol. (d) As, 7 × 2 = 14 and 5 × 2 = 10 (a) M (b) P
Similarly, 9 × 2 = 18 (c) Q (d) S
Sol. (b) If we put A = 1, B = 2, Y = 25, Z = 26, we have
Ex. 3 Find the missing number in the figure.
F + P = 6 + 16 = 22
18 24 32 G + N = 7 + 14 = 21
12 14 16 J + E = 10 + 5 = 15
3 ? 4 Since, K = 11 so value corresponding to missing letter
72 112 128 = (27 − 11 ) = 16.
So, the missing letter is the 16th letter of the English
(a) 2 (b) 3 (c) 4 (d) 5 alphabet which is P.

https://sscstudy.com/
https://sscstudy.com/

GENERAL MENTAL ABILITY 51

Practice Questions
Directions (Q. Nos. 1-27) In each of the following 10. 6
3 11
questions a set of figures carrying certain characters is 4 7 12
6 9 14
given. Assuming that the characters in each set follow a 12
9 17
similar pattern, find the missing character in each case. 13 16 ?

1. 4 5 4
(a) 21 (b) 19 (c) 23 (d) 29
1 10 3 2 14 4 3 ? 5
11. 4 5 9 15 8 6
2 3 6
5 10 ?
(a) 54 (b) 18 (c) 36 (d) 24
3 8 4 12 7 11
2. 4 9 2
3 5 7 (a) 8 (b) 10
8 1 ? (c) 15 (d) 19
(a) 9 (b) 6 (c) 15 (d) 14 27 54 42 84 ? 42
12.
3. 8 6 6
32 4 30 5 ? 7 9 14 7
(a) 38 (b) 36 (c) 42 (d) 40 (a) 8 (b) 21
(c) 14 (d) 16
4. 25
? 21
13. 466 398
1 17 341 282
250 ?
5 13
9
(a) 298 (b) 232
(a) 29 (b) 31 (c) 27 (d) 33 (c) 350 (d) 268
5. 14. 12 6 5
25 2
? 3 27 20 18 16 13 12 10 ? 11
9 4 13 9 4
4 5
(a) 9 (b) 12 (c) 8 (d) 13
(a) 10 (b) 16 (c) 25 (d) 20 15. 1 4 5
4 2 5
6.
2 4 5 2 2 3
3 6 ?
49 64 ?
3 3 4
(a) 125 (b) 196 (c) 169 (d) 144
(a) 12 (b) 8 (c) 16 (d)10
3 4 5 16. 4 10 6
7.
12 96 48
9 7 ?
16 152 ?
8 4 6 5 9 3 (a) 110 (b) 104 (c) 112 (d) 124
(a) 13 (b) 11 (c) 9 (d) 7
17. 7 6 9
8. 4 25 64 2 8 4
? 81 4 3 ?
16 9 49 36
36 42 26
(a) 105 (b) 81 (c) 121 (d) 100 (a) 5 (b) 2 (c) 3 (d) 4
9. 6 1 5 3 18.
5 2 10 8 18 6
28 26 ?
4 2 6 4 20 ? 30 10
3 4
30 9 39 13
(a) 32 (b) 36
(c) 30 (d) 28 (a) 10 (b) 12 (c) 14 (d) 8

https://sscstudy.com/
https://sscstudy.com/

52 CUET (UG) Section III : General Test

19. 6 2 9 7 8 1 23. A B D
444 63 P K G
2 8 4 4 2 1 V C ?
2 4 6 (a) N (b) K
? (c) M (d) O
2 1 3
24.
(a) 218 (b) 342 AZ BY
(c) 330 (d) 356
? CX
20. A D G
? M J (a) DK (b) DW
(a) N (b) Q (c) DL (d) DE
(c) R (d) P 25.
B J Q E ?
21. B
? 2 10 17 ? 8
V G
(a) H and 8 (b) 6 and U (c) H and 6 (d) 5 and H
Q L
26.
(a) A (b) Z P R D ?
(c) C (d) Y 1618 420

22. W
(a) T (b) U (c) B (d) Q
T
27. A 22 I 12 ?
Q
N K ? E 26 E 18 O ?

(a) F (b) G 13 15 U 6
(a) (b) (c) (d)
(c) H (d) L p 6 6 U

ANSWERS
1. (b) 2. (b) 3. (c) 4. (a) 5. (b) 6. (d) 7. (d) 8. (d) 9. (b) 10. (a)
11. (a) 12. (b) 13. (b) 14. (b) 15. (c) 16. (b) 17. (b) 18. (a) 19. (c) 20. (d)
21. (a) 22. (c) 23. (b) 24. (b) 25. (d) 26. (a) 27. (c)

https://sscstudy.com/
https://sscstudy.com/

Hints & Solutions


1. (b) Here, number inside the circle is equal As, in 1st figure 19. (c) Here, the pattern is as follows,
4 + 5 + 8 + 3 20
to the sum of the numbers around it. ⇒ = =5 In 1st figure
i.e. 4 + 3 + 2 + 1 = 10 ⇒ 5 + 4 + 3 + 2 = 14 4 4
In 2nd figure 1 2 3 3 2 1 1 2 3 3 2 1
Similarly, 4 + 5 + 6 + 3 = 18 ⇒ 6 2 9 → 9 2 6 2 8 4 → 4 8 2
9 + 15 + 12 + 4 40
⇒ = = 10
2. (b) Here, the sum of numbers in each row 4 4 926 − 482 = 444
= 15 Similarly, in 3rd figure In 2nd figure ⇒ 187 − 124 = 63
i.e. 4 + 9 + 2 = 15; 3 + 5 + 7 = 15 8 + 6 + 11 + 7 32 Similarly, in 3rd figure
⇒ = =8
Similarly, 8 + 1 + ? = 15 4 4 ⇒ 642 − 312 = 330
⇒ ? = 15 − 9
12. (b) The pattern is as follows, 20. (d) According to English alphabetical
∴ ? =6
As, 9 × 3 = 27 ; 9 × 6 = 54; 14 × 3 = 42 series,
16
3. (c) Here, the pattern is as follows, and 14 × 6 = 84 1
+3 +3
4
+3 +3
7 10 13
+3
A → D → G → J → M → P
8 × 4 = 32 ; 6 × 5 = 30 Similarly, 7 × 3 = 21
Similarly, 6 × 7 = 42 7 × 6 = 42 21. (a) According to English alphabetical
series,
4. (a) Here, the pattern is as follows, 13. (b) Here, the lower number is twice the 2
+5
7
+5
12
+5
17
+5
22
+5
1

1 + 4 = 5; 5 + 4 = 9; 9 + 4 = 13 difference of the first two numbers. B → G → L → Q → V → A


13 + 4 = 17 ; 17 + 4 = 21; 21 + 4 = 25 In 1st figure
22. (c) According to English alphabetical
25 + 4 = 29 ⇒ ( 466 − 341) × 2 = 125 × 2 = 250
series,
Similarly, in 2nd figure
5. (b) Here, the pattern is as follows 23
−3
20
−3
17
−3
14
−3
11
−3
8
−3
5
⇒ ( 398 − 282 ) × 2 = 116 × 2 = 232 W → T → Q → N → K → H → E
2 → 2 2 = 4 ; 3 → 3 2 = 9;
4 → 4 2 = 16 ; 5 → 5 2 = 25 14. (b) Here, the pattern is as follows, 23. (b) According to English alphabetical
In 1st figure series,
6. (d) Here, the pattern is as follows, ⇒ (27 + 18) − (12 + 13) = 45 − 25 = 20 1 2 4 7 11 +5
+3 +2 +4
2×3 6 4 × 3 12 A →
+1
B → D → G → K →
= =3 ⇒ = =6 In 2nd figure 11
2 2 2 2 16 22 3
+8
⇒ (16 + 12 ) − ( 6 + 9) = 28 − 15 = 13 +6
P → +7
V → C → K
5 × 4 20
Similarly, = = 10 Similarly, in 3rd figure
2 2 24. (b) Here, the sum of position of alphabets
⇒ (10 + 11) − ( 5 + 4) = 21 − 9 = 12
7. (d) Here, the pattern is as follows, in each quadrant is 27
15. (c) Here, the pattern is as follows, A + Z = 1 + 26 = 27
8 + 4 − 3 = 9; 5 + 6 − 4 = 7
In 1st column B + Y = 2 + 25 = 27
Similarly, 9+ 3− 5 = 7
⇒ (1 + 4 + 2 )2 = 7 2 = 49 C + X = 3 + 24 = 27
8. (d) Here, the numbers in each circle are In 2nd column Similarly,D + W = 4 + 23 = 27 ⇒ DW
squares of consecutive numbers ⇒ ( 4 + 2 + 2 )2 = ( 8)2 = 64
i.e. 2 2 = 4, 3 2 = 9 Similarly, in 3rd column
25. (d) Here, in each box letters of English
alphabetical series and their corresponding
and 4 2 = 16 and 5 2 = 25, ⇒ 2
( 5 + 3 + 5) = 169 positions are given,
6 2 = 36 and 7 2 = 49 As, B = 2, J = 10 , Q = 17
16. (b) Here, the pattern is as follows,
Similarly, 8 2 = 64, 9 2 = 81 Similarly, E = 5 and H = 8
In 1st row ⇒ 4 2 = 16 = 10 + 6
2
and 10 = 100 In 2nd row ⇒ 12 2 = 144 = 96 + 48 26. (a) Here, the number is obtained from the
position of alphabets in English alphabetical
9. (b) Here, the pattern is as follows, Similarly, in 3rd row
series,
( 5 + 2 + 4 + 3) × 2 = 28 16 2 = 256 = 152 + ?
As, P R ⇒ 1 6 1 8
( 6 + 1 + 2 + 4) × 2 = 26 ⇒ ? = 256 − 152 ⇒ ? = 104
Similarly, ( 5 + 3 + 6 + 4) × 2 = 36
17. (b) Here, the pattern is as follows,
Similarly, D T ⇒ 4 2 0
10. (a) Here, the pattern is as follows, In 1st column
In 1st figure ⇒ (7 + 2 ) × 4 = 9 × 4 = 36
⇒ 3 + 1 = 4, 4 + 2 = 6, 6 + 3 = 9, 9 + 4 = 13 In 2nd column
In 2nd figure ⇒ ( 6 + 8) × 3 = 14 × 3 = 42 27. (c) Here, the vowels appear in zigzag form
⇒ 6 + 1 = 7, 7 + 2 = 9; 9 + 3 = 12 , 12 + 4 = 16 Similarly, in 3rd column from top to bottom and numbers at their
Similarly, in 3rd figure ( 9 + 4) × ? = 26 ⇒ ? = 2 respective positions in reverse English
alphabetical series.
⇒ 11 + 1 = 12 ; 12 + 2 = 14; 14 + 3 = 17 18. (a) Here, the pattern is as follows
and 17 + 4 = 21 10 + 8 = 18 ; 18 ÷ 3 = 6 ? U
∴ =
11. (a) Here, the number in the middle is equal 30 + 9 = 39 ; 39 ÷ 3 = 13 ? 6
to the average of numbers at the corners. Similarly, 20 + ? = 30 ; ? = 30 − 20 = 10

https://sscstudy.com/
https://sscstudy.com/

54 CUET (UG) Section III : General Test

CHAPTER 11

Ranking Test
The process of determining the position of a person/thing Sol. (c) According to the question,
on the basis of comparison or relative position of other
person/thing is called ranking.
There are two types of questions which are asked
1. Based on Comparison Rohit Swati Manoj Sumit Ashish
2. Based on Position (Top/Left or Right/Bottom)
Rohit > Swati; Manoj < Swati
or Swati > Manoj; Sumit > Rohit
1. Based on Comparison and Ashish is shortest.
In this type of questions, comparison of different objects Arranging the above data, we get
based on some factors like ages, marks, size, height etc Sumit > Rohit > Swati > Manoj> Ashish
is given. The candidate is required to arrange the data Clearly, Sumit is the tallest.
in ascending or descending order and then answer the
related question. 2. Based on Position
(Top/Left or Right/Bottom)
Ex. 1 Ram is taller than Mohan and Sohan, while
Sohan is taller than Shyam. Also, Mohan is taller In this type of questions, the position of person (s) from
than Sohan. Who amongst the following is the either of the two ends of a row is given. The candidate is
shortest? required to find the total number of persons in the row
(a) Shyam (b) Mohan or number of persons to the top/left or right/bottom of a
(c) Ram (d) Sohan particular person etc.
Sol. (a) According to the question, While solving this type of questions, keep the following points
Ram > Mohan and Ram > Sohan in mind.
Sohan > Shyam 1. Total number of persons in a row
Mohan > Sohan = (Rank of a person from left end/top
+ Rank of that person from right end/ bottom) − 1
2. Rank of a person from right end/bottom
= (Total number of persons in row)
− (Rank of that person from left end/top) + 1
Ram Mohan Sohan Shyam 3. Rank of a person from top/left end
= (Total number of persons in row)
Arranging the above data, we get – (Rank of that person from right end/bottom) + 1
Ram > Mohan > Sohan > Shyam
Clearly, Shyam is the shortest.
4. In case of interchanging of positions
(a) Total number of persons
Ex. 2 In a group of five friends, Rohit is taller than = [Initial position of 1st person
Swati. Also, Manoj is shorter than Swati. Sumit is + Interchanged position of 2nd person] − 1
taller than Rohit while Ashish is shortest. Who
(b) New position of 2nd person
amongst them is the tallest? = [Difference in the two positions of 1st person]
(a) Swati (b) Rohit
(c) Sumit (d) Manoj + [Initial position of 2nd person]

https://sscstudy.com/
https://sscstudy.com/

GENERAL MENTAL ABILITY 55

Ex. 3 In a class of 45 students rank of Ayush from Sol. (c) Position of Sandeep from left (L) = 10
the top is 15, then rank of Ayush from bottom is Position of Sandeep from right (R) = 16
(a) 30 (b) 32 ∴ Total number of people in the row = (L + R) − 1
(c) 31 (d) 35
= 10 + 16 − 1 = 25
Sol. (c) Here, total number of students = 45
Ex. 5 In a row of students, Anil is 7th from left,
Rank from top = 15
while Sunil is 18th from right. Both of them
∴ Rank of Ayush from bottom
interchanged their positions such that Anil becomes
= (Total number of persons in the class)
21st from left. What will be the total number of
– (Rank of Ayush from top) + 1
students in the class?
= 45 − 15 + 1 = 31
(a) 38 (b) 33
Alternate Method (c) 31 (d) 30
Sol. (a) 7th 18th

15th Anil Sunil


45 Students

Sunil Anil
Total number of students = 45 21st
Given, the rank of Ayush from top is 15, it means there
∴ Anil moves (21–7)
are 45 − 15 = 30 students below Ayush in the class.
= 14 places ahead and changed his place with Sunil.
∴ The rank of Ayush from bottom = 30 + 1 = 31
∴ Sunil will also move 14 places ahead.
Ex. 4 In a row, position of Sandeep is 10th from left
∴ Sunil's new position from right end = 18 + 14 = 32
and 16th from right. How many people are there in
We can also calculate the total number of students
the row?
= (Anil's initial position) + (Sunil's new position) −1
(a) 30 (b) 26
(c) 25 (d) 16 = 7 + 32 − 1 = 38

Practice Questions
1. A pole is taller than a giraffe which is taller than a 5. If Anita is taller than Surjit but shorter than Kusum
tree. A signal is shorter than a pole but taller than a and Surjit is just as tall as Kalpana but taller than
building which is taller than a giraffe. Who is the Vanita, then Kalpana is
shortest? (a) just as tall as Anita
(a) Signal (b) Giraffe (b) taller than Kusum
(c) Tree (d) Building (c) shorter than Anita
2. Mohan is taller than Rohan but shorter than Farhan. (d) shorter than Surjit
Kannan is shorter than Mohan but taller than Rohan,
6. In a group of five districts Akbarpur is smaller than
Shankar is taller than Rohan and Farhan. Who is the
Fatehpur, Dhanbad is bigger than Palamu, and
tallest?
Barabanki is bigger than Fatehpur but not as big as
(a) Mohan (b) Farhan
Palamu. Which district is the biggest?
(c) Shankar (d) Kannan
(a) Akabarpur (b) Fatehpur
3. Sameer is older than Milon but not as old as Arjun, (c) Palamu (d) Dhanbad
Priya is older than Sam but not as old as Milon. Who
amongst them is the oldest? 7. Five friends A, B, C, D and E are sitting in a row in
(a) Milon (b) Sameer decreasing order of their weight from left to right.
(c) Priya (d) Arjun A is heavier than only C, while B is heavier than
4. Five girls took part in a race. Sonam finished before both E and A. D is heavier than B. Who amongst
Neha but after Pooja. Shivani finished before Anju but the following is sitting exactly in centre between
after Neha. Who won the race? them?
(a) Sonam (b) Pooja (a) C (b) A
(c) Neha (d) Shivani (c) E (d) B

https://sscstudy.com/
https://sscstudy.com/

56 CUET (UG) Section III : General Test

Directions (Q. Nos. 8-10) Consider the following 15. If you are 9th person in a queue starting from one end
information and answer questions based on it. and 11th from another end, what is the number of
persons in the queue?
Suresh is taller than Ramesh who is shorter than Rakesh. (a) 20 (b) 19
Jinesh is taller than the shortest person but shorter than (c) 21 (d) 18
Rakesh. Pritesh is taller than Suresh but shorter than
Jinesh. 16. Rajesh’s rank is 12th from top and 30th from bottom
among the children who passed the annual
8. Who is the shortest? examination. If 16 children failed, then find the total
(a) Rakesh (b) Ramesh number of children who appeared for the
(c) Pritesh (d) Suresh examination.
(a) 57 (b) 33
9. If Jayesh who is taller than Suresh joins the group, (c) 68 (d) 45
who will be fourth, if they are arranged in a
descending order? 17. In an examination, Rahul got 11th rank from top and
(a) Jayesh (b) Jinesh he was 47th from the bottom among those who
(c) Pritesh (d) Cannot be determined passed. 3 students could not appear for the exam. 1
student failed. What is the total number of students?
10. Which statement, among the following is correct? (a) 60 (b) 62
(a) Suresh is taller than Jinesh
(c) 59 (d) 61
(b) Pritesh is shorter than Rakesh
(c) Jinesh is taller than Rakesh 18. In a queue of 17 people, when Seema shifts 3
(d) Ramesh is taller than Pritesh positions left, then she becomes 6th from left. Find
11. In a queue of girls, Shama is 11th from the starting her previous position in the queue from right.
and 27th from the last. How many girls are there in (a) 5 (b) 2
the queue? (c) 9 (d) 3
(a) 37 (b) 43
(c) 35 (d) 36
19. In a queue, Mohan is 10th from right side and Sohan
is 25th from left side. When they interchange their
12. In a row of 50 people, Sachin is 24th from right end. place, then Mohan is at 22nd place from right. Find
What is Sachin's position from left end? Sohan's new position from left.
(a) 30th (b) 28th (a) 37th (b) 38th
(c) 27th (d) 24th (c) 35th (d) 36th
13. In the name list of 48 students, Rohit’s rank is 41st
20. In a students queue, Kamal is 12th from left and
from bottom. Find his rank from top.
(a) 12th (b) 8th (c) 7th (d) 9th Deepak is 18th from right. When Kamal and Deepak
interchange their position, then Kamal is 25th from left.
14. In a row of trees, one tree is 7th from either end of the Find the total number of students in the queue.
row. How many trees are there in the row? (a) 40 (b) 41
(a) 11 (b) 15
(c) 42 (d) 43
(c) 14 (d) 13

ANSWERS
1. (c) 2. (c) 3. (d) 4. (b) 5. (c) 6. (d) 7. (c) 8. (b) 9. (d) 10. (b)
11. (a) 12. (c) 13. (b) 14. (d) 15. (b) 16. (a) 17. (d) 18. (c) 19. (a) 20. (c)

https://sscstudy.com/
https://sscstudy.com/

Hints & Solutions


1. (c) According to the question, 7. (c) According to the question, 16. (a) Total number of students who passed
Pole > Giraffe > Tree K (i) A is heavier than only C, that means A is at 4th = (12 + 30) − 1 = 41
Pole > Signal > Building > Giraffe K (ii) place and C is at 5th place from left. Number of students who failed = 16
From (i) and (ii), we get A > C; B > E, A or B > E > A > C ∴Total number of students who appeared for
Pole > Signal > Building > Giraffe > Tree Also, D>B ⇒ D > B > E > A > C the exam = 41 + 16 = 57
Clearly, tree is the shortest. 1 2 3 4 5
17. (d) Total number of students who passed
Clearly, E is sitting exactly in centre between
2. (c) According to the question, = (11 + 47 ) − 1 = 57
them.
Ferhan > Mohan > Rohan K (i) Now, total number of students
Sol. (Q. Nos. 8-10) According to the question,
Mohan > Kannan > Rohan K (ii) = 57 + 3 + 1 = 61
Suresh > Ramesh K (i)
Shankhar > Rohan , Ferhan K (iii)
Rakesh > Ramesh K (ii) 18. (c) Seema shifts 3 positions towards left.
From (i), (ii) and (iii), we get
Rakesh > Jinesh K (iii) So, in order to find her original position we
Shankhar>Ferhan>Mohan> Kannan>Rohan have to shift her 3 places towards right.
Jinesh> Pritesh > Suresh K (iv)
Clearly, Shankhar is the tallest. ∴Seema’s original position from left
From (i), (ii), (iii) and (iv), we get
3. (d) According to the question, Rakesh > Jinesh > Pritesh > Suresh > = 6+ 3 = 9
Arjun > Sameer> Milon Ramesh Also, total number of people = 17
and Milon > Priya > Sam
8. (b) Clearly, Ramesh is the shortest. ∴Total number of people
Arranging in meaningful order, we get = (Position from left + Position from right) −1
9. (d)
Arjun > Sameer > Milon > Priya > Sam ∴ 17 = ( 9 + R ) − 1 ⇒ R = 9
Rakesh > Jinesh > Pritesh > Suresh > Ramesh
Clearly, Arjun is the oldest. ∴Seema is 9th from the right.
Jayesh
4. (b) According to the question, 19. (a)
Hence, it cannot be determined that who is the 25th 10th
Pooja > Sonam > Neha fourth.
and Neha > Shivani > Anju Sohan Mohan
Arranging the data in meaningful order, we get
10. (b) Clearly, Pritesh is shorter than Rakesh.
Pooja > Sonam> Neha > Shivani > Anju 11. (a) We know that, Mohan Sohan
Clearly, Pooja won the race. Total number of girls in queue = (Rank of girl
from starting + Rank of girl from last) −1 22nd
5. (c) According to the question, = 11 + 27 − 1 = 37 ∴ Mohan moves (22 −10) = 12 places ahead
Kusum>Anita>Surjit and changed his place with Sohan.
and Surjit = Kalpana > Vanita
12. (c) Total number of people = ( Position
∴Sohan will also move 12 places ahead.
from left + Position from right ) −1
Arranging in meaningful order, we get ∴Sohan's new position from left end
⇒ 50 = (24 + L ) − 1 ⇒ L = 51 − 24 = 27
Kusum> Anita > Surjit = Kalpana > Vanita = 25 + 12 = 37th
∴ Sachin is 27th from left end.
Clearly, Kalpana is shorter than Anita.
13. (b) Total number of students 20. (c) 12th 18th
6. (d) According to the question, = ( Rank from top + Rank from bottom ) −1 Kamal Deepak
Barabanki >Fatehpur > Akbarpur 48 = ( T + 41) − 1 ⇒ T = 8
and Dhanbad > Palamu ∴Rohit is 8th from top.
and Palamu > Barabanki Deepak Kamal
14. (d) Here, position of tree from both the end
Arranging the data in meaningful order, we get 25th
is 7.
Dhanbad > Palamu > Barabanki ∴ Total number of students
∴ Total number of trees = 7 + 7 − 1 = 13
> Fatehpur> Akbarpur = ( Deepak's initial position
15. (b) Total number of persons = ( Position
Clearly, Dhanbad district is biggest amongst + Kamal’s new position ) −1
them. from left end + Position from right end ) −1
= 18 + 25 − 1 = 42
= ( 9 + 11) − 1 = 19

https://sscstudy.com/
https://sscstudy.com/

58 CUET (UG) Section III : General Test

CHAPTER 12

Sitting Arrangement
Sitting arrangement refers to, how a group of persons or 3. M is not at the centre.
objects allocated their positions in a particular manner. 4. K is to the immediate right of N and immediate left of L.
In these questions, a group of persons or objects with Ex. 2 The student standing to the immediate left of
some predefined conditions is given, on the basis of O is
these conditions candidates are required to arrange (a) L (b) J (c) M (d) P
persons objects in an order as given in the question.
Ex. 3 Who is at the centre of the row?
Based on the order of sitting, questions can be divided (a) N (b) J (c) P (d) K
into following types.
Ex. 4 Who stands fifth from the left of the row?
1. Linear arrangement. 2. Circular arrangement
(a) K (b) P (c) N (d) J
3. Square/Rectangular arrangement
Sol. (Ex. Nos. 2-4) According to the question,
M O J P N K L
1. Linear Arrangement Left Right
In linear arrangement, person/objects are required to be 2. (c) M is standing to the immediate left of O.
placed in proper order in a straight line. 3. (c) P is at the centre of the row.
4. (c) N is standing fifth from the left end of the row.
Left Right

One is required to identify the exact position of the 2. Circular Arrangement


objects and their positions with respect to one another In this type of arrangement, objects/persons are placed
based on the information given. around a circle facing the centre.
A
Ex. 1 Five friends P, Q, R, S and T are sitting in a
row facing North. Here, S is between T and Q and Q
is to the immediate left of R. P is to the immediate D C
Centre
left of T. Who is in the middle?
(a) S (b) T (c) Q (d) R Left Right
B
Sol. (a) Arrangement according to the question is as
follows. Ex. 5 Six girls are sitting in a circle facing towards
P T S Q R the centre. Bindu is to the left of Viji. Rekha is in
(Left) (Right) between Bindu and Mumtaz. Jessa is in between Viji
Facing
(Middle) North
and Nirmala. Who is to the left of Mumtaz?
(a) Rekha (b) Nirmala (c) Viji (d) Bindu
∴ S is in the middle of the row.
Sol. (b) According to the question,
Directions (Ex. Nos. 2-4) Consider the following Mumtaz
information and answer questions based on it. Rekha Nirmala

Seven students J, K, L, M, N, O and P are standing in a


row, in random order, from left to right, such that
Bindu Jessa
1. P, O, K and N do not stand on any of the extreme ends.
Viji
2. J is to the immediate left of P and immediate right
of O. Hence, Nirmala is to the left of Mumtaz.

https://sscstudy.com/
https://sscstudy.com/

GENERAL MENTAL ABILITY 59

3. Square/Rectangular Ex. 6 A, B, C and D are playing cards. A and C, and


B and D are partners. D is to the right of C. The face of
Arrangement C is towards West. Find the direction that D is facing.
In this type of arrangement, four objects/persons are (a) West (b) East (c) South (d) North
placed around a square or a rectangular shaped table Sol. (c) Arrangement according to the question is as
facing the centre. follows.
A D
N
Right A C Left W E
Right D C Left
S
B

B
Clearly, D is facing South.

Practice Questions
1. Five friends are sitting in a row facing South. Here, 7. Five boys are sitting in a row. A is on the right of B, E
Mohan is between Balu and Raju and Raju is to the is on the left of B, but to the right of C. If A is on the
immediate right of Praveen and Amith is to the right left of D. Who is sitting in the middle?
of Balu. Who is in extreme right end? (a) E (b) B
(a) Amith (b) Balu (c) A (d) C
(c) Praveen (d) Mohan
Directions (Q. Nos. 8-10) Read the following
2. Five students Priyanka, Mary, Sunil, Asha and Ryan information and answer the following questions.
are standing in a line. Priyanka and Sunil are Five girls are sitting in front, facing you. Ruhi and
standing ahead of Ryan. Sunil is standing between
Manali respectively are sitting to the right and left of
Ryan and Asha. Between Sunil and Mary, Ryan is
Urja. Dhwani is sitting between Manali and Tanya.
standing. Who is the first in the line?
(a) Asha (b) Sunil 8. How many girls are there between Dhwani and Ruhi ?
(c) Ryan (d) Priyanka (a) 1 (b) 2
(c) 3 (d) 4
3. Five different coloured buses are standing in a row
facing South. Black coloured bus is standing in the 9. Who is sitting in the middle ?
immediate right of Red. Green colour is between Blue (a) Dhwani (b) Urja
and Yellow. Yellow colour is between Black and Green. (c) Manali (d) Tanya
Which coloured bus is standing in the middle?
10. Who is sitting to the right of Tanya?
(a) Yellow (b) Blue
(c) Black (d) Green (a) Ruhi (b) Manali
(c) Dhwani (d) Urja
4. Six friends A, B, C, D, E and F are sitting in a row
facing East. C is between A and E. B is just to the Directions (Q. Nos. 11-13) Use the following passage
right of E but left of D. F is not at the right end. Who for the questions based on it.
is at the right end? There are six flats on a floor in two rows. Out of these,
(a) D (b) B three are North facing and the other three are South
(c) E (d) C facing flats. The flats are to be allotted amongst Arun,
5. There are five different houses, A to E, in a row. A is Biswajyot, Chitra, Derek, Evan and Fatima. Biswajyot
to the right of B and E is to the left of C and right of gets a North facing flat and is not next to Derek. Derek
A, B is to the right of D. Which of the houses is in the and Fatima get diagonally opposite flats. Chitra is next
middle? to Fatima and gets a South facing flat.
(a) A (b) B Evan gets a North facing flat.
(c) C (d) D
11. Except Derek and Fatima, which other pair is
6. A, B, C, D, E and F are sitting in a row. E and F are diagonally opposite to each other?
in the centre and A and B are at the ends. C is sitting (a) Arun and Biswajyot
on the left of A. Then, who is sitting on the right of B? (b) Arun and Chitra
(a) A (b) D (c) Evan and Derek
(c) E (d) F (d) Evan and Chitra

https://sscstudy.com/
https://sscstudy.com/

60 CUET (UG) Section III : General Test

12. Which of the following pairs is exactly opposite to each Directions (Q. Nos. 19-21) Read the following
other? passage and answer the questions based on it.
(a) Derek and Evan (b) Fatima and Chitra (i) Vinay, Vinod, Vikas, Vasu, Vaibhav, Varun and
(c) Evan and Chitra (d) Evan and Arun Vishal are sitting around a circle facing at the
13. Which of the following combination gets South facing centre.
flats? (ii) Vasu is the third to the right of Vikas.
(a) Arun, Chitra, and Fatima (iii) Varun is second to the left of Vinay and second to
(b) Chitra, Biswajyot and Derek the right of Vishal.
(c) Evan, Arun and Fatima
(d) Derek, Arun and Biswajyot (iv) Vaibhav is third to the right of Vinod and is not an
immediate neighbour of Varun.
14. Five persons are sitting facing centre of a circle. Pramod
is sitting to the right of Ranjan. Raju is sitting between 19. Who is second to the right of Vinod?
Brejesh and Naveen. Raju is to the left of Brejesh and (a) Vaibhav (b) Vasu
Ranjan is to right of Brejesh. Who is sitting to the left of (c) Vishal (d) Vikas
Naveen?
20. Who is third to the left of Varun?
(a) Pramod (b) Raju (c) Brejesh (d) Rajan
(a) Vaibhav (b) Vasu
15. Six persons are sitting in a circle facing the centre of (c) Vishal (d) Vikas
the circle. Parikh is between Babita and Narendra.
Asha is between Chitra and Pankaj. Chitra is to the 21. Who is to the immediate right of Vikas?
immediate left of Babita. Who is to the immediate (a) Vinay (b) Vishal (c) Vaibhav (d) Varun
right of Babita? 22. Radha, Sheela, Mahima and Seeta are sitting along a
(a) Parikh (b) Pankaj rectangular table. Radha is to the right of Sheela.
(c) Narendra (d) Chitra Mahima is to the left of Seeta. Which of the persons
16. Six persons are sitting around a circular table. Ajay is given in the options are sitting opposite to each other?
facing Hemant who is sitting to the left of Arvind and (a) Sheela-Seeta (b) Radha-Seeta
right of Sanjay. Suman is to the right of Arvind. (c) Radha-Sheela (d) Mahima-Radha
Manoj is facing Arvind. Who is now sitting to the
right of Manoj? Directions (Q. Nos. 23-25) Consider the following
(a) Arvind (b) Ajay information and answer the questions based on it.
(c) Suman (d) None of these Six animals - Horse, Cow, Pig, Dog, Donkey and Goat
are tied to a pole each, in a circle, facing each other, in
17. A, B, C, D, E, F and G are playing cards sitting in a random order.
circle.
1. Goat is to the immediate right of Pig.
(i) F is 2nd to the right of G.
2. Cow is not tied next to either Donkey or Dog.
(ii) B is neighbour of F but not of E.
3. If the animals mark the vertices of a hexagon, then
(iii) E, the neighbour of C, is 4th to the right of G.
the Horse is diagonally opposite to the Pig.
(iv) D is between E and A.
Who is fourth to the left of G? 23. The Cow is tied to the immediate left of
(a) D (a) Goat (b) Pig
(b) E (c) Horse (d) Dog
(c) C
24. Which pair is tied next to each other?
(d) Cannot be determined
(a) Horse and Goat (b) Pig and Cow
18. Four persons M, N, O and P are playing cards. M is on (d) Goat and Dog (d) Donkey and Dog
the right of N and P is on left of O. Then which of the
25. Which animal is tied to the immediate left of Pig?
following are partners?
(a) Goat (b) Donkey
(a) P and O (b) M and P
(c) Dog (d) Cannot be determined
(c) M and N (d) N and P

ANSWERS
1. (a) 2. (d) 3. (a) 4. (a) 5. (a) 6. (b) 7. (b) 8. (b) 9. (c) 10. (c)
11. (a) 12. (c) 13. (a) 14. (a) 15. (a) 16. (d) 17. (c) 18. (d) 19. (d) 20. (a)
21. (c) 22. (b) 23. (c) 24. (d) 25. (d)

https://sscstudy.com/
https://sscstudy.com/

Hints & Solutions


1. (a) According to the question, sitting Sol. (Q. Nos. 11-13) According to the 18. (d) According to the question, the
arrangement will be question, arrangement is as follows,

Right Left South P


Mohan
Raju
Balu

Praveen
Amith

Fatima Chitra Arun Arun Chitra Fatima


Or O M
Biswajyot Evan Derek Derek Evan Biswajyot
Thus, Amith is at extreme right end.
North N
2. (d) According to the question,
Priyanka 11. (a) Except Derek and Fatima, Arun and Clearly, N and P are partners.
Asha Biswajyot are diagonally opposite to each
Sunil other. Sol. (Q. Nos. 19-21) According to the
Ryan question,
Mary 12. (c) Evan and Chitra are exactly opposite
to each other. Vikas
Hence, Priyanka, is the first in the line. Vaibhav Vinay
13. (a) Arun, Chitra and Fatima get South
3. (a) According to the question, we have facing flats.
following standing arrangement of buses Vishal
Right Left 14. (a) According to the question,
Green

Red
Blue

Yellow
Black

Raju Vasu Vinod


Brijesh Naveen Varun
Hence, yellow coloured bus is standing in the
19. (d) Vikas is second to the right of Vinod.
middle.
20. (a) Vaibhav is third to the left of Varun.
4. (a) According to the question, we have the Pramod
Ranjan
following sitting arrangement. 21. (c) Vaibhav is to the immediate right of Vikas.
F A C E B D Hence, Pramod is sitting on the left of Naveen.
22. (b) According to the question, the
West East
15. (a) According to the question, the arrangement is as follows,
Left Right
arrangement is as follows,
∴D is at the extreme right. Mahima
Narendra Parikh
5. (a) According to the question, the Seeta Radha
arrangement is as follows,
Pankaj Babita
D B A E C Sheela
Hence, House A is in the middle. Clearly, Radha and Seeta are sitting opposite
Asha Chitra to each other.
6. (b) According to the question, the
arrangement is as follows, Hence, Parikh is to the right of Babita. Sol. (Q. Nos. 23-25) According to the
B D
E/F F/E C A 16. (d) According to the question, the question,
Hence, D is on the right of B. arrangement is as follows, Donkey/Dog Horse
Manoj Ajay
7. (b) According to the question, the
Donkey/Dog

arrangement is as follows,
Sanjay Suman Cow
C E B A D
Hence, B is in the middle.
Hemant Arvind
Sol. (Q. Nos. 8-10) According to the question, Hence, Sanjay is on the right of Manoj. Pig Goat

Ruhi Urja Manali Dhwani Tanya 17. (c) According to the question, the 23. (c) Cow is tied to the immediate left of
arrangement is as follows,
Horse.
E
8. (b) Two girls are sitting between Dhwani D C
and Ruhi. 24. (d) Donkey and Dog are tied next to each
F other.
9. (c) Manali is sitting in the middle. A
25. (d) Either Dog of Donkey is tied to the
10. (c) Dhwani is sitting to the right of G B immediate left of Pig.
Tanya. Hence, C is fourth to the left of G.

https://sscstudy.com/
https://sscstudy.com/

62 CUET (UG) Section III : General Test

CHAPTER 13

Puzzle Test
Puzzle refers to something which is baffling or confusing
that is to be solved. Books Colour of cover Volume Law/Medical
In such questions based on Puzzle, some raw D Yellow New Medical
information is given about the persons or items that are
E Green Old Medical
to be arranged in a particular sequence or order. For
this candidates are required to study and analyse the F Green Old Medical
jumbled information given in the question. And finally
It can be seen from the above table that, E and F are old
link them together to form a diagrammatic
medical extracts and have green covers. Hence, option
representation to answer the questions. The question on
(b) is correct.
puzzle test can be based on classification, placing
arrangement, comparison, sequential order, based on Ex. 2 Read the given information carefully and
family, etc. answer the question based on it.
Ex. 1 Read the given information carefully and In a group of five persons A, B, C, D and E, one
answer the question based on it. person is a professor, one is a doctor and one is a
lawyer. A and D are unmarried females who don’t
Six books are labelled A, B, C, D, E and F are placed
work.
side by side. Books B, C, E and F have green covers
while others have yellow covers. Books A, B and D are There is a married couple in the group in which E is
new while the rest are old volumes. the husband, B is the brother of A and is neither
Books A, B and C are law reports while the rest are doctor nor a lawyer. Who is professor?
medical extracts. Which two books are old medical (a) B (b) C
(c) A (d) Data inadequate
extracts and have green covers?
Sol. (a) According to the question,
(a) B and C (b) E and F (c) C and F (d) D and F Married
couple
Sol. (b) According to the given information, the (+)
(Doctor/Lawyer) E ←→ C ( −) (Doctor / Lawyer)
arrangement is as follows
Brother A( − ) (Unmarried)
(Professor) B ( + ) ←→
Books Colour of cover Volume Law/Medical D( − ) (Unmarried)
A Yellow New Law A and D are workless.
B Green New Law B is neither doctor nor lawyer.
Hence, it is clear that B must be professor.
C Green Old Law

https://sscstudy.com/
https://sscstudy.com/

GENERAL MENTAL ABILITY 63

Practice Questions
1. A ate grapes and pineapple, B ate grapes and (iii) The person who belongs to Delhi is sitting to the
oranges, C ate oranges, pineapple and apple, D ate left of the person who belongs to Bhopal.
grapes, apple and pineapple. After taking fruits, B (iv) P is sitting opposite to C.
and C fell sick. A and B belong to which place ?
In the light of the above facts, it can be said that the (a) Bhopal and Lucknow (b) Lucknow and Shimla
cause of sickness was (c) Bhopal and Shimla (d) Bhopal and Delhi
(a) apple (b) pineapple
6. There are four boxes, J, K, L and M in which four
(c) grapes (d) oranges
types of fruits are stored viz. Litchi, Apple, Grapes
2. A, B, C, D and E belong to five different cities P, Q, R, and Mango. Boxes are arranged in a certain manner
S and T (not necessarily in that order). Each one of from top to bottom.
them comes from a different city. Further it is given There are two boxes between K and L. The box in
that, which Grapes are stored is above L, but not immediate
I. B and C do not belong to Q. above. The Box in which Apple is stored is immediate
II. B and E do not belong to P and R. below M, but not stored in Box L. Litchi box is above
III. A and C do not belong to R, S and T. the Mango box, but not immediate above Apple box.
IV. D and E do not belong to Q In which of the following box Litchi is stored?
and T. (a) J (b) M (c) K (d) L
Which one of the following statements is not correct?
(a) C belongs to P (b) D belongs to R 7. In a four day period–Monday through Thursday, each
(c) A belongs to Q (d) B belongs to S of the following temporary office workers worked only
one day, each on a different day. Jai was scheduled to
3. Six friends James, Deepak, Hari, Rahul, Vikram and work on Monday but he traded with Raj, who was
Imran went for a picnic where they decided to play originally scheduled to work on Wednesday. Farid
kabaddi. They divided themselves into two different traded with Kajal, who was originally scheduled to
teams with Deepak, Rahul and Vikram in Team-1 and work on Thursday. Finally, Jai traded with Kajal.
rest in Team-2. Deepak and Imran were fat, but others After all the switching was done, who worked on
were thin. Rahul and James were cheating, but other Tuesday?
played the game fairly. (a) Jai (b) Farid
Who was/were the friend/friends who played the game (c) Raj (d) Kajal
fairly from the Team-2?
(a) Imran and James (b) Imran alone 8. A, B, C, D, E, F and G are members of a family
(c) Hari and Imran (d) Hari alone consisting of 4 adults and 3 children, two of whom, F
and G are girls, A and D are brothers and A is a
4. Five friends, Ramesh, Jim, Manish, Nilesh, Sagar went doctor. E is an engineer married to one of the brothers
for interviews in five different companies A, B, C, D and and has two children. B is married to D and G is their
E, but not in the same order, wearing different ties viz. child. Who is C?
Green, Red, Blue, Yellow and Orange but not in the (a) G’s father (b) F’s father
same order. Nilesh did not go to company B and was not (c) E’s daughter (d) A’s son
wearing a Green or Orange colour tie. The person who
went to company B wearing a Blue colour tie and the 9. In a family of six, there are three men A, B and C and
person who went to company E was wearing a Yellow three women R, S and T. The six persons are
colour tie. Manish and Sagar did not go to the company Architect, Lawyer, CA, Professor, Doctor and Engineer
B. Jim went to company D and was wearing a Red by profession, but not in the same order. There are
coloured tie. Who is working in company B? two married couples and two unmarried persons. C is
(a) Sagar (b) Ramesh not R’s husband. The Doctor is married to a Lawyer.
(c) Nilesh (d) Manish R’s grandfather is a Professor. B is neither A’s son nor
he is an Architect or a Professor. The Lawyer is T’s
5. A, B, C and D are sitting around a circular table and daughter-in-law. C is T’s son and the Engineer’s
discussing about their home town. father. A is married to the CA.
(i) A is sitting opposite to the person who belongs to Which among the following is the correct pair of
Shimla. married couples?
(ii) B is sitting to the right of the person who belongs (a) AS and CR (b) AC and SR
to Lucknow. (c) AT and CS (d) AT and RB

https://sscstudy.com/
https://sscstudy.com/

64 CUET (UG) Section III : General Test

10. Eight students A, B, C, D, E, F, G and H are going to 15. White house is owned by
college in two cars and following are conditions. (a) Q (b) R
There are four students in each car. (c) S (d) Cannot be determined
A is in the same car in which D is sitting but H is not
Directions (Q. Nos. 16-18) Consider the following
in the same car.
information and answer the questions based on it.
B and C are not in the car in which D is sitting. The likes and professions of five friends Rosy, Mary, Lily,
F is sitting with A and E. Andy and Daisy are given.
Four students sitting in the same car are 1. Rosy likes yellow and is a student.
(a) A, B, G, H (b) B, D, F, G 2. The librarian likes green.
(c) B, C, G, H (d) A, C, D, E
3. Mary and Andy like purple and blue respectively
11. P, Q, R, S, T, U are 6 members of a family in which and neither of them is a teacher.
there are two married couples. T, a teacher, is married 4. Lily likes brown and the inspector likes blue.
to a doctor who is mother of R and U. Q, the lawyer, is
5. One of the five is a principal.
married to P. P has one son and one grandson. Of the
two married ladies one is a housewife. There is also 16. Which colour does the teacher like?
one student and one male engineer in the family. (a) Blue (b) Brown (c) Purple (d) Green
Which of the following is true about the 17. Who is the principal?
grand-daughter of the family? (a) Daisy (b) Andy (c) Mary (d) Lily
(a) She is a lawyer (b) She is an engineer
(c) She is a student (d) She is a doctor 18. Which among the following is the wrong pair?
(a) Andy-Inspector (b) Purple-Principal
12. Six students A, B, C, D, E, F are sitting on the (c) Daisy-Green (d) Rosy-Teacher
ground. A and B belong to Ruby House, while the rest
belong to Emerald House. D and F are tall, while Directions (Q. Nos. 19-21) Study the following
others are short. C and D are wearing glasses, while information carefully and answer the questions given
others are not wearing. Which girl of Emerald House below it.
who is tall and wearing glasses? 1. There are six members in a family in which there
(a) B (b) C
are two married couples.
(c) D (d) A
2. Sandhya, a lawyer, is married to the engineer and
Directions (Q. Nos. 13-15) Consider the following is mother of Charu and Suraj.
information and answer questions based on it. 3. Bhuvanesh, the teacher, is married Aruna.
P, Q, R and S are four friends who pursue teaching, law, 4. Aruna has one son and grandson.
banking, cooking and own red, blue, white and yellow 5. Of the two married ladies one is a housewife.
house, in a random order. An individual owns only one 6. There is also one student and one male doctor in
house and pursues only one profession. the family.
1. P owns a red house and is not a banker.
2. The owner of blue house is lawyer. 19. How is Aruna related to Charu?
(a) Sister (b) Mother
3. The colour of S’s house is neither yellow nor white.
(c) Grandfather (d) Grandmother
4. R is a teacher.
20. Who among the following is the housewife?
13. The owner of the blue house is
(a) Charu (b) Aruna
(a) S (b) R
(c) Sandhiya (d) None of the above
(c) Q (d) Cannot be determined
21. Which of the following is true about the
14. Q is a
grand-daughter in the familty?
(a) Lawyer (b) Banker
(a) She is a doctor (b) She is a teacher
(c) Cook (d) Teacher
(c) She is a student (d) Data inadequate

ANSWERS
1. (d) 2. (d) 3. (d) 4. (b) 5. (c) 6. (c) 7. (a) 8. (d) 9. (c) 10. (c)
11. (c) 12. (c) 13. (a) 14. (b) 15. (d) 16. (b) 17. (c) 18. (d) 19. (d) 20. (b)
21. (c)

https://sscstudy.com/
https://sscstudy.com/

Hints & Solutions


1. (d) Hence, A and B belong to Bhopal and Shimla, 12. (c) We have total six student sitting on the
respectively. ground
Fruit
Apple Pineapple Grapes Oranges 6. (c) Ruby House → A, B
Person Box Fruit
Emerald House → C, D, E, F; Tall → D, F
A û ü ü û K Litchi
Short → A, B, C, E
B û û ü ü M Grapes Wearing glasses → C, D
C ü ü û ü J Apple Wearing no glasses → A, B, E, F
D L Mango We have to find out girl of Emerald House who
ü ü ü û
Since, B and C fell sick, so the cause of ∴ Litchi is stored in Box K. is tall and wearing glasses.
sickness is oranges which is eaten by both of So, we have to find out the common students
them.
7. (a) in the given condition.
Day Originally Finally Therefore, D is the only student who is wearing
2. (d) From the given information, following Monday Jai Raj Raj glasses, tall and belongs to Emerald Houses.
table can be drawn
Tuesday Farid Kajal Jai Sol. (Q. Nos. 13-15) According to the
City Wednesday Raj Jai Kajal question, the arrangement is as follows
P Q R S T
Person Thursday Kajal Farid Farid Person Profession House colour
A û ü û û û Hence, Jai worked on Tuesday. P Cooking Red
B û û û û ü Q Banking White/Yellow
8. (d) Family diagram is as follows
C ü û û û û (–) Married (+) R Teacher Yellow/White
Brother
(Engineer) E A (Doctor) D
D û û ü û û S Lawyer Blue
E û û û ü û (–) (+) 13. (a) S is the owner of blue house.
F C
Now, it is clear from the above table that B does 14. (b) Q is a banker.
(+) Married (–)
not belong to S. So, statement (d) is incorrect. B
(+) = Male 15. (d) White house is owned by either Q or R.
3. (d) According to the question, the (–)
Sol. (Q. Nos. 16-18) According to the
G (–) = Female
arrangement is as follows question,
So, C is A’s son.
Friend Team Size Playing Nature Person Colour Profession
9. (c) The family diagram can be drawn
James Team-2 Thin Cheating Rosy Yellow Student
as
Deepak Team-1 Fat Fairly (+) Married (–) Mary Purple Principal
A (Professor) T (CA)
Hari Team-2 Thin Fairly Lily Brown Teacher
Son
ther

Rahul Team-1 Thin Cheating Andy Blue Inspector


Grandfa

(+)
Vikram Team-1 Thin Fairly C (Doctor)
Married
S (Lawyer)(–) Daisy Green Librarian
Imran Team-2 Fat Fairly (+)
(Architect) R(–) B (Engineer) 16. (b) Teacher likes brown colour.
Hence, Hari played fairly from Team-2 and he
is thin. Hence, the correct pair of married couples are 17. (c) Mary is the principal.
AT and CS.
4. (b) According to the question, the 18. (d) Rosy is not a teacher, she is a student.
arrangement is as follows 10. (c) By given conditions, there are two groups Sol. (Q. Nos. 19-21) According to the
of students which are A, D, E, F and B, C, G, H. question,
Person Company Colour of tie Married
11. (c) According to the information, the
Ramesh B Blue (+) couple
arrangement is as follows Bhuvanesh Aruna (–)
Jim D Red (Teacher) (Housewife)
Couple
Manish A/C Green/Orange Q+ P–
Housewife
Nilesh E Yellow Son Daughter-in-law Son
Lawyer
Sagar A/C Green/Orange Son Married
Couple (–) couple
Hence, Ramesh works in company B. T+ S– Sandhye (+)
(Engineer)
Teacher Doctor (Lawyer)
5. (c) According to the question,
A (Bhopal) Children Mother Female – Student
R U Male – Doctor
Charu Suraj
(+, –) (+, –)
(Lucknow) C/P C/P (Delhi)
Engineer/Student Engineer/Student 19. (d) Aruna is the grandmother of Charu.
So, she is a student is correct and either R or 20. (b) Aruna is the housewife.
B (Shimla) U be a student.
21. (c) The grand-daughter of the family is the
student.

https://sscstudy.com/
https://sscstudy.com/

66 CUET (UG) Section III : General Test

CHAPTER 14

Venn Diagram
Venn diagram is the diagrammatic representation of a Here, a group of three items are shown by three circles
particular group of objects. In it, different figures are and their relation can be as follows.
drawn to represent relationship between two or more (a) Three items belong to three different groups
objects. The figures used are generally circular but
sometimes square, triangle and rectangles are also used.
In these Venn diagrams, the basic concept of set, subset, A B C
disjoint set etc. are used.
(1) An object is a subset of another object if former is a (b) One item belongs to the class of second and second
part of latter and this relation is shown by two belongs to the class of third.
concentric circles. e.g. Chair, furniture.

A B C

Chair

Furniture (c) Two separate items belong to the class of third.

(2) An object is said to have an intersection with


another object when two objects share something
in common. e.g. Surgeon, Males A B

C
Surgeon Males
(d) Two items with some common relationship belong
to the class of third.
(3) Two objects are said to be disjoint when neither
one is a subset of another nor they share anything
in common. e.g. Furniture, Car. A B

C
Furniture Car
(e) Two items are partly related to the third and are
themselves independent of each other.
The questions related to Venn diagram are based
on the following types A B C

1. Identification of Relation Based on (f) Three items are partly related to each other.
Venn Diagram
In this type, geometrical figures in the form of circles are A B
given that represent different classes.
These different classes share some common relationship C
or sometimes they are completely disjoint too.

https://sscstudy.com/
https://sscstudy.com/

GENERAL MENTAL ABILITY 67

(g) One item belongs to the class of second while third Ex. 6 Tea, Coffee, Beverages
item is different from the two. Sol. 2. (b) Iron is a metal but chlorine is a non-metal.
B
A C
Iron Chlorine

Metal
(h) One item belongs to the class of second and the
third item is partly related to these two. 3. (d) Some houses and some bridges are made of
bricks.
B
A C House

(i) One item belongs to the class of second and third is Brick
partly related to second.
Bridge
B
A
C
4. (c) Potato and cabbage are entirely different. But
both are vegetables.

(j) Two items are partly related to each other and the Vegetables
third item is entirely different from the two.
Potato Cabbage
A B C

Ex. 1 Which of the following diagrams represents


pens and pencils ?
5. (b) All thieves are criminals.
But judge is different.

Judge
(a) (b) (c) (d)
Thief
Sol. (c) Pens and Pencils both are used for writing but
both are entirely different. Criminal

Pens Pencils 6. (c) Tea and coffee are entirely different. But both
belong to the class of Beverages.

Directions (Ex. Nos. 2-6) In the following questions,


three classes are given. Which of the following figures
will represents the best relationship amongst them? Tea Coffee

Beverages

(a) (b) (c) (d)


2. Analysis Based Venn Diagram
In this type, generally a venn diagram is given. Each
Ex. 2 Metal, Iron, Chlorine geometrical figure in the diagram represents a certain
Ex. 3 Brick, House, Bridge class.
The candidate is required to analyse all the figures
Ex. 4 Vegetables, Potato, Cabbage
carefully and answer the questions.
Ex. 5 Judge, Thief, Criminal

https://sscstudy.com/
https://sscstudy.com/

68 CUET (UG) Section III : General Test

Directions (Ex. Nos. 7 and 8) In the following diagram, Ex. 9 The ratio of total number of train travelers to
three classes of population are represented by three the people who do not travel by train is
figures. The triangle represents school teachers, the 27 28
(a) (b)
square represents the married persons and the circle 16 16
represents the persons living in joint families. 37 38
(c) (d)
16 16
F Sol. (d) Number of people who are train travelers
E B C = 50 + 100 + 50 + 50 + 100 + 10 + 20
D = 380
A
Number of people who do not travel by train
Ex. 7 Married persons living in joint families but not = 100 + 20 + 40
working as school teachers are represented by = 160
(a) C (b) F (c) D (d) A
380
Sol. (c) Married persons living in joint families are ∴ Required ratio =
represented by the region common to the square and the 160
circle, i.e. D and B. But according to the given conditions, 38
the persons should not be school teachers. So, B is to be =
16
excluded. Hence, the required condition is denoted by
region D. Ex. 10 How many people who travel by train also
Ex. 8 School teachers who are neither married nor do travel by car or bus but not cycle?
live in joint families are represented by (a) 50 (b) 70
(a) F (b) C (c) B (d) A (c) 80 (d) 100
Sol. (a) School teachers are represented by the triangle. Sol. (c) Number of people who travel by train also
But according to the given conditions, persons are neither travel by car or bus but not cycle
married nor do they live in joint families. = 10 + 20 + 50
So, the region should not be a part of either the square or = 80
the circle. Such a region is F.
Ex. 11 Which two modes of transport used by people
Directions (Ex. Nos. 9-11) Study the following are mutually exclusive?
diagram and answer questions based on it. (a) Car—Bus
100 (b) Car—Cycle
Car
20 40 (c) Bus—Cycle
10 Bus (d) Cannot be determined
100 50
50 Train
100 Sol. (b) From the diagram it is clear that car → ‘ ’
50 Cycle and cycle ‘ ’ are mutually exclusive.
20

https://sscstudy.com/
https://sscstudy.com/

GENERAL MENTAL ABILITY 69

Practice Questions
1. Which of the following diagrams best depicts the 9. Topic, Book, Chapter
relation among Red, Pink and Colour?
10. Clothes, Saree, Dhoti
11. Forests, Earth, Mountains
12. Which of the following diagrams best depicts the
(a) (b) (c) (d) relation among Liquids, Milk and River water?

2. Identify the diagram that best represents the


relationship among the classes given below.
Book, Pen, Pencil (a) (b) (c) (d)

Directions (Q. Nos. 13 and 14) Study the following


diagram carefully and answer the following questions.
(a) (b) (c) (d)
2
Directions (Q. Nos. 3-6) In the following questions, three 7 3 6
classes are given. Out of the four figures that follows, you 5
4
have to find the figure that best represents the relationship 1
amongst the three given classes.
13. Find the number that lies inside all the figures.
(a) 3 (b) 6 (c) 4 (d) 7

14. Find the number that lies only inside the triangle.
(a) (b) (c) (d) (a) 1 (b) 4
(c) 7 (d) There is no such number
3. Elephants, Wolves, Animals
Directions (Q. Nos. 15-17) In the following figure,
4. Tea, Coffee, Drinks
triangle represents Doctors, Square represents Young
5. Boys, Students, Athletes persons and circle represents Rural peoples. Study the
6. Police, Thief, Criminal diagram carefully and answer the given questions.
7. Which of the the following figures represents the Doctors
A
relationship among Doctor, Man and Actor ?
D
B C Young

E F G

Rural
(a) (b) (c) (d)
15. Find the portion that represents young Doctors that
8. Identify the diagram that best depicts the relation live in rural areas.
among Classroom, Black board and School? (a) F (b) G (c) E (d) C

16. Find the portion that represents doctors living in


rural area but are not young.
(a) E (b) A+B (c) B (d) D+C
(a) (b) (c) (d)
17. Find the portion that represents people living in rural
Directions (Q. Nos. 9-11) In the following questions, area who are neither Doctors nor young.
three classes are given. Out of the four figures that follows, (a) B (b) E
you have to find the figure that best depicts the relation (c) C (d) F
amongst the three given classes. Directions (Q. Nos. 18-20) In the following figure,
rectangle represents males, triangle represents educated
people, square represents public servants and circle
represents people living in urban area. Study the diagram
(a) (b) (c) (d) carefully and answer the given questions.

https://sscstudy.com/
https://sscstudy.com/

70 CUET (UG) Section III : General Test

Urban 3 Public
10 7
servants
12 6 13
14 4 43
31 26
11 Males
5 Educated 12
28
12 44
18. Out of the following options, how many educated 36
males do not live in urban area?
(a) 10 (b) 4 (c) 11 (d) 9
24. How many persons like reading?
19. Out of the following options, how many people live in (a) 170 (b) 154 (c) 176 (d) 117
urban area who are neither public servants nor
educated or males? 25. How many persons like trekking and reading but not
(a) 3 (b) 5 (c) 6 (d) 10 cycling?
(a) 12 (b) 31 (c) 44 (d) 28
20. Out of the following options, how many educated
males are living in urban area? 26. How many persons like reading and cycling but not
(a) 4 (b) 5 (c) 6 (d) 10 trekking?
(a) 31 (b) 12 (c) 26 (d) 28
Directions (Q. Nos. 21-23) Study the following diagram
and answer questions based on it. Directions (Q. Nos. 27-29) Study the following
diagram and answer questions based on it.
The diagram represents the likes of kids of a class.
5
13 11
10 13 16 Jupiter 4
12 2
14 5 10 15 20
3
Mercury 6
12 14
Mars
5
21. The ratio of kids who like only Mars to those who like
all the three is Tennis players
7 7 7 14
(a) (b) (c) (d) Hockey players
5 8 6 13
Chess players
22. What is the difference between the kids who like
Mercury and Jupiter?
27. The number of Non-Chess players is how much more
(a) 6 (b) 8 (c) 10 (d) 12
than the players who play both Chess and Tennis?
23. How many people who like Mars like Jupiter also? (a) 5 (b) 10
(a) 45 (b) 29 (c) 26 (d) 13 (c) It is the same (d) Cannot be determined

Directions (Q. Nos. 24-26) Study the diagram given 28. The number of Tennis players who are not Hockey
below and answer the questions based on it. players is
(a) 10 (b) 50 (c) 55 (d) 35
∆ represents persons who like reading
represents persons who like cycling 29. The ratio of Hockey players to Chess players is
9 10 11 15
represents persons who like trekking (a) (b) (c) (d)
18 18 18 18

ANSWERS
1. (c) 2. (c) 3. (c) 4. (c) 5. (a) 6. (b) 7. (d) 8. (a) 9. (a) 10. (d)
11. (c) 12. (a) 13. (a) 14. (a) 15. (d) 16. (c) 17. (b) 18. (c) 19. (a) 20. (a)
21. (d) 22. (a) 23. (b) 24. (a) 25. (d) 26. (a) 27. (c) 28. (d) 29. (c)

https://sscstudy.com/
https://sscstudy.com/

General Mental Ability 71

Hints & Solutions


1. (c) Red and Pink both belong to the group of 14. (a) The number that lies only inside the
colour. But they both are entirely different. Man triangle is 1.
Doctor Actor 15. (d) The region ‘C’ is common to all the
Red Pink figures. Hence, ‘C’ represents young Doctors
that live in rural areas.
Colour 8. (a) Blackboard is a part of classroom and
classroom is a part of school. 16. (c) The region ‘B’ is common to triangle
2. (c) Book, Pen and Pencil, all the three and circle only. Hence, ‘B’ represents Doctors
belong to stationery items but they are entirely living in rural area but are not young.
different. Black
board 17. (b) The region ‘E’ is present in circle only
Book and hence is the required answer.
Classroom
18. (c) Only 11 educated males do not live in
Pen
School urban area.
Pencil
19. (a) Only 3 persons live in urban area who
9. (a) A book has chapters and chapter has
are neither public servants nor educated or
3. (c) Elephants and Wolves are entirely topics.
males.
different. But, they both belong to the class of
animals. 20. (a) Only 4 educated males are living in
Topic urban area.

Chapter 21. (d) Number of kids who like only Mars


Elephant
= 14 Number of kids who like all the three = 13
Wolves Book 14
∴ Required ratio =
13
Animals
10. (d) Dhoti and Saree are entirely different
but both belongs to the group of clothes. 22. (a) Number of kids who like Mercury
4. (c) Tea and Coffee are entirely different. But = 10 + 12 + 13 = 35
both belong to the class of drinks. Clothes Number of kids who like Jupiter = 16 + 13 = 29
∴Required difference = 35 − 29 = 6
Dhoti Saree
23. (b) Number of kids who like Mars like
Tea Coffee
Jupiter also = 13 + 16 = 29
Drinks 24. (a) Number of persons who like Reading
11. (c) Mountains and Forests are part of = 43 + 31 + 12 + 28 + 12 + 44 = 170
5. (a) Some boys can be students and some Earth. But, Some mountains are forested and
boys can be Athletes. Also, some athletes can some forests are mountainous. 25. (d) Number of persons who like Trekking
be students. and Reading but not Cycling = 28
Earth
26. (a) Number of persons who like Reading
Students Athletes and Cycling but not Trekking = 31
Mountains Forests 27. (c) Number of players who play both
Chess and Tennis = 11 + 10 + 14 = 35
Boys Number of Non-Chess players = 35
Clearly, the number of Non-Chess players are
same the number of players who play both
6. (b) All thieves are criminals, but all criminals Chess and Tennis.
12. (a) Milk and River water both belong to the
are not thieves. Also, police is entirely different.
group of liquids. But they both are entirely
different.
28. (d) The number of Tennis players who are
Criminals not Hockey players = 5 + 11 + 14 + 5 = 35

Thieves 29. (c) Number of Hockey players


= 5 + 2 + 3 + 10 + 15 + 20 = 55
River
water Number of Chess players
Milk
= 2 + 13 + 11 + 4 + 15 + 6
Police Liquid + 14 + 12 + 3 + 10
= 90
7. (d) Doctors and Actors are entirely different. 13. (a) The number common to triangle, circle 55 11
∴Required ratio = =
and rectangle is 3. 90 18
But some doctors and actors are men.

https://sscstudy.com/
https://sscstudy.com/

72 CUET (UG) Section III : General Test

CHAPTER 15

Cube and Dice


Cube Number of smaller cubes with 1 side painted = Number
of central cubes
A cube is a three dimensional solid object bounded by six
square faces Number of cubes with no side painted = Number of
Face nucleus or Inner central cubes.
Ex. 1 How many smaller cubes of 1 cm side can be
Edge
formed with a solid cube of 3 cm side ?
(a) 3 (b) 6 (c) 9 (d) 27
Sol. (d) Required number of smaller cubes
Corner
Volume of bigger cube (3)3
A cube has 6 faces, 8 corners and 12 edges. = = = (3)3 = 27
Volume of smaller cube ( 1 )3
If a larger cube is cut into smaller cubes of equal volume
so that each edge is divided into n parts, then Ex. 2 A cube is coloured red on all its faces. It is then
cut into 64 smaller cubes of equal size. The smaller
A B A cubes so obtained are separated. How many cubes have
B C B A
A B A B no faces coloured?
A B (a) 24 (b) 16
A B A (c) 8 (d) 10
C
B C B
B A Sol. (c) Number of smaller cubes = n 3 =64 ⇒ n = 3 64 = 4
B
A Now, cubes with no painted surface are inner central
A B A
cubes or nucleus cubes.
∴ Number of nucleus cubes = (n − 2)3 = (4 − 2)3 = 8
● Number of smaller cubes = ( n )3
Ex. 3 How many cubes are there in this diagram?
Edge of larger cube
where, n=
Edge of smaller cube
● Number of corner cubes (exist at each corner) = A = 8
● Number of middle cubes (exist at middle of each edge)
= B = 12 ( n − 2)
● Number of central cubes (exist at centre of each face) (a) 16 (b) 12
= C = 6 ( n − 2)2 (c) 10 (d) 8
● Number of nucleus cube/Inner Central Cube (hidden Sol. (b) 1 2 3 4
and exist at centre of large cube) 4
1 2 3 4
3
= D = ( n − 2) 5
12 12 5
Note When a large cube is painted with some colour and then
divided into smaller cubes of equal size, then after separation, 11 6
11 6
number of smaller cubes so obtained are as follows, 9 8
7
Number of smaller cubes with 3 sides painted = Number 10 9 8 7
of corner cubes
Number of smaller cubes with 2 sides painted = Number So, there are 12 cubes in the given figure, which are
of middle cubes illustrated above.

https://sscstudy.com/
https://sscstudy.com/

GENERAL MENTAL ABILITY 73

Dice upper

A dice is a three dimensional figure with 6 square faces. left front right back
lower

upper

back left front right

There are two types of dice. lower

(a) When the common digit is on different faces


Here, Upper face is opposite to Lower face. Front face is
When the common digit is on different faces, then opposite to Back face, and Right face is opposite to Left
list these digits in clockwise direction starting from face.
common digit and then compare them.
e.g. Ex. 4 What number will come at the opposite face
2 4
of 1 in the following dice ?
3 1
1 5
1 1
5 3 2 6
Here, 1 is common in both position. So move in,
clockwise direction starting from 1. (i) (ii)
We get, 1 → 2 → 3
(a) 4 (b) 5 (c) 3 (d) 2
1→ 5→ 4
Sol. (a) The adjacent faces of ‘1’ are 2, 3, 5 and 6. So,
So, 3 is opposite to 4
the remaining face containing number ‘4’ will be
2 is opposite to 5 opposite to ‘1’.
and 1 is opposite to 6
Ex. 5 When the following figure is folded to form a
(b) When two digits are common
cube, then which number will be opposite to 1?
e.g. 2 2
1
4 3 2 6 3
3 6
4
5
Here, uncommon digits in each dice are opposite to
(a) 6 (b) 4 (c) 5 (d) 3
each other.
So, 4 is opposite to 6 Sol. (b) 1 upper
2 6 3 left front right
and 3/2 is opposite to 1/5
4 lower
5
Spread out Form of a Dice back

The spread out form of a dice is shown on the basis of We know that upper face is opposite to lower face. So, 1
the front side of the dice. is opposite to 4.
Upper Ex. 6 On the basis of the given two positions of
single dice, find the letter at the face opposite to the
face having letter A.
Left Right
Front A D
Lower C E
B C
Back
(I) (II)
One can unfold a dice in many ways, some of which are (a) B (b) C (c) E (d) D
given below. Sol. (c) Common letter = C (on different face)
upper
Common

Opposite
Opposite

back From Position I, C B A


left front right
upper
lower From Position II, C D E
left front right
back
lower Clearly, E is opposite to A.

https://sscstudy.com/
https://sscstudy.com/

74 CUET (UG) Section III : General Test

Practice Questions
1. How many cubes are there in the given figure? 10. Find the digit at the face opposite to the face having
digit 2 in the dice given below.
1
2 6

(a) 3, 4 (b) 3, 5 (c) 4, 5 (d) 3, 4, 5


(a) 3 (b) 5 (c) 7 (d) 13
11. Two positions of a dice are given. Based on them, find
2. Count the number of blocks in the given figure. out which number is opposite to number 4 in that
dice?
6 6
3 4
2 3

(a) 1 (b) 2 (c) 3 (d) 4


(a) 7 (b) 11 12. From the given two positions of a single dice, find the
(c) 13 (d) 15 digit at the face opposite to the face having digit 2.
3. A cube of 3 cm side is divided into smaller cubes of 2 6
side 1 cm. What will be the total number of middle 3 1
6 4
cubes?
(a) 6 (b) 12 (c) 18 (d) 24 (a) 4 (b) 5 (c) 3 (d) 1
4. If a cube of 12 cm side is divided into smaller cubes of 13. Which number is on the face opposite to face having
3 cm side, then find the total number of inner central number 1?
cubes?
5 2 5
(a) 18 (b) 9 (c) 8 (d) 81
2 3 1 4 6 1
5. A cube having 24 cm side is divided into 64 smaller
cubes of equal volume. Find the edge of smaller (a) 2 (b) 3 (c) 4 (d) 5
cubes?
14. Which number is on the face opposite to face having
(a) 5 cm (b) 6 cm
number 3?
(c) 3 cm (d) 4 cm
3 5 3 2
Directions (Q.Nos. 6-8) Read the given information 6 1 2 4 5 1 3 6
carefully and answer the questions that follow.
All the surfaces of a cube of side 15 cm are painted with (a) 2 (b) 3 (c) 4 (d) 6
red colour and then it is cut into smaller cubes of side
3 cm. The smaller cubes so obtained are separated. 15. From the given two postions of a single dice, find the
6. How many smaller cubes are there having two letter at the face opposite to the face having letter V.
surfaces painted with red colour? V S
(a) 8 (b) 24 (c) 36 (d) 54
J J
I U
7. How many smaller cubes are there having only three
surfaces painted with red colour?
(i) (ii)
(a) 8 (b) 10 (c) 32 (d) 64
(a) S (b) U (c) J (d) I
8. How many smaller cubes are there having 4 or more
faces painted with red colour? 16. From the given four positions of a single dice, find the
(a) 8 (b) 0 (c) 4 (d) 2 colour at the face opposite to the face having green.

9. Find the digit at the face opposite to the face having g e r


Gree
n
White Oran Silve
n

digit 4 in the given dice.


d

d
Gree
d

Re

Re
Re

Orange Orange Violet Green


5
4 1 (i) (ii) (iii) (iv)

(a) 2 (b) 6/2 (a) Orange (b) Red


(c) 5 (d) 3 (c) Silver (d) Violet

https://sscstudy.com/
https://sscstudy.com/

GENERAL MENTAL ABILITY 75

17. Four positions of dice are given below. Which letter 19. Choose the cube which will be formed on folding the
will be opposite to D? question figure.
B E C D Question figure
A C E B
(i) C (ii) A (iii) D (iv) C O
K
(a) E (b) B (c) A (d) C

T
L
18. When the following figure is folded to form a dice, M U
then which number will be opposite to 5?
1 Answer figures
3 2
K L K O
4 M U L

T
6 5 L K T K

(a) 1 (b) 2 (c) 3 (d) 4 (a) (b) (c) (d)

ANSWERS
1. (c) 2. (a) 3. (b) 4. (c) 5. (b) 6. (c) 7. (a) 8. (b) 9. (d) 10. (d)
11. (b) 12. (d) 13. (b) 14. (c) 15. (a) 16. (d) 17. (c) 18. (c) 19. (a)

Hints & Solutions


1. (c) Number of cubes in upper row = 2 5 + 4 = 9; 5 + 1 = 6 Clearly, violet is opposite to green.
1 + 4 = 5 [not equal to 7] 17. (c) Dice can be shown as,
Number of cubes at the bottom = 5 Hence, it is a standard dice.
∴ Total cubes = 2 + 5 = 7 Now, in a standard dice, sum of opposite B
2. (a) Number of blocks at the top = 3 faces is 7. Hence, 4 is opposite to 3.
D C A
Number of blocks at bottom = 4 10. (d) Here, the sum of adjacent faces E
∴ Total blocks = 3 + 4 = 7 2 + 1 = 3, 2 + 6 = 8, 1 + 6 = 7 ?
3. (b) Number of middle cubes = 12 ( n − 2 ) Hence, it is a general dice.
Digits at the opposite face of 2 = 3, 4 or 5. So, from the unfolded dice, it is clear that the
= 12  − 2  = 12 × 1 = 12
3
11. (b) In the given two figures, two numbers letter opposite to D is A.
1 
are common, i.e. 3 and 6. So, remaining two 18. (c) Taking 2 as front face, we get 1 as
4. (c) Total number of inner central cubes numbers are opposite to each other . upper face and 4 as lower face. Similarly, 3 is
3
= (n − 2 )3 =  − 2  = (2 )3 = 8
12 Hence, 2 is opposite to 4. left face, 5 is right face and 6 is the back face.
 3  Hence, 3 is opposite to 5.
12. (d) In both the positions, 6 is common.
5. (b) We know that, 19. (a) The unfolded dice can be represented
2 6 as
Number of smaller cubes = n3 = 64
3 1
⇒ n =3 64 = 4 6 4 O
Edge of bigger cube
Also, n= Moving in clockwise direction from 6, we get K Opposite
Edge of smaller cube
6 → 2 → 3 Opposite
24 L
T

⇒ Edge of smaller cube = = 6 cm 6 → 1 → 4


4
Hence, 1 is opposite to 2. M U
Sol. (Q. Nos. 6-8)
13. (b) The numbers 2, 4, 5 and 6 are on the Opposite
6. (c) We know that
surfaces adjacent to 1. Hence, 3 is opposite to 1. Here, O and L are opposite, K and M are
Side of bigger cube 15
n= = =5 opposite and T and U are opposite.
Side of smaller cube 3 14. (c) Figure (i), (iii) and (iv) show that
Smaller cubes having 2 surfaces painted numbers 6, 1, 5 and 2 appears on the In answer figure (b), K and M are shown
= Number of middle cubes surfaces adjacent to the number 3. adjacent to each other. In answer figure (c),
= 12 ( n − 2 ) = 12 ( 5 − 2 ) = 36 Hence, 4 will be opposite to number 3. T and U are shown adjacent to each other and
in answer figure (d), O and L are shown
7. (a) Smaller cubes having 3 surface painted 15. (a) Here J is common in both the positions.
adjacent to each other. According to the
Hence, letter ‘S’ is at the face opposite to the question figure these are opposite to each
= Number of corrner cubes = 8
face having letter V.
other, so these alternatives are not possible.
8. (b) There is no such cube with 4 or more
faces painted. 16. (d) From the given four positions of a Only the cube given in answer figure (a) can
single dice, faces adjacent to face having be formed because K, L and T can be on
9. (d) Here, the sum of adjacent faces green colour = Orange, red, white, silver adjacent faces.

https://sscstudy.com/
https://sscstudy.com/

76 CUET (UG) Section III : General Test

CHAPTER 16

Non-Verbal Reasoning
Non-Verbal reasoning tests candidates thinking ability, Sol. (c) First design of first pair of problem figures is
ability to identify similarity and dissimilarity and come inverted and then placed on the original figure to get the
to a conclusion quickly on the basis of given figures, second figure. Similar concept is used in second pair.
patterns and symbols. The following topics are covered Ex. 2 Find the figure from the set of answer figures
in this chapter. that will replace the question mark ‘?’.
1. Analogy 2. Classification
Problem figures
3. Series 4. Embedded figure
5. Counting of Figures 6. Figure completion
?
7. Paper cutting and folding
8. Mirror & Water Image.
Answer figure

1. Analogy
Analogy refers to correspondence or similarity in
relationship. When a figure exhibits some kind of
relationship with another figure on some basis then the (a) (b) (c) (d)
two figures are said to be analogous to each other. In Sol. (c) The figure rotates through 180° anti-clockwise.
questions based on analogy, two sets of figures namely On following the same pattern, the answer figure (c) will
problem figures and answer figures are given. replace the question mark ‘?’.
The set of problem figures consist of two parts which
bear certain relationship between them. The second part 2. Classification
comprises of one figure and a sign of ‘?’.
Classification is the process of putting things or objects
The candidate is asked to select one figure from the into a group and then finding the different object or
set of answer figures to replace ‘?’ but maintaining thing that does not belong to that group based on some
similar relationship as depicted between the first two common properties.
figures.
In questions based on classification, the candidate has to
Ex. 1 Find the figure from the set of answers find an odd figure from a set of four figures based on
figures that will replace the question mark‘?’. some common properties.
Problem figures Ex. 3 Find the odd figure from the set of given
figures.
?

Answer figures
(a) (b) (c) (d)
Sol. (b) Except figure (b), in all other figures the main
figure is divided into four parts while in second figure
(a) (b) (c) (d) two parts are there.

https://sscstudy.com/
https://sscstudy.com/

GENERAL MENTAL ABILITY 77

Ex. 4 Find the odd figure from the set of given Ex. 7 Find the answer figure in which the question
figures. figure is embedded.
+ + + × + Question figure Answer figures
+ + + × + + + +
× + × +
(a) (b) (c) (d)
Sol. (b) Figure (b) has 4 plus signs and other have 3. (a) (b) (c) (d)

Sol. (c) The question figure is embedded in answer Fig.


3. Series (c)
Series is a continuous sequence of figures following a
certain defined pattern. In this type of questions, a
series of figures is given as problem figures and the
candidate is asked to select one figures from the set of
answer figures which will continue the given sequence. Ex. 8 Find the answer figure which is embedded in
the question figure.
Ex. 5 Find the figure that will continue the set of
problem figures. Question figure Answer figure
Problem figures

(a) (b) (c) (d)


Answer figures Sol. (b) The answer figure (b) is embedded in the
question figure.

(a) (b) (c) (d)


Sol. (b) In each step, a pin is added on the left hand
side of existing pin/pins. The head of the pin is in a
direction opposite to adjacent pin. 5. Counting of Figures
Ex. 6 Find the figure that will continue the set of Counting of figures is simply the realisation of different
problem figures. geometrical figures from a complex one. In questions
based on counting of figures the candidate is asked to
Problem figures
count straight lines, triangles, squares, circles etc. from
a given figure.
?
Ex. 9 How many squares are there in the figure
given below?
Answer figures

(a) (b) (c) (d)


Sol. (d) In each successive problem figure, black dot
(a) 4 (b) 5
moves three steps clockwise and white circle moves two
(c) 6 (d) 7
steps clockwise.
Sol. (b) There are 5 squares in the given figure
4. Embedded Figures A
E
B
A figure is said to be an embedded figure when that
figure is completely contained by another figure. H F
I
Questions based on embedded figures comprise of a
question figure and four answer figures and it is asked
to find to correct answer figure embedded in given figure D G C
or the correct answer figure in which the given question viz. ABCD, AEIH, EBFI, FICG, and HIGD.
figure is embedded.

https://sscstudy.com/
https://sscstudy.com/

78 CUET (UG) Section III : General Test

Ex. 10 How many triangles are there in the figure Ex. 12 Choose the answer figure that will complete
given below? the question figure.
Question figure Answer figure

(a) (b) (c) (d)


(a) 20 (b) 27 ?
(c) 18 (d) 29
Sol. (a) Naming the figure, Sol. (b) Clearly, option (b) completes the original figure
A B C which looks like the figure given below
I J

H D
M
K
L
G F E
Smallest triangles are ∆ABI, ∆BCJ, ∆CDJ, ∆DEK, ∆EFK,
Ex. 13 Choose the answer figure that will complete
∆FGL, ∆GHL, ∆HAI = 8 Small triangles formed with two
triangles the question figure.
∆BCD, ∆DEF, ∆FGH, ∆HAB = 4 Question figure Answer figure
Large triangles formed with two triangles and one
square are
∆ACM, ∆CEM, ∆EGM, ∆GAM = 4
Largest triangles ∆ACE, ∆AGE, ∆GAC, ∆GEC = 4 (a) (b) (c) (d)
?
∴ Total triangles = 8 + 4 + 4 + 4 = 20
Ex. 11 How many circles are there in the figure Sol. (a) Clearly, option (a) completes the question
given below? figure which looks like the figure given below.

(a) 4 (b) 8
(c) 9 (d) 10
Sol. (b) 7. Paper Cutting and Folding
1 2 3 4 Paper folding and cutting problems are based on a
5 6
transparent sheet which is folded along a dotted line or
folded and cut (or punched) in a particular manner.
7
In questions based on paper folding, a transparent sheet
8 having a certain design and a dotted line is given on it.
The candidate is required to identify the design that
Clearly, total number of circles = 8 would appear on the sheet when it is folded along the
dotted line.
6. Figure Completion In questions based on paper cutting, few figures are
Figure completion is a process to find out the missing given showing the way in which a piece of paper is to be
part of an incomplete figure to complete it. folded and then cut to form a particular section.

In this type of questions, a figure is given and a part of Ex. 14 In the given question, a square transparent
1 sheet with a pattern is given. Find out from amongst
it is missing, generally th part of the figure and the
4 the four alternatives as to how the pattern would
candidate is asked to find that missing part from the appear when the transparent sheet is folded along the
given set of answer figures. dotted line.

https://sscstudy.com/
https://sscstudy.com/

GENERAL MENTAL ABILITY 79

Transparent sheet Answer figures Object

Water
(a) (b) (c) (d)

Sol. (c) The transparent sheet will appear like option


figure (c)
Water Image
Directions (Ex. Nos. 15 and 16) In the following
Note If the mirror is placed along the horizontal line than the mirror
questions a set of three figure (X), (Y) and (Z) have been image of the object would be same as water image.
given showing a sequence in which paper is folded and
finally cut from a particular section. These figures are Ex. 17 Find the mirror image of the question figure
followed by a set of answer figures marked (a), (b), (c) when the mirror is placed along the line MN.
and (d) showing the design which the paper actually Question figure Answer figures
acquires when it is unfolded. You need to select the M
answer figure which is closest to the unfolded piece of
paper.
Ex. 15
N (a) (b) (c) (d)
Question figures Answer figures
Sol. (c) Option figure (c) is the correct mirror image of
question figure.

X Y Z (a) (b) (c) (d)

Sol. (d)
Ex. 18 If a mirror is placed on the line XY, then
which of the answer figure is the right image of the
given figure?
Ex. 16
Question figure Answer figures
Question figures Answer figures X

X Y Z (a) (b) (c) (d)


Y (a) (b) (c) (d)
Sol. (c) Sol. (a) Option figure (a) is the correct mirror image of
question figure

8. Mirror and Water Image


Mirror image is the reflection of an object into the
mirror, and water image of an object is its reflection into
Ex. 19 Find the Water image of the question figure.
the water. Question figure Answer figures
––––
In mirror image, the left hand side and right hand side
––––
×××× –––– ×××× ––––
×××× –––– ×××× ––––
×××× –––– ×××× ––––
×××× ××××
of an object interchange their places, while top and ––––
––––
××××
××××
– – – – ××××
– – – – ××××
– – – – ××××
––––
––––
××××
××××
××××
–––– ––––
××××
– – – – ×××× ××××
bottom remains constant. ––––
××××
––––

Object Mirror Image (a) (b) (c) (d)

Sol. (d) option figure (d) is the correct water image of


question figure.
––––
––––
––––
Mirror ––––
××××
××××
××××
××××
In water image, the top and bottom of an object
interchange their places, while the left hand side and
right hand side remains constant.

https://sscstudy.com/
https://sscstudy.com/

80 CUET (UG) Section III : General Test

Practice Questions
Directions (Q. Nos. 1-5) Find a Answer figures 12. Question figures
figure from the set of answer figures
that will replace the question mark
?
(?).
1. Question figures (a) (b) (c) (d)
Answer figures

? Directions (Q. Nos. 6-10) In the


following questions select the figure
which is different from the other
Answer figures (a) (b) (c) (d)
three responses.
13. Question figures
6.
?
(a) (b) (c) (d)
(a) (b) (c) (d)
2. Question figures
Answer figures
7.
?

(a) (b) (c) (d) (a) (b) (c) (d)


Answer figures
8.
14. Question figures

?
(a) (b) (c) (d) (a) (b) (c) (d)

3. Question figures 9.
Answer figures

?
(a) (b) (c) (d)
10. (a) (b) (c) (d)
Answer figures
15. Question figures
(a) (b) (c) (d)
?
(a) (b) (c) (d)
Directions (Q. Nos. 11-15) Find
the figure form the set of answer Answer figures
4. Question figures
figure that will follow the pattern of
question figure and will replace
?
question mark(?).
(a) (b) (c) (d)
11. Question figures
Answer figures Directions (Q. Nos. 16-18) In the
following questions from the given
?
answer figures, select the one in
which the question figure is
(a) (b) (c) (d) Answer figures hidden/embedded.
5. Question figures 16. Question figure

?
(a) (b) (c) (d)

https://sscstudy.com/
https://sscstudy.com/

GENERAL MENTAL ABILITY 81

Answer figures 21. What is the minimum number of Answer figures


straight lines required to make
the figure?

(a) (b) (c) (d) (a) (b) (c) (d)

17. Question figure 27. Question figure


(a) 20 (b) 15
(c) 11 (d) 7

Directions (Q. Nos. 22 and 23)


Find the total number of triangles in
Answer figures the given figures.
22. Answer figures

(a) (b) (c) (d)


(a) 9 (b) 10 (a) (b) (c) (d)
18. Question figure
(c) 11 (d) 12
28. Question figure
23.

Answer figures

(a) 5 (b) 12
(c) 9 (d) 10 Answer figures

(a) (b) (c) (d) 24. What is the number of circles in


the following figure?
Directions (Q. Nos. 19 and 20)
Which of the answer figure is hidden (a) (b) (c) (d)
oblique line embedded in the
question figure? 29. Question figure

19. Question figure (a) 8 (b) 10


(c) 12 (d) 15

25. Find the total number of squares ?


in the given figure.

Answer figures Answer figures

(a) 10 (b) 14
(a) (b) (c) (d) (c) 12 (d) 16 (a) (b) (c) (d)

20. Question figure Directions (Q. Nos. 26-30) In the 30. Question figure
following questions, find the answer
figure that will complete the pattern
in the question figure.
?
26. Question figure
Answer figures
Answer figures

?
(a) (b) (c) (d)
(a) (b) (c) (d)

https://sscstudy.com/
https://sscstudy.com/

82 CUET (UG) Section III : General Test

Directions (Q. Nos. 31 and 32) In 33. Answer figures


each of the following questions, a
figure marked as transparent sheet
is given and followed by four answer X Y Z
figures, one out of these four options (a) (b) (c) (d)
resembles the figure which is
obtained by folding transparent sheet 38. Question figure
along the dotted line. Find the answer (a) (b) (c) (d) M N
from these figures.
34.
31. Transparent sheet

X Y Z
Answer figures

Answer figures
(a) (b) (c) (d)
35. (a) (b) (c) (d)

(a) (b) (c) (d) Directions (Q. Nos. 39 and 40)


X Y Z Find the water image of the given
32. Transparent Sheet question figure.
39. Question figure
(a) (b) (c) (d)
Answer figures
Direction (Q. Nos. 36-38) Find the
mirror image of the question figure if
the mirror is placed along the line MN. Answer figures

(a) (b) (c) (d) 36. Question figure


M
Directions (Q. Nos. 33-35) In each
of the following questions, a set of (a) (b) (c) (d)
three figure (X), (Y) and (Z) has been N
given, showing a sequence in which a Answer figures 40. Question figure
paper is folded and finally cut at a
particular section. Below these
figures a set of answer figures
marked (a), (b), (c) and (d) showing (a) (b) (c) (d)
the design which the paper actually Answer figures
37. Question figure
acquires when it is unfolded are also M
given. You have to select the answer
APPROACH
figure which is closest to the
unfolded piece of paper. N (a) (b) (c) (d)

ANSWERS
1. (c) 2. (d) 3. (c) 4. (d) 5. (b) 6. (b) 7. (b) 8. (c) 9. (c) 10. (c)
11. (c) 12. (b) 13. (b) 14. (b) 15. (c) 16. (b) 17. (b) 18. (b) 19. (c) 20. (c)
21. (c) 22. (d) 23. (d) 24. (c) 25. (b) 26. (d) 27. (c) 28. (d) 29. (b) 30. (b)
31. (c) 32. (a) 33. (c) 34. (b) 35. (d) 36. (a) 37. (c) 38. (d) 39. (c) 40. (d)

https://sscstudy.com/
https://sscstudy.com/

GENERAL MENTAL ABILITY 83

Hints & Solutions


1. (c) From first figure to second the main 12. (b) In each subsequent figure one line 21. (c) There are 5 horizontal lines and 6
figure is rotated 90º clockwise. So, answer segment is added. So, option (b) figure will vertical lines.
figure (c) will replace the ‘?’. follow the same pattern. ∴ Number of lines = 5 + 6 = 11

2. (d) From figure first to second, the figure 13. (b) The inner line segments move in A
22. (d)
inside the main figure rotates 90º clockwise anti-clockwise direction. So, option (b) figure
and moves to the sides. So, answer figure (c) will replace symbol ‘?’.
will replace the ‘?’ 14. (b) In each subsequent figure. The design E
rotates clockwise (or anti-clockwise) through F
3. (c) From first figure to second the main
figure is divided into six parts. So, option 90° along with the addition of a line segment.
figure (c) will replace the ‘?’. Hence, option (b) is the correct choice as it B D C
follows the corresponding pattern. Number of triangles=12 viz.
4. (d) From first figure to second the main ∆AFB, ∆AFE, ∆FEC, ∆FDC, ∆BFD,
figure is divided into two equal parts. So,
15. (c) In each step the elements are moving
as follows ∆BFC, ∆AFC, ∆ABD, ∆ADC,
option figure (d) will replace the ‘?’
∆ABE, ∆CBE and ∆ABC
5. (b) From first figure to second, the
outermost figure is removed and the 23. (d) A
second figure from inside becomes the
outermost figure. So, option figure (b) will
replace the ‘?’ Following the above pattern option figure (c)
will replace the question mark(?).
6. (b) Except in figure (b) in all others the
design has been divided vertically. 16. (b) Clearly, the question figure is
embedded in answer figure (b) as shown below B C D E F
7. (b) In the given question option (b) is
different of all other three options because in Number of triangles=10, viz.
every option lines are meeting in shape except ∆ABC, ∆ACD, ∆ADE, ∆AEF,
figure (b). ∆ABD, ∆ACE, ∆ADF, ∆ABE,
∆ACF and ∆ABF
8. (c) Except in figure (c), in all other figures,
two exactly opposite triangles are shaded. In 17. (b) Clearly, question figure is embedded
option (c) shaded figure is not opposite to in answer figure (b) as shown below 24. (c)
1 4 7 10
another one.

9. (c) Except in figure (c), in all other figures 2 5 8 11


the left and right designs consist of similar
3 6 9 12
figures.
In figure (c), the left and right designs consist 18. (b) Clearly, question figure is embedded Thus, there are all 12 circles in the figure.
of different figure, so option (c) is correct. in answer figure (b) as shown below
25. (b) B C
10. (c) Except in figure (c), in all other figures, A D
the inner design consists of less number of F G
E H
sides than that of the outer design.
In option (c) the inner design have more sides J K L
I
that outer design. 19. (c) Clearly, the answer figure (c) is
embedded after inverting in question figure as M N O
11. (c) In each step the elements are moving P
shown below.
as follows. Number of squares=14
viz. ABFE, BCGF, CDHG, EFJI, FGKJ,
GHLK, IJNM, JKON, KLPO, ACKI,
BDLJ, EGOM, FHPN and ADPM.

26. (d) Clearly, option (d) figure will complete


20. (c) Clearly, the answer figure (c) is
the pattern of the given figure.
embedded in question figure as shown below

Following above pattern option figure (c) will


replace the question mark(?).

https://sscstudy.com/
https://sscstudy.com/

84 CUET (UG) Section III : General Test

27. (c) Clearly, option (c) figure will complete 31. (c) The transparent sheet will appear like 37. (c) The mirror image would be like
the pattern of the given figure option figure (c)

32. (a) The transparent sheet will appear like


option figure (a).

33. (c)
28. (d) Clearly, option (d) figure will complete 38. (d) The mirror image would be like
the pattern of the given figure.

34. (b)
39. (c) The water image of the question figure
29. (b) Clearly, option (b) figure will complete would be
the pattern of the given figure.

35. (d)

40. (d) The water image of the question figure


30. (b) Clearly, option (b) figure will complete would be.
the pattern of the given figure. 36. (a) The mirror image would be like

https://sscstudy.com/
https://sscstudy.com/

CHAPTER 01

Number System
Numbers are everywhere in our daily life for counting, measuring, labeling, ordering (sequencing) and coding. In the
most popular, the Hindu-Arabic system, we use the symbol 0, 1, 2, 3, 4, 5, 6, 7, 8 and 9. These symbols are called digits.
Out of these ten digits, 0 is called an insignificant digit whereas the other are called significant digits.

Number Descending Order


A mathematical symbol represented by a set of digits A number is said to be in descending order, if they are
called a number. Every digit has a face value which arranged with greatest number to smallest number.
equals the value of the digit itself, irrespective of its e.g., 13, 9,7, 5, 2 are in descending order.
place in the numeral. Each digit in a number or numeral
has a place value besides its face value. We can express Successor
a number in two ways i.e., in words and in figures. A number (say b) is said to be a successor of another
e.g., If a number figure is 3290, then we can write this number (say a) if a + 1 = b
number in words as ‘three thousand two hundred and
ninety’. Predecessor
A number (say a) is said to be a predecessor of another
Face Value and Place Value of the number (say b ) if b − 1 = a
Digits in a Number
Type of Numbers
Face Value (Real Value) Natural Numbers Counting number is known as natural
In a number, the face value of a digit is the value of the number. e.g., 1, 2, 3, 4, ..., ∞
digit itself. The place of the digit is not considered in the Whole Numbers Natural number including zero (0) is
number. e.g., In the number 43857, the face value of 4 is known as whole numbers.
4, the face value of 3 is 3 and so on. e.g., 0, 1, 2, 3, ..., ∞
Place Value (Local Value) Integers The set of positive and negative whole
numbers including 0 is known as integers.
In a number, the place value of a digit changes
according to the change of its place. e.g., ..., − 3, − 2 , − 1, 0, 1, 2, 3, ...
p
Place value of units digit = (Unit digit) × 1 Rational Numbers The numbers of the form , where
q
Place value of tens digit = (Tens digit) × 10
q ≠ 0 and p and q is integer is known as rational number.
Place value of hundreds digit = (Hundreds digit) × 100 5 2 5 0 1 3
e.g., , − , − , , , etc.
Place value of thousands digit = (Thousands digit) × 1000 1 1 7 1 5 5
and so on. Irrational Numbers The number, which is non-repeating
e.g., In the number 48379, the place value of 4 is and non-terminating, is called irrational number.
4 × 10000 i.e., 40000, the place value of 8 is 8 × 1000 e.g., 3 , 5 , 2 etc.
i.e., 8000 and so on.
Real Numbers All rational and irrational numbers are
Ascending Order known as real numbers.
1 1 12
A number is said to be in ascending order, if they are e.g., 5, 7, 3, , , , 3, 5 etc.
arranged with smallest number to greatest number. e.g., 2, 2 5 17
5, 7, 9, 13 are in ascending order. π is an irrational number.

https://sscstudy.com/
https://sscstudy.com/

4 CUET (UG) Section III : General Test

Even Numbers The number, which is multiple of or as multiple of the divisor. The relation so obtained can
divisible by 2, is called even number. be given by
e.g., 2, 4, 6, 8, ... Dividend = Divisor × Quotient
Odd Numbers The number, which is neither multiple of Here, remainder equals zero.
2 nor divisible by 2, is called odd number.
e.g., 1, 3, 5, 7, ... Divisibility Tests
Prime Numbers The number, which has only two Divisibility by 2 An number is divisible by 2, when its
factors i.e., 1 and itself, is called prime number. unit’s digit is either even or zero.
e.g., 2, 3, 5, 7, 11, 13, ... e.g., The numbers 4, 6, 8, 10, 24, 28, 322, 4886 etc., are
divisible by 2.
2 is only even number which is prime.
3, 5, 7, 11, 13, 221, 995, 4637 etc., are not divisible by 2.
Composite Numbers The number, which has more
than two factors, is known as composite number. The rule emanates form the fact that any number
multiplied by 2 gives product whose unit’s digit is either
e.g., 4, 8, 9, 10, ...
even or zero.
4 is composite number because it is divisible by 1, 2 and 4.
Divisibility by 3 A number is divisible by 3 when the
Consecutive Numbers These are series of numbers sum of its digits is divisible by 3.
differing by 1 in ascending or descending order.
426 ⇒ 4 + 2 + 6 = 12, which is divisible by 3.
e.g., 12, 13, 14, 15, ...
Hence, 426 is also divisible by 3.
Similarly, example of consecutive even numbers are 4, 6,
8349 ⇒ 8 + 3 + 4 + 9 = 24, which is divisible by 3.
8, 10, ..., 22, 24, 26, 28 ... and so on.
Hence, 8349 is also divisible by 3.
e.g. of consecutive prime numbers are 7, 11, 13, 17, ...
7493 ⇒ 7 + 4 + 9 + 3 = 23. 23 is not divisible by 3.
Decimal Numbers A collection of digits (0, 1, 2, 3, ... ,9)
after a point (called the decimal point) is called a decimal Hence, 7943 is also not divisible by 3.
fraction. Note This rule comes from the fact that the multiples of 3 are numbers,
A number containing a decimal point is called a decimal the sum of whose digits is divisible by 3.
number. 3 × 6 = 18; 1 + 8 = 9 9 is divisible by 3
Every decimal fraction represents a fraction. These 3 × 27 = 81; 8+1= 9
fractions have denominators with powers of 10. As
3 × 14 = 42; 4+ 2= 6
4 6 7
35.467 = ( 3 × 101 ) + ( 5 × 100 ) + 1 + 2 + 3 3 × 220 = 660; 6 + 6 + 0 = 12
10 10 10
4 6 7 3 × 12547 = 37641; 3 + 7 + 6 + 4 + 1 = 21 and so on.
= ( 3 × 10) + ( 5 × 1) + + + In the above examples 9, 6, 12 and 21 all are divisible
10 100 1000
by 3.
4 6 7
= 30 + 5 + + +
10 100 1000 Divisibility by 4 A number is divisible by 4, when the
400 + 60 + 7 467 35467 number formed by its two extreme right digits is either
= 35 + = 35 + = divisible by 4 or both these digits are zero.
1000 1000 1000
e.g., 324, 5632, 3500 , 4320 and 89412 are divisible by 4
Perfect Square Numbers The numbers whose square as they satisfy the above mentioned conditions.
root can be determined, are called perfect square
323, 5131, 3510 and 54645 are not divisible by 4 as 23,
numbers.
31, 10 and 45 respectively, are not divisible by 4 nor is
e.g., 4, 36, 225, … etc. the condition of double zero fulfilled.
Two Digits Numbers If unit place digit is y and ten
Note This rule is derived from the fact that 100 or multiples of 100 are
digit is x, then the two digits numbers can be expressed always divisible by 4. So, if any such two digit number which is
as 10x + y. divisible by 4, is added to the multiples of 100, the sum will also be
e.g., 87 = 10 × 8 + 7 divisible by 4.

Divisibility by 5 A number is divisible by 5, when its


Divisibility unit’s digit is either 5 or zero.
A number (dividend) is said to be divisible by another e.g., 2145, 630, 735 and 4500 are divisible by 5 as they
number (called divisor) when the quotient is a natural have either 5 or 0 at their unit’s place.
number and the remainder is zero. In other words, we
1546, 243, 11, 19647 are not divisible by 5 as they do not
can say that when a number (dividend) is divisible by
satisfy the above condition.
another number (divisor), the dividend can be expressed

https://sscstudy.com/
https://sscstudy.com/

NUMERICAL ABILITY 5

Divisibility by 6 A number is divisible by 6, when it is Divisibility by 11 A number is divisible by 11 when the


divisible by 2 as well as 3. This rule emanates from the difference between when the sum of the digits at odd
fact that 2 and 3 are the two factors or sub-multiples of places and the sum of the digits at even places is either
6. Hence, both the conditions i.e., for divisibility by 2 as 0 or divisible by 11.
well as divisibility by 3 must be satisfied simultaneously. 9851833
Alternatively, for a number to be divisible by 6 it must Difference = Sum of digits at odd places
have either zero or an even digit at the unit’s place and − Sum of digits at even places
also simultaneously the sum of its digits must be
= ( 9 + 5 + 8 + 4) − ( 8 + 1 + 3 + 3) = 26 − 15 = 11
divisible by 3.
So, the given number is divisible by 11.
72
602613 Difference = ( 6 + 2 + 1) − ( 0 + 6 + 3) = 9 − 9 = 0
(i) Its unit’s digit i.e., 2 is an even number and hence
So, the given number is divisible by 11.
it is divisible by 2.
(ii) Also, the sum of its digits equals 9 ( = 7 + 2), which Divisibility by 12 A number is divisible by 12, when it is
is divisible by 3. divisible by both the numbers 3 and 4. This is so because
(iii) Therefore, 72 is divisible by 6. 4 and 3 are the two factors or sub-multiples of 12.
Therefore, conditions of divisibility by 4 as well as 3
2247 must be satisfied simultaneously.
(i) Its unit’s digit is an odd number and hence it is not
Alternatively, for a number to be divisible by 12, its last
divisible by 2.
two digits must be either zero or the number formed by
(ii) The sum of its digits equals 15 ( = 2 + 2 + 4 + 7), them, must be divisible by 4 and at the same time the
which is divisible by 3. sum of all the digits of the number must be divisible by 3.
(iii) We see that both the conditions i.e., divisibility by 9612
2 and divisibility by 3 are not satisfied
(i) 12 is divisible by 4. So, the number is divisible by 4.
simultaneously. Therefore, 2247 is not divisible by
6, even though the number is divisibly by 3. (ii) 9 + 6 + 1 + 2 = 18, which is divisible by 3. Hence,
9612 is divisible by 3.
Divisibility by 7 A number is divisible by 7, when the (iii) Therefore, 9612 is divisible by 12.
difference between twice the digit at ones and the
number formed by other digits is either zero or a 7623
multiple of 7. (i) Its last two digits are neither zeros nor is 23
divisible by 4. So, the number is not divisible by 4.
e.g., 658 is divisible by 7
(ii) 7 + 6 + 2 + 3 = 18, which is divisible 3. So, the
because 65 − 2 × 8 = 65 − 16 = 49
number is divisible by 3.
As 49 is divisible by 7, the number 658 is also divisible (iii) We find that conditions for divisibility by 4 and 3
by 7. are not satisfied simultaneously. Therefore, 7623
Divisibility by 8 A number is divisible by 8, when the is not divisible by 12 even though it is divisibly by 3.
number formed by its three extreme right digits is
divisible by 8 or when these last three digits are zeros. To Find the Unit’s Place Digit of a Given Exponential
3648 Since, 648 is divisible by 8, 3648 is also divisible by In case of 0, 1, 5, 6 The unit’s place digit is 0, 1, 5, 6
8. respectively.
11600 Since, 600 is divisible by 8, 11600 is also divisibly In case of 4, 9
by 8. (a) if power is odd → The unit’s place digit is 4 and 9
216000 As, the last three digits of the given number are respectively.
zeros, 216000 is divisible by 8. (b) if power is even → The unit’s place digit is 6 and 1
21700 Since, 700 is not divisible by 8, 21700 is also not respectively.
divisible by 8. In case of 2, 3, 7, 8 See the following example. To find
Divisibility by 9 A number is divisible by 9, when the the unit’s place digit of (134647)553 .
sum of its digits is divisible by 9. Step I 553 ÷ 4 gives 1 as remainder this remainder is
39537 taken as new power.
Sum of the digits = 3 + 9 + 5 + 3 + 7 = 27. Step II (134647)553 = (134647)1 = 71 = 7.
Since, 27 is divisible by 9, 39537 is also divisible by 9.
∴ The unit’s place digit is 7.
Divisibility by 10 A number is divisible by 10, when it
has zero at its unit’s place. On dividing it, the remainder obtained is zero, take 4 as
new power instead of zero. e.g., (134647)552
e.g., The numbers 150, 7250, 1900, 35450 etc., are
divisible by 10. Sol. (134647)552 = (134647)0 = 70 = 74 = 2401
8564, 7509, 29005 etc., are not divisible by 10. ∴ The unit’s place digit is 1.

https://sscstudy.com/
https://sscstudy.com/

6 CUET (UG) Section III : General Test

312, 325, 338, ..., 988


Important Tips/Formulae
Here, a = 312, cd = 13
n ( n + 1)
Sum of the first n natural numbers = Let n be the numbers of terms.
2
Sum of the first n even natural numbers = n( n + 1) ∴ tn = a + (n − 1) d, 988 = 312 + (n − 1) × 13
676
Sum of the first n odd natural numbers = n2 ⇒ = n −1
n( n + 1)( 2n + 1) 13
Sum of the squares of first n natural numbers =
6 ⇒ n = 52 + 1 = 53
2
( + 1)
Sum of the cubes of first n natural numbers =  
n n Alternate

 2  Required number =
1000 300

nth term of the series a, a + d, a + 2d, a + 3d , ... = a + ( n − 1) d 13 13
Where a is the first term and d is common difference. 12 1
= 76 − 23
Sum to the n terms of the series a, a + d, a + 2d ... 13 13
n n
= [2a + ( n − 1) d ] or = [ a + l ] = 76 − 23 = 53
2 2
Where l = last term 3. Find the sum of all natural numbers from 75 to 97.
Dividend = (Divisor × Quotient) + Remainder (a) 2008 (b) 1985
(c) 1895 (d) 1978
74 (74 + 1)
Solved Examples Sol. (d) 1 + 2 + 3 + ... + 74 = = 37 × 75 = 2775
2
1. Find the unit digit in the expression 97 (97 + 1) 97 × 98
and 1 + 2 + 3 + K + 97 = = = 4753
2 2
55725 + 735810 + 22853.
(a) 5 (b) 6 (c) 7 (d) 4 ∴ 75 + 76 + 77 + 78 + K + 97 = 4753 − 2775 = 1978
725
Sol. (b) The unit digit of 55 is 5. 4. A number, when divided by 114, leaves remainder
73 5810
is 9 (Remainder is 2 ∴ (3)2 = 9) 21. If the same number is divided by 19, then the
22853 is 2 remainder will be
(a) 1 (b) 2
∴ The unit a digit of the expression = 5 + 9 + 2 = 16 i.e., 6
(c) 7 (d) 17
2. How many integers lie between 300 and 1000, Sol. (b) Let given number = (14K + 2)
which are exactly divisible by 13?
= (19 × 6K ) + 19 + 2
(a) 50 (b) 54 (c) 53 (d) 52
= 19 (6K + 1) + 2
Sol. (c) The numbers lying between 300 and 1000, which Hence, the required remainder is 2.
are exactly divisible by 13, are

Practice Questions
1. A number of three digits when divided by 2, 5, 9, 6. The sum of the squares of 3 consecutive positive
11 leaves remainder 1 in each case. The number is numbers is 365. The sum of the numbers is
(a) 981 (b) 983 (c) 991 (d) 997 (a) 30 (b) 33 (c) 36 (d) 45
2. The sum of a two digit number and the number 7. In a division sum, the divisor is 10 times the
obtained by reversing its digits is a square number. quotient and 5 times the remainder. If the
How many such numbers are there? remainder is 46, then the dividend
(a) 5 (b) 6 (c) 7 (d) 8 (a) 4236 (b) 4306 (c) 4336 (d) 5336
3. When a number is divided by 24, the remainder is 8. If x is a number midway between 10 and 16 and y
16. The remainder when the same number is y
is half of 78, then is equal to
divided by 12 is x
(a) 3 (b) 4 (c) 6 (d) 8 (a) 6 (b) 5 (c) 4 (d) 3
4. The unit’s digit in the product 7 71 × 663 × 3 65 is 9. The difference between the largest 3 digit number
(a) 1 (b) 2 (c) 3 (d) 4 and the smallest 2 digit number is
(a) 989 (b) 899 (c) 998 (d) 988
5. The units digit of the expression
10. What largest number of four digits is exactly
256251 + 36528 + 7354 is
divisible by 88?
(a) 6 (b) 5
(a) 9988 (b) 8888
(c) 4 (d) 0
(c) 9768 (d) 9944

https://sscstudy.com/
https://sscstudy.com/

NUMERICAL ABILITY 7

11. In 337 337 , the unit digit is occupied by x ( x + 1) (2x + 1)


23. If 12 + 22 + 32 + ... + x2 = , then
(a) 1 (b) 3 (c) 7 (d) 9 6
2 2 2 2
12. When a number is divided by 121, the remainder is 1 + 3 + 5 + K + 19 is equal to
(a) 1330 (b) 2100 (c) 2485 (d) 2500
25. If the same number is divided by 11, the
remainder will be 24. The difference between the largest 4 digit number
(a) 3 (b) 4 (c) 6 (d) 25 and the smallest 3 digit number is
13. In a group of cows and hens, the number of legs (a) 9899 (b) 8999 (c) 9989 (d) 9889
are 14 more than twice the number of heads. The 25. What can be said about the expansion of 212 n − 64 n,
number of cows is where n is a positive integer?
(a) 5 (b) 7 (c) 10 (d) 12 (a) Last digit is 4
14. The simplified value of 1 −
1  1  1 (b) Last digit is 8
 1 −  1 −  ... (c) Last digit is 2
 3   4  5
(d) Last two digits are zero
1 − 1  1 − 1  , is
   26. Find the unit digit in the product of
 99   100
(268 × 539 × 826 × 102)
2 1 1 1 (a) 5 (b) 3 (c) 4 (d) 2
(a) (b) (c) (d)
99 25 50 100
27. On dividing a certain number by 357, the
15. In a division problem, the divison is 4 times the remainder is 39. On dividing the same number by
quotient and 3 times the remainder. If the 17. What will be the remainder?
remainder is 4, the dividend is (a) 5 (b) 3 (c) 7 (d) 6
(a) 36 (b) 40 (c) 12 (d) 30
28. Each member of a picnic party contributed twice
16. Which of the least number should be added at 1000 as many rupees as the total collection was ` 3042.
to make it exactly divisible by 45? The number of members present in the party was
(a) 35 (b) 80 (c) 20 (d) 10 (a) 2 (b) 32 (c) 42 (d) 39
17. The sum of odd numbers from 10 to 60 is 29. A number consists of two digits. If the digits
(a) 775 (b) 468 (c) 921 (d) 875 interchange places and the new number is added
18. The sum of three consecutive natural numbers to the original number, the resulting number will
each divisible by 3 is 72. What is the largest among be divisible by
them? (a) 11 (b) 5 (c) 3 (d) 9
(a) 21 (b) 24 (c) 27 (d) 30 30. An 85 m long rod is divided into two parts. If one
19. In a question on division with zero remainder, a 2
part is of the other part, then the longer part
candidate took 12 as divisor, instead of 21. The 3
quotient obtained by him was 35. The correct (in m) is
2
quotient is (a) 34 (b) 56 (c) 85 (d) 51
(a) 13 (b) 20 (c) 0 (d) 12 3

20. The sum of all integers between 200 and 400 31. The unit’s place digit in the product (3127)173 will
divisible by 9 is be
(a) 3366 (b) 6633 (c) 6336 (d) 6363 (a) 1 (b) 3 (c) 7 (d) 9

21. The unit digit of the expression 32. The sum of all natural numbers between 100 and
813 3703 828 200 which are multiples of 3 is
125 × 553 × 4532 is
(a) 5000 (b) 4950 (c) 4980 (d) 4900
(a) 4 (b) 2
(c) 0 (d) 5 33. A number divided by 899 gives a remainder of 63.
If the number is divided by 29, the remainder will
22. Find the number of divisors of 1420.
be
(a) 14 (b) 15
(a) 2 (b) 5 (c) 13 (d) 28
(c) 13 (d) 12

ANSWERS
1. (c) 2. (d) 3. (b) 4. (d) 5. (d) 6. (b) 7. (d) 8. (d) 9. (a) 10. (d)
11. (c) 12. (a) 13. (b) 14. (c) 15. (b) 16. (a) 17. (d) 18. (c) 19. (b) 20. (b)
21. (c) 22. (d) 23. (a) 24. (a) 25. (d) 26. (c) 27. (a) 28. (d) 29. (a) 30. (d)
31. (c) 32. (b) 33. (b)

https://sscstudy.com/
https://sscstudy.com/

8 CUET (UG) Section III : General Test

Hints & Solutions


1. LCM of 2, 5, 9, 11 = 990 = 230 × 23 + 46 = 5336  1  1  1  1
14. 1 −  1 −  1 −  K 1 − 
8. Value of x between 10 and 16  3  4  5  99
Required number = 990 + 1 = 991
will be 11, 12, 13, 14, and 15.  1 
2. Let the two digit number be 1 − 
According to question,  100
10x + y. 78
y= = 39 2 3 4 5 98 99
By given condition, 2 = × × × × ... × ×
3 4 5 6 99 100
10x + y + 10 y + x = z 2 y 39 2 1
(Square number) Then, = = =
x 13 100 50
11(x + y) = z 2
(Let x = 13 because 39 divided by 15. Let divison = d , then d = 4 q = 3r
z2
x+ y = 13) Since r = 4, then d = 3 × 4 = 12
11 y 4 ×3
=3 and q = =3
Putting the all values of x and y, x 4
which sum is 11, we find 8 such 9. Largest three digit’s number ∴ N = dq + r = 12 × 3 + 4 = 40
numbers. 16. 45)1000(22
= 999
3. When 16 is divided by 12 the Smallest two digit’s number = 10 90
remainder is 4. So, the required 100
Difference = 999 − 10 = 989
remainder is 4. 90
10. Largest number of four digits 10
4. Unit digit of [771 × 663 × 365 ]
= 9999 So, the required number
= Unit digit of 88 ) 9999 ( 113
[74×17+ 3 × 6 × 34×16+ 1 ] = 45 − 10 = 35
88
= Unit digit of [73 × 6 × 31 ] 119 17. Sum to first n odd numbers
2
= Unit digit of [343 × 6 × 3] 88  59 + 1
= 
[Q 73 = 343] 319  2 
= Unit digit of [3 × 6 × 3] 264
{Last odd number = 59}
= Unit digit of [54] = 4 55
= (30)2 = 900
∴ Required number
5. Unit digit of and sum to odd numbers from 1 to
(256251 + 36528 + 7354 ) = 9999 − 55 = 9944 10
= Unit digit of [5 + 6 + (3)54 ] 11. The given number is 337337.  9 + 1
2
 10
2
2
=  =   = (5) = 25
= Unit digit of [5 + 6 + (34 )13 × 32] The unit’s place digit is 7, so by  2  2
= Unit digit of [5 + 6 + 1 × 9] applying the method, we divide
∴ The required odd numbers
the power by 4 i.e.,
= Unit digit of [20] = 0 = 900 − 25 = 875
4 ) 37 ( 84
6. Let the three consecutive 18. Let the three consecutive
32
numbers be x, x + 1 and x + 2 number divisible by 3 be
17
respectively. x, (x + 3), (x + 6), then
16
According to question, x + (x + 3) + (x + 6) = 72
x2 + (x + 1)2 + (x + 2)2 = 365 1
3x + 9 = 72
x2 + x2 + 1 + 2x + x2 + 4 + 4x = 365 The remainder is 1.
3x = 63
3x2 + 6x + 5 − 365 = 0 So, the new power is 1.
x = 21
x2 + 2x − 120 = 0 ∴ (337)337 = (337)1
So, the largest number must be
On solving, x = 10 Hence, the unit’s place digit is 7.
x + 6 = 21 + 6 = 27
∴ Sum of numbers 12. By Shortcut : 25 ÷ 11 gives 3 as
19. The number is 35 × 12 = 420
= 10 + 11 + 12 = 33 remainder
Now, correct quotient = 420 ÷ 21
7. Let remainder = 2x = 46 13. Let there be x cows and y hens. = 20
46 Then, (4x + 2 y) − 14 = 2 (x + y)
x= = 23 20. The numbers are 207, 210, 225,
2 4x + 2 y −14 = 2x − 2 y ..., 396
∴ Quotient = 23, Remainder = 46,
4x − 2x = 14 Tn = a + (n − 1) d
Divisor = 230
QDividend = Divisor × Remainde 2x = 14 396 = 207 + (n − 1) × 9
r 14 396 − 207
x= =7 ⇒ n= +1
+ Quotient 2 9

https://sscstudy.com/
https://sscstudy.com/

NUMERICAL ABILITY 9

n = 22 = (4096 − 1296) [(4096)n − 1 According to the question,



n
S n = (a + l) + (4096)n − 2(1296) 2
x = (85 − x)
2 + … + (1296)n − 1 ] 3
22 = 2800 (k)
= (207 + 396) ⇒ 3x = 170 − 2x
2 Hence, last two digits are 170
⇒ x= = 34
= 6633 always be zero. 5
21. The unit digit in 125813 = 5 26. Product of unit digits of the ∴ The length of first part = 34 m
The unit digit in number = 8 × 9 × 6 × 2 = 864 and the length of second part
(553)3703 = Divide 3703 by 4 the ∴ Required digit = 4 = 85 − 34 = 51 m
remainder is 0 = 0 27. Let the given number be 31. Unit’s place digit in (3127)173
The unit digit in (357 k + 39). = unit’s place digit in (7)173
(4532)828 = Divide 828 by 4 the Then, (357k + 39) = 7172+ 1 = (7)1
remainder is 0 = 0
= (17 × 21 k) + (17 × 2) + 5 = unit’s place digit in 1 × 7 = 7
Hence, unit digit will be
= 17 × (21k + 2) + 5 32. Multiple of 3 between 100 and
=5 ×0 ×0 =0
∴ Required remainder = 5 200 are 102, 105, ......, 198.
22. 1420 = 22 × 51 × 711
28. Let the number of members of a Let the number of numbers is n.
∴ Number of divisors picnic = x ∴ 198 = 102 + (n − 1) ⋅ 3
= (2 + 1) (1 + 1) (1 + 1) = 12 ∴ Contribution of each member 198 − 102
⇒ (n − 1) =
23. 12 + 22 + 32 + ...+ 192 = 2x 3
19 × 20 × 39 ∴ 2x × x = 3042 96
= = 2470 = = 32
3042 3
6 ⇒ x2 = = 1521
22 + 42 + K + 182 = 1140 2 ⇒ n = 33
∴ x = 39 33
= 22(12 + 22 + K + 92) ∴ Required sum = (102 + 198)
4 × 9 × 10 × 19 29. Let the unit’s place digit = x and 2
= = 1140
6 ten’s place digit = y 33
= × 300
∴ 1 + 3 + 5 + K + 192
2 2 2 ∴ number = 10 y + x 2
and new number = 10x + y
= 2470 − 1140 = 1330 = 4950
Sum of number
24. Largest four digit’s number = 10 y + x + 10x + y 33. If a number x is divided by 899
= 9999 = 11 (x + y) gives a remainder 63 and
Smallest three digit’s number This shows that the sum of quotient y.
= 100 numbers is divisible by 11. ∴ x = y × 899 + 63
Difference = 9999 − 100 = 9899 30. Let the length of first part = x m ⇒ x = (31 y + 2) × 29 + 5
25. 212n − 64n = (212)n − (64 )n ∴ The length of another part It is clear from above that if x is
= (4096)n − (1296)n = (85 − x) m divided by 29, then the
remainder is 5.

https://sscstudy.com/
https://sscstudy.com/

10 CUET (UG) Section III : General Test

CHAPTER 02

HCF and LCM


HCF of any given set of number is the greatest factor common to them and LCM of two or more numbers is the smallest
number which is common multiple of the given number.

HCF (Highest Common Factor) LCM (Least Common Multiple)


A common factor of two or more numbers is a number The least number which is exactly divisible by each one
which divides each of them exactly. of the given numbers is called their LCM.
e.g., 2 is a common factor of 18 and 24.
Methods of Finding LCM
It is also known as GCD (Greatest Common Divisor) or
GCM (Greatest Common Multiple) Factorisation Method Break the given numbers into
their prime factors and then find the product of highest
Methods of Finding HCF powers of all the factors, which occur in the given
number and this product is the required LCM.
Factorisation Method Break the given numbers into
prime factors and then find the product of common prime e.g., Find the LCM of 24, 45 and 60.
factors with least powers gives the HCF. Sol. 24 = 2 × 2 × 2 × 3 = 23 × 3
e.g., Find the HCF of 24, 45 and 60. 45 = 3 × 3 × 5 = 32 × 5
60 = 2 × 2 × 3 × 5 = 22 × 3 × 5
Sol. 24 = 2 × 2 × 2 × 3 LCM = 23 × 32 × 5 = 8 × 9 × 5 = 360
45 = 3 × 3 × 5 The product of the divisors and the undivided numbers is
60 = 2 × 2 × 3 × 5 the required LCM.
HCF of 24, 45, 60 = 3
Division Method Write down the given numbers in a
Division Method Divide the larger number by smaller line, separating them by commas and then divide by
number and then divide the divisor by the remainder anyone of the prime numbers which exactly divides
and repeat this process till the remainder becomes zero atleast two of the given numbers. Now, write down the
and in the last divisor is the require HCF. quotients and the undivided numbers in the line below
e.g., Find the HCF of 13281 and 15844. the first and repeat this process untill you get a line of
numbers which are prime to one-another. In the last the
Sol. 13281 ) 15844 ( 1 product of all the divisors and the numbers in the last
13281 line is the required LCM.
2563 ) 13281 ( 5
12815 e.g., Find the LCM of 24, 45 and 60.
466 ) 2563 ( 5 Sol. 2 24, 45, 60
2330
2 12, 45, 30
233 ) 466 ( 2
466 3 6, 45, 15
× 5 2, 15, 5

∴ HCF = 233 2 2, 3, 1
If the successive divisions continue until remainder of 1 is 3 1, 3, 1
obtained this means that the original numbers have no 1, 1, 1
common divisor. LCM = 2 × 2 × 2 × 3 × 3 × 5 = 360

https://sscstudy.com/
https://sscstudy.com/

NUMERICAL ABILITY 11

Now, LCM
Important Tips/Formulae 2 12, 30, 40
Product of two numbers = (their HCF) × (their LCM)
Co-prime Two numbers are coprime if their HCF is 1. 2 6, 15, 20
HCF and LCM of fractions 3 3, 15, 10
HCF of numerators 5 1, 5, 10
(i) HCF =
LCM of denominators 2 1, 1, 2
LCM of numerators
(ii) LCM = 1, 1, 1
HCF of denominators
LCM = 2 × 2 × 3 × 5 × 2 = 120
The greatest number that will divide x, y, z leaving remainders
HCF = 35 = 5 × 7
a, b and c respectively is given by HCF of ( x − a ), ( y − b ) and
( z − c). 49 = 7 × 7 21 = 3 × 7
The least number which when divided by x, y and z leaves the ∴ HCF = 7
same remainder R in each case is given by LCM of [( x, y, z) + R]. Required HCF =
7
The greatest number that will divide x, y and z leaving the same 120
remainder in each case is given by HCF of| x − y |, | y − z| and
| z − x |. 3. Two numbers are in the ratio 5 : 7. If their HCF is
4, find the numbers.
(a) 15, 21 (b) 20, 28
Solved Examples (c) 25, 35 (d) 30, 42

1. Among how many children may 429 mangoes and Sol. (b) Let the required numbers be 5x and 7x.
also 715 oranges be equally divided ? Then, their HCF is x.
(a) 143 (b) 123 So, x=4
(c) 152 (d) 160 ∴ The numbers are (5 × 4) and (7 × 4)
Sol. (a) HCF (429, 715) i.e. 20 and 28.
429 )715( 1
4. Find the least number which when divided by 16,
429
286 )429( 1
24 and 36 leaves the remainders 7, 25 and 27
286 respectively.
143 )286( 2 (a) 140 (b) 125
286 (c) 130 (d) 135
× Sol. (d) Here 16 − 7, 24 − 15, 36 − 27 = 9, 9, 9
∴The number of children must be 143.
Required number = LCM of (16, 24, 36) − 9
35 49 21
2. Find the HCF of , and . = 144 − 9 = 135
12 30 40
4 7 2 16, 24, 36
(a) (b)
38 120 2 8, 12, 18
5 7
(c) (d) 3 4, 6, 9
132 142
35 49 2 4, 2, 3
Sol. (b) HCF of ,
12 30 2 2, 1, 3
21 HCF of 35, 49 and 21 3 1, 1, 3
and =
40 LCM of 12, 30 and 40 1, 1, 1

Practice Questions
3 9 15
1. HCF of the reciprocals of the fractions , and 3. Which is the smallest number of five digits which
4 10 16 is divided by 41?
is (a) 10045 (b) 10004
3 3 1 2 (c) 10041 (d) 10025
(a) (b) (c) (d)
15 80 15 45
4. Which of the following number has the highest
2. The LCM of two numbers is 48. The numbers are divisor?
in the ratio 2 : 3. The sum of the numbers is (a) 99 (b) 101
(a) 28 (b) 32 (c) 40 (d) 64 (c) 176 (d) 182

https://sscstudy.com/
https://sscstudy.com/

12 CUET (UG) Section III : General Test

5. In a morning walk, three persons step off together. 15. If the LCM of three numbers is 9570, then their
Their steps measure 80 cm, 85 cm and 90 cm HCF will be
respectively. What is the minimum distance each (a) 11 (b) 12
should walk so that all can cover the same distance (c) 19 (d) 21
in complete steps? 16. The product of the LCM and HCF of two numbers
(a) 12220 cm (b) 12230 cm is 24. The difference of the two numbers is 2. Find
(c) 12240 cm (d) 12250 cm
the numbers.
6. Mayank, Manoj and Ankit begin to jog around a (a) 8 and 6 (b) 8 and 10
circular stadium. They complete their revolutions (c) 12 and 4 (d) 6 and 4
in 425, 565 and 635, respectively. After how many 17. Find out the HCF of 3 8 , 35 , 3 9 and 314
seconds will they be together at the starting point? (a) 314 (b) 38
(a) 520 s (b) 504 s (c) 35 (d) 39
(c) 380 s (d) 480 s
18. Find out of LCM of 45 , 4 −18 , 412 and 4 7
7. The HCF of 1056, 1584, 2178 is (a) 4−81 (b) 45
(a) 66 (b) 56 (c) 47 (d) 412
(c) 62 (d) 64
19. HCF of ` 1.20, ` 3.40, ` 4.80
8. The LCM of 22, 54, 108, 135 is (a) 80 paise (b) 60 paise
(a) 5940 (b) 5490 (c) 40 paise (d) 20 paise
(c) 5405 (d) 5095
20. The HCF and LCM of two numbers are 21 and
5 10 25
9. The HCF of , , is 4641 respectively. If one of the numbers lies
6 18 36 between 200 and 300, the two numbers are
5 25 25 5
(a) (b) (c) (d) (a) 273, 357 (b) 210, 340
36 6 36 18 (c) 215, 314 (d) 210, 252
1 5 2 4
10. The LCM of , , , is 21. Find the largest number which divides 62, 132 and
3 6 9 27 237 to leave the same remainder in each case.
1 10
(a) (b) (a) 21 (b) 30
54 27 (c) 35 (d) 40
20
(c) (d) None of these
3 22. Five bells first begin to toll together and then at
intervals of 3, 5, 7, 8 and 10 s. Find after what
11. Find the ratio between LCM and HCF of 5, 15 and 20.
(a) 8 : 1 (b) 14 : 3 (c) 12 : 2 (d) 12 : 1
interval they will again toll together. How many
times does they toll together in one hour?
12. The HCF and LCM of two numbers are 18 and (a) 14 min, 3 times (b) 12 min, 4 times
3780 respectively. If one of them is 540, then the (c) 14 min, 4 times (d) 12 min, 3 times
second one is 23. The LCM of (16 − x2 ) and ( x2 + x − 6) is
(a) 142 (b) 126 (a) (x − 3)(x + 3) (4 − x2 )
(c) 118 (d) 112 (b) 4 (4 − x2 ) (x + 3)
13. The least number of square tiles required to pave (c) (4 − x2 ) (x − 3)
the ceiling of a room 15 m 17 cm long and 9 m 2 cm (d) (16 − x2 ) (x − 2) (x + 3)
broad is 24. The LCM of two numbers is ( a + b) and their HCF is
(a) 656 (b) 814 (c) 902 (d) 738
P ( a − b). If one of the numbers is P, then the other
14. What is the largest number which can divide 1356, number is
1868 and 2764 leaving 12 as remainder in each Pa
(a) (b) Pab
case ? b
(a + b)
(a) 64 (b) 124 (c) a 2 − b2 (d)
(c) 156 (d) 260 P (a − b)

ANSWERS
1. (d) 2. (c) 3. (b) 4. (c) 5. (c) 6. (b) 7. (a) 8. (a) 9. (a) 10. (c)
11. (d) 12. (b) 13. (b) 14. (a) 15. (a) 16. (d) 17. (c) 18. (d) 19. (d) 20. (a)
21. (c) 22. (c) 23. (d) 24. (c)

https://sscstudy.com/
https://sscstudy.com/

Hints & Solutions


1. Required HCF 5 10 25 ⇒ y= x−2
9. HCF of , ,
4 10 16 6 18 36 LCM × HCF = x × y = 24
= HCF of , and
3 9 15 =
HCF of 5, 10, 25 5
= ⇒ x (x − 2) = 24
HCF of (4,10 and 16) 2 LCM of 6, 18, 36 36 ⇒ x2 − 2x − 24 = 0
= =
LCM of (3, 9 and 15) 45 1 5 2 4 ⇒ (x − 6) (x + 4) = 0
10. The LCM of , , ,
2. Let the two numbers be 2x, 3x 3 6 9 27 ⇒ x=6
respectively. LCM of 1, 5, 2, 4 The numbers are 6 and 4
=
LCM of numbers = 48 HCF of 3, 6, 9, 27 respectively.
2 × 3x = 48 20 17. HCF = Highest common factor
48 =
x= =8 3 = 35
6 So, 35 the highest common
11. LCM of 5, 15, 20 = 60
∴Sum of two numbers factor in 38 , 35 , 39 , 314.
HCF of 5, 15, 20 = 5
= 2 ×8 + 3 ×8 60
= 16 + 24 = 40 The required ratio = = 12 : 1 18. LCM − Least Common Factor
5 So, 412 is least common factor in
3. Smallest number of 5 digits
12. Product of 1st × 2nd number 45 , 4−81, 412 and 4.
= 10000
= HCF × LCM
When 10000 is divided by 41, 19. ` 1.20 = 120 paise
remainder = 37 ⇒ 540 × 2nd number = 18 × 3780 ` 3.40 = 340 paise
Hence, required number 18 × 3780 ` 4.80 = 480 paise
⇒ 2nd number= = 126
= 10000 + (41 − 37) = 10004 540 So, HCF of 120, 340, 480
4. 99 = 3 × 3 × 11 13. Side of the each square tiles = 20 × 6, 20 × 17, 20 × 24
101 = 101 = HCF of 1517 cm and 902 cm HCF = 20 paise
176 = 2 × 2 × 2 × 2 × 11 = 41 cm (highest common factor)
182 = 2 × 7 × 13 Required numbers of tiles 20. Since, the HCF of two numbers
Hence, 176 has the highest 1517 × 902 is 21, hence let the numbers be
= = 814
divisors. 41 × 41 21x and 21 y respectively.
5. Minimum distance each should We know that product of two
14. Required number numbers = HCF × LCM
walk
= HCF of (1356 − 12),
= LCM of 80 cm, 85 cm, 90 cm 21x × 21 y = 21 × 4641
(1868 − 12), (2764 − 12)
5 80, 85, 90 ∴ x y = 221
= HCF of (1344, 1856, 2752)
2 16, 17, 18 = (13 × 17)
1856)2752(1
8, 17, 9 because x and y are co-prime.
1856
Therefore, the numbers are
Minimum distance 896 )1856(2 (21 × 13 , 21 × 17)
= 5 × 2 × 8 × 17 × 9 = 12240 cm 1792 ⇒ (273 , 357)
64 )896(14
6. Required time = LCM of 42, 56 21. Required number
and 635 64
256 = HCF of (132 − 62), (237 − 132)
2 42, 56, 63
256 and (237 − 62)
3 21, 28, 63 × = HCF of 70, 105 and 175 = 35
3 7, 28, 21 HCF of 2752 and 1856 is 64. 22. Required time interval
7 7, 28, 7 Now, HCF of 64 and LCM 1344 = LCM of 3, 5, 7, 8 and 10
4 1, 4, 1 64)1344(21 = 840 s
128 = 14 min
1, 1, 1
64 Number of times they will toll
∴ Required time = 2 × 3 × 3 × 7 × 4 64 together in one hour
= 504 s × 60
= = 4 times.
8. The LCM of 22, 54, 108, 135 HCF = 64 14
2 22, 54, 108, 135 (ignoring the fraction part)
15. We know that LCM of the given
3 11, 27, 54, 135 numbers is always divisible by 23. (16 − x2) = (4 − x) (4 + x)
9 11, 9, 18, 45 their HCF. So, 11 is the HCF. (x2 + x − 6) = (x + 3) (x − 2) LCM
11, 1, 2, 5 16. Let the numbers be x and y. of (16 − x2) and (x2 + x − 6)
Then, x − y = 2 = (16 − x2) (x − 2) (x + 3)
= 2 × 3 × 9 × 11 × 5 × 2 = 5940

https://sscstudy.com/
https://sscstudy.com/

14 CUET (UG) Section III : General Test

CHAPTER 03

Square Root and


Cube Root
‘‘Square and Cubes are the basic operations with equal importance as the binary operations. To cater such problems
you must have the conceptual clarity regarding the same’’.

Square The square root of 49 is written as 49.


The square of any number is the number multiplied by Thus, 49 = 7 × 7 = 7
itself.
e.g., 22, 32, 42, ..., n 2 Square root of a number can be learnt with the help of
given below.
22 = 2 × 2 = 4
81 = 9 100 = 10 121 = 11
42 = 4 × 4 = 16
n2 = n × n 144 = 12 169 = 13 196 = 14

In 22, 32, 42, ... , n 2, figure 2, 3, 4, ..., n are called the 225 = 15 256 = 16 289 = 17
base and at the top of a number tells you to square it. 324 = 18 361 = 19 400 = 20
The above statements can be expressed by saying that
the square of 2 is 4 or two squared is 4 and so on. 441 = 21 484 = 22 529 = 23
576 = 24 525 = 25 676 = 26
Properties of Square
729 = 27 784 = 28 841 = 29
It cannot be a negative number.
It cannot have odd number of zeros at its end. 900 = 30 and so on ....
It cannot end with 2, 3, 7 or 8. Unit Digit in Square
Number Unit Digit Number Unit Digit
Perfect Square (...1) 2 …1 (...6) 2 …6
A natural number is called a perfect square or a square
(...2) 2 …4 (...7) 2 …9
number. If it is the square of some natural number.
(...3) 2 …9 (...8) 2 …4
n = m 2, for some natural number m, then n is said to be
(...4) 2 …6 (...9) 2 …1
a perfect square. 2 2
(...5) …5 (...0) …0
e.g., 4 is a perfect square of 2.9 is a perfect square of 3.
Note How to Calculate the Square Root?
(i) Squares of even numbers are always even.
There are two methods to calculate the square root.
(ii) Squares of odd numbers are always odd.
Prime Factorisation Method
Square Root This method has the following steps.
The square root of a number is that factor of the number Step I Express the given number as the product of
which, when multiplied by itself, will give that number. prime factors.
The above statement can be expressed by if a is the Step II Keep these factors in pairs.
square root of ‘b’, then ‘b’ is the square of ‘a’. Step III Take the product of these prime factors taking
The square root of a number is indicated by the sign one out of every pair of the same primes. This
product gives us the square root of the given
or 2 .
number.

https://sscstudy.com/
https://sscstudy.com/

NUMERICAL ABILITY 15

e.g. Find the square root of 1089. Step IV Bring down the next period, which is 69. Now,
Sol. Prime factors of 1089 = 11 × 11 × 3 × 3 trial divisor is 13 × 2 = 26 and trial dividend is
1869. So, we take 267 as dividend and 7 as
11 1089 quotient. The remainder is 0 now.
11 99
Step V The process (processes like III and IV) goes on till
3 9
all the periods (pairs) come to an end and we get
3 3
remainder as 0 (zero).
1
Hence, the required square root = 137
⇒ 1089 = 11 × 11 × 3 × 3 e.g. What is the square root of 151321?
Now, taking one number from each pair and multiplying Sol. 389
them, we get
3 15 13 21
1089 = 11 × 3 = 33 9
e.g. Find the square root of 1024. 68 613
544
Sol. Prime factors of 1024
769 6921
= 2× 2× 2× 2× 2× 2× 2× 2× 2× 2 6921
⇒ 1024 = 2× 2× 2× 2× 2× 2× 2× 2× 2× 2 ×
∴ Required square root = 389
Now, taking one number from each pair and multiplying
them, we get 1024 = 2 × 2 × 2 × 2 × 2 = 32 Note Division method should be applied, when the given number is so
large that it is very difficult to find its square root by the prime
factorisation method.
2 1024
2 512 Square of Two Digits
2 256
Let AB is 2 digit number, then
2 128
Step I B2
2 64 Step II 2( A × B)
2 32 Step III A2 = A2 + 2 AB + B2
2 16
e.g. (16)2
2 8
2 4 Sol. Here, 1 = A and 6 = B. Then,
Step I (6)2 = 36
2 2
Step II 2(1 × 6) = 12
Step III (1)2 = 1 = 1 + 12 + 36 = 2 + 1 5 + 3 6 = 256
Division Method e.g. (23) 2
The steps of this method can be easily understood with Sol. Here, 2 = A and 3 = B. Then,
the help of following examples Step I (3)2 = 9
e.g. Find the square root of 18769. Step II 2(2 × 3) = 12
Sol. Step I In the given number, mark off the digits in Step III (2)2 = 4 = 4 + 12 + 9 = 5 + 1 2 + 9 = 529
pairs starting from the unit digit. Each pair e.g. ( 26)2
and the remaining one digit (if any), is called
a period. Sol. Here, 2 = A and 6 = B. Then,
Step II Now, on subtracting, we get 0 (zero) as Step I (6)2 = 36
remainder. Step II 2(2 × 6) = 24
Step III Bring down the next period, i.e., 87. Now, Step III (2)2 = 4 = 4 + 24 + 36 = 6 + 27 + 3 6 = 676
the trial divisor is 1 × 2 = 2 and trial
dividend is 87. e.g. ( 47)2
So, we take 23 as divisor and put 3 as quotient. The
Sol. Here, 4 = A and 7 = B. Then,
remainder is 18 now.
Step I (7)2 = 49
137 Step II 2(4 × 7) = 56
1 1 87 69 Step III (4)2 = 16 = 16 + 56 + 49
1
= 22 + 6 0 + 4 9 = 2209
23 87
69
Square of a Number Ending in 5
267 1869
1869 Step I Multiply the number formed after deleting 5 at
× the unit’s place with the number one higher than it.

https://sscstudy.com/
https://sscstudy.com/

16 CUET (UG) Section III : General Test

Step II Annex 25 on the right side of the product and you 2704
will get the square of the given number. e.g. is equal to
81
e.g. (15)2 2704 2704 52
Sol. = =
Sol. Step I 1× 2 = 2 81 81 9
Step II = 25 ⇒ (15)2 = 225 52
e.g. ( 35)2 5 2704
25
Sol. Step I 3 × 4 = 12 102 204
Step II = 25 ⇒ (35)2 = 1225 204
×
Square of Decimal Numbers
Note Sometimes, numerator and denominator are not a complete
Step I Find the square of the number ignoring the square. In these types of cases, it is better to convert the given
decimal point. fraction into decimal fraction to find the square root.
Step II Put the decimal point in such a way that the
number of decimal places in the square is twice Cube
of that in the original number. The cube of any number is the number multiplied by
Step III The square of a decimal number will lie between itself and by itself again.
the square of its integral part and the square of e.g. 23 , 53 , ..., n3
the number one higher than the integral part. A small figure 3 at the top of given number tells you to
e.g. ( 3.6)2 = ( 36)2 = 1296 = 12.96 cube it. 23 is usually read as ‘two cubed’ or the ‘cube of 2’
and so on.
e.g. ( 4.5)2 = ( 45)2 = 2025 = 20.25
The cube of 5 is 5 × 5 × 5 = 125
e.g. (10.5)2 = (105)2 = 11025 = 110.25
Perfect Cube
Square of a Number Ending in 25 A natural number is said to be a perfect cube, if it is the
Step I Multiply the number formed after leaving 25 by cube of some natural number. A natural number n is a
a number 5 suffixed to it. perfect cube, if n = m3 , where m is a natural number.
Step II Annex 625 to the right side of the product. Numbers such as 1, 8, 27, 64, ... are called perfect cube.
e.g. (125)2 = 1 × 15 = 15 hence, (125)2 = 15625
Note Cube of a number is the triple product obtained on multiplying the
e.g. ( 225)2 = 2 × 25 = 50 hence, ( 225)2 = 50625 number by itself.
e.g. ( 325)2 = 3 × 35 = 105 hence, ( 325)2 = 105625 e.g. 93 = 9 × 9 × 9 = 729
113 = 11 × 11 × 11 = 1331
How to Calculate the Square Root of 123 = 12 × 12 × 12 = 1728 and so on....
Decimal Numbers?
Cube of a number can be learnt through this below
If in a given decimal number, the number of digits after
decimal are not even, then we put a 0 (zero) at the 03 = 0 13 = 1 23 = 8
3 3
extreme right. Now, we mark off the periods and try to 3 = 27 4 = 64 53 = 125
calculate the square root applying the division method 63 = 216 73 = 343 83 = 512
mentioned above.
e.g. Find the square root of 147.1369. 93 = 729 103 = 1000 113 = 1331
Sol. 123 = 1728 133 = 2197 143 = 2744
12.13 3 3
15 = 3375 16 = 4096 173 = 4913
1 1 47 . 13 69
1 183 = 5832 193 = 6859 203 = 8000
22 47 213 = 9261 223 = 10648 233 = 12167
44
3 3
241 313 24 = 13824 25 = 15625
241
242 7269 Cube Root
3 7269
The cube root of a number multiplied by itself and by
×
itself again gives the number.
∴ Required square root = 12.13
A natural number m is the cube root of a number n if
How to Calculate the Square Root of n = m3 .
a Fraction? The above statement can be expressed by if m is the
To find square root of a fraction, we have to find the cube root of n, then n is the cube of m. e.g., 125 is the
square roots of numerators and denominators, separately. cube of 5 and therefore 5 is the cube root of 125.

https://sscstudy.com/
https://sscstudy.com/

NUMERICAL ABILITY 17

How to Calculate the Cube Root? 2. If 1369 = 37; find the value of
Prime Factorisation Method 13.69 + 01369
. + 0001369
. + 000001369
.
This method has following steps
(a) 4.1207 (b) 4.1109
Step I Express the given number as the product of (c) 4.1107 (d) 3.8506
prime factors.
Sol. (c) We have,
Step II Keep these factors in a group of three. 1369 1369
Step III Take the product of these prime factors picking Given expression = +
100 10000
one out of every group (group of three) of the 1369 1369
same primes. This product gives us the cube root + +
1000000 100000000
of given number.
37 37 37 37
e.g., Find the cube root of 9261. = + + +
10 100 1000 10000
Sol. Prime factors of 9261 = (3 × 3 × 3) × (7 × 7 × 7) = 3.7 + 0.37 + 0.037 + 0.0037 = 4.1107
9261 = 3 3 × 3 × 3 × 7 × 7 × 7
3
3. Find the smallest number which should be added
Now, taking one number from each group of these,
to 8958 so that the sum is a perfect square.
3 9261 (a) 65 (b) 63
3 3087 (c) 69 (d) 67
3 1029 Sol. (d) 9 8958 94
7 343
9 81
7 49
184 858
7 7
4 736
1
188 122
we get 3
9261 = 3 × 7 = 21
∴Required number = (95) 2 − 8958 = 9025 − 8958
= 67
Important Tips/Formulae
91
ab = a× b 4. Find 3 1 − .
216
a a 4 5 7 3
= (a) (b) (c) (d)
b b 5 6 8 4
The value of x ± x± x ± K∞ 216 − 91 3 125
Sol. (b) 3 =
Suppose ‘a’ and ‘b’ are consecutive factors of x where b > a. If there 216 216
sign is (+) in the expression, the answer is b i.e., bigger factor and if 5× 5× 5 5
=3 =
there sign is (–), the answer is a i.e., the smaller factor. 6× 6× 6 6

e.g., 20 − 20 − 20 − 20 K ∞ is equal to 5. Evaluate 3 0064


. + 1 . 21 .
Sol. 20 = 4 × 5. Since, the sign is ( − ), then the required answer is (a) 1.8 (b) 1.3 (c) 1.5 (d) 2.2
the smaller factor i.e., 4. 64 121
Sol. (c) 3
0.064 + 1.21 = 3 +
1000 100
x ⋅ x ⋅ x ⋅ x K ∞ If the root goes upto ∞ in multiplication, 3
64 121
=3 +
the answer is x itself. 1000 100
4 11
= + = 0. 4 + 1.1 = 1. 5
10 10
Solved Examples 6. Find the value of n, if 3 n = 729
(a) 11 (b) 10
288 (c) 12 (d) 13
1. Find the value of .
128
2 3 1 Sol. (c) 3n = 729
(a) (b) (c) (d) 2
3 2 2 3n = 36
288 288 144 12 3 ∴ 3n = (36 )2 = 312
Sol. (b) = = = =
128 128 64 8 2 ⇒ n = 12

https://sscstudy.com/
https://sscstudy.com/

18 CUET (UG) Section III : General Test

Practice Questions
1. Evaluate 129 + 216 + 68 + 169 16. If a = 0 . 1039, then the value of
4 a2 − 4 a + 1 + 3 a is
(a) 13 (b) 15 (c) 9 (d) 12
112 576 256 (a) 0.1039 (b) 0.2078 (c) 1.1039 (d) 2.1039
2. The simplified value of × × is
196 12 8 17. The largest number of five digits when it is a
(a) 12 (b) 8 (c) 16 (d) 32 perfect square is
6 (a) 99967 (b) 99764 (c) 99856 (d) 99999
3. The value of 0000729
. is
(a) 0.027 (b) 0.3 (c) 0.03 (d) 0.09 18. The least number to be added to 269 to make it a
4. 1499 × 1499 = ? perfect square is
(a) 31 (b) 16 (c) 7 (d) 20
(a) 19501 (b) 1900501
(c) 2247001 (d) 2204701 19. Find the least number which when subtracted
5. 14161 = ? from 1850 makes it a perfect square.
(a) 129 (b) 119 (a) 5 (b) 7 (c) 1 (d) 11
(c) 121 (d) None of these
20. The value of 6 + 6 + 6 + 6 + K is
6. 004
. =? 2 1
(a) 0.002 (b) 0.02 (a) 6 (b) 3 (c) 6 (d) 3
(c) 0.2 (d) None of these 3 2

0.441 21. Find the value of 5 ⋅ 5 ⋅ 5 ⋅ 5 ⋅ K ⋅ ∞ .


7. =?
0625
. (a) 125 (b) 25 (c) 5 (d) 5
(a) 0.048 (b) 0.084
22. The least perfect square number divisible by each
(c) 0.48 (d) 0.84
one of 3, 4, 5, 6, 8 is
1.21 × 0.9 (a) 1200 (b) 1500
8. The value of is
1.1 × 0.11 (c) 3600 (d) 700
(a) 2 (b) 3 (c) 9 (d) 11 7. 2 (?) 3
23. = , then find the value of (?)
9. If 4 n = 1024, then the value of n is
3
0729
. (2) 3
(a) 5 (b) 4 (c) 3 (d) 6
(a) 5 (b) 8 (c) 10 (d) 12
24. What least number should be subtracted from
10. The value of 248 + 52 + 144 is 6860 so that 19 be the cube root of the result from
(a) 14 (b) 16 (c) 16.6 (d) 18.2 this subtraction?
x 13 (a) 3 (b) 2 (c) 4 (d) 1
11. If 1 + = , then x is equal to 2
144 12 25. If p = 999, then 3 p ( p + 3 p + 3) + 1 = ?
(a) 1 (b) 12 (c) 13 (d) 25 (a) 1000 (b) 999 (c) 1002 (d) 998
12. Three-fifth of the square of a certain number is 26. By what least number, 3600 be divided to get a
126.15. What is the number ? perfect cube?
(a) 14.5 (b) 75.69 (a) 9 (b) 50 (c) 300 (d) 450
(c) 145 (d) 210.25
27. By what least number, 675 be multiplied to obtain
13. Given that 1225 = 35, then the value of
a number which is a perfect cube?
12 . 25 + 0 . 1225 + 0 . 001225 is (a) 5 (b) 6 (c) 7 (d) 8
(a) 0.3885 (b) 388.5 28. The largest four-digit number which is a perfect
(c) 38.85 (d) 3.885 cube is
2 5
14. If 5 = 2. 24 and 6 = 2. 45, then the value of + (a) 9999 (b) 9261
3 6 (c) 8000 (d) None of these
is
4
(a) 1.37 (b) 1.57 29. The value of 2 4 + 3 64 + 28 is
(c) 1.73 (d) 1.75 (a) 12 (b) 16 (c) 18 (d) 24
1296 x
15. If = , then find the value of x. 30. The value of 3 (216) −3 ÷ (343) −2 is
x 2.25 36 17 49 49
(a) 10 (b) 9 (c) 8 (d) 6 (a) (b) (c) (d)
156 54 216 412

https://sscstudy.com/
https://sscstudy.com/

ANSWERS
1. (d) 2. (d) 3. (b) 4. (c) 5. (b) 6. (c) 7. (d) 8. (b) 9. (c) 10. (b)
11. (d) 12. (a) 13. (d) 14. (c) 15. (b) 16. (c) 17. (c) 18. (d) 19. (c) 20. (d)
21. (d) 22. (c) 23. (b) 24. (d) 25. (a) 26. (d) 27. (a) 28. (b) 29. (a) 30. (c)

Hints & Solutions


= 248 + 64 ⇒ x2 = 2. 25 × 2 × 2 × 3 × 3
1. 129 + 216 + 68 + 169
⇒ x2 = 2. 25 × 36
= 248 + 8 = 256
= 129 + 216 + 68 + 13 ⇒ x = 2. 25 × 36
= 16
x 13 ⇒ x = 1.5 × 6
= 129 + 216 + 81 11. 1+ = ⇒ x=9
144 12
= 129 + 216 + 9 x 169 16. 4a 2 − 4a + 1 + 3a
⇒ 1+ =
144 144 = (1)2 − 2 × 2a + (2a )2 + 3a
= 129 + 225 x 169
⇒ = −1 = (1 − 2a )2 + 3a
= 129 + 15 = 144 = 12 144 144
112 576 256
=
25 = 1 − 2a + 3a
2. × ×
196 12 8 144 =1 + a
112 24 16 25 = 1 + 0.1039
= × × ⇒ x= × 144 = 25
14 12 8 144 = 1.1039
= 8 × 2 × 2 = 32 Alternate Method 17. Largest number of 5 digits is
3. 6
0.000729 = (0.3) = 0.3
6 6 ⇒ x = 13 + 12 = 25 99999.
? = 1499 × 1499 3 2 3 9 99 99 316
4. 12. x = 126 . 15
5 3 9
⇒ ? = 2247001 126.15 × 5 61 99
x2 = = 42.05 × 5
5. ? = 14161 3 1 61
⇒ ? = 119 x = 210 . 25 626 3899

6. ? = 0.04 x = 14.5 3756


143
⇒ ? = 0.2 13. 12 . 25 + 0 . 1225 + 0 . 001225
∴Required number
0.441 1225 1225 1225
7. ?= = + + = 99999 − 143
0.625 100 10000 1000000 = 99856
441 35 35 35
⇒ ?= = + + 18. As we know that 162 = 256
625 10 100 1000
21 = 3.5 + 0.35 + 0.035 and 172 = 289
⇒ ?=
25 = 3.885 Hence, 289 − 269 (Given number)
⇒ ? = 0.84 2 5 2 5 = 20
14. + = + So, 20 is the required number to
1.21 × 0.9 121 × 9 3 6 3 6
8. = = 9 =3 be added to make given number
1.1 × 0.11 11 × 11 2× 2 5
= + a perfect square.
n 3× 2 6
9. 4n = 1024 ⇒ (4) 2 = 1024 20. 6+ 6+ 6+ 6+K∞
2 + 5 2 + 2.24
n = =
⇒ (4) 2 = 45 6 2.45 6 has two factors 2 and 3.
4.24 Since, here is (+ ve ) sign, so that
n = = 1.73
⇒ =5 2.45 bigger factor is the answer i.e., 3
2 is the answer.
⇒ n = 10 15.
1296
=
x
21. Clearly, 5 is answer
x 2.25
10. 248 + 52 + 144 Since, root goes upto ∞
⇒ x2 = 2.25 × 1296
= 248 + 52 + 12 ⇒ x2 = 2. 25 5 ⋅ 5 ⋅ 5 ⋅ 5 K ∞ = 5 (itself)
× 2 ×2 × 2 × 2 × 3 × 3 × 3 × 3

https://sscstudy.com/
https://sscstudy.com/

20 CUET (UG) Section III : General Test

22. By Hit and Trial 25. 3 p ( p2 + 3 p + 3 ) + 1 ∴ 675 = 52 × 33


As we know that 3600 is the only 1 To get perfect cube, it must be
given option which is a perfect = { p3 + 3 p2 + 3 p + 1}3 multiplied by 5.
square too. And again it is 1
divisible by all the given numbers = [( p + 1)3 ]3 28. By given option,
3, 4, 5, 6, 8 9261 is clearly a perfect cube.
= p + 1 = 999 + 1 = 1000
Hence, 3600 is the required 29. 24 + 3 64 + 4 28
26. Prime factorisation of 3600 is
number.
7.2 (?)3 2 3600 = 22 + 4 + 4 24 × 24
23. 3 = 3 2 1800
0.729 (2) =4 + 4 + 4
3 2 900
7.2 (?) = 12
⇒ = 2 450
729 8 (216)−3
3 3 225
1000 30. 3 (216)−3 ÷ (343)−2 = 3
3 75 (343)−2
7.2 × 8
⇒ (?)3 = 5 25
9 ×9 ×9 (343)2
3 5 5 =3
10 × 10 × 10 1 (216)3
7.2 × 8
⇒ (?)3 = ∴ 3600 = 23 × 2 × 32 × 52 (73 )2
9 / 10 To get perfect cube, it must be =3
7.2 × 8 × 10 (63 )3
⇒ 3
(?) = divided by 2 × 32 × 52 = 450
9 27. Prime factorisation of 675 is 73 × 73
=3
⇒ (?)3 = 64 6 × 63 × 63
3
5 675
⇒ ? = 3 64 = 4
5 135 7×7
24. Q (19)3 = 6859 =
3 27 6 ×6 ×6
∴ Required number 3 9
49
= 6860 − 6859 = 1 3 3 =
216
1

https://sscstudy.com/
https://sscstudy.com/

NUMERICAL ABILITY 21

CHAPTER 04

Indices and Surds


The term indices refer to the power to which a number is raised. Thus, n2 is a number with an index of 2. It prefers the
phrase ‘n’ to the power of ‘2’. Term surd is not often used, instead roots are used, occassionally you will be asked to
given an answer to surd form, this implies that you should provide the answer in the terms of constant and square
roots instead of working out an imprecise decimal approximation.

Indices Surds
Let a is a real number and m is a positive integer, then Let a is a rational number and m is a positive integer,
1 1
a × a × a × a × ... m times = a m where m is known as
indices to the base a. then if a m be a irrational number, then a m is known as
1
surds of power m. i.e. m = m a = mth root of a.
Rules of Indices a
Let a and b be two real numbers and m and n are two Rules of Surds
positive integers, then Let a is a rational number and m and n are two positive
am
am × an = am + n = am − n integers, then
an m a ma
am ( n a )n = a m
ab = m a ⋅ m b
m =
 a b mb
( a m )n = a m × n   = m
 b b ( m a )n = m a n m n
a = mn a
−m 1
( ab) = a × b
m m m
a = m
a Similarity
( a )0 = 1 If the product of two surds is rational, then each of the two surds is
e.g. If 4a = 5, 5b = 6, 6c = 7, 7d = 8, then find the value of called a rationalizing factor of the other.
1
abcd. Rationalizing factor of = a
a
Sol. We have, 4a = 5 ...(i) Rationalizing factor of
1
=am b
5b = 6 ...(ii) a± b
6c = 7 ...(iii) 1
Rationalizing factor of = am b
7d = 8 ...(iv) a± b
On putting value of 5 from Eq. (i) in Eq. (ii), we get
(4a )b = 6 Addition and Subtraction of Surds
⇒ 4ab = 6 [Q (am )n = amn] For addition and subtraction of surds, first make the
Likewise (4 ) = 7
ab c possible factors of the terms, then add or subtract the
⇒ 4abc = 7 equivalent surds.
and (4 ) = 8 ⇒ 4abcd = 8
abc d e.g. Find the value of 80 + 3 245 − 125.
⇒ (22 ) abcd = 23
Sol. 80 = 16 × 5 = 4 5
⇒ 22abcd = 23
Since, there exist equality between them with same base, 3 245 = 3 49 × 5 = 21 5
then their power must be equal. 125 = 25 × 5 = 5 5
3
2abcd = 3 ⇒ abcd = = 15 . ∴ 80 + 3 245 − 125 = 4 5 + 21 5 − 5 5 = 20 5
2

https://sscstudy.com/
https://sscstudy.com/

22 CUET (UG) Section III : General Test

Multiplication and Division of Surds 153 × 212 (5 × 3)3 × (7 × 3)2


Sol. (c) =
352 × 34 (5 × 7)2 × 34
For multiplication and division of surds, first make the
denominates of the powers, same as we did while 53 × 33 × 72 × 32 34
= = 4 =1
arranging them in increasing or decreasing order (or in 52 × 72 × 34 3
comparison). Then, multiply or divide as usual. 1
2
e.g. Find the product of 5 , 6 6 and 3 4. 3. If [3 m ÷ (3 m )2 ] m = 81, then find the value of m.
(a) 0, 6 (b) 2, 4
Sol. LCM of 2, 6 and 3 = 6
(c) 1, 4 (d) 5, 1
∴ 5 = 5 = 5 = (125) , 6 = 6 = (6)
1/ 2 3/ 6 1/ 6 6 1/ 6 1/ 6
1
4 = 41/3 = 42/ 6 = (16)1/ 6
3
 3m2 m
∴ Required product = (125 × 6 × 16)1/ 6 = (12000)1/ 6 Sol. (a)  2m  = 81
 3 
e.g., Divide 12 × 41/ 3 by 3 2. 2
⇒ 3 m − 2m = 81m
12 × 41/3 4 × 42/ 6 4 × (16)1/ 6 ⇒ 3m
2 − 2m
= 34m
Sol. = =
3 × 21/ 2 1 × 2 3/ 6 (8)1/ 6
⇒ m2 − 2m − 4m = 0 ⇒ m (m − 6) = 0
1/ 6
= 4  
16 ⇒ m = 0, 6
= 4(2)1/ 6 = 4 6 2
 8
4. If 5 + 3
x = 3, then find the value of x.

Solved Examples (a) 216 (b) 64 (c) 125 (d) 27


3 −6 2 x−1
5+ 3
x =3
1. If     =  
3 3 3 Sol. (b)
, then x is equal to
 5  5  5 ⇒ 5+ 3
x=9
(a) − 2 (b) 2 (c) − 1 (d) 1 ⇒ 3
x = 4 ⇒ x = (4)3 = 64 ⇒ x = 64
3 −6 2x − 1
 3  3 =  
3 4 4
5. Simplify  6 29  ×  3 29  .
Sol. (c)     3 6
 5  5  5
   
3 −6 2x − 1
⇒  3 =  
3 (a) 215 (b) 210
 
 5  5 (c) 212 (d) 216
4 4
⇒ 2x − 1 = − 3 ⇒ 2x = − 3 + 1 3 6  6 3 
−2 Sol. (d)  29  ×  29 
∴ x= = −1    
2 4 4 4 4
 1  1  1   1 
153 × 212 =  ( 6 29 )3  ×  (3 29 ) 6  =  (29 )18  ×  (29 )18 
2. The value of the expression will be      
352 × 34  

(a) 2 (b) 0 (c) 1 (d) 3 = (22 )4 × (22 )4 = (22 × 22 )4 = 216

Practice Questions
1. What should come in place of the question mark (?) 4. The number of boys raised ` 400 for a relief fund,
in the following equation? each boy giving as many 25 paise coin as there
317.5 ÷ 313/ 2 × 31− 3 = ( 31) ? were boys. The number of boys was
(a) 9/2 (b) 6 (a) 40 (b) 16
(c) 7/2 (d) 4 (c) 20 (d) 100
1 1 1 1 1 5. If 6440 soldiers were asked to stand in rows to
2. − + − +
3− 8 8− 7 7− 6 6− 5 5−2 form a perfect square, it was found that 40 soldiers
were left out. What was the number of soldiers in
is equal to each row?
(a) 5 (b) 3 (a) 40 (b) 80
(c) 1 (d) 0 (c) 64 (d) 60
3. In the equation ( x / 21) × ( x / 189) = 1, which of the
6. The value of 5 + 11 + 19 + 29 + 49 is
following number will replace both the x?
(a) 21 (b) 63 (a) 3 (b) 9
(c) 3969 (d) None of these (c) 7 (d) 5

https://sscstudy.com/
https://sscstudy.com/

NUMERICAL ABILITY 23

17. If x = 7 − 4 3, then the value of  x +  is


7. Each member of picnic party contributed twice as 1
many rupees as the total number of members and  x
the total collection was ` 3042. The number of (a) 3 3 (b) 8 3
members present in the party was (c) 14 + 8 3 (d) 14
(a) 2 (b) 32 (c) 40 (d) 39 18. (16) 0.16 × (16) 0. 04 × (2) 0.2 is equal to
8. If cube root of 175616 is 56, then the value of (a) 1 (b) 2 (c) 4 (d) 16
3
175616
. + 0175616
. + 0000175616
. 3
is equal to 3
19. Given 2 = 1.414, then the value of
(a) 0.168 (b) 62.16 8 + 2 32 − 3 128 + 4 50 is
(c) 6.216 (d) 6.116 (a) 8.484 (b) 8.526 (c) 8.426 (d) 8.876
3 ⋅ 9 n + 1 + 9 ⋅ 32 n − 1  1 1 
9. The value of is 20. The value of 2 + 2 + +  is
9 ⋅ 32 n − 6 ⋅ 9 n − 1  2+ 2 2 − 2
3 2
(a) 3 (b) 3 (c) 3n + 1 (d) 3n − 16 (a) 2 (b) 2 − 2
5 5
(c) 2 + 2 (d) 2 2
10. The least number which is a perfect square and 1 1
has 7936 as one of its factors is equal to (6 . 25) 2 × (0 . 0144) 2 + 1
(a) 12.008 (b) 246016 (c) 61504 (d) 240616 21. Simplify 1 1

11. Each student of class 10 contributed some money (0 . 027) 3 × (81) 4


for a picnic. The money contributed by each (a) 0.14 (b) 1.4 (c) 1 (d) 1. 4
student was equal to the cube of the total number ( x 3)2 × x4
of students. If the total collected amount was 22. If = x p , then the value of p is
x10
` 29791, find the total number of students. (a) 26 (b) 2 (c) 1 (d) 0
(a) 15 (b) 27 (c) 31 (d) 34
23. The value of x is257.5 × 52.5 ÷ 125 1.5 = 5x
12. 17 3.5 × 17 7. 3 ÷ 174.2 = 172 , then find the value of (?). (a) 16 (b) 17.5 (c) 8.5 (d) 13
(a) 6.5 (b) 7.2 (c) 6.6 (d) 15.8
9 × 3 × (27)
n 5 3
2 24. If = 27, then n = ?
13. If  2
2
 = 2 , then x is equal to
x 3 × (81)4
 
(a) 3 (b) 2 (c) 1 (d) 0
(a) 2 (b) 0 (c) 3 (d) 1 300 200
2 25. Which of the two is greater 2 or 3 ?

(a) 3200 (b) 2300
14.  −
1  3
 is equal to (c) Both are equal (d) Can’t say
 343 
1 1 1
(a) − (b) (c) − 49 (d) 49 26. If A = 5 + 2 6, then the value of A + is
49 49 A
3 (a) 2 3 (b) 3 (c) 2 (d) 7

15. If 5 5 × 53 ÷ 5 2 = 5x + 2 , then the value of x is  1+ 2   1− 2 
(a) 4 (b) 5 (c) − 3 (d) − 6 27. The value of   +  is
 5 + 3  5 − 3
3 3 6 12
16. Which is greater 4 , 6, 15, 245?
(a) 5 + 6 (b) 2 5 + 6
3 3 6
(a) 4 (b) 6 (c) 15 (d) 12 245 (c) 5 − 6 (d) 2 5 − 3 6

ANSWERS
1. (b) 2. (a) 3. (b) 4. (a) 5. (b) 6. (a) 7. (d) 8. (c) 9. (a) 10. (b)
11. (c) 12. (c) 13. (d) 14. (d) 15. (a) 16. (b) 17. (d) 18. (b) 19. (a) 20. (a)
21. (d) 22. (d) 23. (d) 24. (a) 25. (a) 26. (a) 27. (c)

Hints & Solutions


1. 317.5 ÷ 31
3
2 × 31−3 = ( 31 )? 1 1 9+ 8
2. = × = ( 9 + 8)
⇒ 317.5 ÷ 311.5 × 31−3 = ( 31 )? 3− 8 9 − 8 9+ 8
1 1 1 1 1
⇒ 317.5 −1.5 −3 = ( 31 )? ∴ − + − +
− 8 8 − 7 7− 6 6− 5 5 −2
? ?
⇒ 313 = (31) 2 ⇒ = 3 ⇒ ? = 6 = (3 + 8 ) − ( 8 + 7 ) + ( 7 + 6 ) − ( 6 + 5 ) + ( 5 + 2)
2

https://sscstudy.com/
https://sscstudy.com/

24 CUET (UG) Section III : General Test

=3 + 8 − 8 − 7 + 7 + 6 12. 173.5 × 177.3 ÷ 174.2 = 17? 4 + 2 2 + 2 − 2 −2 − 2


=
− 6 − 5 + 5 + 2 =5 173.5 + 7.3 2
x x ⇒ = 17? 4
3. Q × =1 17 4.2 = =2
21 189 2
⇒ 1710. 8 − 4.2 = 17? 1 1
⇒ x2 = 21 × 189 ⇒ 176. 6 = 17?
(6 . 25) 2 × (0 . 0144) 2 + 1
x = 21 × 21 × 3 × 3 = 21 × 3 = 63 Since, base are same. 21. 1 1
4. Let the number of boys = x ∴ ? = 6.6 (0 . 027)3 × (81) 4
2
According to question,  2
25 2 13. Given,  2  = 2x 2 . 5 × 0 . 12 + 1 1 . 3 13
x = 400 ⇒ x2 = 1600 ⇒ x = 40   = = =
100 2× 2 0 .3 × 3 0 .9 9
⇒ 2 = 2x 13
5. Total number of soldiers = 6440 = = 1 .4
⇒ ( 2 ) = 2x
2
9
and number of left soldiers = 40 1
×2
∴Number of soldiers in square ⇒ 22 = 2x (x3 )2 × x4 x6 + 4
22. = x p ⇒ 10 = x p
= 6440 − 40 = 6400 ⇒ 2 = 2x ⇒ x = 1
1 x 10
x
∴Number of soldiers in a row −
2
2 ⇒ x p = 1 = x0
 1  3 ⇒ x p = x0 ⇒ p = 0
= 6400 = 80 14.  −  = (− 343)3
 343 23. 257.5 × 525.
÷ 1251.5 = 5x
6. 5+ 11 + 19 + 29 + 7 = (− 73 )23
/
= (− 7)2 = 49 ⇒ 5 × 5 . ÷ 54.5 = 5x
15 25
3
− 1
15. 5 5 × 53 ÷ 5 2 = 5x + 2 ⇒ 515 × 2 = 5x
= 5 + 11 + 19 + 36 1 5
5 × 52 × 53 ⇒ 5x = 513 ⇒ x = 13
= 5 + 11 + 19 + 6 ⇒ = 5x + 2
3 9n × 35 × (27)3
− 24. = 27
= 5 + 11 + 25 5 2 3 × (81)4
1 3
= 5 + 11 + 5 = 5 + 16 (32)n × 35 × (33 )3
⇒ 5 × 5 2 × 53 × 5 2 = 5x + 2 ⇒ = 33
= 5 + 4 = 9 =3 1 3 3 × (34 )4
1+ +3 +
7. Let the number of members = x ⇒ 5 2 2 = 5x + 2 32n × 35 × 39
⇒ = 33
According to question, ⇒ 54 + 2 = 5x + 2 3 × 316
2x2 = 3042 ⇒ x+ 2 =6 32n × 314
∴ x=4 ⇒ = 33
⇒ x2 = 1521 317
3
⇒ x = 39 × 39 = 39 16. 4 , 3 6 , 6 15 , 12 245 320
⇒ 32n × 314 = 317 × 33 ⇒ 32n =
8. Q 3
175616 = 56 LCM of 3, 3, 6, 12 = 12 314
Now, ∴ 3
4 = 12 44 = 12 256 ⇒ 32n = 36 ⇒ 2n = 6 ⇒ n = 3
3
175.616 + 3 0.175616 25. In these types of questions,
+ 3 0.000175616
3
6 = 12 64 = 12 1296 either the base or the exponent is
made same.
=3
175616 3 175616
+
6
15 = 12 152 = 12 225
2300 = (23 )100 = 8100
1000 1000000 3200 = (32)100 = 9100
245 = 12 245 = 12 245
12
175616 Now, clearly 9100 > 8100
+3 Clearly, 3 6 is greater.
1000000000 So, 3200 > 2300
56 56 56 17. x= 7 −4 3
= + + 1 26. A = 5 + 2 6 = ( 2 + 3 )2
10 100 1000 ∴ =7+4 3 (Conjugate)
= 5.6 + 0.56 + 0.056 = 6.216 x = 3+ 2
1 1
3 ⋅ 9n + 1 + 9 ⋅ 32n − 1 ∴ x + = 7 − 4 3 + 7 + 4 3 = 14 ∴ = 3− 2
9. x A
9 ⋅ 32n − 6 ⋅ 9n − 1 1
18. (16)0.16 × (16)0. 04 × (2)0. 2 So, A +
3 ⋅ 32n + 2 + 32 ⋅ 32n − 1 A
= 2 2n = (2)0. 64 × (2)0.16 × (2)0. 20
3 ⋅ 3 − 3 × 32n − 2 × 2 = (2)1. 00 = 2 = 3+ 2+ 3− 2
=2 3
32n + 3 + 32n + 1 32n + 1 [9 + 1] 19. 8 + 2 32 − 3 128 + 4 50
= 2n + 2 2n − 1
= 2n − 1 1+ 2 1− 2
3 − 2 ⋅3 3 [27 − 2] = 2 2 + 8 2 − 24 2 + 20 2 27. +
5+ 3 5− 3
32 × 10 18 3 =6 2
= = =3 5 − 3 + 10 − 6 + 5 − 10
25 5 5 = 6 × 1.414 = 8.484
11. Total collected amount = ` 29791 1 1 + 3− 6
20. 2 + 2 + + =
∴Total number of students 2+ 2 2 −2 ( 5 + 3) ( 5 − 3)
= 3 29791 2− 2 2+ 2 2 5 −2 6
=2 + 2 + + = = 5− 6
= 3 31 × 31 × 31 = 31 2 −2 2

https://sscstudy.com/
https://sscstudy.com/

NUMERICAL ABILITY 25

CHAPTER 05

Simplification
In mathematics, we normally use four mathematical operations + (addition), − (subtraction), × (multiplication) and
÷ (division). Now, we will study about these operations in detail.

Addition Multiplication
Adding the objects of different sets means finding the It is a mathematical concept which indicates that how
total number of objects that are being considered. many times a number is added to itself, e.g., if 2 is added
e.g., ‘‘2 marbles and 3 marbles make 5 marbles,’’ you can 4 times, then 2 + 2 + 2 + 2 = 8. This can be done by
write 2 + 3 = 5 on the board or on paper. When you say multiplication also i.e., 2 × 4 = 8
“2 pencils and 3 pencils make 5 pencils”, you could again In other words, we can say that, the basic idea of
write 2 + 3 = 5 and so on. multiplication is repeated addition.
The number to be repeated is called the multiplicand
Addition of Fractional Numbers and the number which indicates how many times the
If any two or more than two fractional numbers are given multiplicand is to be repeated is called the multiplier.
P The sum of the repetition obtained is called the product.
in the form of , then we can do the sum in the following
Q e.g., (i) 5 × 3 = 5 + 5 + 5 = 15
ways.
(ii) 649 × 3 = 649 + 649 + 649 = 1947
P 2P 5P Numerator = P + 2P + 5P 8P
(i) + + = =
Q Q Q Denominator = Q Q General Method of Multiplication
LCM LCM LCM We all know the general method of 48
P× + 2P × + 12P × × 23
P 2P 12P Q R S multiplication, i.e., how multiplication is done.
(ii) + + = let us now derive a general formula. 144
Q R S LCM of Q, R and S 96
The actual multiplication is done in the following 1104
Q T Q T 
P +S = (P + S ) +  +  way.
R U R U
I. 48

Subtraction 23
The process of subtraction is the reverse of addition. In 8 × 3 = 24 we write ‘4’ and carry ‘2’
subtraction we take away or remove some number of
4 8
objects from the given objects to make it smaller.
II. ( 4 × 3) + ( 2 × 8) = 28
Subtraction of Fractional Numbers 2 3
We subtract the fractional numbers in the following Add carry ‘2’ of step I = 28 + 2 = 30
ways
We write ‘0’ at the ten’s place and carry ‘3’.
R P
(i) If the fraction is subtracted from , III. 4 8
Q Q
b
P R P−R 2 3
then − =
Q Q Q 4× 2= 8
Q T Q −T Add carry 3 of step II = 8 + 3 = 11
(ii) P −S = (P − S ) +
R R R IV. The resultant product is 1104.

https://sscstudy.com/
https://sscstudy.com/

26 CUET (UG) Section III : General Test

Multiplying the Fractional Numbers How to Solve the Questions?


While multiplying two or more than two fractions, multiply Following are the steps which are used to solve the questions
numerator with numerator and multiply denominator with Step I First of all read and understand the relation
denominator. which is given for the symbol properly.
P R P× R Step II Keeping the given relation in your mind,
(i) × =
Q S Q× S arrange the numbers according to it.
(ii)
P
×
R
×
T
=
PRT Step III At last, find the value of the expression using
Q S U QSU original symbols.
5 4 20
e.g., (i) × =
1 1 5 13 65
(ii) 2 × 4 = × = e.g. if a * b = a 2 + b2, then 5 * 3 = 52 + 32 = 25 + 9 = 34
3 7 21 2 3 2 3 6
5 4 9 180 90 30
(iii) × × = = =
2 3 7 42 21 7 Solved Examples
1. Add the numbers 5936, 87569, 75.003, 7.8753,
Division 7135.990.
Division is the method of finding how many times one (a) 100732.8638 (b) 100723.8863
given number called the divisor is contained in another (c) 100723.8683 (d) 100732.8683
given number called the dividend. The number Sol. (c) 5936.0000
expressing this, is called the quotient and the excess of 87569.0000
the dividend over the product of the divisor and the 75.0030
quotient is called the remainder. 7.8753
e.g., 265 ÷ 48 Dividend + 7135.9900
Sum = 100723.8683
Divisor 48) 265 ( 5 Quotient
240 2. Simplify 1856 − 3287 + 5432 − 679.
25 Remainder
(a) 3122 (b) 2233
(c) 3322 (d) 4500
Formulae for Division Based Problems Sol. (c) Simplified value
Dividend = Divisor × Quotient + Remainder = 1856 + 5432 − (3287 + 679)
Divisor) Dividend (Quotient = 7288 − 3966 = 3322
Remainder
Dividend − Remainder 3. Find the multiplication of the numbers
Divisor = 5384 and 7329.
Quotient
(a) 39429336 (b) 39459336
Dividend − Remainder (c) 39898736 (d) 38459336
Quotient =
Divisor
Sol. (b) 5384
× 7329
Other Mathematical Operations 48456
10768×
Sometime we face some other operations but actually 16152××
these are not new. 37688 × × ×
e.g., if we assume the symbol * as +, then 39459336
5* 3 = 5 + 3 = 8 4. If m ⊕ n = m 3 − n2 + mn, find 2 ⊕ 3.
It means that our basic mathematical operations come (a) 1 (b) −1
in the form of other symbols but questions are solved by (c) 5 (d) −7
taking the original symbols. 3 2
Sol. (c) Q m ⊕ n = m − n + mn
The symbols given in the question can be *, δ , π , £ etc. ∴ 2 ⊕ 3 = 2 3 − 32 + 2 × 3 = 8 − 9 + 6 = 5

https://sscstudy.com/
https://sscstudy.com/

NUMERICAL ABILITY 27

Practice Questions
3 1 − 2 1 ÷ 1 of 1 1 8. Which of the following will come in place of both the
 
004
.  3 2 2 4 =?
1. of questions marks (?) in the following equation?
003
. 1 1 1 128 ÷ 16 × ? − 7 × 2
+ of =1
3 5 9
1 1 72 − 8 × 6 + ? 2
(a) 1 (b) 5 (c) (d)
5 2 (a) 17 (b) 16
(c) 18 (d) 3
2. The value of
1 1 1 1 9. If a number is decreased by 4 and divided by 6 the
+ + + ... + is
2 +1 3+ 2 4+ 3 100 + 99 result is 8. What would be the result if 2 is
(a) 1 (b) 9
subtracted from the number and then it is divided by
(c) 99 (d) 99 − 1 5?
2 1 2
1 1 2 1 (a) 9 (b) 10 (c) 11 (d) 10
+ − 3 5 5
3. Simplify
3 4 5 2
2 3 3 4 10. If 567567567 is divided by 567, the quotient is
1 of − of (a) 111 (b) 10101
3 4 4 5 (c) 1001001 (d) 3
37 37 74 74
(a) (b) (c) (d) 1 1
78 13 78 13 11. How many are in ?
8 2
4. Ram went to a market and bought one copy of a (a) 8 (b) 4
Mathematics book and two pencils for ` 165. Rahim (c) 2 (d) 16
went to the same market and bought another copy of
12. The difference of the place value and the face value
the same book and ten pencils of the same brand for
of the number 3 in 12345 is
` 169. The price of each pencil was
(a) 299
(a) ` 0.50 (b) ` 1
(b) 297
(c) ` 0.75 (d) ` 2
(c) 298
5. A man has some hens and cows. If the number of (d) None of the above
heads : number of feet = 12 : 35, find out the number 13. When 121012 is divided by 12, the remainder is
of hens, if the number of heads alone is 48. (a) 0
(a) 28 (b) 26 (b) 2
(c) 24 (d) 22 (c) 3
1 (d) 4
6. 1 + is equal to
1 1
1+ 14. A chocolate has 12 equal pieces. Manju gave th of it
5 4
(a) 11/6 (b) 13/6 1 1
to Anju, rd of it to Sujata and th of it to Fiza. The
(c) 15/6 (d) None of these 3 6
7. In the following, which is the greatest number? number of pieces of chocolate left with Manju is
(a) [(2 + 2) 2] 2 (b) ( 2 + 2 + 2) 2 (a) 1 (b) 2
(c) (4) 2 (d) ( 2 × 2 × 2) 2 (c) 3 (d) 4

ANSWERS
1. (b) 2. (b) 3. (a) 4. (a) 5. (b) 6. (a) 7. (a) 8. (d) 9. (d) 10. (c)
11. (b) 12. (b) 13. (d) 14. (c)

https://sscstudy.com/
https://sscstudy.com/

28 CUET (UG) Section III : General Test

Hints & Solutions


 1 1 1 1 4. According to the question, On putting ‘x’ in place of question
3 − 2  ÷ of 1
0.04  3 
2 2 4 Cost price of → mark ‘?’, we get
1. of
0.03 1 1 1 x2 − 8x + 15 = 0
+ of (1 book + 2 pencils) = ` 165 …(i)
3 5 9 (x − 3) (x − 5) = 0
Cost price of →
 10 5 5 ∴ x = 3 or 5
 − ÷ (1 book + 10 pencils) = ` 169 …(ii)
0.04  3 2 8 9. Let the number be N.
= × On subtracting Eq. (i) from Eq.
0.03 1 1 According to the question,
+ (ii), we get
3 45 8 pencils = ` 4 N −4
5 8 =8
× 4 6
4 6 5 ∴ 1 pencil = ` = ` 0.50
= × 8 ⇒ N = (8 × 6) + 4
3 1+ 1
5. Suppose, number of heads = 12K ⇒ N = 52
3 45
and number of feet = 35 K N − 2 52 − 2 50
4 Now, = = = 10
4 3 Then, 12K = 48 ⇒ K = 4 5 5 5
= × ∴ Number of feet = 35 × 4 = 140 567567567
3 16 10. Clearly, = 1001001
45 Again, suppose number of hens 567
=x 1 1
4 4 45 11. Required number of in
= × × =5 and number of cows = y 8 2
3 3 16
∴ x + y = 48 …(i) 1
1 2 −1 8
2. Q × = 2 −1 and 2x + 4 y = 140 = 2 = =4
2 +1 2 −1 1 2
Similarly, ⇒ 2x + 4 (48 − x) = 140
8
1 1 [from Eq. (i)]
+ +... 12. The place value of the given
2 +1 3+ 2 ⇒ 2x + 192 − 4x = 140
number is 300 and the face
1 ⇒ 2x = 52 value of that number is 3.
+
100 + 99 ⇒ x = 26 So, the required difference is
= ( 2 − 1) + ( 3 − 2 ) + 1
6. Expression = 1 + 300 − 3 = 297
1
....+ ( 100 − 99 ) 1+ 13. 12)121012(10084
5
= 100 − 1 = 10 − 1 = 9 12
1 5
1 1 2 1  =1 + =1 +
+ − 5+1 6 101
3 4 5 2  96
3. 5
2 3 3 4 6 + 5 11 52
1 of − of = =
3 4 4 5 48
6 6
1 1 4 − 5  Hence, when 121012 is divided
+ 7. [(2 + 2)2] 2 = (16) 2 = 256
3 4  10  by 12, then remainder is 4.
= (2 + 2 + 2)2 = (6)2 = 36
3 5 4 3
× − × (4)2 = 16 14. The number of pieces of
4 3 5 4 chocolate left with Manju
(2 × 2 × 2)2 = (8)2 = 64
1 1 1  1 1 1
+ − Therefore, 256 > 64 > 36 > 16 =1 −  + + 
3 4  10  ∴ [(2 + 2)2] 2 is the greatest.
 4 3 6
=
5 3  3 + 4 + 2
− 128 ÷ 16 × ? − 7 × 2 =1 −  
4 5 8. =1  12 
72 − 8 × 6 + ?2
1 1
− 8 × ? − 14 9 12 − 9 3
37 20 37 ⇒ =1 =1 − = =
= 3 40 = × = 49 − 48 + ?2 12 12 12
13 120 13 78
⇒ 2
1 + ? = 8 × ? − 14 Hence, number of pieces of
20 chocolate left with Manju is 3.
⇒ ?2 − 8 × ? + 15 = 0

https://sscstudy.com/
https://sscstudy.com/

NUMERICAL ABILITY 29

CHAPTER 06

Average
An average or arithmetic mean of given data is the sum of the given observations divided by number of observation.

Average
The average of a number of quantities belongs to
some kind is their sum divided by the number of Points to be Remember
those quantities. The quantities whose average is to be determined, should be in the same
unit.
In other words an average of a given observation Average of the given observations is x.
or data is a number which is found on dividing the (i) If each observation is increased by a, then new average = x + a
sum of observations or data by the number of (ii) If each observation is decreased by a, then new average = x − a
observations or data given. (iii) If each observation is multiplied by a, then new average = ax
Sum of observations x
Average = (iv) If each observation is divided by a, then new average =
Number of observations a

Important Tips/Formulae
Average of two groups are a and b, respectively. If number of The average of square of consecutive natural numbers upto n
items in first and second group are x and y respectively, then ( n + 1)( 2n + 1)
ax + by =
Average of both the groups = 6
x+ y
The average of cubes of consecutive natural numbers upto n
The average weight/age of group of n person is x. If a person of 2
 n + 1 
y kg/yr is replaced by another person of z kg/yr, then = n 
  2  
z − y
New average weight/age = x +  
 n  n
The average of consecutive even numbers upto n = + 1
The average of marks of n students was calculated as x. But it was 2
later found that the marks of one student had been wrongly n+ 1
The average of consecutive odd numbers upto n =
entered as a instead of b and of another as c instead of d. 2
The correct average The average of first n consecutive even numbers = ( n + 1)
Sum of correct marks −Sum of wrongly entered marks  The average of first n consecutive odd numbers = n
=x+  A man covers a distance d at x km/h and returns back to the
 n 
starting point at y km/h. Then,
n+ 1 2xy
The average of consecutive natural numbers upto n = Average speed during whole journey = km/h
2 ( x + y)

https://sscstudy.com/
https://sscstudy.com/

30 CUET (UG) Section III : General Test

Solved Examples
1. If average of given numbers 3, 5, 6, 7, 9, x is 7, then ⇒ 8x − 65 + w = 8x + 12 ⇒ w = 77 kg
find the value of x. Alternate Method
(a) 12 (b) 10 (c) 14 (d) 11
3+ 5+ 6+ 7+ 9+ x Weight of new person = number of persons × increment of
Sol. (a) Average = average + weight of the person replaced
6
30 + x = 8 × 1.5 + 65 = 77 kg
i.e., 7=
6 4. The average of 9 numbers is 30. The average of
⇒ 30 + x = 42 ⇒ x = 12 first 5 numbers is 25 and that of the last 3
numbers is 35. What is the 6th number?
2. A tabulator while calculating the average marks of (a) 42 (b) 40 (c) 39 (d) 41
100 students of an examination, by mistake enters
Sum of observations
68, instead of 86 and obtained the average as 58; Sol. (b) Average =
Number of observations
the actual average marks of those students is
(a) 58.18 (b) 57.82 (c) 58.81 (d) 57.28 ⇒ Total sum = 9 × 30 = 270
Sum of first five = 5 × 25 = 125
Sol. (a) Actual total marks of 100 students
= 5800 + (86 − 68) = 5818 Sum of last three = 3 × 35 = 105
5818 ∴ 6th number = 270 − (125 + 105) = 40
∴ Required actual average = = 58.18
100
5. A library has an average number of 510 visitors on
3. The average weight of 8 persons increased by Sunday and 240 on other days. The average
1.5 kg when a person weighing 65 kg is replaced by number of visitors per day in a month of 30 days
a new person. What could be the weight of new beginning with Sunday is
person? (a) 290 (b) 285
(a) 75 kg (b) 82 kg (c) 77 kg (d) 70 kg (c) 295 (d) 300
Sol. (c) Let the average weight of 8 persons be x and the Sol. (b) A month beginning with Sunday will have = 5
weight of the new person be w kg. Sundays total number of other days = 30 − 5 = 25 days
Sum of the weight of 8 persons = 8x 510 × 5 + 25 × 240
∴Average number of visitor =
By given condition, new sum = 8x − 65 + w 30
8x − 65 + w 2250 + 6000
∴ New average = = x + 1.5 = = 285 visitors
8 30

Practice Questions
1. The average of 10, 12, 16, 20, p and 26 is 17. Find 5. In a class, the average score of girls in an
the value of p. examination is 73 and that of boys is 71. The
(a) 17 (b) 18 (c) 15 (d) 16 average score for the whole class is 71.8. Find the
2. The average of 11 results is 60 marks. If the percentage of girls.
(a) 40% (b) 50%
average of first six results is 59 marks and that of
(c) 55% (d) 60%
the last six is 62 marks, then the sixth result
contains 6. The average of three numbers is 135. The largest
(a) 65 marks (b) 66 marks number is 195 and the difference between the
(c) 60 marks (d) 61 marks other two is 20. The smallest number is
(a) 65 (b) 95
3. The average wage of workers in a factory is ` 6000.
(c) 105 (d) 115
There are 12 officers having an average wage of
` 14000, while the average wage of the remaining 7. The average of 6 numbers is 30. If first four
persons is ` 5600, the number of the workers is numbers average is 25 and last three numbers
that factory is average is 35, then fourth number will be
(a) 242 (b) 252 (c) 240 (d) 230 (a) 25 (b) 30
(c) 35 (d) 40
4. The average of 5 numbers is 20 and 4 of the
numbers are 10, 15, 20 and 25. If the number are 8. In the first 10 overs of a cricket game, the run rate
arranged in ascending order, then the average of was only 3.2. What should be the run rate in the
the last three i remaining 40 overs to reach the target of 282 runs?
(a) 25 (b) 18.75 (c) 24 (d) 22.33 (a) 6.25 (b) 6.5 (c) 6.75 (d) 7

https://sscstudy.com/
https://sscstudy.com/

NUMERICAL ABILITY 31

9. A grocer has a sale of ` 6435, ` 6927, ` 6855, ` 7230 17. A motorist travels to a place 150 km away at an
and ` 6562 for 5 consecutive months. How much average speed of 50 km/h and returns at 30 km/h. His
sale must he have in the sixth month, so that he average speed for the whole journey (in km/h) is
gets an average sale of ` 6500? (a) 35 (b) 37 (c) 37.5 (d) 40
(a) ` 4991 (b) ` 5991 (c) ` 6001 (d) ` 6991
18. Three years ago, the average age of a family of 5
10. The population of a town increased from 175000 to members was 17 yr. A baby having been born, the
262500 in a decade. The average per cent increase average age of the family is the same today. The
of population per year is present age of the baby is
(a) 4.37% (b) 5% (c) 6% (d) 8.75% (a) 2 yr (b) 2.4 yr (c) 3 yr (d) 1.5 yr
11. The average of the even numbers from 1 to 30 is 19. Five years ago, the average age of P and Q was
(a) 15 (b) 17 (c) 19 (d) 16 15 yr. Average age of P, Q and R today is 20 yr. How
12. The average of 5 consecutive numbers is 18. The old will R be after 10 yr?
(a) 35 yr (b) 40 yr (c) 30 yr (d) 50 yr
highest of these numbers will be
(a) 24 (b) 18 (c) 20 (d) 22 20. The average of 11 observations is 60. If the average
of first five observations is 58 and that of last five
13. The average of two numbers is M. If one number is
is 56, the sixth observation is
N, then the other number is (a) 90 (b) 110 (c) 85 (d) 100
(a) 2N (b) 2M (c) M − N (d) 2M − N
21. A cricketer has completed 14 innings and his
14. The average of marks of a student in 7 subjects is average is 30 runs. How many runs must he make
75. His average in 6 subjects excluding Science is in his next innings so as to raise his average to 32?
72. How many marks did he get in Science? (a) 60 (b) 55 (c) 65 (d) 50
(a) 72 (b) 90
(c) 93 (d) None of these
22. The average of five numbers is 42 while the
average of another eight numbers is 81. What is
15. The average age of a class is 15.8 yr. The average the combined average of all numbers together?
age of the boys in the class is 16.4 yr while that of (a) 66 (b) 60.5 (c) 68.5 (d) 64
the girls is 15.4 yr. What is the ratio of boys to girls
23. The average of first five multiple of 3 is
in the class ?
(a) 3 (b) 9 (c) 12 (d) 15
(a) 1 : 2 (b) 3 : 4
(c) 3 : 5 (d) None of these 3 1 1 1
24. Find the average of four numbers 2 , 5 , 4 , 8 .
4 3 6 2
16. The average of 50 numbers is 38. If two numbers, 3 3
namely 45 and 55 are discarded, the average of (a) 5 (b) 3
16 16
remaining numbers is 5 16
(c) 16 (d) 3
(a) 36.5 (b) 37 (c) 37.5 (d) 37.52 3 3

ANSWERS
1. (b) 2. (b) 3. (b) 4. (a) 5. (a) 6. (b) 7. (a) 8. (a) 9. (a) 10. (b)
11. (d) 12. (c) 13. (d) 14. (c) 15. (d) 16. (c) 17. (c) 18. (a) 19. (c) 20. (a)
21. (a) 22. (a) 23. (b) 24. (a)

Hints & Solutions


1. Average of 10, 12, 16, 20, p and 26 = 17 12 × 14000 + (n − 12) × 5600
6000 = ⇒ 6000n
10 + 12 + 16 + 20 + p + 26 n
⇒ = 17 = 168000 + 5600n − 67200
6
⇒ 84 + p = 102 ⇒ p = 18 ⇒ 400n = 100800
∴ n = 252
2. Required perimeter
= 6 × 59 + 6 × 62 − 11 × 60 4. Sum of 5 numbers = 20 × 5 = 100
Since, 4 of the number are 10, 15, 20 and 25,
= 6 [59 + 62 − 110]
therefore fifth number = 100 − (10 + 15 + 20 + 25)
= 6 [121 − 110] = 6 × 11 = 66 = 100 − 70 = 30
3. Let the total number of employee in factory be n, then Now, arranging in ascending order, last three
According to question, numbers are 20, 25 and 30.

https://sscstudy.com/
https://sscstudy.com/

32 CUET (UG) Section III : General Test

∴ Required average The average per cent increase of 18. Total age of family three years
20 + 25 + 30 75 population ago
= = = 25 87500
3 3 = × 100 = 17 × 5 = 85 yr
5. Let the number of girls and boys 175000 × 10 Total age of family today
are x, y respectively. 8750 = 85 + 5 × 3 = 100 yr
= = 5%
By given condition, 1750 Let the age of the baby be x yr.
73x + 71 y = 71.8 (x + y) 11. The average of even numbers 100 + x
∴ = 17
⇒ 1. 2x = 0. 8 y from 1 to n is 6
2 n + 2 30 + 2
x= y = = ⇒ x = 2 yr
3 2 2
32 19. Total age of P and Q today
∴ Percentage of girls = = 16
x 2 = (15 × 2 + 5 × 2) = 40 yr
= × 100% Total age of P , Q and R today
(x + y) 12. The four consecutive numbers
2 are x, x + 1, x + 2, x + 3, x + 4 = 20 × 3 = 60 yr
y x + (x + 1) + (x + 2) Therefore, present age of R
= 3 × 100% = 40% + (x + 3) + (x + 4) = 60 − 40 = 20 yr
2  ⇒ = 18
 y + y 5 Age of R after 10 yr
3 
5x + 10 = 20 + 10 = 30 yr
⇒ = 18
6. Let smallest number = x 5 20. Sum of 11 observation
∴ Second number = x + 20 ⇒ 5x = 90 − 10 = 60 × 11 = 660
80
According to question, ⇒ x= Sum of first 5 observation
5 = 5 × 58 = 290
Average of three numbers = 135 ∴ x = 16 Sum of last 5 observation
x + x + 20 + 195 The highest number = x + 4 = 5 × 56 = 280
= 135
3 = 16 + 4 = 20 ∴6th observation
⇒ 2x = 135 × 3 − (20 + 195) N +x = 660 − (290 + 280)
13. Given, M = ,
⇒ 2x = 405 − 215 2 = 90
190 where x is the other number. 21. Total runs scored in 14 innings
x= = 95 Then, x = 2 M − N = 30 × 14 = 420
2
14. Average Suppose he makes x runs in his
7. Sum of 6 numbers = 6 × 30 = 180 Sum of numbers in all subjects next inning.
= 420 + x
Sum of first 4 numbers = 4 × 25 Number of subjects Then, = 32
= 100 15
∴ Sum of numbers in 7 subjects
Sum of last 3 numbers = 3 × 35 ∴ x = 32 × 15 − 420 = 60 runs
= 75 × 7 = 525
= 105 Sum of numbers in 6 subjects 22. Sum of five numbers
Hence, fourth number (excluding Science) = 42 × 5 = 210
= (100 + 105) − 180 = 72 × 6 = 432 Sum of eight numbers
= 205 − 180 = 25 ∴ Marks in Science = 81 × 8 = 648
= 525 − 432 = 93 ∴ Average of all numbers
8. Total run in 10 overs
15. Boys Girls 210 + 648 858
= 3. 2 × 10 = 32 runs = = = 66
Remaining run in 40 overs 16.4 15.8 13 13
= 282 − 32 = 250 runs 15.4 23. First five multiple of 3
∴Required run rate in 40 overs 0.4 0.6
Required ratio = 2 : 3 = 3, 6, 9, 12, 15
250
= = 6 .25 3 + 6 + 9 + 12 + 15
40 16. Average of remaining numbers Average =
5
9. Let total sell will be in 6 months (38 × 50) − (45 + 55) 45
= = =9
=`x 48 5
Total amount 1800
Average = = = 37.5 24. Sum = (2 + 5 + 4 + 8) +
Number of months 48
 3 1 1 1
6435 + 6927 + 6855 17. Average speed  + + + 
 4 3 6 2
+ 7230 + 6562 + x Total distance
6500 = = 7
= 19 + = 20 +
3
6 Total time 4 4
⇒ 6500 × 6 = 34009 + x 300 3
= 20 +
∴ x = 39000 − 34009 150 150
+ 4
= ` 4991 Average =
30 50 4
300 20 3 3 3
10. Population increased in a decade = = 37.5 km/h = + =5+ =5
= 262500 − 175000 = 87500 8 4 16 16 16

https://sscstudy.com/
https://sscstudy.com/

NUMERICAL ABILITY 33

CHAPTER 07

Ratio and Proportion


The number of times one quantity contains another quantity of the same kind is called ratio of the two unit.

Ratio Proportion
It is a tool to compare two or more numbers of same The equality of two ratios is called proportion.
quantities. Let a , b, c and d are four quantities, then the
Or proportional are a : b :: c : d.
The ratio of two quantities in the same units is the Note In the proportion a : b :: c : d , a and d are extreme values and b
fraction that one quantity is of the other. and c are mean values.
i.e., Product of means = Product of extreme
a
Thus, the ratio a to b is the fraction written as a : b.
b Properties of Proportion
Note In the ratio a : b, the first term a is antecedent and second term b (i) If x is the third proportional to a , b, then a : b :: b : x.
is consequent. (ii) Mean proportional between a and b is ab.
a c a+b c+d
Properties of Ratios (iii) If = , then =
b d a−b c−d
(i) The value of a ratio remains unchanged, if each one a−b c−d
of its terms is multiplied or divided by a same and = (Componendo and Dividendo)
non-zero number. a+b c+d
(ii) a 2 : b2 is the duplicate ratio of a : b. e.g., Divide ` 1024 among, A , B and C so find the value of
A and B.
(iii) a3 : b3 is the triplicate ratio of a : b.
Sol. Sum of the ratios = 4 + 5 + 7 = 16
First part =  1024 ×  = ` 576
(iv) a : b is the sub-duplicate ratio of a : b. 9

 16 
(v) If a : b and c : d are two ratios, then the compounded
and second part =  1024 ×  = ` 448
7
ratio is ac : bd.  16 

Important Tips/Formulae
a a+ x a−x e.g., If A : B = 5 : 4
If = 1, = 1 and =1
b a+ x a−x and B : C = 3 : 2, then A : B : C = ?
a c e
If = = = ... = k Sol. 5 : 4
b d f
a + c + e + ...
Then, =k 3 : 2
b + d + f + ... = 5× 3: 3× 4: 4×2
If A : B and B : C are given, then A : B : C is given by = 15 : 12 : 8
A : B
a N1 b N2 c N3
If = , = , = , then a : b : c: d is given by
B : C b D1 c D2 d D3

i.e., A ×B :B ×B :B ×C N1 N2 N3 : D1 N2 N3 : D1 D2 N3 : D1 D2 D3

https://sscstudy.com/
https://sscstudy.com/

34 CUET (UG) Section III : General Test

Solved Examples
a 3 b 5 c 7 7+ x 3
1. If = , = , = , then find a : b : c : d. Sol. (d) Let the required number is x, then =
b 4 c 6 d 8 11 + x 4
(a) 105 : 120 : 135 : 180 (b) 105 : 140 : 168 : 192 ⇒ 28 + 4x = 33 + 3x ⇒ 4x − 3x = 33 − 28 ⇒ x = 5
(c) 105 : 140 : 158 : 212 (d) 115 : 140 : 168 : 182
Sol. (b) a : b : c : d 5. If x : y = 3 : 1, then find the ratio of x 3 − y 3 : x 3 + y 3.
(a) 12 : 11 (b) 13 : 21
= N1 N 2N3 : D1 N 2N3 : D1 D2N3 : D1 D2D3
(c) 13 : 14 (d) 21 : 42
= 3 × 5 × 7 : 4 × 5 × 7 : 4 × 6 × 7 : 4 × 6 × 8 = 105 : 140 : 168 : 192
Sol. (c) Let x = 3 k; y = k
2. What is the ratio of 80 paise and ` 5?
(a) 2 : 25 (b) 4 : 35 (c) 4 : 25 (d) 6 : 35 Then, x − y = 27 k3 − k3 = 26 k3
3 3

and x3 + y3 = 27 k3 + k3 = 28 k3
Sol. (c) ` 5 = 5 × 100 paise
80 26 k3
Their ratio = = 4 : 25 Their ratio = = 13 : 14
500 28 k3

3. Two natural numbers are in the ratio 3 : 5 and 6. If two numbers are in the ratio of 5 : 8 and if 9 be
their product is 2160. The smaller of the numbers added to each, the ratio becomes 8 : 11. Find the
is lower number.
(a) 36 (b) 24 (c) 18 (d) 12 (a) 10 (b) 13 (c) 12 (d) 15
Sol. (a) Let the natural numbers are 3x and 5x, then Sol. (d) Let the numbers are 5 x and 8 x.
3x × 5x = 2160 ⇒15x2 = 2160 5x + 9 8
By given condition, =
⇒ x2 = 144 ⇒ x = 12 8x + 9 11
Hence, smaller = 3x = 3 × 12 = 36 ⇒ 64x + 72 = 55x + 99
⇒ 9x = 27
4. What must be added to each term of the ratio
7 : 11 so as to make it equal to 3 : 4? ⇒ x=3
(a) 8 (b) 7.5 (c) 6.5 (d) 5 Hence, lower number = 5 × 3 = 15

Practice Questions
1. The least ratio among 10 : 18, 7 : 21, 12 : 16, 8 : 20 6. What sum of money is to be divided among
is 3 persons in the ratio 3 : 4 : 7, so that the second
(a) 12 : 16 (b) 8 : 20 person receives ` 12 only?
(c) 10 : 18 (d) 7 : 21 (a) ` 21 (b) ` 32
2. If x be the mean proportion between ( x − 2) and (c) ` 9 (d) ` 42
( x − 3), then the value of x is 7. In a certain examination, the number of those who
(a) 6 (b) 5 passed was 4 times the number of those who failed.
(c) 6/5 (d) 5/6 If there had been 35 fewer candidates and 9 more
3. The fourth proportional of the numbers 12, 16, 18 had failed, the ratio of passed and failed
is candidates would have been 2 : 1, then the total
(a) 28 (b) 30 number of candidates was
(c) 20 (d) 24 (a) 135 (b) 155
4. The ratio of land and water in the whole world is (c) 145 (d) 150
1 : 2 . If this ratio in the Northern hemisphere be 8. The monthly income of H and W is in the ratio 4 : 3
2 : 3, then the ratio of land and water in the and the expenditure is in the ratio 3 : 2. If each of
Southern hemisphere is
them saves ` 600 per month, the income of W, in
(a) 4 : 7 (b) 4 : 11
(c) 3 : 4 (d) 4 : 3 rupees is
(a) 1200 (b) 2400
5. A certain number is divided into two parts such (c) 1800 (d) 9000
that 5 times the first part added to 11 times, the a 2 b 4
second part makes 7 times the whole. The ratio of 9. If = and = , then ( a + b) :( b + c) = ?
the first part to the second part is b 3 c 5
(a) 2 : 1 (b) 5 : 11 (a) 3 : 4 (b) 4 : 5
(c) 1 : 2 (d) 2 : 3 (c) 5 : 9 (d) 20 : 27

https://sscstudy.com/
https://sscstudy.com/

NUMERICAL ABILITY 35

10. Ratio between the monthly incomes of A and B is 20. If there are ` 495 in a bag in denominations of ` 1,
9 : 8 and the ratio between their expenditures is 50 paisa and 25 paisa coins, which are in the ratio
8 : 7. If they save ` 500 each, find A’s monthly 1 : 8 : 16. How many 50 paisa coins are there in the
income. bag?
(a) ` 3500 (b) ` 4000 (a) 50 (b) 220
(c) ` 4500 (d) ` 5000 (c) 440 (d) None of these
5x − 2 y 21. The monthly salary of A, B, C are in the ratio of
11. If x : y = 3 : 4, then the value of .
7x + 2y 2 : 3 : 5. If C’s monthly salary is ` 1200 more than
7 7 7 7 that of A, then B ’s annual salary is
(a) (b) (c) (d)
25 23 29 17 (a) ` 14400 (b) ` 24000
1 1 1 1 (c) ` 1200 (d) ` 2000
12. If : = : , then the value of x is
5 x x 1.25 22. The ratio of 40 m and 2 km is
(a) 1.5 (b) 2 (c) 2.5 (d) 3.5 (a) 1 : 50 (b) 2 : 35
ma + nc (c) 1 : 40 (d) 3 : 25
13. If a : b = c : d, then is equal to
mb + nd 23. If p : q = 3 : 4 and q : r = 8 : 9, then p : r is
(a) m : n (b) na : mb (a) 1 : 3 (b) 3 : 2
(c) a : b (d) md : nc (c) 2 : 3 (d) 1 : 2
24. If a + b : b + c : c + a = 6 : 7 : 8 and a + b + c = 14 ,
14. A bag contains an equal number of ` 1, 50 paise and
then the value of c is
25 paise coins respectively. If the total value is ` 35, (a) 6 (b) 7
how many coins of each type are there? (c) 8 (d) 14
(a) 20 (b) 16 (c) 18 (d) 25 1
25. If of A = 75% of B = 06
. of C, then A : B : C is
a2 + b2 3
15. If a : b = c : d, then the value of 2 is
c + d2 (a) 4 : 5 : 9 (b) 5 : 9 : 4
(c) 9 : 5 : 4 (d) 9 : 4 : 5
1 a+ b a−b b2
(a) (b) (c) (d)
2 c+ d c−d d2 26. A certain sum of money is divided between P and
1 1
Q in the ratio of 3 : 5 . If P gets ` 180 less than Q,
16. A dog pursues a cat and takes 5 leaps for every 2 2
6 leaps of the cat, but 4 leaps of the dog are equal
then the square of Q is
to 5 leaps of the cat. Compare the speeds of the dog
(a) ` 315 (b) ` 495
and the cat.
(c) ` 630 (d) ` 810
(a) 15 : 22 (b) 9 : 25
(c) 25 : 21 (d) 25 : 24 27. Four numbers in the ratio 1 : 3 : 4 : 7 add upto give
17. What should be subtracted from each of the a sum of 105. Find the value of the biggest number.
(a) 42 (b) 35 (c) 49 (d) 63
numbers 54, 71, 75 and 99, so that the remainders
are in continued proportion? 28. What number must be taken from each term of the
(a) 9 (b) 7 fraction 27/35 that it may become 2 : 3?
(c) 4 (d) 3 (a) 9 (b) 10 (c) 7 (d) 11
18. Divide 170 into three parts such that the first part 29. The present ratio of ages of A and B is 4 : 5. 18 yr
is 10 more than the second and its ratio with third ago, this ratio was 11 : 16. Find the sum of total of
part is 2 : 5. their present ages.
(a) 22 : 35 : 113 (b) 35 : 55 : 80 (a) 90 yr (b) 105 yr (c) 110 yr (d) 80 yr
(c) 40 : 30 : 100 (d) 35 : 65 : 70
30. If two numbers are in the ratio of 5 : 8 and if 9
19. Ratio of boys to the girls in a class is 5 : 4. Which of added to each, the ratio becomes 8 : 11. Find the
the following cannot be the number of students in sum of the numbers.
the class? (a) 39 (b) 49 (c) 64 (d) 19
(a) 45 (b) 72
(c) 108 (d) 98

ANSWERS
1. (c) 2. (c) 3. (d) 4. (b) 5. (a) 6. (d) 7. (b) 8. (c) 9. (d) 10. (c)
11. (c) 12. (c) 13. (c) 14. (a) 15. (d) 16. (d) 17. (d) 18. (c) 19. (d) 20. (c)
21. (a) 22. (a) 23. (c) 24. (a) 25. (d) 26. (b) 27. (c) 28. (d) 29. (a) 30. (a)

https://sscstudy.com/
https://sscstudy.com/

36 CUET (UG) Section III : General Test

Hints & Solutions


10 2 7 1 7. Let number of failed and passed 1 1 1
1. = = 0.22 , = = 0.33 , 12. × = 2
18 9 21 3 candidates be x and 4x, 5 1. 25 x
12 3 8 2 respectively. Therefore, total
= = 0.75, = = 0 .4 ⇒ x2 = 5 × 1.25
16 4 20 5 number of candidates was 5x.
Thus, 10:18 is the least among x = 6.25 = 2.5
According to the question, if
a c
the given ratios. total number of students had 13. Let = =k
been 5x − 35, then b d
2. Since, x is mean proportion of Then a = bk, c = dk
4x − 35 − 9 2
(x − 2) and (x − 3). =
x+9 1 ma + nc mbk + ndk a
∴ x2 = (x − 2)(x − 3) ∴ = =k=
mb + nd mb + nd b
⇒ x2 = x2 − 5 x + 6 ⇒ 4x − 44 = 2(x + 9)
⇒ 5x = 6 ⇒ 4x − 2x = 18 + 44 14. Number of coins of each type
6 ⇒ 2x = 62 Total value
⇒ x= =
5 ∴ x = 31 Sum of value of each type of coin
35
3. Let x be the fourth proportion of Thus, total number of = = 20
candidates was 31 × 5 i.e., 155. 1 + 0.5 + 0.25
12, 16 and 18, then
16 × 18 a c
x= ⇒ x = 24 8. Let the monthly income of H 15. = = k; a = bk, c = dk
12 b d
and W be `4x, ` 3x respectively
 a 2 + b2 b2(k2 + 1) b2
4. Let the whole world having 30 and the expenditure be ` 3 y,` 2 y  2  = =
equal parts the fraction of the respectively.  c + d2  d 2(k2 + 1) d 2
1
land to the world = × 30 = 10 By given condition, 5 ×5
16. 5 leaps of dog = leaps of cat
3 4x − 3 y = 600 ...(i) 4
parts 3x − 2 y = 600 ...(ii) ∴ Speed of dog : Speed of cat
and the fraction of the water to From Eqs. (i) and (ii), 25
2 x = 600 = : 6 = 25 : 24
the world = × 30 = 20 parts 4
3 ∴Monthly income of W = 600 × 3
= ` 1800 17. Let number x be subtracted from
Q Northern hemisphere carries a 2 b 4
15 parts of a whole, hence the 9. = , = each of the numbers, then
fraction of land at Northern b 3 c 5 54 − x 75 − x
=
hemisphere ⇒ a : b : c = 8 : 12 : 15 71 − x 99 − x
2 Let a = 8k, b = 12k and c = 15k For x = 3, this relation is correct.
= 15 × = 6 parts a + b 8k + 12k 20
5 ∴ = = 18. Let first and third part be
b + c 12k + 15k 27 2x and 5x, then second part be
and the fraction of water at
2x − 10.
Northern hemisphere 10. Let A’s monthly income = ` 9x
(2x) + (2x − 10) + 5x = 170
3 and B ’s monthly income = ` 8x
= 15 × = 9 parts ⇒ x = 20
5 According to question,
Ratio between their Therefore, three parts are 40, 30
∴Ratio of land and water in
expenditures = 8 : 7 and 100.
Southern hemisphere 9x − 500 8
= (10 − 6) : (20 − 9) = 4 : 11 ⇒ = 19. The total number of students
8x − 500 7 should be divisible by (5 + 4) = 9
5. Let first and second part of the 6x − 3500 = 64x − 4000 20. Ratio of number of coins
number be x and y, respectively. x = 500 = 1 : 8 : 16
Then, ∴ A’s monthly income = 9 × 500 Ratio of value of each type of
5x + 11 y = 7(x + y) = ` 4500
coins = 1 : 4 : 4
⇒ 11 y − 7 y = 7x − 5x x 3 4x
11. Q = ⇒ y= ∴ Value of 50 paisa coins
⇒ 4 y = 2x y 4 3 4
∴ x : y = 2 :1  4x = × 495 = 220
5x − 2   9
5x − 2 y  3
6. Let required money be ` x. ∴ = Therefore, number of coins of
7x + 2 y  4x 50 paisa = 220 × 2 = 440
Then, 7x + 2  
 3
4 21. Let the monthly salary of
x = 12 8x
14 5x − A , B and C be 2x, 3x and 5x.
12 × 14 = 3 = 15x − 8x = 7x = 7
5x = 2 x + 1200
⇒ x= = ` 42 8x 21x + 8x 29x 29
4 7x + ⇒ x = ` 400
3

https://sscstudy.com/
https://sscstudy.com/

NUMERICAL ABILITY 37

So, annual salary of B 21 9k 9 4 ⇒ 15x = 105


= k − 6k = = ×
= 3 × 400 × 12 2 2 2 3 ⇒ x=7
= ` 14400 ∴ c=6 ∴ Biggest number = 7 × 7 = 49
1 27 − x 2
22. 24 m = 2 × 1000 = 2000 m 25. of A = 75% of B = 0.6 of C 28. Let x be taken, then =
3 35 − x 3
So that, ratio of 40 m and 2000 1 75 × B
m ⇒ ×A= = 0.6 × C ⇒ 70 − 2x = 81 − 3x ⇒ x = 11
3 100
= 40 : 2000 = 1 : 50 A 3B 3C 29. Let the present ages of A and B
⇒ = =
23. p : q = 3 : 4 and q : r = 8 : 9 3 4 5 are 4x and 5x, respectively.
9 5 4x − 18 11
i.e.,
p 3
= and =
q 8 Now, A = B and C = B By given condition, =
q 4 r 9 4 4 5x − 18 16

On multiplying these two


Then, ratio of ⇒ 64x − 18 × 16 = 55x − 18 × 11
9 5
p p q 3 8 2 A : B :C = B : B : B ⇒ 9x = 18(16 − 11)
= × = × = 4 4
r q r 4 9 3 ⇒ x = 10
Required ratio = 9 : 4 : 5
∴ p: r = 2 :3 ∴The sum of their present ages
26. The ratio of P ’s share and Q ’s = 40 + 50 = 90 yr
24. Given a + b : b + c : c + a = 6 : 7 : 8
share
Let a + b = 6k, b + c = 7k 1 1 7 11 30. Let the numbers are 5x and 8x.
= 3 :5 = : = 7 : 11 By given condition,
and c + a = 8k 2 2 2 2
5x + 9 8
∴ 2(a + b + c) = 6k + 7k + 8k By given condition, =
8x + 9 11
21 11x − 7x = 180 ⇒ x = 45
⇒ a + b+ c= k ⇒ 64x + 72 = 55x + 99
2 ∴ The share of Q
⇒ 9x = 27
21 = 11x = 11 × 45 = ` 495
⇒ 14 = k ⇒ x=3
2 27. Let the numbers are x, 3x, 4x
Hence, the numbers are 5 × 3, 8 × 3
28 4 and 7x. i.e., 15 and 24.
⇒ k= =
21 3 By given condition, The sum of the numbers
Therefore, c = (a + b + c) − (a + b) x + 3x + 4x + 7x = 105 = 15 + 24 = 39

https://sscstudy.com/
https://sscstudy.com/

38 CUET (UG) Section III : General Test

CHAPTER 08

Percentange
Percentage is a method of expressing fractions or parts of any quantity into a equal form. It is expressed in terms of
hundredths.

Per cent
The word per cent means per hundred. Thus 21 per cent
1 1 1 4 1 5
means 21 part out of 100 parts, which can also be 1% = 12 % = 16% = 83 % =
21 100 2 8 25 3 6
written as . Therefore, per cent is a fraction whose 1 2 1 16
100 2% = 16 % = 64% =
50 3 6 25
denominator is 100 and the numerator of this fraction is
1 1 2 1 4
called the rate per cent. 4% = 20% = 40% = 133 %=
25 5 5 3 3
21
So, = 21%. Here 21 per cent is the rate. The sign for 5% =
1
25% =
1
60% =
3 2
66 % =
2
100 20 4 5 3 3
per cent is %. 1 1 1 1 4 2
6 %= 33 % = 80% = 8% =
(i) To convert a fraction or a decimal or a whole number 4 16 3 3 5 25
into a per cent, multiply if by 100. 1 1 1 1 3
8 %= 50% = 37 % =
(ii) To convert a per cent to a fraction or a decimal, 3 12 2 2 8
divided it by 100 and delete the % sign. 1 1 1 7
10% = 100% = 87 % =
10 1 2 8
Also, any fraction or decimal can be converted into its
equivalent percentage by multiplying with 100%. Note Every number is 100% of itself.

Important Tips/Formulae
If A’s income is x% more than that of B,B ’ s income is less than Suppose the price of a certain commodity increases by R% and
100 ⋅ x  expenditure on this commodity remains the same. Then,
that of A by  %.
reduction percentage in consumption =  × 100 %
R
 100 + x 
 100 + R 
If A’s income is x% less than that of B, B’s income is more than Suppose the price of commodity decreases by R% and
100 ⋅ x  expenditure on this commodity remains the same. Increase
that of A by  %.
percentage in consumption =  × 100 %
R
 100 − x 
 100 − R 

https://sscstudy.com/
https://sscstudy.com/

NUMERICAL ABILITY 39

Solved Examples
1. 60% of a number is 24 less than 3/4th of that 3. If the price of the cooking gas increased by 15%, by
number. Find the number how many per cent should a family reduce its
(a) 150 (b) 180 (c) 160 (d) 175 consumption so as not to exceed its budget on
Sol. (c) Let the required number be x. cooking gas?
Then, by given condition (a) 14% (b) 13%
1
3 (c) 17% (d) 13 %
60% of x = x − 24 23
4
Sol. (d) Suppose the price of cooking gas = ` 100
− 24 ⇒ 
60 3x 3x 3x 
⇒ ×x= −  = 24
100 4  4 5 Increased price = ` 115
⇒ 15x − 12x = 480 ⇒ x = 160 ⇒ ` 115, he should reduce ` 15
15
2. If A earns 10% more than B’ s, then how many per ⇒ ` 1, he should reduce `
115
cent less does B earn than A ?
1 1 1 15 1
(a) 11 % (b) 11 % (c) 13% (d) 12 % ⇒ ` 100 he should reduce ` × 100 = 13 %
3 9 2 115 23
 R
× 100 %
Alternate Method
Sol. (b) Required percentage = 
 100 − R  Reduction percentage in consumption
10  R 
= × 100 = × 100 %
100 − 10  (100 + R ) 
10 1
= × 100 = 11 % =
15
× 100 = 13 %
1
90 9 115 23

Practice Questions
1. The length of a rectangle is increased by 40% and 7. The cost of an article worth ` 100 is increased by
its breadth decreased by 30%. The change in the 10% first and again increased by 10%. The total
area of the rectangle is increase in rupees is
(a) 2% increase (b) 2% decrease (a) 20 (b) 21 (c) 110 (d) 121
(c) 10% increase (d) 10% decrease
8. In measuring the sides of a rectangle, there is an
2. The ratio of the number of boys and girls in a excess of 5% on one side and 2% deficit on the
government aided school is 3 : 2. 20% of the boys other. Then, the error per cent in the area is
and 25% of the girls are not scholarship holders. (a) 3.3 (b) 3 (c) 2.9 (d) 2.7
The percentage of students who are scholarship
9. In an examination, 1100 boys and 900 girls
holders is?
appeared. 50% of the boys and 40% of the girls
(a) 70% (b) 48% (c) 60% (d) 78%
passed the examination. The percentge of
3. The side of a square increases by 10%, then find, candidates who failed is
by what per cent, does its area increase? (a) 45 (b) 45.5
(a) 27 (b) 30 (c) 19 (d) 21 (c) 50 (d) 54.5
1 10. When the price of cloth was reduced by 25%, the
4. A number increased by 137 % and the increment
2 quantity of cloth sold increased by 20%. What was
is 33. The number is the effect on gross receipt of the shop?
(a) 27 (b) 22 (c) 24 (d) 25 (a) 5% increase (b) 5% decrease
5. A’s salary is 20% less than B’s salary. Then, B’s (c) 10% increase (d) 10% decrease
salary is more than A’s salary by 11. 20% of (50% of 5000) is
1 2
(a) 33 % (b) 16 % (a) 400 (b) 600 (c) 250 (d) 500
3 3
(c) 20% (d) 25% 12. 23% of 8040 + 42% of 545 = ? % of 3000
(a) 56.17 (b) 63.54
6. In an office 40% of the staff is female, 40% of the (c) 69.27 (d) 71.04
female and 60% of the male voted for me. The
percentage of votes I got was 13. ?% of 8745 = 5159. 55
(a) 24 (b) 42 (a) 47 (b) 49
(c) 50 (d) 52 (c) 54 (d) None of these

https://sscstudy.com/
https://sscstudy.com/

40 CUET (UG) Section III : General Test

14. If 16% of x is same as 12% of 48, then x =? 22. If x earns 25% less than y, what per cent more does
(a) 24 (b) 36 (c) 32 (d) 40 y earn, then x?
1 2
15. 80% of 1450 is (a) 25% (b) 15% (c) 33 % (d) 16 %
3 3
(a) 1160 (b) 1235 (c) 1045 (d) 1250
1
16. 0.008 is what per cent of 0.2? 23. A number increased by 137 % gives 33, then the
2
(a) 0.4 (b) 2 (c) 40 (d) 4
number is
25 (a) 16 (b) 24 (c) 20 (d) 28
17. If x% of is 150, then the value of x is
2
24. An ore contains 26% copper. To get 91 kg of copper
(a) 1000 (b) 1200 (c) 1400 (d) 1500
the quantity of the ore required is
18. 10% of 15% of 20% of ` 500 is (a) 350 kg (b) 250 kg
(a) ` 225 (b) ` 150 (c) ` 67 (d) ` 1.50 (c) 240 kg (d) 450 kg
19. If 35% of a number is 12 less than 50% of that 25. The price of petrol went up by 25% . In order that
number, then the number is expenses on petrol should not increase. One must
(a) 80 (b) 60 (c) 50 (d) 40 reduce travel by
20. What per cent of 400 is 60? (a) 25% (b) 20% (c) 18% (d) 15%
(a) 6 (b) 12 (c) 15 (d) 20 26. In an examination, 35% of the students passed and
21. A reduction of 20% in the price of sugar enable a 455 failed. How many students appeared for the
purchaser to obtain 3 kg more for ` 120. The examination?
original price of sugar per kg is (a) 490 (b) 700
(c) 1300 (d) 845
(a) ` 15 (b) ` 12 (c) ` 8 (d) ` 10

ANSWERS
1. (b) 2. (d) 3. (d) 4. (c) 5. (d) 6. (d) 7. (b) 8. (c) 9. (d) 10. (d)
11. (d) 12. (c) 13. (d) 14. (b) 15. (a) 16. (d) 17. (b) 18. (d) 19. (a) 20. (c)
21. (d) 22. (c) 23. (b) 24. (a) 25. (b) 26. (b)

Hints & Solutions


1. Here, x = 40, y = 30 3. Required percentage increment ∴ Number of my votes
4x 6x
Required change in the area of  x2   (10)2 = 40% of + 60% of
rectangle = 2x +  = 2 × 10 +  10 10
 xy   100  100  16x 36x 52x
= x − y − % = + =
 100 = 20 + 1 = 21 % 100 100 100
 40 × 30 4. Let the number be x. ∴Required percentage = 52%
= 40 − 30 − % Then, 137.5% of x = 33
 100  7. x = 10, y = 10
33 × 100
= −2% ∴ x= = 24 ∴Total increase
Where negative sign shows the 137. 5  10 × 10 
= 10 + 10 + % = 21%
decrement. 5. Required percentage  100 
2. Let number of boys and girls in 20
= × 100% 8. x = 5%, y = − 2%
the school be 3x and 2x, 100 − 20
respectively making 5x as the ∴ Required error per cent in the
20
total number of students. = × 100% = 25%  xy 
80 area = x + y +
Now, number of students who  100 
are scholarship holders 6. Let the total number of employes 5 ×2
= 80% of 3x + 75% of 2x =x =5 −2 − = 2 . 9%
100
= 2 . 4x + 1 . 5x Number of female employes
= 3. 9x 4x 9. Number of total candidates
= 40% of x = = 1100 + 900 = 2000
Therefore, required percentage 10
3. 9x Number of boys passed the
= × 100% and number of male employes examination = 50% of 1100 = 550
5x 6x
= Number of girls passed the
= 78%
10 examination

https://sscstudy.com/
https://sscstudy.com/

NUMERICAL ABILITY 41

900 × 40 14. 16% of x = 12% of 48 ⇒ 3x = (150 − 120) = 30


= 40% of 900 = = 360
100 48 48 × 12 ⇒ x = ` 10 per kg
⇒ x× =
Total number of candidates 100 100 22. Let y earns a % more than x.
passed the examination 48 × 12  25 
= 550 + 360 = 910 ∴x= = 3 × 12 = 36 a% =  × 100 %
16  100 − 25 
Total number of candidates 80
15. × 1450 = 1160  25 
failed the examination a%= × 100 %
= 2000 − 910 = 1090
100  75 
∴ Required percentage 16. Let x% of 0.2 = 0.008 100
a% = %
1090 x 3
= × 100 = 54. 5% ⇒ × 0.2 = 0.008 1
2000 100 ⇒ a % = 33 %
0.008 × 100 3
10. x = − 25%, y = 20% ⇒x= ⇒x=4
0.2 23. Let the number be x, then
∴ Required effect x 25 according to question
17. × = 150
 25 × 20  100 2 x + 37.5% of x = 33
= −25 + 20 − % 150 × 100 × 2 3
 100  ⇒ x= ⇒ x = 1200 x + × x = 33
25 8
= − 10% or 10% decrease  3
18. 10% of 15% of 20% of ` 500 ⇒ x 1 +  = 33
11. 20% of (50% of 5000) 10 15 20  8
 50  = × × × 500 = `1.50 11
= 20% of  × 5000 100 100 100 x× = 33 ⇒ x = 24
 100  8
19. Let the number be x. Then,
= 20% of 2500 24. Since 26% ≡ 91 kg
20 × 2500 35% of x = 50% of x − 12 91
= = 500 35 x Hence, 100% = × 100
100 ⇒ ×x= × 50 − 12 26
100 100
12. ?% of 3000 = 23% of 8040 + 42% 100% = 350 kg
⇒ 35x = 50x − 1200
So, 350 kg ore is required.
of 545 ⇒ x = 80
3000 25. One must reduce travel by x %.
⇒ ?× 20. Let x% of 400 is 60  25 
100 x Then x% =  × 100  %
23 42 i.e., × 400 = 60  100 + 25 
= 8040 × + 545 × 100
100 100  25 
60 × 100 x% =  × 100 %
3000 ⇒x= = 15 ⇒ x = 15  125 
⇒ ?× = 1849.2 + 228 .9 400
100 x% = 20%
⇒ ? × 30 = 2078 .1 21. Let the original price be
2078 .1 ` x per kg. 26. Total number of failed students is
⇒ ?= 445 ≡ (100 − 35)%
30 Reduced price = ` (80% of x)
80 4x 445 ≡ 65%
⇒ ? = 69.27 =` ×x=`
100 5 So, the total number of students
13. ?% of 8745 = 5159.55 120 120 appeared for the examination is
? − =3 ⇒ 455
⇒ 8745 × = 5159.55 4x x given by 100% = × 100
100 5 65
515955 120 × 5 120 91
⇒ ?= = 59 − =3 = × 100 = 700 student
8745 4x x 13

https://sscstudy.com/
https://sscstudy.com/

42 CUET (UG) Section III : General Test

CHAPTER 09

Profit, Loss and


Discount
On sell of an article when a person received greater amount, then the cost price of an article is known as profit and
when received less amount, then the price of an article is known as loss.
The following basic terms are very useful to solve the problems on profit and loss.
Cost Price (CP) The cost price of an article is the price at which the article is bought.
Selling Price (SP) The selling price of an article is the price at which the article is sold.
Profit or Gain If selling price of an article is more than its cost price, there is profit or gain.
Loss If selling price of an article is less than its cost price (CP), there is loss.
In fact, Profit = SP − CP; Loss = CP − SP
Discount Discount is a term used during a business. It is the rebate given to the buyer by the seller to increase the sale.
Discount is always given on marked price or printed price of the article.
i.e., Discount = Marked/Print Price − Selling Price

Important Tips/Formulae
Profit SP − CP A person sells goods at a profit of x%. Had he sold it for ` X
Profit percentage = × 100 = × 100
CP CP more, y% would have been gained.
Loss CP − SP X
Loss percentage = × 100 = × 100 Then, CP is given by = ` × 100
CP CP y−x
SP × 100 SP × 100
CP = = (P = profit, L = loss) A person sells goods at a loss of x%. Had he sold it for ` X more,
(100 + P %) (100 − L %)
he would have been gained y%.
CP (100 + P %) CP(100 − L %)
SP = = Then, CP is given by = `
X
× 100
100 100 y+x
Discount
Discount percentage = × 100 When there is a gain of x% and a loss of y% the net effect is
Marked price
given by.
If two successive discount of x% and y% is given on an article,
Net effect =  x − y −
xy 
100 − x   100 − y  %
then Selling Price = Marked Price ×   ×   100 
 100   100 
If CP of x articles is equal to SP of y articles where x > y.
Equivalent discount =  x + y −
xy 
 x−y
 100  Then, profit percentage = × 100
If a false weight is used during selling of an article, then Gain per y
Error If CP of x articles is equal to SP of y articles where x < y.
cent = × 100%
True value − Error y−x
Here, Error = True weight − False weight Then, loss percentage = × 100
y

https://sscstudy.com/
https://sscstudy.com/

NUMERICAL ABILITY 43

Solved Examples
1. If selling price of an article is 8/5 times its cost Sol. (c) Let CP be ` 100.
price, the profit per cent on it will be? Then, marked price is 20% higher than CP
(a) 50% (b) 60% (c) 65% (d) 70% 120 × 100
8 ∴ Marked Price = 120% of CP = = 120
Sol. (b) SP = CP 100
5
Now, 20% discount is given on marked price, then
⇒ 5 SP = 8 ⇒ Profit = 3 unit (100 − Discount) × Marked Price
3 SP =
⇒ Profit percentage = × 100 = 60% 100
5 (100 − 20) × 120 80 × 120
= = = 96
2. A chair is sold for ` 705 at a gain of 6%. Find its 100 100
cost price. ∴ Loss = CP − SP = 100 − 96 = 4
5 Loss 4
(a) ` 666 (b) ` 670 (c) ` 665 (d) ` 680 Loss percentage = × 100 = × 100 = 4%
53 CP 100
Sol. (c) Let the CP be ` x. Alternate Method
6 53x (± a × ± b) 
Then SP = x + x × = Effective change =  ± a ± b + %
100 50  100 
But SP = ` 705 a = 20% (+ ve for increase/profit)
53x
⇒ = 705 b = − 20% (− ve for decrease/profit)
50 (− 20 × 20) 
705 × 50 5 =  20 − 20 + = − 4 = 4% loss
⇒ x= = 665  100 
53 53
5 5. A chair listed at ` 350 is available at successive
∴ CP = ` 665
53 discounts of 25% and 10%. The selling price of the
chair is
3. If the cost price of 9 pens is equal to the selling
(a) ` 240.25 (b) ` 242.25 (c) ` 236.25 (d) ` 230.25
price of 11 pens, then what is gain or loss
percentage? Sol. (c) Effective discount of 2 successive discounts
(a) 19%
8
(b) 17 %  ± a ± b + (± a × ± b)  − ve for discount
 
3  100 
2 2 + ve for increase a = − 25 ; b = − 10
(c) 18 % (d) 19 %
11 3  − 25 × − 10  
= − 25 − 10 +  
Sol. (c) Let CP of each pen be ` 1,   100 
then CP of 11 pens = ` 11 = − 35 + 2.5 = − 32.5 = 32.5% effective discount
SP of 11 pens = CP of 9 pens = ` 9 (100 − discount) × CP
SP =
Then, loss = ` 2 100
2 200 (100 − 32.5) × 350 67.5 × 350
∴ Loss percentage = × 100 = % ∴ SP = = = ` 236.25
11 11 100 100
2
= 18 % 6. On selling an article for ` 264 a man loses 4%. In
11
order to gain 12%. Find the selling price of the
Alternate Method article.
CP of 9 pens = SP of 11 pens ( y) (a) ` 300 (b) ` 310 (c) ` 295 (d) ` 308
Clearly, x < y Sol. (d) Let CP be ` x.
11 − 9
Then, loss per cent = × 100 96
11 Then, 96% of x = 264 ⇒ × x = 264
100
2 2
= × 100 = 18 % 264 × 100 
11 11 ∴ x =   = 275
 96 
4. The marked price is 20% higher than cost price. A ∴ CP = ` 275, Gain = 12%
discount of 20% is given on the marked price. By 112
SP = 112% of ` 275 = 275 × = 308
this type of sale, there is 100
(a) No loss no gain (b) 4% gain ∴ SP = ` 308
(c) 4% loss (d) 2% loss

https://sscstudy.com/
https://sscstudy.com/

44 CUET (UG) Section III : General Test

Practice Questions
1. If the loss on an article is 5% and its cost price is 12. The cost price of 50 cups is equal to the sale price of
` 90, find the selling price. 40 cups. The percentage of profit in the transaction is
(a) ` 95.50 (b) ` 85.50 (a) 25 (b) 30
(c) ` 85 (d) ` 95 (c) 15 (d) 20
2. A defective TV costing ` 5000 is being sold at a loss of 13. A shopkeeper purchases 10 pieces of certain items for
50%. If the price is further reduced by 50%, then its ` 8.00 and sells them at 8 pieces for ` 10.00. The profit
selling price is percentage of the shopkeeper is
(a) ` 1225 (b) ` 1250 (a) 56.50 (b) 56.25
(c) ` 1025 (d) ` 1200 (c) 25.50 (d) 26.25
3. A grocer buys 10 dozens of eggs at ` 18 per dozen from 14. The marked price is 10% higher than the cost price. A
the wholesale market. Out of these, 10 eggs were found discount of 10% is given on the marked price. In this
broken and had to be thrown away. At what price per kind of sale, the seller
egg should he sell them so as to make a profit of 10%, if (a) losses 1.5% (b) bears no loss, makes no gain
he spent ` 24 on transportation? (c) gains 1% (d) losses 1%
(a) ` 2.00 (b) ` 2.25 15. If an article is sold at a gain of 6% instead of at a loss
(c) ` 2.04 (d) ` 2.50
of 6%, then the seller gets ` 6 more. The cost price of
4. A businessman marks his goods at such price that after the article is
allowing a discount of 15%, he makes a profit of 20%. (a) ` 106 (b) ` 50
The marked price (in `) of an article having cost price (c) ` 94 (d) ` 100
` 170 is 16. The selling price of 20 articles is equal to the cost price
(a) 236 (b) 220 (c) 240 (d) 204 of 22 articles. The gain percentage is
5. Find the rate of discount when marked price is ` 250 (a) 12% (b) 9%
and selling price is ` 235. (c) 10% (d) 11%
(a) 6.0% (b) 7.0% (c) 6.5% (d) 5.0% 17. A man gains 10% by selling an article for a certain price.
6. The list price of a watch is ` 160. After two successive If he sells it at double the price, then the profit made is
discounts, it is sold for ` 122.40. If the first discount is (a) 120% (b) 20%
10%, what is the rate of the second discount? (c) 40% (d) 100%
(a) 13% (b) 18% (c) 16% (d) 15% 18. The cost price of a book is ` 300. The shopkeeper wants
1 1 to gain 20% after allowing a discount of 10% on the
7. If the discount sales reduce from 3 % to 3 %, what
2 3 marked price. Then, the marked price of the book must be
difference does it make to purchases for ` 12102? (a) ` 360 (b) ` 336
(a) ` 20.17 (b) ` 16.35 (c) ` 400 (d) ` 396
(c) ` 15.35 (d) ` 17.35 19. The printed price of a book is ` 60, but the seller allows
8. Hundred apples were bought for ` 300. Out of these successive discounts of 20% and 30%. The net sale price
four were rotten and the rest were sold at ` 50 per is subject to a sales tax of 5%. The net sale price is
dozen. Then, the net profit is (a) ` 36.28 (b) ` 33.60
(a) ` 150 (b) ` 100 (c) ` 50 (d) ` 125 (c) ` 36.60 (d) ` 35.28
3
9. Rekha purchased a scooter for ` 20000 and sold it for 20. A dealer sold of his articles at a gain of 24% and the
4
` 22000. The percentage of profit is remaining at the cost price. Percentage of gain in the
(a) 15 (b) 12 whole transaction is
(c) 10 (d) 20
(a) 15 (b) 18
10. The cost price of a refrigerator is ` 28000. The (c) 24 (d) 32
shopkeeper offers a discount of 20% on it and loses 21. The marked price of a watch is ` 1600. The shopkeeper
10%. The marked price of the refrigerator is gives successive discounts of 10% and x% to the
(a) ` 28000 (b) ` 31500 customer. If the customer pays ` 1224 for the watch, the
(c) ` 25200 (d) ` 27000 value of x is
11. An article listed at ` 26580 is sold at a discount of 10%. (a) 5 (b) 10 (c) 15 (d) 20
Due to festival season the shopkeeper allows a further
22. If the cost price is 95% of the selling price, what is the
discount of 5%. Find the selling price of the article.
profit per cent?
(a) ` 22750.00 (b) ` 22825.00
(a) 4 (b) 4.75 (c) 5 (d) 5.26
(c) ` 22725.90 (d) ` 23922.00

https://sscstudy.com/
https://sscstudy.com/

NUMERICAL ABILITY 45

23. The difference between a discount of 35% and two 35. If a man reduces the selling price of a fan from ` 400 to
successive discounts of 20% on a certain bill was ` 22. ` 380, his loss increases from x% to ( x + 4) %. What is
The amount of the bill was the cost price of the fan?
(a) ` 200 (b) ` 220 (c) ` 1100 (d) ` 2200 (a) ` 420 (b) ` 550
(c) ` 500 (d) ` 400
24. If I purchased 11 books for ` 100 and sold 10 books for
` 110, the percentage of profit per book sold is 36. If the cost price of 36 books is equal to the selling price
(a) 10 (b) 11.5 (c) 17.3 (d) 21 of 30 books, then the gain percentage is
4 2
25. A cloth merchant sold half of his cloth at 40% profit, half (a) 20% (b) 16 % (c) 16% (d) 8 %
6 6
of remaining at 40% loss and the rest was sold at the cost
price. In the total transaction, his gain or loss will be 37. A man buys 6 dozen eggs for ` 10.80 and 12 eggs are
(a) 20% gain (b) 25% loss found rotten and the rest are sold at 5 eggs per rupee.
(c) 10% gain (d) 15% loss Find his gain or loss percentage.
1 1
(a) 11 % , gain (b) 11 %, loss
26. A bookseller sells a book on 10% profit. If he purchase 9 9
this book on 4% loss and sells ` 6 more, then he earns 1 1
(c) 9 % , gain (d) 9 %, loss
18 3 % profit. Find the cost price of the book. 11 11
4
(a) ` 130 (b) ` 140 (c) ` 150 (d) ` 160 38. Sunanda is making a profit of 25% on his selling price,
what is her actual profit percentage?
27. Ravi purchased two toffees in one rupee of some 100
quantity and sells it five toffees in one rupee. Find the (a) (b) 20 (c) 25 (d) 30
3
loss per cent.
(a) 120 (b) 90 (c) 30 (d) 60 39. If a commission of 10% given on the marked price of an
article, the gain is 25%. Find the gain per cent, if
28. The percentage profit earned by selling an article for commission is increased to 20%.
` 1920 is equal in the percentage loss incurred by 1 100 50
(a) 11 (b) 12 (c) (d)
selling the same article for ` 1280. At what price should 9 3 3
the article he sold to make 25% profit?
(a) ` 2000 (b) ` 2200 40. The cost price of an article is 40% of the selling price.
(c) ` 24000 (d) Data inadequate The per cent that the selling price is of cost price is
(a) 250% (b) 240% (c) 60% (d) 40%
29. If 5 lemons are bought for ` 16, then the selling price of
a lemon at 25% profit will be 41. A shopkeeper has certain number of eggs of which 5%
(a) ` 5 (b) ` 4 (c) ` 6 (d) ` 8 are found to be broken. He sells 93% of the remainder
30. Alfred buys an old scooter for ` 4700 and spends and still has 266 eggs left. How many eggs did he
` 800 on its repairs. If he sells the scooter for ` 5800, originally have?
his gain per cent is (a) 3800 (b) 4000
4 5 (c) 4200 (d) None of these
(a) 4 (b) 5 (c) 10 (d) 12
7 11 42. A dishonest dealer marks his goods 20% above the cost
31. A vendor bought toffees at 6 for a rupee. How many for price. He also makes a profit by using a false weight of
a rupee must be sell to gain 20%? 900 g in place of 1 kg while buying or selling. Find the
(a) 3 (b) 4 (c) 5 (d) 6 percentage profit earned by the shopkeeper.
32. It cost ` 1 to photocopy a sheet of paper. However, 2% (a) 20 (b) 12 (c) 42 (d) 46.6
discount is allowed on all photocopies done after first 43. Mukul bought 80 kg of rice for ` 1200 and sold it at a
1000 sheets. How much will it cost to photocopy 5000 loss of as much money as he received for 20 kg rice. At
sheets of paper? what price per kg did he sell the rice?
(a) ` 3920 (b) ` 3980 (c) ` 4900 (d) ` 4920 (a) ` 12 per kg (b) ` 10 per kg
(c) ` 8 per kg (d) ` 11 per kg
33. When the selling price of an article is ` 280, the loss
percentage is 20%. What is the loss or gain percentage, 44. A furniture shop allows 20% discount on the marked
if the selling price is increased to ` 380? price of each item what price must be marked on a
4 5 table costing ` 560, so as to make a profit of 25%?
(a) 8 %, profit (b) 4 % , loss
7 7 (a) ` 800 (b) ` 825
50 9 (c) ` 700 (d) ` 875
(c) %, profit (d) 4 % , profit
3 11 45. A shopkeeper allows a discount of 12.5% on the marked
34. A man buys 25 chairs for ` 375 and sells them at a price of a certain article and makes a profit of 20%. If
prof it equal to the selling price of 5 chairs. What is the the article costs the shopkeeper ` 210, what price must
selling price of one chair? be marked on the article ?
(a) ` 18.75 (b) ` 14.50 (a) ` 280 (b) ` 288
(c) ` 15.20 (d) ` 17.20 (c) ` 300 (d) None of these

https://sscstudy.com/
https://sscstudy.com/

46 CUET (UG) Section III : General Test

ANSWERS
1. (b) 2. (b) 3. (c) 4. (c) 5. (a) 6. (d) 7. (a) 8. (b) 9. (c) 10. (b)
11. (c) 12. (a) 13. (b) 14. (d) 15. (b) 16. (c) 17. (a) 18. (c) 19. (d) 20. (b)
21. (c) 22. (d) 23. (d) 24. (d) 25. (c) 26. (c) 27. (d) 28. (a) 29. (b) 30. (b)
31. (c) 32. (d) 33. (a) 34. (a) 35. (c) 36. (a) 37. (a) 38. (a) 39. (a) 40. (a)
41. (b) 42. (d) 43. (a) 44. (d) 45. (b)

Hints & Solutions


1. Required SP = (100 − 5)% of 90 7 10 12. Required percentage gain
= %− %
2 3 x− y
= 95% of 90 = × 100
1 y
= ` 85.50 = %
6 50 − 40
2. Selling price of the TV ∴ Required difference = × 100
40
= 50% of 50% of 5000 1
= % of 12102 = ` 20.17 10
= ` 1250 6 = × 100 = 25%
40
3. Total cost price of eggs including 8. Cost price of 96 apples (8 dozen) 13. Cost price of 1 peice of article
transportation cost = 10 × 18 + 24 = `300
= ` 0. 8
= 180 + 24 [Q 4 apples were rotten.] Selling price of 1 peice of article
= ` 204 Selling price of 96 apples 10
To get 10% profit, their SP (8 dozen) = 8 × 50 = `400 = = ` 1.25
8
= 110% of 204 = ` 224.40 ∴Required gain = 400 − 300
Profit while selling on a peice of
But this is the SP of 110 eggs as = ` 100 article = 1. 25 − 0. 8 = `0.45
10 eggs were broken.
9. Profit on selling the scooter ∴ Required gain percentage
Hence, sale price per egg
= 22000 − 20000 0.45 × 100
224.40 =
= = ` 2.04 = ` 2000 0.8
110
∴ Required profit percentage = 56.25 %
4. Sale price of the article 2000 × 100
= (100 + 20)% of 170 = ` 204 = = 10% 14. Here, x = 10, y = 10
20000
Now, (100 − 15)% of MP = SP ∴ Resultant gain or loss
10. Let the MRP of refrigerator be
100  xy 
⇒ MP = 204 × = ` 240 ` x , then = x − y − %
85  100
(100 − 20)% of x = 90% of 28000
5. Rate of discount  10 × 10
⇒ 80% of x = 90% of 28000 = 10 − 10 − %
250 − 235  100 
= × 100% 80 90
250 ⇒ ×x= × 28000 = − 1%
100 100
15 Thus, seller losses 1%.
= × 100% = 6% 28000 × 90
250 ⇒ x=
80 15. Let CP of the article be ` x, then
6. Let the rate of the second ∴ x = `31500 106% of x − 94% of x = 6
discount be r %.
11. Equivalent discount of 10% and ⇒ 12% of x = 6
Then, (100 − 10)% of (100 = r )%
5% 6 × 100
of 160 = 122.40 ∴ x=
10 × 5
90 100 − r = 10 + 5 − 12
⇒ × × 160 = 122.40 100
100 100 = ` 50
= 15 − 0. 5
122.40 × 100 × 100 16. Here, x = 20 and y = 22, then
⇒ 100 − r = = 14. 5%
90 × 160 y−x
∴ Required selling price ∴Per cent gain = × 100%
= 85 x
= (100 − 14.5)% of 26580
⇒ r = 15% 22 − 20
26580 × 85.5 = × 100%
7. Difference is discount = 20
100
1 1 = 10%
= 3 % −3 % = ` 22725.90
2 3

https://sscstudy.com/
https://sscstudy.com/

NUMERICAL ABILITY 47

17. Let CP of the article be ` x. 23. Let the amount of the bill be ` x . Q When ` 4 get more, cost price
of the book = ` 100
Then, SP = 110% of x = ` 1.1x Equivalent discount of two Q When ` 6 get more, cost price
If SP be double i.e., 2.2 x, then successive discounts of 20% 100
2. 2x − x of the book = × 6 = ` 150]
Profit per cent = × 100% 20 × 20  4

x = 20 + 20 − % 27. Q Cost price of 2 toffees = ` 1
= 120%  100 
1
= 36% ∴ Cost price of 1 toffee = `
18. CP = ` 300 2
SP = 120% of 300 According to question, Q Selling price of 5 toffees = ` 1
= ` 360 36% of x − 35% of x = ` 22 1
∴Selling price of 1 toffee = `
x × 36 x × 35 x 5
∴ MP = 360 ×
100 − = 22 = 22 1 1
90 100 100 100 −
= ` 400 ⇒ x = ` 2200 ∴ Loss per cent = 2 5 × 100
1
100
19. After discounts, the sale price of 24. CP of 1 book = ` 2
the book = 80% of 70% of 60 11
SP of 1 book = `
110
= ` 11 (5 − 2) × 2
= ` 33.60 = × 100
10 10 × 1
But there is sale tax of 5%,
∴Percentage profit 3 ×2
therefore required SP = 105% of = × 100 = 60%
33.60 = ` 35.28 SP − CP 10
= × 100
CP 28. Let cost price of article = ` x
20. Let the CP of articles = ` x
100
By given condition, 11 − Profit = 1920 − x
3 124 x = 11 × 100 1280 + x = Loss
SP of articles = x × + 100 / 11
4 100 4 Q Profit = Loss
= 21%
=`
472
x ∴ 1920 − x = 1280 + x
400 25. Let CP of cloth be ` 100, then SP 3200 = 2x
of half of his cloth at 40% profit x = ` 1600
∴ Gain percentage
140 125
472 = 50 × = ` 70 Profit on 25 % = 1600 ×
x−x 100 100
= 400 × 100 1 = ` 2000
x SP of th of his cloth at 40% loss
72 4 29. Cost price of 5 lemons = ` 16
= × 100 = 18% 60 16
400 = 25 × = ` 15 Cost price of 1 lemon = `
100 5
21. Marked price = ` 1600 1
SP of remain th cloth = ` 25 Selling price of 1 lemon on 25%
Equivalent discount 4 16 125 16 5
10 × x  profit = × = × =`4
 ∴Total SP = ` (70 + 15 + 25) 5 100 5 4
= 10 + x −
 100 
= ` 110 30. Total cost price of scooter
 9x 
∴ Required profit = 4700 + 800
= 10 + %
 10  110 − 100 = ` 5500
= × 100 = 10%
and selling price = ` 1224 100 Selling price of scooter = ` 5800
Q Percentage discount 26. Suppose, cost of book = ` 100 Profit = 5800 − 5500 = ` 300
1600 − 1224  ∴ First selling price of the book Profit × 100
= × 100 % Profit per cent =
 1600  = 100 + 10 = ` 110 Cost price
 9x   376  ∴ Reduced cost price of the book 300 × 100 60
⇒ 10 + %= × 100 % 96 = =
 10  1600  = 100 × = ` 96 5500 11
100 5

9x
= 23 . 5 − 10 ∴ Second selling price of the =5 %
100 11
book
13 . 5 × 10 1
x= = 15% 96 ×  75 31. Cost price of 1 toffee = `
= 100 +  6
9 100  4
Selling price of 1 toffee on 20%
22. Required profit per cent 475
= 96 × = ` 114 1 120 1 6 1
Profit = × = × =
100 − 95  4 × 100
= × 100 % 6 100 6 5 5
 95 
Q Difference of selling price Hence, vender selling 5 toffees
= 5 . 26% = ` 114 − ` 110 = ` 4 in a rupee.

https://sscstudy.com/
https://sscstudy.com/

48 CUET (UG) Section III : General Test

32. Cost of first 1000 sheets = ` 1000 m−n  100 − 5  100 − 93
= × 100 ∴x×    = 266
n  100   100 
Remaining sheets
36 − 30 19 7
= 5000 − 1000 = × 100 ⇒ x× × = 266
30 20 100
= 4000
= 20% ⇒ x = 4000
Given, 2% discount on
remaining sheets 37. CP of 60 eggs = ` 10.80 (as 12 eggs 42. The dealer uses a false weight in
98 were found rotten) buying as well as selling.
= 4000 × = 40 × 98 SP of 60 eggs = ` 12.00 gain It means that he buys 1100 g
100
= ` 1.20 and sells 900 g in place of 1 kg.
= ` 3920
∴ Gain per cent Now, let the CP of 1100 g be
Total cost = 1000 + 3920 1.2 100
= × 100 = % ` 1000.
= `4920 10.8 9 Marked price of 1100 g of goods
33. The cost price of an article at a 1
= 11 % = ` 1200 (20% up), and this is
280 × 100 9
loss of 20% = = ` 350 the SP of 900 g of goods.
80 38. Let the selling price = ` 100. CP of 900 g = ` 818.18
New SP of an article = ` 380 If profit is ` 25 on SP, the cost
price = ` 75 Profit = ` 381.82
∴ Profit = 380 − 350 = ` 30
Actual profit per cent and profit per cent
Hence, profit per cent
30 Profit 381.82
= × 100 = × 100 = × 100 = 46.6%
350 CP 818.18
4 25 43. SP of 80 kg of rice = CP of 80 kg
=8 % = × 100
7 75 − loss
100 SP of 80 kg rice = 1200 − SP of
34. SP of 25 chairs = CP of 25 chairs = %
+ Profit 3 20 kg of rice
SP of 25 chairs 39. Gain per cent = (100 + % first SP of 100 kg of rice = ` 1200
= CP of 25 chairs + SP of 5 chairs profit) ∴ SP of 1 kg of rice = ` 12
(Q Profit = SP of 5 chairs) 100 − % 2nd discount  44. CP of the table = ` 560
− 100
⇒ SP of 20 chairs = CP of 25  100 − % 1st discount  SP of the table at a profit of 25%
chairs  100 − 20 = ` 700
= (100 + 25)   − 100
⇒ SP of 20 chairs = ` 375  100 − 10 Since SP is arrived after a
⇒ SP of one chair = ` 18.75 80 discount of 20% on marked price
= 125 × − 100
35. Difference of SP = Difference of 90 700
100 1 ∴ Marked price = = ` 875
loss/profit = = 11 % 0.8
9 9
⇒ 400 − 380 = [(x + 4) − (x)]% 45. Cost price of the article = ` 210
40. The CP will be ` 40, if the SP is
20 = 4% Selling price at a profit of 20%
` 100.
20 = ` 252
∴ 100% = × 100 Percentage of SP to CP
4
100 ∴ MP × 0.875 = ` 252
= ` 500 = × 100 = 250% 252
40 ⇒ MP = ` = ` 288
36. If the CP of m book is equal to 0.875
41. Let the shopkeeper has x eggs.
SP of n books, then gain
percentage

https://sscstudy.com/
https://sscstudy.com/

NUMERICAL ABILITY 49

CHAPTER 10

Simple and
Compound Interest
Interest is the money paid by the borrower to the lender for the use of money but the interest calculated at principal
borrowed known as simple interest and the interest calculated after addition of principal and interest known as
compound interest.

Simple Interest When Interest added half yearly and time, rate is given
Interest is the amount paid by the borrower to the in yearly, then time becomes double and rate becomes
lender for using his money. half.
When the interest is calculated uniformly on the amount When time and rate is given in yearly and interest added
throghout the loan period, then that is simple interest. quarterly, then rate becomes 1/4 of yearly rate and times
The amount on which simple interest is calculated is becomes four times of yearly.
known as principal. The following table will illustrate the conceptual working
PTR of simple interest and compound interest. Rate of
SI = interest per annum is 10%
100
Where, SI → Simple Interest For the Simple Interest Compound Interest
T → Time year
Principal SI Principal CI
P → Principal on which interest is calculated
R → Rate 1 1000 100 1000 100

Amount ( A) = P + SI 2 1000 100 1000 + 100 = 1100 110


3 1000 100 1100 + 110 = 1210 121
Compound Interest On the basis of above calculation
It is the interest calculated on a sum of money which
It is clear that
includes principal and interest calculated for previous
year. The SI and CI for first year is same and for second (i) Simple interest for each year is constant.
and subsequent years differ by an amount which is (ii) Compound interest calculated for each year
arrived by calculating interest on interest for previous includes—simple interest on principal and simple
years. interest on interest calculated for previous year.

Important Tips/Formulae
Let principal = ` P, Rate = R % per annum, Time = n yr (ii) When interest is annually and rate of interest be R1 % during
(i) When interest is compounded annually, then 1st year, R2 % during 2nd year, R3 % during 3rd year.

Amount = P  1 + 1   1 + 2   1 + 3 
n R R R
Amount = P  1 +
R 
  100   100   100 
 100 
P × r2 (iii) When interest is half-yearly Amount
Difference between CI and SI for 2 yr = 2n
= P ×  1 +
2
( 100 ) R 

 2 × 100 
Difference between CI and SI for 3 yr
P × r2 ( 300 + r ) (iv) When interest is quarterly.
= 4n
Amount = P ×  1 +
3 R 
( 100 ) 
 4 × 100 

https://sscstudy.com/
https://sscstudy.com/

50 CUET (UG) Section III : General Test

Solved Examples
1. A sum doubles in 20 yr at simple interest. How 3. The simple interest on a certain sum of money for
1
much is the rate? 2 1 yr at 8 % per annum is ` 560. Then, find the sum.
(a) 5% per annum (b) 8% per annum 3 3
(c) 5.5% per annum (d) 8.5% per annum (a) ` 2880 (b) ` 2800 (c) ` 8880 (d) ` 8280
Sol. (a) Let sum = P Sol. (a) Let the sum be ` x. Then,
Then, amount = 2P, SI = 2P − P = P 25 1 7 7
x× × × = 560 ⇒ x × = 560
SI × 100 3 100 3 36
∴ R= 560 × 36
P ×T ⇒ x= = 2880 ⇒ x = ` 2880
7
P × 100
= 4. Find the compound interest on ` 10000 in 2 yr at
P × 20
4% per annum (p.a.), the interest being
R = 5% per annum compounded half-yearly.
2. Find simple interest on ` 4800 at 8% per annum (a) ` 836 (b) ` 824.32 (c) ` 324 (d) ` 868
for 10 months. Sol. (b) Principal = ` 10000, Rate = 2% per half-year
(a) ` 400 (b) ` 420 Time = 2 yr = 4 half-year
(c) ` 320 (d) ` 510  2  
4
∴ Amount = 10000 ×  1 +  
Sol. (c) P = ` 4800, R = 8% per annum,   100  
10 5 51 51 51 51
T = yr = yr = 10000 × × × ×
12 6 50 50 50 50
P × R × T 4800 × 8 × 5 = ` 10824.32
∴ SI = = = ` 320
100 6 × 100 ∴ CI = ` 10824.32 − 10000 = ` 824.32

Practice Questions
1. If a sum of money at simple interest doubles in 6. A sum of ` 8448 is to be divided between A and B
5 yr, it will become 4 times in how many years? who are respectively 18 and 19 yr old, in such a
(a) 18 (b) 16 way that if their shares be invested at 6.25% per
(c) 15 (d) 20 annum compound interest, they will receive equal
2. A certain sum of money becomes ` 2250 at the end amounts on attaining the age of 21 yr. The present
of 2 yr and becomes ` 2625 at the end of 5 yr. If the share of A is
person receives only simple interest, then the rate (a) ` 4225 (b) ` 4352
of interest is (c) ` 4096 (d) ` 4000
(a) 6.25% (b) 5% 7. On a certain sum of money, the difference between
(c) 8% (d) 6.5% the compound interest for a year, payable
3. A sum was put at simple interest at a certain rate half-yearly, and the simple interest for a year is
for 4 yr. Had it been put at 3% higher rate it would ` 180. If the rate of interest in both the cases is
have fetched ` 1440 more. Find the sum. 10%, then the sum is
(a) ` 14400 (b) ` 11000 (a) ` 60000 (b) ` 72000
(c) ` 10000 (d) ` 12000 (c) ` 62000 (d) ` 54000
4. If the compound interest on a certain sum for 2 yr 8. In a certain time, the ratio of a certain principal
at 4% is ` 102, what would be the simple interest and the simple interest obtained from it are in the
at the same rate for 2 yr? ratio 10 : 3 at 10% interest per annum. The
(a) ` 75 (b) ` 100.50 number of years the money was invested is
(c) ` 100 (d) ` 98 (a) 1 (b) 3
(c) 5 (d) 7
5. A sum of money amounts to ` 2240 at 4% per
annum simple interest in 3 yr. The interest on the 9. Find the simple interest due after 120 days for
same sum for 6 months at 3.5% per annum is ` 4800 at 10%.
(a) ` 30 (b) ` 50 (a) ` 157.80 (b) ` 157
(c) ` 35 (d) ` 150 (c) ` 156.01 (d) ` 124.93

https://sscstudy.com/
https://sscstudy.com/

NUMERICAL ABILITY 51

10. Find the rate of interest at which ` 100 becomes 18. At what rate per cent compound interest, will ` 400
` 200 in 10 yr. amount to ` 441 in 2 yr?
(a) 10% (b) 15% (a) 4% (b) 5%
(c) 8% (d) 17% (c) 6% (d) 3%

11. Rina borrows a loan of ` 1200 at simple interest 19. Find the simple interest on ` 1600 at 6% per
and the number of years is equal to the rate per annum for 146 days.
cent per annum and she give ` 432 as interest. (a) ` 42.50 (b) ` 30.60
(c) ` 38.40 (d) ` 40
Find the rate of interest.
(a) 3.6% 20. Arun invests ` 10250 at 4% per annum simple
(b) 6% interest to obtain a total amount of ` 12710. For
(c) 18% how many years did he invest the sum?
(d) Cannot be determined (a) 6 (b) 8
12. A man took loan from a bank at the rate of 12% (c) 5 (d) 4
par annum simple interest. After 3yr he had to pay 21. The simple interest on ` 2000 for 7 months at
` 5400 interest only for the period. The principal 5 paise per rupee per month is
amount borrowed by him was (a) ` 700 (b) ` 70
(a) ` 2000 (b) ` 10000 (c) ` 350 (d) ` 305
(c) ` 15000 (d) ` 20000 22. A certain sum is invested at simple interest. If it
13. A sum of money at simple interest amount to ` 815 trebles in 10 yr, what is the rate of interest?
in 3 yr and to ` 854 in 4 yr. The sum is (a) 18% per annum (b) 20% per annum
(c) 22% per annum (d) 25% per annum
(a) ` 650 (b) ` 690
(c) ` 698 (d) ` 700 23. A certain sum of money at simple interest amounts
14. The rate of interest is 6% per annum in a bank. If to ` 4320 in 2 1 yr and to ` 4752 in 4 yr. The rate of
2
an interest of ` 1350 is to be earned in 5 yr, then interest per annum is
the amount of money deposited is 1 1
(a) 8% (b) 8 % (c) 7 % (d) 9%
(a) ` 13500 (b) ` 4500 2 2
(c) ` 5400 (d) ` 4800
24. A lent be ` 5000 to B for 2 yr and ` 3000 to C for
15. A man borrows ` 50000 at 4% compound interest 4 yr on simple interest at the same rate of interest
per annum. Then, the total amount of money he and received ` 2200 in all from both as interest.
has to pay after 2 yr is The rate of interest per annum is
(a) ` 54080 (b) ` 50480 (a) 8% (b) 9%
(c) ` 54000 (d) ` 54800 (c) 10% (d) 12%
16. Sudha borrowed ` 400 from her friend at the rate 25. The simple interest on ` 7380 from 11 May, 1987 to
of 12% per annum for 2 1 yr. The interest and the 11 September, 1987 at 5% per annum.
2 (a) ` 123 (b) ` 103
amount paid by her were (c) ` 200 (d) ` 223
(a) ` 140, ` 540 (b) ` 130, ` 530 1 1
26. A man invested of his capital at 8%, at 9% and
(c) ` 125, ` 525 (d) ` 120, ` 520 3 4
17. The compound interest on a sum at the rate of 5% the remainder at 10%. If his annual income is
for 2 yr is ` 512.50. The sum is ` 1180, the whole capital is
(a) ` 5200 (b) ` 4800 (a) ` 9000 (b) ` 12000
(c) ` 5000 (d) ` 5500 (c) ` 15000 (d) ` 13500

ANSWERS
1. (c) 2. (a) 3. (d) 4. (c) 5. (c) 6. (b) 7. (b) 8. (b) 9. (a) 10. (a)
11. (b) 12. (c) 13. (c) 14. (b) 15. (a) 16. (d) 17. (c) 18. (b) 19. (c) 20. (a)
21. (a) 22. (b) 23. (a) 24. (c) 25. (a) 26. (b)

https://sscstudy.com/
https://sscstudy.com/

52 CUET (UG) Section III : General Test

Hints & Solutions


1. Here, n1 = 2, t1 = 5, n2 = 4, t2 = ? x 6. 25 12. Let amount of loan = ` x
⇒ =1 +
t n −1 5 1 8448 − x 100
Using, 1 = 1 ⇒ = Simple interest
t2 n2 − 1 t2 3 x 1 17 Amount × Rate × Time
⇒ =1 + = =
8448 − x 16 16 100
⇒ t2 = 15 yr
⇒ 16x = 17 (8448 − x) x × 12 × 3
3. Let the principal be ` P and 5400 =
rate of interest r % per annum. ⇒ 16x = 143616 − 17x 100
Then, according to question, ⇒ 33x = 143616 5400 × 100
x=
P × (r + 3) × 4 P × r × 4 ∴ x = ` 4352 36
− = 1440
100 100 7. Let the sum be ` P. x = ` 15000
⇒ 4P + 12P − 4 Pr = 1440 × 100 Compound interest 13. Let amount = ` x
⇒ 12 P = 1440 × 100  5 
2  41 Let interest rate = r %
⇒ P = 120 × 100 = P 1 +  − 1 = ×P
 100  Simple interest in 3 yr,
  400 x × r ×3
⇒ P = ` 12000 ⇒ 815 − x = ...(i)
and simple interest 100
4. Let the principal be ` P. P × 10 × 1 1
According to question, = = ×P Simple interest in 4 yr
2
100 10 x × r ×4
 4  Given, CI − SI = ` 180 ⇒ 854 − x = ...(ii)
102 + P = P 1 +  100
 100 41 1
2
×P− × P = 180 On dividing Eq. (i) by Eq. (ii),
 26 100 10 815 − x 3
⇒ 102 + P = P   ⇒ =
 25 P = 180 × 400 854 − x 4
P × 676 = ` 72000 ⇒ 3260 − 4x = 2562 − 3x
⇒ 102 + P =
625 8. Let time = t yr ∴ x = ` 698
676P − 625P ∴ P = 10x and SI = 3x
⇒ 102 = 14. Given, Rate = 6% per annum,
625 Prt Time = 5 yr
Q SI =
⇒ 51P = 108 × 625 100 Interest = ` 1350
P = `1250 10x × 10 × t Let amount = ` x
3x =
Required single interest 100 Then, Simple Interest
1250 × 4 × 2 t = 3 yr Amount × Rate × Time
= = `100 =
100 9. Given, principal = ` 4800 100
5. If the sum be ` P, then x×6 ×5
Rate = 10%, Time = 120 days ⇒ 1350 =
P ×4 ×3 P × R×T 100
2240 − P = Simple interest =
100 1350 × 100
100 ⇒ =x
12P 4800 × 10 × 120 30
⇒ 2240 = +P =
100 100 × 365 ⇒ x = 45 × 100
112 = ` 4500
⇒ 2240 = P = ` 157. 8
100 15. Given, Rate = 4%
2240 × 100 10. Principal = ` 100, Time = 10 yr
∴ P= = ` 2000 Amount = ` 50000
112 Amount = ` 200,
Simple interest = 200 − 100 Time = 2 yr
Now, required interest,
Prt 7 1 1 = ` 100 Then, the total amount of money
SI = = 2000 × × × 2
100 2 2 100 SI × 100 100 × 100  4 
Rate = = = 10% = 50000 1 + 
= ` 35 P ×T 100 × 10  100
6. Let shares of A and B are ` x 11. Given, time = rate of interest = r 26 26
= 50000 × ×
and ` (8448 − x), respectively. 1200 × r × r 25 25
Then, 432 =
Amount got by A after 3 yr 100 = 80 × 676
= Amount got by B after 2 yr 2 432 × 100 = ` 54080
3 ⇒ r = 400 × 5 × 12
 6. 25 1200 16. SI = = ` 120
x 1 +  2 × 100
 100  ⇒ r 2 = 36
4
⇒ r = 36 A = P + SI
 6. 25
= (8448 − x) 1 +  = (400 + 120)
 100  ∴ r = 6%
= ` 520

https://sscstudy.com/
https://sscstudy.com/

NUMERICAL ABILITY 53

 R 
2  =
192 ⇒ 100 r + 120 r = 2200
17. CI = P 1 +  − 1
 100 5 ⇒ 220 r = 2200
 
= ` 38.40 2200
 2  ⇒ r=
5  20. P = ` 10250, R = 4% per annum, 220
⇒ 12.50 = P 1 +  − 1
 100 = 10%
  SI = ` (12710 − 10250) = ` 2460
512 .50 × 400 100 × SI 7380 × 5 × 1
∴ P= Now, T = 25. SI = = ` 123
41 P×R 100 × 3
= ` 5000 100 × 2460  4 1 
= T = 4 months = = yr
18. P = ` 400, A = ` 441, n = 2 yr 10250 × 4  12 3 
R=? = 6 yr
n ∴ T = 6 yr 26. Let the capital be ` x.
 R 
A = ? 1 +  21. SI on ` 1 for 1 yr = (5 × 12) paise
x
 100 Then, Ist capital = `
3
2 = 60 paise
 R  x
⇒ 441 = 400 1 +  SI on ` 100 for 1 yr IInd capital = `
 100 4
 60 
2 = × 100
441  R   100    x x 
⇒ = 1 +  IIIrd capital = ` x −  +  
400  100 = ` 60   3 4 
2 2 Now, P = ` 2000, 5x
 21  R  =`
⇒   = 1 +  R = 60 %
 20  100 12
7
21  R  and T = yr x 8  x 9 
⇒ = 1 +  12 ∴ × × 1 +  × × 1
20  100  3 100   3 100 
 2000 × 60 × 7
R 21 ∴ SI =    5x 10 
⇒ = −1  100 × 12  + × × 1 = 1180
100 20  12 100 
R 1 = ` 700
⇒ = 2x 3x x
100 20 22. Let the sum be ` x. Then, ⇒ + + = 1180
SI = ` (3x − x) = ` 2x 75 100 24
⇒ R = 5%
x × R × 10
19. Here, P = ` 1600, R = 6 % per ∴ 2x = ⇒ 16x + 18x + 25x = 600 × 1180
100
146 ⇒ 59x = 600 × 1180
annum and T = yr ∴ R = 20 % per annum
365
P × R ×T 24. Suppose rate of interest per 600 × 1180
∴ SI = ⇒ x=
100 annum = r% 59
 6 146 According to the question, = 12000
= 1600 × ×  5000 × 2 × r 3000 × 4 × r
 100 365 + = 2200
100 100 ∴ Capital = ` 12000

https://sscstudy.com/
https://sscstudy.com/

54 CUET (UG) Section III : General Test

CHAPTER 11

Mixture and Alligation


When two or more than two pure substances are mixed in a certain ratio, they create a mixture.
Here, we shall confine ourselves to mostly homogeneous mixtures in view of the question commonly asked.

Mixture e.g. A mixture of a certain quantity of milk with 16


The new product obtained by mixing two or more L of water is worth ` 0.75 per litre.
ingredients in a certain ratio is called a mixture of those If pure milk be worth ` 2.25 per litre, how much
particular ingredients.
milk is there in the mixture?
Mean Price Sol. According to the rule of alligation,
The cost price of a unit quantity of the mixture is called
Water (0) Milk (2.25)
the mean price.

Rules of Mixture or Alligation Mean price 0.75


This rule of mixture is used to determine the mean price
of the mixture, when the prices of the individual items
2.25 – 0.75 = 1.5 0.75 – 0 = 0.75
being mixed together and the proportion in which they
are being mixed are given. ∴ Water : Milk = 1.5 : 0.75 = 2 : 1
When two ingredients at given prices are known, then 1
Clearly, quantity of milk = of water
the ratio in which these two are mixed to obtain a 2
mixture of known price is given by 1
= × 16
Amount of cheaper Cost price of dearer – Mean price 2
= = 8L
Amount of dearer Mean price – Cost price of cheaper
It is also known as rule of alligation. It can also be
expressed, as Keep in Mind!
The value of the mixture is always higher than the lowest
CP of cheaper CP of dearer value and lower than the highest value of the items being
(per unit) (C) (per unit) (D) mixed.
Mean price
Rule of mixture or alligation is applied for rate, ratio, percentage
(M)
(D-M) (M -C) value, speed, prices etc. and not for absolute values.
In other words, whenever per cent, per hour, per km, per kg
Hence, Amount of cheaper : Amount of dearer etc. are being compared, we can use this method.
= (D − M ) : (M − C)

https://sscstudy.com/
https://sscstudy.com/

NUMERICAL ABILITY 55

Solved Examples
1. Tea worth ` 126 per kg and ` 135 per kg are mixed Ratio in which they are mixed
with a third variety in the ratio 1 : 1 : 2 . If the 8 5
mixture is worth ` 153 per kg. The price of the 13 9 7
third variety per kg will be 65 – 63 13 9 – 8
(a) ` 169.5 (b) ` 170.0 91 13 13
(c) ` 175.5 (d) ` 180.0 2
Sol. (c) Let the common ratio be x. 2 1 91 2 13 2
= : = = × = = 2: 7
91 13 1 91 1 7
Price of 3rd variety of tea = ` p
13
Now, by given condition,
126 × x + 135 × x + p × 2x = 153 (x + x + 2x) 3. A mixture of milk and water is such that the
⇒ 126x + 135x + p × 2x = 153 × 4x quantity of milk is 3 that of water. The proportion
⇒ 261x + p × 2x = 612x 5
⇒ p × 2x = 612x − 261x of milk in the mixture is
351x
∴ p= (a) 1/8 (b) 1/2 (c) 3/8 (d) 5/8
2x 3
⇒ p = ` 175.50 Sol. (d) Milk = water ⇒ 5 milk = 3 water
5
2. Two vessels A and B contain milk and water mixed Total mixture = 5 + 3 = 8 units
in the ratio 8 : 5 and 5 : 2 , respectively. The ratio in 5
Portion of milk in mixture =
which these two mixtures be mixed to get a new 8
3
mixture containing 69 % milk is 4. Two solutions of 90% and 97% purity are mixed,
13
resulting in 21 L of mixture of 94% purity. The
(a) 2 : 7 (b) 3 : 5
(c) 5 : 2 (d) 5 : 7 quantity of the second solution in the resulting
8 mixture, in litres, is
Sol. (a) Total ratio of milk in vessel A = (a) 15 (b) 12 (c) 9 (d) 6
13
5 Sol. (b) By mixture method,
Total ratio of milk in vessel B =
7 90% 97%
3 94%
69
Ratio of milk in new mixture = 13 3% 4%
100 ∴Quantity of second solution in 21L of mixture
900 9
= = 4
= × 21 = 12 L
13 × 100 13 7

Practice Questions
1. A mixture of 20 kg of spirit and water contains 3. By mixing two different quantities of pulses in the
10% water. After adding a certain amount of water, ratio 2 : 3 and selling the mixture at the rate of
the weight of the new mixture is 25 kg. What is the ` 22 per kg a shopkeeper makes a profit of 10%.
percentage of water in the new mixture? If the cost of the smaller quantity be ` 14 per kg,
(a) 18 (b) 28 then the cost per kg of the larger quantity is
(c) 12.5 (d) 15 (a) ` 23
2. 60 kg of a certain variety of rice at ` 32 per kg is (b) ` 24
(c) ` 25
mixed with 48 kg of another variety of rice and (d) None of the above
the mixture is sold at the average price of ` 28
4. A drum contain 20 L of a paint. From this 2 L of
per kg. If there be no profit or loss due to the new
paint is taken out and replaced by 2 L of oil. Again
sale price, then the price of the second variety of
2 L of this mixture is replaced by 2 L of oil. If the
rice is
operation is performed once again, then final ratio
(a) ` 25.60 per kg (b) ` 25 per kg
(c) ` 23 per kg (d) ` 30 per kg of the paint and the oil in the drum would be
(a) 729 : 271 (b) 172 : 279
(c) 3 : 7 (d) 217 : 972

https://sscstudy.com/
https://sscstudy.com/

56 CUET (UG) Section III : General Test

5. Several litres of acid were drawn off a 54 L vessel 10. Atul bought 30 kg of rice at ` 8.50 per kg and 20 kg
full of acid and an equal amount of water added. of rice at ` 8.00 per kg. If he has to make a 20%
Again the same volume of the mixture was drawn profit, at approximately, what rate per kg should he
off and replaced by water. As a result, the vessel sell the rice?
contained 24 L of pure acid. How much of the acid (a) ` 10.00 (b) ` 12.00
was draw off initially? (c) ` 8.50 (d) ` 8.00
(a) 12 L (b) 16 L
11. Raghav buys milk at a certain price and after
(c) 18 L (d) 24 L
mixing it with water sells it again at the same
6. There are three containers of equal capacity. The price. How many of water he mixes in a litres of
ratio of sulphuric acid to water in the first milk, if he makes a profit of 20%?
container is 3 : 2, that in the second container is (a) 200 mL (b) 250 mL
7 : 3 and in the third container 11: 4. If all the
(c) 150 mL (d) 20 mL
liquids are mixed together, then the ratio of
sulphuric acid to water in the mixture will be 12. Pure ghee costs ` 100 per kg. After adulterating it
(a) 61 : 29 (b) 61 : 28 with vegetable oil costing ` 50 per kg. A
(c) 60 : 29 (d) 59 : 29 shopkeeper sells the mixture at ` 96 per kg,
7. A container is filled with liquid, 6 part of which thereby making a profit of 20%. In what ratio does
are water and 10 part milk. How much of the he mix the two?
mixture must be drawn off and replaced with (a) 1 : 2 (b) 3 : 2
water so that the mixture may be half water and (c) 3 : 1 (d) 2 : 3
half milk? 13. Two types of oils having the rates of ` 4 per kg and
1 1 1 1
(a) (b) (c) (d) ` 5 per kg respectively are mixed in order to
3 7 5 8
produce a mixture having the rate of ` 4.60 per kg.
8. A man buys milk at a certain price per litre and What should be the amount of the second type of
after mixing it with water sells it again at the oil if the amount of the first type of oil in the
same price. How many of water does he mix in mixture is 40 kg?
every litre of milk if he makes a profit of 25%? (a) 75 kg (b) 50 kg
(a) 250 mL (b) 200 mL (c) 60 kg (d) 40 kg
(c) 150 mL (d) 30 mL
14. 400 students took a mock exam in Delhi 60% of the
9. A barrel contains a mixture of wine and water in
boys and 80% of the girls cleared the cut off in the
the ratio 3 : 1. How much fraction of the mixture
must be drawn off and substituted by water so that examination. If the total percentage of students
the ratio of wine and water in the resultant qualifying is 65%, how many girls appeared in the
mixture in the barrel become 1 : 1? examination?
1 1 1 2 (a) 100 (b) 120
(a) (b) (c) (d)
4 3 2 3 (c) 150 (d) 300

ANSWERS
1. (b) 2. (c) 3. (b) 4. (a) 5. (c) 6. (a) 7. (c) 8. (a) 9. (a) 10. (a)
11. (a) 12. (b) 13. (c) 14. (a)

Hints & Solutions


1. Quantity of water in mixture of 20 kg = 10% of 20 kg 2. Let price of the second variety of rice be ` x per kg.
= 2 kg Then, total cost of first variety of rice
Weight of new mixture = 25 kg = 60 × 32 = ` 1920
Quantity of water in new mixture = 25 − (20 − 2) and total cost of second variety of rice = 48 × x
= 25 − 18 = 7 kg = ` 48x
7 × 100 ∴Total SP of both varities of rice
∴ Required percentage =
25 = (60 + 48) × 28 = 108 × 28
= 7 × 4 = 28% = ` 3024
Since, there is neither profit nor loss, therefore,

https://sscstudy.com/
https://sscstudy.com/

NUMERICAL ABILITY 57

1920 + 48x = 3024 (54 − x) 41. 50


∴ 54 − x − x = 24 ∴ x= = ` 8. 30
⇒ 48x = 1104 54 5
∴ x = 23 ⇒ x2 − 108x + 1620 = 0 ∴ SP of mix per kg at 20% profit
Thus, price of second variety of ⇒ x = 90, 18 8. 30 × 120
= = 9. 96
rice is ` 23 per kg. ∴ 90 > 54 100
So, x = 90 is ruled out = ` 10.00 (approx.)
3. Let the total quantity of pulses
= 5 kg Hence, x = 18 11. Let the price of milk per litre be
SP of the mixture= ` 22 × 5 = ` 110 6. Amount of Sulphuric acid in the ` 1.
Profit = 10% 3 7 11 61 ∴ SP of adulterated milk per
Q new mixture = + + = litres = ` 1
∴ CP of the mixture = ` 100 5 10 15 30
∴ CP of adulterated milk per litres
Q Cost of the first quantity and amount of water in the new 1 × 100 5
= 2 × 14 = ` 28 2 3 4 29 = =`
mixture = + + = (100 + 20) 6
Cost of the second quality = ` 72 5 10 15 30 5 1
72 1−
∴ CP of the second quality = Ratio of sulphuric acid and Quantity of water 6 =6
3 water in the new mixture Q =
Quantity of milk 5 − 0 5
= ` 24 61 29
= : = 61 : 29 6 6
4. The quantity of paint = 18 L 30 30 ∴ Quantity of water with 1 L of
and quantity of oil = 2 L 7. Let the container initially milk = 200 mL
Now, 2 L of mixture is taken out contains 16 L of liquid. 12. CP of mixture per kg
and 2 L of oil is mixed. Let ‘a’ L of liquid be replaced with
100 × 96
∴Now, the quantity of paint water. = = ` 80
Quantity of water in the new (100 + 20)
9 81
= 18 − = L  6a  Quantity of pure ghee
5 5 mixture = 6 − + a L
 16  Quantity of vegetable oil
and quantity of oil
50 − 80
1 19
=2 + 2 − = L
Quantity of milk in the new =
5 5  10a  80 − 100
mixture = 10 −  L 30
Now, this process is performed  16  ∴ Required ratio = = 3 :2
again. 6a 10a 20
∴ 6− + a = 10 −
Then, the quantity of paint 16 16 13. Given, first type of oil in the
81 81 729 mixture = 40 kg
= − = L ⇒ 96 − 6a + 16a = 160 − 10a
5 50 50 ⇒ 96 + 10a = 160 − 10a By mixture and alligation rule,
and quantity of oil ⇒ 20a = 64 Type-I Type-II
19 19 271 64 16 `4 `5
= − +2= L ⇒ a= =
5 50 50 20 5 ` 4.6
Hence, the ratio of paint and oil ∴ Part of mixture replaced
1 16 1 0.4 0.6
= 729 : 271 = × =
16 5 5 ∴ Required ratio = 2 : 3
5. Let x L of several litres of acid
8. Let 1 L of milk is bought for ` 810. Let the quantity of type-I and
were drawn off initially
type-II be 2x and 3x respectively.
∴ Remaining acid in the vessel Let x L of water is added to it, so
2x = 40
= (54 − x) L that (1 + x) L of the mixture is
sold at ` 10 per litre. x = 20
and quantity of water in the
vessel = x L ∴ Quantity of second type oil
∴ CP of (1 + x) L = ` 10
Now, x L of mixture is drawn off and SP of (1 + x) L = ` 10 (1 + x) = 3 × 20 = 60 kg
∴ Quantity of acid drawn off Profit per cent 14. Given, total students = 400
 54 − x  1 By mixture and alligation rule,
= × x L ∴ 100x = 25 ⇒ x =
 54  4 Boys Girls
and quantity of water drawn off ∴ 250 mL of water should be 60% 80%
x2 mixed in every litre of milk.
65%
= L 30 x−8
54 10. =
20 8. 50 − x 15 5
Now, the quantity of acid
  54 − x  Where, x is the CP of the mix ∴ Required ratio = 3 : 1
= 54 − x −   x L
  54   per kg.
Number of girls appeared in
⇒ 25.50 − 3x = 2x − 15 1
examination = × 400 = 100
⇒ 5x = 25. 50 + 16 4

https://sscstudy.com/
https://sscstudy.com/

58 CUET (UG) Section III : General Test

CHAPTER 12

Time and Work


In this chapter conceptual clarity of relationship between working efficiency and time is very important to understand.
Working efficiency is the work done by an individual in one day and this efficiency is inversely proportional to the
number of days to complete a work. It means that a person who takes less days to complete a work is said to be more
efficient than a person who takes more days to complete the same work.

Important Tips/Formulae
1
If A can do a piece of work in n days, then the piece of work completed by A in one day is .
n
 nm 
If A complete a piece of work in n days and B in m days, then they together can complete the work in   days.
 n + m
1 1
If a pipe A can fill a tank in x h and a pipe B can empty the full tank in y h (where y > x), then net part filled in 1 h =  −  .
x y
M1 D1 H1 M2 D2 H2
= ; Here M1 , M2 = Number of men
W1 W2
D1 , D2 = Number of days, H1 , H2 = Number of hours and W1 , W2 = Works

Solved Examples
1. If A and B together can finish a piece of work in 7 1
∴ (A + B + C )’s 1 day work = ×
20 days, B and C in 10 days and C and A in 30 2
12 days, then A, B and C jointly can finish the 7
=
same work in 60
2 60
(a) 4 days (b) 30 days Hence, A, B and C can finish the work together in days
7 7
4 7 4
(c) 8 days (d) days or 8 days.
7 60 7
1
Sol. (c) (A + B )’s 1 day work = 2. In a fort there was sufficient food for 200 soldiers
20
1
for 31 days. After 27 days 120 soldiers left the fort.
(B + C )’s 1 day work = For how many extra days will the rest of the food
10
1 last for the remaining soldiers?
(C + A )’s 1 day work = (a) 10 (b) 8
12 (c) 12 (d) 9
1 1 1
2(A + B + C )’s 1 day work = + + Sol. (a) Suppose extra days = x
20 10 12
3+ 6+ 5 Then, 200 × 31 = 200 × 27 + 80 × x
=
60 ⇒ 6200 = 5400 + 80x
14 7 800
= = ⇒ x= = 10 days
60 30 80

https://sscstudy.com/
https://sscstudy.com/

NUMERICAL ABILITY 59

Alternate Method 4. Two pipes A and B can fill a cistern in 12 min and
Required number of days 16 min, respectively. Both are opened together at
200 (31 − 27) the end of 4 min, B is turned off. In how much time
i.e., D=
200 − 120 will the cistern be full?
200 × 4 (a) 8 min (b) 10 min (c) 9 min (d) 11 min
= = 10 days
80 Sol. (c) Part filled by (A + B ) in 4 min
= 
1 1 7 7
3. If 10 men or 20 boys can make 260 mats in 20 +  ×4= × 4=
 12 16  48 12
days, then how many mats will be made by 8 men
Remaining part =  1 −  =
7 5
and 4 boys in 20 days?
 12  12
(a) 250 (b) 280
(c) 255 (d) 260 1
part is filled by A in 1 min.
12
Sol. (d) Let the number of mats be x.
part is filled by A in  12 ×  = 5 min
5 5
10 men ≡ 20 boys ⇒ (1 man ≡ 2 boys)
12  12 
⇒ (8 men + 4 boys) ≡ (8 × 2 + 4) ≡ 20 boys
In 20 days, 20 boys make 260 mats. ∴Total time taken to fill the cistern = (4 + 5) = 9 min

Practice Questions
1. 30 men can produce 1500 units in 24 days working together but B left 2 days before the completion of
6 h a day. In how many days, can 18 men produce work and C left 5 days before the completion of
1800 units working 8 h a day? work. The share of A from the assured money is
(a) 18 (b) 32 (c) 36 (d) 45 (a) ` 2700 (b) ` 540
(c) ` 1800 (d) ` 600
2. 4 men and 6 childern can complete a work in
8 days while 3 men and 7 childern can complete 7. A and B can do a piece of work in 72 days, B and C
the same work in 10 days. If 20 childern only work, can do it in 120 days, and A and C can do it in
the work will be completed in 90 days. When A, B and C work together, how
(a) 20 days (b) 22 days much work is finished by them in 3 days?
(c) 15 days (d) 18 days 1 1 1 1
(a) (b) (c) (d)
40 30 20 10
3. A can do a piece of work in 10 days and B can do the
same piece of work in 20 days. They start the work 8. A is twice as good a workman as B and together
together, but after 5 days A leaves. B will do the they finish a piece of work in 14 days. The number
remaining piece of work in of days taken by A alone to finish the work is
(a) 5 days (b) 10 days (a) 11 days (b) 21 days
(c) 6 days (d) 8 days (c) 28 days (d) 42 days
4. A and B undertake to do a piece of work for ` 720. A 9. A is thrice as good a workman as B and therefore
alone can do it in 8 days and B alone can do it in is able to finish a job in 40 days less than B.
12 days. With the help of C they finish it in 4 days. Working together, they can do it in
Then, the share of C (in `) is (a) 14 days (b) 13 days
(a) 120 (b) 300 (c) 20 days (d) 15 days
(c) 360 (d) 240 10. A and B can do a piece of work in 8 days, B and C
4
5. Three pipes A, B and C can fill a tank in 30 min, can do it in 24 days, while C and A can do it in 8
20 min and 10 min, respectively. When the tank is 7
empty, all the three pipes are opened. If A, B and C days. In how many days can C do it alone?
discharge chemical solutions P, Q and R (a) 60 (b) 40
(c) 30 (d) 10
respectively, then the part of solution R in the
liquid in the tank after 3 min is 11. A coach helper repairs a coach in 12 days. His
8 5 6 7 technician completes the same job in 18 days. If
(a) (b) (c) (d)
11 11 11 11 both of them work together, in how many days
6. A, B and C can do a piece of work in 20, 24 and would the job be completed?
5
30 days, respectively. They undertook to do the (a) (b) 7.3
36
piece of work for ` 5400. They begin the work (c) 7.2 (d) 7

https://sscstudy.com/
https://sscstudy.com/

60 CUET (UG) Section III : General Test

12. 15 trackmen complete rail renewal job in 12 days. 20. Rani and Sneha working separately can finish a job
How many days would 18 trackmen take to complete in 8 h and 12 h respectively. If they work for an hour
the same job? alternately, Rani beginning at 9 : 00 am when will
(a) 10 (b) 18 the job be finished?
(c) 15 (d) 17
(a) 7 : 30 pm (b) 7 : 00 pm
13. A man can do a work in 15 days. His father do this (c) 6 : 30 pm (d) 6 : 00 pm
work in 20 days and man’s son do this work in 25 21. After working for 8 days, Anil finds that only 1/3 of
days. If all do the work together, how many days the work has been done. He employs Rakesh who is
will they take? 60% efficient as Anil. How many more days will
(a) Less than 6 days (b) 6 days Anil take to complete the job?
(c) Approximately 6.4 days (d) More than 10 days
(a) 5 (b) 8
14. If 6 men and 8 boys can do a piece of work in (c) 10 (d) 12
10 days, while 26 men and 48 boys can do the same 22. A, B and C can do a job alone in 6, 12 and 24
in 2 days, the time taken by 15 men and 20 boys in days respectively. In what time will they do it
doing the same type of work will be together?
(a) 4 days (b) 5 days
(c) 6 days (d) 7 days 3 3
(a) 2 days (b) 3 days
7 7
15. A can do a work in 15 days and B in 20 days. If they 2 4
work on it together for 4 days, then the fraction of (c) 3 days (d) 3 days
7 7
the work that is left is
1 1 7 8 23. A can do 1/3 of the work in 5 days and B can do 2/5
(a) (b) (c) (d)
4 10 15 15 of the work in 10 days. In how many days both A
and B together can do the work?
16. 8 horses’ food is equal to 6 cows’ food. How many
3 4
cows can eat the food meant for 20 horses? (a) 7 (b) 8
4 5
(a) 15 (b) 16
3
(c) 17 (d) 18 (c) 9 (d) 10
8
17. X can complete a piece of work in 30 h and Y can
do it in 24 h. In how many hours will it be 24. A is thrice as efficient as B and B is twice as
completed if they work together? efficient as C, if A, B and C work together how long
(a) 13.33 (b) 14.33 would they take to complete a job that B takes
(c) 15.33 (d) 16.33 10 days to complete?
20 11
18. A can do 3/4 of a work in 12 days. In how many (a) days (b) days
9 9
days can he finish 1/8 of the work? (c) 3 days (d) None of these
(a) 1 (b) 2
(c) 3 (d) 4 25. 12 men can complete a work in 18 days. 6 days
after they started working, 4 men joined them.
19. If 18 men and 10 boys can do in one day as much
work as 10 men and 22 boys, the amount that a men How many days will all of them take to finish the
should be paid per day, if a boy gets ` 10 per day remaining work?
(a) ` 7.5 (b) ` 12.5 (a) 9 (b) 10
(c) ` 20 (d) ` 15 (c) 12 (d) 15

ANSWERS
1. (c) 2. (a) 3. (a) 4. (a) 5. (c) 6. (a) 7. (b) 8. (b) 9. (d) 10. (a)
11. (c) 12. (a) 13. (a) 14. (a) 15. (d) 16. (a) 17. (a) 18. (b) 19. (d) 20. (c)
21. (c) 22. (b) 23. (c) 24. (a) 25. (a) 26. (d) 27. (a) 28. (d) 29. (b) 30. (c)

https://sscstudy.com/
https://sscstudy.com/

NUMERICAL ABILITY 61

Hints & Solutions


1. Here, M1 = 30, W1 = 1500, Quantity of solution R in 3 min 9. Ratio in work efficiency of
D1 = 24 , T1 = 6, M 2 = 18, 3 A and B = 3 : 1
=
W 2 = 1800, D2 = ?, T2 = 8 10 and then ratio in time taken by
∴ Part of solution R A and B = 1 : 3
Using, M1 T1 D1 W 2 = M 2 T2 D2 W1
3 According to question,
30 × 6 × 24 × 1800 3 × 20 6
= 10 = = 3x − x = 40
= 18 × 8 × D2 × 1500 11 10 × 11 11 2x = 40 ⇒ x = 20
30 × 6 × 24 × 1800
⇒ D2 = 20 ∴ A takes 20 days and B takes 30
18 × 8 × 1500 days.
6. Let the number of days to
= 36 days complete the work be x, then ∴ If they work together, the time
2. Let 1 man’s one day’s work be x x x−2 x−5 20 × 60
+ + =1 taken by A and B =
and 1 child's one day's work be y 20 24 30 20 × 60
there, 6x + 5 (x − 2) + 4 (x − 5)
1 ⇒ =1 = 15 days
4x + 6 y = ...(i) 120
8 10. According to question,
1 ⇒ 6x + 5x + 4x = 120 + 10 + 20 1
3x + 7 y = ...(ii) ⇒ 15x = 150 ( A + B)’s 1 day’s work =
10 8
Solving both the equations, ∴ x = 10 1
1 10 1 (B + C )’s 1 day’s work =
y= ∴ Work done by A = = 24
400 20 2 7
Therefore, 1 child can complete ∴ Share of A from the assured (C + A )’s 1 day’s work =
60
the work in 400 days and thus, 20 money
children can complete the work ∴ 2 ( A + B + C )’s 1 day’s work
1
400 = × 5400 = ` 2700 1 1 7 17
in i.e. 20 days. 2 = + + =
20 8 24 60 60
7. Work done by A and B together in 17
3. Let B will do the remaining 1 ( A + B + C )’s 1 day’s work =
work in x days. 1 day = 120
72 17 1 1
According of the question, Work done by B and C together C’s 1 day’s work = − =
5 x+5 1 120 8 60
+ =1 in 1 day = ∴ C can complete the work in
10 20 120
10 + x + 5 and work done by C and A
60 days.
⇒ =1
20 1 11. Both work together
together in 1 day =
⇒ x + 15 = 20 90 x × y 12 × 18 12 × 18
= = =
∴ x = 5 days ⇒ 2( A + B + C )’s 1 day’s work x + y 12 + 18 30
1  1 1 1 = 7.2 days
4. C can do the work = = + + 
1 1 1   72 120 90 12. T1 = 15 trackmen, D1 = 12 days
 − − 
 4 8 12
⇒ ( A + B + C )’s 1 day’s work T2 = 18 trackmen , D2 = ?
1
= = 24 days 1 1 1 1 According to the question,
 6 − 3 − 2 =  + + 
  2  72 120 90 T1 D1 = T2D2
 24 
∴ ( A + B + C )’s 2 day’s work ⇒ 15 × 12 = 18 × D2
Ratio of one day work of A, B
and C 1  5 + 3 + 4 1 15 × 12
=2×   = ⇒ D2 = = 10 days
1 1 1 2  360  30 18
= : : = 3 :2 :1
8 12 24 8. Let A completes the work in x 1
1 13. A man 1 day’s work =
∴ Share of C = × 720 days, then B completes the work 15
(3 + 2 + 1) in 2x days. 1
Man’s father 1 day’s work =
1 A and B together completes the 20
= × 720 = `120 1
6 work in = 14 days Man’s son 1 day’s work =
x + 2x 25
5. Total quantity of solutions P, Q = 14 If three do together, then 1 day’s
and R from A, B and C x + 2x
1 1 1
respectively, after 3 min 3 work = + +
⇒ x = × 14 15 20 25
3 3 3 3 × 11 11 2
= + + = = 30 + 15 + 12 57
30 20 10 60 20 = 21 days = =
300 300

https://sscstudy.com/
https://sscstudy.com/

62 CUET (UG) Section III : General Test

Hence, they do the work 4+5 9 3 Man 2 × Efficiency 2 × Days 2


300 = = = =
together in days 120 120 40 Work2
57 Both complete the work 1 × 1 × 8 1.6 × x
or 5.26 days. 40 ⇒ =
= = 13. 33 h 1  1
1 − 
14. 6 men and 8 boys can do a work 3 3  3
= 10 days 18. Using formula, 1 2
60 men and 80 boys can do a ⇒ × 1.6x = × 8
M1 D1 H 1 M 2D2H 2 3 3
work = 1 day =
W1 W2 ⇒ x = 10 days
Similarly, 26 men and 48 boys
can do a work = 2 days 12 D2 ∴10 days more to compete the
⇒ =
60 men + 80 boys 3 1 job.
4 8 1
= 52 men + 96 boys 22. Taken time =
1 4 1 1 1
8 men = 16 boys D2 = 12 × × + +
1 man = 2 boys 8 3 6 12 24
D2 = 2 1
6 men + 8 boys (4 men) can do a =
work = 10 days 4+2+1
∴ He can finish in 2 days.
10 men can do a work = 10 days 24
19. 18 men + 10 boys
15 men + 20 boys (10 men) will 1 24
= 10 men + 22 boys = =
complete the work = x days 7 7
Man 3
25 men will complete the work ⇒ = 24
Boy 2 3
= x days = 3 days
M1 D1 = M 2D2 ⇒ Their wages paid will be in 7
the ratio 3 : 2.
10 × 10 = 25 × x 23. A does one work in 15 days
100 Hence, if a boy gets ` 10 per day
∴ x= a mean get ` 15. B does one work in 25 days
25 ∴( A + B)’s one day work
⇒ x = 4 days 20. Capacity of Rani and Sneha per
1 1
1 hour = +
15. One day work of A = 1 1 25 25
15 = and respectively. 5+3 8
1 8 12 = =
One day work of B = Total work done by them in 1 h 75 75
20 Hence, ( A + B) will complete one
1 1 5
One day work of both = + = per hour 3
1 1 8 12 24 work in 9 days.
= + 48 1 8
15 20 ∴ Full work = 1 × = 9 h, then 24. Since, A = 3B and B = 2C
4+3 7 5 6
= = of work will left and now turn is ∴ B takes 10 days
60 60 of Sneha. Sneha will do the ⇒ C takes 20 days and A takes
7 7 remaining work in 30 min. 10
4 day work of both = ×4 = days.
60 15 Total time taken 3
7 8
= 9 h and 30 min ∴ If they all work together, then
∴ Remaining work = 1 − =
15 15 3 1 1 9
Time = 9 + 9 h and 30 min + + =
16. Food of 8 horses’ 10 10 20 20
= 18 : 30 h 20
= Food of 6 cows’ Rani finished the job in Hence, time taken is days.
6 9
Food of 1 horse = Food of cow 6 : 30 pm. 1
8 25. 12 men’s 6 days work =
6 21. If efficiency of Anil is taken as 1, 3
Food of 20 horses = × 20 then efficiency of Rakesh = 0.6 2
8 Remaining work =
∴After employing Rakesh, total 3
= 15 cows effective man;
1 m1 × d1 × w2 = m2 × d2 × w1
17. 1 h work of X = man × efficiency = 1 × 1 + 0.6 × 1 2 1
30 12 × 1 × = 16 × d2 ×
= 1 + 0.6 = 1.06 3 18
1
1 h work of Y = Using the formula 12 × 2 × 18
24 Man1 × Efficiency1 × Days1 d2 = = 9 days
1 1 3 × 16
1 h work of both = + Work1
30 24

https://sscstudy.com/
https://sscstudy.com/

NUMERICAL ABILITY 63

CHAPTER 13

Time, Speed and Distance

The concept of time, speed and distance is related to a particular object in motion. A set of typically asked questions
from the topic ‘time, speed and distance’ that acquaints you to different concepts in the topic including relative speed,
average speed, different units for measurement of time, speed and distance and the conversion of these units. Also
includes questions on boats and stream and train and platform.

Time 2. Distance and time are directly proportional. If


Time is defined as quantity, which governs the order or distance to be travelled is doubled, then the time
sequence of an occurrence. In the absence of time, the taken would also be doubled at the same speed.
actual sequence of any occurrence or incident would be 3. Time is inversely proportional to speed. If the
lost. If we did not have the concept of time, we would not distance remains the same and speed is doubled,
be able know in what period or in what order something then time taken to travel the same distance
took place. becomes half of the original time taken at the
original speed.
Unit of Time Hour (h) and second (s) are mostly taken as
the unit of time. Average Speed
Speed When a certain distance is covered at speed A and the
same distance is covered at speed B, then the average
Speed is defind as the distance covered per unit time. It 2AB
is the rate at which the distance is covered. speed during the whole journey is given by .
A+ B
Unit of Speed Though we commonly take km/h. As units
of speed. e.g. A person goes to Delhi from Mumbai at the speed of
60 km/h and comes back at the speed of 50 km/h.
Calculate the average speed of the person for the entire
Distance trip.
When an object is moving with a certain speed in a 2 × 60 × 50
particular time, the displacement made by an object is Sol. Average Speed =
60 + 50
called the distance.
Unit of Distance kilometre (km) and metre (m) is [QA = 60 km/h, B = 50 km/h]
usually taken as the unit of distance. 6000
= = 54. 54 km/h
110
Relationship between Time, Speed and
Distance Relative Speed
Relationship between time, distance and speed is Relative speed is the speed of a moving object in relation
expressed by to other moving object.
Distance Let two objects are in motion and their speeds are a and
Speed = or Distance = Speed × Time
Time b, respectively, then
This expression shown that (a) Relative speed = a + b (if two objects are moving in
1. Speed is directly proportional to distance. If the opposite directions)
speed is doubled, then distance travelled in the (b) Relative speed = a − b (if two objects are moving in
same time, will also be doubled. same direction and a > b)

https://sscstudy.com/
https://sscstudy.com/

64 CUET (UG) Section III : General Test

e.g., Two persons are moving in the direction opposite to Boats and Streams
each other. The speeds of the both persons are 5 km/h
The problems of boats and streams are also based on the
and 3 km/h, respectively. Find the relative speed of the
basic relation of speed, distance and time
two persons in respect of each other.
Distance
Sol. Required relative speed = 5 + 3 = 8 km/h i. e. , Speed =
Time
In there questions, the direction along the stream
Concept of Relative Speed in (water) is called downstream and direction against the
Motion of Trains stream is called upstream.
Some important points in problems on trains If the speed of a boat in still water is x km/h and the
1. If two trains of length x km and y km are moving in speed of the stream is y km/h, then
opposite directions at u km/h and v km/h, then time Downstream speed = ( x + y ) km/h
 x + y Upstream speed = ( x − y ) km/h
taken by the trains to cross each other =   h ˜ From
 u + v the above relationship we conclude, if the
downstream speed is u km/h and upstream speed v km/h
2. If two trains of length x km and y km are moving in then,
the same direction at u km/h and v km/h are, where  u + v
u > v , then time taken by faster train to cross the Speed of boat in still water =   km/h
 2 
 x + y
slower train =   h u − v 
 u − v Speed of stream =   km/h
 2 

Solved Examples
1. The speed of a bus is 72 km/h. The distance L1 + L2
Sol. (d) T =
covered by the bus in 5 s is S1 + S2
(a) 50 m (b) 74.5 m (c) 100 m (d) 60 m 160 + L2
5 ⇒ 6=
Sol. (c) Speed of bus in m/s = 72 × = 20 m/s (126 + 90)
5
18 18
∴Distance travelled in 5s = 20 × 5 (Speed × Time) = 100 m 5
⇒ 6 × 216 × = 160 + L2
18
2. Two men start together to walk a certain distance,
⇒ L2 = 200 m
one at 4 km/h and another at 3 km/h. The former
arrives half an hour before the latter. Find the 5. A train takes 18 s to pass completely through a
distance. station 162 m long and 15 s through another
(a) 6 km (b) 9 km (c) 8 km (d) 7 km station 120 m long. Find the length of the train.
Sol. (a) Let the distance = x km (a) 80 m (b) 90 m
(c) 85 m (d) 95 m
x x 1 4x − 3x 1 12 162 + l 120 + l
By given condition, − = ⇒ = ⇒x = Sol. (b) 18 = and 15 =
3 4 2 12 2 2 5 5
∴ x = 6 km
∴ l − 185 + 162 = 0
3. If a train 110 m long passes a telegraph pole in 3 s, and l − 155 + 120 = 0
then the time taken by it to cross a railway On solving the above two equations, we have l = 90 m
platform 165 m long will be
(a) 6.50 s (b) 8.5 s (c) 7.5 s (d) 6.55 s 6. A train 100 m long completely passes a man
Distance 110 walking in the same direction at 6 km/h in 5 s and
Sol. (c) Speed = = m/s a car travelling in the same direction in 6 s. At
Time 3
what speed was the car travelling?
110 + 165 275 × 3
Time taken to cross the platform = = (a) 15 km/h (b) 20 km/h
110/3 110 (c) 18 km/h (d) 16 km/h
= 7.5 s Sol. (c) In case of man,
4. Two trains A and B, travelling in opposite 100 × 18
5= ⇒ x = 78 km/h
directions, cross each other in 6 s. The speed of train (x − 6) × 5
A is 126 km/h, while that of train B is 90 km/h. If 100 × 18
the length of train A is 160 m, what is the length of In case of car, 6 =
(78 − y)5
train B?
(a) 150 m (b) 120 m (c) 220 m (d) 200 m ⇒ y = (78 − 60) = 18 km/h

https://sscstudy.com/
https://sscstudy.com/

NUMERICAL ABILITY 65

Practice Questions
1. A motorcycle covers 40 km with a speed of 11. Nalanda and Nawada are two towns. Sabir goes
20 km/h. Find the speed of the motorcycle for the from Nalanda to Nawada at 30 km/h and comes back
next 40 km journey so that the average speed of to the starting point at 70 km/h. What is the average
the whole journey will be 30 km/h. speed during the whole journey?
(a) 70 km/h (b) 52.5 km/h (a) 12 km/h (b) 60 km/h (c) 24 km/h (d) 42 km/h
(c) 60 km/h (d) 60.5 km/h
12. A train 200 m long is running at 72 km/h. In how
2. A bus can complete a journey in 12 h. The first much time will it pass a platform 200 m long?
half is completed at 22 km/h and the second (a) 10 s (b) 20 s (c) 400 s (d) 17 s
half at 26 km/h. Find the distance.
13. A railway officer standing on a railway bridge
(a) 280 km (b) 284 km
(c) 286 km (d) 288 km which is 200 m long finds that the train crosses the
bridge in 19 s but himself in 9 s . Find the length
3. A man takes 6 h 30 min in walking to a of the train.
certain place and riding back. He would have (a) 135 m (b) 180 m (c) 72 m (d) 90 m
gained 2 h 10 min by riding both ways. How 1
long would he take to walk both ways? 14. Two buses, one of them takes 7 h to travel
2
(a) 8 h 20 min (b) 4 h 10 min
(c) 8 h 40 min (d) 4 h 20 min
300 km and another takes 9 h to travel 450 km.
Find the ratio of speed of two buses.
4. Raghubir after travelling 84 km, found that if he (a) 2 : 3 (b) 4 : 3 (c) 4 : 5 (d) 8 : 9
travelled 5 km an hour more, he would take 5 h
15. A train 110 m long is running at the speed of
less, he actually travelled at a rate of
(a) 7 km/h (b) 10 km/h 72 km/h to pass a 132 m long platform in how
(c) 5 km/h (d) 6 km/h many times?
1 (a) 9.8 s (b) 12.1 s (c) 12.42 s (d) 14.3 s
5. If a man covers 10 km in 3 h, the distance
5 16. A boatman can row his boat at a speed of 8 km/h in
covered by him in 5 h is still water. If a river flows at a speed of 2 km/h,
(a) 16 km (b) 15 km then how long will the boatman take to row his
(c) 18 km (d) 17 km boat 1200 m with the direction of the current?
(a) 7.2 min (b) 7.4 min (c) 7.6 min (d) 7.8 min
6. A man covers a certain distance on scooter. Had he
moved 3 km/h faster, he would have taken 40 min 17. Two cyclists start from the same place in opposite
less. If he had moved 2 km/h slower, he would have directions. One goes towards north at 18 km/h and
token 40 min more. The distance (in km) is the other goes towards south at 20 km/h. What
(a) 42.5 (b) 36 time will they take to be 47.5 km apart?
(c) 37.5 (d) 40 1
(a) 1 h (b) 2 h
4
7. A man walking with 3/4 of his usual speed, reaches
(c) 3 h (d) None of these
office 20 min late. His usual time is
(a) 50 min (b) 80 min 18. A man goes uphill with an average speed of
(c) 70 min (d) 60 min 24 km/h and comes down with an average speed of
8. When a person cycled at 10 km/h he arrived at his
36 km/h. The distance travelled in both the cases
office 6 min late. He arrived 6 min early, when he being the same, the average speed for the entire
increased his speed by 2 km/h. The distance of his journey is
(a) 30 km/h (b) 28.8 km/h
office from the starting place is
(c) 32 km/h (d) None of these
(a) 6 km (b) 7 km
(c) 12 km (d) 16 km 19. A train 700 m long is running at the speed of
9. Walking at the rate of 4 km an hour, a man covers
72 km/h. If it crosses a tunnel in 1 min, then the
a certain distance in 3 h 45 min. If he covers the length of the tunnel is
(a) 650 m (b) 500 m
same distance on cycle, cycling at the rate of
(c) 550 m (d) 700 m
16.5 km/h, the time taken by him is
(a) 55.45 min (b) 54.55 min 20. Two trains of length 120 m and 80 m are running
(c) 55.44 min (d) 45.55 min in the same direction with velocities of 40 km/h
10. If a man runs at 2m/s, how many kilometres does
and 50 km/h respectively. The time taken by them
he run in 1 h 20 min? to cross each other is
(a) 60 s (b) 75 s
(a) 8.4 (b) 6.9
(c) 72 s (d) 80 s
(c) 9.6 (d) 7.4

https://sscstudy.com/
https://sscstudy.com/

66 CUET (UG) Section III : General Test

21. A motor boat takes 2 h to travel a distance of 9 km 30. A constable is 114 m behind a thief. The constable
down the current and it takes 6 h to travel the runs 21 m the thief 15 m in a minute. In what time
same distance against the current. The speed of will the constable catch the thief?
the boat in still water and that of the current (in (a) 17 min (b) 19 min
km/h) respectively are (c) 16 min (d) 18 min
(a) 3, 2 (b) 3.5, 2.5 31. The distance between two places A and B is 15 km.
(c) 3, 1.5 (d) 3, 1
Ram walks from A to B at 4 km/h and Shyam
22. Distance between two towns P and Q is 240 km. A walks from B to A at 6 km/h. Both start at 7 am. At
motor cycle rider starts from P towards Q at 8 pm what time will they meet?
at a speed of 40 km/h. At the same time another (a) 7 : 45 am (b) 8 : 30 am
motor cycle rider starts from Q towards P at (c) 7 : 14 am (d) 8 : 30 pm
50 km/h. At what time will they meet? 32. A car covers a certain distance in 8 h. If the speed
(a) 9 : 45 pm (b) 10 : 40 pm
is increased by 4 km/h. This distance could be
(c) 11 pm (d) 10 : 30 pm
covered in 7 1 h. This distance is
23. A man standing on a railway platform observes 2
that a train going in one direction takes 4 s to pass (a) 420 km (b) 480 km
him. Another train of same length going in the (c) 640 km (d) 700 km
opposite direction takes 5 s to pass him. The time 33. The ratio of the speeds of three cars is 2 : 3 : 4.
taken (in seconds) by the two trains to cross each What is the ratio of the times taken by them in
other will be covering the same distance?
49 40 (a) 2 : 3 : 4 (b) 4 : 3 : 2
(a) (b)
9 9 (c) 4 : 3 : 6 (d) 6 : 4 : 3
50 31
(c) (d) 34. If the speed of a train is increased by 5 km/h from
9 9
its normal speed it would have taken 2 h less to
24. A boy is running at a speed of p km/h to cover a cover 300 km. What is its normal speed?
distance of 1 km. But due to the slippery ground, (a) 20 km/h (b) 25 km/h (c) 30 km/h (d) 45 km/h
his speed reduced by q km/h ( p > q) . If he takes r h
35. A 200 m long train crosses a platform of double its
to cover the distance, then
1 pq 1 length in 36 s. The speed of the train is
(a) = (b) = p+ q (a) 60 km/h (b) 48 km/h (c) 64 km/h (d) 66 km/h
r p+ q r
1 36. A train 270 m long is moving at a speed of
(c) r = p − q (d) = p−q
r 25 km/h. It will cross a man coming from the
25. A train is going at a speed of 180 km/h. Its speed is
opposite direction at 2 km/h in
(a) 36 s (b) 32 s (c) 28 s (d) 24 s
(a) 15 m/s (b) 30 m/s
(c) 40 m/s (d) 50 m/s 37. A train running at 36 km/h takes 10 s to pass a
26. A man completes 30 km of a journey at 6 km/h and telegraph pole. How long would it take to cross a
the remaining 40 km in 5 h. His average speed for platform 110 m long?
whole journey is (a) 24 s (b) 31 s (c) 21 s (d) 33 s
4 38. A train running at a uniform speed crosses a 122
(a) 6 km/h (b) 7 km/h
11 m long platform in 17 s and a 210 m long bridge in
1 25 s. The speed of the train is
(c) 7 km/h (d) 8 km/h
2 (a) 46.5 km/h (b) 37.5 km/h
27. A is twice as fast as B and B is thrice as fast as C. (c) 37.6 km/h (d) 39.6 km/h
The journey covered by C in 42 min will be covered 39. A train is moving at a speed of 132 km/h. If the
by A in length of the train is 110 m, how long will it take
(a) 7 min (b) 28 min (c) 63 min (d) 14 min to cross a railway platform 165 m long?
28. A man covers half of his journey at 6 km/h and the (a) 7.5 s (b) 15 s (c) 10 s (d) 5 s
remaining half at 3 km/h. His average speed is 40. A train crosses a platform 100 m long in 60 s at a
(a) 4.5 km/h (b) 3 km/h speed of 45 km/h. The time taken by the train to
(c) 4 km/h (d) 9 km/h cross an electric pole is
29. If a train runs at 40 km/h, it reaches its (a) 8 s (b) 52 s (c) 1 min (d) 40 s
destination late by 11 min. But, if it runs at 41. Two trains 105 m and 90 m long run at the speeds
50 km/h, it is late by 5 min only. The correct time of 45 km/h and 72 km/h respectively in opposite
for the train to complete its journey is directions on parallel tracks. How much time do
(a) 15 min (b) 21 min they take to cross each other?
(c) 13 min (d) 19 min (a) 5 s (b) 6 s (c) 7 s (d) 8 s

https://sscstudy.com/
https://sscstudy.com/

NUMERICAL ABILITY 67

ANSWERS
1. (c) 2. (c) 3. (c) 4. (a) 5. (d) 6. (d) 7. (d) 8. (c) 9. (b) 10. (c)
11. (d) 12. (b) 13. (b) 14. (c) 15. (b) 16. (a) 17. (a) 18. (b) 19. (b) 20. (c)
21. (c) 22. (b) 23. (b) 24. (d) 25. (d) 26. (b) 27. (a) 28. (c) 29. (d) 30. (b)
31. (b) 32. (b) 33. (d) 34. (b) 35. (a) 36. (a) 37. (c) 38. (d) 39. (a) 40. (b)
41. (b)

Hints & Solutions


1. Let speed of the motor cycle for 6. Let distance and original speed x x 12
− =
the next 40 km journey be of the man be d km and s km/h. 10 12 60
x km/h. Then, Then, 12 × 60
2 × 20 × x d d 2 x= = 12 km
30 = − = 60
20 + x s s+3 3 9. Speed = 4 km/h
⇒ 3(20 + x) = 4x d (s + 3 − s) 2 3
⇒ = Time = 3 h 45 min = 3 h
⇒ 60 + 3x = 4x s (s + 3) 3 4
⇒ x = 60 km/h ⇒ 9 d = 2s (s + 3) ∴ Distance covered by man
2. Let distance of the journey be ...(i) 3
= 4 × 3 = 15 km
d km. d d 2 4
d d and − =
Then, + = 12 s−2 s 3 15
2 × 22 2 × 26 ∴Required time = h
d (s − s + 2) 2 16. 5

d
+
d
= 48 ⇒ =
s (s − 2) 3 15
11 13 = × 60 min
24d ⇒ 3 d = s (s − 2) 16. 5
⇒ = 48 = 54.55 min
143 ...(ii)
⇒ d = 286 km From Eqs. (i) and (ii), we get 10. Speed of man = 2 m/s
2 × 18 36
4. Let the speed of Raghubir be 3s (s − 2) = 2s (s + 3) = km/ h = km/ h
5 5
x km/h. ⇒ 3s2 − 6s = 2s2 + 6s
80 4
According to the question, ⇒ s2 = 12s Time = 1 h 20 min = h= h
60 3
84 84 ⇒ s = 12
− =5 Distance = Time× Speed
x x+5 From Eq. (ii), we get 36 4 48
1 1  = × = = 9.6 km
⇒ 84  − =5 3 d = 12 (12 − 2) 5 3 5
 x (x + 5)  ∴ d = 40 km 11. Average speed of Sabir
 x + 5 − x 7. Let usual speed and usual time 2xy 2 × 30 × 70
⇒ = =
84   =5 x+ y 30 + 70
 x(x + 5)  taken by the man are S km/h
84 × 5 and T h, respectively. 4200
⇒ =5 = = 42 km/h
x(x + 5) ∴ D = ST ...(i) 100
According to the question, we 12. Length of train = 200 m
⇒ x(x + 5) = 84
2 get Length of platform = 200 m
⇒ x + 5x − 84 = 0
3  1 Total distance cover by train to
⇒ x(x + 12) − 7(x + 12) = 0 D = S ⋅ T +  ...(ii)
4  3 pass the platform = 400 m
⇒ (x + 12) (x − 7) = 0
From Eqs. (i) and (ii), we get Speed of train = 72km / h
⇒ x− 7 =0 72 × 5
∴ x = 7 km/h 3  1 = = 20 m/s
ST = S ⋅ T +  18
5. Distance covered by man in 3h 4  3
Time taken by train to pass the
1 ⇒ 4T = 3T + 1 platform
= 10 km
5 ∴ T = 1 h = 60 min Distance 400
= = = 20 s
Distance covered by man in 1 h Speed 20
51 17 8. Let the distance of his office
= = km from the starting point be x km. 13. Let length of train = x m
5 ×3 5
By given condition, According to the question,
Distance covered by man in 5 h
17
x

6
=
x
+
6 x + 200
will be = × 5 = 17 km Speed = ...(i)
10 60 (10 + 2) 60 19
5

https://sscstudy.com/
https://sscstudy.com/

68 CUET (UG) Section III : General Test

x 5 24. Actual speed of boy


Speed = ...(ii) 19. Speed = 72 × = 20 m/s
9 18 = ( p − q) km/h
From Eqs. (i) and (ii), Let the length of tunnel be x m. Time taken to cover 1 km
x x + 200 700 + x 1
= Then, = 60 =
9 19 20 p−q
⇒ 19x − 9x = 1800 ⇒ x = 500 m 1
∴ =r
⇒ 10x = 1800 20. Since, the train is running in p−q
⇒ x = 180 m same direction therefore relative 1
Hence, length of train is 180 m. speed ⇒ = p−q
r
300 = (50 − 40) km/h
14. First bus speed = ×2  5
15 = 10 ×
5 25
= 25. 180 km/h = 180 ×  m/s
m/s  18
= 40 km/h 18 9
∴ Required time = 50 m/s
450
Second bus speed = = Time taken to cover 26. Total journey = (30 + 40) = 70 km
9
25
= 50 km/h (120 + 80) at m/s  30 
9 Total time taken =  + 5
∴ Required ratio = 40 : 50 = 4 : 5 6 
9
15. Speed of the train = 72 km/h = 200 × = 72 s = 10 h
25
70
= 72 ×
5
m/s 21. Rate of downstream Average speed = km/h
18 10
9
= = 4 .5 km/h = 7 km/h
= 20 m/s 2
110 + 132 9 27. Let C ’s speed be x m/min.
∴Required time = Rate of upstream = = 1 .5 km/h
20 6 B’s speed = 3x m/min
242 ∴Rate in still water A’s speed = 6x m/min
= = 12 .1 s
20 1 Ratio of speeds of A and C
= (4 .5 + 1 .5) = 3 km/h
16. Speed of boatman in still water 2 = 6x : x = 6 : 1
= 8 km/h Rate of the current ∴ 6 : 1 = 42 : y
1 42
Speed of river = 2 km/h = (4 .5 − 1 .5) = 1 .5 km/h ⇒ y= = 7 min
2 6
Speed of boatman with the
22. Suppose they meet x h after ∴A takes 7 min to cover it.
direction of current
8 pm. Then, sum of distance 2xy
= 8 + 2 = 10 km/h 28. Average speed = km/h
covered by them in hours x+ y
Distance cover by boat in the = 240 km 2 ×6 ×3
direction of current = 1200 m =
∴ 40x + 50x = 240 6+3
Distance
Time = ⇒ x=
240
= 2 h 40 min 36
Speed 90 = = 4 km/h
1200 9
= Hence, they will meet in
10 × 1000 29. Let the distance travelled by the
10:40 pm.
60 train is x km.
1200 × 60 72 23. Let the length of each train be Then,
= = x m. x x
10 × 1000 10 − = 60
x
Then, speed of first train = m/s 40 × 1000 50 × 1000
= 7 . 2 min 4 60 60
17. Since, they are in opposite and speed of second train 60x 60x
direction. So, total distance ⇒ − =6
x 40000 50000
apart = 18 + 20 = 38 km = m/s
5 60  x x 6
For 47.5 km apart, it takes time ⇒  −  =
 x x 10000  4 5 1
1 Relative speed =  +  m/s
= × 47.5  4 5 x
38 ⇒ = 1000
9x 20
1 = m/s
=1 h 20 ⇒ x = 20 km
4
2xy ∴ Time taken to cross each other ∴The correct time for the train
18. Average speed = = Time taken to cover 2x m to complete its journey
x+ y
 9x 20
2 × 24 × 36 at   m/s = × 60 − 11
=  20 40
24 + 36
20 40 = 30 − 11
= 28 . 8 km/h = 2x × = s
9x 9 = 19 min

https://sscstudy.com/
https://sscstudy.com/

NUMERICAL ABILITY 69

30. (21 − 15) = 6 m is covered in 35. Length of train = 200 m 187


= m/s
and length of platform 17
1 min
= 2 × 200 m = 400 m  187 18
114 m will be covered in = ×  km/h
200 + 400  17 5
1  Speed of the train =
 × 114 min = 19 min 36 = 39.6 km/h
6 
600 39. Speed of train
31. Suppose they meet after x h. = m/s
36  5  110
Then, 4x + 6x = 15 600 18 = 132 ×  = m/s
= ×  18 3
⇒ 10x = 15 36 5
= 60 km/h Time taken to cross the platform
⇒ x = 1.5 h
110 + 165  275 × 3
So, they meet at 8 : 30 am. 36. Relative speed = (25 + 2) = = 
110 / 3  110 
32. Let the required distance be = 27 km/h 15
x km. 5 = s
x x = 27 × m/s 2
Then, − =4 18 = 7.5 s
15 / 2 8 15
2x x = m/s 40. Let the length of the train be
⇒ − =4 2 x m.
15 8 Required time  5 25
⇒ 16x − 15x = 480 270 270 × 2 Speed = 45 ×  m/s = m/s
= = s = 36 s  18 2
⇒ x = 480 km 15 15 x + 100 25
1 1 1
33. Required ratio = : : 2 ∴ =
2 3 4 60 2
 5
37. Speed of train = 36 ×  m/s ⇒ 2x + 200 = 1500
= 6 :4 :3  18
⇒ 2x = 1300
34. Let the normal speed be x km/h. = 10 m/s ⇒ x = 650
300 300 Length of the train = (10 × 10) m
− =2 Time taken to cross a pole
x (x + 5) = 100 m  2
1 1 1 Time taken to cross the platform = 650 ×  = 52 s
⇒ − =  25
x x + 5 150 100 + 110 210
= = = 21 s 41. Sum of the lengths of trains
x+5−x 1 10 10
⇒ = = (105 + 90) m = 195 m
x(x + 5) 150 38. Let the length of train be x m/s.
Relative speed = (72 + 45)
⇒ x(x + 5) = 750 x + 122 x + 210
= = 117 km/h
⇒ x2 + 5x − 750 = 0 17 25 5 65
⇒ x2 + 30x − 25x − 750 = 0 ⇒ 17x + 3570 = 25x + 3050 = 117 × = m/s
18 2
⇒ x(x + 30) − 25(x + 30) = 0 ⇒ 8x = 520 ⇒ x = 65 Time taken to cross each other
⇒ (x − 25)(x + 30) = 0 ∴Length of the train = 65 m  2
65 + 122 = 195 ×  = 6 s
∴ x = 25 km/h Speed of train = m/s  65
17

https://sscstudy.com/
https://sscstudy.com/

70 CUET (UG) Section III : General Test

CHAPTER 14

Area and Perimeter


In this chapter, we often have to deal with the problem of finding the areas of plane figures. As we know that, plane
figures have only length and breadth (i.e., 2D only), hence this chapter can be featured as 2D mensuration too.

Area
It is the space enclosed within the boundary of a 2D figure. Unit Unit of its is same as the unit of side i.e., m, cm etc.
Unit sq cm, sq m (i.e., square units) etc. Triangle An enclosed figure formed by three line segment.
Quadrilateral A close figure having four sides.
Perimeter Circle A plane figure enclosed by a curve on which every point is
Sum of length of all the sides of 2D figure is known as equally distant from a fixed point.
its perimeter.

2D Figures with their Postulates

Different Types of Triangle Related Formulae Different Types of Triangle Related Formulae
● Equilateral Triangles Its Area = 3 4 a 2 ● Scalene Triangle Its all Area = s (s − a ) (s − b) (s − c)
all three sides are equal. Height, h = 3 2 a sides are unequal. 1
Side = a, Height = h = ×c×h
Perimeter = 3a 2
Each angle = 60° b
a a+ b+ c
h Where, s =
a a 2
h Perimeter = a + b + c
c
a
a, b, c = Unequal sides of the
triangle
● Isosceles Triangles Its two b 1
Area = 4a 2 − b2 ● Right Angled Triangle Area = × b× p
sides are equal. 4 2
One of the angle in this
a = Equal sides 2
triangle is 90°. Perimeter = p + b + h
a 2 −  
b
b = Third unequal side Height, h =
 2 h = Hypotenuse, b = Base h 2 = p 2 + b2
h = Height
1 p = Perpendicular
A = 4a 2 − b2
2
Perimeter = a + a + b
a a = 2a + b, h
h p
∠ABC = ∠ACB
D BD = DC
B C b
b

https://sscstudy.com/
https://sscstudy.com/

NUMERICAL ABILITY 71

Different Types of Quadrilateral Related Formulae


● Parallelogram It is a quadrilateral with opposite sides parallel Area = Base × Height = b × h
and equal. Perimeter = 2(a + b)
A b ∠ABC = ∠ADC
B
and ∠BAD = ∠BCD
a h a Area (∆ ADC ) = Area ( ∆ABC )

D b C
a, b → Opposite parallel sides, h = Height
1
● Trapezium Quadrilateral with one of the pair of opposite sides Area = (Sum of parallel sides) × Height
equal is called trapezium. 2
a 1
= (a + b)h
2
c h d Perimeter = a + b + c + d

b
a, b → Opposite parallel sides, h = Height

Rhombus It is a parallelogram whose all the four sides are equal. 1



Area = × d1 × d 2
a 2
1 2 2
Sides (a ) = d1 + d 2
a O 2
a Perimeter = 4a
d2 2 2
d1 4a 2 = d1 + d 2
a
a = Sides, d1 , d 2 = Diagonals
● Rectangle It is a parallelogram with equal opposite and each angle Area = Length × Breadth = l × b
is equal to 90°. Perimeter = 2(l + b)
l
A B Diagonal (d ) = l 2 + b2

d
b b

D l C
l = Length, b = Breadth, d = Diagonal

Square It is a parallelogram with all 4 sides equal and each angle d2


Area = (Side) 2 = a 2 =

is equal to 90°. 2
A a B Perimeter = 4 × Side = 4a
Diagonal (d ) = a 2
a d a

D a C
a = Sides, d = Diagonal
Area = π r 2
● Circle It is a plane figure enclosed by a curve on which every point
Circumference (Perimeter) = 2 πr
is equally distant from a fixed point called centre inside the curve.
Diameter = 2r
πrθ
Length of the arc (l) =
180°
πr 2 θ
O Area of sector AOB =
360°
r r
r = Radius
22
A B π=
7
l

https://sscstudy.com/
https://sscstudy.com/

72 CUET (UG) Section III : General Test

Regular Polygon ◆ The area of the largest inscribed in a semicircle of radius r is equal
to r 2 .
In a regular polygons, all sides and all interior angles ◆ Area of a square inscribed in a circle of radius r is equal to 2 r 2 .
are equal.
360° Some Common Polygon
Each exterior angle = (n = Number of sides of
n Number of Sides (n) Polygon Area
polygon) 5 3
Pentagon 5 a2
360° 4
Each interior angle = 180° −
n 6 Hexagon 3
6a 2
 n ( n − 1)  4
Number of diagonals of a polygon =  − n
 2  7 Heptagon

Note 8 Octagon 2 ( 2 + 1) a 2
◆ If the length of a rectangle are increased by a% and b% 9 Nanogon
respectively, then area will be increased by  a + b +
ab  10 Decagon
 %.
 100 

Solved Examples
1. A rectangular garden is 100 m long and 60 m Alternate Method
broad. It is surrounded by a 5m wide road. What is
Percentage increase in area =  x + y +
xy 

the area of the road?  100 
(a) 1600 m 2 (b) 1200 m 2 (c) 1000 m 2 (d) 1700 m 2
Here, x = 10 and y = 8
Sol. (d) Area of the rectangular garden ABCD 10 × 8 4
= 10 + 8 + = 18 %
= 100 × 60 = 6000 m 2 100 5
and area of the rectangular garden with road EFGH
= (100 + 2 × 5) × (60 + 2 × 5) = 110 × 70 = 7700 m 2 3. A wire in the form of a square is cut and bent in
E F the form of a circle. If the area of the square is 110
cm2 , what is the area of the circle?
A B (a) 160 cm 2 (b) 140 cm 2
(c) 120 cm 2 (d) 180 cm 2
2
Sol. (b) Area of the square = 110 cm
D C ∴One side of the square = 110 cm
∴Perimeter of the square = 4 × 110 cm
H G But the perimeter of the square = Circumference of the
2
∴ Area of road = 7700 − 6000 = 1700 m circle
∴ Circumference of the circle = 4 × 110 cm
2. If the length and breadth of a rectangle are
increased by 10% and 8% respectively, then by 4 110
∴ Radius of the circle =
what per cent does the area of the rectangle 2× π
increase? 4 110 × 7 7 110
2 2 4 2 = = cm
(a) 16 (b) 14 (c) 18 (d) 18 2 × 22 11
3 7 5 5
2
Sol. (c) Let the original length and breadth of the  7 110 
∴ Area of the circle = π ×  
rectangle are x and y.  11 
∴Its original area = xy 22 49 × 110
= × = 140 cm 2
x × 110 11x 7 11 × 11
Increased length = =
100 10 Alternate Method
108 27 y Perimeter of the square = 4 110 cm
and increased breadth = y × =
100 25
(Perimeter of the square)2
11x 27 y 297xy ∴ Area of the circle =
∴Resulting area = × = 4p
10 25 250
297xy 47xy (4 110 )2 × 7
∴Increase in area = − xy = =
250 250 4 × 22
47xy 100 94 4 16 × 110 × 7
∴Percentage increase in area = × = = 18 % = = 140 cm 2
250 xy 5 5 4 × 22

https://sscstudy.com/
https://sscstudy.com/

NUMERICAL ABILITY 73

4. The radius of a circle is 11 cm. What is the area of Diagonal of the square = Diameter of the circle
the square inscribed in the circle? = 2 × 11 = 22 cm
(a) 212 cm 2 (b) 232 cm 2 1
∴Area of the square = (Diagonal)2
(c) 242 cm 2 (d) 244 cm 2 2
1
Sol. (c) = × 22 × 22 = 242 cm 2
2
Alternate Method
Area of the square inscribed in a circle = 2r 2
Here, r = 11= 2 × (11)2 = 242 cm 2

Practice Questions
1. If the ratio of the areas of two squares is 1 : 4, the 10. A semicircular shaped window has diameter of 63 cm.
ratio of their perimeters is  22
Its perimeter equals to  π = 
(a) 1 : 6 (b) 1 : 8  7
(c) 1 : 2 (d) 1 : 4 (a) 126 cm (b) 162 cm (c) 198 cm (d) 251 cm
2. Perimeter of a triangle is 24 m and circumference of 11. Tiling work of rectangular hall 60 m long and 40 m
its incircle is 44 m. Find the area of the triangle. broad is to be completed with a square tile of 0.4 m
(a) 42 sq m (b) 24 sq m side. If each tile cost ` 5, find the total cost of the
(c) 48 sq m (d) 84 sq m
tiles.
3. The ratio of the length of the parallel sides of a (a) ` 60000 (b) ` 65000 (c) ` 75000 (d) ` 12000
trapezium is 3 : 2. The shortest distance between
12. A horse is placed for grazing inside a square field
them is 15 cm. If the area of the trapezium is
12 m long and is tethered to one corner by a rope
450 cm 2, then sum of lengths of the parallel sides is
8 m long. The area it can graze is
(a) 15 cm (b) 36 cm
(a) 50.18 sq m (b) 50.28 sq m
(c) 42 cm (d) 60 cm
(c) 50.38 sq m (d) 50.48 sq m
4. A parallelogram has sides 15 cm and 7 cm long. The
13. The diameters of two concentric circles are 8 cm and
length of one of the diagonals is 20 cm. The area of
10 cm. The area of the region between them is
then parallelogram is
(a) 2π sq cm (b) 4π sq cm
(a) 42 cm 2 (b) 60 cm 2 (c) 36π sq cm (d) 9π sq cm
(c) 84 cm 2 (d) 96 cm 2
14. The diagonal of square field is 50 m. The area
5. An equilateral triangle and a regular hexagon have
(in m 2) of the field is
the same perimeter. The ratio of the area of the
(a) 625 (b) 1250 (c) 2500 (d) 5000
triangle to that of the hexagon is
(a) 3 : 2 (b) 2 : 3 15. The area of the shaded region in the given figure is
(c) 1 : 2 (d) 1 : 4
6. If the edge of a cube is increased by 100%, then the
surface area of the cube is increased by
(a) 100% (b) 200% (c) 300% (d) 400% 45° 45°
7. A street of width 10 m surrounds from outside a a2 a2
(a) 2a 2 ( π − 2) (b) ( π − 2) (c) a 2 ( π − 1) (d) ( π − 1)
rectangular garden whose measurement is 200 m × 180 2 2
m. The area of the path (in sq m) is
(a) 8000 (b) 7000 16. The area of the shaded region shown in the given
(c) 7500 (d) 8200 figure is
8. The length of the three sides of a right angled
triangle are ( x − 2) cm, x cm and ( x + 2) cm,
7 cm
respectively. Then, the value of x is
(a) 10 (b) 8 (c) 4 (d) 0 7 cm
14 cm

14 cm
9. The lengths of two sides of an isosceles triangle are
15 and 22, respectively. What are the possible values 7 cm
of perimeter? 28 cm
(a) 52 or 59 (b) 52 or 60 (a) 515 cm 2 (b) 535 cm 2
(c) 15 or 37 (d) 37 or 29 (c) 505 cm 2 (d) 525 cm 2

https://sscstudy.com/
https://sscstudy.com/

74 CUET (UG) Section III : General Test

17. A square field with side 30 m is surrounded by a path 20. If the radius of a circle is tripled, its perimeter will
of uniform width. If the area of the path is become how many times of its previous perimeter?
256 m 2, the width of the path is 1
(a) 2 (b) (c) 9 (d) 3
(a) 14 m (b) 16 m 3
(c) 4 m (d) 2 m 1
21. A room is half as broad as it is high and 1 times as
18. An equilateral triangle of side 6 cm has its corners 2
cut off to form a regular hexagon. The area of this high as long. If the cost of carpeting the floor at `
hexagon is 3 per sq m, is ` 144, the height (in m) of the room is
(a) 6 3 sq cm (b) 3 3 sq cm (a) 8 (b) 10
5 3 (c) 12 (d) 16
(c) 3 6 sq cm (d) sq cm
2 22. A rectangular plot 90 m × 50 m has two 10 m wide
19. The area of the four walls of a room is 128 sq m. The roads running in the middle of it, one parallel to the
length is equal to the width and the height is length and the other parallel to the breadth. Area of
4 m. The area of the floor of the room is the plot used in roads is
(a) 32 sq m (b) 49 sq m (a) 1300 sq m (b) 1400 sq m
(c) 64 sq m (d) 81 sq m (c) 1500 sq m (d) 1200 sq m

ANSWERS
1. (c) 2. (d) 3. (d) 4. (c) 5. (b) 6. (c) 7. (a) 8. (b) 9. (a) 10. (b)
11. (c) 12. (b) 13. (d) 14. (b) 15. (b) 16. (d) 17. (d) 18. (a) 19. (c) 20. (d)
21. (c) 22. (a)

Hints & Solutions


A 15 cm
1. Ratio of perimeters B 6. The required surface area
= Ratio of areas increased by
= 1 : 4 = 1: 2 20 cm  100 + x 2 
7 cm =   − 1 × 100%
 
2. If inradius of the triangle be r,  100 
there  100 + 100 2 
2πr = 44 D C =   − 1 × 100%
 100 
44 ∴ Area of the parallelogram  
⇒ r= =7m
2π = 2 × Area of ∆ ABC  200 2 
∆ =   − 1 × 100%
Using r = = 2 × 42  
S  100 
= 84 cm 2
24 = [(2)2 − 1] × 100%
⇒ ∆ = 7× 5. Let the side of an equilateral
2 = (4 − 1) ×100%
triangle is a and side of hexagon
= 7 × 12 = 84 m2 is b. = 300%
3. Let the length of the parallel According to question, 7. 10
sides of a trapezium be 3x cm, Perimeter of equilateral triangle 200m
2x cm.
= Perimeter of hexagon
Given,
180m

10 10
3a = 6b
Area of trapezium = 450 a
1 =2
× (2x + 3x) × 15 = 450 b
2 3 2 10
450 × 2 a
5x = = 60 cm ∴ Ratio of the areas = 4
15 3 2 ∴ The area of the path
∴ Required sum = 60 cm 6× b
4 = (200 + 10 × 2) × (180 + 10 × 2)
15 + 7 + 20
4. In ∆ ABC, S = = 21 a2 1  a
2
− 200 × 180
2 = 2=  
6b 6  b = 220 × 200 − 200 × 180
Area of ∆ ABC
1 2 = 44000 − 36000
= 21(21 − 15)(21 − 7)(21 − 20) 2
= (2) =
6 3 = 8000 m 2
= 21 × 6 × 14 × 1 = 42 cm 2

https://sscstudy.com/
https://sscstudy.com/

NUMERICAL ABILITY 75

8. In right angled triangle and area of the semicircle ∴ Area of the hexagon
(Hypotenuse)2 = (Base)2 1 3 3 3
= πa 2 = × (6)2 − × (2)2
+ (Perpendicular)2 2 4 4
⇒ (x + 2) = x2 + (x − 2)2
2 ∴Area of the shaded region 3
1 = [36 − 12]
⇒ x + 4 x + 4 = x2 + x2 − 4 x + 4
2
= πa 2 − a 2 4
2
⇒ x2 = 8 x =
3
× 24
1
⇒ x=8 = a 2(π − 2) 4
2
9. Perimeter of isosceles triangle = 6 3 cm 2
= 15 + 15 + 22 or 16. Area of the whole external
rectangle = 28 × 14 = 392 cm 2 19. Let the length of the room be
15 + 22 + 22 = 52 or 59 units x m.
Area of the whole semicircle
10. Perimeter of semicircular 1 22 Q 128 = 2 × 4 (x + x)
shaped window = (πr + 2r ) cm = × × 14 × 14 ∴ x=8m
2 7
= r (π + 2) cm So, area of the floor = 64 m 2
63  22  = 308 cm 2
=  + 2 cm ∴ Area of the whole external 20. Let the radius of the initial
2  7 
figure = 392 + 308 = 700 cm 2 circle be r.
63 36
= × = 162 cm and area of the whole inner ∴Radius of the resulting circle
2 7 = 3r
figure
60 × 40
11. Number of tiles = 1 22 ∴ Perimeter of the initial circle
0.4 × 0.4 = 14 × 7 + × ×7×7
2 7 = 2πr
= 15000 = 98 + 77 and perimeter of the resulting
Total cost of the tiles = 15000 × 5 = 175 cm 2 circle = 2π (3r ) = 6πr
∴ Area of the shaded region 6 πr
= ` 75000 ∴ Required ratio = =3
= 700 − 175 2 πr
1
12. The required area = [π (8)2] = 525 cm 2 21. Let the height of the room be
4
1 22 17. Let the breadth of the path x m. x m.
= × × 64 = 50. 28 sq m 2
4 7 ∴(30 + 2x)2 − (30)2 = 256 ∴ Length = x ×
3
13. The area of the region between ⇒ (30 + 2x + 30)(30 + 2x − 30)
2x
two concentric circles = 256 = m
3
= π (R2 − r 2) = π (52 − 42) 30 m x
= π (25 − 16) = 9π cm 2 and Breadth = m
2
14. Area of the square field 2x x
1 ∴ Area of the floor = ×
= (Diagonal)2 3 2
2 x2
1 = sq m
= (50)2 ⇒ (60 + 2x) × 2x = 256 3
2 x 2
⇒ x2 + 30x − 64 = 0 × 3 = 144
= 1250 sq m Q
⇒ (x − 2)(x + 32) = 0 3
15. If one side of the triangle is x, ⇒ x = 12 m
then in right angled triangle ∴ x = 2 or −32
So, the breadth of the path be 22. Area of road = Width of road
x2 + x2 = 4 a 2
2 m. × [Length of plot + Breadth of plot
∴ x=a 2
− Width of road]
∴ Area of the right angled 18. Q One side of an equilateral
triangle triangle = 6 cm = 10 × [90 + 50 − 10]
1 ∴One side of the regular = 10 × 130
= × a 2 × a 2 = a2 hexagon = 2 cm
2 = 1300 sq m

https://sscstudy.com/
https://sscstudy.com/

76 CUET (UG) Section III : General Test

CHAPTER 15

Volume and
Surface Area
In this chapter, we have to deal with the problem of finding the volume and surface area of solid figures. As we know
that, solid figures have length, breadth and height (thickness). Hence, this chapter can be featured as 3D and
mensuration too.

Volume Surface Area


It is the space occupied within the boundary of a 3D It is the total area that can be measured on the entire
figure. surface. This can only be measured, if the object is a 3D
Unit cu cm, cu m (i.e., cube units) etc. object surface area is measured in square unit.

Important Formulae Related to Solid Figures

Name Figure Lateral/Curved Total Surface Volume Nomenclature


Surface Area Area

l = Length
h b = Breadth
Cuboid 2h (l + b) 2 (lb + bh + h l ) lbh h = Height
b
I

a
Cube 4a 2 6a 2 a3 a = Edge

a
a

b
2 (Area of base)
h (Perimeter of base)
Right prism + Lateral surface Area of base × Height
× Height
area

Right circular r = Radius of base


cylinder h 2πrh 2πr (r + h ) πr 2h h = Height of the
cylinder

https://sscstudy.com/
https://sscstudy.com/

NUMERICAL ABILITY 77

Name Figure Lateral/Curved Total Surface Volume Nomenclature


Surface Area Area

l 1
h (Area of the base)
(Perimeter of the base) Area of the base +
Right pyramid 3
× (Slant height) Lateral surface area
× Height
r

h = Height
l 1 r = Radius
Right circular cone h πrl πr (l + r ) πr 2h
3 l = Slant
height
r

r 4
Sphere — 4 πr 2 πr 3 r = Radius
3

2
Hemisphere 2 πr 2 3 πr 2 πr 3 r = Radius
3
r

R 4 R = Outer radius
Spherical shell r — 4 π (R 2 − r 2 ) π (R 3 − r 3 )
3 r = Inner radius

Solved Examples
1. If the surface of a cube is 216 sq cm, its volume 3. One rectangular tank is 2.1 m long and 2 m broad.
will be If the capacity of the tank is 21 hectolitre, then the
(a) 108 sq cm (b) 36 cu cm height is
(c) 216 sq cm (d) 216 cu cm (a) 0.9 m (b) 0.5 m (c) 0.7 m (d) 10 m
2
Sol. (d) Surface of a cube = 6 × (Side) Sol. (b) Capacity = 21 hectolitre = 2100 L = 21
. cu m
∴ 216 = 6 × x2 ∴ Capacity = l × b × h ⇒ 21 . = 21
. × 2× h
Where, x is the side of cube. 2.1
∴ h= = 0.5 m
216 2.1 × 2
x2 = = 36
6
∴ x = 36 = 6 cm
4. If each edge of a cube is decreased by 40%. Find
∴Volume of the cube = 6 × 6 × 6 = 216 cu cm
the per cent decrease in the surface area of the cube.
(a) 64 (b) 84 (c) 80 (d) 74
2. If the area of the floor of a rectangular room be
Sol. (a) According to the formula,
15 sq m and the height of the room be 4 m, then
Percentage decrease in surface area
how much air is in the room?
15  (− 40)2   (1600) 
(a) 11 cu m (b) cu m =  2 (− 40) +  =  − 80 +
4  100   100 
(c) 60 cu m (d) 19 cu m = (− 80 + 16) = − 64%
Sol. (c) Volume = Area × h = 15 × 4 = 60 cu m Negative sign shows that decrease takes place here.

https://sscstudy.com/
https://sscstudy.com/

78 CUET (UG) Section III : General Test

Practice Questions
1. The base of right prism is an equilateral triangle with 11. Three cubes of iron of edges 9 cm, 12 cm and 15 cm
a side of 7 m and its height is 24 m. Find its volume. respectively are melted to form a large single cube.
(a) 509 m3 (b) 1018 m3 The edge of the new cube is
(c) 529 m3 (d) 519 m3 (a) 10 cm (b) 14 cm
2. To raise the height of a low land 48 m long and 31.5 (c) 18 cm (d) 16 cm
m broad to 6.5 dm, a ditch 27 m long and 12. If the radius of base and height of a cone are
18.2 m broad was dug in a side plot, the depth of the increased by 10%, then the volume of the cone is
ditch will be increased by
(a) 5 m (b) 7 m (a) 30% (b) 33.1%
(c) 1 m (d) 2 m (c) 40% (d) 42%
3. Weight of a solid metal sphere of radius 4 cm is 4 kg. 13. A sphere of radius 2 cm is put into water contained in
The weight of a hollow sphere made with same metal, a cylinder of radius 4 cm. If the sphere is completely
whose outer diameter is 16 cm and inner diameter is immersed in the water, the water level in the cylinder
12 cm, is rises by
(a) 20.5 kg (b) 15.5 kg 1 1 2
(a) 2 cm (b) cm (c) cm (d) cm
(c) 16.5 kg (d) 18.5 kg 3 2 3
4. The radii of a sphere and a right circular cylinder are 14. A sphere and a right circular cylinder have the same
equal and their curved surface areas are also equal. radius r. If their volumes are equal, the height of the
The ratio of their volumes is cylinder is
(a) 3 : 4 (b) 2 : 3 4 3 2 3
(a) r (b) r (c) r (d) r
(c) 3 : 2 (d) 4 : 3 3 4 3 2
5. A sphere and a cylinder have equal volume and equal 15. If the ratio of surface areas of two sphere is 9 : 16,
radius. The ratio of the curved surface area of the then the ratio of their volume is
cylinder to that of the sphere is (a) 3 : 4 (b) 9 : 16
(a) 4 : 3 (b) 2 : 3 (c) 27 : 64 (d) 81 : 256
(c) 3 : 2 (d) 3 : 4 16. The radius of the base and height of a cone are 3 cm
6. A sphere and a cube have equal surface areas. The and 5 cm respectively whereas the radius of the base
ratio of the volume of the sphere to that of the cube is and height of a cylinder are 2 cm and 4 cm
(a) π: 6 (b) 6: π respectively. The ratio of the volume of the cone to
(c) 2: π (d) π:3 that of the cylinder is
(a) 15 : 8 (b) 45 : 16
7. A sphere exactly fits inside a hollow cylinder closed at (c) 15 : 16 (d) 1 : 3
both ends. The ratio of the volume of the empty space
in the cylinder to the volume of the sphere is 17. A rectangular block 6 cm × 42 cm × 45 cm is cut up
(a) 2 : 1 (b) 1 : 2 into exact number of equal cubes. The least possible
(c) 2 : 3 (d) 3 : 2 number of cubes will be
(a) 30 (b) 210 (c) 330 (d) 420
8. If the length of longest rod that can be placed within
the cuboid is 5 5 m long and the sum of lenght 18. Two cylindrical buckets have their diameters in the
breadth and height is 19 m long, then find the whole ratio 3 : 1 and their heights are as 1 : 3. Their
surface area of that cuboid. volumes are in the ratio
(a) 1 : 2 (b) 2 : 3 (c) 3 : 1 (d) 3 : 4
(a) 236 m 2 (b) 256 m 2
(c) 125 m 2 (d) 361 m 2 19. A sphere is cut into two hemispheres. One of them is
9. The number of spherical bullets that can be made out used as a bowl. It takes 8 bowlfuls of this to fill a
of a solid cube of lead whose edge measures 44 cm, conical vessel of height 12 cm and radius 6 cm. The
 22 radius of the sphere is
each bullet being of 4 cm diameter, is  take π =  (a) 2 cm (b) 3 cm
 7 (c) 4 cm (d) 6 cm
(a) 2541 (b) 2451
(c) 2514 (d) 2415 20. A cone of height 7 m and of base radius 3 m is carved
from a rectangular block of wood of dimensions 10 m
10. The length of longest pole that can be placed in a × 5 m × 4 m. The percentage of volume of the block
room of 12 m long, 8 m broad and 9 m high is left out is
(a) 12 m (b) 17 m (a) 67% (b) 66%
(c) 19 m (d) 21 m (c) 34% (d) 33%

https://sscstudy.com/
https://sscstudy.com/

NUMERICAL ABILITY 79

21. Three cubes of metal whose edges are in the ratio 23. The surface areas of a cylinder, a cone and a
3 : 4 : 5 are melted to form a single cube whose diagonal hemisphere of smae radii are equal. The
is 12 3 cm. The edges of the three cubes (in cm) are ratio between height of the cylinder and
(a) 9, 12, 15 (b) 15, 20, 25 cone is
(c) 6, 8, 10 (d) 8, 10, 12 (a) 2 3 : 1 (b) 1 : 2 3
22. A solid cylinder of diameter 14 mm and length 25 mm (c) 2 : 3 (d) 3 : 2
has a volume 3850 mm 3. If the length were doubled 24. If the side of two cubes are in the ratio 3 : 1, the ratio
and the diameter halved, the new volume would be of their total surface areas is
(a) 1172 mm3 (b) 1925 mm3 (a) 3 : 1 (b) 8 : 1
(c) 3850 mm3 (d) 7700 mm3 (c) 9 : 1 (d) 12 : 1

ANSWERS
1. (a) 2. (d) 3. (d) 4. (b) 5. (c) 6. (b) 7. (b) 8. (a) 9. (a) 10. (b)
11. (c) 12. (b) 13. (d) 14. (a) 15. (c) 16. (c) 17. (d) 18. (c) 19. (b) 20. (a)
21. (c) 22. (b) 23. (b) 24. (c)

Hints & Solutions


3
1. Area of base of right prism 4. Given, 4πr 2 = 2πrh 4  r
3
4  3 
= π  = π 
3 49 3 2 ⇒ h = 2r 3  a 3  2π
= (7)2 = m
4 4 4 3
Now, required ratio = πr : πr 2h 4π × 3 3 6
Volume of right prism 3 = =
3 ×2 2 × π π π
= Area of base × Height = 4r : 3h
49 3 7. Radius of the sphere = Radius of
= × 24 = 4r : 6r (Q h = 2r )
the base of the cylinder
4 = 2 :3 and height of the cylinder =
= 6 × 49 × 3
5. According to question, Diameter of the sphere = 2r
= 294 × 1.732 4
Volume of sphere = Volume of Volume of sphere = πr3
= 509.222 ≈ 509 m3 cylinder 3
2. Let the depth of the ditch be 4 3
πr = πr 2h
h m. 3
According to the question, 4
h= r
6. 5 3
48 × 31. 5 ×
10 ∴ Required ratio
h=
27 × 18.2 Curved surface of sphere
=
9828 Curved surface of cylinder and volume of the cylinder
= =2m
4914 = πr 2(2r ) = 2πr3
4 πr 2 4 πr 2 3
3. Volume of solid sphere of radius = = = ∴ Volume of the empty space
2πrh 4  2
4 2πr  r 4
4 cm = π (4)3 3  = 2πr3 − πr3
3 3
Volume of hollow sphere 6. Let radius of sphere = r 2 3
= πr
4 and side of cube = a 3
= π [(8)3 − (6)3 ] 2πr3 4 πr3
3 According to question, ∴ Required ratio = :
4 3 3
Q Weight of π (4)3 cm3 = 4 kg Surface area of sphere
= 1 :2
3
= Surface area of cube
4
∴ Weight of π [(8)3 − (6)3 ] cm3 8. Given, l + b + h = 19 ...(i)
3 4πr = 6a 2
2
2 2 2
r 3 and l + b + h =5 5 ...(ii)
4 4
= ⋅ π [(8)3 − (6)3 ] =
4 a 2π On squaring Eq. (i), we get
π (4)3 3
3 4 3 (l + b + h )2 = (19)2
4 (512 − 216) πr
= = 18.5 kg ∴ Ratio of volumes = 3 3 l2 + b2 + h 2 + 2(lb + bh + hl)
43 a = 361 ...(iii)

https://sscstudy.com/
https://sscstudy.com/

80 CUET (UG) Section III : General Test

Now, on squaring Eq. (ii), 32 2 3


(4)2h = (Q r = 4 cm) πr × 8 = 144π
3 3
( l2 + b2 + h 2 )2 = (5 5 )2 2
⇒ h = cm ∴ r = 3 cm
l2 + b2 + h 2 = 125 3
4 20. Volume of the block left
Now putting the obtaining value 14. Volume of the sphere = πr3  1 22 
in Eq. (iii), we get 3 = (10 × 5 × 4) −  × × 3 × 3 × 7
3 7 
125 + 2(lb + bh + hl) = 361 and volume of the cylinder
= πr 2h = 200 − 66 = 134 m3
2(lb + bh + hl) = 361 − 125 4 134 × 100
Whole surface area of cuboid Q πr 2h = πr3 ∴Required percentage =
3 200
= 236 m 2 4 = 67%
∴ h= r
9. Total number of spherical 3
21. Let the edges of the three cubes
bullets 15. Let r1 and r2 be the radii of be 3x cm, 4x cm and 5x cm.
Volume of solid cube spheres. ∴ Volume of the resulting cube
=
Volume of 1 bullet 4πr12 9 = (3x)3 + (4x)3 + (5x)3
44 × 44 × 44 ∴ =
= = 2541 4πr22 16 = 216 x3
4 22
× ×2 ×2 ×2 r12 9 r 3 ∴Edge of the resulting cube
3 7 ⇒ = ⇒ 1 =
r22 16 r2 4 = 6x cm
10. Length of the longest pole ∴Diagonal the resulting cube
4 3 4 3
∴ πr1 : πr2 = r13 : r23 = 6x 3 cm
= 122 + 82 + 92 3 3
= 144 + 64 + 81 = 33 : 43 6x 3 = 12 3
= 289 = 17 m = 27 : 64 ∴ x=
12 3
= 2 cm
1 6 3
11. Volume of the new cube 16. Volume of the cone = π (3)2 × 5
3 ∴The edges of three cubes are
= (9)3 + (12)3 + (15)3
= 15π cm3 6 cm, 8 cm and 10 cm.
= 729 + 1728 + 3375 = 5832 cm3
and volume of the cylinder 22. Q Diameter = 14 mm
∴One side of the new cube
= π (2)2 × 4 ∴ Diameter of new cylinder
= 18 cm
= 16 π cm3 = 7 mm
12. Let the original radius and ∴ Required ratio = 15 : 16 ∴ Length of new cylinder
height be r and h, respectively.
17. For the least number of cubes, = 50 mm
∴ Volume of the original cone the edge of the cube will be 2
1  7
= πr 2h largest. π   × 50 = x
 2
3 ∴ HCF of 6, 42 and 45 = 3 ∴ x = 1925 mm3
and increased volume ∴ Volume of 1 cube = 3 × 3 × 3
1  110r 
2 23. Let the radius of each solid be r
 110h  = 27 cm3
= π  ×  and the heights of the cylinder
3  100   100  ∴ Required number of cubes and the cone be h1 and h2
1 6 × 42 × 45
= π × 1.331 r 2h = = 420 respectively.
3 27 2πr 2 + 2πrh1 = 3πr 2
∴Percentage of increase 18. Let the diameter and height of
1 2 r
πr h (1.331 − 1) one bucket 3x m and h m. ∴ h1 =
2
=3 × 100 ∴The diameter and height of
1 2 and also 3πr 2 = πr h22 + r 2 + πr 2
πr h other bucket will x m and 3 h m.
3 2 2
= 33.1%  3x  x ⇒ πr h22 + r 2 = 2π r 2
∴V1 : V 2 = π   × h : π   ⋅ 3 h
 2  2
13. Volume of the sphere ⇒ h22 + r 2 = 4 r 2
4 32π 2 2
9 πx h 3 πx h ⇒ h22 = 3 r 2
= π (2)3 = cm3 = : = 3 :1
3 3 4 4 ∴ h2 = 3 r
Let the water raised h m when a r
19. Volume of the cone
sphere is immersed in it. h1
1
= π × 36 × 12 ∴ = 2 = 1 :2 3
Volume of water (cylinder) h2 3r
3
= Volume of sphere
= 144π cm3 24. The ratio of their total surface
32π (3)2
π r 2h = If the radius of the sphere be r
areas = = 9 :1
3 cm, then (1)2

https://sscstudy.com/
https://sscstudy.com/

NUMERICAL ABILITY 81

CHAPTER 16

Algebra
Polynomial
Ex. 1 Find the quotient and the remainder when x4 + 1
f ( x ) = a0x n + a1x n − 1 +..... + an ( a0 ≠ 0) is called a
is divide by x − 1.
polynomial in variable x, where a0 , a1 , ... , an are real
(a) x3 + x2 + x + 1, 2 (b) x3 + x2 − x + 1, 2
numbers and n is a non-negative integer, is called
(c) x3 + x2 − x + 1, 3 (d) None of these
degree of polynomial.
e.g. Polynomial ( x − a ) is a degree of 1 and polynomial Sol. (a) Using long division method,
x 2 − 7x + 12 is a degree of 2. x3 + x2 + x + 1
x − 1) x4 + 1
Note ¿ If f (x) = 0, then it is said to be polynomial equation. x4 − x3
− +
Fundamental Operations on Polynomials x3 + 1
Some operations based on polynomials are discussed x3 − x2
below − +

1. Addition of Polynomials Polynomials can be x2 + 1


added by arranging their like terms and combining x2 − x
− +
them. x+ 1
2. Subtraction of Polynomials Polynomials can be x −1
subtracted by arranging their like terms and by − +
changing sign of each term of the polynomial to be 2
subtracted and then added. 3 2
Hence, quotient = x + x + x + 1and remainder = 2.
3. Multiplication of Polynomials We know that,
(i) the product of two factors with like signs is Linear Equations
positive and product of unlike signs is negative. In One Variable
(ii) if x is any variable and m , n are positive The expression of the form ax + b = 0, where a and b are
integers, then x m × x n = x m + n real numbers and a ≠ 0, is a linear polynomial of one
Thus, x3 × x 6 = x (3 + 6) = x 9. variable and equation involving only linear polynomial
4. Division of a Polynomial by a Polynomial are called linear equations of one variable.
The following steps are given below e.g. 5x + 8 = 9 − x is a linear equation in one variable.
Firstly, arrange the terms of the dividend and
● ● Graph of linear equation of one variable is a straight
divisor in descending order of their degrees. line, which is either parallel to the horizontal and
Divide the first term of the dividend by the first
● vertical axis.
term of the divisor to obtain the first term of the ● Linear equation in one variable has unique solution.
quotient.
Multiply all the terms of the divisor by the first
● In Two Variables
term of the quotient and subtract the result from An equation of the form ax + by + c = 0, where a , b, c,
the dividend. ∈ R , a ≠ 0, b ≠ 0 and here x , y are variables is called a
Consider the remainder (if any) as a new

linear equation in two variables.
dividend and proceed as before. e.g. 2x + 3 y = 5, 2 x + 3 y = 0,
Repeat this process till we obtain a remainder

which is either 0 or a polynomial of degree less 2a + 3b = 0


than the degree of the divisor. are linear equations in two variables.

https://sscstudy.com/
https://sscstudy.com/

82 CUET (UG) Section III : General Test

The linear equation in two variables ax + by + c = 0 a1 b1 c1


= = , then system has infinite solutions and

● If
has an infinite number of solutions. a2 b2 c2
● The graph of equation ax + by + c = 0 is a straight line, represents overlapping lines.
so it is called as linear equation. Inconsistent System The given system will be
● Every point on graph of ax + by + c = 0 gives it a b c
inconsistent, if 1 = 1 ≠ 1 and do not have any solution.
solution. a2 b2 c2
2 3 5 Hence, it represent a pair of parallel lines.
Ex. 2 Solve + = , where
x−3 x− 4 x
x ≠ 3, x ≠ 4 and x ≠ 0 Ex. 4 Which option is correct, for the following pair of
1 1 equations?
(a) 3 (b) 3
3 2 x + 2 y − 4 = 0 and 3 x + 6 y − 12 = 0
1 (a) Consistent
(c) 3 (d) None of these
4 (b) Consistent (dependent)
2 3 5 (c) Inconsistent
Sol. (a) Given that, + =
x−3 x−4 x (d) None of these
2(x − 4) + 3(x − 3) 5 Sol. (b) Given, pair of linear equations is
⇒ =
(x − 3) (x − 4) x x + 2 y − 4 = 0 and 3x + 6 y − 12 = 0
⇒ (5x − 17) x = 5 (x2 − 7x + 12) On comparing with standard form of pair of linear
⇒ 5x2 − 17x = 5x2 − 35x + 60 equations,
60 1 we get
⇒ 18x = 60 ⇒ x = =3
18 3 a1 = 1, b1 = 2, c1 = − 4
1
So, x = 3 is a solution of the given equation. and a2 = 3, b2 = 6, c2 = − 12
3 a1 1 b1 2 1
Now, = , = =
a2 3 b2 6 3
Ex. 3 The length of a rectangle is 8 cm more than its c1 −4 1
breadth. If the perimeter of the rectangle is 68 cm, its and = =
c2 − 12 3
length and breadth are respectively
(a) 21 cm, 13 cm (b) 22 cm, 21 cm a1 b1 c1 1
Clearly, = = =
(c) 23 cm, 13 cm (d) 24 cm, 21 cm a2 b2 c2 3
Sol. (a) Let the breadth of the rectangle be x cm. Hence, the given pair of linear equations is consistent
(dependent).
Then, its length = (x + 8) cm
∴ Perimeter of rectangle = 2 [x + (x + 8)] Factor and Factorisation
= 4x + 16 ● A polynomial g( x ) is called a factor of polynomial p( x ),
According to the given condition, 4x + 16 = 68 if g ( x ) divides p( x ) exactly.
⇒ 4x = 52 ⇒ x = 13 ● To express polynomial as the product of polynomials of
∴Breadth of rectangle = 13 cm degree less than that of the given polynomial is called
and length of rectangle = 13 + 8 = 21cm . as factorisation.

Consistency of the System of Linear Equations Factorisation by Common Factors


A set of linear equations is said to be consistent, if there A factor which occurs in each terms, is called the
exists atleast one solution for these equation. A set of common factor. e.g. Factorise 16x 2 y + 4xy
linear equations is said to be inconsistent, if there is no We have, 16x 2 y = 2 × 2 × 2 × 2 × x × x × y and
solution for these equation. 4xy = 2 × 2 × x × y
Here, 2 × 2 × x × y is common in these two terms.
Let us consider a system of two linear equations as
shown. Factorisation by Splitting Middle Term
a1x + b1 y + c1 = 0 and a2x + b2 y + c2 = 0.
Let factors of the quadratic polynomial ax 2 + bx + c be
Consistent System The above system will be ( px + q ) and (r x + s). Then, ax 2 + bx + c = ( px + q ) (r x + s)
consistent,
a b a b c = prx 2 + ( ps + qr )x + qs
if 1 ≠ 1 or 1 = 1 = 1
a2 b2 a2 b2 c2 On comparing the coefficients of x 2, x and constant terms
a1 b1 from both sides, we get a = pr, b = ps + qr and c = qs.
● If ≠ , then system has unique solution and
a2 b2 Here, b is the sum of two numbers ps and qr, whose
represents a pair of intersecting lines. product is ( ps)( qr ) = ( pr )( qs ) = ac.

https://sscstudy.com/
https://sscstudy.com/

NUMERICAL ABILITY 83

Thus, to factorise ax 2 + bx + c, write b as the sum of two ● If p( x ) is divided by ( x − α ), then remainder is f(α ).
numbers, whose product is ac. Note ¿ Remainder can be evaluated by substituting, x = α in p(x).
2 2
Note To factorise ax + bx − c and ax − bx − c, write b as the
2. Factor Theorem
difference of two numbers whose product is (– ac).
● Let p( x ) be a polynomial in x of degree not less than
Ex. 5 Factors of 2x + 7 x + 3 are2
one and α be a real number.
(a) (x + 2) (x + 1) (b) (2x + 1) (x + 3) ● If p(α ) = 0, then ( x − α ) is factor of p( x ).
(c) (x + 3) (2x − 1) (d) (2x − 2) (x − 3) Note If (x − α ) is a factor of p (x ), then p(α ) = 0.
Sol. (b) Given polynomial is 2x2 + 7x + 3
Ex. 7 The value of p, if (2x − 1) is a factor of
On comparing with ax2 + bx + c, we get
2x 3 + px2 + 11x + p + 3, is
a = 2, b = 7 (a) − 7 (b) 7
and c=3 (c) − 6 (d) 5
Now, ac = 2 × 3 = 6 Sol. (a) Let q (x) = 2x3 + px2 + 11x + p + 3
So, all possible pairs of factors of 6 are 1 and 6, 2 and 3. If q (x) is divisible by 2x − 1, then (2x − 1) is a factor of q (x).
Clearly, pair 1 and 6 gives 1
Consider 2x − 1 = 0 ⇒ x =
1+ 6 = 7 = b 2
∴ 2x + 7x + 3 = 2x2 + (1+ 6)x + 3
2 1
On putting x = in q(x), we have
= 2x2 + x + 6x + 3 = x(2x + 1) + 3(2x + 1) 2
3 2
q   = 2 ×   + p   + 11  + p + 3 = 0
= (2x + 1) (x + 3) 1 1 1 1
 2  2  2  2
Factorisation by Algebraic Identities 1 1 11
⇒ 2× + p × + + p + 3 = 0
Sometimes, we do a factorisation with the help of 8 4 2
algebraic identities, which are given below. 1 p 11
⇒ + + + p+ 3 = 0
1. ( a 2 − b2 ) = ( a + b)( a − b) 4 4 2
2. ( a + b)2 = a 2 + b2 + 2ab and ( a − b)2 = a 2 + b2 − 2ab 1+ p + 22 + 4 p + 12
⇒ =0
2 2 4
3. ( a + b) − ( a − b) = 4ab and
⇒ 5 p + 35 = 0
( a + b)2 + ( a − b)2 = 2 ( a 2 + b2 )
⇒ 5 p = − 35
4. ( a + b + c)2 = a 2 + b2 + c2 + 2 ( ab + bc + ca )
∴ p=−7
5. ( a + b)3 = a3 + b3 + 3ab ( a + b)
6. ( a − b)3 = a3 − b3 − 3ab ( a − b) Quadratic Equation
7. ( a3 + b3 ) = ( a + b)( a 2 + b2 − ab)
● The second degree equation of polynomial is called
quadratic equation.
3 3 2 2
8. ( a − b ) = ( a − b)( a + b + ab) ● The general quadratic equation is given by
9. a3 + b3 + c3 − 3abc ax 2 + bx + c = 0, where a , b, c are real numbers and a ≠ 0.
= ( a + b + c)( a 2 + b2 + c2 − ab − bc − ac)
10. If a + b + c = 0, then a3 + b3 + c3 = 3abc
Roots of a Quadratic Equation
A value of a variable which satisfies the particular
Ex. 6 Factorise 8 a 3 − 343 b3 quadratic equation is called root of that equation or
(a) (2a + 7b) (4a 2 + 14ab + 49b2 ) solution of the equation.
(b) (2a − 7b) (4a 2 + 14ab + 49b2 ) e.g. Let the equation is x 2 − 6x + 8 = 0.
(c) (2a − 7b) (4a 2 − 14ab + 49b2 ) Here, we take x = 2 , then 22 − 6 ( 2) + 8 = 0
(d) None of the above
So, x = 2 is a root of the quadratic equation.
Sol. (b) 8a3 − 343b3 = (2a )3 − (7b)3
= (2a − 7b) [(2a )2 + (2a ) (7b) + (7b)2 ] Solution of a Quadratic Equation
= (2a − 7b) (4a 2 + 14ab + 49b2 ) The solution of a quadratic equation can be find by two
methods.
Factorisation by Using Theorems 1. By Factorisation Method Let the quadratic
equation be ax 2 + bx + c = 0. If the factors of
1. Remainder Theorem ax 2 + bx + c are ( x + α ) ( x + β ), then the solution is
● Let p( x ) be the polynomial in x of degree not less than x = − α , − β.
one and α be a real number.

https://sscstudy.com/
https://sscstudy.com/

84 CUET (UG) Section III : General Test

2. By Quadratic Formula If given equation is Sol. (c) Given equation is 2x2 + 14x + 9 = 0.
ax 2 + bx + c = 0, then roots of a quadratic equation
Then,
can be determined by the formula
− 14 ± (14)2 − 4 (2) (9)
− b ± b2 − 4ac x=
x= 2 (2)
2a
− 14 ± 196 − 72 − 14 ± 124
= =
This formula is known as Sridharacharya Formula, 4 4
where ( b2 − 4ac) is the discriminant (4) of the equation. − 14 ± 2 31 − 7 ± 31
= =
4 2
Nature of Roots of a Quadratic Equation − 7 + 31 − 7 − 31
∴The roots are and ⋅
Let D = b2 − 4ac be the discriminant of the quadratic 2 2
equation ax 2 + bx + c = 0.
Ex. 9 If  a −  = 6, then ( a4 + 1 / a4 ) = ?
1
1. If D > 0, then the two roots are real and unequal.  a
2. If D = 0, then the two roots are real and equal. (a) 1444 (b)38
3. If D < 0, then there are no real roots. (c) 34 (d) 1442
1
4. If D > 0 and D is perfect square, then roots are Sol. (d) a − =6
a
rational. 2
On squaring both side  a −  = (6)2
1
5. If D > 0 and D is not a perfect square, then roots  a
are irrational.
1
Note ¿ If one of the roots of the quadratic equation is a + b, ⇒ a 2 + 2 − 2 = 36
then its another root will be a − b. a
1
⇒ a 2 + 2 = 38
Sum and Products of the Roots a
Let α , β be the roots of the equation Again squaring both side
2
ax 2 + bx + c = 0 ⇒  a 2 + 1  = 382
 
b  a2 
1. The sum of the roots, α + β = − 1
a ⇒ a 4 + 4 + 2 = 1444
c a
2. The product of the roots , α ⋅ β = 1
a ⇒ a 4 + 4 = 1442
a
Formation of a Quadratic Equation
Ex. 10 Divide 36 into two parts, such that 5 times of
If the roots of equation are given to us say α and β, then the first part is more than 8 times of the second part by
S = Sum of roots = α + β 24?
and P = Product of roots = αβ (a) 20, 16 (b) 24, 12
(c) 26, 10 (d) 22,14
∴ The quadratic equation will be
Sol. (b) Let two parts are ‘x’ and ‘y’
x 2 − (α + β )x + αβ = 0 or x 2 − Sx + P = 0
then 5x − 8 y = 24 …(i)
2
Ex. 8 Solve the equation 2x + 14 x + 9 = 0 . and x + y = 36
−7 + 31 −7 − 31 On multiplying with 8 both side, 8x + 8 y = 288
(a) (b)
2 2 Adding in Eq. (i)
−7 + 31 −7 − 31 13x = 288 + 24 ⇒ 13x = 312
(c) , (d) None of these
2 2 x = 24 and y = 36 − 24 = 12

https://sscstudy.com/
https://sscstudy.com/

NUMERICAL ABILITY 85

Practice Questions
1. The degree of polynomial 336x2 + 210x + 42 is 11. If ax + by = 3, bx − ay = 4 and x2 + y2 = 1, then the
(a) 3 (b) 4 value of a2 + b2 is
(c) 42 (d) 2 (a) 25 (b) 26
(c) 27 (d) 28
2. If 2x2 + ax + b, when divided by x − 3, leaves a
remainder of 31 and x2 + bx + a, when divided by 12. If 3 x − 2 = 2 3 + 4, then the value of x is
x − 3, leaves a remainder of 24, then a + b equals (a) 2 (1 − 3 ) (b) 2 (1 + 3 )
(a) −23 (b) −7 (c) 1 + 3 (d) 1 − 3
(c) 7 (d) 23
3 x + 6 11x − 8 x 3x x + 7
3. If a + b + c = 2s, then 13. If − + = − , then the value
8 24 3 4 24
[( s − a)2 + ( s − b)2 + ( s − c)2 + s2 ] = ?
of x is
(a) (a 2 + b2 + c2 ) (b) (4s2 − a 2 − b2 − c2 ) 3
(a) − 3 (b)
(c) (s2 − a 2 − b2 − c2 ) (d) (s2 + a 2 + b2 + c2 ) 2
1
4. In a test, ( +5) marks are given for every correct (c) 3 (d)
3
answer and ( −2) marks are given for every incorrect
answer, Rakesh answered all the questions and 14. The value of y in the solution of the equation
scored 30 marks though he got 10 correct answers. 2x + y
= 2x − y
= 8 is
How many incorrect answers had he attempted? 1
(a) 0 (b)
(a) 10 (b) 12 4
(c) −10 (d) −12 1 3
(c) (d)
2 7
5. The sum of a number and its reciprocal is −12.
What would be the sum of cubes of the two (the 15. If 5 is added to twice of a number it becomes 6,
number and its reciprocal)? then the number is
(a) 0.5 (b) 5
(a) −1764 (b) −1728
(c) 0.25 (d) None of these
(c) −1681 (d) −1692
16. The sum of the two numbers is 11 and their
3 −1
6. If = a + b 3 ; then a2 + b2 = ? product is 30, then the numbers are
3 +1 (a) 8, 3 (b) 9, 2
(a) 8 (b) 7 (c) 7, 4 (d) 6, 5
(c) 5 (d) 6 17. If one number is thrice the other and their sum is
20, then the numbers are
7. Solve for x;x ∈N: ( x − 4)2 − 36 = 0 . (a) 5, 15 (b) 4, 12
(a) −2 (b) −10
(c) 3, 9 (d) None of these
(c) 10 (d) 2
18. If x + y = 7 and 3 x − 2 y = 11, then
8. Find the values of k for which x2 + 5kx + k2 + 5 is (a) x = 2 , y = 5
exactly divisible by x + 2 but not divisible by x + 3. (b) x = 5, y = 5
(a) Both 1 and 9 (b) 1
(c) x = 5, y = 2
(c) Neither 1 nor 9 (d) 9
(d) x = 0, y = 3
9. If y = − 1, then the value of 1 + (1 / y) + (1 / y2 ) + (1 / y 3)
19. The solution of the system of linear equations
+ (1 / y4 ) + (1 / y5 ) is
(a) −1 (b) 0 0 .4 x + 0 .3 y = 1 . 7 and 0 . 7 x − 0 . 2 y = 0 . 8 is
(c) 1 (d) 2 (a) x = 3, y = 2 (b) x = 2 , y = − 3
(c) x = 2 , y = 3 (d) None of these
10. If x and y are positive with x − y = 2 and xy = 24,
20. If  x +  :  x −  = 5 : 4, then the value of x is
1 1 1 1
then + is equal to  x  x
x y
(a) 0 (b) ± 1
5 1 1 25
(a) (b) (c) (d) (c) ± 2 (d) ± 3
12 12 6 6

ANSWERS
1. (d) 2. (c) 3. (a) 4. (a) 5. (d) 6. (c) 7. (c) 8. (d) 9. (b) 10. (a)
11. (a) 12. (b) 13. (c) 14. (a) 15. (a) 16. (d) 17. (a) 18. (c) 19. (c) 20. (d)

https://sscstudy.com/
https://sscstudy.com/

86 CUET (UG) Section III : General Test

Hints & Solutions


1. We know, degree of ax2 + bx + c 1
x3 + + 3(−12) = −1728 10. We have,
is 2. x3 x − y =2 …(i)
1
So, degree of 336x2 + 210x + 42 is 2. x3 + 3 − 36 = −1728 xy = 24 …(ii)
x
2. If 2x2 + ax + b in divided by (x − 3) ⇒ y ( y + 2) = 24
1
x3 + 3 = −1692 [Q from Eq. (i). x = y + 2]
then remainder is 31 and x
2x2 + xa + b − 31 in divisible by ⇒ y2 + 2 y − 24 = 0
3 −1
(x − 3) 6. =a+b 3 ⇒ y = 4, y = − 6 but x and y are
3 −1 positive, so y = 4
thus x = 3 is a solution of
2x2 + ax + b − 31 By using componendo and and x = y + 2 =4 + 2 ⇒ x =6
dividendo 1 1 1 1 5
2(3)2 + 3a + b − 3 = 0 ∴ + = + =
( 3 − 1) ( 3 − 1) x y 4 6 12
18 + 3a + b − 31 = 0 =a+b 3
( 3 + 1 ) ( 3 − 1)
3a + b = 13 …(i) 11. Given equations are
( 3 − 1 )2
Now if x2 + bx + a in divided by =a+b 3 ax + by = 3 …(i)
(x − 3) and remainder is 24 then 3 −1
[Q a 2 − b2 = (a + b)(a + b)] bx − ay = 4 …(ii)
x2 + bx + a − 24 = 0 3 + 1 −2 3 and x 2 + y2 = 1 …(iii)
=a+b 3
(3)2 + 3x + a − 24 = 0 [Q x = 3] 2 On squaring Eqs. (i) and (ii) and
9 + 3x + a − 24 = 0 4 −2 3 then adding, we get
=a+b 3
3x + a = 15 …(ii) 2 a 2x2 + b2y2 + 2ax by + a 2y2 + b2x2
On Solving Eqs. (i) and (ii) b = 4 2− 3 = a + b 3 − 2ax by = 9 + 16
on putting Eq. (i) a = 3 thus a = 2 and b = −1 ⇒ a 2 (x2 + y2) + b2 (x2 + y2) + 2ax by
a + b =4+3= 7
Thus, a 2 + b2 = 22 + (−1)2 −2axby = 25
3. [(s − a )2 + (s − b)2 + (s − c)2 + s2] a 2 + b2 = 5 ⇒ a 2 × 1 + b2 × 1 = 25
⇒ s2 + a 2 − 2sa + s2 + b2 − 2sb + s2 7. (x − 4)2 − 36 = 0 ⇒ (x − 4)2 = 36 [put x2 + y2 = 1]
2 2
+ c2 − 2sc + s2 (x − 4)2 = (6)2 ∴ a + b = 25
[Q (a − b)2 = a 2 + b2 − 2b] On taking square root both side 12. Given that, 3 x − 2 = 2 3 + 4
⇒ 4s − 2(sa + sb + sc) + a 2 + b2 + c2
2
x − 4 = 6 ⇒ x = 10
⇒ 3 x=2 3 + 6
⇒ 4s2 − 2s × 2s + a 2 + b2 + c2 8. If x2 + 5kx + k2 + 5 is divisible by 2 3+6
[Q a + b + c = 2s Given ] (x + 2) then x = −2 is a solution then ⇒ x=
3
⇒ 4s − 4s + a 2 + b2 + c2
2 2
(−2)2 + 5(−2)k + k2 + 5 = 0 2 3+6 3
⇒ a 2 + b2 + c2 k2 − 10k + 9 = 0 ⇒ x= ×
3 3
4. Let he attempted ‘x’ correct answer (k − 1)(k − 9) = 0
⇒ x = 2 (1 + 3 )
and y incorrect answer, then k = 1 and 9 also given that (x + 3)
5x − 2 y = 30 in not a solution then 13. Given that,
3x + 6 11x − 8 x 3x x + 7
5(10) − 2 y = 30 [x = 10 [given ]] (−3)2 + 5(−3)(k) + k2 + 5 ≠ 0 − + = −
8 24 3 4 24
50 − 2 y = 30 9 + k2 − 15k + 5 ≠ 0
9x + 18 − 11x + 8 + 8x
50 − 30 k2 − 14k − k + 14 ≠ 0 ∴
y= ⇒ y = 10 24
2 (k − 1)(k − 14) ≠ 0 18x − x − 7
k ≠ 1 and 14; thus only k = 9 =
5. Let the number is x then 24
1 9. Now, ⇒ 6x + 26 = 17x − 7
x + = −12
x  1  1   1   1   1  ⇒ 11x = 33 ⇒ x = 3
1 +   +  2 +  3  +  4  +  5 
On taking cube both side  y  y   y   y   y 
1 14. Since, 2x + y = 8 and 2x − y = 8
(x + )3 = (−12)3  1   1   1 
x =1+  + +  ⇒ x+ y=
3
 (−1)  (−1)2  (−1)3 
3 1 1 2
x + 3 + 3(x + ) = −1728  1   1  3
x x + +  [Q put y = − 1] and x− y=
 (−1)4   (−1)5  2
[Q (a + b)3 = a3 + b3 + 3ab (a + b)] = 1 −1 + 1 −1 + 1 −1 =0 ⇒ 2 x + 2y = 3

https://sscstudy.com/
https://sscstudy.com/

NUMERICAL ABILITY 87

and 2 x − 2y = 3 ⇒ 3 y + y = 20 ∴ 4x + 3 y = 17 …(i)
3 ⇒ 4 y = 20 and 7x − 2 y = 8 …(ii)
On solving, we get x = , y = 0
2 ⇒ y = 5 and x = 15 On solving Eqs. (i) and (ii), we get
15. Let the number be x. Hence, two numbers are 5 and 15. x = 2 and y = 3
1 1
∴ 2x+ 5 =6 ⇒ x= = 0.5 18. Given equations are x+
2
x + y = 7 and 3x − 2 y = 11 20. Given, x =5
1 4
16. Let the two numbers be x and y. On multiplying Eq. (i) by 2 and x−
x
∴ x + y = 11 and xy = 30 …(i) then adding Eq. (ii), we get
 1  1
2 2
Now, (x − y) = (x + y) − 4xy 5x = 25 ⇒ 4 × x +  =5 x − 
 x  x
= (11)2 − 4 × 30 ⇒ x=5
4 5
∴ 5+ y=7 ⇒ 4x + = 5x −
= 121 − 120 = 1 x x
⇒ x− y=1 …(ii) ⇒ y=2 4 5
⇒ 5x − 4x = +
On solving Eqs. (i) and (ii), we get 19. Given system of linear equations x x
x = 6, y = 5 are 9
⇒ x = ⇒ x2 = 9
4x 3 y 17 7x 2 y 8 x
17. Let the two numbers be x and y. + = and − =
10 10 10 10 10 10 ∴ x= ± 9 =±3
∴ x = 3 y and x + y = 20

https://sscstudy.com/
https://sscstudy.com/

88 CUET (UG) Section III : General Test

CHAPTER 17

Geometry
Acute Angled Triangle A triangle each of whose angle
Plane Geometry is less than 90° is called an acute angled triangle.
Plane geometry is about flat shapes like line, circle and
triangle etc. These types of shapes can be easily drawn 3. Circle
on a piece of paper. A circle is a set of points which are equidistant from a
given point. The given point is known as the centre of
1. Lines and Angles that circle.
Two lines in the same plane are said to be parallel, if 1 Side
they never meet. A line which cuts a pair of parallel line Inradius (OD) = × Height = ,
3 2 3
is called a transversal.

E
1 O
A 2 B
4 3
D
C 5 6 D 2 Side
8
Circumradius (OA) = × Height =
7 3 3
F A

Here, two parallel lines AB and CD are cut by a


transversal i. e. , EF. Then, O
The corresponding angles are
∠1 = ∠ 5, ∠2 = ∠6, ∠4 = ∠8 and ∠3 = ∠7.
The alternate angles are ∠1 = ∠7, ∠2 = ∠8, ∠3 = ∠5
and ∠4 = ∠6. 4. Quadrilateral
2. Triangles It is a plane figure bounded by four straight lines. The sum
of the internal angles of a quadrilateral is equal to 360°.
A figure bounded by three straight lines is called a
triangle. The small of all interior angles of a triangle is 180°. Parallelogram
Types of Triangle A quadrilateral in which the opposite sides are equal
Equilateral Triangle A triangle having all sides equal and parallel, is called a parallelogram.
is called an equilateral triangle and each angle equal D C
to 60°.
Breadth

Scalene Triangle A triangle having all sides of different O


(B)

length is called a scalene triangle.


Isosceles Triangle A triangle having two sides equal is
called an isosceles triangle. A Length (L) B
Right Angled Triangle A triangle one of whose angles The opposite angles are equal in magnitudes.
measures 90° is called a right angled triangle.
The digonals of a parallelogram are not equal in
Obtuse Angled Triangle A triangle one of whose angles
magnitudes, but they bisect each other.
lies between 90° and 180° is called an obtuse angled
AC ≠ BD but AO = OC and OB = OD.
triangle.

https://sscstudy.com/
https://sscstudy.com/

NUMERICAL ABILITY 89

Rectangle The coordinates of any point on X-axis are of the form


A parallelogram in which the adjacent sides are ( x , 0).
perpendicular to each other, is called a rectangle. The coordinates of any point on Y -axis are of the form
The diagonals of a rectangle are of equal magnitudes ( 0, y ).
and bisect each other i. e. , AC = BD and
Distance Formula
OA = OB = OC = OD.
Distance between two points
Square If A ( x1 , y1 ) and B( x2 , y2 ) are two points, then
A parallelogram in which all the sides are D C
Y B (x2, y2)
equal and perpendicular to each other, is
called a square. O A C
The diagonals bisect each other at right (x1, y1)
angles and form four isosceles right
A B
angled triangles. X
O D E
The diagonals of a square are of equal mangnitudes
i. e. , AC = BD.
AB = ( x2 − x1 )2 + ( y2 − y1 )2 = ( x1 − x2 )2 + ( y1 − y2 )2
Polygon For example The distance between A( 1, 2) and B( 5, 6)
Regular polygon A polygon in which all the sides are is
equal and also all the interior angles are equal, is called
AB = ( 5 − 1)2 + ( 6 − 2)2 = 42 + 42
a regular polygon.
Sum of all interior angles = ( n − 2) × 180° = 32 = 4 2 units
= ( 2n − 4) × 90° Area of a Triangle
Each interior angle = 180° − Exterior angle If A( x1 , y1 ), B( x2 , y2 ) and C( x3 , y3 ) are three vertices of a
 360°  ∆ABC, then its area is given by
Each exterior angle =   (in degrees)
 Number of sides 1
Area of ∆ ABC = [x1 ( y2 − y3 ) + x2( y3 − y1 ) + x3 ( y1 − y2 )]
2
Sum of all exterior angle = 360° (always constant).
n( n − 3) Y
Number of diagonals of polygon of n sides =
2 4 B (0, 4)
3
Coordinate Geometry 2
It is a system of geometry, where the position of points 1
A (5, 0)
on the plane is described by using an ordered pair of O X
numbers. 1 2 3 4 5
(0, 0)

Quadrants For example If we have to find the area of a triangle


Y having the vertices (0, 0), (5, 0) and (0, 4), then
The X and Y-axes divide the
cartesian plane into four regions 2nd 1st 1
quadrant quadrant ∴ Area of triangle = { 0 ( 0 − 4) + 5 ( 4 − 0) + 0 ( 0 − 0)}
referred as quadrants. 2
X′ X
The table of sign conventions of O 1
3rd 4th
coordinates in various quadrants quadrant quadrant
= × 20 = 10 sq units
2
is given below
Y′ Q ( x1 , y1 ) = ( 0, 0), ( x2 , y2 ) = ( 5, 0) and ( x3 , y3 ) = ( 0, 4)

Quadrant Region Sign of (x, y) Example Collinearity of Three Points


I XOY (+ , + ) (2, 3) Three points A ( x 1 , y1 ), B( x2 , y2 ) and C( x3 , y3 ) are
collinear, if
II YOX ′ (− , + ) (−2, 4)
(i) Area of ∆ABC is 0, i.e.
III X ′ OY ′ (− , − ) (−1, − 2)
x1( y2 − y3 ) + x2( y3 − y1 ) + x3 ( y1 − y2 ) = 0.
IV Y ′ OX (+ , − ) (1, − 3)
(ii) Slope of AB = Slope of BC = Slope of AC
The coordinates of point O (origin) are taken as ( 0, 0).

https://sscstudy.com/
https://sscstudy.com/

90 CUET (UG) Section III : General Test

(iii) Distance between A and B + Distance between B Section formulae


and C = Distance between A and C Let A( x1 , y1 ) and B( x2 , y2 ) be two points on the cartesian
plane. Let point P ( x , y ) divides the line AB in the ratio of
Centroid of a Triangle m : n internally.
Centroid is the point of A (x1, y1)
mx2 + nx1 my2 + ny1
intersection of all the three Then, x= ,y=
medians of a triangle. G m+n m+n
If A( x1 , y1 ), B( x2 , y2 ) and If P divides AB externally, then
C( x3 , y3 ) are the vertices of B C mx2 − nx1 my2 − ny1
x= ,y=
∆ABC, then the coordinates of (x2, y2) (x3, y3)
m−n m−n
its centroid are
If P is the mid-point of AB, then
1 1 
 3 ( x1 + x2 + x3 ), 3 ( y1 + y2 + y3 ). x=
x1 + x2
,y= 1
y + y2
2 2

Solved Examples
Ex. 1 A chord AB is drawn in a circle with centre O and Ex. 3 In the given figure, AB and CD are parallel lines.
radius 5 cm. If the shortest distance between centre and If ∠EGB = 50° , find ∠CHG.
chord is 4 cm, find the length of chord AB ? (1) 120° (2) 130° (3) 125° (4) 140°
(1) 6 cm (2) 5 cm (3) 4 cm (4) 3 cm
E
Sol. (1) In the adjoining figure AO = 5 cm (radius)
50°
A G B

H
O
C D
cm

4 cm
5

F
A x C x B
Sol. (2) ∠AGH = ∠EGB [Vertically opposite angles]
OC = 4 cm (shortest distance between centre and chord) ∠AGH = 50°
Let length of chord AB be 2x, then AC = x. Now, ∠ AGH + ∠ CHG = 180°
In ∆AOB, AO 2 = AC 2 + OC 2 [Interior angles on the same side of
⇒ (5)2 = x2 + (4)2 ⇒ 25 = x2 + 16 the transversal are supplementary.]
∴ x = 25 − 16 = 9 = 3 cm ∴ 50° + ∠ CHG = 180° ⇒ ∠ CHG = 180° − 50° = 130° .
∴ Length of chord AB = 2x = 2 × 3 = 6 cm
Ex 4. An angle θ° is one-fourth of its supplementary
Ex. 2 In the figure given, ∠ BAC : ∠ ABC = 2 : 3. Find the angle. What is the measure of the angle θ°?
(1) 36° (2) 34° (3) 32° (4) 31°
measure of ∠ ABC.
Sol. (1) If the sum of two angles is 180°, the angles are said
A to be supplementary.
∴ The supplementary angle of θ° is (180° − θ° ).
1
Given that, θ° = (180° − θ° )
4
⇒ 4 θ° = 180° − θ° ⇒ 5 θ° = 180°
120° 180°
⇒ θ° = = 36°
B C 5
(1) Obtuse angle (2) Acute angle
Ex. 5 The number of diagonals in a 27-sided polygon is
(3) Right angle (4) None of these (1) 324 (2) 325
Sol. (2) Let ∠ A = 2x and ∠ B = 3x (3) 322 (4) 320
Then, 2x + 3x = 120° Sol. (1) Number of diagonals of polygon of n sides
[Exterior angle is equal to the sum of the interior opposite (n )(n − 3)
=
angles] 2
⇒ 5x = 120° Number of diagonals of polygon of 27 sides
⇒ x = 24° 27 × 24
= = 324
∴ ∠ ABC = 3x = 3 × 24° = 72° 2

https://sscstudy.com/
https://sscstudy.com/

NUMERICAL ABILITY 91

Ex. 6 Find the distance between the points A ( − 6, 8) and Ex. 7 Find the area of ∆ABC, whose vertices are
B ( 4, − 8). A (8, − 4), B (3, 6) and C ( − 2, 4).
(1) 18.86 units (2) 16.76 units (1) 20 sq units (2) 30 sq units
(3) 11.77 units (4) 16.76 units (3) 35 sq units (4) 29 sq units
Solution (1) Here, A (− 6, 8) = A (x1 , y1 ) and Solution (2) Here, A (8, − 4), then x1 = 8 , y1 = − 4
B (4, − 8) = B (x2 , y2 ) B (3, 6), then x2 = 3, y2 = 6
So, x1 = − 6, y1 = 8, x2 = 4 and y2 = − 8 C (− 2, 4), then x3 = − 2, y3 = 4
∴ Required distance, ∴ Area of ∆ABC
2 2 1
AB = (x2 − x1 ) + ( y2 − y1 ) = {x1 ( y2 − y3 ) + x2 ( y3 − y1 ) + x3 ( y1 − y2 )}
2
= {4 − (− 6)}2 + (− 8 − 8)2 1
= { 8 (6 − 4) + 3 (4 − (− 4)) + (−2)(− 4 − 6)}
2
= (4 + 6)2 + (− 16)2 = 100 + 256
1 1
= {16 + 24 + 20} = × 60 = 30 sq units
= 356 −
~ 18.86 units 2 2

Practice Questions
1. In the given figure, if l ||m, then find the value of x 6. In the given figure, ABCD is a parallelogram in
which ∠BAD = 75° and ∠CBD = 60°.
(in degrees). Then, ∠BDC is equal to
A D C
l l
100°

O (x + 5)°

30°
m m
B 60°
75°
(1) 105° (2) 100°
(3) 110° (4) 115° A B

2. In a ∆ABC, ∠A = 90° , ∠C = 55° and AD ⊥ BC. What (1) 60° (2) 75° (3) 45° (4) 50°
is the value of ∠BAD? 7. A D
(1) 60° (2) 45° (3) 55° (4) 35° x y

3. In the figure given below, ∠PQR = 90° and QL is a


120º
median, PQ = 5 cm and QR = 12 cm. Then, QL is E F
equal to z
P
90º 50º
B C
5 cm

L
In the figure given above, ABCD is a trapezium.
EF is parallel to AD and BC. Then, ∠y is equal to
(1) 30° (2) 45° (3) 60° (4) 65°
Q 12 cm R 8. In the given figure, AB||CD. If ∠CAB = 80° and
(1) 5 cm (2) 5.5 cm ∠EFC = 25°, then ∠CEF is equal to
(3) 6 cm (4) 6.5 cm
4. An angle which is less than 360° and more than F
180°, is called B D
(1) a reflex angle (2) a straight angle 25°
80° C
(3) an acute angle (4) an obtuse angle
5. In a ∆ABC, ∠A : ∠B : ∠C = 2 : 4 : 3. The shortest side
A
and the longest side of the triangle are respectively
E
(1) AC and AB (2) AC and BC
(3) BC and AC (4) AB and AC (1) 65° (2) 55° (3) 45° (4) 75°

https://sscstudy.com/
https://sscstudy.com/

92 CUET (UG) Section III : General Test

9. AB is the diameter of a circle with centre O and P is 18. ∆XYZ is similar to ∆PQR. If ratio of perimeter of
a point on it. If ∠POA = 120°, then the value of ∆XYZ and perimeter of ∆PQR is 4 : 9 and if
∠PBO is PQ = 27 cm, then what is the length of XY (in cm)?
(1) 30° (2) 50° (3) 60° (4) 40° (1) 9 (2) 12 (3) 16 (4) 15

10. Three angles of a quadrilateral are 80°, 95° and 19. G is the centroid of the equilateral ∆ABC. If
112°. Its fourth angle is AB = 10 cm, then length of AG is
(1) 78° (2) 73° (3) 85° (4) 100° 5 3 10 3
(1) cm (2) cm (3) 5 3 cm (4) 10 3 cm
3 3
11. What is the value of θ ?
Information 20. If the angles of a triangle are in the ratio of 1 : 2 :
C 3, then find the value of the largest angle.
(I)
(1) 30° (2) 60° (3) 90° (4) 120°
30°
21. An angle is 10° more than one-third of its
O
complement. Find the greater angle.
θ (1) 30° (2) 60° (3) 45° (4) 75°
A B
22. If the distance between the points ( x, 0) and ( −7, 0)
is 10 units, then the possible values of x are
(II) 0 < θ < 90°
(1) 3 and 17 (2) − 3 and 17
(1) Neither I nor II is sufficient
(3) 3 and −17 (4) − 3 and −17
(2) Either I or II is sufficient
(3) Only I is sufficient 23. The distance between the points ( 4, − 8) and ( k, 0) is
(4) Only II is sufficient 10. Find k.
(1) k = 6 or −2 (2) k = 10 or −2
12. Find the area of the triangle formed with the three
(3) k = 10 or −4 (4) k = 6 or −4
straight lines represented by
(i) x + y = 0; (ii) 3 x = 5 y; and (iii) y = 3 x − 12 24. Coordinates of a point is (0, 1) and ordinate of an
(1) 15 units (2) 20 units (3) 12 units (4) 16 units another point is − 3. If distance between both the
points is 5, then abscissa of second point is
13. Find the ratio in which line 3 x + 2 y = 17 divides the
(1) 3 (2) − 3 (3) ± 3 (4) 1
line segment joined by points (2,5) and (5,2).
(1) 1 : 3 (2) 1 : 2 (3) 2 : 5 (4) 3 : 4 25. What is the reflection of the point (6, − 3) in the
line y = 2 ?
14. T U (1) (−2, − 3) (2) (6, 7)
(3) (−6, 7) (4) (− 2, 3)
26. If the point ( x, y) is equidistant from points (7, 1)
P Q R S and (3, 5), then find (x − y).
In the given diagram , TU || PS and points Q and (1) 2 (2) 4 (3) 6 (4) 8
R lie on PS. Also, 27. The vertices of a triangle are A( 4, 4), B (3, − 2) and
∠PQT = x ° , ∠RQT = ( x − 50) º and ∠TUR = ( x + 25) ° C ( − 3, 16). The area of the triangle is
(1) 30 sq units (2) 36 sq units
What is the measure of ∠URS?
(3) 27 sq units (4) 40 sq units
(1) 130° (2) 140° (3) 135° (4) 115°
28. Two vertices of an equilateral triangle are origin
15. The ratio of the measures of the interior angles of
and (4, 0) What is the area of the triangle?
a regular octagon to that of a regular dodecagon is
(1) 4 sq units (2) 3 sq units
(1) 8 : 12 (2) 12 : 8 (3) 9 : 10 (4) 4 : 5
(3) 4 3 sq units (4) 2 3 sq units
16. If one of the acute angles of a right-angled triangle
is 55°, what is the measure of the other acute 29. If the graph of the equation 2x + 3 y = 6 form a
angle? triangle with coordinates axes, then the area of
(1) 35° (2) 40° (3) 30° (4) 25° triangle will be
A (1) 2 sq units (2) 3 sq units
17. B (3) 6 sq units (4) 1 sq unit
C
E 30. If the points A (1, − 1), B ( 5, 2 ) and C( k, 5) are
collinear, then k equals
(1) 2 (2) 4 (3) 6 (4) 9
31. If two vertices of a triangle are (5, 4) and (–2, 4)
D
and centroid is (5, 6), then third vertex is
In the figure given above, ∠BAE = 30° , ∠ABE = 80° (1) (12, 10) (2) (10, 12)
and ∠DBE = 50°. What is measure of ∠BCE? (3) (− 10, 12) (4) (12, − 10)
(1) 20° (2) 10° (3) 25° (4) 5°

https://sscstudy.com/
https://sscstudy.com/

NUMERICAL ABILITY 93

32. A point C divides the line AB, where A(1, 3) and 35. The area of the triangle with vertices at
B (2, 7), in the ratio of 3 : 4. The coordinates of C ( a, b + c), ( b, c + a) and ( c, a + b) is
are (1) a − b − c (2) ab + bc + ca
(1)  , 5
5
(2) (− 2, − 9) (3)  , 5
3
(4) 
10 33  (3) 0 (4) a + b + c
, 
3  5   7 7
ANSWERS
33. Point A( 4, 2) divides segment BC in the ratio 2 : 5.
Coordinates of B are (2, 6) and C are (9, y). What is 1 (1) 2 (3) 3 (4) 4 (1) 5 (3)
the value of y ? 6 (3) 7 (3) 8 (2) 9 (3) 10 (2)
(1) 8 (2) – 8 (3) 6 (4) – 6 11 (3) 12 (3) 13 (2) 14 (2) 15 (3)
16 (1) 17 (1) 18 (2) 19 (2) 20 (3)
34. In what ratio, the line made by joining the points 21 (2) 22 (3) 23 (2) 24 (3) 25 (2)
A ( − 4, − 3) and B (5, 2) intersects X-axis? 26 (1) 27 (3) 28 (3) 29 (2) 30 (4)
(1) 3 : 2 (2) 2 : 3 31 (1) 32 (4) 33 (2) 34 (1) 35 (3)
(3) − 3 : 2 (4) − 2 : 3

Hints & Solutions


3. Given that, PQ = 5 cm, triangle = side opposite to the Coordinate of Vertex A (3,− 3)
longest angle = AC. = (x1 y1 )
QR = 12 cm and QL is a median.
P 6. ∠C = A = 75° Coordinate of vertex B (0, 0)
[opposite angles of parallelogram] = (x2 , y2 )
In ∆BCD, Coordinate of vertex C (5, 3) = (x3 , y3 )
L ∠CBD + ∠BCD + ∠BDC = 180° Area of ∆ABC
5
⇒ 60° + 75° + ∠BDC = 180° 1
= {x1 ( y2 − y3 ) + x2 ( y3 − y1 )
⇒ 135°+∠BDC = 180° 2
90º ∠BDC = 45° + x3 ( y1 − y2 )}
Q 12 R 1
8. Let ∠CEF = x° = {3(0 − 3) + 0(3 + 3) + 5(−3 − 0)}
PR 2
∴ PL = LR = ...(i) Now, AB ||CD and AF is a
2 1 1
transversal. = {−9 − 15} = × (−24) = 12 units
In ∆PQR, (PR )2 = (PQ )2 + (QR )2 2 2
[by Pythagoras theorem] ∴ ∠DCF = ∠CAB = 80°
[corresponding angles] 13. Let the given lives divides
= (5)2 + (12)2 m1 : m2 at point C
= 25 + 144 = 169 = (13)2 In ∆CEF, side EC has been
produced to D. ∴Coordinate of point C
⇒ PR = (13)2 ⇒ PR = 13
2
⇒ x + 25 = 80° 3x+2y=17
Now, by theorem, if L is the
mid-point of the hypotenuse PR of ⇒ x = 55°
a right angled ∆PQR, then 10. Let the fourth angle be x°. A m1 m2 B
1 1 80 + 95 + 112 + x = 360
QL = PR = (13) = 6.5 cm Then, C (5, 2)
2 2 ⇒ 287 + x = 360
4. An angle which is less than 360° ⇒ x = (360 − 287) = 73°
and more than 180°, is called a
11. From Statement (1)
reflex angle.
∠C 30 m1 × 5 + m2 × 2
θ = 90 + = 90 + = 105° x= ,
5. Let ∠A = 2x 2 2 m1 + m2
∠B = 4x and ∠C = 3x Hence, only statement (1) is m × 2 + m2 × 5
y= 1
We know, ∠A + ∠B + ∠C = 180° sufficient. m1 + m2
∴ 2x + 4x + 3x = 180° 12. Given equation x + y = 0; 3x = 5 y, This point satisfying the given
⇒ 9x = 180° ⇒ x = 20° and y = 3x − 12 equation
Now, ∠A = 40°, ∠B = 80° C  5m + 2m2   2m + 5m2 
3 1  + 2 1  = 17
and ∠C = 60°  m1 + m2   m1 + m2 
2
–1

3x=5y
Hence, the shortest side of triangle
3x

⇒15m1 + 6m2 + 4m1 + 10m2


y=

= side opposite to the smallest


= 17m1 + 17m2
angle = BC and the longest side of
⇒ 19m1 − 17m1 = 17m1 − 16m2
A x+y=0 B

https://sscstudy.com/
https://sscstudy.com/

94 CUET (UG) Section III : General Test

⇒ 2m1 = m2 K (PQ + QR + PR ) 4 4
⇒ = ⇒k = 24. Let the abscissa be x.
m1 1 PQ + QR + PR 9 9
⇒ = Then, (x − 0)2 + (−3 − 1)2 = 52
m2 2 XY 4 4
or = ⇒ XY = × 27 ⇒ x2 + 16 = 25
PQ 9 9
⇒ m1 : m2 ⇒1 : 2 ⇒ x2 = 9
∴ XY = 12 cm
14. Given ⇒ x=±3
∠PQT = x° , ∠RQT = (x − 50)° , 19. In equilateral triangle,
25. Here, the point (6, − 3) is five units
∠TUR = (x + 25)° Altitude = Median
away from line y = 2.
∠PQT + ∠RQT = 180° A Y
4
⇒ x + x − 50 = 180 3 y=2

cm
2
⇒ 2x = 230 G 1

10
X′ X
–3 –2 –1
⇒ x = 115° –1 1 2 3 4 5 6
B C –2
Q ∠TUR + ∠URQ = 180° So, length of altitude, AD –3
–4 (6, –3)
⇒ (115 + 25) + ∠URQ = 180° 3 3 Y′
= a= × 10 = 5 3
⇒ ∠URQ = 180°−140 = 40° 2 2 So, its reflection point will also be
∴ ∠URS = 180°−∠URQ Now, 5 units away from the line y = 2.
= 180° − 40° = 140° 2 2 × 5 3 10 3 ∴ Required point = (6, 7)
AG = × AD = = cm
(8 − 2) × 180° 3 3 3 26. Let P (x, y) be equidistant from
15. ∴Required ratio = 8 20. According to the questions, largest
(12 − 2) × 180° points A (7, 1) and B (3, 5).
180 × 3 180
12 angle = = × 3 = 90° Then, AP = BP , so AP 2 = BP 2
(1 + 2 + 3) 6
(n − 2) × 180° ∴ (x − 7)2 + ( y − 1)2 = (x − 3)2
[Qinterior angle = ]
n 21. Let the other angle be x°. + ( y − 5)2
180 × 6 × 12 9 According to question, 2 2
⇒ x − 14x + 49 + y − 2 y + 1
= =
10 × 8 × 180 10 1
x = 10°+ (90°− x) = x2 − 6x + 9 + y2 − 10 y + 25
3
16. ∴Other acute angle = 180°−90° − 55° 1 ⇒ 8x − 8 y = 16
⇒ x = 10° + 30°− x
= 180°−145° 3 ∴ After solving, x − y = 2
= 35° 1
⇒ x + × = 40° 27. Let x1 = 4, x2 = 3, x3 = − 3,
3
17. Given, 4x y1 = 4, y2 = − 2 and y3 = 16
A ⇒ = 40° ⇒ x = 30°
B 30°
3 ∴ Area of triangle
80° ∴The greater angle is 1
50°
= {x1 ( y2 − y3 ) + x2 ( y3 − y1 )
C E 90° − 30° = 60°. 2
+ x3 ( y1 − y2 )}
22. Given, distance between the points 1
= [4 (− 2 − 16) + 3 (16 − 4)
(x, 0) and (− 7, 0) = 10 units 2
D Here, x1 = x, y1 = 0, x2 = − 7 + (− 3 ) {4 − (− 2)}]
∠AEB = 180° − 80° − 30° = 70° and y2 = 0 1
= [4 × (− 18) + 3 × 12 + (− 3 ) (6)]
⇒ ∠EBC = 180° − 80° − 50 = 50° ∴Required distance 2
1
= ∠CED = (x2 − x1 )2 + ( y2 − y1 )2 = (− 72 + 36 − 18)
2
∠AEB = 180°−80°−30° = 70°
⇒ (− 7 − x)2 + (0 − 0)2 = 10 1 1
∴ ∠CEB = 180°−70°−50° = 60° = × (− 54) = × 54 = 27 sq units
2 2
∴ ∠BCE = 180°−100°−60° = 20° ⇒ ± (x + 7) = 10 [neglecting negative sign]
18. Given, ∆XYZ − ∆PQR If x + 7 = 10, then x = 3 28. Since, triangle is equilateral.
If − (x + 7 ) = 10, then x = −17 ∴ AB = BC = CA = 4
X P
23. Here, (k − 4)2 + (0 + 8)2 = (10)2 ⇒ BD = = 2
4
2
[Q distance = (x2 − x1 )2 + ( y2 − y1 )2 ]
⇒ k 2 + 16 − 8k + 64 = 100 A
Y Z Q R
⇒ k 2 − 8k − 20 = 0
XY YZ XZ
∴ = = = K (say) 2
⇒ k − 10k + 2k − 20 = 0 4
PQ QR PR
⇒ k (k − 10) + 2(k − 10) = 0
According to the question,
⇒ (k + 2)(k − 10) = 0
XY + YZ + XZ 4 (0, 0) O B 2 D C(4, 0)
= ⇒ k = −2, k = 10 4
PQ + QR + PR 9
Hence, the value of k is 10 or – 2.

https://sscstudy.com/
https://sscstudy.com/

Numerical Ability 95

In ∆ ADC, + x3 ( y1 − y2 )} = 0 mx2 + nx1 my2 + ny1


x= , y=
AD 2 = 42 − 22 = 16 − 4 = 12 ⇒ {1 (2 − 5 ) + 5 (5 − (− 1) m+ n m+ n
2 y + 5(6)
⇒ AD = 2 3 + k (− 1 − 2)} = 0 ∴ 2=
1 2+ 5
∴ Area of ∆ABC = × BC × AD ⇒ {− 3 + 30 − 3k } = 0
2 ⇒ 3k = 27 ⇒ 2 y = −30 + 14 ⇒ 2 y = −16
1 ⇒ y = −8
= × 4× 2 3 ∴ k=9
2
31. Let the third vertex be (x, y). 34. We know that, y-coordinate is zero
= 4 3 sq units
on X-axis.
29. Q2x + 3 y = 6 ∴Coordinates of centroid
Given, y1 = − 3, y2 = 2
x + x2 + x3 y1 + y2 + y3 

2x 3 y
+
x y
=1 ⇒ + =1 =  1 ,  m y 2 + n y1
6 6 3 2  3 3  ∴ y=
m+ n
Comparing with equation of line Given, x1 = x, x2 = 5, x3 = − 2, y1 = y,
m (2) + n (−3)
x y y2 = 4, y3 = 4 and centroid = (5, 6) ⇒ 0=
+ = 1, we get m+ n
a b x+ 5− 2
∴ 5=
Intercept at X-axis = 3 3 ⇒ 2m − 3n = 0
y + 4+ 4 ⇒
m 3
=
and intercept at Y -axis = 2 and 6=
3 n 2
Y
⇒ x = 12 and y = 10 35. Here,
32. Given, m = 3, n = 4, x1 = 1, x2 = 2, x1 = a , y1 = b + c
B (0, 2) y1 = 3 and y2 = 7 x2 = b, y2 = c + a
∴ Coordinates of C x3 = c, y3 = a + b
A (3, 0) mx2 + nx1 my2 + ny1 
(0, 0) O
X =  ,  Area of triangle
 m+ n m+ n  1
= [x1 ( y2 − y3 ) + x2 ( y3 − y1 )
3 × 2 + 4 × 1 3 × 7 + 4 × 3
∴ Area of ∆OAB =
1
× 3 × 2 = 3 sq =  ,  2
 3+ 4 3+ 4  + x3 ( y1 − y2 )]
2
=  , 
units 10 33 1
= [a (c + a − a − b) + b(a + b − b − c)
 7 7 2
30. Given, x1 = 1, x2 = 5, x3 = k,
33. Using section formula, i.e. if a line + c(b + c − c − a )]
y1 = − 1, y2 = 2 and y3 = 5. 1
is divided by a point in certain = [a × (c − b) + b(a − c) + c(b − a )]
Since, A, B and C are collinear. ratio (m : n ), then coordinates of 2
∴ Area of triangle = 0 point (x, y) 1
= [ac − ab + ba − bc + cb − ca ] = 0
⇒ {x1 ( y2 − y3 ) + x2 ( y3 − y1 ) 2

https://sscstudy.com/
https://sscstudy.com/

96 CUET (UG) Section III : General Test

CHAPTER 18

Data Interpretation
Data Interpretation as the same suggests tests your skills to understand data presented in different forms like bar
graphs, tables, charts, line graphs etc. The questions are given with a set of data and candidate is required to
deduce the required results from the set of data.

In this section, the most important thing that an portion of entire pie chart. In this, data can be plotted
individual must be able to do is to calculate fast and with respect to only one parameter.
accurately. Adequate practice should generally set an Pie charts are useful for representing
individual well known about the course for cracking this (i) percentage of various elements with respect to
section. total quantity.
(ii) proportions of various elements with respect to
Objectives of Data Interpretation total quantity.
The objectives of data interpretation is as follows (iii) shares of various elements for a particular
To test the analytical ability of the candidate. quantity.
Examine the candidate’s ability to derive the useful Note In the questions of pie chart, the total quantity distributed over
Information from a bulk of informations. a total angle of 360°.
Testing the decision making ability from a given
situation. Bar Chart
Ability to check the data usability to find the solution It is a chart with rectangular bars with length
of a problem. proportional to the value which they represent. In this
method of data representation, the data is plotted as
Data Interpretation bars on the X and Y-axes, where X-axis represents a
It can be defined as applying statistical procedure to discreate variable and Y-axis represents the scale for the
analyse specific facts from a study or body of research. variables.
Data It is the term used to refer to the row numbers Following are the various bar charts
(facts and figures). (i) Simple Bar Chart This type of chart relates to
only one variable.
Discrete
(ii) Subdivided Bar Chart This chart is used for
Data variables represent various parts of the total magnitude of a
Continuous given variable.
Generally, in DI, questions are asked in exams on the (iii) Multiple Bar Chart In this, two or more bars are
following topics constructed adjoining to each other to represent
either different components of a total or to show
1. Pie chart/ Circle graph
multiple variables.
2. Bar chart
3. Line graph Line Graph
It indicates the variation of a quantity with respect to
Pie Chart/Circle Graph the two parameters plotted on X and Y-axes. Line graph
Pie chart is a special technique of data represented in the simplifies the data as it gives a pictorial representation
form of circle. It is divided into various sections or sector, of data and then it is very useful in determining the
each representing a different category and shows the trends and rate of change.

https://sscstudy.com/
https://sscstudy.com/

NUMERICAL ABILITY 97

Types of Line Graph 2. What is the difference (in tonne) between average
Single Line Graph Used for single variable demand and average production of the five
representation. companies taken together?
(a) 320 (b) 420 (c) 2100 (d) 1050
Multiple Line Graph Used for more than one variable
representation. Sol. (b) Required difference
3000 + 600 + 2400 + 1200 + 3300 
=  
Important Points  5 
+ + + + 1500 
− 
1500 1800 900 2700
Slope of graph represents the absolute growth and not the 
percentage growth.  5 
Simple average growth is found out by simple interest method. = 2100 − 1680 = 420 tonne
Cumulative average growth rate is found by compound interest
method. Directions (Q.Nos. 3 and 4) The following line
Cumulative average growth is different from simple average diagram represents the yearly sales figures of a company
growth in the sense that simple average growth is the growth
between two points of time.
in the years 2001-2010. Examine the diagram and
answer the questions.
10
Solved Examples 9
Directions (Q.Nos. 1 and 2) The following graph 8
shows the demand and production of cotton by 7

Sales (in ` crore)


5 companies A, B, C, D and E. Study the graph and 6
answer the given questions. 5
4
Demand (in tonne) Production (in tonne)
3300 3
3000 2
2400 2700
1
1500
1500

1800
2001 2002 2003 20042005 2006 2007 2008 2009 2010
1200
900

Years
600

3. By what per cent did the sales in 2008 decrease in


comparison to the sales in 2006?
2 2
A B C D E (a) 20 (b) 18 (c) 16 (d) 15
Companies 3 3
 6− 5
× 100 =
50 2
1. What is the ratio of companies having more Sol. (c) Percentage decrease =  % = 16 %
 6  3 3
demand than production to those having more
production than demand? 4. The ratio of sales in 2002 to that in 2007 is
(a) 2 : 3 (b) 4 : 1 (a) 2 : 3 (b) 1 : 3
(c) 3 : 2 (d) 1 : 4 (c) 1 : 1 (d) 3 : 2
Sol. (c) Required ratio = 3 : 2 Sol. (b) Required ratio = 2 : 6 = 1 : 3

https://sscstudy.com/
https://sscstudy.com/

98 CUET (UG) Section III : General Test

Practice Questions
1. The line graph below shows the number of houses sold 4. Study the following bie chart carufully and answer the
each month by a real estate agent for the first six question given below.
months of the year. Between which two months did Percentage of literate and illiterate males and females
sales increase the most? in a city having a population 250000
y
Real Estate Sales
11 Illiterate female 8%
Number of Houses

10
9 Literate male 35%
8
7 Illiterate male 24%
6
5 Literate female 33%
4
3
2
1
x
What is the difference between the number of literate
x′
0 Jan Feb Mar Apr May Jun males and literate females?
y′ Months (a) 5000 (b) 500 (c) 75000 (d) 1500
(a) April-May (b) May-June
(c) January-February (d) March-April Directions (Q.Nos. 5 and 6) The adjoining pie chart
represents the proposed outlay of the fifth-five year plan
2. The pie-diagram shows the expenditure incurred on
the preparation of a book by a publisher, under various
of ` 40000 (in crores). Examine the chart and answer the
heads. questions.
Agriculture
A. Paper 20% B. Printing 25%
C. Binding etc., 30% D. Miscellaneous 10% Irrigation and
Power
E. Royalty 15% 45° 108°
Industries and
72° 54° Minerals
B 81°
C Education
25%
30%
Roads and
0% A Communications
D1 20%
E
15% 5. The amount proposed on agriculture is more than that
on industries and minerals by
Which two expenditures together will form an angle of
(a) 7.5% (b) 10% (c) 12% (d) 12.5%
108° at the centre of the diagram?
(a) A and D (b) D and E (c) A and E (d) B and E 6. The amount (in ` crore) proposed on irrigation and
power is less than that on industries and minerals by
3. Study the following in formation carefully and answer
(a) 3000 (b) 3500
the question given below.
(c) 2000 (d) 2500
Percentage share of goods transported through Air,
Road, Rail and Sea in five countries India, China, Directions (Q. Nos. 7-10) Study the following graph
Korea, Japan, Iran. and give the answers of the following questions.
Given the total weight of goods transported in the 5 Production of Fertilizers by a Company
countries in a year are 1086, 3140, 1855, 2360 and 1465
(in 10000 tonne) Over the Years 1995-2002
thousand tonne respectively.
100
120 Sea Rail Road Air
Production (in 10000 tonnes)

90
100 80
80
75
80 70 65
60
60 60
50
50 45
40 40
40
20
30 25
0 20
India China Korea Japan Iran
10
Which country transported the maximum quantity of
goods by road? 0 1995 1996 1997 1998 1999 2000 2001 2002
(a) Iran (b) Japan (c) China (d) Korea Years

https://sscstudy.com/
https://sscstudy.com/

NUMERICAL ABILITY 99

7. What was the percentage decline in the production of Directions (Q.Nos. 15-18) The following bar
fertilizers from 1997 to 1998? diagram represents the use of different modes of travel
1
(a) 33 (b) 30 to school by students in a certain locality of the town.
3
Study the graph and answer the questions.
(c) 25 (d) 20
y
8. In how many years was the production, fertilizers 24

(1 unit =10 students)


more than the average production of the given years? 22
(a) 1 (b) 2 20
(c) 3 (d) 4 18
9. In which year was the percentage increase in 16
production as compared to the previous year, the 14
maximum? 12

Number of Students
(a) 2002 (b) 2001 10
(c) 1996 (d) 1997 8
10. The ratio of total production of fertilizers in the years 6
1996 and 1997 to that of total production in the years 4
1995, 1998 and 2000 is 2
(a) 5 : 6 (b) 6 : 5 0 x
(c) 20 : 29 (d) 13 : 24 Car Bus Moped Bicycle Rickshaw
Model of Travel
Directions (Q. Nos. 11-14) Study the following 15. How many students are coming from that locality?
graph and answer the questions given below. (a) 500 (b) 600 (c) 560 (d) 660
Production of Salt by a Company
16. How many students use Bicycle and Rickshaw combined?
(in 1000 tonne) Over the Years
(a) 240 (b) 340 (c) 140 (d) 440
120
17. What is the percentage of students using Bus from that
110
locality?
100 14 2 8
Production (in 1000 tonnes)

(a) 22 (b) 18 (c) 22 (d) 22


90 33 3 11
80
70 18. What is the ratio of the students using their means of
60
transport as Car with those using Rickshaw?
(a) 7 : 2 (b) 8 : 3 (c) 2 : 7 (d) 3 : 8
50
40 Directions (Q. Nos. 19-22) The following diagram
30 shows the percentage of population of Hindus, Sikhs and
20 Muslims with respect to total population in a town during
10 2005 to 2008. Study the diagrams and answer the question.
0 2001 2002 2003 2004 2005 2006 2007 2008 75
Hindus Sikhs Muslims
Years
70
65
11. What was the percentage increase in production of salt 65
in 2008 compared to that of 2001? 60
60

(a) 55.5 (b) 125 55


Percentage of Total Population

55
(c) 150 (d) 220
50
12. In how many of the given years was the production of 45
45
salt more than the average production of the given years?
40
(a) 1 (b) 2 35
(c) 3 (d) 4 35
30
13. The average production of 2004 and 2005 was exactly 30
25 25
equal to the average production of which of the 25
20
following pairs of years? 20
(a) 2006, 2007 (b) 2005, 2006 15 15 15
(c) 2002, 2006 (d) 2001, 2005 10 10
5
14. What was the percentage decline in the production of 0
salt from 2003 to 2004? HSM HSM HSM HSM
2005 2006 2007 2008
(a) 64.2 (b) 180 (c) 62.4 (d) 107
Years

https://sscstudy.com/
https://sscstudy.com/

100 CUET (UG) Section III : General Test

19. If the total population in 2007 was 80 lakh, then the 26. Which train has the second highest number of
number of Hindus in 2007 was (in lakh) passengers?
(a) 25 (b) 16 (c) 18 (d) 20 (a) A (b) Q (c) S (d) M
20. Percentage decrease in Hindu population form 2005 to 27. How many more per cent (approximately) number of
2008 is passengers are there in train M as compared to the
(a) 50 (b) 40 number of passengers in train L?
(c) 25 (d) 15 (a) 29 (b) 49
(c) 43 (d) 33
21. Difference of percentage of population of Hindus in
2005 and 2008 is Directions (Q. Nos. 28-32) The pie chart provided
(a) 20 (b) 15 below gives the distribution of land (in a village) under
(c) 25 (d) 30
various food crops. Study the pie chart carefully and
22. If the total number of Hindus in 2008 was 12 lakh, the answer the questions based on it.
number of Muslims in 2008 was (in lakh)
(a) 18 (b) 12
(c) 24 (d) 16 Wheat
Rice
Directions (Q.Nos. 23-27) Study the following pie Barley 72°
36° 72°
chart carefully to answer the questions.
Jowar 18° 99°
Percentage of Passenger Travelling 18°
Bajra 45°
in Six Different Trains
Maize Others

Train A
13% Train L
15%
28. If the total area under bajra was three hundred acre,
Train Q
then the total area (in hundred acre) under rice and
19%
Train M barley together is
20% (a) 18 (b) 12
Train R
9% (c) 15 (d) 20
Train S
24% 29. The combination of three crops which contribute to
more than 50% of the total area under the food crops is
Total Number of Passengers = 8500 (a) wheat, rice and maize (b) wheat, rice and jowar
(c) wheat, rice and bajra (d) rice, barley and maize
23. What was the approximate average number of
passengers in train S, train M and train L together? 30. The ratio of the land used for rice and barley is
(a) 1521 (b) 1641 (a) 3 : 1 (b) 1 : 2
(c) 1651 (d) 1671 (c) 2 : 1 (d) 3 : 2

24. If in train R, 34% of the passengers are females and 31. If 10% of the land reserved for rice be distributed to
26% are children, what is the number of males in that wheat and barley in the ratio 2 : 1, then the angle
train? corresponding to wheat in the new pie chart will be
(a) 306 (b) 316 (a) 38.4° (b) 76.8°
(c) 308 (d) 318 (c) 75.6° (d) 45.5°

25. The number of passengers in train Q is approximately 32. If the production of rice is 5 times that of jowar and the
what percentage of the total number of passengers in production of jowar is 2 times that of bajra, then the
trains A and R? ratio between the yield per acre of rice and bajra is
(a) 90 (b) 70 (a) 5 : 2 (b) 3 : 1
(c) 75 (d) 86 (c) 4 : 1 (d) 6 : 1

ANSWERS
1. (d) 2. (a) 3. (a) 4. (a) 5. (b) 6. (a) 7. (c) 8. (d) 9. (d) 10. (a)
11. (b) 12. (c) 13. (b) 14. (a) 15. (d) 16. (b) 17. (c) 18. (d) 19. (d) 20. (a)
21. (b) 22. (d) 23. (d) 24. (a) 25. (d) 26. (d) 27. (d) 28. (a) 29. (a) 30. (c)
31. (b) 32. (a)

https://sscstudy.com/
https://sscstudy.com/

NUMERICAL ABILITY 101

Hints & Solutions


1. From the line graph, it is clear 100 5 23. Required average number of
∴ Required ratio = =
that from March-April, the sales 120 6 passenger
increased the most. 90 − 40 1
11. Per cent increase = × 100 = [(24 + 20 + 15 )% of 8500]
2. In percentage form 108° is 40 3
100 = 125% 1 8500 × 59
equivalent to 108°× = 30% = × = 1671
360° 12. Average production 3 100
and from the pie diagram it is 40 + 30 + 70 + 25 24. Number of passengers in train R
clear that expenditures of A and 8500 × 9
+ 55 + 50 + 80 + 90 = = 765
D together make 30%. = 100
8
3. It is clear from the graph that 440 ∴ Number of males
Iran had maximum = = 55 = (100 − 34 − 26)% of 765
transportation of good via road. 5
765 × 40
i.e. (70, 80, 90) > 50 = = 306
4. Required difference = (35 − 33)% 100
of 250000 = 2% of 250000 13. Average production of 2004 and
2005 25. Required per cent
250000 × 2 19
= 25 + 55 = × 100 = 86%
100 = = 40 (13 + 9)
= 2500 × 2 = 5000 2
As, average production of 2005 26. M, it is clear from the pie chart.
5. Amount Spend on agriculture
and 2006
108° 27. Required per cent
= × 40000 = ` 12000 30 + 50
360° = = 40  20 − 15 
2 = × 100 = 33%
Amount Spend on Industries  15 
70 − 25
and minerals 14. Required decline = × 100 28. Corresponding angle for rice and
72° 70
= × 40000 = `8000 barley
360° = 64.2% (approx.)
= 72° + 36° = 108°
∴Required percentage 15. Number of students coming Q 18° = 300 acre
12000 − 8000 from the locality 300
= × 100 = 10% ∴ 1° =
40000 = 6 + 15 + 11 + 18 + 16
18
6. Required amount = 66 units 300
= 66 × 10 = 660 ∴ 108° = × 108
72° − 45° 18
= × 40000
360° 16. Number of students use bicycle = 1800 acre
27° and rickshaw
= × 40000 = ` 3000 29. Q 100% = 360°
360° = 18 × 10 + 16 × 10
∴ 50% = 180°
7. Required percentage decline = 180 + 160 = 340 Now, wheat + rice + maize
60 − 45
= × 100 17. Required percentage = 72° + 72° + 45°
60 15 8
= × 100 = 22 % = 189° > 180°
= 25% 66 11 30. Required ratio = 72° : 36° = 2 : 1
8. Average production 18. Required ratio = 6 : 16 = 3 : 8
25 + 40 + 60 + 45 + 65 + 50 31. 10% of 72 = 7.2°
+ 75 + 80 19. Total number of Hindus in 2007 ∴ Increase in the corresponding
= 25 angle of wheat
8 = 8000000 × = 2000000
440 100 2
= = 55 = × 7.2 = 4.8°
8 20. Required decrease percentage 3
∴ Required number of year = 4 30 − 15 New corresponding angle for
= × 100 = 50% wheat = 72° : 4. 8°
9. Percentage increase in 1997 30
= 76.8°
60 − 40 21. Required difference = (30 − 15) %
= = 50% (maximum) 32. If the production of bajra be x
40 = 15% tonne, then
10. Total production in years 1996 22. According to the question, Production of jowar = 2x tonne
and 1997
15% = 1200000 Production of rice = 10x tonne
= 40 + 60 = 100
1200000 ∴ Required ratio
and total production in years ∴ 20% = × 20 10x x
1995, 1998 and 2000 15 = : = 5 :2
= 1600000 72 18
= 25 + 45 + 50 = 120

https://sscstudy.com/
https://sscstudy.com/

QUANTITATIVE REASONING 3

Quantitative Reasoning
Quantitative reasoning involves the ability to solve Sol. (a) Price increased = 85 - 80 = ` 5
various problems through mathematical ability along 5
\ Percentage increased = ´ 100 = 6.25%
with logical and mental ability. Quantitative reasoning 80
test is designed to test the ability of a candidate to solve
Ex. 05 In an exam of 80 questions, a correct
the various Mathematical problems which are
answer is given 1 mark, a wrong answer is given -1
encountered in day to day life. To solve the problems on
Quantitative reasoning, a candidate should have a mark and if a question is not attempted there are zero
knowledge of concepts of Arithmetic or basic marks. If a student attempted only 80% of the
Mathematics. questions and got 32 marks, then how many questions
did he answer correctly?
Examples given below, will give you a better idea about (a) 48 (b) 16 (c) 56 (d) 32
the types of questions asked in various examinations. 80 ´ 80
Sol. (a) Question attempted = 80% of 80 = = 64
Ex. 01 A man climbing up a wall of 24 m high. He 100
climbs 16 m in a day but slipped back by 3 m 40 cm in Now, let the number of questions answered correctly = x
the evening. How far had the man reached on that day? \ Number of questions answered wrongly = 64 - x
(a) 11.4 m (b) 12.6 m According to the question,
(c) 12 m 40 cm (d) 19 m 40 cm x ´ 1 + (64 - x) x - 1 = 32
Sol. (b) Distance covered by man in a day x - 64 + x = 32
= (16 - 3.4) m = 12.6 m Þ 2x = 64 + 32 Þ 2x = 96
\ x = 48
Ex. 02 The weights of 4 boxes are 90, 40, 80 and 50
kilograms. Which of the following cannot be the total \ Number of questions answered correctly = 48
weight, in kilograms, of any combination of these Ex. 06 How many odd numbered pages are there in
boxes and in a combination a box can be used only once? a book of 1089 pages?
(a) 200 (b) 260 (a) 542 (b) 545 (c) 544 (d) 546
(c) 180 (d) 170 Sol. (b) Odd pages in the book are 1, 3, 5, 7, 9, … 1089.
Sol. (a) For option (b), 90 + 40 + 80 + 50 = 260 Here, every alternate page is odd starting from 1. So, if the
For option (c), 90 + 40 + 50 = 180 total number of pages was an even numbered, then the half
For option (d), 90 + 80 = 170 number of pages will be odd numbered and half will be
But in option (a), the sum of any numbers given in question even. But, here the total number of pages is odd.
is not 200. Total number of pages + 1
So, required number of pages =
2
Ex. 03 A shepherd had 17 sheeps. All but nine 1089 + 1 1090
died. How many was he left with? = = = 545
2 2
(a) 17 (b) 9
(c) Nil (d) 8 Ex. 07 At the end of a business conference all the
Sol. (b) According to the question, ‘All but nine died’. ten people present, shake hands with each other only
This statement means that ‘All except nine died’ i.e. nine once. How many handshakes were there altogether?
sheeps remained alive and others died. (a) 20 (b) 45 (c) 55 (d) 90
So, shepherd was left with 9 sheeps. Sol. (b) Clearly, the total number of hand shakes
= 9 + 8 + 7 + 6 + 5 + 4 + 3 + 2 + 1 = 45
Ex. 04 John used to buy petrol at the rate of ` 80
It can also be calculated with the help of formula.
per litre till last month. Now he buys it at the rate of
n (n - 1)
` 85 per litre. By what percentage did the petrol price Total number of handshakes =
2
increase as compared to last month? where, n = number of people
(a) 6.25% (b) 8.35% 10 (10 - 1) 10 ´ 9 90
(c) 5.5% (d) 10% Total number of handshakes = = = = 45
2 2 2

https://sscstudy.com/
https://sscstudy.com/

4 CUET (UG) Section III : General Test

Ex. 08 There are deer and peacocks in a zoo. By Students who can play only Flute and Guitar
counting heads they are 80. The number of their legs = 10 - (3 + 2 + 4) = 10 - 9 = 1
is 200. How many peacocks are there? \ Students who can play only Flute = 12 - (3 + 3 + 1)
(a) 60 (b) 50 (c) 20 (d) 30 = 12 - 7 = 5
Sol. (a) Assuming deer = d and peacock = p Ex. 10 A survey of 100 candidates with respect to
´4
Head ® d + p = 80 ¾ ¾¾® 4d + 4 p = 320l …(i) their choice of icecream flavour-vanilla, chocolate and
Legs ® 4d + 2 p = 200 ® 4d + 2 p = 200 …(ii) strawberry produced the following information. 50
(As, a deer has 4 legs and a peacock has 2 legs) candidates like vanilla, 43 like chocolate, 28 like
From subtracting Eq. (ii) from Eq. (i), we get strawberry, 13 like vanilla and chocolate, 11 like
120 chocolate and strawberry, 12 like strawberry and
2 p = 120 Þ p = = 60
2 vanilla and 5 like all the three flavours. How many
Ex. 09 In a music class, 12 students can play flute, candidates like chocolate and strawberry, but not vanilla?
(a) 24 (b) 10 (c) 32 (d) 6
11 can play guitar and 10 can play violin. 6 students
Sol. (d) According to the given information,
can play flute as well as guitar and 3 out of these can
also play violin. 3 students can play only guitar and 100 13

4 students can play only violin. How many students


Vanilla Chocolate
can play only flute? (50) 5 (43)
(a) 4 (b) 5 (c) 6 (d) 7
Sol. (b) According to the given information, 12 4 11
6
Strawberry (28)
Flute 5 3 3 Guitar Since, number of candidates who like chocolate and
(12) 3 (11)
1 2 strawberry = 11
4 Violin And, number of candidates who like all the three flavours
(10)
(chocolate, vanilla, strawberry) = 5
Students who can play only Guitar and Violin \ Number of candidates who like chocolate and strawberry,
= 11 - (3 + 3 + 3) = 11 - 9 = 2 but not vanilla = 11 - 5 = 6

Practice Questions
1. The sum of all the 3-digit numbers which are formed (c) is equal to Yellow balls minus Green balls
by the digits 1, 2 and 3 without repetition of digits, is (d) Cannot be ascertained
(a) 1233 (b) 1321 (c) 1323 (d) 1332 6. Two horses A and B run at a speed of 3 : 2 ratio in the
(e) None of these first lap; during the second lap the ratio differs by 4 :
2. A florist had 133 roses. She sold 5/7 of them. How 7; during the third lap the ratio differs by 8:9. What is
many roses had she left? the difference in ratio of speed altogether between the
(a) 58 (b) 38 (c) 57 (d) 19 two horses?
(a) 4 (b) 1 (c) 3 (d) 2
3. ‘4 is even and 8 is odd’. What is the truth value of this?
(a) Given statement is true (b) Given statements is false 7. The number of students in an art class is increasing
(c) 32 (d) Cannot be determined month after month as follows. Find the number of
students in June from the following information.
4. At a party, the number of girls is half the number of
boys. After an hours, five boys leave the party and Month Number of students
three girls join the party. How many people were January 1
present at the party an hour before?
February 2
(a) 16 (b) 8 (c) 24 (d) 22
March 4
5. There are some balls of Red, Green and Yellow colour
lying on a table. There are as many Red balls as there April 7
are Yellow balls. There are twice as many Yellow balls May 11
as there are Green ones. The number of Red balls is
June ?
(a) is equal to the sum of Yellow and Green balls
(b) is double the number of Green balls (a) 16 (b) 13 (c) 15 (d) 14

https://sscstudy.com/
https://sscstudy.com/

QUANTITATIVE REASONING 5

8. The heights of three towers are in the ratio 5 : 6 : 7. If 18. A man has ` 480 in the denominations of one-rupee
a spider takes 15 min to climb the smallest tower, how notes, five-rupee notes and ten-rupees notes. The
much time will it take to climb the highest one? number of notes of each denomination is equal. What
(a) 15 min (b) 18 min (c) 21 min (d) 54 min is the total number of notes that he has?
(a) 45 (b) 60 (c) 75 (d) 90
9. The weights of 4 boxes are 80, 60, 90 and 70 kg.
Which of the following cannot be the total weight, in 19. In a company, 60% workers are males. If the number
kilograms, of any combination of these boxes and in a of female workers in the company is 800, what is the
combination a box can be used only once. number of male workers in the company?
(a) 300 (b) 230 (c) 220 (d) 290 (a) 1600 (b) 1400 (c) 1900 (d) 1200
10. The price of onions is shown below for every fiftheen 20. The flowers kept in a basket doubles in every one
days. Find the price in the 3rd week of February. minute. If the basket gets completely filled by flowers
in 30 min, then in how many minutes 1 th of the
Period Dec. Dec. Jan. Jan. Feb. 1st Feb. 4
1st 3rd 1st 3rd week 3rd basket was filled with flowers?
week week week week week 15 45
(a) 15 min (b) 28 min (c) min (d) min
Price 20 60 40 120 100 ? 2 2
21. Some birds are sitting on two branches A and B. If
(a) 140 (b) 300 (c) 180 (d) 320
one bird of the branch A fly away and sit on branch B,
11. 8 people are present in a meeting. In the end of then the number of birds on both the branches will
meeting all people shake hand to each other. Find the become equal. But if a bird from branch B flies and
number of handshakes at the end of a meeting? sits on the branch A then the number of birds on
(a) 56 (b) 28 (c) 48 (d) 64 branch A is double of branch B. What number of
birds were sitting on branch A at the beginning?
12. A person is given 1 rupee for shooting at the target
(a) 3 (b) 4 (c) 5 (d) 7
and if he misses, then he has to give 1 rupee. If he
gets 30 rupees for 100 chances then how many 22. A factory produced 1858509 cassettes in the month of
chances did he miss? January, 7623 more cassettes in the month of
(a) 25 (b) 35 (c) 40 (d) 45 February and owing to short supply of electricity
produced 25838 less cassettes in March than in
13. There were a total of 10 bicycles and tricycles. If the
February. Find the total production in all?
total number of wheels was 24, how many tricycles
(a) 5557312 (b) 5983245
were there?
(c) 5564935 (d) 5608988
(a) 2 (b) 6 (c) 18 (d) 4
23. Which number will be in the middle if the following
14. P is greater than Q by 60% and greater than R by
numbers are arranged in descending order? 4456,
30%. The ratio of Q and R is
4465, 4655, 4665, 4565
(a) 1 : 2 (b) 2 : 1
(a) 4456 (b) 4465 (c) 4565 (d) 4655
(c) 13 : 16 (d) 16 : 13
24. Two persons A and B get the same salary. Their basic
15. In a group of cows and hens, the number of legs are
pay are different. The allowances of A and B are 65%
14 more than twice the number of heads. Find the
and 80% of the basic pay respectively. What is the
number of cows?
ratio of the basic pay?
(a) 5 (b) 7
(a) 17 : 15 (b) 7 : 5
(c) 10 (d) 12
(c) 11 : 10 (d) 12 : 11
16. If it takes two workers, working separately but at the
25. A train is running 3 min late and further being late
same speed, 2 h and 40 min to complete a particular
3 s/min. Find how long this train will be delayed
task about how long will it take one worker,
one hour?
working at the same speed, to complete the same task
(a) 1140 min (b) 1150 min
alone?
(c) 1160 min (d) 1200 min
(a) 1 h 20 min (b) 4 h 40 min
(c) 5 h (d) 5 h 20 min 26. A machine cuts the wooden log into 10 m pieces and
take 6 s to cut a 10 m piece. Find the time to cut the
17. Sonu and his friend Rahul went for shopping. Sonu
3 km long wooden log into pieces?
had ` 500 with him while Rahul had ` 240. Sonu spent
(a) 174 s (b) 180 s (c) 1794 s (d) 1800 s
twice as much as Rahul on shopping. Now, Sonu has
three times as much money as is left with Rahul. How 27. A tailor has to cut 10 shirt pieces of equal length from
much money did Sonu spend? a roll of fabric. He cuts at the rate of 45 shirt pieces in
(a) ` 220 (b) ` 60 a minute. How many rolls would be cut in 24 min?
(c) ` 440 (d) ` 120 (a) 120 (b) 108 (c) 84 (d) 72

https://sscstudy.com/
https://sscstudy.com/

6 CUET (UG) Section III : General Test

28. In a zoo, there are rabbits and pigeons. If heads are Directions (Q. Nos. 36-38) Consider the following
counted, there are 200 heads and if legs are counted information and answer the questions based on it.
there are 580 legs. How many pigeons are there? In a group of 75 students, 12 like only cabbage, 15 like only
(a) 90 (b) 100 (c) 110 (d) 120 cauliflower, 21 like only carrot, 12 like both carrot and
29. In a plane, line X is perpendicular to line Y and cabbage, 13 like only capsicum and 2 like both capsicum
parallel to line Z, line U is perpendicular to both lines and cauliflower.
V and W, line X is perpendicular to line V. Which one of 36. The difference between the people who like carrot and
the following statement is correct? cauliflower is
(a) Z, U and W are parallel (a) 6 (b) 18 (c) 16 (d) 4
(b) X, V and Y are parallel
(c) Z, V and U are all perpendicular to W 37. What is the percentage of students that do not like
(d) Y, V and W are parallel cabbage?
(a) 16 (b) 32 (c) 24 (d) 68
30. Marry said, ‘‘a number which I am thinking is divisible
by 2 or 3.’’ This statement is false, if the number which 38. How many students like only one vegetable?
I am thinking. (a) 60 (b) 61 (c) 65 (d) 71
(a) 6 (b) 8 (c) 11 (d) 15
Directions (Q. Nos. 39-41) Read the following
31. A father invites some boys and girls on his son’s birthday. information and answer the following questions.
The number of boys is less than 2 the number of girls.
In a class of 40 students, 28 can speak Tamil and 30 can
Father gives ` 10 to boys and ` 20 to girls as a gift. If total
amount spent is ` 280, find the number of boys. speak Telugu. All students can speak at least one of the
(a) 8 (b) 10 two languages.
(c) 12 (d) 14 39. Find the number of students who can speak only
Tamil.
32. 5 students A, B, C, D and E are present in an
(a) 8 (b)10 (c) 12 (d) 14
examination. If C got 5 marks less than B, D got 10
marks more than B and 20 marks less than A and E 40. Find the minimum number of students who can speak
got 22 marks more than B and B got 40 marks. Then, both Tamil and Telugu.
how many marks did A get? (a) 12 (b) 15 (c) 18 (d) 22
(a) 52 (b) 60
41. Find the number of persons who can speak only
(c) 64 (d) 70
Telugu?
33. A group has some cows, bulls and 45 hens. Every 15 (a) 8 (b) 10 (c) 12 (d) 14
animals are herded by a herdsman. The number of
42. A travel agent surveyed 100 people to find out how
bulls are twice the number of cows. If total numbers
many of them had visited the cities of Mumbai and
of heads is 186 less than total number of legs (with
Bengaluru. 31 people had visited Mumbai, 26 people
herdsman). Find the number of herdsmen?
had visited Bengaluru and 12 people had visited both
(a) 6 (b) 8
cities. Find the number of people who had visited
(c) 10 (d) 12
neither Mumbai nor Bengaluru.
34. Consider the following venn-diagram. (a) 12 (b) 55 (c) 19 (d) 45

Math
43. A survey of 500 TV viewers produced the following
50
information. 285 viewers watch football games, 195
12 12 viewers watch hockey games, 115 viewers watch
5
English 30 10 75 basketball game, 45 viewers watch football and
Hindi
basketball games, 70 viewers watch football and
hockey games, 50 viewers watch hockey and
500 candidates appeared in an examination comprising basketball games and 50 viewers do not watch any of
of tests in English, Hindi and Mathematics. The three games. How many viewers watch exactly one of
diagram gives the number of candidates who failed in the three games?
different tests. What is the percentage of the (a) 440 (b) 365
candidates who failed in atleast two subject? (c) 205 (d) 325
(a) 0.078 (b) 1.0 (c) 6.8 (d) 7.8
44. In an airline, hot drinks and iced tea were served to
35. According to a survey report, 62% people watch news 120 passengers. 75 passengers enjoyed hot drinks and
on doordarshan, 44% people read newspaper and 24% 62 enjoyed iced tea. If 40 enjoyed both, then how many
people read newspaper and watch doordarshan both. passengers enjoyed none of the beverages?
How many percentage of people neither read (a) 15 (b) 12
newspaper nor watch doordarshan. (c) 23 (d) 25
(a) 8% (b) 18% (c) 10% (d) 0%

https://sscstudy.com/
https://sscstudy.com/

ANSWERS
1. (d) 2. (b) 3. (b) 4. (c) 5. (b) 6. (c) 7. (a) 8. (c) 9. (d) 10. (b)
11. (b) 12. (b) 13. (d) 14. (c) 15. (b) 16. (d) 17. (c) 18. (d) 19. (d) 20. (b)
21. (d) 22. (c) 23. (c) 24. (d) 25. (a) 26. (c) 27. (b) 28. (c) 29. (d) 30. (c)
31. (a) 32. (d) 33. (a) 34. (d) 35. (b) 36. (c) 37. (d) 38. (b) 39. (b) 40. (c)
41. (c) 42. (b) 43. (d) 44. (c)

Hints & Solutions


1. (d) The 3-digit numbers which 15 According to the question,
´ 7 = 21 min
are formed by digits 1, 2 and 3 5 3x + 2(10 - x) = 24
without repetition of digits are 9. (d) For option (a), [Q Tricycles = 3 wheels;
given below Bicycle = 2 wheels]
= 80 + 60 + 90 + 70
312 + 321 + 123 + 231 + 132 + 213 Þ x + 20 = 24
= 1332 = 300
5 For option (b), Þ x=4
2. (b) Florist sold = 133 ´ Therefore, the required number
7 = 80 + 60 + 90 = 230
of tricycles x = 4
= 95 roses For option (c),
14. (c) Given, P = 160% of Q = 130%
\Remaining roses = 60 + 90 + 70 = 220
of R
= 133 - 95 = 38 And in option (d) the sum of any
Þ 160% of Q = 130% of R
3. (b) 4 is even but 8 is not odd. numbers given in option is 290.
160 130
So, the statement is false. So, option (d) cannot be the total Þ Q´ =R´
weight, in kg, of any 100 100
4. (c) Let the number of boys = 2x
combination of these boxes. Þ Q ´ 16 = R ´ 13
\Number of girls = x Q 13
10. (b) In each month the price of Þ =
\Total number of persons onions in 3rd week in 3 times to R 16
= 2x + x = 3x the price of onions in 1st week. Þ Q : R = 13 : 16
So, the total number of persons So, the price of onions in 3rd
must be divided by 3. week of Feb = 100 ´ 3 = 300 15. (b) Let the number of hens be x
and cows be y.
\From the given alternatives, 11. (b) Number of handshakes
only 24 is divided by 3. Number of legs in the group
n (n - 1)
Hence, 24 people were present = = 2x + 4 y
2
at the party an hour before. Number of heads in the group
Here, n = 8
5. (b) Let the number of Green =x+ y
\Required number of
balls = x 8(8 - 1) According to the question,
handshakes = legs = 2 (heads) + 14
\Number of Yellow balls = 2x 2
\Number of Red balls = 2x 8´7 2x + 4 y = 2(x + y) + 14
= = 28 Þ 2x + 4 y = 2x + 2 y + 14
It is clear from above that the 2
number of Red balls is twice the Þ 2x - 2x + 4 y - 2 y = 14
12. (b) Assuming the number of
number of Green balls. Þ 2 y = 14
chances missed by the person
6. (c) Difference between ratio of be x. y=7
speed of both horses Number of chances to shoot the (therefore number of cows = 7)
= (2 + 7 + 9) - (3 + 4 + 8) target = 100 - x 16. (d) Given that the two workers,
= 18 - 15 = 3 According to the question, working separately but at the
\Required difference = 3 (100 - x ) - x = 30 same speed takes 2 h and 40
7. (a) January Þ 1 min to complete a particular
Þ 100 - 2x = 30
task.
February Þ 1 + 1 = 2 Þ 2x = 100 - 30 = 70 Then, one worker, working at
March Þ 2 + 2 = 4 70 the same speed, to complete the
x= = 35
April Þ 4 + 3 = 7 2 same task will take double time
May Þ 7 + 4 = 11 13. (d) Let the required number of i.e. 5 h 20 min.
\June Þ 11 + 5 = 16 tricycles = x 17. (c) Let expenditure of Rahul = `x
8. (c) Spider climbs 5x units 15 min Then, the number of bicycles and expenditure of Sonu = ` 2x
\Spider will climb 7x units in = 10 - x Now, 500 - 2x = 3 (240 - x )

https://sscstudy.com/
https://sscstudy.com/

8 CUET (UG) Section III : General Test

Þ 500 - 2x = 720 - 3x \Total production 28. (c) Let the number of rabbits is x
Þ x = ` 220 = 1858509 + 1866132 + 1840294 and the number of pigeons is y.
\ Expenditure of Sonu = 5564935 According to the question,
= 2x = `440 23. (c) According to the question, x + y = 200 …(i)
18. (d) Let the number of notes of arrangement of numbers in and 4x + 2 y = 580 …(ii)
each denomination = x descending order (Q A rabbit has 4 legs and a
Total amount = 480 4665, 4655, 4565, 4465, 4456 piegon has 2 legs)
x + 5x + 10x = 480 \ Number in middle = 4565 or 2x + y = 290 …(iii)
Þ 16x = 480 24. (d) Suppose basic pay of A = ` x From Eqs. (i) and (iii),
Þ x = 30 x = 90
Basic pay of B = ` y and y = 110
So, required total number of Ratio between basic pays
notes = x + x + x Hence, number of piegons = 110
x ´ 165 y ´ 180
[number of notes of each Þ = 29. (d) According to the question,
100 100
denomination is equal] x 180 W
Þ 3x Þ 3 ´ 30 Þ 90 Þ =
y 165 X Z U
19. (d) Let the total number of 12
workers = x = = 12 : 11
11 Y V
Given, number of female workers
25. (a) Given, the train is running Clearly, Y, V and W are parallel.
= 800
3 min late. Therefore, to be 30. (c) From the given options,
Then, according to the question, delayed by one hour it has to be number 6, 8 and 15 are divisible
60 delayed = 60 - 3 = 57 min.
of x + 800 = x by either 2 or 3. Only number
100 Q The train is being late 3 sec or 11, is not divisible by 2 or 3.
60x 800 3 Hence, the required number
\ x- = 800 Þ x = min in one minute.
100 0.4 60 is 11.
8000 \The train will be late by
= = 2000 31. (a) Let the number of girls = x
4 57 min in
57 ´ 1 57 ´ 60 \Number of boys = (x - 2)
Number of male workers
= = According to the question,
60 3 3
= ´ 2000 = 1200 (x - 2) ´ 10 + x ´ 20 = 280
100 60
Þ 10x - 20 + 20x = 280
20. (b) Q 30 min ® 1 [Full] = 1140 min
Þ 30x = 280 + 20
1 26. (c) The length of the wooden
\ (30 - 1)min Þ 29 min ® Þ 30x = 300
2 log = 3 km
Þ x = 10
1 = 3 ´ 1000 = 3000 m
\ (29 - 1)min Þ 28 min ® \Number of boys = 10 - 2 = 8
4 The length of the piece cut by
Therefore, the expected time is machine = 10 m 32. (d) Given, B got = 40 marks
28 min. \Total number of pieces cut by Then, C got = 40 - 5 = 35 marks
21. (d) Assuming the number of 3000 D got = 40 + 10 = 50 marks
the machine = = 300
birds on A = x and B = y 10 A got = 50 + 20 = 70 marks
In Ist stage, x - 1 = y + 1 …(i) To cut 300 pieces the machine E got = 40 + 22 = 62 marks
In IInd stage, x + 1 = 2( y - 1) …(ii) has to cut 299 pieces because in Hence, it is clear from above
From Eqs. (i) and (ii), we get the last step when machine cut that A got 70 marks in the
299th piece then only one piece examination.
(x + 1) = 2(x - 2 - 1)
i.e. 300th piece will remain. 33. (a) Let the number of cows be x,
Þ x + 1 = 2 (x - 3)
Q Time required to cut a piece number of bulls be y and the
Þ x + 1 = 2 x -6 number of herds man be z.
= 6 sec
Þ 2x - x = 7 Then, according to the question,
\Time required to cut 299
Þ x=7 pieces x + y + 45 + z
Therefore, total number of birds =z
= 299 ´ 6 = 1794 s 16
on branch ‘A’ = 7.
27. (b) Shirt pieces cut by the tailor Þ x + y + 45 = 16z - z
22. (c) Production in January in 24 min 24 ´ 45 = 1080 Þ x + 2x + 45 = 15z (Q y = 2x)
= 1858509 Q Number of shirt pieces in a Þ 3x + 45 = 15z …(i)
Production in February roll = 10
and (3x + z + 45)
= 1858509 + 7623 \Required number of rolls
= (4 ´ 3x + 2 ´ z + 2 ´ 45) - 186
= 1866132 1080
= = 108 Þ 15z + z = 12x + 2z + 90 - 186
Production in March 10
Þ 16z - 2z - 12x = -96
= 1866132 - 25838 = 1840294

https://sscstudy.com/
https://sscstudy.com/

GENERAL MENTAL ABILITY 9

(15z - 45) 51 People who watch Football and


Þ 14z - 12 ´ = -96 \ Required percent = ´ 100
3 75 Hockey, n (H Ç F ) = 70
Þ 14z - 4 ´ (15z - 45) = -96 = 68% People who watch Basketball
Þ 14z - 60z + 180 = - 96 38. (b) Number of students who like and Football, n (B Ç F ) = 45
Þ - 46z = -276 only one vegetable People who do not watch any
\ z =6 = 12 + 21 + 15 + 13 games, n (H È B È F ) = 50
Hence, it is clear from above = 61 Now, n (H È B È F ) = n (P )
that there are 6 herds man. Sol. (Q. Nos. 39-41) According to the - n (H È B È F )¢
34. (d) According to the question, question, Þ n (H È B È F ) = n (P )
Total number of candidates Tamil Telugu - [n (H ) + n (B)
= 500 + n (F ) - n (H Ç B) - n (H Ç F )
Candidates failed in Math and - n (B Ç F ) + n (H Ç B Ç F )]
Hindi = 12 10 18 12 Þ 50 = 500 - [285 + 195 + 115
Candidates failed in Math and -70 - 50
English = 12 -45 + n (H Ç B Ç F )]
Candidate failed in Hindi and Let x students can speak both Þ 50 = 500 - 430 - n (H Ç B Ç F )
English = 10 Tamil and Telugu.
Þ n (H Ç B Ç F ) = 70 - 50 = 20
Candidates failed in three So, only Tamil = 28 - x
\20 people watch all three
subjects = 5 Only Telugu = 30 - x games.
So, number of candidates who According to the question, Number of people who only
failed in atleast two subjects Þ (28 - x) + (30 - x) + x = 40 watch Football
= 12 + 12 + 10 + 5 = 39 Þ 58 - x = 40 = 285 - (50 + 20 + 25)
\Percent candidates failed in x = 18 = 190.
atleast two subjects
39 39 39. (b) Number of students who can Number of people who only
= ´ 100 = = 7.8 speak only Tamil = 28 - x watch Hockey
500 5
= 28 - 18 = 10 = 195 - (50 + 20 + 30)
35. (b)
18% 40. (c) Students who can speak both = 195 - 100
Tamil and Telugu x Þ 18. = 95
38% 24% 20% 41. (c) Number of students who can Number of people who only
Newspaper
Doordarshan speak only Telugu 30 - 18 Þ 12. watch Basketball
42. (b) Number of people who = 115 - (25 + 20 + 30)
Total number of people who = 115 - 75
visited Mumbai = 31
watch doordarshan and who
Number of people who visited = 40
read newspaper (A È B )
Bengaluru = 26 \Number of people who watch
= A + B - (A Ç B )
Number of people who visited exactly one of the three games
= 62 + 44 - 24 = 82%
both cities = 12 = 190 + 95 + 40
Hence, percentage of people who
And, total number of people = 325
neither read newspaper nor
= 100 44. (c) Total passenger n (P ) = 120
watch doordarshan = 100 - 82
= 18% \Number of people who visited
Number of passenger enjoyed
neither Mumbai nor Bengaluru
Sol. (Q. Nos. 36-38) According to the hot drinks n (H ) = 75
information = 100 - {(31 + 26) - 12}
Number of passenger enjoyed
= 100 - 57 + 12 iced tea n (I ) = 62
= 112 - 57 = 55
12 12 21 15 2 13 Number of passenger who
43. (d) Total number of people, enjoyed both n (I Ç H ) = 40
n (P ) = 500 Number of passenger who
Cabbage Carrot Cauliflower Capsian enjoyed at least one beverage
People who watch Basketball,
36. (c) Required difference n (B) = 115 n (H È I ) = n (H ) + n (I ) - n (H Ç I )
= (12 + 21) - (15 + 2) People who watch Football, = 75 + 62 - 40
n (F ) = 285 = 97
= 33 - 17 = 16
People who watch Hockey, \Number of passenger enjoyed
37. (d) Students who do not like n (H ) = 195 none of the beverages
cabbage
People who watch Basketball = n (P ) - n (H Ç I )
= 75 - (12 + 12)
and Hockey, n (B Ç H ) = 50. = 120 - 97 = 23
= 51

https://sscstudy.com/
https://sscstudy.com/

LOGICAL AND ANALYTICAL REASONING 3

Logical and Analytical


Reasoning
Analytical thinking involves objective analysis of facts to Propositions
form a judgement. Proposition is a statement giving a relation between two
Analytical reasoning questions are designed to assess terms. It is also known as premise.
the ability to consider a group of facts and rules and
determine what could or must be true. Classification of Proposition
Analytical reasoning questions test a range of deductive This classification is based upon the quantity and
reasoning skills. These include quality of proposition. Here, quantity denotes whether
the proposition is universal or particular and quality
● Comprehending the basic structure of a set of
denotes the proposition is affirmative or negative.
relationships by determining a complete solution to
the problem posed. There are four types of propositions which are as
follows:
● Reasoning with conditional (‘‘if-then’’) statements and
recognising logically equivalent formulation of such Propositions Quantity Quality
statements. A All A are B Universal Affirmative
● Inferring what could be true or must be true from E No A are B Universal Negative
given facts and rules.
I Some A are B Particular Affirmative
● Recognising when two statements are logically
O Some A are not B Particular Negative
equivalent in context by identifying a condition or rule
that could replace one of the original conditions while Venn Diagram Representation of Four Propositions
still resulting in the same possible outcomes.
Types of
In this chapter we will discuss six types of problems Venn Diagram
Propositions
1. Syllogism
A : All S is P
2. Statement and conclusion S Always
P
3. Statement and Argument
4. Statement and Assumptions E : No S is P S p Always

5. Assertion and Reason Some S are P


Either S p [All S are P]
6. Cause and Effects
I : Some S are P or S Some S are P
1. Syllogism P [All S are P]

Syllogism is a greek word which means inference or


deduction. or P Some S are not P
S [All P are S]
It is deductive argument in which conclusion has to be
drawn from the given statements. Either P S
Some S are not P
[Some P are not S]
In this type of questions, a set of statements along with O : Some S are not P
two or more conclusions is given on the basis of these or P Some S are not P
[All P are S]
statements candidates are required to check the S
correctness of conclusions whether the conclusions
or S p Some S are not P
follows from statement or not. [No S are P]

https://sscstudy.com/
https://sscstudy.com/

4 CUET (UG) Section III : General Test

Directions (Ex. Nos. 1 and 2) In each of the following Sol. We can draw all possible cases as given below
questions two statements along with a set of conclusions is
Car Buses Scooters Buses
given. You have to take the statements to be true even if they
seem to be at variance from the commonly known facts and
decide which of the following conclusions logically follows
Scooters Cars
from the given statements (a) (b)
Give answer
Here, using both diagrammatical representations we can
(a) if only Conclusion I follows
conclude either ‘Some cars are buses’ or ‘No cars are buses’
(b) if only Conclusion II follows
Hence, atleast one of the conclusions must be true.
(c) if neither Conclusion I nor II follows
(d) if both Conclusions I and II follow Ex. 3 Some statements are given in following
Ex. 1 Statements Some birds are Donkeys. question followed by some conclusions. You have to
take the given statements to be true even, if they
All donkeys are stupid.
seems to be at variance from commonly known facts.
Conclusions I. All birds are stupid. Then answer the following question.
II. Some birds are stupid.
Statements Some walls are doors.
Sol. (b) Some doors are coats.
Some costs are chairs.
Birds Donkeys Conclusions I. Some chairs are doors.
II. Some coats are walls.
Stupid III. No Chair is doors.
(a) Only II follows (b) Only III follows
Conclusions I. ✗ II. ✓ (c) Either I or III follows (d) Only I follows
Here, only Conclusion II follows. Sol. (c)
Ex. 2 Statements All teachers are students. Walls Doors Coats Chairs
No student is girl.
Conclusions I. × II. × III. ×
Conclusions I. All girls are students.
or
II. Some girls are student.
I or III makes complementary pairs. Either I or III follows.
Sol. (c)

Teachers Girls 2. Statement and Conclusions


In these type of questions a statement is given followed
Students
by some conclusions. The candidate is required to go
Conclusions through the statements throughly and then decide
which of the given conclusion follows on its basis.
I. ✗ II. ✗
A statement is a formal account of certain facts, views,
Hence, neither Conclusion I nor II follows.
problems or situations expressed in words. A conclusion
Complementary Pair of Conclusions is a belief or an opinion that is the result of reasoning
out a given statement.
(‘either’ or Situation)
In drawing inferences from given statements, students Directions (Ex. Nos. 4 and 5) In the following questions a
are required to be more attentive to select statement is followed by two conclusions I and II. Taking
complementary pair of conclusions, where neither of the the statement to be true decide which of the given
conclusions is definitely true but a combinations of both conclutions definitely follows from the given statement.
makes a complement argument pair.
Ex. 4 Statement In a one day cricket match, the
Let us consider the example
total runs made by a team were 200. Out of these,
Statements Some cars are scooters. 160 runs were made by spinners.
Some scooters are buses. Conclusions
Conclusions I. Some cars are buses. I. 80% of the team consists of spinners.
II. No cars are buses. II. The opening batsmen were spinners.

https://sscstudy.com/
https://sscstudy.com/

LOGICAL AND ANALYTICAL REASONING 5

(a) Only Conclusion I follows Give answer


(b) Only Conclusion II follows (a) if only Argument I is strong
(c) Neither I nor II follows (b) if only Argument II is strong
(d) Both I and II follow (c) if neither I not II is strong
Sol. (c) According to the statement, 80% of the total runs (d) if both I and II are strong
were made by spinners. So, I does not follow. Nothing about
Ex. 6 Statement Should luxury hotels be banned
the opening batsmen is mentioned in the statement. So, II
also does not follow.
in India?
Arguments I. Yes, these are place form where
Ex. 5 Statement followed by some conclusions are international criminals operate.
given below.
II. No, affluent foreign tourists will
Statements I. Knowledge is not a skill. It is learning have no place to stay.
and experience.
Sol. (b) The luxury hotels are symbols of country’s
II. The illiterate are not those who
development and a place for staying the affluent foreign
cannot read and write but those who
tourists. So. argument II is a strong one. Argument I is a
cannot learn.
weak argument because ban on luxury hotels is not a way
Conclusions I. Those who cannot read and write are to stop the international criminals.
literate.
Ex. 7 Statement Should there be no examination
II. Knowledge and skill are two
upto Std IX in all the schools in India?
different things.
Arguments I. No, students need to go through the
Find which of the given conclusions logically follows
process of giving examinations right
from the given statements.
from young age.
(a) Only Conclusion I follows
II. Yes, this will help students to think
(b) Only Conclusion II follows
laterally and achieve their creative
(c) Both I and II follow
pursuits.
(d) Neither I nor II follows
Sol. (d) Argument I is strong as school is the ground where
Sol. (b) From Statement I, knowledge is learning and we prepare for the future battles of life. Argument II is
experience, it is not a skill. Hence, knowledge and skill are
strong as examinations kill our creativity, turning us all
two different things. So, Conclusion II follows. Statement II
into more clerks.
does not mean that those who cannot read and write are
literate. So, Conclusion II does not follows.
4. Statement and Assumptions
3. Statement and Argument In this, a statement is given and assumptions are drawn
from it. An assumption is something assumed, supposed
An argument is a fact or set of facts that is being given
and taken for granted. The implicity of the two
by a person to support an idea, statement or theory in
assumptions is in question. The implication means the
its favour or against it.
hidden meaning, something which is derived from and
In this topic, a statement followed by certain arguments as such based upon the statement.
is given. The statement is concerned with an issue.
These issues may be political, social or economical. The Directions (Ex. Nos. 8 and 9) In each question below is
arguments are in favour or against the given statement. given a statement followed by two Assumptions I and II.
Generally, the two arguments are contrary to each other You have to consider the statement and the following
and refer to the positive and negative results of the assumptions and decide which of the assumptions is
action as mentioned in the statement issue. implicit in the statement.
Give answer
Directions (Ex. Nos. 6 and 7) Study the following (a) if only I is implicit (b) if neither I nor II is implicit
instructions carefully and then answer the questions that (c) if only II is implicit (d) if both I and II are implicit
follow. In making decisions about important questions, it
is desirable to be able to distinguish between ‘strong’ and
Ex. 8 Statement The Prime minister is expected
‘weak’ arguments so far as they relate to the questions. to announce an expansion in his ministry shortly. A
‘Weak’ arguments may not be directly related to the newspaper report.
question and may be of minor importance or may be Assumptions
related to the trivial aspect of the question. Each question I. The newspaper has quoted authentic sources.
below is followed by two arguments numbered I and II. II. The newspaper has reliable sources for the news.
You have to decide which of the arguments is a ‘strong’
Sol. (c) The newspaper has not quoted any authentic
argument and which is a ‘weak’ argument?
sources, so I is not implicit, but II is implicit.

https://sscstudy.com/
https://sscstudy.com/

6 CUET (UG) Section III : General Test

Ex. 9 Statement Education and social change are Ex. 12 Assertion (A) We feel colder on mountains
complementary; and leads to a steady loss of tradition than on plains.
on people’s mind.
Reason (R) Temperature decreases with altitude.
Assumptions Sol. (a) Both A and R are true and R is the correct
I. There is a direct relationship between education explanation of A. Above the sea level, temperature decreases
and social change and decline of tradition on with an increase in altitude which makes mountain peaks
people’s mind. colder.
II. Traditions can make priceless contribution to Ex. 13 Assertion (A) Plants convert light energy
society. into chemical energy during the process of
Sol. (a) Only Assumption I is implicit in the given photosynthesis.
statement. Assumption II is a generalised conclusion. Reason (R) Sugar produced from photosynthesis by
plants is used by human beings as a source of energy.
5. Assertion and Reason Sol. (a) Both A and R are true, but R is not the correct
Assertion is a strong and forceful statement or claim explanation of A.
made in regard with a thing, element for its use and
effects. Reason means a fact, event or statement that 6. Caused and Effects
provides an explanation to the assertion.
Cause is the logical or scientific reason of an event that
This topic is basically designed to judge the candidate’s has occured and Effects to this cause are the
technical knowledge and his ability to reason out correctly. consequences of that event.
In the questions, two statements are given. Out of these
two statements, one is the Assertion (a) and other is the In this type of questions, two statements are given and
Reason (R). It is required to analyse whether the reason the student has to identify whether they are
is an optimum and correct explanation of the assertion. independent causes or effects of independents causes or
Some times both assertion and reason are correctly a common cause etc., and accordingly have to select the
stated facts but the reason does not correctly explain the answer options.
assertion. So, different possibilities can exist between Directions (Ex. Nos. 14 and 15) In each of these
these two statements and accordingly the correct questions, two Statements I and II are given. These may
answer is marked from the given alternatives. have a cause and effect relationship or may have
independent causes or be the effects of independent
Directions (Ex. Nos. 10-13) Each of these questions has causes.
an Assertion (A) and a Reason (R).
Give answer
Give answer
(a) if Statement I is the causes and Statement II is its effect
(a) if both ‘A’ and ‘R’ are true and ‘R’ is the correct (b) if Statement II is the cause and Statement I is its effect
explanation of ‘A’
(c) if both Statements I and II are effects of independent
(b) if both ‘A’ and ‘R’ are true but ‘R’ is not the correct causes
explanation of ‘A’
(d) if both Statements I and II are effects of some common
(c) if ‘A’ is true but ‘R’ is false cause
(d) if ‘A’ is false but ‘R’ is true
Ex. 14
Ex. 10 Assertion (A) A body weights less when
I. The prices of petroleum products dropped
immersed in water.
marginally last week.
Reason (R) Newton’s law explains the above
phenomenon. II. The State Government reduced the tax on
petroleum products last week.
Sol. (c) A is true and R is false. A body weights less when
immersed in water but this law was given by Archimedes’, Sol. (b) State Government reduced the tax on petroleum
not by Newton. products last week, that is why the price of petroleum
products dropped marginally, so Statement II is the cause
Ex. 11 Assertion (A) The steam engine was and Statement I is the effect.
invented by James Watt.
Ex. 15
Reason (R) There was a problem of taking out water
from flooded mines. I. Many people visited the religious place during the
weekend.
Sol. (a) Both A and R are true and R is the correct
explanation of A. The problem of pumping out water from II. Few people visited the religious place during the
the flooded mines required the need of a self-working week days.
engine which led James Watt to invent the same. Sol. (c) Both statements are effects of independent causes.

https://sscstudy.com/
https://sscstudy.com/

LOGICAL AND ANALYTICAL REASONING 7

Practice Questions
Directions (Q. Nos. 1-20) In the following questions (a) Only Conclusion I follows
certain statements and conclusions are given. You have (b) Only Conclusion II follows
to take statements as true even if they seems to be at (c) Both Conclusion I and II follow
variance from commonly known facts, then answer the (d) Neither Conclusion I nor II follows
following questions. 6. Statements
1. Statement Medals are awards. 1. Some food are sweet.
Conclusions 2. Some food are sour.
I. All awards are not medals. Conclusions
II. All medals received are called awards. I. All food are either sweet or sour.
(a) Only Conclusion I follows II. Some sweets are sour.
(b) Only Conclusion II follows (a) Only Conclusion I follows
(c) Both Conclusions I and II follow (b) Only Conclusion II follows
(d) Neither Conclusion I nor II follows (c) Both Conclusions I and II follow
(d) Neither Conclusion I nor II follows
2. Statements
1. Some fruits are vegetables. 7. Statements
2. All vegetables are plants. 1. All children are students.
Conclusions 2. All students are players.
I. Some plants are vegetables. Conclusions
I. All players are students.
II. Some fruits are plants.
II. All children are players.
(a) Only Conclusion I follows
(b) Only Conclusion II follows (a) Only Conclusion I follows
(c) Both I and II follow (b) Only Conclusion II follows
(d) Neither of them follows (c) Both Conclusion I and II follow
(d) Neither Conclusion I nor II follows
3. Statements
8. Statements All Student are boys.
1. All animals are dogs.
No boy is dull.
2. All dogs are birds.
Conclusions
Conclusions
I. There are no girl in the class.
I. All animals are birds.
II. No student is dull.
II. All birds are animals.
(a) Only Conclusion I follows
(a) Only Conclusion I follows
(b) Only Conclusion II follows
(b) Only Conclusion II follows
(c) Both Conclusions I and II follow
(c) Both Conclusions I and II follow
(d) Neither Conclusion I nor II follows
(d) Neither Conclusion I nor II follows
9. Statements
4. Statements
1. No girl is a parrot.
1. Some books are mobiles.
2. Sowmya is girl.
2. Some calculators are mobiles.
Conclusions
Conclusions
I. Sowmya is not a parrot.
I. Some mobiles are calculators.
II. All girls are not Sowmya.
II. Some mobiles are books.
(a) Only Conclusion I follows
(a) Only Conclusion I follows
(b) Only Conclusion II follows
(b) Only Conclusion I follows
(c) Either Conclusion I or II follows
(c) Both Conclusions I and II follow
(d) Niether Conclusion I nor II follows
(d) Neither Conclusion I nor II follows
10. Statements
5. Statement
1. Most players are men.
1. No flower is a pot.
2. Some men are singers.
2. No pot is a garden.
Conclusions
Conclusion
I. Some singers are men.
I. No flower is a garden. II. Some players are singers.
II. All garden are pots.

https://sscstudy.com/
https://sscstudy.com/

8 CUET (UG) Section III : General Test

(a) Only Conclusion I follows 16. Statements


(b) Only Conclusion II follows 1. Some yeas are decades.
(c) Neither Conclusion I nor II follows
2. All centuries are decades.
(d) Both Conclusions I and II follow
Conclusions
11. Statements I. Some centuries and years.
1. All cupboards are watches. II. Some decades are years.
2. All watches are costly. III. No century is a year.
Conclusions (a) Conclusion I and II follow.
I. All cupboards are costly. (b) Neither Conclusion I nor II follow
II. Some costly things are cupboards. (c) Only Conclusion I follows
(a) Only Conclusion I follows (d) Only Conclusion II follows
(b) Only Conclusion II follows
17. Statements
(c) Both of them follow
1. No cow is a chair.
(d) Neither of them follows
2. All chairs are tables.
12. Statements
Conclusions
1. All English movies are violent.
I. Some tables are chairs
2. Some people like watching English movies.
II. Some tables are cows.
Conclusions
III. Some chairs are cows.
I. All people watching English movies like violence.
IV. No table is a cow.
II. All people who like violence watch English (a) Either II or III follow (b) Either II or IV and I follow
movies.
(c) Only I follows (d) All conclusions follow
(a) Only I follows (b) Only II follows
(c) Neither I nor II follows (d) Both I and II follow 18. Statements
1. Some buds are flowers.
13. Statements
2. All flowers are trees.
1. Some human creature are angles.
3. All trees are leaves.
2. All angles are doctors.
Conclusions
Conclusions
I. Some leaves are buds.
I. Some human creatures are doctors.
II. Some doctors are human creatures. II. All flowers are leaves.
(a) Only I follows (a) Only Conclusion I follows
(b) Only II follows (b) Only Conclusion II follows
(c) Both I and II follow (c) Both I and II follow
(d) Neither I nor II follows (d) Neither I nor II follows

14. Statements 19. Below are given statements. You have to take the
given statements to be true even if they seem to be at
1. All teachers are experienced.
variance with the commonly known facts and then
2. Some teachers are spinsters. decide which of the given conclusions logically follows
Conclusions from the given statements.
I. Some experienced are spinsters. Statements
II. Some spinsters are experienced. 1. All rats are hills.
(a) Only Conclusions II follows 2. All hills are rivers.
(b) Either Conclusion I or II follows (a) Some rivers are rats (b) No river is a hill
(c) Both Conclusions I and II follow (c) All hills are rats (d) No river is a rat
(d) Only Conclusion I follows
20. Read the given statements carefully and answer the
15. Statements question.
1. Blue is black and some black is red. Statements
2. All red is green but not yellow. 1. All teachers get angry.
Conclusions 2. Some teachers are sad.
I. Some Blue is Green. 3. Sad people may cry.
II. No Black is Yellow. Which of the following conclusion is true?
(a) All sad people cry
III. Some Black is not Yellow
(b) Some teachers may cry
(a) Only I and II follow (b) Only II and IV follow (c) All angry people are teachers
(c) Only I and III follow (d) Only I, II and IV follow (d) All sad people get angry

https://sscstudy.com/
https://sscstudy.com/

LOGICAL AND ANALYTICAL REASONING 9

Directions (Q. Nos. 21-28) In the following questions, 29. A statement follows by some conclusions are given
each question has a statement followed by two conclusions. below.
Taking the statement to be true, decide which of the given Statement Based on his performance, Rajesh got a
conclusions definitely follows from the given statement. poor rating in his office.
Indicate your answer as Conclusions
Give answer I. Rajesh did not perform well.
(a) if only I follows (b) if only II follows II. The rating given to Rajesh was not up to the mark.
(c) if neither I nor II follows (d) if both I and II follow
Find which of the given conclusions logically follow
21. Statement Good health is dependent on right eating from the given statements.
habits. Most of the people do not follow any rule (a) Only Conclusion I follows
regarding eating. (b) Only Conclusion II follows
Conclusions (c) Both I and II follow
I. Most of the people have poor health. (d) Neither I nor II follows
II. People are ignorant of proper eating habits. 30. Statements followed by some conclusions are given below.
22. Statement Black cloud follows thunder. Rains follow Statements
thunder. 1. Depleting natural resources is a major concern in
Conclusions our country.
I. Thunder is the cause of rain. 2. Unless we go for renewable energy sources in a big
II. Black cloud is the cause of thunder. way increase in population being a heavy burden
on the energy resources, will reduce the quality of life.
23. Statement Workers feel highly motivated when they
Conclusions
get sense of involvement by participating in the
management of companies. I. Depleting natural impacts quality of life.
Conclusions II. Use of renewable energy sources is a solution to
counter increase in population.
I. Workers should be motivated to produce more.
Find which of the gives conclusions logically follows
II. Workers should be allowed to participate in the
from the given statements.
management of companies.
(a) Only Conclusion I follows
24. Statement Industrial revolution which first of all (b) Only Conclusion II follows
started in Europe has brought about modern age. (c) Both I and II follow
Conclusions (d) Neither I and II follows
I. Disparity between rich and poor results in revolution.
II. Revolution overhauls society. Directions (Q. Nos. 31-35) In making decisions about
25. Statement America’s defence secretary reiterated important questions, it is desirable to distinguish between
that they would continue to supply arms to Pakistan. a ‘strong’ argument and a ‘weak’ argument. A ‘strong’
Conclusions argument must be both important and directly related to
the question. A ‘weak’ argument may not be directly related
I. Pakistan is incapable of manufacturing arms.
to the question and may be of minor importance or may be
II. It would ensure peace in the region.
related to the trivial aspect of the question. Each question
26. Statement Success cannot be achieved without hard below is followed by two arguments numbered I and II. You
work.
have to decide which of the arguments is ‘strong’ and which
Conclusions is ‘weak’.
I. Every hardworking person is successful.
Give answer
II. Every successful person is hardworking. (a) if only Argument I is strong
27. Statement Unlike Aryabhata, moon is a natural (b) if only Argument II is strong
satellite of the Earth. (c) if neither I nor II is strong
Conclusions (d) if both I and II are strong
I. Aryabhatta is not a satellite. 31. Statement Should there be a total ban on use of
II. Moon is star and Aryabhatta is a satellite. plastic bags?
Find which of the given conclusions logically follows Arguments
from the given statement. I. No, instead the thickness of plastic bags, which can
28. Statement Customer service cannot be enforced. It be used without much damage to the environment,
has to come from within. should be specified.
Conclusions II. Yes, use of plastic bags causes various problems
I. Customer service should be voluntary. like water pollution and water logging and hence,
it is necessary to ban it.
II. Employees do not serve customers.

https://sscstudy.com/
https://sscstudy.com/

10 CUET (UG) Section III : General Test

32. Statement Should coal engines be replaced by electric 38. Statement In Mumbai, railway trains are
engines in trains? indispensable for people in the suburbs to reach their
Arguments places of work on time.
I. Yes, coal engines cause a lot of pollution. Assumptions
II. No, India does not produce enough electricity to I Railway trains are the only mode of transport
fulfil even the domestic needs. available in the suburbs of Mumbai.
33. Statement should English be the medium of II Only railway trains run punctually.
instruction for higher education in India? 39. Statement The mangoes are too cheap to be good.
Arguments Assumptions
I. Yes, even in advanced countries like UK and USA, I When the mango crop is abundant, the prices go
the medium of instruction is English for higher down.
education. II The lower the selling price, the inferior is the
II. Yes, English is much widely spoken language in quality of the commodity.
the world today. 40. Statement Of all the newspapers published in Delhi,
‘The Time and Space’ has the largest number of readers.
34. Statement Should there be only a uniform rate of
income tax irrespective of the level of income? Assumptions
I Volume of readership of all the newspapers in
Arguments
Delhi is known.
I. Yes, this will substantially reduce the work of the
II No newspaper in Delhi other than ‘The Time and
officials of the income tax department.
Space’ has large readership.
II. No, this will reduce government tax collection to a
large extent. Directions (Q. Nos. 41-44) Each of these questions has
35. Statement Should all the factories in the cities be an Assertion (A) and a Reason (R).
shifted to the outskirts, far away from the main city? Give answer
(a) if both ‘A’ and ‘R’ are true and ‘R’ is the correct
Arguments
explanation of ‘A’
I. Yes, this is an essential Step for controlling
(b) if both ‘A’ and ‘R’ are true but ‘R’ is not the correct
pollution in the city.
explanation of ‘A’
II. No, such a Step will lead to lot of inconvenience to (c) if ‘A’ is true but ‘R’ is false
the employees of the factories and their families as
(d) if ‘A’ is false but ‘R’ is true
well.
41. Assertion (A) India’s ‘Republic Day’ falls on 26th
Directions (Q. Nos. 36-40) In each question below is January.
given a statement followed by two Assumptions I and II. An Reason (R) Constitution of India, declaring India as
assumption is something supposed or taken for granted. a ‘Republic’, came into force on 26th January 1950.
You have to consider the statement and the following 42. Assertion (A) India is a democracy.
assumptions and decide which of the assumptions is
Reason (R) India is a developing country.
implicit in the statement.
43. Assertion (A) In India, the judiciary is independent
Give answer
of the executive.
(a) if Assumption I is implicit
Reason (R) Judiciary favours the government and
(b) if Assumption II is implicit
helps in the implementation of its plans.
(c) if neither I nor II is implicit
(d) if both I and II are implicit
44. Assertion (A) A salt water fish drinks sea water
whereas a fresh water fish never drinks water.
36. Statement Apart from the entertainment value of Reason (R) A salt water fish is hypertonic to its
television, its educational value cannot be ignored. environment while a fresh water fish is not hypertonic
Assumptions to its environment.
I People take the television to be means of 45. An Assertion (A) and Reason (R) are given below.
entertainment only. Assertion (A) Leakages in household gas cylinders
II The educational value of television is not realised can be detected.
properly. Reason (R) LPG has a strong smell.
37. Statement Ten candidates who were on the waiting Choose the correct option.
list could finally be admitted to the course. (a) Both A and R are true and R is the correct explanation
of A
Assumptions
(b) Both A and R are true and R is not the correct
I Wait-listed candidates get admission with difficulty. explanation of A
II A large number of candidates were on the waiting (c) Both A and R are false
list. (d) A is true but R is false

https://sscstudy.com/
https://sscstudy.com/

LOGICAL AND ANALYTICAL REASONING 11

46. An Assertion (A) and Reason (R) are given below. Directions (Q. Nos. 51-60) Below in each question are
Assertion (A) Beri - Beri is a viral infection. given two Statements A and B. These statements may be
Reason (R) Vitamin deficiency causes diseases. either independent causes or may be effects of independent
Choose the correct option. causes or of a common cause. One of these statements may
(a) A is false but R is true be the effect of the other statement. Read both the
(b) A is true but R is false statements and decide which of the following answer
(c) Both A and R are false choices correctly depicts the relationship between these two
(d) Both A and R are true and R is the correct explanation statements.
of A Give answer
47. An Assertion (A) and a Reason (R) are given below. (a) if Statement A is the cause and Statement B is its effect
(b) if Statement B is the cause and Statement A is its effect
Assertion (A) We prefer to wear white clothes in
winter. (c) if both statements are effects of independent causes
Reason (R) White clothes are good reflectors of heat. (d) if both statements are effects of some common cause
Choose the correct option. 51. A. Domestic price of petrol has gone down.
(a) A is true but R is false B. International price of crude oil has decreased.
(b) A is false but R is true 52. A. Ahmed is a healthy boy.
(c) Both A and R are true and R is the correct explanation B. His mother is very particular about the food he
of (A) eats.
(d) Both A and R are true and R is not the correct
53. A. Ravi died while on the way to the hospital.
explanation of A
B. A car dashed into the motorcycle Ravi was driving.
48. An Assertion (A) and a Reason (R) are given below.
54. A. The average day temperature of the city has
Assertion (A) Rainwater harvesting reduces soil increased by about 2 degrees in the current year
crosion. over the average of past ten years.
Reason (R) Rainwater harvesting is not important for B. More people living in rural areas of the state have
conservation of water. started migrating to the urban areas in comparison
Choose the correct option. with the earlier year.
(a) Both A and R are true and R is the correct explanation
of A 55. A. Most of the shopkeepers in the locality closed their
shops for the second continuous day.
(b) Both A and R are true, but R is not the correct
explanation of A B. Two groups of people living in the locality have
(c) A is true, but R is false
been fighting with each other with bricks and
stones, forcing people to stay indoors.
(d) A is false, but R is true
56. A. The Government has decided to increase the prices
49. An Assertion (A) and a Reason (R) are given below. of LPG cylinders with immediate effect.
Assertion (A) Penguins are birds, found in the B. The Government has decided to increase the prices
hottest regions of the earth. of Kerosine with immediate effect.
Reason (R) Birds in hot regions do not have wings.
Choose the correct option. 57. A. Party ‘X’ won clear majority in the recently held
(a) Both A and R are true and R is the correct explanation
state assembly elections.
of A
B. Of late, there was unrest in public and also among
(b) Both A and R are ture, but r is not the correct
the members of the ruling party of the state.
explanation of A
(c) A is true, but R is false 58. A. Staff members of the university decided to go on
(d) Both A and R are false strike in protest during the examinations.
B. The university administration made all the
50. An Assertion (A) and a Reason (R) are given below. arrangements for smooth conduct of examination
Assertion (A) Perennial rivers mostly originate from with the help of outsiders.
the Himalayas.
59. A. In the university examination, overall performance
Reason (R) The fountains in the Himalayas feed the of students from college ‘X’ was better
rivers. than that of students from college ‘Y’.
Choose the correct option.
(a) Both A and R are true and R is the correct explanation B. Majority of the students depend upon coaching
of A classes for university examinations.
(b) Both A and R are true, but R is not the correct 60. A. The Government of state ‘X’ decided to ban
explanation of A working of women in night shifts and also in late
(c) A is true, but R is false evening hours.
(d) A is false, but R is true B. The percentage of working women has a significant
rise in the last one decade.

https://sscstudy.com/
https://sscstudy.com/

12 CUET (UG) Section III : General Test

ANSWERS
1. (b) 2. (c) 3. (a) 4. (c) 5. (d) 6. (d) 7. (b) 8. (c) 9. (a) 10. (a)
11. (c) 12. (a) 13. (c) 14. (c) 15. (c) 16. (d) 17. (b) 18. (c) 19. (a) 20. (b)
21. (a) 22. (b) 23. (d) 24. (b) 25. (c) 26. (b) 27. (c) 28. (a) 29. (c) 30. (c)
31. (d) 32. (d) 33. (b) 34. (b) 35. (a) 36. (d) 37. (a) 38. (c) 39. (b) 40. (a)
41. (a) 42. (b) 43. (c) 44. (a) 45. (a) 46. (a) 47. (b) 48. (c) 49. (d) 50. (c)
51. (b) 52. (b) 53. (b) 54. (c) 55. (b) 56. (d) 57. (d) 58. (a) 59. (c) 60. (c)

Hints & Solutions


1. (b) 7. (b) Conclusions I.ü II. ✗
medals awards or medals awards Students From the above figure, only I follows.
13. (c)
Children
Conclusions I. û II. ü
Human
So, only Conclusion II follows. Angles
Creature
Players
2. (c)
Conclusions I. û II. ü Doctors
From the above figure,
Fruits Conclusions I. ü II. ü
Vegetables Only Conclusion II follows.
From the above figure, both I and II follow.
8. (c)
14. (c)
Plants
Students Dull
Conclusions I. ü II. ü Teacher Spinsters
Both the conclusions follow.
Boys
3. (a) Birds
Conclusions I. ü II. ü Experienced
From the above figure, Conclusions I. ü II. ü
Animals Both Conclusions I and II follow. From the above figure. Both Conclusion I and
II follow.
Dog 9. (a) Girls
Parrot 15. (c)
S
Yellow
From the above figure. S = Sowmya Green
Only Conclusion I follows. From above figure, only Conclusion I follows.
4. (c) Blue Red
10. (a) Black
Books Moblies Calculators Players Men Singers

Conclusions I. ü II.ü Conclusions I. ü II. ✗ Conclusions I.ü II. ✗ III. ü


From the above figure. Form above figure. Only Conclusion I follows. From the above figure. Both Conclusions I and
Both Conclusions I and II follow.
11. (c) III follow.
5. (d) Flower Pot Garden 16. (d)
Y C or Y C
Cup-
or Garden board
D D
Flower Pot Watches
Conclusions I. ✗ II. ü III. ✗
Conclusions I. ✗ II. ✗ Costly
From above it is clear that only Conclusion II
Form the above figure. Conclusions I. ü II. ü follows.
Neither Conclusion I nor II follow. Form the above figure, Both of the Conclusion 17. (b)
6. (d) follow.
Sour Food Sweet Cow Chair
12. (a)
Conclusions I. ✗ II. ✗ English Table
Liking
From the above figure. movie
Conclusions I. ü II. ü III. ✗ IV. ü
Neither Conclusion I nor II follows.
Violent From above figure either II or IV and I follow.

https://sscstudy.com/
https://sscstudy.com/

LOGICAL AND ANALYTICAL REASONING 13

Also, it is clearly given in the statement that 43. (c) A is true but R is false. As in India, the
moon is a natural satellite. Hence, neither I nor judiciary is completely independent of the
or Cow Chair II follows. executive the government has no interference
28. (a) From the statement it is clear that, in the judicial affairs.
Table customer service should be voluntrary. 44. (a) Assertion is true, and this happens
18. (c) Hence, only Conclusion I follows. because a salt water fish is hypertonic to its
29. (c) Rajesh got a poor rating because of environment while a fresh water fish is not
his performance so it is clear that Rajesh did hypertonic to its environment.
not perform well. It is also clear from the 45. (a) LPG contains ethyl mercaptan which
Buds Flowers
statement that the rating given to Rajesh was has strong smell. Because of this, smell
Tree not up to the mark. leakages in household gas cylindren can be
Hence, both Conclusions I and II follow. detected.
Leaves
30. (c) It is clear that, depleting resources 46. (a) Beri-Beri is a disease in which the
impacts quality of life and use of renewable body doesn’t have enough vitamin B1
Conclusions I. ü II. ü
energy sources is a solution to counter (thiamin). Hence A is false but R is true.
Both the conclusions follow.
increase in population. 47. (b) We prefer to wear dark clothes in
19. (a)
31. (d) Argument I is strong as it takes a wise, winter as they absorb the heat and keep the
reconciliatory approach to the problem. body warm. However, white clothes are good
Argument II is also strong as water pollution reflectors of heat and are worn in summer.
etc., may severally harm mankind. 48. (c) A is true, but R is false One of the
Rats main reason of rain water harvesting is
32. (d) Both the arguments are logical and
directly related to the statement and hence, conserve water for future.
Hills are strong arguments. 49. (d) Both A and R are false.
Leaves 33. (b) Anything successful in other countries
may not succeed in India. 50. (c) A is true, but R is false Perennial rivers
Clearly, Some rivers are rats. However, since English is much widely spoken usually originate from mountainous snowy
20. (b) language in the world today and hence, regions or glaciers.
should be adopted, is a strong idea. 51. (b) The effect of decrement in
Hence, Argument II is the strong argument. International price of crude oil is the domestic
Teachers Sad price of petrol also gone down.
34. (b) Argument I is weak because reduction
Angry
of work load of IT officials is not too desirable 52. (b) Since, Ahmed’s mother take care of
Cry a motive. Argument II is strong as reduced tax what he eats, Ahmed has a good health.
collection will have a bad impact on state
53. (b) Ravi died because of the accident.
Here, sad people many cry. So, all sad people activities.
cry does not definitely follow. 54. (c) The two statements seem to have
35. (a) Argument I is strong because pollution contrary causes.
Clearly, Some teachers may cry. control is highly desirable. Argument II is weak.
21. (a) Conclusion I definitely follows. 55. (b) The fighting has led to the closure of
36. (d) Both Assumptions I and II are implicit. the shops.
Conclusion II is not confirmed by the given
statement. 37. (a) Assumption II is not implicit in the 56. (d) It seems the price of Petroleum has
statement.
22. (b) Only Conclusion II follows. increased in general. Alternatively, subsidies
Only assumption I is implicit. may have been reduced, again a cause
Conclusion I is wrong as rains are the cause
of thunder. 38. (c) Neither Assumption I nor Assumption common to both the statements.
II is implicit in the statement because of the 57. (d) It can be safely assumed that party ‘X’
23. (d) Both Conclusions I and II follow from word ‘only’.
the statement as involvement of workers was earlier in the opposition and has
boosts production and benefits the companies. 39. (b) Assumption I is not implicit in the benefitted from anti-incumbancy. But A and B
statement, because it states the principle of are consequences of a common cause, i.e.
24. (b) Only Conclusion II follows. Conclusion bad governance by the ruling party.
supply and prices. Only Assumption II is
I is not related to the statement.
implicit which states the principle of price 58. (a) Since, the staff members have gone
25. (c) Neither Conclusion I nor conclusion II quality parity. on strike, the help of outsiders has been
follows. Both the conclusions are unrelated to sought.
the statement. 40. (a) Assumption I is implicit in the
statement because the largest readership may 59. (c) A seems to have happened as X is a
26. (b) Success cannot be achieved without be established only if the volume of readership better college. B seems to be the result of the
hardwork it means that every successful of all the newspapers is known. falling standard of teaching in colleges.
person is hardworking.
Hence, only Conclusion II follows.
41. (a) Both A and R are true and R is the 60. (c) A might have happened because
correct explanation of A. harassment of woman is on the rise while B
27. (c) It is clear from the statement that seems to be the result of a change in gender
Aryabhata is a satellite but it is not a natural
42. (b) Both A and R are true but R is not the
correct explanation of A. role perception.
satellite.

https://sscstudy.com/
https://sscstudy.com/

PRACTICE SET 01 3

CUET (UG) Section III : General Test

Practice Paper 01
Instructions 60 questions to be attempted out of 75. Time : 60 Min

1. Which state became the first state 7. Who was the first woman to receive 15. Which Indian won the 2014 Nobel
to launch India’s first Night Jnanpith Award? Prize for Peace?
Navigation mobile application for (a) Ashapurna Devi (a) Kailash Satyarthi
ferry services? (b) Mahaswetah Devi (b) Malala Yousafzai
(a) Haryana (b) Bihar (c) Amrita Pritam (c) Sanjiv Chaturvedi
(c) Rajasthan (d) Assam (d) Mahadevi Verma (d) Anshu Gupta

2. National Institute of Urban Affairs 8. The longest river bridge of India is 16. Army Training Command is located
(NIUA) signed MoU with which (a) Dibang River bridge (Roing-Dambuk) in
institution to collaborate on (b) Mahatma Gandhi Setu (Patna-Hajipur) (a) Chandigarh (b) Shimla
‘Sustainable Cities India programme’? (c) Bandra-Worli Sea Link (Mumbai) (c) Jaipur (d) Pune
(a) NITI Aayog (d) Bhupen Hazarika Setu (Tinsukia)
17. Admiral is the Chief of which force
(b) World Economic Forum 9. Which is the largest river Island of of Indian Defence?
(c) World Bank
the world located in India? (a) Indian Navy (b) Indian Army
(d) UNDP
(a) Dibru-saikhua (b) Majuli (c) Indian Air Force (d) Coast Guard
3. Which financial company has (c) Umananda (d) None of these
18. From where was Mangalyaan
partnered with International Air 10. Which among the following is most launched?
Transport Association (IATA) to populous city of India? (a) Chennai
launch ‘IATA Pay’ payment
(a) Mumbai (b) New Delhi (b) Sriharikota
platform for airline industry in (c) Kolkata (d) Bengaluru (c) Trombay
India? (d) Gopalpur, on sea
(a) DBS Bank 11. The first woman to get the Bharat
(b) State Bank of India Ratna Award is 19. Who developed Arjun Tank?
(c) Standard Chartered (a) Mother Teresa (a) DRDO
(d) HSBC (b) Indira Gandhi (b) Ministry of Defence
(c) Lata Mangeshkar (c) Russian Army
4. India’s first Biosafety level-3 (d) Sarojini Naidu (d) Indian Army
containment mobile laboratory has
been inaugurated in 12. Which boxer is nicknamed ‘The 20. India procured T90 tank from
(a) Jaipur (Rajasthan) Real Deal’? which country?
(b) Nashik (Maharashtra) (a) Mike Tyson (a) China (b) Russia
(c) Chennai (Tamil Nadu) (b) Mohammed Ali (c) France (d) USA
(d) Gurugram (Haryana) (c) Evander Holyfield
21. Which is the first missile which has
(d) Joe Louis
5. Which country has topped the been developed in India?
medal table of Winter Olympics 13. Jnanpith Award is conferred in the (a) Akash (b) Prithvi
Games 2022 held in Beijing? field of (c) Agni (d) Trishul
(a) India (b) Norway (a) Literature (b) History
22. Air Force Academy is located in
(c) Germany (d) China (c) Drama (d) Dance
(a) Hyderabad (b) Coimbatore
6. First Governor of Reserve Bank of 14. Who received both Bharat Ratna (c) Kakinada (d) Mumbai
India is and Nishan-e-Pakistan?
23. Indian and Russian Scientists
(a) CD Deshmukh (a) Lal Bahadur Shastri
successfully test fired a Supersonic
(b) Hilton Young (b) Ayub Khan
cruise missile named
(c) Edwin Lutyens (c) Morarji Desai
(a) GSAT-1 (b) INSAT-3B
(d) Osborne Smith (d) Maulana Abdul Kalam Azad
(c) Trishul (d) Brahmos

https://sscstudy.com/
https://sscstudy.com/

4 CUET (UG) Section III : General Test

24. Nuclear powered attack submarine Who is the father of K? either side or both side are
of India is (a) O (b) L (c) M (d) N interchanged. Select the correct
(a) INS Kitten (b) INS Chennai numbers to be interchanged from
30. Select the letter-cluster that can
(c) INS Kolkata (d) INS Arihant the given options.
replace the question mark (?) in the
25. Which is first Indian ship to have a following series. 3 + 5 × 4 − 24 ÷ 3 = 7 × 4 − 3 + 36 ÷ 6
super structure of carbon fibre (a) 6, 3 (b) 5, 7
aYd, fTi, kOn, pJs, ?
(c) 4, 7 (d) 24, 36
composite material? (a) VeX (b) uEw (c) uFw (d) uEx
(a) INS Sindurakshak 38. Select the set of letters that when
31. Four letter-clusters have been
(b) INS Kiltan sequentially placed in the blanks of
(c) INS Kalvari
given, out of which three are alike
the given letters series will
(d) INS Chakra
in same manner, while one is
complete the series.
different. Select the odd
26. Select the option that is related to letter-cluster. f_hg_fh_gf_hg_fh_g
the third number in the same way (a) TVW (b) FHJ (a) f, g, h, f, g, h (b) g, h, f, g, h, f
as the second number is related to (c) LNP (D) DFH (c) g, f, g, f, h, f (d) h, f, g, h, f, g
the first number and the sixth 39. In a certain code language, U is
number is related to the fifth 32. Select the option in which the
words share the same relationship written as C, K is written as H. L is
number. written as U. N is written as E. S is
as that shared by the given pair of
72 : 108 : : 84 : ? : : 102 : 153 words. written as L. E is written as K and
(a) 144 (b) 126 (c) 117 (d) 135 C is written as N. How will,
Blunder : Error
27. Identify the diagram that best ‘KNUCKLES’ be written as in that
(a) Euphoria : Happiness
represents the relationship among language?
(b) War : Peace
the given classes. (a) KECNKUHL (b) CHUECKN
(c) Speak : Hear
(c) HECNHUKL (d) HECNHULK
Planet, Earth, Venus (d) Anger : Rage
40. Two positions of the same dice are
33. In each letter of the English
shown. Select the number that will
alphabet is assigned an odd
be on the face opposite to the one
numerical value in increasing
showing 6.
(a) (b) (c) (d) order, such as A=1, B =3 and so on,
then what will be the code of 3 6
28. In the given Venn diagram, the HONEY? 5 5
1 3
triangle represents students (a) 132725747 (b) 152927949
playing table tennis,the rectangle (c) 132725745 (d) 152927947
represents students playing (a) 4 (b) 5
badminton, the circle represents 34. Select the number that can replace (c) 1 (d) 3
female students and the pentagon the question mark (?) in the
41. The sequence of folding a piece of
represents students playing following series.
paper and the manner in which the
football. The numbers given in the 40, 37, 43, 34, 46, ? folded paper has been cut is shown
diagram represent the number of (a) 31 (b) 41 in the following figures. How would
persons in that particular category. (c) 51 (d) 61 this paper look when unfolded?
35. Which of the option figures is the Question Figures
exact mirror image of the given
22 24 figure when the mirror is held at
7 12 3
24
the right side?
9 6 11 26
qv56jk89lm
9 6 Answer Figures
(a) m l 98 k j 6 5 v q (b) m l 8 k j 5v q
q v56 j k89 lm
(d) m 9 8 k j 65 q
l v
( c)
How many female students play
both table tennis and badminton? 36. Select the option that is related to
(a) 22 (b) 18 (c) 7 (d) 9 the third word in the same way as
(a) (b) (c) (d)
the second word is related to the
29. There is a family of five members first word.
K, L, M, N and O. Among them, 42. Select the option that is related to
Ministers : Council : : Sailors : ?
there is one married couple. O is the third word in the same way as
(a) Sea (b) Ship
unmarried and is the brother of K. the second is related to the first
(c) Captain (d) Crew
N is the sister of O. M is the only word.
married female and the mother of 37. The two given expression on both Medicine : Disease : : Food : ?
N. L and O are the only males in the side of the ‘=’ sign will have the (a) Energy (b) Nutrition
the group. same value if two numbers from (c) Thirst (d) Hunger

https://sscstudy.com/
https://sscstudy.com/

PRACTICE SET 01 5

43. Study the given pattern carefully 52. If the diagonal of a square is 12 cm, 61. A sum was invested on simple
and select the number that can then what is the area (in cm 2 ) of interest at a certain rate for 3 yr. If
replace the question mark (?) in it. the square? the interest rate is increased by
(a) 72 (b) 72 2 5%, then the interest increases by `
6 21 14
(c) 36 (d) 36 2 225. What is the sum (in `)
40 500 ?
invested?
8 25 7 53. A can do a piece of work in 12 days
(a) 1000 (b) 1500
and B in 15 days. With help of C,
(a) 98 (b) 84 (c) 1750 (d) 2000
they finish the work in 4 days. In
(c) 91 (d) 78
how many days C alone can do the 62. If P : Q = 2 : 3 and P : R = 4 : 5, then
44. How many rectangles are there in same work? what is (P + Q) : (Q + R)?
the given figures? (a) 10 (b) 12 (c) 8 (d) 15 (a) 20 : 27 (b) 20 : 17
54. A is twice as efficient as B and (c) 10 : 11 (d) 20 : 29
together they finish a piece of work x 5
63. If = , then what is the ratio of
in 18 days. How many days will A y 4
take to complete the work alone? (5x + 6 y) and (5x − 2 y)?
(a) 27 (b) 24 (a) 49 : 17 (b) 49 : 19
(c) 36 (d) 42 (c) 37 : 17 (d) 37 : 19
(a) 34 (b) 32 55. If chairs are bought at ` 500 each 64. The mean of 17 numbers is 12. If
(c) 30 (d) 35 and sold at ` 700 each, then what two numbers 9 and 15 are removed,
45. Select the figure that can replace will be the profit percentage? then what will be the mean of
the question mark (?) in the (a) 10% (b) 20% remaining number?
following series. (c) 30% (d) 40% (a) 11 (b) 12
Question Figures 56. An article is sold at 10% profit (c) 13 (d) 14
instead of 10% loss, the man gains 65. What is the HCF of 8, 12 and 15?
? ` 20 more. What is the cost price (in (a) 1 (b) 2
`) of that article? (c) 3 (d) 120
(a) 50 (b) 100
Answer Figures (c) 200 (d) 400 66. If a number is multiplied by
three-fourth of itself, then the
57. If P got 20% marks less than Q, value thus obtained is 10800. What
then the marks of Q is how much is that number?
per cent more than P? (a) 210 (b)180
(a) (b) (c) (d)
(a) 20 (b) 10 (c) 120 (d) 160
3
46. What is the value of 22 ? (c) 25 (d) 15
67. The number of boys in a class is
(a) 128 (b) 64 58. If the price of a commodity is three times the number of girls.
(c) 256 (d) 512 decreased by 20% and its Which of the following numbers
47. What is the sum of all natural consumption is increased by 40%, cannot represent the total number
numbers between 1 and 150 which then what will be the increase or of children in the class?
are multiples of 3? decrease in the expenditure of the (a) 44 (b) 48
(a) 3675 (b) 4235 commodity? (c) 40 (d) 42
(c) 3735 (d) 4415 (a) 12% decrease (b) 12% increase
(c) 20% increase (d) 20% decrease
68. Neela is now three times as old as
48. What is the sum of first 25 odd her daughter Leela. Ten years
natural number? 59. Speeds of a boat along the current back, Neela was five times as old as
(a) 475 (b) 575 and against the current are 16 Leela. The age of Leela is
(c) 600 (d) 625 km/h and 10 km/h respectively. (a) 15 yr (b) 25 yr
What is the speed of boat (in km/h) (c) 30 yr (d) 20 yr
49. What is the HCF of 12, 15 and 48? in still water?
(a) 5 (b) 1 (a) 4 (b) 12
69. A shepherd had 17 sheeps. All but
(c) 3 (d) 6 (c) 13 (d) 14
nine died. How many was he left
with ?
50. How many numbers less than 100 60. A bus travels some distance at a (a) 17 (b) 9 (c) Nil (d) 8
are multiples of both 3 and 4? speed of 12 km/h and returns at a
(a) 7 (b) 8 speed of 8 km/h. If the total time 70. In a zoo, there are Rabbits and
(c) 6 (d) 12
taken by the bus is 20 h, then what Pigeons. If heads are counted,
51. Which of the following number is is the distance there are 200 and if legs are
not divisible by 4? (in km)? counted, there are 580. How many
(a) 80 (b) 84
pigeons are there?
(a) 113424 (b) 213552
(c) 96 (d) 92 (a) 90 (b) 100 (c) 110 (d) 120
(c) 314250 (d) 52644

https://sscstudy.com/
https://sscstudy.com/

6 CUET (UG) Section III : General Test

Directions (Q. Nos. 71 and 72) In each (a) Only Conclusion I follows II. Discard old ideas because they
of the following questions two statements (b) Only Conclusion II follows are old.
alongwith a set of conclusions is given. (c) Only Conclusion III follows (a) Only Conclusion I follows
(d) All Conclusions follow (b) Only Conclusion II follows
You have to take the statements to be true
(c) Either I or II follows
even if they seem to be at variance from Directions (Q. Nos. 73 and 74) In the (d) Both I and II follow
the commonly known facts and decide following questions a statement is
which of the following conclusions followed by two conclusions I and II. Direction (Q. No. 75) In given
logically follows from the given statements. Taking the statement to be true decide question below is given a statement
71. Statements which of the given conclustions difinitely followed by two Assumptions I and II.
follows from the given statement. You have to consider the statement and
Some stars are planet.
73. Statement In a one day cricket the following assumptions and decide
All planets are galaxies. which of the assumptions is implicit in
match, the total runs made by a
Conclusions the statement.
team were 200. Out of these, 160
I. Some stars are galaxies. runs were made by spinners. Give Answer
II. Some galaxies are planet. (a) If only I is implicit
Conclusions
(a) Only Conclusion I follows (b) If neither I nor II is implicit
I. 80% of the team consists of (c) If only II is implicit
(b) Only Conclusion II follows spinners.
(c) Neither Conclusion I nor II follows (d) If both I and II are implicit
II. The opening batsmen were
(d) Both Conclusions I and II follow spinners. 75. Statement The Prime Minister is
72. Statements (a) Only Conclusion I follows expected to announce an expansion
(b) Only Conclusion II follows in his Ministry shortly. A
All teachers are students.
(c) Neither I nor II follows newspaper report.
No student is girl.
(d) Both I and II follow Assumptions
Conclusions I. The newspaper has quoted
74. Statement The old order changed
I. All girls are students. yielding place to new. authentic sources.
II. Some girls are students. II. The newspaper has reliable
Conclusions
III. Some students are teachers. sources for the news.
I. Change is the law of nature.

ANSWERS
1. (d) 2. (b) 3. (c) 4. (b) 5. (b) 6. (d) 7. (a) 8. (d) 9. (b) 10. (a)
11. (b) 12. (c) 13. (a) 14. (c) 15. (a) 16. (b) 17. (a) 18. (b) 19. (a) 20. (b)
21. (b) 22. (a) 23. (d) 24. (d) 25. (b) 26. (b) 27. (d) 28. (c) 29. (b) 30. (d)
31. (a) 32. (a) 33. (b) 34. (a) 35. (d) 36. (d) 37. (b) 38. (b) 39. (c) 40. (c)
41. (b) 42. (d) 43. (c) 44. (d) 45. (a) 46. (c) 47. (a) 48. (d) 49. (c) 50. (b)
51. (c) 52. (a) 53. (a) 54. (a) 55. (d) 56. (b) 57. (c) 58. (b) 59. (c) 60. (c)
61. (b) 62. (c) 63. (a) 64. (b) 65. (a) 66. (c) 67. (d) 68. (d) 69. (b) 70. (c)
71. (d) 72. (c) 73. (c) 74. (a) 75. (c)

https://sscstudy.com/
https://sscstudy.com/

PRACTICE SET 02 7

CUET (UG) Section III : General Test

Practice Paper 02
Instructions 60 questions to be attempted out of 75. Time : 60 Min

1. India has launched which 7. Which agency in India is 16. ‘Queensberry rules’ is the code
operation to bring back its responsible for development of followed in which sport?
nationals from Ukraine? technology for All Military (a) Tennis (b) Cricket
(a) Operation Yamuna Wings? (c) Boxing (d) Equestrian
(b) Operation Saraswati (a) IGMPP
17. With which sport is the term ‘Tee’
(c) Operation Ganga (b) Ministry of Defence
connected?
(d) Operation Bharat (c) Home Ministry
(a) Hockey (b) Polo
(d) DRDO
2. India, along with which country, (c) Golf (d) Badminton
agreed on ‘Roadmap on Blue 8. Which Fighter Aircraft is bought
18. Which two teams played the first
Economy and Ocean Governance’? from France?
official international cricket match?
(a) The USA (b) The UAE (a) Dhruv (b) Rafale
(a) England and Australia
(c) France (d) Japan (c) Mig-29 (d) Sukhoi-30
(b) England and West Indies
3. Which bank has collaborated with 9. Sukhoi-30 is procured from which (c) USA and Canada
NPCI to launch exclusive MSME country? (d) Australia and India
RuPay Credit Card for the Micro, (a) USA (b) Russia
Small and Medium Enterprises 19. The famous tennis player Steffi
(c) France (d) Germany
(MSMEs) industry? Graf belongs to which among the
(a) Punjab National Bank 10. Which helicopter is developed by following countries?
(b) Bank of Baroda Hindustan Aeronautics Limited? (a) USA (b) England
(c) State Bank of India (a) Dhruv (b) Tejas (c) Germany (d) Switzerland
(d) Union Bank of India (c) Nishant (d) Ajaya 20. How high is the badminton net at
4. With which country’s Air Force, 11. The Headquarter of International the center?
India has organised bilateral air Atomic Energy Agency is in (a) 5 feet (b) 5.1 feet
exercise titled ‘Eastern Bridge-VI’ (a) Geneva (b) Paris (c) 5.5 feet (d) 4.8 feet
at Jodhpur (Rajasthan)? (c) Vienna (d) Washington 21. Four pairs of words are given. Find
(a) Qatar (b) Oman the odd one out.
12. When did India join the United
(c) Kuwait (d) Saudi Arabia (a) 5th June : World Environment Day
Nations?
5. Which sports federation has (a) 1945 (b) 1947 (b) 22nd April : Earth Day
suspended Vladimir Putin as (c) 1950 (d) 1954 (c) 22nd March : World Water Day
Honorary President? (d) 22nd May : World Sparrow Day
13. The International Court of Justice
(a) International Judo Federation 22. With which sport is Karnam
sits in
(b) International Volleyball Malleswari associated?
(a) Geneva (b) The Hague
Federation (a) Tennis (b) Swimming
(c) Vienna (d) Rome
(c) International Football (c) Athletics (d) Weightlifting
Federation 14. The Non-Permanent members
(d) International Chess Federation of the Security Council are elected 23. The ‘Constitution Day’ of India is
for observed on
6. Which programme is launched for (a) 24th November (b) 25th November
(a) 1 year (b) 2 years
the development of a (c) 26th November (d) 27th November
(c) 3 years (d) 6 months
comprehensive range of missiles in
India? 15. Earth Summit was sponsored by 24. Which is the capital of Cyprus?
(a) DRDO (b) IGMDP (a) UNESCO (b) UNCED (a) Nicosia (b) Polis
(c) PRAKRAM (d) IRSP (c) WHO (d) UNICEF (c) Lamaca (d) Aradipou

https://sscstudy.com/
https://sscstudy.com/

8 CUET (UG) Section III : General Test

25. World Tuberculosis (TB) day is Answer Figures 36. Study the given pattern carefully
observed on? and select the number that can
(a) 28th March (b) 24th March replace the question mark (?) in it.
(c) 24th May (d) 28th May
21 18 32
26. Select the letter-cluster that can (a) (b) (c) (d) 19 22 ?
replace the question mark (?) in the 31. Select the option in which the 299 296 348
following series. numbers are related in the same (a) 30 (b) 24
SAT, VEW, YIZ, ?, EUF way as are the numbers in the (c) 28 (d) 14
(a) BUK (b) FIC (c) BOC (d) COD given set.
37. Pointing to the photograph of
27. Select the correct mirror image of (7, 98, 196)
Sanchi, Nitin said, “Her mother’s
the given figure when a mirror is (a) (11, 154, 308) (b) (20, 267, 520)
father’s son’s wife is my mother-in-
placed on its right side. (c) (15, 190, 380) (d) (18, 185, 360)
law’s only daughter”. How is Nitin
Question Figure 32. Which two signs and two numbers related to Sachin’s mother?
should be interchanged to make the (a) Paternal uncle
given equation correct? (b) Paternal grandfather
11 × 7 ÷ 35 − 64 + 56 = 47 (c) Maternal uncle
(a) + and − ; 7 and 11 (d) Brother
Answer Figures
(b) × and ÷ ; 35 and 56 38. T is 5 yr older to U. U is 8 yr
(c) × and ÷ ; 35 and 11 younger to V. V is 3 yr older to W.
(d) + and − ; 35 and 11 W is 9 yr younger to X. Who are of
33. Which of the following Venn diagram same age?
(a) (b) (c) (d) (a) W and T (b) W and V
best represents the relationship
28. In a certain code language between the following classes? (c) V and X (d) X and T
“CAUGHT” is written as Income tax payers, Employees, 39. From the given answer figures,
“326212087”. Then in this language Males select the one in which the question
“SOLDER” is written as figure is hidden/embedded.
(a) 1812122459 (b) 1915124359 Question Figure
(c) 2012152358 (d) 1912122359
29. In the question given below an (a) (b) (c) (d)
unfolded dice is given. You are
34. Select the figure that can replace
required to select the correct
the question mark (?) in the Answer Figures
answer choice (s) which is/are
following series.
formed by folding the unfolded dice.
Question Figures
Question Figure
2
? (a) (b) (c) (d)
1
3 6 40. Which answer figure will completes
Answer Figures
4 the pattern in the question figure?
5 Question Figure
Answer Figures
6 5 2 1
1 3 1 6 3 6 4 2 (a) (b) (c) (d)
?
(a) (b) (c) (d) 35. Select the option that is embedded
in the given figure X (rotation is Answer Figures
(a) Only A (b) Only B not allowed).
(c) Only A and B (d) Only C and D
Question Figure
30. The sequence of folding a piece of
paper and manner in which the (a) (b) (c) (d)
folded paper has been cut is shown
41. How many rectangles are there in
in the following figures. How would
the given figure?
this paper look when unfolded? Answer Figures
Question Figures

(a) (b) (c) (d) (a) 4 (b) 3 (c) 5 (d) 6

https://sscstudy.com/
https://sscstudy.com/

PRACTICE SET 02 9

42. If ‘+’ means ‘÷’, ‘×’ means ‘−’, ‘−’ consumption is increased by 20%, Directions (Q. Nos. 62-65) The line
means ‘+’ and ‘÷’ means ‘×’, then then what will be the increase or chart given below shows the number of
16 − 5 × 2 + 4 ÷ 12 = ? decrease in the expenditure of the cars stolen by City P and Q for the years
(a) 15 (b) 9 commodity? 2008 to 2016.
(c) 12 (d) 27 (a) 28% decrease (b) 28% increase
City P
(c) 20% increase (d) 20% decrease
43. If 15 C 5 D 45 = 135 and City Q

Number of cars stolen


1000 860 920
13 C 4 D 12 = 39, then 11 C 33 D 66 53. The marked price of an article is 770 760
800 700 670
=? 60% more than its cost price. If a 575 575 575
600 625
(a) 22 (b) 32 discount of 20% is given, then what 475
525
is the profit percentage? 400 525 425
(c) 44 (d) 11 395 375 425
(a) 14 (b) 16 200 250
44. In the following question, select the (c) 18 (d) 28 0
odd number-pair from the given

2008
2009
2010
2011
2012
2013
2014
2015
2016
alternatives. 54. The marked price of an article is
(a) 8-136 (b) 17-289 twice the cost price. For a gain of Year
(c) 6-102 (d) 19-361 20%, what should be the discount
percentage? 62. In which year from 2008 to 2016,
45. In the following question, select the (a) 20 (b) 40 the combined number of stolen cars
odd letters from the given (c) 45 (d) 60 from City P and Q is maximum?
alternatives. (a) 2008 (b) 2009
(a) MN (b) NM 55. If 6/7 of P = 4/5 of Q, then what is (c) 2012 (d) 2016
(c) FU (d) ZA P :Q ?
(a) 7 : 5 (b) 14 : 7
63. In how many years the number of
46. If the difference of two numbers is cars stolen from City P is more
(c) 14 :15 (d) 8: 7
4 and the difference of their than the number of cars stolen
squares is 64, then which is the 56. What is the net discount for two from City Q?
smaller of the two number? successive discounts of 20% and (a) 1 (b) 2
(a) 10 (b) 6 20%? (c) 3 (d) 5
(c) 4 (d) 8 (a) 36 (b) 40
(c) 42 (d) 44 64. What is the percentage decrease in
47. What is the value of 0.03010? the number of car stolen in year
(a)
301
(b)
301 57. A trader allows two successive 2015 from year 2014 from City P ?
100 1000 discounts of 20% and 10% while (a) 50 (b) 20
301 301 selling an article. If he gets ` 720
(c) (d) (c) 25 (d) 33.33
10000 100000 for that article, then what is the
marked price of the article? 65. The number of stolen cars from
48. A number when divided by 12 the two cities in year 2015 is
leaves remainder 9 is obtained. (a) 1000 (b) 1150
(c) 960 (d) 1200 what per cent of the total number
What will be the remainder when of cars stolen from the two cities
the same number is divided by 3? 58. If the ratio of edges of two cubes is in 2016?
(a) 1 (b) 0 2 : 3, then what is the ratio of the (a) 82.5 (b) 86.5
(c) 2 (d) 4 volume of the cubes? (c) 87.5 (d) 79.5
49. What is the LCM of 4, 6 and 9? (a) 8 : 27 (b) 4: 9
(c) 2 : 3 (d) 4 : 27 66. How many odd numbered pages are
(a) 18 (b) 36
there in a book of 1089 pages ?
(c) 1 (d) 42 59. A is twice as good as B and (a) 542 (b) 545
50. Working 12 h a day, Vaibhav can together they finish a piece of work (c) 544 (d) 546
complete a piece of work in 28 days. in 24 days. In how many days can
A alone to finish the work? 67. There were a total of 10 bicycles
How many hours a day should he and tricycles. If the total number of
(a) 27 (b) 36
work so as to finish the work in 21 wheels was 24, how many tricycles
(c) 42 (d) 48
days? were there ?
(a) 12 (b) 16 60. A man gains 10% by selling and (a) 2 (b) 6
(c) 15 (d) 18 article for a certain price. If he sells (c) 18 (d) 4
it at double the price, then what
51. If the price of an article decreases 68. A monkey climbs a 12 m high pole.
will be the profit percentage?
by 25%, then to restore its former In the first minute he climbs 2 m
(a) 110 (b) 120
value by how much per cent should and slips down in the next minute
(c) 150 (d) 140
the new price be increased? by 1 m. If this continues, then in
(a) 25 (b) 20 61. If chairs are bought at ` 400 each
how minutes will the monkey climb
(c) 33.33 (d) 10 and sold at ` 500 each, then what
at the top of the pole ?
will be the profit percentage?
52. If the price of a commodity is (a) 10 (b) 21
(a) 10% (b) 20%
decreased by 40% and its (c) 12 (d) 13
(c) 25% (d) 60%

https://sscstudy.com/
https://sscstudy.com/

10 CUET (UG) Section III : General Test

69. A group of 1200 persons consisting (c) Both Conclusions I and II follow 74. Statement
of captains and soldiers is travelling (d) Neither I nor II follows
Black cloud follows thunder.
in a train. For every 15 soldiers 72. Statements Rains follow thunder.
there is one captain. The number of All flowers are trees.
captain in the group is Conclusions
No fruit is tree. I. Thunder is the cause of rain.
(a) 85 (b) 80
(c) 75 (d) 70 Conclusions II. Black cloud is the cause of
I. No fruit is flower. thunder.
70. A person is given ` 1 for shooting at II. Some trees are flowers.
the target and if he misses. Then Direction (Q. No. 75) In each question
(a) Only Conclusion I follows
he has to give ` 1. If he gets ` 30 for below is given a statement followed by
(b) Only Conclusion II follows
100 chances, then how many two Assumptions I and II. An
(c) Both Conclusions I and II follow
chances did he miss ? assumption is something supposed or
(d) Neither I nor II follows
(a) 25 (b) 35 taken for granted. You have to consider
(c) 40 (d) 45 Directions (Q. Nos. 73 and 74) the statement and the following
In the following questions, each assumptions and decide which of the
Directions (Q. Nos. 71 and 72) In question has a statement followed by assumptions is implicit in the statement.
each of the following questions two two conclusions. Taking the statement Give Answer
statements followed by two or three to be true, decide which of the given (a) If Assumption I is implicit
conclusions are given. You have to take conclusions definitely follows from the (b) If Assumption II is implicit
the given two statements to be true even given statement. Indicate your answer as
if they seem to be at variance from (c) If neither I nor II is implicit
(a) if only I follows
commonly known facts. Read the (b) if only II follows (d) If both I and II are implicit
conclusion and then decide which of the (c) if neither I nor II follows 75. Statement Of all the newspapers
given conclusions logically follows from (d) if both I and II follow
published in Delhi, ‘The Time and
the two given statements.
73. Statement Good health is Space’ has the largest number of
71. Statements dependent on right eating habits. readers.
Some books are tables. Most of the people do not follow any
Some tables are mirrors. Assumptions
rule regarding eating.
Conclusions I. Volume of readership of all the
Conclusions newspapers in Delhi is known.
I. Some mirrors are books. I. Most of the people have poor
II. Some tables are books. health. II. No newspaper in Delhi other
(a) Only Conclusion I follows than ‘The Time and Space’ has
II. People are ignorant of proper
(b) Only Conclusion II follows large readership.
eating habits.

ANSWERS
1. (c) 2. (c) 3. (d) 4. (b) 5. (a) 6. (b) 7. (d) 8. (b) 9. (b) 10. (a)
11. (c) 12. (a) 13. (b) 14. (b) 15. (b) 16. (c) 17. (c) 18. (c) 19. (c) 20. (a)
21. (d) 22. (d) 23. (c) 24. (a) 25. (b) 26. (b) 27. (b) 28. (d) 29. (a) 30. (b)
31. (a) 32. (c) 33. (a) 34. (d) 35. (b) 36. (d) 37. (d) 38. (a) 39. (d) 40. (c)
41. (c) 42. (a) 43. (a) 44. (d) 45. (b) 46. (b) 47. (c) 48. (b) 49. (b) 50. (b)
51. (c) 52. (a) 53. (d) 54. (b) 55. (c) 56. (a) 57. (a) 58. (a) 59. (b) 60. (b)
61. (c) 62. (b) 63. (b) 64. (d) 65. (a) 66. (b) 67. (d) 68. (b) 69. (c) 70. (b)
71. (b) 72. (c) 73. (a) 74. (b) 75. (a)

https://sscstudy.com/
https://sscstudy.com/

PRACTICE SET 03 11

CUET (UG) Section III : General Test

Practice Paper 03
Instructions 60 questions to be attempted out of 75. Time : 60 Min

1. Which Union Ministry launched 8. Who was the first woman 17. Grammy Award is given in the field
the ‘National Strategy for Additive Cosmonaut in space? of
Manufacturing’? (a) Valentina Tereshkova (a) Literature (b) Music
(a) Ministry of Electronics and (b) Junko Tabei (c) Science (d) Invention
Information Technology (c) Marlyn Phipps
18. In which of the fields Nobel Prize is
(b) Ministry of MSME (d) Louis Frechette
not given?
(c) Ministry of Commerce and Industry 9. Which is the first country to make a (a) Physics (b) Mathematics
(d) Ministry of Science and Technology Constitution? (c) Chemistry (d) Medicine
2. France government has decided to (a) Sweden (b) Norway
19. Who received Booker Prize for the
withdraw its military from Mali (c) Japan (d) USA
Book ‘Half a Life’?
after nine years, and shift its 10. Which country is first to issue (a) Arundhati Roy
military base to which country? paper currency? (b) Anita Desai
(a) Chad (b) Senegal (a) Australia (b) USA (c) VS Naipaul
(c) Niger (d) Mauritania (c) China (d) New Zealand (d) Kiran Bedi
3. Which bank has become the first to 11. The famous Newspaper of 20. Who was the first recipient of
enter in the Metaverse? Germany is known as Dhyan Chand Award in 2002?
(a) Barclays (b) JPMorgan (a) The Age (b) Herald Tribune (a) Sachin Tendulkar
(c) Citibank (d) UBS (c) Die Welt (d) The Press (b) Aparna Ghosh
4. Which country hosted multilateral 12. Novosti is a major News Agency of (c) Arjun Chopra
which country? (d) Vishwanathan Anand
exercise MILAN 2022?
(a) India (b) France (a) USA (b) Russia 21. Which was the oldest Aircraft
(c) The UK (d) Malaysia (c) China (d) Germany Carrier of India?
13. UK’s famous News Agency is (a) INS Viraat
5. Who won Mexican Open 2022
(a) Associated Press (b) Reuters (b) INS Vikrant
men’s singles title?
(c) Europa Press (d) PTI (c) INS Vikramaditya
(a) Rafael Nadal (d) INS Mysore
(b) Cameron Norrie 14. Anna Karenina is the famous
(c) Daniil Medvedev work of 22. Which is an Air to Air Missile?
(d) Stefanos Tsitsipas (a) Stephen Hawking (a) Prithvi(b) Agni
(b) Charles Dickens (c) Akash (d) Astra
6. Who was the first man to walk in
(c) Sidney Sheldon
space? 23. Which of the following is a Surface
(d) Leo Tolstoy
(a) M Yuri Gagarin
to Air Missile?
(b) Alexey Leonov 15. Charles Dickens famous work is (a) Trishul
(c) Neil Arm Strong (a) Bleak House (b) Blood Cive (b) K-15 Sagarika
(d) Dennis Tito (c) Candide (d) Care of the Soul (c) Brahmos
(d) Agni
7. Who was the first woman President 16. Which award is considered the
most prestigious awards in cinema 24. Nuclear explosive devices were
of any country in the world?
world? tested in India at
(a) Junko Tabei
(a) Grammy Award (a) Sriharikota
(b) Arunima Sinha
(b) Booker Prize (b) Bengaluru
(c) Maria Estela Peran
(c) Jnanpith Award (c) Pokharan
(d) Margaret Thatcher
(d) Oscar Award (d) Kanchipuram

https://sscstudy.com/
https://sscstudy.com/

12 CUET (UG) Section III : General Test

25. What is the name of the Light 32. Which answer figure will complete 39. In the following question, select the
Combat Aircraft developed by India the pattern in the question figure? related number from the given
indigenously? Question Figure alternatives.
(a) Brahmos (b) Chetak 100 : 121 : : 36 : ?
(c) Astra (d) Tejas (a) 64 (b) 49 (c) 25 (d) 16
?
26. In the following question, select the 40. If a mirror is placed on the line AB,
number which can be placed at the then which of the answer figures is
sign of question mark (?) from the the right image of the given figure?
given alternatives.
Answer Figures Question Figure
21 18 32 A
19 22 ? 4
299 296 348 3 2
B
(a) 30 (b) 14 (a) (b) (c) (d)
(c) 28 (d) 24 Answer Figures
33. Arrange the given words in the
27. How many triangles are there in
sequence in which they occur in the
the given figure?
dictionary.
1. Paper 2. Parade (a) (b) (c) (d)
3. Postal 4. Praise
5. Picture 41. A piece of paper is folded and
(a) 4 (b) 6 punched as shown below in the
(a) 12543 (b) 21543
(c) 5 (d) 7 question figures. From the given
(c) 21534 (d) 12534
28. In a certain code language answer figures, indicate how it will
34. In the following question, select the appear when opened?
‘SANCTION’ is coded as
missing number from the given
‘XFSHODJI’. Then in the same Question Figures
series
code language ‘PROFOUND’ is
(need to write options).
coded as?
(a) UWTKKPIZ (b) VWUKMPIY 104, 52, 26, 13, 6. 5, ?
(c) UWTKJPIY (d) UXTKNQIY (a) 3.5 (b) 3.75
(c) 4.25 (d) 3.25
29. In a certain code language, ‘round’ Answer Figures
is written as ‘tall’, ‘tall’ is written 35. In the following question, which
as ‘red’, ‘red’ is written as ‘wood’, one of letters when sequentially
‘wood’ is written as ‘bread’ and placed at the gaps in the given
‘bread’ is written as ‘soil’. What do letter series shall complete it?
(a) (b) (c) (d)
we eat in breakfast? _b c a b _ a b c _ b _
(a) soil (b) wood (a) acac (b) bbca (c) caca (d) aacc 42. If ‘J’ menas ‘+’, ‘K’ means ‘−’, ‘T’
(c) bread (d) tall means ‘×’ and ‘U’ means ‘÷’ then
36. In the following question, select the
30. The present ages of P and Q are 15 odd letter from the given 18 T 3 U 27 J 2 = ?
yr and 21 yr respectively. What will alternatives. (a) 6 (b) 2
(c) 4 (d) 3
be the respective ratio of the ages (a) QP (b) DC
of P and Q after 6 yr? (c) XY (d) ML 43. If 8 × 16 × 9 = 8169 and
(a) 5 : 7 (b) 5 : 6 37. Identify the diagram that best 9 × 23 × 4 = 9234, then
(c) 3 : 4 (d) 7 : 9 4 × 10 × 11 = ?
represent the relationship among
31. From the given answer figures, the given classes. (a) 11014 (b) 41011
select the one in which the question Haryana, Chandigarh, Uttar Pradesh (c) 14610 (d) 10114
figure is hidden/embedded. 44. In the following question, select the
Question Figure odd word-pair from the given
alternatives.
(a) (b) (c) (d) (a) Ground : Play (b) Solar system : Sun
(c) Body : Hand (d) Tree : Stem
38. In the following question, select the 45. In the following question, select the
Answer Figures related word from the given related letters from the given
alternatives. alternatives.
Liver : Organ : : Earth : ? EI : AE : : UO : ?
(a) Round (b) Brown (a) PK (b) QJ
(a) (b) (c) (d) (c) Home (d) Planet (c) PJ (d) QK

https://sscstudy.com/
https://sscstudy.com/

PRACTICE SET 03 13

46. Which of the following number is 57. If the cost price of 5 articles is 66. A factory produced 1858509
divisible by 11? equal to the selling price of 8 cassettes in the month of January,
(a) 59609 (b) 45332 articles, then what is the loss 7623 more cassettes in the month
(c) 23581 (d) 44433 percentage (in %)? of February and owing to short
(a) 40 (b) 37.5 supply of electricity produced
47. What is the value of
(c) 50 (d) 56.25 25838 less cassettes in March than
[( a −2b3 ) ÷ ( a1b−1 )] × [( a 2b−4 ) in February. Find the total
÷ ( a −1b2 )] ? 58. Average of 9 consecutive numbers
production in all?
is 37. What is the smallest number
(a) b2 (b) 1/b2 (a) 5557312 (b) 5983245
out of these 9?
(c) a 2 (d) a 2b2 (c) 5564935 (d) 5608988
(a) 26 (b) 38
48. What is the unit of digit of (c) 29 (d) 33 67. The sum of the ages of 4 children
5124 × 1245 ? born at the intervals of 4 yr is 48.
59. Monthly income of X and Y is in
(a) 5 (b) 1 (c) 0 (d) 2 Find the age of the youngest child.
ratio of 5 : 11 respectively. The
(a) 4 yr (b) 5 yr
49. How many times digit ‘5’ appears difference in their income is
(c) 6 yr (d) 7 yr
in the number from 1 to 100? ` 24000. What will be the monthly
(a) 20 (b) 21 (c) 19 (d) 18 income of Y? 68. Mr. and Mrs. Gopal have two
50. What is the value of (a) ` 44000 (b) ` 20000 daughters and each daughter has
(c) ` 22000 (d) ` 33000 one brother. How many persons are
162 + 172 + 182 + ... 252 ?
there in the family ?
(a) 4325 (b) 4465 60. Total runs scored by three players
(c) 4105 (d) 4285 (a) 8 (b) 6 (c) 7 (d) 5
A, B, C and C are 1800. Ratio of
51. If a number is divided by 30 then it runs scored by A and B is 5 : 7 and 69. In a group of equal number of cows
leaves 17 as a remainder. What B and C is 3 : 2. How much runs and herdsmen, the number of legs
will be the remainder when the are scored by A? was 28 less than four times the
same number is divided by 10? (a) 756 (b) 612 number of heads. The number of
(a) 7 (b) 3
(c) 562 (d) 540 herdsmen was
(c) 1 (d) 2 (a) 7 (b) 28
61. Selling price of an article is ` 2695.
(c) 21 (d) 14
52. The marked price of an article is If 23% discount is given, then what
50% more than its cost price. If a is the marked price (in `)? 70. A book has 300 pages and each
discount of 10% is given, then what (a) 3700 (b) 3100 page has 20 lines of 10 words each.
is the profit percentage? (c) 3500 (d) 3800 How many words are there in the
book altogether ?
(a) 25 (b) 30 62. If x : y = 3 : 2 and x + y = 90, then the
(c) 35 (d) 20 (a) 6000 (b) 60000
value of ( x − y) is (c) 66000 (d) 600000
53. The marked price of an article is (a) 16 (b) 14
` 750 and a customer pays (c) 18 (d) 12 Directions (Q. Nos. 71 and 72) In each
` 600 for it. What is the discount of the following questions two statements
63. If the length of a rectangle is
percentage? followed by two or three conclusions are
increased by 25%, then by how
(a) 20 (b) 30 given. You have to take the given two
much per cent breadth should be
(c) 15 (d) 10 statements to be true even if they seem to
reduced so that the area remains
be at variance from commonly known
54. P can do a piece of work in 36 days. the same?
facts. Read the conclusion and then
If Q is 50% more efficient than P, (a) 15 (b) 20
decide which of the given conclusions
then in how many days can Q do (c) 12.5 (d) 25
logically follows from the two given
the same work? statements.
64. A train is moving at a speed of
(a) 18 (b) 24
84 km/h and covers a certain 71. Statements Some pins are clips.
(c) 12 (d) 21
distance in 12 h. If the same
Some clips are pens.
55. P is twice as good as Q and distance is to be covered in 14 h,
together they finish a piece of work then what will be the speed (in Conclusions
in 36 days. The number of days km/h) of the train? I. Some pins are pens.
taken by P alone to finish the (a) 72 (b) 76 II. Some pens are clips.
work? (c) 66 (d) 60 (a) Only conclusion I follows
(a) 48 (b) 60 (b) Only conclusion II follows
65. An amount of ` 35000 is invested in
(c) 54 (d) 72 (c) Both conclusions I and II follow
a scheme of compound interest. If
(d) Neither I nor II follows
56. If each edge of a cube is increased rate of interest is 20% per annum,
by 10%, then the percentage then what will be the amount (in `) 72. Statements
increase in its surface area is obtained after 2 yr? Some horses are lions.
(a) 22% (b) 19% (a) 56200 (b) 50400 All lions are foxes.
(c) 20% (d) 21% (c) 48600 (d) 42500

https://sscstudy.com/
https://sscstudy.com/

14 CUET (UG) Section III : General Test

Conclusions 73. Statement Workers feel highly 75. Two statements are given followed
I. No fox is horse. motivated when they get sense of by two Conclusions I and II. You
II. Some foxes are lions. involvement by participating in the have to consider the statement to
(a) Only Conclusion I follows management of companies. be true, even if it seems to be at
(b) Only Conclusion II follows Conclusions variance from commonly known
(c) Both Conclusions I and II follow I. Workers should be motivated facts. You are to decide which of
(d) Neither I nor II follows to produce more. the given conclusions can definitely
be drawn from the given statement.
II. Workers should be allowed to
Directions (Q. Nos. 73 and 74) Indicate your answer.
participate in the
In the following questions, each Statements
management of companies.
question has a statement followed by
All children are students.
two conclusions. Taking the statement 74. Statement Industrial revolution
All students are players.
to be true, decide which of the given which first of all started in
conclusions definitely follows from the Europe has brought about modern Conclusions
given statement. Indicate your answer age. I. All cricketers are students.
as Conclusions II. All children are players.
(a) If only I follows (a) Only Conclusion II follows
(b) If only II follows
I. Disparity between rich and
(b) Both Conclusions I and II follow
(c) If neither I nor II follows
poor results in revolution.
(c) Neither Conclusion I nor II follows
(d) If both I and II follows II. Revolution overhauls society. (d) Only Conclusion I follows

ANSWERS
1. (a) 2. (c) 3. (b) 4. (a) 5. (a) 6. (b) 7. (c) 8. (a) 9. (d) 10. (c)
11. (c) 12. (b) 13. (b) 14. (d) 15. (a) 16. (d) 17. (b) 18. (b) 19. (b) 20. (b)
21. (a) 22. (d) 23. (a) 24. (c) 25. (d) 26. (b) 27. (b) 28. (c) 29. (a) 30. (d)
31. (d) 32. (d) 33. (d) 34. (d) 35. (a) 36. (c) 37. (d) 38. (d) 39. (b) 40. (c)
41. (a) 42. (c) 43. (b) 44. (a) 45. (d) 46. (a) 47. (b) 48. (c) 49. (b) 50. (d)
51. (a) 52. (c) 53. (a) 54. (b) 55. (c) 56. (d) 57. (b) 58. (d) 59. (a) 60. (d)
61. (c) 62. (c) 63. (b) 64. (a) 65. (b) 66. (c) 67. (c) 68. (d) 69. (d) 70. (b)
71. (b) 72. (b) 73. (d) 74. (b) 75. (a)

https://sscstudy.com/
https://sscstudy.com/

PRACTICE SET 04 15

CUET (UG) Section III : General Test

Practice Paper 04
Instructions 60 questions to be attempted out of 75. Time : 60 Min

1. National Means-cum-Merit 9. Male is the capital of which 18. Hopmen Cup is the famous
Scholarship Scheme (NMMSS) has Country? Tournament of which sport?
been extended till which year? (a) Mauritius (b) Lakshadweep (a) Lawn Tennis (b) Table Tennis
(a) 2023-24 (b) 2025-26 (c) Maldives (d) Malaysia (c) Badminton (d) Golf
(c) 2029-30 (d) 2031-32
10. Vientiane is the capital city of 19. Term ‘Spikers’ is related to which
2. Which country will become the which country? sport?
first country to adopt India’s UPI (a) Fiji (b) Ecuador (a) Football (b) Hand ball
platform? (c) Laos (d) Togo (c) Billiards (d) Volleyball
(a) Nepal (b) Myanmar
11. Which is the latest country to be 20. Who was the first person to stay in
(c) Bangladesh (d) Sri Lanka
admitted as new member state of Rashtrapati Bhawan?
3. Which city in India has highest UN? (a) William Bentinck
number of dollar-millionaire (a) Switzerland (b) East Timor (b) Lord Irwin
households according to the Hurun (c) South Sudan (d) Montenegro (c) C Rajagopalachari
India Wealth Report 2021? (d) Lord Canning
12. Where is the permanent
(a) Bengaluru (b) Kolkata
Secretariat of the SAARC? 21. What is the range of Agni V
(c) Delhi (d) Mumbai
(a) New Delhi (b) Islamabad missiles?
4. Which regiment’s units were (c) Columbo (d) Kathmandu (a) 5000 km (b) 6000 km
honoured with the President’s (c) 8000 km (d) 10000 km
13. Where is the headquarter of
Colours on 23rd February? 22. First aircraft carrier built in India is
INTERPOL?
(a) Rajputana Rifles
(a) Paris (b) London (a) INS Viraat (b) INS Vikrant
(b) Parachute Regiment
(c) Lyons (d) Geneva (c) INS Arihant (d) INS Kalvart
(c) Sikh Regiment
(d) Jat Regiment 14. Headquarter of World Health 23. Barak–8 is a missile which is
Organisation (WHO) is situated codeveloped by India and
5. Which Indian city has been elected
at (a) Israel (b) Japan
as the host of International Olympic
(a) Hague (b) Oslo (c) US (d) France
Committee’s (IOC) session 2023?
(c) Geneva (d) New York
(a) New Delhi (b) Mumbai 24. India’s National flag was adopted on
(c) Chennai (d) Hyderabad 15. The Panchatantra fables are (a) 22nd July, 1947
thought to be composed by (b) 26th January, 1950
6. Which is the 1st Spaceship landed (a) Mullah Nasruddin (c) 24th January, 1950
on Mars? (b) Vishnu Sharma (d) 15th August, 1947
(a) Chandran-I (b) Viking-I (c) King Sudharshan
(c) Columbus-IV (d) Atlanta-III 25. When was national calendar
(d) Tenali Raman
adopted?
7. Who was the first cloned human 16. Sendy Lodge Stadium is located in (a) 22nd July, 1947
baby of world? which country? (b) 24th January, 1950
(a) Dolly (b) Eve (a) USA (b) Australia (c) 26th January, 1950
(c) Oliver (d) Jack (c) Scotland (d) England (d) 22nd March, 1957

8. Which is the largest sea port of the 17. Twickenham Stadium is related to 26. Which of the following Venn
world? which sport? diagram best represents the
(a) Shanghai (b) Mumbai (a) Football (b) Cricket relationship between the following
(c) Los Angels (d) San Diago (c) Rugby Football (d) Boxing classes?

https://sscstudy.com/
https://sscstudy.com/

16 CUET (UG) Section III : General Test

Cricketers, Players, Human beings 32. Select the option in which the 38. Select the letter-cluster that can
words share the same relationship replace the question mark (?) in the
as that shared by the given words following series.
pair TULG, WRPC, ZOTY, CLXU, ?
(a) (b) (c) (d) Player : Team (a) FJCQ (b) FIBQ
(a) Flower : Bouquet (c) FICR (d) GIAQ
27. Four number-pairs have been given (b) Ant : Bee
out of the which three are alike in 39. Select the letter that can replace
(c) Student : Teacher
same manner and one is different. the question mark (?) in the
(d) Purpose : Crowd
Select the number-pair that is following series.
different from the others. 33. Four words have been given, out of A, T, E, U, I, V, O, W, ?, X
(a) 4209 : 15 (b) 5121 : 12 which three are alike in same (a) R (b) M (c) U (d) Z
(c) 8101 : 10 (d) 7456 : 22 manner and one is different. Select
the odd word. 40. Select the number that can replace
28. The sequence of folding a piece of (a) Kidney (b) Pancreas the question mark (?) in the
paper and the manner in which the (c) Renin (d) Liver following series.
folded paper has been cut is shown 45, 47, 53, 65, 85?
in the following figures. How would 34. How many triangles are there in
(a) 115 (b) 130 (c) 145 (d) 105
this paper look when unfolded? the given figure?
41. Select the option that is related to
Question Figures
the third number in the same way
as the second number is related to
the first number.
13 : 109 : : 24 : ?
Answer Figures (a) 194 (b) 201 (c) 197 (d) 216
(a) 25 (b) 24 42. Select the option in which the
(c) 27 (d) 26
numbers are related in the same
35. Select the figure that can replace way as the numbers in the given set.
(a) (b) (c) (d) the question mark (?) in the (8, 7, 407)
following series. (a) (12, 8, 555) (b) (10, 8, 576)
29. In a certain code language
Question Figures (c) (11, 12, 450) (d) (9, 6, 339)
‘AUSTERE’ is coded as ‘13102595’.
How will ‘SETTING’ be coded as in 43. Which two signs and two numbers
that language? ? should be interchanged to make the
(a) 11544957 (b) 10522948 given equation correct?
(c) 10522957 (d) 11522867 Answer Figures 28 − 32 ÷ 2 × 8 + 34 = 132
30. Select the correct mirror image of (a) × and − ; 32 and 34
the given figure when a vertical (b) + and − ; 28 and 32
mirrors is placed on the right side (c) + and − ; 32 and 34
of the figure. (d) × and − ; 8 and 2
(a) (b) (c) (d)
Question Figure 44. ‘Criticise’ is related to ‘Condemn’ in
A
36. A + B means ‘A, is the mother of B’; the same way as ‘Solidarity’ is
A − B means ‘A is the brother of B’ ; related to’ ……… .
A × B means ‘A is the father of B’; (a) Unity (b) Atrocious
B
A ÷ B means ‘A is the daughter of B’. (c) Guilty (d) Prosperity
Answer Figures
If P − K × Y − J ÷ S + R, then which 45. Select the option in which the given
of the following statements is not figure X is embedded (rotation is
correct? not allowed).
(a) J is daughter of P. Question Figure
(a) (b) (c) (d) (b) P is paternal uncle of R.
(c) K is husband of S.
31. Study the given pattern carefully
(d) Y is son of S.
and select the number that can
replace the question mark (?) in it. 37. Four letter-clusters have been
given, out of which three are alike Answer Figures
42 37 ?
in same manner and one is
18 24 36
different. Select the odd
252 296 540 letter-cluster.
(a) 45 (b) 55 (a) PSVX (b) ZCFI
(c) JMPS (d) ORUX (a) (b) (c) (d)
(c) 40 (d) 50

https://sscstudy.com/
https://sscstudy.com/

PRACTICE SET 04 17

46. A can do a piece of work in 20 days 55. A man saves 30% of his income in 62. S6 are what per cent of total
and B can do the same piece of 1 yr. If he wants to save the same number of shoes?
work in 30 days. They start amount in 8 months, then by how (a) 60 (b) 30
working together and work for 5 much percentage should he (c) 20 (d) 25
days and then both leave the work increase his monthly savings? 63. What is the average number
C alone finished the remaining (a) 20 (b) 30 of shoes of all 6 types of
work in 14 days. In How many (c) 40 (d) 50 shoes?
days will C alone finish the whole
56. If some articles are bought at ` 10 (a) 166.67 (b) 333.33
work?
each and sold at `7 each, then what (c) 116.66 (d) 163.33
(a) 24 (b) 18
(c) 36 (d) 42 is the loss percentage? 64. S4 is what per cent of S3?
(a) 60% (b) 16.67% (a) 90 (b) 80
47. Marked price of an article is 40% (c) 25% (d) 30% (c) 70 (d) 75
more than its cost price. If a
discount of 45% is given, then what 57. By selling 50 m of cloth, a person 65. S2 are how much per cent more
will be the loss percentage? gains the cost price of 20 m of cloth. than S5?
(a) 23 (b) 29 What is his gain per cent?
(a) 350 (b) 150
(c) 19 (d) 25 (a) 40 (b) 25 (c) 250 (d) 300
(c) 20 (d) 10
48. If 123457Y is completely divisible 66. In a company 60% workers are
by 8, then what will be the digit in 58. Average of 9 numbers is 20. If a males. If the number of female
place of Y? number 30 is also included, then workers in the company is 800.
(a) 4 (b) 5 what will be the average of these What is the number of male
(c) 8 (d) 6 10 numbers? workers in the company ?
49. A number when divided by 18 (a) 20.5 (b) 21 (a) 1200 (b) 1600
leaves remainder 15. What is the (c) 19.5 (d) 21.5 (c) 1900 (d) 1400
remainder when the same number 59. B starts some business by investing 67. Aarna cut a cake into two halves
is divided by 6 ? ` 90000. After 4 months, D joins and cuts one half into smaller
(a) 3 (b) 2 business by investing ` 80000. At pieces of equal size. Each of the
(c) 1 (d) 4 the end of the year, in what ratio small pieces is twenty grams in
50. What is the value of will they share the profit? weight. If she has seven pieces of
1 3 2 1 (a) 10 : 7 (b) 9 : 4 the cake in all with her, how heavy
8 +5 +4 +3 ?
7 7 7 7 (c) 27 : 16 (d) 7 : 3 was the original cake ?
(a) 22 (b) 23 60. A man gains 25% by selling an (a) 120 g (b) 140 g
(c) 24 (d) 21 (c) 240 g (d) 280 g
article at a certain price. If he sells
51. What is the value of 13 + 14 ..... 20? the same article at half the price, 68. A certain number of horses and an
(a) 182 (b) 144 then what is the loss percentage? equal number of men are going
(c) 132 (d) 124 (a) 62.5 (b) 27.5 somewhere. Half of the owner are
(c) 37.5 (d) 32.5 on their horse’s back while the
52. What is the digit in unit’s place of
remaining ones are walking along
the product 25 × 252 × 3 7? 61. What is the value of
leading their horses. If the
(a) 1 (b) 2 19 + 8 3 ? number of legs walking on the
(c) 0 (d) 5 ground is 70, how many horses are
(a) 4 + 3 (b) 4 − 3
53. Which smallest number to be (c) 8 + 3 (d) 8 − 3 there ?
subtracted from 300, so that the (a) 10 (b) 12
resulting number is completely Directions (Q. Nos. 62-65) The bar (c) 14 (d) 16
divisible by 9? chart given below shows the stock of 6 69. Mani is double the age of Prabhu,
(a) 5 (b) 6 different types of shoes in a store. Ramona is half the age of Prabhu.
(c) 3 (d) 1 350 If Mani is sixty years old, find out
300
54. A man spends 80% of his income 300 the age of Ramona.
and saves the rest. If his income 250 220 210 (a) 20 yr (b) 15 yr
Stock

and spending both increases by 200 (c) 10 yr (d) 24 yr


10%, then what is the percentage 150 120
90 70. John’s age is 42 yr and Kelvin’s age
change in his savings? 100 60 is 26 yr and how many years ago
(a) 10% increase 50
was Kelvin’s age half of John’s
(b) 5% decrease 0
S1 S2 S3 S4 S5 S6 age ?
(c) 5% increase (a) 6 yr (b) 4 yr
Type of shoes
(d) 15% decrease (c) 10 yr (d) 8 yr

https://sscstudy.com/
https://sscstudy.com/

18 CUET (UG) Section III : General Test

Directions (Q. Nos. 71 and 72) In each (c) Both Conclusions I and II follow Assumptions
of the following questions two statements (d) Neither I nor II follows I. Wait-listed candidates get
followed by two or three conclusions are admission with difficulty.
given. You have to take the given two
Directions (Q. Nos. 73 and 74) In each
question below is given a statement II. A large number of candidates
statements to be true even if they seem to were on the waiting list.
be at variance from commonly known followed by two Assumptions I and II. An
facts. Read the conclusion and then assumption is something supposed or 75. A statement is given, followed by
decide which of the given conclusions taken for granted. You have to consider two Conclusions I and II. You have
logically follows from the two given the statement and the following to consider the statement to be true
statements. assumptions and decide which of the even, if it seem to be at variance
assumptions is implicit in the statement. from commonly known facts. You
71. Statements have to decide which of the given
Given answer
All wheels are brakes. (a) If Assumption I is implicit conclusions, if any, follow from the
Some gears are wheels. (b) If Assumption II is implicit given statement.
Conclusions (c) If neither I nor II is implicit
Statement
(d) If both I and II are implicit
I. No brake is gear. The Government of India has
II. Some gears are brakes. 73. Statement Apart from the recently announced several
(a) Only Conclusion I follows entertainment value of television, concessions and offered attractive
(b) Only Conclusion II follows its educational value cannot be package tours to foreign visitors.
(c) Both Conclusions I and II follow ignored.
Conclusions
(d) Neither I nor II follows Assumptions I. Now more number of foreign
72. Statements I. People take the television to be tourists will visit India.
All men are dogs. means of entertainment only. II. The Government of India
II. The educational value of seems to be serious in
All dogs are cats. television is not realised attracting tourists.
Conclusions properly. (a) Only Conclusion I follows
I. All men are cats. 74. Statement Ten candidates who (b) Only Conclusion II follows
II. All cats are men. were on the waiting list could (c) Neither Conclusion I nor II follows
(a) Only Conclusion I follows (d) Both Conclusions I and II follow
finally be admitted to the course.
(b) Only Conclusion II follows

ANSWERS
1. (b) 2. (a) 3. (d) 4. (b) 5. (b) 6. (b) 7. (b) 8. (a) 9. (c) 10. (c)
11. (c) 12. (d) 13. (c) 14. (c) 15. (b) 16. (c) 17. (c) 18. (a) 19. (d) 20. (b)
21. (a) 22. (b) 23. (a) 24. (a) 25. (d) 26. (c) 27. (b) 28. (b) 29. (c) 30. (b)
31. (a) 32. (a) 33. (c) 34. (a) 35. (d) 36. (a) 37. (a) 38. (b) 39. (c) 40. (a)
41. (c) 42. (d) 43. (c) 44. (a) 45. (c) 46. (a) 47. (a) 48. (d) 49. (a) 50. (d)
51. (c) 52. (c) 53. (c) 54. (a) 55. (d) 56. (d) 57. (a) 58. (b) 59. (c) 60. (c)
61. (a) 62. (b) 63. (c) 64. (d) 65. (c) 66. (a) 67. (c) 68. (c) 69. (b) 70. (c)
71. (b) 72. (a) 73. (d) 74. (a) 75. (a)

https://sscstudy.com/
https://sscstudy.com/

PRACTICE SET 05 19

CUET (UG) Section III : General Test

Practice Paper 05
Instructions 60 questions to be attempted out of 75. Time : 60 Min

1. The Smart Card Arms License and 9. Official report of the Government 17. Who was the first Indian to travel
Shastra App has been launched by of Japan is known as in space?
which the police department of (a) Blue Book (b) Grey Book (a) Rakesh Sharma
which State/UT? (c) Yellow Book (d) Green Book (b) Kalpana Chawla
(a) Uttar Pradesh (b) Maharashtra (c) Ravish Malhotra
10. What does ‘Satyameva Jayate’
(c) Delhi (d) Tamil Nadu (d) Sunita Williams
mean?
2. The World Sustainable Development (a) ‘Truth alone triumphs’ 18. The first Central Agricultural
Summit is an annual flagship event (b) ‘True Faith is Rare’ University of India was constructed
of which organisation? (c) ‘Truth is Divine’ in
(a) World Wide Fund for Nature-India (d) ‘Truth is a Treasure’ (a) Manipur (b) Uttar Pradesh
(b) Confederation of Indian Industry (c) Bihar (d) Assam
11. Which among the following is not
(c) The Energy and Resources Institute
an official language of United 19. Rukmini Satellite is dedicated to
(d) Quality Council of India
Nation? which purpose?
3. Name the bank which has recently (a) Russian (b) German (a) Education
acquired the highest stake in India (c) Arabic (d) French (b) Remote Sensing
Debt Resolution Company Ltd (c) Agriculture
12. The Indian recipient of Noble Peace
(IDRCL)? (d) Defence
Prize is
(a) Canara Bank (a) Hargovind Khurana 20. Central tobacco research Institute
(b) State Bank of India (b) Rabindranath Tagore is locate at
(c) Punjab National Bank (c) Mother Teresa (a) Rajahmundry (b) Coimbatore
(d) Union Bank of India (d) Amartya Sen (c) Hyderabad (d) Pune
4. Which nation has successfully 13. First Nobel Prize in Economics was 21. Who wrote the National song of
tested its new naval air defence given in India?
system called ‘C-Dome’? (a) 1967 (b) 1968 (a) Rabindranath Tagore
(a) Israel (b) Iran (c) 1970 (d) 1969 (b) Bankim Chandra Chatterjee
(c) Turkey (d) North Korea (c) Mohammed Iqbal
14. Pulitzer Prize is given in which
(d) Chitragupta
5. Sadia Tariq won a gold medal for field?
India in which event? (a) Music (b) Movie 22. Which is the largest fresh water
(a) Wushu (b) Fencing (c) Literature (d) Sport lake in the world?
(c) Boxing (d) Weightlifting (a) Lake Victoria (b) Lake Erie
15. The Magsaysay Award for Social
(c) Lake Superior (d) Lake Ontario
6. Name the capital of Uganda? Service was instituted by
(a) Mogadishu (b) Kampala (a) The Government of India 23. National Good Governance Day is
(c) Lusaka (d) Bulenga (b) The UNO observed in India on
(c) The ILO (a) 24th December
7. Denmark Legislature is called as (d) Philippines Government (b) 25th December
(a) Shora (b) Diet (c) 26th December
16. Who is the first Indian to fly a
(c) Tasongidu (d) Folketing (d) 31st December
plane?
8. White paper is the official paper of (a) JRD Tata 24. Which is the fastest land animal in
which government? (b) Mihir Sen the World?
(a) Japan (b) France (c) Saifuddin Kitchlew (a) Dog (b) Cheetah
(c) India (d) Iran (d) WC Banerjee (c) Tiger (d) Horse

https://sscstudy.com/
https://sscstudy.com/

20 CUET (UG) Section III : General Test

25. When is International day for (c) Ductile : Bendable 3. Million


Preservation of Ozone layer (d) Integrity : Honesty 4. Octillion
observed? 33. In a certain code language ‘PAGE’ 5. Trillion
(a) 16th September (b) 4th July is coded as ‘161495’. How will (a) 3, 5, 1, 4, 2 (b) 4, 3, 5, 1, 2
(c) 23rd January (d) 1st May ‘PART’ be coded as in that (c) 2, 3, 5, 1, 4 (d) 3, 5, 1, 2, 4
26. Study the given pattern carefully language?
40. In the given Venn diagram, the
and select the number that can (a) 16125020 (b) 16132420
‘rectangle’ represents travellers
replace the question mark (?) in it. (c) 16128620 (d) 16136220
who like to travel by plane, the
? 13 49 34. Four words have been given, out of ‘circle’ represents ‘travellers who
9 17 69
which three are alike in some like to travel by bus’, and the
manner and one is different. Select ‘triangle’ ‘represents travellers who
13 11 59
the odd word. like to travel to train’. The numbers
(a) 9 (b) 5 (c) 10 (d) 21 (a) Clarinet (b) Trumpet given in the diagram represents
(c) Xylophone (d) Saxophone the number of travellers in that
27. Select the letter-cluster that can
replace the question mark (?) in the 35. ‘Astronomy’ is related to ‘Stars’ in particular category.
following series. the same way as ‘Agronomy’ is
ADG, CGK, EJO, ?, IPW related to ‘………’.
(a) GMZ (b) GNS (a) Mines (b) Crops 10 11
(c) Planets (d) Emotions 2
(c) GNZ (d) GMS
5 9 6
28. A + B means ‘A is the husband of B’; 36. Which two signs should be
interchanged to make the given 13
A − B means, ‘B is the sister of A’
equation correct?
A × B means ‘A is the mother of B’;
A ÷ B means ‘B is the son of A’. 12 + 81 − 27 × 9 ÷ 3 = 36
(a) − and ÷ (b) ÷ and × How many travellers like to travel
If P + R × T − Q ÷ S + U, then how either by train or plane but not by
(c) × and − (d) + and ×
is P related to S? bus?
(a) Maternal grandfather 37. Three different positions of the (a) 36 (b) 27
(b) Father-in-law same dice are shown. Select the (c) 30 (d) 29
(c) Paternal grandfather symbol that will be on the face
(d) Uncle opposite to the one showing +. 41. Select the option that depicts how
the given transparent sheet of
29. Four number-pairs have been paper would appear if it is folded at
# % #
given, out of which three are alike @ % the dotted line.
$
in some manner and one is @ * +
Question Figure
different. Select the number-pair
that is different from the rest. (a) # (b) *
(a) 11 : 121 (b) 13 : 169 (c) @ (d) $
(c) 17 : 289 (d) 15 : 250
38. Select the correct mirror image of
30. Select the number that can replace the given figure when a mirror is
the question mark (?) in the placed on the right of the figure. Answer Figures
following series. Question Figure
5, 10, 26, ?, 122, 170
(a) 82 (b) 50
(c) 65 (d) 77
(a) (b) (c) (d)
31. Four letter-clusters have been
given, out of which three are alike 42. How many triangles are there in
in some manner and one is the given figure?
different. Select the odd Answer Figures
letter-cluster.
(a) BJT (b) GCV
(c) DDP (d) FBL
32. Select the option in which the words (a) (b) (c) (d) (a) 32 (b) 30
share the same relationship as that (c) 33 (d) 44
shared by the given pair of words. 39. Arrange the following words in a
logical and meaningful order. 43. Select the option that is embedded
North : South
(a) Frown : Smile
1. Quadrillion in the given figure. (Rotation is not
(b) Whole : Total 2. Quintillion allowed)

https://sscstudy.com/
https://sscstudy.com/

PRACTICE SET 05 21

Question Figure 49. The length of two parallel sides of 59. X alone can complete a work in
trapezium are 30 cm and 40 cm. 6 days and Y alone can complete
If the area of the trapezium is the same work in 30 days. If X
350 cm 2, then what is the value and Y work together, then in
(in cm) of its height? how many days work will be
(a) 8 (b) 10 completed?
Answer Figures (c) 15 (d) 12 (a) 5 (b) 4
(c) 6 (d) 4.5
50. P, Q and R undertook a work for
`48000. Together P and Q complete 60. The population of a town increases
5/12th part of the work. What is at the rate of 15% per annum. If
(a) (b) the present population is 108445 of
the share (in `) of R ?
town, then what was the
(a) 21000 (b) 28000
(c) 27000 (d) 31000
population 2 yr ago?
(a) 72000
51. When 80 is subtracted from 20% of (b) 79000
a number, the result is 30. What is (c) 82000
(c) (d)
the value of the number? (d) 85000
44. Select the figure that can replace (a) 330 (b) 440 61. A 450 m long train crosses a bridge
the question mark (?) in the (c) 550 (d) 220
650 m long in 36 s. What is speed
following series.
52. If the rice is sold at ` 48 per kg, (in km/h) of the train?
Question Figures then there would be a 20% loss. To (a) 110 (b) 125
earn a profit of 20% what should be (c) 150 (d) 95
J Q
M
V
? the price of rice (per kg)? 62. A car travels at a speed of 25 m/s
(a) 72 (b) 76 for 8 hours. What is the distance
(c) 78 (d) 84 (in km) travelled by the car?
Answer Figures
53. A number is first decreased by (a) 360 (b) 720
AB 30% and then increased by 30%. If (c) 450 (d) 900
B
BA A the number so obtained is 72 less 63. If the ratio of the cost price and
than the original number, then selling price of an article is 5 : 6,
(a) (b) (c) (d)
what is the value of the original then what will be the profit
45. Select the number that can replace number? percentage?
the question mark (?) in the (a) 720 (b) 800 (a) 10 (b) 12
following series. (c) 960 (d) 1080 (c) 15 (d) 20
62, 66, 74, 90, ? 54. What is the unit digit of 64. The average of 6 numbers is 18. If
(a) 106 (b) 120 342 × 743 + 175? one number is excluded, the
(c) 116 (d) 122 (a) 1 (b) 2 average becomes 17. What is the
(c) 3 (d) 7 excluded number?
46. The ratio of two positive numbers
is 9 : 11. Their product is 6336. 55. Which one is the largest fraction (a) 22 (b) 23
What is the smallest number? among 3/4, 7/8 and 5/6? (c) 21 (d) 20
(a) 32 (b) 72 (a) 3/4 (b) 7/8 65. If 2/3 of P = 1 / 5 of Q, then what is
(c) 88 (d) 48 (c) 5/6 (d) All are equals
P : Q?
47. If a shopkeeper marks the price of 56. What is the value of (a) 3 : 5 (b) 3 : 10
goods 40% more than their cost 2 + 2 −1 + 2 2 + 2 −2 ? (c) 3 : 7 (d) 3 : 9
price and allows a discount of 40%,
(a) 27/4 (b) 9/4 66. The total age of a mother and her
then what is his gain or loss
(c) 9/5 (d) 25/4 daugher is 60 yr. The difference
percent?
between their age is 30 yr. Find out
(a) 16% loss (b) 16% profit 57. If 1062 = 11236 , then what is the
the age of mother.
(c) 10% loss (d) 12% profit value of (a) 40 yr (b) 50 yr
48. If the difference between discount 112.36 + 11236? (c) 45 yr (d) 55 yr
of 35% and two successive (a) 106.6 (b) 100.6
67. Which number will be in the
discounts of 20% on a certain bill is (c) 116.6 (d) 126.6
middle if the following numbers are
`3, then what is the amount (in `) 58. Which of the given values is arranged in descending order?
of the bill? completely divisible by 18?
(a) 250 (b) 300
4456, 4465, 4655, 4665, 4565
(a) 1642 (b) 3612 (a) 4456 (b) 4465
(c) 350 (d) 400
(c) 7218 (d) 2427 (c) 4565 (d) 4655

https://sscstudy.com/
https://sscstudy.com/

22 CUET (UG) Section III : General Test

Directions (Q. Nos. 68-70) Read the Conclusions Assumptions


following information and answer the I. All houses are pens. I. Railway trains are the only
following questions. II. Some houses are pens. mode of transport available in
In a class of 40 students, 28 can speak III. All pens are houses. the suburbs of Mumbai.
Tamil and 30 can speak Telugu. (a) Only Conclusion I and II follows II. Only railway trains run
All students can speak at least one of the (b) Only Conclusion II and III follows punctually.
two languages. (c) Both Conclusions I and III follow
74. Statement The mangoes are too
(d) No conclusion follow
68. Find the number of students who cheap to be good.
can speak only Tamil. 72. Statements
Assumptions
(a) 8 (b) 10 All pens are chalks.
I. When the mango crop is
(c) 12 (d) 14 All chairs are chalks.
abundant, the prices go down.
69. Find the minimum number of Conclusions II. The lower the selling price, the
students who can speak both Tamil I. Some pens are chairs. inferior is the quality of the
II. Some chalks are pens. commodity.
and Telugu.
III. Some chalks are chairs.
(a) 12 (b) 15 (a) Only Conclusion I and II follows 75. Two statements are given followed
(c) 18 (d) 22
(b) Only Conclusion II and III follows by two conclusions I and II. You
70. Find the number of persons who (c) Both Conclusions I and III follow have to consider the statements to
can speak only Telugu? (d) No conclusion follow be true even if they seem to be at
(a) 8 (b) 10 Directions (Q. Nos. 73 and 74) In each variance from commonly known
(c) 12 (d) 14 question below is given a statement facts. You have to decide which of
Directions (Q. Nos. 71 and 72) In each followed by two Assumptions I and II. An the given conclusions, if any, follow
assumption is something supposed or from the given statements. Indicate
of the following questions two statements
taken for granted. You have to consider your answer.
followed by two or three conclusions are the statement and the following
given. You have to take the given two assumptions and decide which of the Statements
statements to be true even if they seem to assumptions is implicit in the statement. All animals are dogs.
be at variance from commonly known Give Answer All dogs are birds.
facts. Read the conclusion and then (a) If Assumption I is implicit
decide which of the given conclusions Conclusions
(b) If Assumption II is implicit
logically follows from the two given (c) If neither I nor II is implicit I. All animals are birds.
statements. (d) If both I and II are implicit II. All birds are animals.
71. Statements (a) Only Conclusion I follows
73. Statement In Mumbai, railway
(b) Only Conclusion II follows
All pens are roads. trains are indispensable for people (c) Both Conclusion I and II follows
All roads are houses. in the suburbs to reach their places (d) Neither conclusion I nor II follows
of work on time.

ANSWERS
1. (c) 2. (c) 3. (a) 4. (a) 5. (a) 6. (b) 7. (d) 8. (c) 9. (b) 10. (a)
11. (b) 12. (c) 13. (d) 14. (c) 15. (d) 16. (a) 17. (a) 18. (a) 19. (d) 20. (a)
21. (b) 22. (c) 23. (b) 24. (b) 25. (a) 26. (b) 27. (d) 28. (a) 29. (d) 30. (b)
31. (b) 32. (a) 33. (b) 34. (c) 35. (b) 36. (a) 37. (c) 38. (a) 39. (d) 40. (c)
41. (b) 42. (c) 43. (c) 44. (b) 45. (d) 46. (b) 47. (a) 48. (b) 49. (b) 50. (b)
51. (c) 52. (a) 53. (b) 54. (a) 55. (b) 56. (a) 57. (c) 58. (c) 59. (a) 60. (c)
61. (a) 62. (b) 63. (d) 64. (d) 65. (b) 66. (c) 67. (c) 68. (b) 69. (c) 70. (c)
71. (b) 72. (b) 73. (c) 74. (b) 75. (a)

https://sscstudy.com/

You might also like